You are on page 1of 392

FOR MORE JOIN, MISSION SSC / BANKING

[ii] FOR MORE JOIN, MISSION SSC / BANKING


• Head Office : B-32, Shivalik Main Road, Malviya Nagar, New Delhi-110017

• Sales Office : B-48, Shivalik Main Road, Malviya Nagar, New Delhi-110017
Tel. : 011-26691021 / 26691713

Price : ` 290

Typeset by Disha DTP Team

DISHA PUBLICATION
ALL RIGHTS RESERVED

© Publisher
No part of this publication may be reproduced in any form without prior permission of the publisher. The author and the
publisher do not take any legal responsibility for any errors or misrepresentations that might have crept in. We have tried
and made our best efforts to provide accurate up-to-date information in this book.

For further information about the books from DISHA,


Log on to www.dishapublication.com or email to info@dishapublication.com
[iii]

CONTENTS
RBI ASSISTANT Solved Paper 2012[Held on 29-04-2012] 2012-1-16

Section A : TEST OF REASONING A-1-108

1. Alphabet & Numbers Arrangement 1-3

2. Analogy & Classification 17 - 26

3. Coding-Decoding 27 - 32

4. Direction & Distance 35 - 41

5. Blood Relation 42 - 48

6. Time Sequence, Number & Ranking Test 49 - 54

7. Problem Solving 55 - 73

8. Coded Inequalities 74 - 84

9. Syllogisms 85 - 97

10. Data Sufficiency 98 - 108

Section B : TEST OF NUMERICAL ABILITY B-1-100

1. Number System & Simplifcation 1 - 13

2. Average and Problem on Ages 14 - 20

3. Ratio & Proportion 21 - 34

4. Percentage 35 - 42

5. Profit and Loss 43 - 50

6. Simple and Compound Interest 51 - 59

7. Time & Work / Pipes & Cisterns 60 - 70

8. Distance, Speed & Time (Boats and Streams) 71 - 79

9. Mensuration 80 - 91

10. Series 92 - 100


[iv]

Section C : TEST OF ENGLISH LANGUAGE C-1-82

1. Vocabulary 1 - 20

2. Grammar 21 - 44

3. Reading Comprehension 45 - 66

4. Parajumbles 67 - 75

5. Cloze Test 76 - 82

Section D : TEST OF COMPUTER KNOWLEDGE D-1-26

1. Computer Knowledge 1 - 26

Section E : TEST OF GENERAL AWARENESS E-1-56

1. General Awareness 1 - 56

FOR MORE JOIN, MISSION SSC / BANKING


Held on 29-04-2012

RBI ASSISTANT SOLVED PAPER 2012


Marks : 200 Time : 2 hrs

REASONING 7. ‘HC’ is related to ‘KQ’ in a certain way. Similarly ‘OG’ is


related to ‘AB’ in the same way. To which of the following is
1. How many meaningful English words, can be formed with
‘RK’ related to following the same pattern?
the letters PCYO using all the letters but each letter only
(a) TJ (b) SI
once in each word?
(c) TI (d) HD
(a) None (b) One (e) IM
(c) Two (d) Three 8. Each vowel of the word SAVOURY is changed to the next
(e) More than three letter in the English alphabetical order and each consonant
2. The positions of first and the fifth letters of the word is changed to the previous letter in the English alphabetical
SUITABLE are interchanged, similarly, the positions of order. If the new alphabets thus formed are arranged in
second and sixth letters, third and seventh letters and fourth alphabetic order (from left to right), which of the following
and eighth letters are interchanged. In the new arrangement will be fifth from the right?
how many letters are there in the English alphabetical series (a) U (b) R
between the alphabet which is third from the left end and
(c) Q (d) P
the alphabet which is second from the right end? (e) X
(a) None (b) One
9. How many such pairs of letters are there in the word
(c) Two (d) Four
PACKETS, each of which has as many letters between them
(e) More than four
in the word (in both forward and backward directions) as
Directions (Qs. 3 and 4): Read the following information carefully
they have between them in the English alphabetical order?
and answer the questions which follow:
(a) One (b) Two
Among A, B, C, D and E each scored different marks in an
(c) Three (d) Four
examination. Only one person scored more than C. E scored more
(e) More than four
than A but less than D. D did not Score the highest marks. The
10. Point P is 10 m to the West of Point A. Point B is 2 m to the
one who scored the second lowest, scored 71 % marks. C scored
South of Point P. Point Q is 6 m to the East of Point B. Point
92% marks.
C is 2 m to the North of Point Q. Which of the following
3. Who amongst the following is most likely to have scored
three points fall in a straight line?
87% marks?
(a) A, C, P (b) B, C, P
(a) A (b) B
(c) Q. C, A (d) A, B, Q
(c) D (d) E
(e) A, B, C
(e) Either A or D
4. Which of the following percentages is most likely to be B’s Directions (Qs. 11-15): In these questions relationship between
different elements is shown in the statements. The statements are
percentage in the exam?
(a) 68% (b) 71% followed by two conclusions.
(c) 84% (d) 76% Give Answer (A) : If only Conclusion I is true.
(e) 97% Give Answer (B) : If only Conclusion II is true.
Directions (Qs. 5-7): The following questions are based upon Give Answer (C) : If only Conclusion I or II is true.
the alphabetical series given below: Give Answer (D) : If neither Conclusion I nor II is true.
M J L I T Q S R K U F H C B D E AV P O G N Give Answer (E) : If both Conclusions I and II are true.
5. What will come in place of question (?) mark in the following 11. Statement : H³I=J>K£L
series based on the above alphabetical series? Conclusions : I. K < H
N PO BDE ? II. L ³ I
(a) UKR (b) SRKU 12. Statement : S > C ³ O; P < C
(c) RKUF (d) QSRK Conclusions : I. O < P
(e) FUK II. S < P
6. If in a certain code ‘BIND’ is coded as ‘CLGB’ and ‘HELD’ is 13. Statement : A = B £ C; A > R
coded as ‘FDJB’ based on the series given above, how will Conclusions : I. B > R
‘FORK’ be coded in the same code language? II. R< C
(a) HGKU (b) UPKR 14. Statement : D > E £ F; J < F
(c) KPSR (d) UPSR Conclusions : I. D > J
(e) UGSR II. E < J
2012-2 SOLVED PAPER-2012

15. Statement : P < Q > T; R ³ Q 24. Which of the following may represent ‘money matters’?
Conclusions : I. R > P (a) ki to (b) ma pa
II. T< R (c) fi ma (d) ha ma
Directions (Qs. 16-20): Study the following information to (e) ma jo
answer the given questions : 25. What does ‘ru’ stand for?
Eight people are sitting in two parallel rows containing four people (a) well (b) manage
each, in such a way that there is an equal distance between (c) time (d) enough
adjacent persons. In row-l A, B, C and D are seated (but not (e) Either (c) or (d)
necessarily in the same order) and all of them are facing North. In 26. Which of the following may represent ‘good enough’?
row-2 P, Q, R and S are seated (but not necessarily in the same (a) ru si (b) da ha
order) and all of them are facing South. Therefore, in the given (c) si pa (d) si da
seating arrangement each member seated in a row faces another (e) ki ru
member of the other row. Directions (Qs. 27-33): Study the following information carefully
S sits second to left of Q. A faces the immediate neighbour and answer the given questions.
of S. Only one person sits between A and C. P does not face A. B Eight friends A, B, C, D, E, F, G and H are sitting around a circle
is not an immediate neighbour of A. facing the centre, but not necessarily in the same order.
16. Which of the following is true regarding D ? • D sits third to left of A. A is an immediate neighbour of both
(a) D sits at one of the extrem ends of the line F and H.
(b) A sits to immeditate left of D • Only one person sits between C and F.
(c) Q faces D • B is not an immediate neighbour of D.
(d) C is an immediate neighbour of D • Only one person sits between B and G.
(e) No immediate neighbour of D faces R 27. A is related to G in a certain way. Similarly C is related to H
17. Who amongst the following faces C? in the same way following the given seating arrangement.
(a) P (b) Q To whom amongst the following is F related following the
(c) R (d) S
same pattern?
(e) Cannot be determined
(a) A (b) B
18. Who amongst the following sits to the immediate right of
(c) C (d) D
the person who faces C?
(e) E
(a) P (b) Q
(c) R (d) S 28. Who amongst the following sits second to the right of C?
(e) Cannot be determined (a) F (b) A
19. Four of the following five are alike in a certain way based on (c) D (d) G
the given seating arrangement and thus form a group. (e) H
Which is the one that does not belong to that group? 29. What is the position of C with respect to the position of E?
(a) A (b) Q (a) Third to the left (b) Second to the left
(c) R (d) B (c) Immediate right (d) Third to the right
(e) S (e) Second to the right
20. Who amongst the following faces R? 30. Which of the following is true with respect to the given
(a) A (b) B seating arrangement?
(c) C (d) D (a) A sits to the immediate left of H
(e) Cannot be determined (b) B sits exactly between C and G
Directions (Qs. 21-26): Study the following information to (c) F sits second to right of C
answer the given questions : (d) E is an immediate neighbour of C
‘time and money’ is written as ‘ma jo ki’ (e) None of these
‘manage time well’ is written as ‘pa ru jo’ 31. Which of the following pairs represents the immediate
‘earn more money’ is written as ‘zi ha ma’ and neighbours of G?
‘earn well enough’ is written as ‘si ru ha’. (a) A, C (b) C, D
21. What is the code for ‘earn’? (c) D, H (d) D, E
(a) si (b) ru (e) C, F
(c) ha (d) ma 32. Who amongst the following sits exactly between C and F?
(e) Cannot be determined (a) A (b) D
22. Which of the following represents ‘more time’? (c) G (d) H
(a) pa jo (b) zi ki (e) B
(c) ma ki (d) si jo 33. Starting from A, if all the friends are made to sit in the
(e) jo zi alphabetical order in clockwise direction, the positions of
23. What is the code for ‘manage’? how many (excluding A) will remain unchanged?
(a) ru (b) pa (a) None (b) One
(c) jo (d) ha (c) Two (d) Three
(e) Either (a) or (c) (e) Four
SOLVED PAPER-2012 2012-3

Directions (Qs. 34-37): In each question below are two I. S sits third to left of Q. S is an immediate neighbour of
statements followed by two conclusions numbered I and ll. You both P and T.
have to take the two given statements to be true even if they II. Two people sit between T and R. R does not sit at any
seem to be at variance from commonly known facts and then of the extreme ends. P sits second to right of T.
decide which of the given conclusions logically follows from
the given statements disregarding commonly known facts. ENGLISH LANGUAGE
Give Answer (A) : If only Conclusion I follows. Directions (Qs. 41-45): Pick out the most effective word from
Give Answer (B) : If only Conclusion II follows. the given words to fill in the blank to make the sentence
Give Answer (C) :If either Conclusion I or II follows. meaningfully complete.
Give Answer (D) : If neither Conclusion I nor II follows. 41. The government is planning to set ................ .family welfare
Give Answer (E) : If both Conclusions I and II follow. centres for slums in cities.
34. Statements : All exams are tests. (a) another (b) with
No test is a question. (c) for (d) in
Conclusions : I. Atleast some exams are questions. (e) up
II. No exam is a question. 42. Economic independence and education have ........ women
35. Statements : No bangle is an earring. more assertive.
Some earrings are rings. (a) prepared (b) made
Conclusions : I. No ring is a bangle. (c) marked (d) resulted
II. Some rings are definitely not earrings. (e) adjusted
36. Statements : Some banks are colleges. 43. In the modern world the........... of change and scientific
All colleges are schools. innovation is unusually rapid.
Conclusions : I. Atleast some banks are schools. (a) supplies (b) context
II. All schools are colleges. (c) pace (d) fantasy
37. Statements : All rivers are lakes. (e) requirement
All lakes are oceans.
44. The unprecendented economic growth of ............ China has
Conclusions : I. All rivers are oceans.
worldwide attention.
II. Atleast some oceans are lakes.
(a) perceived (b) proposed
Directions (Qs. 38-40): Each of the questions below consists of
(c) neither (d) astonished
a question and two statements numbered I and II given below it.
(e) attracted
You have to decide whether the data provided in the statements
45. Each business activity .......... employment to people who
are sufficient to answer the question. Read both the statements
would otherwise be unemployed.
and-
(a) taking (b) finds
Give Answer (A) : If the data in Statement I alone are sufficient to
(c) creates (d) provides
answer the question, while the data in
(e) given
Statement II alone are not sufficient to answer
Directions (Qs. 46-55): Read the following passage carefully
the question.
and answer the questions given below it. Certain words have
Give Answer (B) : If the data in Statement II alone are sufficient
been printed in bold to help you locate them while answering
to answer the question, while the data in
some of the questions.
Statement I alone are not sufficient to answer
The importance of communication skills cannot be underestimated
the question.
especially so in the teachinglearning process. Teaching is
Give Answer (C) : If the data either in Statement I alone or in
generally considered as only fifty per cent knowledge and fifty
Statement II alone are sufficient to answer the
per cent interpersonal or communication skills. For a teacher, it is
question.
not just important to give a lecture rich in content that provides
Give Answer (D) : If the data neither in the Statement I nor II are
sufficient to answer the question. abundant information about the subject or topic in question, but
Give Answer (E) : If the data in both the Statements I and II a successful teacher develops an affinity with, an understanding
together are necessary to answer the question. of and a harmonious interrelationship with her pupils. Building
38. On which date of the month was Parul born? rapport becomes her primary task in the classroom. But what
I. Her mother correctly remembers that she was born after exactly is rapport? Rapport is a sympathetic relationship or
15th but before 21st of April. understanding that allows you to look at the world from someone
II. Her father correctly remembers that she was born after else’s perspective. Making other people feel that you understand
18th but before 24th of April. them creates a strong bond. Building rapport is the first step to
39. How many brothers does Meghna have? (Meghna is a girl) better communication the primary goal of all true educators.
I. Kishore, the father of Meghna is the only child of Kamal. Communication skills for teachers are thus as important as their
Kamal has only two grand children. in-depth knowledge of the particular subject which they teach.
II. Jyoti, the daughter-in-law of Kamal has a son and a To a surprising degree, how one communicates determines one’s
daughter. effectiveness as a teacher. A study on communication styles
40. Among P, Q, R, S and T sitting in a straight line, facing suggests that 7% of communication takes place through words,
North, who sits exactly in the middle of the line? 38% through voice intonation and 55% through body language.
2012-4 SOLVED PAPER-2012

Much of teaching is about sending and receiving messages. 3. Receiving the information with as little distortion as
The process of communication is composed of three elements; possible.
the source (sender, speaker, transmitter or instructor), the symbols (a) Only 1 and 3 (b) Only 2
used in the composing and transmitting of the message (words (c) Only 1 (d) Only 3
or signs), and the receiver (listener, reader or student). The three (e) Only 2 and 3
elements are dynamically interrelated since each element is 52. Which of the following is true about Rapport as per the
dependent on the others for effective communication to take passage?
place. Effective communication is all about conveying your 1. It is a sympathetic relationship.
message to the other people clearly and unambiguously. It’s also 2. It is based on understanding of other people’s frame
about receiving information the others are sending to you, with of reference.
as little distortion as possible. Doing this involves effort from 3. It helps in creating a strong bond.
both the sender and the receiver. And it’s a process that can be 4. It is important for teachers to build rapport with
fraught with error, with messages muddled by the sender, or students.
misinterpreted by the recipient. When this isn’t detected it can (a) Only 1 and 2
cause tremendous confusion, wasted effort and missed (b) Only 2 and 4
opportunity. (c) Only 1, 2 and 4
Good communication skills are a prerequisite for those in (d) Only 1, 2 and 3
the teaching profession. Carefully planned and skillfully delivered (e) All are true
messages can issue invitations to students that school is a place 53. Which of the following must the teachers keep in mind to
to share ideas, investigate, and collaborate with others. Effective facilitate learning in students as per the passage?
communication is essential for a well-run classroom. A teacher, (a) To control the students such that they do not share
who is able to communicate well with students, can inspire them ideas with others within the lecture hours.
to learn and participate in class and encourage them to come (b) To maintain rapport with students and compromise on
forth with their views thus creating a proper rapport. Although the course content.
this sounds simple and obvious, it requires much more than a (c) To realise that all students have different levels of
teacher saying something out loud to a student. They must also strengths and weaknesses.
realise that all students have different levels of strengths and (d) Only to keep the lecture rich in course content.
weaknesses. (e) To ensure that students adhere to her views only.
Directions (Qs. 46 and 47): Choose the word which is most 54. Which of the following are the three elements of
opposite in meaning to the word/s printed in bold as used in the communication as per the passage?
passage. (a) Source, Signs and Students
46. ABUNDANT (b) Source, Sender and Speaker
(a) Small (b) Little (c) Signs, Words and Students
(c) False (d) Sufficient (d) Instructor, Listener and Reader
(e) Rare (e) Transmitter, Student and Receiver
47. MUDDLED 55. Which of the following is the finding of a study on
(a) Skillfully organised communication styles?
(b) Strongly controlled (a) The body language and gestures account for 38% of
(c) Clearly conveyed communication and outweighs the voice intonation.
(b) Only 9% communication about content whereas the
(d) Isolated
rest is about our tone and body language.
(e) Complicated
(c) The tone of our voice accounts for 55% of what we
Directions (Qs. 48-50): Choose the word which is most similar
communicate and outweighs the body languages.
in meaning to the word/s printed in bold as used in the passage.
(d) More than 90% of our communication is not about
48. SOUNDS
content but about our tone and body language.
(a) Seems (b) Corrects
(e) Teaching is fifty per cent knowledge and fifty per cent
(c) Noises (d) Takes
interpersonal or communication skills.
(e) Silences
Directions (Qs. 56-65): Read each sentence to find out whether
49. AFFINITY there is any grammatical error in it. The error, if any, will be in
(a) Partnership (b) Partiality one part of the sentence. The number of that part is the answer.
(c) Weakness (d) Compatibility If there is ‘No error’, the answer is (E). (Ignore the errors of
(e) Discord punctuation, if any).
50. DEGREE 56. The economic disparity (A)/ has grown rapid in(B)/ the era
(a) Extent (b) Goal of globalisation(C)/ and free market forces.(D)/ No error (E).
(c) Affect (d) Situation 57. Research shows that people(A)/ is more sensitive to
(e) Direction perceiving(B)/ messages that are consistent(C)/ with their
51. Which of the following are essential for effective opinions and attitudes.(D)/ No error (E).
communication? 58. Many poverty alleviation schemes(A)/ are not applicable
1. Conveying the message clearly. of(B)/ slum dwellers in metro cities(C)/ as they are above
2. Not to waste effort and opportunity. the poverty line. (D)/ No error (E). .
SOLVED PAPER-2012 2012-5

59. Rather than considering its(A)/ human capital as a drain 73. (a) shrinking (b) blooming
on(B)/ resources, India needs to(C)/ resource devtlop its (c) returned (d) same
into a huge.(D)/ No error (E). (e) small
60. The European nations have(A)/ become one of the(B)/ 74. (a) against (b) to
favourite destinations of the Indian students(C)/ seek (c) over (d) up
specialised knowledge and training.(D)/ No error (E). (e) for
61. If a credit card bill(A)/ is paid in full and(B)/ on time, none 75. (a) lines (b) relatively
finance(C)/ charges are levied.(D)/ No error (E). (c) accordance (d) proper
62. Training have a(A)/ positive effect on(B)/ development of (e) toning
various (C)/ skills and abilities.(D)/ No error (E). Directions (Qs. 76-80): Rearrange the following five sentences
63. Lack of ability to(A)/ read or write is just one of(B)/ the all 1, 2, 3, 4 and 5 in a proper sequence so as to form a meaningful
barriers that keep(C)/ the poor people under developed.(D)/ paragraph, and then answer the questions given below :
No error (E). 1. Understandably, the newly married woman herself
64. Science and technology have(A)/ become dominant wants to spend more time with the family.
factors(B)/ affecting our economic, cultural (C)/ and spiritual 2. They also worry that she might not be able to defend
development.(D)/ No error (E). herself in case of trouble.
65. A social business sells products(A)/ at prices that make it 3. Once married, the in-laws exert a lot of pressure for
self-sustaining, pays(B)/ no dividends and reinvestment similar cause of security.
(C)/ the profits in the business.(D)/ No error (E). 4. Initially the family does not want the ‘decent’ girl going
Directions (Qs. 66-75): In the following passage there are all around.
blanks, each of which has been numbered. These numbers are 5. Retaining female workers at door-to-door sales jobs is
printed below the passage and against each, five words are just as hard as ever.
suggested, one of which fits the blanks appropriately. Find out
76. Which of the following should be the SECOND sentence?
the appropriate word in each case.
(a) 5 (b) 2
The latest technology (66) put to use or about to arrive in market
(c) 4 (d) 3
must be (67) to all entrepreneurs. The reason is that it may have
(e) 1
an (68) effect on business. Value radios gave way to transistor
77. Which of the following should be the FOURTH sentence?
radios and with micro chips, technology is giving way to digital
equipment. Business has (69) the same but the technology has (a) 1 (b) 3
kept changing. A notable feature is that the size of the receivers (c) 4 (d) 2
decreased (70) so did the use of its material and consequently its (e) 5
price. The traditional flour mills are losing business (71) customers 78. Which of the following should be the FIRST sentence?
now buy flour (72) from the market. As a result of this, the business (a) 4 (b) 2
is (73). Following the same lines as technology, the social trends (c) 3 (d) 5
also go on changing and influence the market. The Indian sarees (e) 1
are being taken (74) by readymade stitched clothes. Every 79. Which of the following should be the THIRD sentence?
entrepreneur must note such changes in the environment and (a) 3 (b) 1
also the technology and plan in (75) with these to ensure the (c) 5 (d) 4
success of his endeavour. (e) 2
66. (a) to (b) needed 80. Which of the following should be the LAST (FIFTH)
sentence?
(c) decided (d) besides
(a) 2 (b) 4
(e) being
(c) 3 (d) 1
67. (a) hoped (b) welcome
(e) 5
(c) released (d) known
(e) aware
NUMERICAL ABILITY
68. (a) approximate (b) huge
(c) uniform (d) excellence Directions (Qs. 81-95): What should come in place of the
(e) enormous question mark (?) in the following questions?
69. (a) maintained (b) remained 81. ? ÷ 0.5 ´ 24 = 5652
(c) often (d) mentioned (a) 171.75 (b) 117.25
(e) become (c) 171.25 (d) 117.75
70. (a) mainly (b) and (e) None of these
(c) how (d) also 82. 5 ´ ? = 4808 ÷ 8
(e) some (a) 122.2 (b) 112.2
71. (a) reason (b) due (c) 120.2 (d) 102.2
(c) young (d) as (e) None of these
(e) old 83. 65% of 654 - ? % of 860 = 210.1
72. (a) knowingly (b) ease (a) 25 (b) 15
(c) cheap (d) directly (c) 20 (d) 30
(e) forcefully (e) None of these
2012-6 SOLVED PAPER-2012

84. 35154 – 20465 – 5201 = ? (a) 10 (b) 9


(a) 9488 (b) 9844 (c) 11 (d) 8
(c) 9484 (d) 9848 (e) None of these
(e) None of these 97. The owner of an electronic store charges his customer 11 %
8 192 more than the cost price. If a cutomer paid ` 1,33,200 for an
85. ¸ =? LED T.V., then what ws the original price of the T. V. ?
13 559
(a) ` 1,20,000 (b) ` 1,14,500
19 19 (c) ` 1,22,500 (d) ` 1,18,000
(a) 1 (b) 4
24 28 (e) None of these
98. The average age of a woman and her daughter is 19 years.
17 17
(c) 2 (d) 3 The ratio of their ages is 16 : 3 respectively. What is the
28 28 daughter’s age?
(e) None of these (a) 9 years (b) 3 years
86. 243 ´ 124 – 25340 = ? (c) 12 years (d) 6 years
(a) 4729 (b) 4792 (e) None of these
(c) 4972 (d) 4927
(e) None of these 3 1 5 7 8
99. If the fractions , , , and are arranged in ascending
87. 92 ÷ 8 ÷ 2 ? 5 4 6 9 11
(a) 4.75 (b) 5.75 order of their values, which one will be the third?
(c) 4.25 (d) 5.25
5 3
(e) None of these (a) (b)
88. (121)3 ´ 11 ÷ (1331)2 = (11)? 6 5
(a) 3 (b) 2 7 8
(c) 1 (d) 0 (c) (d)
9 11
(e) None of these
89. 283.56 + 142.04 + 661.78 = ? (e) None of these
(a) 1084.28 (b) 1087.28 100. A car covers a certain distance in 3 hours at the speed of
(c) 1080.38 (d) 1082.48 124 kms./hr. What is the average speed of a truck which
(e) None of these travels a distance of 120 kms less than the car in the same
90. 7028 ÷ 25 = ? time?
(a) 218.12 (b) 281.21 (a) 88 kms./hr. (b) 84 kms./hr.
(c) 218.21 (d) 282.12 (c) 78 kms./hr. (d) 73 kms./hr.
(e) None of these (e) None of these
101. The cost of 4 Calculators and 2 Stencils is ` 6,200. What is
91. 390.5 ´ ? = 284 ´ 22
the cost of 10 Calculators and 5 Stencils?
(a) (256)2 (b) 16 (a) ` 15,500
(c) 16 (d) 256 (b) ` 14,875
(e) None of these (c) ` 16,200
92. 12.5 ´ 8.4 ´ 7.6 = ? (d) Cannot be determined
(a) 787 (b) 788 (e) None of these
(c) 799 (d) 789 102. Find the average of the following set of scores: 214, 351,
(e) None of these 109, 333, 752, 614, 456, 547
93. 4477 ÷ (44 ´ 5.5) = ? (a) 482 (b) 428
(a) 24.5 (b) 21.5 (c) 444 (d) 424
(c) 16.5 (d) 18.5 (e) None of these
(e) None of these 103. The average of four consecutive odd numbers A, B, C and
94. 33.5% of 250 = ? D respectively is 54. What is the product of A and C?
(a) 76.25 (b) 82.25 (a) 2907 (b) 2805
(c) 78.75 (d) 83.75 (c) 2703 (d) 2915
(e) None of these (e) None of these
104. The sum of 55% of a number and 40% of the same number
1 3 4
95. of of of 5820 = ? is 180.5. What is 80% of that number?
2 5 9 (a) 134 (b) 152
(a) 766 (b) 777 (c) 148 (d) 166
(c) 776 (d) 767 (e) None of these
(e) None of these 105. There are 950 employees in an organization, out of which
96. 12 men alone can complete a piece of work in 6 days. whereas 28% got promoted. How many empolyees got promoted?
10 men and 21 women together take 3 days to complete the (a) 226 (b) 256
same piece of work. In how many days can 12 women alone (c) 266 (d) 216
complete the piece of work? (e) None of these
SOLVED PAPER-2012 2012-7

106. What is the least number to be added to 3000 to make it a Directions (Qs.116-120): In each of these questions an equation
perfect square? is given with a question mark (?) in place of the correct figure
(a) 191 (b) 136 on the right hand side which satisfies the eqality. Based on the
(c) 25 (d) 84 values on the left hand side and the symbol of equality given,
(e) None of these you have to decide which of the following figures will satisfy the
107. What would be the compound interest obtained on an equality and thus come in place of the question mark.
amount of `7,640 at the rate of 15 p.c.p.a after two years? Symbols stand for
(a) ` 2,634.9 (b) ` 2,643.9 > (greater than)
(c) ` 2.364.9 (d) ` 2,463.9 = (equal to)
(e) None of these < (lesser than)
108. In an examination it is required to get 65% of the aggregate ³ (either greater than or equal to)
marks to pass, A student gets 847 marks and is declared £ (either lesser than or equal to)
failed by 10% marks. What are the maximum aggregate 116. –[{92 ÷ 184} ´ 1.5] < (?)
marks a student can get? (a) –0.7 (b) –0.753
(a) 1450 (c) –0.8 (d) –0.75
(b) 1640 (e) –0.82
(c) 1500 117. [{84 – (3)2} ´ 10] > (?)
(d) Cannot be determined
( )
2
(e) None of these (a) 784 (b) (28)2
109. A juice centre requires 35 dozen guavas for 28 days. How
many dozen guavas will it require for 36 days? (c) 750 (d) 751.5
(a) 50 (b) 52 (e) 749.9
(c) 40 (d) 45 118. [85 – {58 –76}] ³ (?)
(e) None of these (a) –103 (b) 103
110. Mohan sold an item for ` 4,510 and incurred a loss of 45%.
At what price should he have sold the item to have gained (c) ± 51.5 (d) (51.5)2
a profit of 45%? (e) ± 103
(a) ` 10,900 119. éë{ 324 - 256 } ´ -4.5ùû = (?)
(b) ` 12,620
(c) ` 11,890 (a) –7 (b) 8
(d) Cannot be determined (c) –8 (d) 9
(e) None of these (e) –9
111. What will come in place of both the question marks (?) in 120. ± [(81÷ 6) + (45 ÷ 2)] £ (?)
the following question? (a) 36 (b) - 1296
1.5 (c) (d) –36
(?) 6 1296
= 1.5
288 (?) (e) 1296
(a) 6 (b) 12
(c) 7 (d) 14 GENERAL AWARENESS
(e) None of these 121. USA has asked India to reduce its dependence on crude oil
112. What would be the circumference of a circle whose area is supply from which of the following countries which is also
745.36 sq.cm? a member of OPEC?
(a) 94.4 cm (b) 88.8 cm (a) Venezuela (b) Iraq
(c) 96.8 cm (d) 87.4 cm (c) Libya (d) Iran
(e) None of these (e) Nigeria
Directions (Qs. 113-115): What will come in place of the question 122. Which of the following is the most essential financial service
mark (?) in the following number series? which should be provided to the poor people to bring them
113. 5 15 35 75 155 (?) in the network of financial inclusion?
(a) 295 (b) 315 (a) Insurance for life
(c) 275 (d) 305 (b) Investment plan for future
(e) None of these (c) Pension for old age
114. 3 6 18 72 360 (?) (d) A bank account where he/she can save small amount
(a) 2160 (b) 1800 (e) Health insurance for minor illnesses and
(c) 2520 (d) 1440 hospitalization in case of need
(e) None of these 123. Who among the following is the President of a country at
115. 688 472 347 283 256 (?) present?
(a) 236 (b) 229 (a) Rupert Murdoch (b) Ban ki-moon
(c) 255 (d) 248 (c) Yoshihiko Noda (d) Nicolas Sarkozy
(e) None of these (e) None of these
2012-8 SOLVED PAPER-2012

124. What does the letter F denote in ‘NBFCs’ a term seen very (d) Deciding rate of interest on Saving Bank Accounts in
frequently in Banking world these days? Public Sector Banks
(a) Formal (b) Fiscal (e) Representing India in World Bank and other such
(c) Federal (d) Functional agencies
(e) Financial 133. Dipika who won Crocodile Challenge Cup Finals, played in
125. Who among the following is a Deputy Governor of the RBI December 2011. is a famous
at present? (a) Badminton player
(a) Sri Sunil Mitra (b) Table Tennis player
(b) Sri Azim Premji (c) Lawn Tennis player
(c) Sri H.R. Khan (d) Chess player
(d) Mrs. Sushma Nath (e) Squash player
(e) None of these 134. Who among the following is the recipient of Nobel Prize in
126. Standard and Poor ’s is a Credit Rating Agency of Literature given in 2011?
international repute. Which of the followng is one such (a) Bruce A Beutler (b) V.S. Naipual
agency of Indian origin? (c) Tomas Transtromer (d) Ralph M. Steinman
(a) IBA (b) BASEL (e) Chetan Bhagat
(c) SEBI (d) IRDA 135. The 59th National Film Award for Best Actress was given to
(e) CRISIL (a) Vidya Balan (b) Kareena Kapoor
127. Who among the following is the Chief Minister of Uttar (c) Priyanka Chopra (d) Lara Dutta
Pradesh at present? (e) Roopa Ganguly
(a) Akhilesh Yadav 136. Which of the following is the abbreviated name of the body/
(b) Mulayam Singh Yadav agency set up to boost foreign investments in India?
(a) FOREX (b) FCCB
(c) Mayawati
(c) FIPB (d) FEMA
(d) Amar Singh
(e) AITAF
(e) None of these
137. Performance of which of the following is NOT considered
128. Coin of which of the following denominations is called Small infrastructural sector of economy?
Coin? (a) Electricity (b) Textile Sector
(a) ` 1 (b) ` 2 (c) Telecom (d) Cement
(c) ` 5 (d) 50 paisa (e) Road and Railways
(e) ` 10 138. Sachin Tendulkar made his 100th century in the match
129. Which of the following is NOT a Highlight of the Union played between India and ...........
Budget 2012-13? (a) Pakistan (b) England
(a) No change in rate of Corporate Tax (c) Australia (d) Bangladesh
(b) All types of loans upto ` 35 lacs will be given on 6% (e) Sri Lanka
interest only 139. Who amongst the following was awarded Padma Vibhushan
(c) Service Tax raised from 10% to 12% in 2012?
(d) Fiscal Deficit is targeted at 5.1% of GDP (a) T.V. Rajeswar (b) Aruna Irani
(e) Substantial increase in Defence Budget (c) A.R. Rehman (d) Shabana Azmi
130. Which of the following services/products of banks is (e) Anup Jalota
specially designed and launched to help students? 140. Which of the following countries is a member of BRICS?
(a) Personal loan (b) Corporate loan (a) Bhutan (b) Iran
(c) Business loan (d) Medical loan (c) Romania (d) Sudan
(e) Education loan (e) South Africa
131. Which of the following terms is NOT directly associated 141. Which of the following countries recently placed its first
with the functioning of RBI? Spacelab ‘Tiangong-1’ into orbit ?
(a) Open Market Operations (a) North Korea (b) Japan
(b) Cash Reserve Ratio (c) India (d) France
(c) SENSEX (e) China
(d) Liquidity Adjustment Facility 142. Who among the following is the Solicitor-General of India
(e) Public Debt Office at present?
132. Which of the following is one of the major activities of the (a) Jacob Mathew (b) Rohinton Nariman
National Bank for Agriculture and Rural Development (c) Gopal Subramaniam (d) Ashok Chawla
(NABARD)? (e) None of these
(a) On-site inspection of Cooperative Banks and Regional 143. Which of the following schemes is launched by the
Rural Banks (RRBs) Government of India to motivate school children to attend
(b) Helping Government of India in preparing Union school regularly?
Budget and presenting it in the Cabinet Meeting (a) Kutir Jyoti (b) Mid-Day-Meal
(c) Acting as custodian of the foreign exchange reserves (c) MGNREGA (d) RAY
of the country (e) Bharat Nirman
SOLVED PAPER-2012 2012-9

144. As per the news published in various newspapers, 154. Which of the following terms is used in the game of Cricket?
Government is planning to set up a Regulatory Body in (a) Heave (b) Silly Point
educational field specially for ....... (c) Tee (d) Smash
(a) Higher Education (e) Grand slam
(b) Medical Education 155. A new Nuclear Power Plant is being set up in which of the
(c) Elementary Education following places in India?
(d) Secondary Education (a) Amethi (b) Firozabad
(e) Adult Education (c) Jaitapur (d) Joshi Math
145. Which of the following countries is selected as a host of (e) Satna
Common Wealth Games 2018? 156. Which of the following schemes is launched to make cities
in India slum-free?
(a) India (b) Australia
(a) Jawaharlal Nehru National Urban Renewal Mission
(c) Pakistan (d) South Africa
(b) Bharat Nirman
(e) Sri Lanka (c) Rajiv Awas Yojana
146. France has agreed to supply ‘Rafale’ to India. The deal is (d) Indira Awas Yojana
about the supply of ............ (e) None of these
(a) Warships (b) RADAR system 157. Who among the following is a famous author of Indian
(c) Fighter Aircrafts (d) Nuclear Reactors origin?
(e) Submarines (a) Homi K. Bhabha (b) Kiran Desai
147. ‘Seychelles’, where China is going to set up its new military (c) Swati A. Piramal (d) Shabana Azmi
base, is a country in............ (e) Ronen Sen
(a) Bay of Bengal (b) China Sea 158. Who among the following has written the famous book
“Malgudi Days”?
(c) Indian Ocean (d) Red Sea
(a) V.S. Naipaul
(e) Caspian Sea (b) Deepak Chopra
148. Which of the following terms is used in the field of (c) Rabindranath Tagore
economics ? (d) Vijay Tendulkar
(a) Absolute Zero (b) Molecular Equation (e) R.K. Narayan
(c) Zero Point Energy (d) Balance of Payment 159. Which of the following is NOT the name of the currency of
(e) Mass Defect a country?
149. ‘Kyoto Protocol’, an agreement signed by various countries, (a) Rand (b) Pound
is associated with the field of ................ (c) Dinar (d) Ecuador
(a) International trade (e) Dollar
160. Which of the following is the Unit of heat?
(b) Clean environment and climate change
(a) Joule (b) Ohm
(c) Currency swap
(c) Ampere (d) Volt
(d) Deep sea mining and oil exploration (e) Newton
(e) Building a common food stock to save mankind in case
of any natural calamity COMPUTER KNOWLEDGE
150. World’s AIDS Day is observed on which of the following 161. What is backup?
days? (a) Adding more components to your network
(a) 1st December (b) 1st March (b) Protecting data by copying it from the original source
(c) 1st April (d) 1st May to a different destination
(e) 1st January (c) Filtering old data from the new data
151. USA withdrew its army from which of the following (d) Accessing data on tape
countries after a 9 year’s long stay of the same there? (e) Using earlier data
(a) Syria (b) Libya 162. The legal right to use software based on specific restrictions
(c) Egypt (d) Ireq is granted via a.............
(e) Iran (a) software privacy policy
152. Which of the following Cups/Trophies is associated with (b) software license
the game of Laws Tennis? (c) software password manager
(a) FIFACup (b) Champions Trophy (d) software log
(c) Ranji Trophy (d) Subrato Cup (e) None of these
163. What is an E-mail attachment?
(e) Davis Cup
(a) A receipt sent by the recipient
153. Which award is given to the coaches of Sports persons?
(b) A separate document from another program sent along
(a) Dronacharya Award with an E-mail message
(b) Arjuna Award (c) A malicious parasite that feeds off of your messages
(c) Kalidas Samman and destroys the contents
(d) Rajiv Gandhi Khel Ratna Award (d) A list of CC : or BCC : recipients
(e) Saraswati Samman (e) A friend to whom E-mail is sent regularly
2012-10 SOLVED PAPER-2012

164. The type of software that controls the internal operations 175. You organize files by storing them in ...............
in the computer, and controls how the computer works with (a) Archives (b) Lists
all its parts is which of the following? (c) Indexes (d) Folders
(a) Shareware (e) None of these
(b) Public domain software 176. A ............... pre-designed document that already has
(c) Application software coordinating fonts, a layout, and a background.
(d) Operating system software (a) Guide (b) Model
(e) None of these (c) Ruler (d) Template
165. When data changes in multiple lists and all lists are not (e) Design-plate
updated, this causes .............. 177. What is the default file extension for all Word documents?
(a) Data redundancy (a) WRD (b) TXT
(b) Information overload (c) DOC (d) FIL
(c) Duplicate data (e) WD
(d) Data consistency 178. Removing and replacing devices without turning off your
(e) Data inconsistency computer is referred to as ...........
166. What is the main folder on a storage device called? (a) Hot swapping (b) Plug-N-Play
(c) Bay swap (d) USB swapping
(a) Root directory (b) Interface
(e) None of these
(c) Device driver (d) Platform
179. Specialized programs that assist users in locating
(e) Main directory information on the web are called .............
167. To view information on the web you must have a ............... (a) Information engines (b) Locator engines
(a) Cable modem (b) Web browser (c) Web browsers (d) Resource locators
(c) Domain Name Server (d) Hypertext viewer (e) Search engines
(e) None of these 180. Compiling creates a(n) .................
168. A file is often referred to as a(n) ............... (a) Error-free program (b) Program specification
(a) Wizard (b) Document (c) Subroutine (d) Algorithm
(c) Pane (d) Device (e) Executable program
(e) Documentation 181. Expansion cards are inserted into ...................
169. To protect yourself from computer hacker intrusions you (a) Slots
should install a ................. (b) Peripheral devices
(a) Firewall (b) Mailer (c) The CPU
(d) The back of the computer
(c) Macro (d) Script
(e) None of these
(e) None of these
182. A device that connects to a network without the use of
170. What type of computers are client computers (most of the cables is said to be ..............
time) in a client-server system? (a) Distributed (b) Non-Wired
(a) Mainframe (b) Mini-computer (c) Centralized (d) Open source
(c) Microcomputer (d) PDA (e) Wireless
(e) None of these 183. A complete electronic circuit with transistors and other
171. What happens when you boot up a PC? electronic components on a small silicon chip is called a(n)
(a) Portions of the operation system are copied from disk ..............
into memory (a) Workstation (b) CPU
(b) Portions of the operating system are copied from (c) Magnetic disk (d) Integrated circuit
memory onto disk (e) Complex circuit
(c) Portions of the operating system are compiled 184. Junk e-mail is also called .....................
(d) Portions of the operating system are emulated (a) Crap (b) Spoof
(e) The PC gets switched off (c) Sniffer script (d) Spool
172. Linux is an example of ........... (e) Spam
(a) Freeware (b) Open source software 185. A program designed to destroy data on your computer
(c) Shareware (d) Complimentary which can travel to ‘infect’ other computers is called a .........
(e) None of these (a) Disease (b) Torpedo
173. Which one of the following software applications would be (c) Hurricane (d) Virus
the most appropriate for performing numerical and statistical (e) Infector
calculations? 186. .............. shows the files, folders, and drives on your
(a) Database (b) Document processor computer, making it easy to navigate from one location to
(c) Graphics package (d) Spreadsheet another within the file hierarchy.
(e) Power Point (a) Microsoft Internet Explorer
174. A .......... is used to read handwritten or printed text to make (b) Windows Explorer
a digital image that is stored in memory. (c) My Computer
(a) Printer (b) Laser beam
(d) Folders Manager
(c) Scanner (d) Touchpad
(e) None of these (e) Windows Locator
SOLVED PAPER-2012 2012-11

187. The .............. manual tells you how to use a software program. (c) You can see only that page which have no graphics
(a) Documentation (b) Programming (d) You can see only title page of your document
(c) User (d) Technical (e) You can see only the last page of your document
(e) Designer 194. When you will start, your computer Boot routine will
188. A collection of interrelated records is called a .............. perporm
(a) Utility file (a) RAM Test (b) Disk drive test
(b) Management information system (c) Memory Test (d) Power-on-self Test
(e) Whether power supply is on or off
(c) Database (d) Spreadsheet
195. Machine language uses
(e) Datasheet (a) Numeric code (b) English Language code
189. File extension is used (c) Java Language (d) CPU Processing code
(a) For naming the file (e) None of these
(b) To ascertain that file name is not lost 196. What is the keyboard short-cut for new slide?
(c) To identify file (a) Ctrl + M (b) Ctrl + N
(d) To identify file type (c) Ctrl + Shift + N (d) Ctrl + S
(e) To make items complex (e) None of these
190. What is Gutter margin? 197. Vertical space between lines of text in document is called
(a) Double space (b) Line gap
(a) Margin added to left margin while printing
(c) Single space (d) Vertical spacing
(b) Margin added to right margin while printing (e) Line spacing
(c) Margin added to binding side of the page while printing 198. Full form of CD-RW is
(d) Margin added to outside of page while printing (a) Compact Drum, Read, Write
(e) None of these (b) Compact Diskette, Read, Write
191. ALU of CPU has (c) Compact Disc, Read-only then Write
(a) RAM space (b) Register (d) Compact Diskette with Random Write Capability
(c) Byte space (e) Compact Disc-Rewritable
(d) Secondary storage space 199. Password makes users capable
(a) To enter into system quickly
(e) None of these
(b) To use time efficiently
192. What happens when operating system is located in RAM? (c) To retain the secrecy of files
(a) Copying (b) Device driving (d) To make file structure simple
(c) Booting (d) Multitasking (e) To feel special
(e) None of these 200. Files deleted from hard disc are sent to
193. In page preview mode (a) Dustbin (b) Floppy Disc
(a) You can see all the pages of your document (c) Clip board (d) Mother board
(b) You can see only that page on which your are currently (e) Recycle bin
working

ANSWER KEY
1 (b) 21 (c) 41 (e) 61 (c) 81 (d) 101 (a) 121 (d) 141 (e) 161 (b) 181 (a)
2 (c) 22 (e) 42 (b) 62 (a) 82 (c) 102 (e) 122 (d) 142 (b) 162 (b) 182 (e)
3 (c) 23 (b) 43 (c) 63 (c) 83 (a) 103 (b) 123 (d) 143 (b) 163 (b) 183 (d)
4 (e) 24 (c) 44 (e) 64 (e) 84 (a) 104 (b) 124 (e) 144 (a) 164 (d) 184 (e)
5 (b) 25 (a) 45 (d) 65 (b) 85 (a) 105 (c) 125 (c) 145 (b) 165 (a) 185 (d)
6 (d) 26 (d) 46 (b) 66 (e) 86 (b) 106 (c) 126 (e) 146 (c) 166 (a) 186 (b)
7 (a) 27 (d) 47 (a) 67 (d) 87 (b) 107 (d) 127 (a) 147 (c) 167 (b) 187 (a)
8 (c) 28 (c) 48 (a) 68 (e) 88 (c) 108 (e) 128 (d) 148 (d) 168 (b) 188 (c)
9 (b) 29 (a) 49 (d) 69 (b) 89 (e) 109 (d) 129 (a) 149 (b) 169 (a) 189 (d)
10 (a) 30 (e) 50 (a) 70 (b) 90 (e) 110 (c) 130 (e) 150 (a) 170 (a) 190 (c)
11 (a) 31 (b) 51 (a) 71 (d) 91 (d) 111 (b) 131 (c) 151 (d) 171 (a) 191 (b)
12 (b) 32 (e) 52 (e) 72 (d) 92 (e) 112 (c) 132 (a) 152 (e) 172 (b) 192 (c)
13 (e) 33 (b) 53 (c) 73 (a) 93 (d) 113 (b) 133 (e) 153 (a) 173 (d) 193 (a)
14 (d) 34 (b) 54 (a) 74 (c) 94 (d) 114 (a) 134 (c) 154 (b) 174 (c) 194 (d)
15 (e) 35 (d) 55 (d) 75 (c) 95 (c) 115 (d) 135 (a) 155 (c) 175 (d) 195 (a)
16 (d) 36 (a) 56 (b) 76 (c) 96 (b) 116 (a) 136 (c) 156 (c) 176 (d) 196 (a)
17 (a) 37 (e) 57 (b) 77 (b) 97 (a) 117 (e) 137 (b) 157 (b) 177 (c) 197 (e)
18 (b) 38 (d) 58 (b) 78 (d) 98 (d) 118 (e) 138 (d) 158 (e) 178 (b) 198 (e)
19 (e) 39 (e) 59 (d) 79 (e) 99 (d) 119 (d) 139 (a) 159 (d) 179 (e) 199 (c)
20 (a) 40 (b) 60 (d) 80 (d) 100 (b) 120 (c) 140 (e) 160 (a) 180 (e) 200 (e)
2012-12 SOLVED PAPER-2012

Answers &
Explanations
1. (b) Meaningful Word Þ COPY
2. (c) After rearrangement 9. (b) 16 1 3 11 5 20 19
P A C K E T S
A B L E S U I T
3rd from left 2nd from right
(3-4) : B> C > D > E >A 10. (a) 6m 4m
P C A
92% 71% 2m 2m
3. (c) D is most likely to have scored 87% marks. B Q
6m North
4. (e) B scored more than D. There-fore, B must have scored
more than 97% marks.
West East
–3 –5 –7
5. (b) N P B S
+1 +1 +1
South
O D R 11. (a) H ³ 1 = J > K £ L
Conclusions:
+1 +1
I. K < H : True
E K II. L ³ I : Not True
+1 12. (b) S > C ³ O
S>C>P
U P< C³O
Conclusions :
6. (d) B I N D C LG B
I. O < P : Not True
–1
–1 II. S > P : True
–1 13. (e) R < A = B £ C
–1 Conclusions :
I. B > R : True
HELD FDJB
–1 II. R < C : True
–1 14. (d) D > E £ F > J
–1 Conclusions :
–1 I. D > J : Not True
Similarly, II. E < J : Not True
FOR K UP SR 15. (e) P < Q £ R
–1 R³Q>T
–1 Conclusions :
–1 I. R > P : True
–1 II. T < R : True
-3 Row-2
7. (a) H ¾¾® K (16-20): R L
-7 I E
C ¾¾® Q G F
-3 H T
O ¾¾® A T Q P S R
-7
G ¾¾® B
L B C D A R
Similarly, E I
F G
R ¾¾® T
-3
T H
Row-1 T
-7
K ¾¾® J
16. (d) D sits third from the left or second from the right.
8. (c) S A V O U R Y A sits to the immediate right of D.
–1 +1 –1 +1 +1 –1 –1 S faces D.
A, the immediate neighbour of D,
R B U P V Q X faces R.
17. (a) P faces C.
B P Q R U V X 18. (b) P faces C and Q is to the immediate right of P.
19. (e) Except S, all others are seated at the extreme ends of
5th from right
lines.
SOLVED PAPER-2012 2012-13

20. (a) A faces R.


(21-26) : 35. (d)
Bangles
time and money ma jo kt
manage time well pa ru Earrings Rings
jo
earn more money zi ha ma
Conclusions I : Not True (û)
earn well enough si ru ha II : Not True (û)
21. (c) earn Þ ha Hencly, none follows.
22. (e) more Þ zi; time Þ jo Banks
Schools
23. (b) manage Þ pa
24. (c) money Þ ma: matters Þ fi College
25. (a) ru Þ well 36. (a)
26. (d) enough Þ si
good Þ da
Conclusions I : True (ü)
D II : Not true (û)
(27-33) : Hence, only I follows.
E G
Oceans
Lakes
H C Rivers
37. (e)

A B
F Conclusions I : True (ü)
II : True (ü) Inverse of second premise.
27. (d) A and G are just opposite to each other. Hence, Both follow.
C and H are just opposite to each other. 38. (d) From statement I
Similarly, F and D are just opposite to each other. Parul was born on 16th, 17th, 18th, 19th or 20th April.
28. (c) D sits second to the right of C. From statement II.
29. (a) C is third to the left of E. Parul was born on 19th, 20th, 21st, 22nd or 23rd April.
30. (e) A sits to the immediate right of H. 39. (e) From Statement I
B sits exactly between C and F. Kamal
F sits second to left of C.
E sits third to the right of C.
Kishore (+)
31. (b) Immediate neighbours of G are C and D.
32. (e) B sits exactly between C and F.
Meghna (–) (?)
D
33. (b) D From statement II
C
E E
G Kamal

B H C F
Kishore Jyoti
(+)

A B Meghna (+)
A G (–)
F
H From both the statements Meghna has one brother
40. (b) From statement I
(34-37) :
P/T S P/T R Q
Test

Exams Question From statement II


34. (b)
T P R

Conclusions I : Not true (û) 46. (b) The word Abundant (Adjective) means : plentiful,
II : True (ü) existing in large quantities; more than enough;
Hence, only II follows sufficient.
Its antonym should be little.
2012-14 SOLVED PAPER-2012
47. (a) The word Muddled (Adjective) means; confused; 91. (d) 390.5 ´ ? = 284 × 22
jumbled; deranged.
Its antonym should be skillfully organised. 284 ´ 22
48. (a) The word Sound (Verb) means : seem, give impression. Þ ?= = 16
390.5
Look at the sentence : \ ? = 16 × 16 = 256
His explanation sounds reasonable to me. 92. (e) ? = 12.5 × 8.4 × 7.6 = 798
49. (d) The word Affinity (Noun) means : rapport: close
relationship between two people or the things that 4477
93. (d) ? = = 18.5
have similar quantities, structures, freatures: 44 ´ 5.5
compatibility. 250 ´ 33.5
50. (a) The word Degree (Noun) means: amount or level of 94. (d) ? = = 83.75
something: extent. 100
56. (b) An adverb adds to the meaning of a verb, an adjective 4 3 1
or an other adverb. 95. (c) ? = 5820 ´ ´ ´ = 776
9 5 2
Hence, has grown rapidly....in should be used here. 96. (b) Q 12 men complete the work in 6 days.
57. (b) Here, Subject (people) is plural.
Hence, are more sensitive to per ceiving... should be 1
\ 1 man’s 1 day’s work =
used. 72
59. (d) Here, develop it into a huge resource ... should be
10 ´ 3 5
used. There is need of meaningful arrangement. \ 10 men’s 3 day’s work = = work
60. (d) Here, seeking/who seek specialised ... should be used. 72 12
61. (c) Here, on time no other charges should be used. 5 7
62. (a) Here, Training has a (Singular) ... should be used. Remaining work = 1 - =
12 12
63. (c) Here, all the or the barriers that keep ... should be used.
65. (b) Here, at prices that make them (products) self- 7
sustaining .. should be used. \ 21 women do work in 3 days.
12
? M1D1 M 2 D2
81. (d) ´ 24 = 5652
0.5 \ By W = W
1 2
5652 ´ 0.5
Þ ?= = 117.75 21 ´ 3 12 ´ D 2 21 ´ 3 ´ 12
24 Þ = Þ = D2
7 1 7 ´ 12
4808 12
82. (c) 5 × ? = = 601
8 Þ D2 = 9 days
601 97. (a) Let original price of TV = x
Þ ?= = 120.2 Customer paid = 111% of x = Rs 133200
5
133200 ´ 100
654 ´ 65 860 ´ ? \ x= = Rs 1,20,000
83. (a) - = 210.1 111
100 100
98. (d) Let Woman’s age = 16x
860 ´ ? daughter’s age = 3x
Þ 425.1 - = 210.1
100 Now (16x + 3x)/2 = 19
Þ 19x = 38 Þ x = 2
860 ´ ?
Þ = 425.1 –210.1 = 215 \ Daughter’s age = 3 × 2 = 6 years
100
3 1 5 7 8
215 ´ 100 99. (d) = 0.6: = 0.25; = 0.83: = 0.78; = 0.73
Þ?= = 25 5 4 6 9 11
860
84. (a) ? = 35154 – 20465 – 5201 = 9488 1 3 8 7 5
\ < < < <
4 5 11 9 6
8 559 43 19
85. (a) ? = ´ = =1 8
13 192 24 24 Hence will be the third number
86. (b) ? = 243 × 124 – 25340 = 30132 – 25340 = 4792 11
100. (b) Distance covered by car in 3 hours = 124 × 3 = 372 km
92
87. (b) ? = = 5.75 Distance covered by truck in 3 hours
8´ 2 = 372 – 120 = 252 km.
88. (c) ((11)2 )3 ´ 11 ¸ ((11)3 )2 = (11)? Average speed of truck =
252
= 84 km/hr
Þ 116 × 11 ¸ 116 = (11)? 3
Þ (11)6 + 1 – 6 = (11)? Þ ? = 1 101. (a) Let cost of 1 calculator = Rs. x
89. (e) ? = 283.56 + 142.04 + 661.78 = 1087.38 Let cost of 1 stencil = Rs. y
\ 4x + 2y = 6200
7028 Multiplying both sides by 2.5
90. (e) ? = = 281.12
25 10x + 5y = 6200 × 2.5 = Rs. 15500
SOLVED PAPER-2012 2012-15
102. (e) Required average
745.36 ´ 7
214 + 351 + 109 + 333 + 752 + 614 + 456 + 547 3376 Þ r2 = = 237.16
= = 22
8 8
\ r= 237.16 = 15.4 cm
= 422
103. (b) Let four numbers be A, A + 2, A + 4, A + 6 A + A + 2 + 22
A + 4 + A + 6 = 54 × 4 \ Circumference of circle = 2pr = 2 ´ ´ 15.4
7
Þ 4A + 12 = 216 Þ 4A = 216 – 12 = 204
= 96.8 cm
204 113. (b) The pattern is :
ÞA = = 51
4 15 – 5 = 10; 35 – 15 = 20
B = A + 2 = 53 75 – 35 = 40; 155 – 75 = 80
\ C = A + 4 = 51 + 4 = 55 Now ? = 155 + 160 = 315
D = A + 6 = 57 114. (a) The pattern is :
\ A × C = 51 × 55 = 2805 3×2=6
104. (b) Let the number be x. 6 × 3 = 18
Now (55 + 40)% of x = 180.5 18 × 4 = 72
x ´ 95 108.5 ´ 100 72 × 5 = 360
Þ = 180.5 Þ x = = 190 360 × 6 = 2160
100 95
115. (d) The pattern is :
190 ´ 80 688 – 472 = 216 = 63
Now 80% of 190 = = 152
100 472 – 347 = 125 = 53
347 – 283 = 64 = 43
950 ´ 28
105. (c) Number of promoted employees = = 266 283 – 256 = 27 = 33
100 \ ? = 256 – 23 = 256 – 8 = 248
106. (c) (55)2 = 3025
\ Required number = 3025 – 3000 = 25 éì 1 ü ù
116. (a) LHS = - êí92 ´ ý ´ 1.5ú
éæ ù T ëî 184 þ û
R ö
107. (d) We know compound interest = P ê çè1 + ÷ø - 1ú = – 0.75 < – 0.7
ë 100 û
117. (e) LHS = (84 – 9) × 10 = 750 > 749.9
éæ 2 ù é æ 23 ö 2 ù 118. (e) LHS = 85 – 58 + 76 = 103 ³ ± 103
15 ö
= 7640 ê çè1 + ÷ø - 1ú = 7640 ê çè ÷ø - 1ú
ë 100 û ë 20 û { }
119. (d) LHS = – éë 324 - 256 ´ -4.5ùû
é 529 ù 7640 ´ 129 = – [(18 - 16 ) ´ -4.5] = 9 = 9
= 7640 ê -1 = = Rs. 2463.9
ë 400 úû 400
108. (e) Let maximum marks = x æ 81 45 ö
120. (c) LHS = ± çè + ÷ø
Student got 55% x = 847 6 2
847 ´ 100 æ 27 45 ö 72
\ x= = 1540 = ± çè + ÷ø = ± = ± 36 £ 1296
55 2 2 2
109. (d) For 28 days º 35 dozens of guavas 121. (d) the member countries of Organisation of Petroleum
35 Exporting Countries (OPEC) are : Algeria, Indonesia, Iran,
For 1 day = dozens Iraq, Kuwait, Libya, Nigeria, Qatar, Saudi Arabia, United Arab
28
Emirates and Venezuela.
35 123. (d) Nicolas Sarkozy, the President of France was defeated
Now 36 days = ´ 36 = 45 dozens
28 by Francois Hollande in the presidential election held on
110. (c) Let cost price of article = x May 6, 2012, Mr. Hollande succeeded Mr. Sarkozy as
Now 55% of x = Rs. 4510 President on May 15. 2012.
124. (e) NBFCs : Non-Banking Financial Companies
4510 125. (c) Harun Rashid Khan on July 4, 2011 assumed charge as
Therefore x = ´ 100 = Rs 8200
55 the Deputy Governor of the Reserve Bank of Indian for three
To gain profit of 45% selling price is 145 % of 8200 is years.
Rs 11890. 126. (e) CRISIL : The Credit Rating Information Services of India
111. (b) (?)1.5 × (?)1.5 = 288 × 6 Ltd. It is India’s first credit rating agency which started
Þ ?3 = 2 × 2× 2 × 6 × 6 × 6 functioning in January 1998.
?3 = 23 × 63 IBA : Indian Banking Association BASEL : The
implementation of Basel III capital regulation in India will
?3 = (12)3
kick start from January 1, 2013. It will be fully implemented
? = 12
by March 31,2018.
112. (c) We know area, pr2 = 745.36
IRDA : Insurance Regulatory and Development
22 2 Authority
Þ ´ r = 745.36
7
2012-16 SOLVED PAPER-2012
127. (a) Akhilesh Yadav, the son of Samajwadi Party supremo 142. (b) Rohinton Nariman was on July 23, 2011 appointed
Mulayam Singh Yadav was sworn in as Chief Minister of Solicitor General of India. He succeeded Gopal Subramaniam,
Uttar Pradesh on March 15,2012. who resigned on July 9, 2011.
128. (d) The government issues coins of denominations 50 143. (b) Mid-Day Meal scheme is the popular name for school
paisa, one rupee, two rupees, five rupees and ten rupees. meal programme in India which started in the 1960s. Kutir-
129. (a) Service tax rate was increased from 10 per cent to 12 per Jyoti Programme launched in the late 1980s, envisaged
cent, with corresponding changes in rates for individual extending single point light connections to households of
services. The proposals for service tax expected to yield rural BPL families.
additional revenue of Rs. 18,660 crore. MGNREGA (Mahatma Gandhi National Rural
Fiscal deficit at 5.9 per cent of GDP in the Revised Employment Guarantee Act) was enacted by legislation on
Estimates for 2011-12 while fiscal deficit is targeted at 5.1 per August 25. 2005 and launched February 2, 2006. Initially,
cent of GDP in Budget Estimates for 2012-13. The Budget NREGA, but was renamed on October 2, 2009 as MGNREGA.
2012-13 proposed to increase defence spending by some 18 RAY (Rajiv Awas Yojana) : With an aim of creating a
per cent and hiked the budget for capital acquistion by 15 slum-free India, the Government on June 2, 2011 approved
per cent. A sum of Rs. 193407 crore were earmarked for the launch of the phase-1 of Rajiv Awas Yojana to facilitate
defence. affordable housing for slum dwellers.
130. (e) Education loan has been specially designed and Bharat Nirman is a plan for creating basic rural
launched to help students. infrastructure. It comprises projects on Irrigation, roads,
131. (c) Sensex is related to share market. (Pradhan Mantri Gram Sadak Yojanal. housing (Indira Awas
132. (a) The National Bank for Agriculture and Rural Yojana), water supply, electrification (Rajiv Gandhi Grameen
Development (NABARD) is the apex developmental bank Vidyutikaran Yojana’ and telecommunication connectivity.
for agriculture and rural development. It was set up on July 144. (a) The Union Cabinet on December 20, 2011 approved
12,1982. the National Council for Higher Education and Research
NABARD has been entrusted with three types of (NCHER), a single overarching Independent body to cover
functions, namely. (i) the credit functions (ii) the all streams of higher education except the medical and
developmental functions and (iii) the regulatory functions. agricultural areas.
The Banking Regulations Act. 1949 empowers NABARD to 145. (b) Commonwealth Games 2018 will be held in Gold Coast,
undertake inspection of Regional Rural Banks and Co- Australia from April 4 to April 15, 2018.
operative Banks (other than primary cooperative banks). 146. (3) India on January 31, 2012 selected the French Fighter
133. (e) Crocodile Challenge Cup Finals was played on Rafale over the Eurofighter Typhoon in a multi-billion dollar
December 18,2011 at the Hong Kong Squash Centre. Dipika contract for the supply of 126 Medium Multi Role Combat
Pallikal beat Joey Chan of Hong Kong. Aircraft (MMRCA) - the country’s largest defence deal to
134. (c) Swedish poet Tomas Transtromer received the 2011 date.
Nobel Prize in Literature in December 2011. Before 147. (c) Seychelles is a group of about 100 islands scattered in
Transtromer, two Swedish authors, Eyvind Johnson and the Western Indian Ocean. Its capital is Victoria.
Harry Maritinson. shared the Nobel Prize in Literature in 148. (d) Balance of Payment is a tabulation of the credit and
1974. debit transactions of a country with foreign countries and
135. (a) Vidya Balan received the Best Actress award for her international institutions.
role in The Dirty Picture at the 59th National Film Award on 149. (b) Kyoto Protocol, which was adopted in 1997, is part of
May 3, 2012. the United Nations Framework Convention on Climate
136. (c) FOREX: Foreign Exchange FCCB : Foreign Currency Change. Canada is the first country to formaly withdraw
Convertible Bond from the Kyoto Protocol.
FIPB : Foreign Investment Promotion Board 150. (b) World’s AIDS Day is observed on 1st December.
FEMA: Foreign Exchange Management Act 1st April : Foundation Day of Orissa (Utkal) : Fool’s Day
AITAF: ARTS in the Armed Forces 1st May : International Laobur Day; Statehood Day of
137. (b) Eight core infrastructure Industries are crude oil, Maharashtra and Gujarat
petroleum refinery products, natural gas, fertilisers, coal, Ist January : Nagaland Day; Army Medical Corps Foundation
Day: Liberation Day of Cuba: Revolution Day of Palestine:
electricity, cement and finished steel.
National Day of Sudan: Republic Day of Slovakia.
138. (d) On March 16, 2012 Sachin Tendulkar became (he first
151. (d) USA withdew its Army from Iraq after nine years. USA
cricketer in the world to notch up century of International
will withdraw its army from Afghanistan by 2014.
centuries after scoring 114 against Bangladesh.
152. (e) FIFA Cup: Football: Champions Trophy : Hockey: Ranji
139. (a) Padma Vibhushan awardees in 2012 : KG Subramany
Trophy : Cricket.
an; Mario de Miranda, Bhupen Hajarika, Dr. Kantimal
Subrato Cup : Football
Hastimal Sancheti and T V Rajeswar.
Davis Cup : Tennis
140. (e) The Fourth BRICS Summit was held on March 29. 2012
154. (b) Heave : Volleyball: Silly Point: Cricket: Tee : Golf : Smash
in New Delhi. The grouping was formalised with the first
: Badminton: Grand Slam : Tennis
meeting of the Foreign Ministers of Brazil, Russia, India
156. (c) Rajiv Awas Yojana under Jawaharlal Nehru National
and China on the margins of the United Nations General
Urban Renewal Mission
Assembly in New York in September 2006. South Africa
157. (b) Kiran Desai won Booker Prize in 2006 for her book ‘The
Joined the grouping at the third Summit in Sanya, China in
Loss of Inheritance’.
April 2011.
159. (d) Ecuador is a country in South America. Its capital is
141. (e) China on September 29, 2011 successfully launched its
Quito.
first space laboratory module, Tiangong-1. 160. (a) Unit of heat is Joule. One calorie is equal to 4.2 Joules.
Section A : TEST OF REASONING

Alphabet & Numbers


Arrangement
1 Chapter
As we know that English alphabet is a group of English letters, RULES FOR QUICKER METHOD:
hence the problems based on alphabet are the problems based (a) If both the directions are same then subtraction of numbers
on English letters. Problems under this segment are the very takes place.
important part of the questions asked in various competitive exams (b) If the directions are opposite then addition of numbers takes
to be conducted for the purpose of requirement of officers and place.
clerks. Particularly for getting job in banking sector, this type of Now, for solving the sample question we apply this rule. As
questions can not be ignored. This is the reason that we will we want to find out the 7th letter to the right of the 13th letter
discuss every aspect of such problems so that students do not from the left, the directions are opposite and thus rule (b)
face any kind of difficulty while solving the problems related to will be applied here. Hence we add 7 + 13 = 20. Therefore,
English alphabet. the answer will be 20th from left. Also, 20th from left less
mean 26 – 20 + 1 = 7th from right. We can easily see.
TYPES OF PROBLEMS : \ 20th letter from left = T
(1) General series of alphabet Also 7th letter from right = T
(2) Random series of alphabet \ This method also gives the answer choice (b).
(3) Words in alphabetical order After solving the sample question, you must have noticed
(4) Problems of word formation that the above mentioned trick is to calculate the actual
position of the required letter before going to search for it.
(5) Problems of letter gap
(6) Rule identification problems
OTHER VARIATIONS OF SUCH TYPE OF PROBLEMS
Now we will discuss all the six types of problems one by
one in detail. EXAMPLE 2. If alphabet series is given in backward or
reverse order, then find out the eighth letter to right of O?
(1) General Series of Alphabet :
(a) H (b) G (c) U
EXAMPLE 1. Which of the following options is seventh to (d) X (e) None of these
the right of the 13th letter from the left in a forward Alphabet Z Y X W V U T S R Q P O
series? Sol. N M L K J I H G F E D C B A
(a) R (b) T (c) V 1 2 3 4 5 6 7 8
(d) W (e) None of these. It’s clear (b) is the correct answer.
Sol. Now the question is how to solve it?
1st of all we will write the forward alphabet series as given
Note : Even with the forward alphabet series we can solve
this problem because the letter which is eight to the right of O in
below:
the reverse order alphabet series must be eight to the left of O in
A B C D E F G H I J K L M forward alphabet series.

13th letter from left EXAMPLE 3. If the 1st half of the alphabet is written in
N O P Q R S T U V W X Y Z reverse order, then find out the letter that would be 20th letter
1 2 3 4 5 6 7 from the right.
(a) G (b) F (c) D
From above series it is clear that M is the 13th letter from
(d) H (e) None of these
left and to the right of M (13th letter from left), T is the 7th
Sol. As the 2nd half is not reversed, the 1st 13 letter would be
letter. Hence (b) is the correct option. same when we do counting from right. But not letters coming
Here, we have solved this problem with a general method. after 13th will be actually from the left. Hence 14th letter from
But this type of problem can also be approached through right would be A; 15th would be B; 16th would be C and we
quicker method that will help you save some extra consumed move further in the same manner. Hence from left which is G.
time. \ Option (a) is the correct answer.
A- 2 ALPHABET AND NUMBERS ARRANGEMENT

Extra Tips
I: While solving the problems based on alphabet, you must have in your mind the exact positions of every letters of alphabet in
forward order as well as in backward or reverse order as given below:
Letters positions in forward alphabetical order:
A B C D E F G H I J K L M N O P Q R S T U V W X Y Z
1 2 3 4 5 6 7 8 9 10 11 12 13 14 15 16 17 18 19 20 21 22 23 24 25 26
Letters positions in backward or reverse alphabetical order:
Z Y X W V U T S R Q P O N M L K J I H G F E D C B A
1 2 3 4 5 6 7 8 9 10 11 12 13 14 15 16 17 18 19 20 21 22 23 24 25 26
II: Just keep in mind, the following positions of the letters in the English alphabet (forward order).

(i) E J O T Y (ii) C F I L O R U X

5 10 15 20 25 3 6 9 12 15 18 21 24

EJOTY Remember this word CFILORUX Remember

(iii) D H L P T X

4 8 12 16 20 24

DHLPTX Remember

III: mth element to be counted from left to right of a series of x characters is equal to (x + 1 – m)th element to be counted from right
to left of that series. This rule can be better illustrated by an example which is given below:
Let us take the forward order alphabet series,
A B C D E F G H I J K L M N O P Q R S T U V W X Y Z
1 2 3 4 5 6 7 8 9 10 11 12 13 14 15 16 17 18 19 20 21 22 23 24 25 26
As we know that English alphabet has 26 characters, hence, we have x = 26.
Now suppose, we have to find out the position of K in the above given series counting from right to left.
Position of ‘K’ in the English alphabet from left to right is 11. Thus m = 11
\ Position of K in the above given series from right to left would be (26 + 1 – 11) = 16

Note : I, II & III given under extra tips are very important as Sol. General method:
they are very helpful in solving problems based on general series A B C D E F G H I J K L M N O
of alphabet. Renders are advised to take them as a rule. P Q R S T U V W X Y Z
Here, delated letters have been encircled and we find the
EXAMPLE 4 : How to solve problems when letters are dropped new series as given below:
or deleted at regular intervals? A B D E G H J K M N P Q S T V W Y Z
Sample Question: If every 3rd letter from left to right of English 1 2 3 4 5 6 7 8 9 10 11 12 13 14 15 16 17 1
alphabet is deleted, then what would be the 6th letter from left in D E G H J K M N P Q S T V W Y Z
the new series obtained? 1 2 3 4 5 6 7 8 9 10 11 12 13 14 15 16 17 18
It is clear, that 6th letter from left in the new series is H.
ALPHABET AND NUMBERS ARRANGEMENT A- 3
QUICKER APPROACH: in2nd segment the next 8 letters get reversed and in the 3rd segment
No doubt, that general method gives the correct answer. But we the remaining 10 letters get reversed. Just see the presentation
need to save extra consumed time and this is the reason we go for given below:
quicker approach. A B C D E F G H I J K L M N O P Q R S T U V W X Y Z
As per the question, every third letter is deleted in the original
Get reversed Get reversed Get reversed
series. It does mean that we are left of two letters after every
deletion. Here, ‘2’ is the key digit for us and we have to find out H G F E D C B A P O N M L K J I Z Y X W V U T S R Q
6th letter from the left in the new obtained series. Therefore, we (8 letters) (8 letters) (10 letters)
have to find a digit which is just less than 6 but divisible by 2. For
this question the digit just less than 6 and divisible by 2 is 4. Now Now if you are asked to find out the 4th letter from left
in the new
we follow the operation given below: obtained series, then through general method, we simply do
counting from left in the new series and find out our required
4 answer as ‘E’ because ‘E’ is at 4th position from left in the new
6th letter from the left in the new series = 6 +
2 obtained series. But while solving such type of problems, we
= 8th letter from the left in the original series, which is it. have to do some time consuming formalities like (a) writing the
In the same manners, we can find out any letter at a particular original series (b) writing and reversing the letters of original
position in the new obtained series. series as per the question says and (c) counting them to get the
14 required answer. Such time consuming processes can be avoided
\ 16th letter from the left in the new obtained series = 16 + if we go through “Extve Tips III” and solve the question with
2
quicker approach.
= 23rd letter from the left in the original series which is W.
18th letter from the left in the new obtained series QUICKER APPROACH:
16 It is clear that 4th letter from left in the new obtained series falls
= 18 + into first segment which has 8 letters. Hence 4th letter in the new
2
= 26th letter from the left in the original series which is Z. obtained series = (8 + 1 – 4) = 5th letter from the left in the original
The sample question can be asked in following way also: series. As we know that exact position of 5th letter from left in the
“If every third letter from left to right in English alphabet is dropped original alphabet series is the position of E. Hence E is our required
(or deleted), then find out the 13th letter from right in the new answer.
obtained series”. If we have to find out 18th letter from left in the new obtained
To solve this, we find 1st of all the number of letters in the new series, then that will be 16 + (10 + 1 – 2) = 25th letter from left in the
obtained series. original alphabet series (why?) which is Y.
As every third letter is dropped, hence we have In fact, while finding out 18th letter, we can easily see that 18th
letter is the 2nd letter of 3rd segment and hence it will be not
æ 26 ö
çè 26 – ÷ø = 26 – 8 = 18 letters in the new series. affected by 1st two segments having 8 letters each. In other words
3 to find out 18th letter in the new obtained series, we have to find
Point to be noted that here we divide 26 by 2 as every 3rd letter is out the 2nd letter in the 3rd segment. This is the reason we find out
26 the 2nd letter in the 3rd segment and then add the 16 letters of 1st
dropped and after division we take approximate value of in two segment to get the 18th letter in the new obtained series.
3
From this, we find that 18th letter from left in the new obtained
26 series is the 25th letter from left in the original series. As 25th letter
round figure (approximate value of will be 8).
3 from left in the original series is Y. It (Y) will be our required
As per the question we have to find out 13th letter from right in answer.
the new obtained series. This loss mean (18 + 1 – 13) = 6th letter Readers are advised to practice such type of problems as much
from left which is H. as possible and after a certain time type will notice that they have
get a skill to solve such problems in a few seconds and that too,
Note : This quicker approach can also be applied to the without the use of pen and paper.
dropping of every 4th, 5th, 6th, 7th..... and so on letters from left
to right at regular intervals. HOW TO SOLVE IF POSITIONS OF LETTERS ARE
INTERCHANGED?
How to solve problems based on the backward (reversed) alphabet
There is no any rule for such type of problems. Only the hard
series?
practice can given you a skill to solve such questions in a quick
While solving problems based on general series of alphabet, we
time.
come across the various cases. In some cases we see that whole
alphabet series is reversed but in some other cases 1 st half of the EXAMPLE 5. If A and C interchange their places, B and D
series is reversed, or second half of the series is reversed or many interchange their places, F and H interchange their places and
segments of the alphabet series are reversed. so on, then which letter will be 5th to the left of Q?
Let us take a case when a forward order alphabet series get (a) P (b) N (c) M
reversed in three segments. In 1st segment 8 letters get reversed; (d) T (e) None of these.
A- 4 ALPHABET AND NUMBERS ARRANGEMENT

Sol. As per the question the interchanges take place as follows: Sol. Here m = 8 and n = 15.

(8 –15) + 27 ù = é 20 ù
Then middle letter = é th
A B C D E F G H I J K L M N O P êë 2 úû êë 2 úû = 10
letter from left in the English alphabet = J.
Q R S T U V W X Y Z
Note : In case III (m – n) + 27 must be divisible by 2.
Here we can see that Q interchanges with S. Then to left of
2. Random series of alphabet :
Q, the 5th letter would be P because P interchanges with N.
This series is not in the proper sequence and letters take
How to find the middle letter?
their position in the series in jumbled manner. Further, this
Case I : Remember that if mth and nth letter from the left in is also a possibility that all the 26 letters of English alphabet
the English alphabet are given then are not available in the series. Even same letters may be
repeated in the series.
æ m + nö
Middle letter = çè ÷ th letter from the left.
2 ø EXAMPLE 9. How many as in the following series are
immediately preceded by B but not immediately followed by D?
EXAMPLE 6. Which letter will be midway between 8th letter
R S P Q B A H M A C F B A D N O P B A C D.
from the left and 16th letter from the left in the English alphabet?
Sol. × ×
Sol. Here m = 8 and n = 16
R S P Q B A H M A C F B A D N O P B A C D
8 + 16 24
then middle letter = = = 12th letter from left in the
2 2 ü ü

alphabet = L \ Only the two times A fulfill the given condition and those
Case II: Remember that if mth and nth letter from the right in the A have been marked with the correct sign (ü). Those not
English alphabet are given then fulfilling the condition have been marked with the cross
sign (×). \ Required answer is 2.
æ m + nö (3). Words in alphabetical order
Middle letter = ç th letter from right
è 2 ÷ø
In such type of questions, words are given and you have to
find out which word will appear in the dictionary 1st . 1st or
é æ m + nö ù é æ m + nö ù 2nd or 3rd or 4th etc.
= ê 26 + 1 – ç ÷ø ú = ê27 – çè ÷ th
ë è 2 û ë 2 ø úû
EXAMPLE 10. Which of the following words will come 2nd in
letter from the left in the English alphabet. the dictionary?
EXAMPLE 7. Which letter will be midway between 8th letter (a) Name (b) Shame (c) Fame
from the right and 16th letter from the right in the English (d) Came (e) None of these.
alphabet. Sol. ‘Came’ comes 1st in the dictionary.
‘Fame’ comes 2nd in the dictionary
é æ 8 + 16 ö ù
Sol. Middle letter = ê 27 – ç th letter from left in the
ë è 2 ÷ø úû ‘Name’ comes 3rd in the dictionary
‘Shame’ comes 4th in the dictionary
alphabet.
\ (c) is the required answer.
or middle letter = (27 – 12) = 15th letter from left = 0
EXAMPLE 11. Find out the word coming last in the dictionary..
Note : In case I and case II (m + n) must be divisible by 2.
(a) Large (b) Long (c) Lust
Case III :Remember that if the mth letter from the left and the nth (d) Love (e) None of these
letter from the right are given then middle letter
Sol. Step I : In this question the 1st letter of all the words are
é (m – n) + 27 ù same. Hence, from 1 st letter we can not find out this
= ê úû th letter from the left in the alphabet. arrangement in the dictionary.
ë 2
Step II: We move on the 2nd letter of the words and find
EXAMPLE 8. Which letter will be midway between 8th letter that 2nd letter of Large is ‘a’; 2nd letter of Long is ‘O’; 2nd
from the left and 15th letter from the right? letter of Lust is ‘u’ 2nd letter of Love is ‘O’. Now its clear
ALPHABET AND NUMBERS ARRANGEMENT A- 5

that in the dictionary ‘a’ comes before ‘O’ & u. Hence the
Sol. C O N T R O V E R S I A L
word ‘Large’comes 1st in the dictionary.
Step III: Now we will compare the remaining three words 1st 4th 6th 8th
‘Long’, Lust and ‘Love’. Here, when we see the 2nd letter of Now from letters N T O and E, the two words ‘NOTE’ and
these words we find ‘O’ comes before ‘U’ in the dictionary. ‘TONE’ can be formed.
Hence we can come to the conclusion that the words ‘Long’ 5. Case I : Problems of letter gap
and ‘Love’ will definitely not the last word. Thus we came EXAMPLE 13. How many pairs of letters are there in the
to our required answer that the word ‘Lust’ or option (c) will word ‘DREAMLAND’ which have as many letters between them
definitely come last in the dictionary. as in the English alphabet?
\ Option (c) will be our answer. But if we want to know Sol. Here, we are asked to solve problem according to English
the 2nd and 3rd word also then we can move on to the next alphabet. In this case we have to count both ways. It does
mean that we have to count from left to right and from right
step.
to left. Let us see the following presentation:
Step IV: In the words ‘Long’ and ‘Love’, 1st two letters
are common. Therefore, to know the arrangement of these
D R E A M L A N D
two words in the dictionary we move on to the third letter.
Third letter in the word ‘Long’ is ‘N’ and in ‘Love’ the third
letter is ‘v’. As ‘n’ comes before ‘v’ alphabetically, the word The above presentation makes it clear that the required
‘Long’ will come before the word ‘Love’ in the dictionary. pairs of letters are 4. (Pairs: DA, EA, ML and LN)
Case II:
Hence, it is clear that ‘Long’ comes 2nd and ‘Love’ comes
3rd in the dictionary. EXAMPLE 14. How many pairs of letters are there in the
Step V: Find arrangement : (1) Large (2) Long (3) Love word ‘DREAMLAND’ which have the same number of letters
between them as in the English alphabet in the same sequence.
(4) Lust.
Sol. Here we are asked to solve problems according to the
4. Problems of word formation. alphabetical sequence. It does mean that we have to do
In such problems a word is given and you have to find out counting only from left to right. Let us, see the following
the number of words to be formed out of some letters drawn presentation:
from that particular word. D R E A M L A N D

EXAMPLE 12. How many meaningful words can be formed The above presentation makes it clear that the required pair
from the 3rd, 4th, 6th and 8th letter of the word ‘CONTROVERSIAL’? of letters is only 1 (Pair: LN)

EXERCISE
Directions (Qs. 1-5): Answer these questions referring to the 3. In the alpha-numerical sequence/series given below, how
symbol-letter-number sequence given below: many numbers are there which are (i) immediately followed
1. If every third letter from the following English alphabet is by a letter at the even place in English alphabet and (ii) not
dropped, which letter will be seventh to the right of eleventh immediately preceded by a letter at the odd place in the
letter from your right? English alphabet?
ABC DE FG HI JK LM NO PQRS T UV W XYZ W2 N1 V9 G2 P4 X6 K7 R1 T 8 L3 H5 Q8 U2 J
(a) V (b) U (a) 3 (b) 5
(c) K (d) I (c) 2 (d) 4
(e) None of these (e) None of these
2. If the first half of the English alphabet is reversed and so is 4. If the positions corresponding to the multiples of five in the
the second half, then which letter is seventh to the right of following alphabet are replaced by symbols and that of
twelfth letter from the left side? multiples of seven by digits, how many letters will be left?
ABCDEFGHIJKLMNOPQRSTUVWXYZ ABC DE FG HI JK LM NO PQRS T UV W XYZ
(a) S (b) V (a) 15 (b) 18
(c) U (d) T (c) 21 (d) 17
(e) None of these (e) None of these
A- 6 ALPHABET AND NUMBERS ARRANGEMENT
5. If only the first half of the following English alphabet is and E in the new order?
reversed how many letters will be there between K and R? (a) Z (b) N
ABC DE FG HI JK LM NO PQRS T UV W XYZ (c) D (d) There is no such letter
(a) 6 (b) 16 (e) None of these
(c) 14 (d) 10 Directions (Qs. 13-15): Answer these questions referring to the
(e) None of these symbol-letter-number sequence given below:
QD T P52 3F G4 B7 HJ 9K6 M N8
Directions (Qs. 6-8) : Following questions are based on English
13. If the positions of the letters in the sequence remain
alphabet.
unchanged and the positions of the numbers in the
ABC DE FG HI JK LM NO PQRS T UV W XYZ
sequence are reversed then which of the following letter/
6. If the letters from T to Z are interchanged by the letters A to number is fifth to right of ninth letter/number from the right?
G in such a way that A takes the position of T and so on, (a) P (b) 6
then which will be the third letter to the left of 18th letter (c) 3 (d) K
from right?
(e) None of these
(a) Y (b) U
14. How many letters are there in the sequence which are
(c) B (d) C immediately preceded by a number and immediately
(e) None of these followed by a letter?
7. If first 6 letters shall be written in opposite order, then the (a) One (b) Two
next 6 letters shall be written in opposite order and so on, (c) Three (d) Four
and at the end Y will be interchanged by Z then which will
(e) None of these
be the fourth letter to the right of 13th letter from left?
15. What will come in place of the question mark (?)?
(a) J (b) H
Q8B, ?, 5MH, 36J
(c) I (d) O
(a) TN7 (b) TM7
(e) None of these
8. What will come in place of the question mark (?) in the (c) TMH (d) TNH
following series? DGK GKP ? PVC (e) None of these
(a) GKV (b) KPV Directions (Qs. 16-20): Study the following arrangement carefully
(c) PVZ (d) KPU and answer the questions given below:
(e) None of these J1#P 4 E K 3 A D $ R U M 9 N 5 1% T V * H2 ÷
Directions for Q.9 : Question is based on the following alphabet F6G8QW
series. 16. How many such numbers are there in the above arrangement,
ABC DE FG HI JK LM NO PQRS T UV W XYZ each of which is either immediately preceded by or
9. If the alphabet is written in the reverse order and every immediately followed by a vowel or both?
alternate letter starting with Y is dropped, which letter will
(a) None (b) One
be exactly in the middle of the remaining letters of the
(c) Two (d) Three
alphabet.
(a) M (b) N (e) More than three
(c) O (d) M or O 17. Which of the following is exactly in the middle between the
Directions (Qs. 10 - 12): Answer the following questions based tenth from the left and the eighth from the right end in the
on the alphabet given below: above arrangement?
ABC DE FG HI JK LM NO PQRS T UV W XYZ (a) M (b) N
10. If the letters of the word PRODUCTIVE which are at the (c) 1 (d) 5
odd-numbered position in the English alphabet are picked (e) None of these
up and arranged in alphabetical order from left and if they 18. If the order of the last fifteen elements in the above
are now substituted by Z, Y, X and so on, beginning from arrangement is reversed, which of the following will be the
left, which letter will get substituted by W?
ninth to the right of the eleventh element from. the left end?
(a) No letter will get substituted by W
(b) O (c) E (a) G (b) %
(d) C (e) None of these (c) 8 (d) 3
11. What will come in the place of question (?) mark in the (e) None of these
following series? 19. How many such consonants are there in the above
GPW, GPUW, GIPUW, GIPSUW, ? arrangement, each of which is immediately preceded by a
(a) GIHPSUW (b) GIPQSUW symbol but not immediately followed by either a number or
(c) GIKPSUW (d) GIJPSUV a vowel?
(e) None of these (a) None (b) One
12. If the alphabets are written in reverse order after
(c) Two (d) Three
inter-changing alphabets from ‘D to L’ with those from ‘R to
Z’ respectively, which letter would be mid-way between W (e) More than three
ALPHABET AND NUMBERS ARRANGEMENT A- 7
20. Four of the following five are alike in a certain way based on Directions.(Qs.28-32): Study the following elements (letters,
their position in the above arrangement and so form a group. digits and symbols sequence) to answer the questions given
Which is the one that does not belong to that group? below:
(a) A $ E (b) % V N A B 7 C D 9 Z Y «P 2 M © K S 3 ­ 5 N T @
(c) 2 F V (d) 4 K 1
Note: ‘A’ is to the left of ‘B’ and ‘@’ is to right of ‘T’.
(e) 6 Q ÷
28. If each symbol of the above sequence is replaced with a
Directions (Qs. 21-24): Study the following information and letter and each digit is replaced with a new symbol, then
answer the questions given below: how many letters will be there in the sequence?
25 boy-scouts bearing names from A to Y were standing in a row. (a) 16 (b) 17
The teacher wanted to select various teams from among them. He
(c) 4 (d) 12
gave them random number from 3 to 8 as shown below:
(e) None of these
A B C D E F G H I J K L M N O P Q R S T UV W X Y
29. How many such digits are there in the sequence each of
4 4 6 3 5 4 3 3 5 67 5 8 5 8 3 3 8 4 8 6 5 4 6 6 which is immediately preceded as well as followed by letters?
21. If he decides to pick up those exclusive pairs of adjacent (a) None (b) One
boys whose numbers if totalled turn out to be exactly 12, (c) Two (d) Three
how many such pairs would be available?
(e) None of these
(a) Nil (b) Six
30. Which of the following letters is exactly midway between
(c) Five (d) Four
the letters falling between ‘C’ and ‘5’?
(e) None of these (a) Y (b) K
22. If he decides to pick up those boys who bear even numbers (c) P (d) M
and have boys bearing even numbers on both sides, how
(e) None of these
many boys will be picked up?
31. If each symbol of the above sequence is replaced with the
(a) One (b) Two
digits from ‘1’ to ‘9’ which are not there in the sequence,
(c) Three (d) Four
then what will be the sum of all digits? [Each symbol should
(e) None of these
be replaced with a different digit].
23. If he decides to pick up. those boys who bear odd numbers
(a) 19 (b) 45
but have boys bearing 7 and/or 8 on either side, how many
(c) 55 (d) 60
boys will be picked up?
(e) None of these
(a) Four (b) Three
32. If the first element from the left interchanges place with the
(c) Two (d) One
tenth element from the left, similarly, second with ninth,
(e) None of these
third with eighth, fourth and seventh, and so on, then which
24. If he decides to pick up only those boys who bear even
of the following will be seventh to the left of eight element
numbers but have on both sides students bearing odd
numbers, how many boys will be picked up? from the right?
(a) Six (b) Five (a) 9 (b) 7
(c) Four (d) Three (c) D (d) C
(e) None of these (e) None of these
Directions (Qs.25 -27): Study the following five numbers and Directions (Qs. 33-40): Study the following arrangement of
answer the questions given below. letters/symbols and answer the questions given below:
517 325 639 841 792 D F JT $ # PR ZQ * C MAB@ HK LS + ?
25. What will be last digit of the third number from top when 33. How many such symbols are there each of which is
they are arranged in descending order after reversing the immediately preceded by a symbol and immediately followed
position of the digits within each number? by a letter?
(a) 7 (b) 3 (a) One (b) Two
(c) 5 (d) 2 (c) Three (d) Four
(e) None of these (e) None of these
26. What will be the middle digit of the second lowest number 34. If the order of the first half of the arrangement is reversed
after the position of only the first and the second digits which of the following letters/symbols will be the fifth to
within each number are interchanged? the left of the fifteenth letter/symbol from the left?
(a) 5 (b) 2 (a) * (b) Q
(c) 7 (d) 3 (c) T (d) J
(e) None of these (e) None of these
27. What will be the first digit of the second highest number 35. If all the symbols of the above sequence are denoted by 7
after the position of only the second and the third digits and each letter is denoted by 5, then what will be the sum of
within each number are interchanged? all the elements of the sequence?
(a) 7 (b) 2 (a) 142 (b) 138
(c) 8 (d) 9 (c) 132 (d) 122
(e) None of these (e) None of these
A- 8 ALPHABET AND NUMBERS ARRANGEMENT
36. If all the symbols from the above sequence are dropped, (a) $ (b) 4
which letter will be seventh to the right of twelfth letter from (c) 7 (d) A
the right? (e) None of these
(a) H (b) B Directions (Qs. 46-48): Study the following letter/number series
(c) K (d) A carefully and answer the questions given below it.
(e) None of these W3 7 HJ Q T 5 1 2 GK4 F PT 6 L BE 9 4 DMR8 2 V
37. Which of the following is related to ‘FT’ in the same way as 46. If the numbers from the first half of the sequence are
‘DJ’ is related to‘? S’ ? dropped, which letter/number will be fifth to the right of
(a) L+ (b) KS sixth letter/number from the left?
(c) HL (d) + L (a) 6 (b) T
(e) None of these (c) Q (d) J
38. How many such letters are there in the above sequence (e) None of these
each of which occupies the same position from the left in 47. How many such letters are there in the sequence which are
the sequence as in the alphabet from left? immediately followed by a number and immediately
(a) None (b) One preceded by a letter?
(c) Two (d) Three (a) Four (b) Two
(e) None of these (c) Three (d) Five
39. Four of the following five are alike on the basis of their (e) None of these
position in the above sequence and hence form a group. 48. Four of the following five are alike in a certain way on the
Which of the following does not belong to that group? basis of their position in the sequence and so form a group.
(a) DJ ? (b) T # L Which is the one that does not belong to the group?
(c) FT + (d) PZ @ (a) WVH (b) JM1
(e) J # S (c) HRT (d) 78Q
40. Which of the following will be exactly midway between (e) 59G
fifth element from the left and eighth element from the right? Directions (Qs. 49-53): Study the following arrangement of digits,
(a) C (b) * letters and symbols and answer the questions given below:
(c) Q (d) M M K 3$ RE 5F %T UJ * 8 PHB N 2 IS #A3 7 D 4
(e) None of these 49. How many such consonants are there each of which is
Directions (Qs. 41-45): Study the following arrangement carefully either immediately preceded by a number and/or immediately
and answer the questions given below: followed by a symbol?
M £ 5 T R E 3 $ PJ 1 7 D 1 2 NA4 F H 6 * U 9 # V B @ W (a) None (b) One
41. If the positions of the first fourteen characters of the above (c) Two (d) Three
arrangement are reversed, which of the following will be the (e) None of these
twenty-second from the right end? 50. Four of the following five are alike in a certain way based on
(a) J (b) I the above arrangement and form a group.
(c) P (d) 3 Which is the one that does not belong to that group?
(e) None of these (a) 3RF (b) %U8
42. How many such numbers are there in the above arrangement, (c) 8H2 (d) I # 7
each of which is immediately preceded by a vowel and (e) H8U
immediately followed by a consonant? 51. If the positions of F and B are interchanged, similarly, the
(a) None (b) One positions of U and A are interchanged, how many such
(c) Two (d) Three vowels will be there each of which will be both immediately
(e) More than three preceded and immediately followed by a consonant?
43. What should come in place of the question mark (?) in the (a) None (b) One
series given below based on the above arrangement? (c) Two (d) Three
R3 £ PIE ?AF I (e) None of these
(a) DNJ (b) D21 52. What should come in place of the question mark (?) in the
(c) IN1 (d) N4D following series
(e) None of these R5K, U * F, ?, A7I
44. How many such consonants are there in the above (a) B28 (b) PBJ
arrangement each of which is immediately preceded by a (c) B2H (d) HNJ
symbol but not immediately followed by a number? (e) NJP
(a) None (b) One 53. Which of the following is neither immediately preceded by
(c) Two (d) Three a letter nor immediately followed by a letter?
(e) More than three (a) None (b) B
45. Which of the following is the fifth towards right of the (c) $ (d) 7
seventeenth from the right end? (e) None of these
ALPHABET AND NUMBERS ARRANGEMENT A- 9
Directions (Qs. 54-58) : Study the following arrangement carefully the second letter of that word is your answer. If no such
and answer the questions given below: word can be made mark ‘X’ as your answer, and if more than
B A 5 D % R I F H 6 # V 9 $ 3 E 7G 1 ÷ 2 M K X 8 U F W Z N one such word can be formed mark ‘M’ as your answer.
54. Which of the following is exactly in the middle of the (a) I (b) N
eleventh element from the left end and the fifteenth element (c) S (d) X
from the right end? (e) M
(a) V (b) $ 63. If each of the letters in the English alphabet is assigned odd
(c) 7 (d) E numerical value beginning A = 1, B = 3 and so on, what will
(e) None of these be the total value of the letters of the word ‘INDIAN’?
55. Four of the following five are alike in a certain way based on (a) 96 (b) 89.
their position in the above arrangement and so form a group. (c) 88 (d) 86
Which is the one that does not belong to that group? (e) None of these
(a) EG$ (b) RFD 64. If it is possible to make a meaningful word with the third,
(c) 127 (d) XUM the fifth, the sixth and the eleventh letters of the word
(e) H# MERCHANDISE, using each letter only once, which of the
56. How many such numbers are there in the above arrangement following will be the third letter of that word? If no such
each of which is immediately preceded by a consonant and word can be formed, give ‘X’ as answer and if more than
not immediately followed by a symbol? one such word can be formed, mark ‘T’ as answer.
(a) None (b) One (a) H (b) E
(c) Two (d) Three (c) R (d) X
(e) More than three (e) T
57. What should come in place of the question mark (?) in the 65. If it is possible to make a meaningful word with the first, the
following series based on the above arrangement? fifth, the ninth and the eleventh letters of the word
ADI F69 37 ÷ ? PENULTIMATE, using each letter only once, which of the
(a) 2KU (b) MXU following will be the third letter of that word? If no such
(c) MXF (d) XUM word can be made give ‘N’ as the answer and if more than
(e) H#
one such word can be formed give ‘D’ as the answer.
58. How many such consonants are there in the above
(a) E (b) P
arrangement each of which is immediately followed by a
(c) L (d) D
consonant but not immediately preceded by a digit?
(e) N
(a) None (b) One
66. How many such pairs of letters are there in the word
(c) Two (d) Three
CREDIBILITY each of which has only one letter between
(e) More than three
them in the word as also in the alphabet?
59. If it is possible to make a meaningful word with the second,
(a) None (b) One
the fourth, the sixth and the ninth letters of the word
PERMEABILITY, which of the following will be the first (c) Two (d) Three
letter of that word? If no such word can be formed give 'N' (e) More than three
as the answer. If only two such words can be formed give 67. If the letters in the word POWERFUL are rearranged as
‘D' as the answer and if more than two such words can be they appear in the English alphabet, the position of how
formed give ‘Z’ as the answer. many letters will remain unchanged after the rearrangement?
(a) M (b) L (a) None (b) One
(c) N (d) D (c) Two (d) Three
(e) Z (e) More then three
60. How many such pairs of digits are there in the number 68. How many such pairs of letters are there in the word
95137248 each of which has as many digits between them in PRODUCTION each of which has as many letters between
the number as when they are arranged in ascending order? them in the word as in the English alphabet?
(a) None (b) One (a) None (b) One
(c) Two (d) Three (c) Two (d) Three
(e) More than three (e) More than three
61. Find the two letters in the word EXTRA which have as 69. If it is possible to make only one meaningful word with the
many letters between them in the word as in the alphabet. If fourth, the fifth, the seventh and the eleventh letters of the
these two letters are arranged in alphabetical order which word PREDICTABLE, which of the following will be the
letter will come second? first letter of that word? If only two such words can be
(a) E (b) X formed, give ‘P’ as the answer; if three or more than three
(c) T (d) R such words can be formed, give ‘Z’ as the answer; and if no
(e) A such word can be formed, give ‘X’ as the answer.
62. If it is possible to make only one meaningful English word (a) D (b) T
from the sixth, the fifth, the twelfth and the fourth letters of (c) P (d) Z
the word IMAGINATIONS, using each letter only once, (e) X
A-10 ALPHABET AND NUMBERS ARRANGEMENT
70. If it is possible to make a meaningful word from the first, the (a) Code (b) Lack
fourth, the eighth, the tenth and the thirteenth letters of the (c) Meet (d) Deaf
word ESTABLISHMENT, using each letter only once, the (e) Road
last letter of that word is your answer. If more than one such 78. How many such pairs of letters are there in the word
‘CORPORATE’ each of which has as many letters in the
word can be formed write ‘P’ as your answer and if no such
same sequence between them in the word as in the English
word can be formed write ‘X’ as your answer. alphabet?
(a) X (b) P (a) None (b) One
(c) T (d) E (c) Two (d) Three
(e) M (e) More than three
71. How many meaningful words can be formed by replacing 79. If it is possible to make only one meaningful word with the
only the consonants in the word BREAK by the next letter second, the seventh, the tenth and the eleventh letters of
in the English alphabet and keeping the vowels unchanged? the word ‘TRADITIONAL’, what will be the second letter
(a) None (b) One of the word? If no such word can be formed, give ‘X’ as the
(c) Two (d) Three answer. If only two such words can be formed give ‘Y’ as
(e) More than there the answer and if more than two such words can be formed
72. How many such pairs of letters are there in the word give ‘Z’ as the answer.
ORIENTAL each of which has as many letters between them (a) L (b) I
in the word as in the English alphabet? (c) X (d) Y
(a) None (b) One
(e) Z
(c) Two (d) Three
80. How many pairs of letters are there in the word
(e) More than three
SPONTANEOUS which have number of letters between
73. The positions of the first and the eighth letters in the word
WORKINGS are interchanged. Similarly, the positions of them in the word one less than the number of letters between
the second and the seventh letters are interchanged, the them in Engiish alphabet?
positions of the third letter and the sixth letter are (a) Five (b) One
interchanged, and the positions of the remaining two letters (c) Four (d) Two
are interchanged with each other. Which of the following (e) Three
will be the third letter to the left of R after the rearrangement? 81. If each of the vowels i.e., A, E, I, O & U alongwith the 3rd
(a) G (b) S letter to its right in the alphabet are taken out and arranged
(c) I (d) N one after the other in the same order followed by the
(e) None of these remaining letters of the alphabet, which of the following
74. If only the consonants in the word MEAT are changed in will be 5th to the left of the 19th letter from the left in the
such a way that each of the them becomes the next letter in new arrangement?
the English alphabet and the remaining letters are kept ABC DE FG HI JK LM NO PQRS T UV W XYZ
unchanged, then how many meaningful words can be formed (a) G (b) H
with the new set of letters using each letter only once in a (c) J (d) W
word? (e) None of these
(a) None (b) Two 82. If each alphabet is assigned a sequential numerical value in
(c) Three (d) One terms of odd numbers on the basis of their position in the
(e) None of these English alphabet; viz. A = B = 3; C = 5 and so on. What will
75. If th e first and the second letters of the word be the value of the word DESK?
UNPRECEDENTED are interchanged with the last and the (a) 76 (b) 74
secondlast letters, and similarly the third and the fourth
(c) 64 (d) 68
letters are interchanged with the third and the fourth letters
(e) None of these
from the last respectively, and so on, then what will be the
83. If it is possible to make a meaningful word from the third,
7th letter to the right of the 3rd letter from the left?
(a) P (b) R sixth, eighth and eleventh letters of the word ‘DISTINGUISH’
(c) E (d) C using each letter only once, first letter of the word would be
(e) None of these your answer. If more than one such word can be formed, your
76. In the word ‘PRESENCE’, how many such pairs of letters answer would be ‘M’ and if no such word can be formed,
are there as have as many letters between its units in the answer is ‘X’.
word as there are in the English alphabet? (a) N (b) S
(a) One (b) Two (c) H (d) M
(c) Three (d) Four (e) X
(e) None of these 84. How many such pairs of letters are there in the word
77. If the letters in each of the following five words are first DREAMLAND each of which has as many letters between
rearranged in the alphabetical order and then the groups of them in the word as in the alphabet?
letters so formed are rearranged as in a dictionary, which (a) None (b) One
word would have its group of letters in the MIDDLE among (c) Two (d) Three
the five? (e) More than three
ALPHABET AND NUMBERS ARRANGEMENT A-11
85. If it is possible to make a meaningful word with the second, 93. If it is possible to make a meaningful word with the second,
the seventh, the ninth and the eleventh letters of the word the fifth and the eighth letters of the word ‘CARETAKER’,
ORGANISATION, which of the following will be the third which of the following will be the first letter of that word? If
letter of that word? If no such word can be formed, give ‘X’ no such word can be made, give X as answer. If more than
as the answer and if more than one such word can be made, one such word can be made, give M as the answer.
given answer as ‘M’. (a) A (b) E
(a) S (b) R (c) X (d) M
(c) T (d) X (e) None of these
(e) M 94. If it is possible to make a meaningful word with the first, the
86. How many pairs of letters are there in the word ‘NURSING’ fourth, the seventh and the eleventh letters of the word
which have as many letters between then as in the alphabet? ‘INTERPRETATION’, which of the following will be third
(a) One (b) Three letter of that word? If more than one such word can be made
(c) Five (d) Six give M as the answer and if no such word can made, give X
(e) None of these as the answer.
87. If it is possible to make a meaningful word from the second, (a) I (b) R
fourth, tenth and twelfth letters of the word ADVERTISEMENT, (c) X (d) M
using each letter only once, write the last letter of the word (e) None of these
as your answer. If more than one such word can be formed, 95. If it is possible to make a meaningful word with the third,
write ‘P’ as your answer and if no such word can be formed, the fifth, the seventh and the tenth letters of the word
write ‘X’ as your answer. ‘PROJECTION’ which of the following is the third letter of
(a) P (b) X that word? If no such word can be made, give X as the
(c) N (d) M answer. If more than one such word can be made, give M as
(e) D the answer.
88. How many pairs of letters are there in the word CRYS- (a) O (b) N
TALLIZE, which have as many letters between them as in (c) X (d) M
the alphabet? (e) None of these
(a) 1 (b) 2 96. If it is possible to make a meaningful word with the first, the
(c) 3 (d) 4 third, the seventh and the ninth letters of the word
(e) None of these SEPARATION, which of the following will be the third letter
89. If letters in the word UNIVERSAL are arranged in the of that word? If no such word can be made, give X as the
alphabetical order and each letter is assigned numerical answer and if more than one such word can be made, give
value equal to its serial number from the left in this rearranged M as the answer.
order, what is the difference in the total of numerical values (a) O (b) P
of vowels and that of consonants? (c) M (d) X
(a) 19 (b) 17 (e) None of these
(c) 21 (d) 20 97. If the second, third, fifth, eighth and ninth letters of the
(e) None of these word CONTEMPLATION are combined to form a meaningful
90. How many pairs of letters are there in the word EXCLUSIVE word, what will be the middle letter of that word ? If more
which have as many letters between them as in the alphabet? than one such words can be formed, your answer is X and
(a) 2 (b) 3 if no such word can be formed, your answer is Y.
(c) 4 (d) Nil (a) X (b) O
(e) None of these (c) A (d) Y
91. If it is possible to make a meaningful word from the fifth, (e) None of these
seventh, eighth, ninth and thirteenth letters of the word 98. How many such pairs of letters are there in the word
‘EXTRAORDINARY’ using each letter only once, write the CORPORATE each of which has as many letters in the same
second letter of that word as your answer. If no such word sequence between them in the word as in the english
can be formed write ‘X’ as your answer and if more than one alphabet ?
such word can be formed, write ‘M’ as your answer. (a) None (b) One
(a) A (b) I (c) Two (d) Three
(c) R (d) M (e) None of these
(e) X 99. If the first three letters of the word COMPREHENSION are
92. The letters of the name of a vegetable are I, K, M, N, P, P, U. reversed, then the last three letters are added and then the
If the letters are rearranged correctly, then what is the last remaining letters are reversed and added, then which letter
letter of the word formed ? will be exactly in the middle. ?
(a) M (b) N (a) H (b) N
(c) K (d) P (c) R (d) S
(e) None of these (e) None of these
A-12 ALPHABET AND NUMBERS ARRANGEMENT
100. If the first and second letters in the word DEPRESSION 105. If it is possible to form a word with the first, fourth, seventh
were interchanged, also the third and the fourth letters, the and eleventh letters of the word ‘SUPERFLOUS’, write the
fifth and the sixth letters and so on, which of the following first letter of that word. Otherwise, X is the answer.
would be the seventh letter from the right ? (a) S (b) L
(a) R (b) O (c) O (d) X
(c) S (d) P (e) None of these
(e) None of these 106. How many independent words can ‘HEARTLESS’ be
101. If the positions of the third and tenth letters of the word divided into without changing the order of the letters and
DOCUMENTION are interchanged, and likewise the using each letter only once ?
positions of the fourth and seventh letters, the second and (a) Two (b) Three
sixth letters is interchanged, which of the following will be (c) Four (d) Five
eleventh from the right end ? (e) None of these
(a) C (b) I 107. If the last four letters of the word ‘CONCENTRATION’ are
(c) T (d) U written in reverse order followed by next two in the reverse
(e) None of these order and next three in the reverse order and then followed
102. Arrange the given words in alphabetical order and tick the by the first four in the reverse order, counting from the end
one that comes in the middle. which letter would be eighth in the new arrangement ?
(a) Restrict (b) Rocket (a) N (b) T
(c) Robber (d) Radom (c) E (d) R
(e) None of these (e) None of these
103. Select the combination of numbers so that letters arranged 108. How many independent words can ‘STAINLESS’ be divided
accordingly will form a meaningful word. into without changing the order of the letters and using
R A C E T each letter only once ?
1 2 3 4 5 (a) Nil (b) One
(a) 1, 2, 3, 4, 5 (b) 3, 2, 1, 4, 5 (c) Two (d) Three
(c) 5, 2, 3, 4, 1 (d) 5, 1, 2, 3, 4 (e) None of these
(e) None of these 109. Select the combination of numbers so that the letters
104. Rearrange the first four letters, in any way, of the word arranged accordingly will form a meaningful word.
DECISION. Find how many words can be formed by using VA R S T E
all the four words. (a) 2, 3, 1, 6, 4, 5 (b) 4, 5, 2, 3, 1, 6
(a) One (b) Two (c) 6, 3, 4, 5, 2, 1 (d) 3, 2, 4, 5, 6, 1
(c) Three (d) More than three (e) None of these
(e) None of these

ANSWER KEY
1 (a) 14 (c) 27 (a) 40 (c) 53 (a) 66 (c) 79 (e) 92 (b) 105 (b)
2 (c) 15 (a) 28 (a) 41 (d) 54 (e) 67 (b) 80 (a) 93 (d) 106 (b)
3 (b) 16 (d) 29 (d) 42 (c) 55 (b) 68 (d) 81 (a) 94 (d) 107 (d)
4 (b) 17 (d) 30 (c) 43 (e) 56 (b) 69 (d) 82 (b) 95 (e) 108 (c)
5 (c) 18 (a) 31 (b) 44 (d) 57 (c) 70 (b) 83 (b) 96 (c) 109 (b)
6 (a) 19 (b) 32 (d) 45 (b) 58 (e) 71 (c) 84 (e) 97 (b)
7 (e) 20 (e) 33 (a) 46 (a) 59 (e) 72 (c) 85 (e) 98 (c)
8 (b) 21 (e) 34 (e) 47 (d) 60 (b) 73 (d) 86 (b) 99 (d)
9 (b) 22 (c) 35 (d) 48 (b) 61 (a) 74 (a) 87 (e) 100 (d)
10 (b) 23 (a) 36 (b) 49 (e) 62 (e) 75 (b) 88 (c) 101 (c)
11 (c) 24 (b) 37 (d) 50 (e) 63 (a) 76 (c) 89 (b) 102 (a)
12 (b) 25 (b) 38 (b) 51 (b) 64 (e) 77 (e) 90 (b) 103 (d)
13 (e) 26 (d) 39 (e) 52 (a) 65 (d) 78 (d) 91 (d) 104 (a)
ALPHABET AND NUMBERS ARRANGEMENT A-13

Answers &
Explanations
1. (a) After dropping every third letter, we get 18. (a) After the changing, the series becomes as follows;
ABD E GHJ KMNP QST V WYZ J 1 # P4 E K 3AD $ RU M 9 NW Q8 G 6 F ¸ 2 H
(11– 7 =) 4th from the right. * V T % 15
3. (b) W2 N 1 V 9 G 2 P 4 X 6 K 7 R 1 T 8 L 3 H 5 Q 8 U 2 J Now, ninth to the right of the eleventh element from
4. (b) Positions corresponding to the multiples of five are E, the left® (11 + 9 =) 20th element from the left, i.e., G.
J, O, T, Y and that of multiples of seven are G, N and U. 19. (b) We have to look for Symbol Consonant – Consonant
Hence, the total number of remaining letters in the sequence and Symbol–Consonant–Symbol sequences.
series = 26 – 8 = 18 J I # P 4 E K 3 A D $ R U M 9 N 51 % T V H
5. (c) After reversing the first half of the given series. the 2¸ F6G8QW
series becomes as follows: Only T is such a consonant.
M L K J I HG F E D C B AN O PQ R S T UVW XYZ 20. (e) See the difference between each two successive
There are 14 letters between K and R. element.
6. (a) The changed sequence becomes (a) A +2 $ – 5 E; A+2=$–5=E
T UV W XY Z HI J KL M N OP Q RS A BC D E F G (b) % + 2V – 5 N; %+2=V–5=N
3rd letter to the left of 18th letter from right (c) 2 + 2 F – 5 V; 2+2 = F–5 =V
= (18 + 3) = 21th from right
= (26 + 1 = 21) = 6th from left = F (d) 4 + 2 K – 51; 4+2=K–5=1
7. (e) 4th to the right of 13th from left (e) 6 + 3 Q – 5 ¸ ; 6 +2=8–5=2
= (13 + 4 =) 17th from left = Q
Now, in the changed sequence, MNOPQR becomes Note that the difference between two successive
RQPONM. Thus N takes the place of Q. Hence in the elements in 5 is not similar to others.
changed get M in the req. place. 21. (e) KL, RS, ST, XY — We have four such pairs of adjacent
8. (b) First letter moves + 3, + 4, + 5 places forward, second boys. But note the phrase “exclusive pairs” Since, S
letter + 4, + 5, + 6, and third letter + 5, + 6, + 7 in is common in RS and ST, we can have only one of the
consecutive terms. two pairs. Hence, three pairs.
9. (b) Cancelling every second letter after reversing the 22. (c) B, S and X
alphabet the series becomes. 23. (a) L, N, P and Q.
ZXVT RPNLJHFDB 24. (b) F, J, M, O and V.
The middle letter is N. 25. (b) Here, the given numbers are:
517 325 639 841 792
10. (b) ZYXWV
After reversing the numbers become as follows:
13. (e) Q D T P 8 6 9 F G 7 B 4 H J 3 K 2 M N 5
715 523 936 148 297
(9 – 5=) 4th from right, which is 2. When arranged in descending order the numbers
14. (c) Q D T P 5 2 3 F G 4 B 7 H J 9 K 6 M N 8 become as follows:
15. (a) The first element of each group is two elements forward 936 715 523 297 148
to the corresponding element of the previous group. Now, the third number from top is 523. Hence, the last
The second element of each group is one element digit of 523 is 3.
backward and the third element of each group is one 26. (d) After interchanging the first and the second digits,
element forward to the corresponding elements of the numbers become as follows;
previous group as given in the sequence. 157 235 369 481 972
16. (d) We have to look for Vowel-Number and Number-Vowel When arranged in descending order the numbers
sequences. become as follows;
J 1 # P 4 E K 3 A D $ R U M 9 N 51 % T V * 972 481 369 235 157
H 2 ¸F6 G8 QW Here, the second lowest number is 235.
4, 3 and 5 are the required numbers. Hence, middle digit of 235 is 3.
27. (a) If the positions of only the second and the third digits
17. (d) D $ RU M 9 N 5 1 % T V * H 2 within each number are interchanged, the numbers
E5555555555F E555555555F
7 elements 7 elements become as follows.
571 352 693 814 729
A-14 ALPHABET AND NUMBERS ARRANGEMENT
Now, when the numbers are arranged in descending
æ 15 + 5 ö
order, we get left, is ç = ÷ 10th element from left, i.e., Q.
814 729 693 571 352 è 2 ø
Here, 729 is the second highest number 41. (d) After changing the series becomes as follows :
Hence, the first digit of 729 is 7. I D 7 1 J P $ 3 E R T 5 £ M 2 N A4 F H 6 H U 9 # VB @ W
28. (a) There are 21 elements in the series. Among them, there Now, twenty-second element from the right end is 3.
are only 5 digits. Since, symbols replaced by letters 42. (c) We have to look for
and digits are replaced by symbols, ultimately there Vowel-number-consonant sequence.
will be (21 – 5 =) 16 letters in the series. M £ 5TR E 3 $ PJ 1 7 D I 2 NA4 F H 6 H U9 #V B @W
29. (d) We have to search for Letter-Digit-Letter sequence.
Only 2 and 4 are such numbers.
Note the bold digits given in the series below.
43. (e) D 2 J
AB7CD9ZY*P2M©KS3­5NT@
30. (c) The letters falling between C and 5 are as follows: R +4 P +4 D +4A
DZYPMKS. Hence, P is the required letter. 3 +4 1 +4 2 +4F
31. (b) Note that there are already five digits (7, 9, 2, 3, 5,) in £ +4 E +4 J +4I
the series. If the four symbols are replaced by the 44. (d) M £ 5 T R E 3 $ P J 1 7 D I 2 N A 4 F H 6 U9 # VB@W
remaining digits from 1 to 9 (1, 4, 6, 8) then sum of the
45. (b) Fifth element towards right of the seventeenth element
digits = 1 + 2 + .....
from the right end implies twelfth element from the
9 ´10 right end. Hence, the required element is 4.
+ 9= = 45
2 46. (a) If the numbers from the first half of the sequence are
dropped, the series becomes as follows:
n × (n +1)
[Sum of n natural numbers = ] WHJ Q T GK FPT 6 LBE 9 4 DM R 8 2 V
2
Hence, 5th to the right of the sixth letter/ number from
32. (d) Seventh to the left of eighth element from right
the left Þ l lth element from the left, ie 6.
= (7 + 8 =) 15th element from the right.
47. (d) Here, we have to find out letter-letter-number
In original series 'Z' occupies the 15th place from right
but after the changes 'Z' interchanges its position sequence. Bold letters in the sequence given below
with 'C'. represent those letters:
33. (a) D F J T $ # PR ZQ * C MAB @ HK LS + ? W3 7HJ QT512GK 4F PT6LBE 94DM182V
only $ # P is the required answer. 48. (b) Following is the common property found in others:
34. (e) * Q Z RP# $ TJ FD C MAB@ HK LS + ? If first element of each group occupies nth position in
35. (d) Number of total symbols = 6; Number of total letters the given sequence then the last element of the
= 16. Since, all the symbols are denoted by 7 and all corresponding group occupies (n + 3)th position in
letters are denoted by 5, sum of the elements of the the given sequence.
sequence = 6 × 7 + 16 × 5 = 122 49. (e) MK 3 $ RE 5 F% T UJ * 8 PH BN 2 I S#A3 7 D4
36. (b) When all the symbols are dropped the series becomes 50. (e) Here the rule followed is: All the groups consist of
as follows: three elements. Where, 1st element + 2 = 2nd element
D F JT PR ZQ C MAB HK LS and 2nd element + 3 = 3rd element.
Now, seventh to the right of twelfth letter from the 51. (b) After re-arrangement the new arrangement will be
right = (12 – 7 =) 5th letter from the right, i.e., B. M K 3 $ R E 5 B % TAJ * 8 PH F N 2 I S # U 3 7 D 4
37. (d) Compare ‘DJ’ and ‘?S’. ‘D’ is the first element from left
54. (e) B A 5 D % R I * F H 6 # V 9 $ 3 E 7 G
end of the series and ‘?’ is the first element from right
end. Similarly, ‘J’ and ‘S’ are third elements from left 1÷2 M K X 8 U F W Z N
and right end respectively. Hence, ‘FT’ is related Hence, the required element is ‘9’.
to' + L'. 55. (b) See the difference between each two successive
38. (b) D F J T $ #P R ZQ * C M A B @ H K L S + ? elements.
ABC DE F G HI JK LMNO P QRS T UV (a) E + 2 G – 4 $
39. (e) Except it there is only one element between first and (b) R + 3 F – 5 D
second letter of each group of words when the position (c) 1 + 2 2 – 4 7
of the letters in the series is taken into consideration.
(d) X + 2 U – 4 M
40. (c) Eighth element from right = (22 + 1 – 8 =)15th element
(e) H + 2 # – 4 *
from left.
Note that the difference between two successive
Hence, the required element which is exactly midway
elements in (b) is not similar to others.
between 5th element from left and 15th element from
ALPHABET AND NUMBERS ARRANGEMENT A-15
56. (b) We have to look for Consonant-Number-Letter and 69. (d) The specified letters are D, I, T and E. Words formed
Consonant-Number-Number sequences. by these letters are as follows:
BA 5 D % R I * F H 6 # V 9 $ 3 E 7 G 1 ÷ 2 M K X 8 U F (i) EDIT (ii) DIET
WZN (iii) TIDE (iv) TIED
Hence, only one such number (8) exists in the above 70. (b) Here specified letters are: E, A, S, M and T. Words
series. formed from these letters are as follows:
57. (c) 1. STEAM 2. MATES
A D I F 6 9 3 7 ÷ 3. TEAMS
+2 +3 +2 +2 +3 +2 +2 +3 71. (c) Here letters are: B, R, E, A and K. When the consonants
are replaced by the next letter then we have C, S, E. A
Hence, the missing group will be MXF
and L to form words. These words are as follows:
58. (e) We have to look for the sequences Letter-Consonant-
1. SCALE 2. LACES
Consonant, and Symbol-Consonant-Consonant in the
given series.
72. (c) O R I E NTAL
BA5D% RI * FH6 #V9 $3 E 7 G
­ 73. (d) After interchanging, the order of the letters in the word
1÷2 M KX 8 U F W Z N becomes as follows:
­ ­ ­ SGNIKROW
There are four such consonants as shown above. Thus, the third letter to the left of R is N.
59. (e) Here specific letters are E, M, A and L. Words formed 74. (a) The new set of letters are: N, E, A, U. Hence no
with these letters are as follows meaningful word can be made.
1. LAME 2. MALE 3. MEAL 75. (b) 7th letter to the right of 3rd letter from the left Þ 10th
Since, no. of words formed by the given letters is more letter from the left. After changing the word becomes
than two, our answer is choice (e). as follows
60. (b) Here, given number is 95137248. When the number is
arranged in ascending order number becomes as DETNEDECERPNU
follows 12345789. Now, look at the pairs: 35. What do 76. (c)
you observe? These pairs are those pairs each of which
has as many digits between them in the number as PRESENCE
when they are arranged in ascending order.
E X T R A 77. (e) When the letters in each of the words are arranged in
61. (a)
alphabetical order it becomes as follows: cdeo, ackl.
When E and A are arranged in alphabetical order then eemt. adef and ador. Now when the words are
i.e. AE, E will be second. rearranged as in a dictionary then their respective
62. (e) SING, SIGN position becomes as follows: ackl, adef, ador, cdeo
63. (a) INDIAN = 17 + 27 + 7 + 17 + 1 + 27 = 96 and eemt.
64. (e) Selected letters of the given word are R, H, A and E. By
using each letter only once we can make the following 78. (d) C O R P O R A T E
words:
Note that we have to find the pairs keeping the
1. HEAR 2. HARE
sequence of the letters of pair according to their
This is more than one.
sequence in English alphabet. Therefore go for search
65. (d) The letters are: P, L, A, E. Meaningful words: PALE, only from left to right.
LEAP, PEAL.
79. (e) Here specified letters are: R, I, A and L. Words formed
C R E D I B I L I T Y with these letters are:
66. (c)
1. RAIL 2. LIAR 3. LAIR
67. (b) P O W E R F U L
E F LO PR UW 80. (a) S P O N TAN E O U S
only U remains unchanged.
68. (d) PI, RU and ON. In each shown pairs there is one letter less than the
number of letters between them in English alphabet.
81. (a) Arranging English alphabet according to the instruc-
PRODUCTION
tions given, we get
A-16 ALPHABET AND NUMBERS ARRANGEMENT
ADEH ILORUXBCFGJKMNPQSTV 97. (b) Only one meaningful word ‘ALONE’ can be made. O is
WYZ the middle letter.
(19 – 5 =) 14th from the left 98. (c)
82. (b) According to the question, the value of all the letters
will be: its sequential numerical value as in English C O R P O R A T E
alphabet + the no. of letters before it in English
alphabet.
Three pairs — (P, R), (R, T) and (P, O) have as many
Hence D = 4 + 3 = 7, S = 19 + 18 = 37 letters between them in the word as in the English
E = 5 + 4 = 9 K = 11 + 10 = 21 alphabet. But since the letters must be in the same
\ Numerical value of DESK = 7 + 9 + 37 + 21 = 74 sequence in the word as in English alphabet, so that
83. (b) The 3rd, 6th, 8th and 11th letters are S, N, U and H desire pairs are (P, R) and (R, T) only.
respectively. The word that can be made is SHUN. 99. (d) Clearly, we have :
84. (e) Four COMPREHENSION ® (COM) (PREHENS) (ION)
® COMIONSNEHERP
The middle letter is the seventh letter, which is S.
D R E A M L A N D 100. (d) The new letter sequence is EDRPSEISNO.
The seventh letter from the right is P.
85. (e) The specified letters are R, S, T and O. Meaningful
word formed from these letters are SORT and ROTS. D E P R E S S I O N

86. (b) N U R S I N G 1 2 3 4 5 6 7 8 9 10
87. (e) The respective letters are D, E, M and N. Of these 101. (c)
letters, only MEND can be formed.
C R Y S T A L L I Z E
88. (c)
D O C U M E N T A T I O N
89. (b) Arranging ‘UNIVERSAL’ alphabetically and assigning
values from leftward, we get A E I L N R S U V 1 2 3 4 5 6 7 8 9 10 11 12 13
123456789
Now, sum of position nos. of vowels (A, E, I, U)
1+ 2 + 3 + 8 = 14 102. (a) Arranging the words in alphabetical order, we have
and sum of position nos. of consonants (L N, R, S, V) Random, Restaurant, Restrict, Robber, Rocket.
= 4 + 5 + 6 + 7 + 9 = 31
So the work in the middle is Restrict and the correct
Difference = 31 – 14 = 17 answer is (a).
90. (b) Letter are (X, U), (L,I) and (E,C). 103. (d) Clearly, the given letters, when arranged in the order 5,
91. (d) A, R, D, I, Y. We can make DIARY, DAIRY 1, 2, 3, 4 from the word ‘TRACE’.
92. (b) PUMPKIN 104. (a) The first four letters are D, E, C, I and only word DICE
93. (d) The second, fifth and eighth letters of the word can be formed so the answer is (a).
CARETAKER are A, T and E respectively. The words 105. (b) The letters selected are S, E, L and S respectively. The
formed are EAT, ATE and TEA. word formed is LESS. The first letter is L.
94. (d) The first, fourth, seventh and eleventh letters of the 106. (b) The words are HE, ART, LESS
word INTERPRETATION are I, E, R and T respectively. 107. (d) The new letter sequence is
The words formed are TIER, RITE and TIRE.
NOITARTNECNOC
95. (e) The third, fifth, seventh and tenth letters of the word
The eighth letter from the end is R.
PROJECTION are O, E, T and N respectively. The words
108. (c) Only two such words can be formed. The words are
formed are NOTE and TONE.
STAIN and LESS.
96. (c) The first, third, seventh and ninth letter of the word
109. (b). Clearly the given letters, when arranged in the order
SEPARATION are S, P, T and O respectively. The words
formed are SPOT, POTS and TOPS. 4, 5, 2, 3, 1, 6 form the word ‘STRAVE’.
Analogy &
Classification
2 Chapter
ANALOGY 4. Worker & product based analogy: This type of analogy
The meaning of analogy is ‘similar properties’ or similarity. If an gives a relationship between a person of particular
object or word or digit or activity shows any similarity with profession and his/her creations.
another object or word or digit or activity in terms of properties, Examples:
type, shape, size, trait etc., then the particular similarity will be
called analogy. For example, cricket : ground and chess: table are Batsman : Run
the analogous pairs (why?). In fact, both pairs of words have Writer : Book
similar relationship in terms of place of playing as cricket is played Author : Novel
in the ground and similarly chess is played on the table. In this
chapter we will discuss different types of analogy because Singer : Song
problems based on analogy are an important category of questions Poet : Poem
to be asked in almost all examinations of competitive level. Journalist : News
TYPES OF ANALOGY.
5. Causes & effect based analogy: In such type of analogy
1. Tool & object based analogy: This establishes a relationship 1st word acts and the 2nd word is the effect of that action.
between a tool and the object in which it works. Similar
relations has to be discovered from answer choices. Examples:
Examples: Work : Tiredness
Pencil : Paper Bath : Freshness
Pen : Paper Race : Fatigue
Scissors : Cloth Shoot : Kill
Saw : Wood 6. Opposite relationship (Antonym) based analogy : In such
Eraser : Paper type of analogy the two words of the question pair are
2. Synonym based analogy : In such type of analogy two words opposite in meaning. Similar relations has to be discovered
have similar meaning. from the answer choice word pairs.
Examples: Examples:
Big : Large
Poor : Rich
Huge : Gigantic
Fat : Slim
Endless : Eternal
Tall : Short
Thin : Slim
Benevolent : Kind Big : Small
Notion : Idea Light : Dark
Huge : Big Avoid : Meet
3. Worker & tool based analogy: This establishes a 7. Gender based analogy: In such type of analogy, one word
relationship between a particular tool and the person of is masculine and another word is feminine of it. In fact, it is
that particular profession who uses that tool. a ‘male and female’ or ‘sex’ relationship.
Examples: Examples:
Writer : Pen
Man : Woman
Painter : Brush
Boy : Girl
Cricketer : Bat
Nephew : Niece
Blacksmith : Hammer
Barber : Scissors Bull : Cow
Hunter : Gun Duck : Drake
A-18 ANALOGY & CLASSIFICATION
8. Classification based analogy: This type of analogy is based 14. Adult & young one based analogy : In such type of analogy,
on biological, physical, chemical or any other classification. the 1st word is the adult one and 2nd word is the young one
In such problems the 1st word may be classified by the 2nd of the 1st word or vice-versa.
word and vice-versa.
Examples:
Examples:
Cow : Calf
Cow : Animal
Girl : Human Human : Child
Oxygen : Gas Dog : Puppy
Water : Liquid Duck : Duck ling
Snake : Reptile 15. Subject & specialist based analogy: In such type of analogy
Parrot : Bird the 2nd word is the specialist of 1st word (subject) or vice-
9. Function based analogy : In such type of analogy, 2nd versa.
word describes the function of the 1st word. Examples:
Examples:
Heart : Cardiologist
Singer : Sings
General : Commands Skin : Dermatologist
Player : Plays 16. Habit based analogy: In this type of analogy 2nd word is the
Surgeon : Operates habit of 1st and vice-versa.
10. Quantity and unit based analogy: In such type of analogy Examples:
2nd word is the unit of the first word and vice-versa. Cat : Omnivorous
Examples: Tiger : Carnivorous
Distance : Mile Cow : Herbivorous
Mass : Kilogram Goat : Herbivorous
Length : Meter 17. Instrument and measurement based analogy: We see in
11. Finished product & raw material based analogy : In such this type of analogy, the 1st word is the instrument to
type of analogy the 1st word is the raw material and 2nd measure the 2nd word and vice-versa:
word is the end product of that raw material and vice-versa. Examples:
Examples: Hygrometer : Humidity
Yarn : Fabric Barometer : Pressure
Milk : Curd Thermometer : Temperature
Flour : Bread Sphygmomanometer : Blood pressure
Latex : Rubber 18. Individual & group based analogy : Second word is the
Grape : Wine group of 1st word (or vice-versa) in such type of analogy.
Fruit : Juice Examples:
12. Utility based analogy : In such type of analogy the 2nd word Cow : Herd
shows the purpose of the 1st word or vice-versa. Sheep : Flack
Examples: Grapes : Bunch
Pen : Writing Singer : Chorus
Food : Eating 19. State & capital based analogy: 1st word is the state and 2nd
Chair : Sitting word is the capital of that state (1st word) (or vice-versa) in
Bed : Sleeping the analogy like this.
Bat : Playing Examples:
13. Symbolic relationship based analogy: In such type of Bihar : Patna
analogy, the 1st word is the symbol of the 2nd word and West Bengal : Kolkata
vice-versa. Maharashtra : Mumbai
Examples: Karnataka : Bangluru
White : Peace Note: Analogy based on country and capital is very similar
Red : Danger to this type of analogy in which we put name of the country
Black : Sorrow in place of the name of state and country capital in place
Red cross : Hospital of state capital. For example India: New Delhi and
Nepal : Kathmandu.
Swastika : Fortune
ANALOGY & CLASSIFICATION A-19

20. Analogy based on individual & dwelling place : In such Example:


type of analogy 1st word is the individual & 2nd word is the (i) LAIN : NAIL : : EVOL : Love
dwelling place of that individual (1st word) and vice-versa. Here the 1st term gets reveresed to produce the 2nd term and
Examples: similar relation is shown in between 3rd and 4th term.
Horse : Stable (ii) ABCD : OPQR : : WXYZ : KLMN
Bee : Apiary In (ii) each letter of the 1st group ‘ABCD’ is moved fourteen
Dog : Kennel steps forward to obtain the corresponding letter of the 2nd
Birds : Aviary group ‘OPQR’. A similar relation is established between the
Monk : Monastery third group ‘WXYZ’ and the fourth group ‘KLMN.’
Human : House CLASSIFICATION
21. Analogy based on worker and working place: In this type You must have in your mind that what does classification mean.
of analogy the 1st word represents a person of particular In fact, in classification we take out an element out of some given
profession and 2nd word represents the working place of elements and the element to be taken out is different from the rest
that person (1st word) and vice-versa. of the elements in terms of common properties, shapes, sizes,
Examples : types, nature, colours, traits etc. In this way the rest of the
elements form a group and the element that has been taken out is
Doctor : Hospital
not the member of that group as this single element does not
Clerk : Office
possesses the common quality to be possessed by rest of the
Cook : Kitchen elements. For example, if we compare the elements like, lion, cow,
Professor : College tiger, panther, bear and wolf then we find that this is a group of
Teacher : School animals. How do we classify them? To understand this let us see
22. Analogy based on topic study: 1st word is the study of the the presentation given below :-
2nd word (or vice-versa) in the analogy like this.
Lion Cow Tiger Panther Bear Wolf
Examples:
Birds : Ornithology
Earth quakes : Seismology Wild Domestic Wild Wild Wild Wild
animal animal animal animal animal animal
Eggs : Zoology
23. Analogy based on letters (or meaningless words) Here, if we want to separate out one animal then definitely that
Case I : (Forward alphabetical sequence) animal will be cow because cow is the only animal in the group
which is a domestic animal. Rest of the animals (Lion, Tiger,
Examples:
Panther, Bear and Wolf) are wild animals. Hence rest of the animals
CD : FG : : PQ : UV
(Lion, Tiger, Panther, Bear & Wolf) form a group of wild animals
Here CD and FG are in the natural alphabetical sequence.
separating out the domestic animal (Cow).
Similarly, PQ & UV are in the natural alphabetical sequence.
Similarly, out of 6 letters A, M, N, U, P & Q, we will take out A and
Case II: (Backward or opposite alphabetical sequence)
form a group of 5 letters M, N, U, P & Q because out of given six
Example: letters only A is a vowel while rest of the letters form a group of
DC : GF : : QP : VU consonants.
In fact this case is opposite of case I Types of classification :
Case III: ( Vowel – consonant relation) (1) Letter/meaningless word based classification
Example (2) Meaningful word based classification
ATL : EVX : : IPR : ORS (3) Digit based classification
Here, the 1st two words start with the 1st two vowels A & E (4) General knowledge based classification
and the next two words start with the next two vowels I & Now we will discuss these three types of classifications one by
O. Last two letter of every word are consonants. one:-
Case IV: Example (Skip letter relation) (1) Letter/meaningless word based classification :- Such
ABC : FGH : : IJK : NOP classifications are based on letters of English alphabet. So
Here between ABC & FGH two letters skip and they are D & many groups of letters are given in the question in which
E. Similarly, between IJK & NOP two letters skip and they one group is different from remaining groups and hence the
are L & M. different group will be our answer.
Case V: (Jumbled letters relation)
A-20 ANALOGY & CLASSIFICATION

EXAMPLE 1. Find the odd word out of the following options. Sol. (d) is the correct answer because this is the only word which
has 5 letters while the remaining words have 4 letters and
(a) PQT (b) UVY hence options (a), (b), (c) & (e) form a group separating out
(c) DEH (d) IJN option (d).
(e) FGJ EXAMPLE 4. Out of the 5 words given below, four have certain
thing common and so they form a group. Find out the word which
Sol. (a) P Q R S T (b) U V W X Y one is not a part of that group.
2 letter gap 2 letter gap (a) Slim (b) Trims
(c) Greets (d) Grid
(c) D E F G H (d) I J K L M N (e) Fight
Sol.: Option (c) is the correct option because this is the only
2 letter gap 3 letter gap option which has two vowels while the other options have
only one vowel. Let us see the following presentation:-
(e) F G H I J
Sl i ms Tr i ms
(a) (b)
2 letter gap 1 vowel 1 vowel

As it is clear that except option (d) all the other options have 2 Gr ee ts Gr i d
letters gap between 2nd and third letters and the 1st two letters (c) (d)
are in consecutive order. While in case of option (d) 1st two 2 vowels 1 vowel
letters are in consecutive order but there is 3 letters gap between
F i ght
2nd and third letter separating it out of the remaining group of the (e)
letters. Hence option (d) is the correct option. 1 vowel
3. Digit based classification :- In such type of classifications
EXAMPLE 2. Following are given four options and out of digits or numbers are given to find out one number that is
them 3 form a group in terms of some similarity. Find out the not a part of the group of remaining numbers.
option which does not fit into that group.
EXAMPLE 5. Find the odd number out.
(a) LMNO – ONML (b) PQRS – SRQP
(a) 122 (b) 128 (c) 199
(c) IJKL – LKJI (d) UVWX – VUXW
(d) 200 (e) 388
L M N O O N M L Sol.: Option (c) is the correct option because this is an odd number
Sol. (a) ¾¾
®
1 2 3 4 4 3 2 1 while all the other options are even numbers.

P Q R S S R Q P EXAMPLE 6. Four of the following numbers have some


(b) ¾¾
®
1 2 3 4 4 3 2 1 similarity and hence they form a group. Find out the number
which does not suit in the group.
I J K L L K J I (a) 7842 (b) 7562 (c) 7122
(c) ¾¾
®
1 2 3 4 4 3 21 (d) 7722 (e) 8952
Sol.: Option (e) is the correct answer as except option (e) all
U V W X V U X W
(d) ¾¾
® other options start with 7 & end with 2.
1 2 3 4 2 1 4 3
4. General knowledge based classification :- Such
Above presentation makes it clear that (d) does not fit into classification is done on the basis of our general knowledge.
the groups. No doubts that this is a word based classification but
2. Meaningful words based classification :- In such type of without having general knowledge this type of questions
classification we have to take odd word out of the given can not be solved.
group of meaningful words. EXAMPLE 7. Find the odd man out.
EXAMPLE 3. Which one of the following words is not a part (a) Patna (b) Mumbai
of the group formed by remaining words. The remaining words (c) Kolkata (d) Bangluru
form the group on the basis of certain similarity.
(e) Madhya Pradesh
(a) Name (b) Game
Sol.: Option (e) is the correct answer because Madhya Pradesh
(c) Fame (d) Shame is an Indian state while all other options are capitals of
(e) Lame Indian states. Patna is the capital of Bihar; Mumbai is the
ANALOGY & CLASSIFICATION A-21
capital of Maharashtra; Kolkata is the capital of West Bengal (a) Tendulkar (b) Dravid
and Bangluru is the capital of Karnataka. In case of Madhya (c) Dhoni (d) Yuvraj Singh
Pradesh (it is an Indian state), we can say that it has its
(e) Sania Mirza
capital in Bhopal.
Sol.: Option (e) is the correct option because Sania Mirza is an
EXAMPLE 8. Which of the following animals does not fit Indian tennis player. While the rest of the options are Indian
into the group formed by remaining four animals? cricketers.
(a) Cat (b) Dog Now, this chapter has come to an end. Readers are advised
(c) Tiger (d) Octopus to move as per the following steps while solving the
problems related to classification :-
(e) Lion
Step I : See all the given options with a serious eye.
Sol.: Option (d) is the correct option as this is the only animal
out of given options which is a water animal. Rest of the Step II : Try to make relation of similarity among the given
options are land animals. options.
Step III : Find out the one word not having the common
EXAMPLE 9. Four of the following given options have some
similarity like other four option and that one word will
similarity and so they form a group separating out one option. be your answer.
Find out that separate option.
A-22 ANALOGY & CLASSIFICATION

EXERCISE
1. Which of the following has the same relationship as that of 11. Which pair of the letters in the word BEAUTIFUL has the
Money : Wealth same relationship between its letters with respect to their
(a) Pity : Kindness (b) Cruel : Anger position in the English alphabet as the pair EA in that word
(c) Wise : Education (d) Pride : Humility has between its letters?
(e) None of these
(a) IB (b) LF
2. Which of the following is related to ‘Melody’ in the same
(c) IE (d) FL
way as ‘Delicious’ is related to ‘Taste’?
(a) Memory (b) Highness (e) TL
(c) Tongue (d) Speak 12. ‘Income’ is related to ‘Profit’ in the same way as ‘Expenditure’
(e) Voice is related to
3. In a certain way ‘Diploma’ is related to ‘Education’. Which (a) Sale (b) Receipts
of the following is related to ‘Trophy’ in a similar way? (c) Surplus (d) Loss
(a) Sports (b) Athlete (e) Balance
(c) Winning (d) Prize 13. ‘Electricity’ is related to ‘Wire’ in the same way as ‘Water’ is
(e) None of these related to
4. In a certain code CHEMISTRY is written as NFIDITUSZ. (a) Bottle (b) Jug
How is BEANSTOCK written in that code?
(c) River (d) Pipe
(a) CFBOSLDPU (b) CFBOSUPDL
(e) None of these
(c) OBFCSUPDL (d) OBFCSLDPU
(e) None of these 14. ‘Hospital’ is related to ‘Nurse’ in the same way as ‘Court’
5. Which of the following does not have the same relationship is related to
as that of Cloth : Garments? (a) Justice (b) Lawyer
(a) Leather : Footwear (b) Wood : Furniture (c) Judgement (d) Trial
(c) Earthen pots : Clay (d) Gold : Ornaments (e) None of these
(e) None of these 15. By following certain logic ‘THEIR’ is written as ‘TRIHE’
6. ‘Clock’ is related to ‘Time’ in the same way as ‘Vehicle’ is and ‘SOLDIER’ is written ‘SROLIED’. How is CUSTOM
related to which of the following? written in that logic?
(a) Driver (b) Road (a) UTSOMC (b) CTSUOM
(c) Passenger (d) Journey
(c) CUTSOM (d) YUSOMC
(e) Fuel
(e) None of these
7. “Illness” is related to “Cure” in the same way as “Grief’ is
Directions (Qs. 16-40) : In each of the following questions, there
related to
are two words / set of letters / numbers to the left of the sign
(a) Happiness (b) Ecstasy :: which are connected in some way. The same relationship
(c) Remedy (d) Solicitude obtains between the third words / set of letters / numbers and
(e) Consolation one of the four alternatives under it. Find the correct alternative
8. ‘Necklace’ is related to ‘Jewellery’ in the same way as ‘Shirt’ in each question.
is related to 16. Flying : Bird :: Creeping : ?
(a) Cloth (b) Cotton (a) Aeroplane (b) Snail
(c) Apparel (d) Thread
(c) Ground (d) Flower
(e) Ornament
9. ‘Bouquet’ is related to ‘Flowers’ in the same way as (e) None of these
‘sentence’ is related to 17. Clock : Time :: Thermometer : ?
(a) Letters (b) Paragraph (a) Heat (b) Radiation
(c) Content (d) Words (c) Energy (d) Temperature
(e) Construction (e) None of these
10. Which of the following relates to FLOWER in the same way 18. Man : Walk :: Fish : ?
as RTERBN relates to SECTOR?
(a) Swim (b) Eat
(a) RWLGPF (b) EOFKUQ
(c) EOFMXS (d) RWLEPD (c) Live (d) Sleep
(e) RWLEND (e) None of these
ANALOGY & CLASSIFICATION A-23
19. Import : Export :: Expenditure : ? 31. NUMBER : UNBMER : : GHOST : ?
(a) Deficit (b) Income (a) HOGST (b) HOGTS
(c) Debt (d) Tax (c) HGOST (d) HGSOT
(e) None of these (e) None of these
20. Ocean : Water :: Glacier : ? 32. 11 : 17 : : 19 : ?
(a) Refrigerator (b) Ice (a) 29 (b) 27
(c) Mountain (d) Cave (c) 23 (d) 21
(e) None of these (e) None of these
21. PRLN : XZTV :: JLFH : ? 33. Court : Justice : : School : ?
(a) NPRT (b) NRPT (a) Teacher (b) Student
(c) NTRP (d) RTNP (c) Ignorance (d) Education
(e) None of these (e) None of these
22. DRIVEN : EIDRVN :: BEGUM : ? 34. Breeze : Cyclone : : Drizzle : ?
(a) EUBGM (b) MGBEU (a) Earthquake (b) Storm
(c) BGMEU (c) UEBGM (c) Flood (d) Downpour
(e) None of these (e) None of these
23. 14 : 9 :: 26 : ? 35. Oxygen : Burn : : Carbon dioxide : ?
(a) 12 (b) 13 (a) Isolate (b) Foam
(c) 15 (d) 31 (c) Extinguish (d) Explode
(e) None of these (e) None of these
24. ACFJ : OUZJ :: SUXB : ? 36. Teheran : Iran : : Beijing : ?
(a) GNSA (b) GLQZ (a) China (b) Japan
(c) GKPY (d) GMRB (c) Turkey (d) Malaysia
(e) None of these (e) None of these
25. 6: 24 :: 5 : ? 37. 3 : 27 : : 4 : ?
(a) 23 (b) 22 (a) 140 (b) 75
(c) 26 (d) 20 (c) 100 (d) 64
(e) None of these (e) None of these
26. Medicine : Sickness :: Book : ? 38. Disease : Pathology :: Planet : ?
(a) Ignorance (b) Knowledge (a) Astrology (b) Geology
(c) Author (d) Teacher (c) Astronomy (d) Palaeontology
(e) None of these (e) None of these
27. Bank : River :: Coast : ? 39. Foresight : Anticipation :: Insomnia : ?
(a) Flood (b) Waves (a) Treatment (b) Disease
(c) Sea (d) Beach (c) Sleeplessness (d) Unrest
(e) None of these (e) None of these
28. Supervisor : Worker :: 40. CG : EI : : FJ : ....
(a) Junior : Senior (b) Elder : Younger (a) LM (b) IJ
(c) Debtor : Creditor (d) Officer : Clerk (c) GK (d) HL
(e) None of these (e) None of these
29. Thunder : Rain :: Night : ... 41. Four of the following five are alike in a certain way and so
(a) Day (b) Dusk form a group. Which is the one that does not belong to that
(c) Darkness (d) Evening group?
(a) 29 (b) 85
(e) None of these (c) 147 (d) 125
30. ACE : HIL :: MOQ : ? (e) 53
(a) XVT (b) TVX 42. Four of the following five are alike in a certain way and so
(c) VTX (d) TUX form a group. Which is the one that does not belong to that
group?
(e) None of these (a) Crow (b) Vulture
A-24 ANALOGY & CLASSIFICATION
(c) Bat (d) Ostrich (e) 157
(e) Eagle 50. Four of the following five are alike in a certain way and so
43. Four of the following five are alike in a certain way and so form a group. Which is the one that does not belong to the
form a group. Which is the one that does not belong to that group’?
group? (a) Listen (b) Feel
(a) Food : Hunger (b) Water : Thirst (c) Think (d) Sing
(c) Air : Suffocation (d) Talent : Education (e) Hear
(e) Leg : Lame 51. Four of the following five are alike in a certain way and so
44. Four of the following five are alike in a certain way and so form a group. Which is the one that does not belong to that
form a group. Which is the one that does not belong to that group?
group? (a) Jowar (b) Wheat
(a) Teacher (b) Engineer (c) Paddy (d) Bajra
(c) Architect (d) Doctor (e) Mustard
(e) Scientist 52. Four of the following five are alike in a certain way and so
45. Four of the following five are alike in a certain way and form a group. Which is the one that does not belong to that
hence form a group. Which one of the following does not group?
belong to that group? (a) Volume (b) Size
(a) 126 (b) 122 (c) Large (d) Shape
(c) 65 (d) 50 (e) Weight
(e) 170 53. Four of the following five are alike in a certain way and so
46. Four of the following five are alike in a certain way and form a group. Which is the one that does not belong to that
hence form a group. Which one of the following is different group?
from the group? (a) 72 (b) 42
(a) 226 (b) 290 (c) 152 (d) 110
(c) 360 (d) 170 (e) 156
(e) 122 54. Four of the following five are alike in a certain way and so
47. Four of the following five are alike in a certain way and form a group. Which is the one that does not belong to that
hence form a group. Find the one which is different from the group?
other four. (a) Guava (b) Orange
(a) Rice (b) Wheat (c) Apple (d) Lichi
(c) Barley (d) Mustard (e) Pear
(e) Bajra 55. Four of the following five are alike in a certain way and so
48. Four of the following five are alike in a certain way and from a group. Which is the one that does not belong to that
hence from a group. Find the one which is different from the group?
other four. (a) Aluminium (b) Copper
(a) Arrow (b) Sword (c) Mercury (d) Iron
(c) Knife (d) Axe (e) Zinc
(e) Pistol
49. Four of the following five are alike in a certain way and so
form a group. Which is the one that does not belong to the
group?
(a) 169 (b) 179
(c) 135 (d) 149

ANSWER KEY
1 (a) 8 (c) 15 (a) 22 (b) 29 (c) 36 (a) 43 (d) 50 (d)
2 (e) 9 (d) 16 (b) 23 (c) 30 (d) 37 (d) 44 (a) 51 (e)
3 (a) 10 (e) 17 (d) 24 (d) 31 (d) 38 (c) 45 (a) 52 (c)
4 (c) 11 (c) 18 (a) 25 (d) 32 (a) 39 (c) 46 (c) 53 (c)
5 (c) 12 (d) 19 (b) 26 (a) 33 (d) 40 (d) 47 (d) 54 (d)
6 (d) 13 (d) 20 (b) 27 (c) 34 (d) 41 (c) 48 (a) 55 (c)
7 (c) 14 (b) 21 (d) 28 (d) 35 (c) 42 (c) 49 (a)
ANALOGY & CLASSIFICATION A-25

Answers &
Explanations
1. (a) They are synonymous. +8 +8
L ¾¾® T F ¾¾® N
2. (e) ‘Delicious’ is the adjective used for ‘Taste’. Similarly,
‘Melodious’ is the adjective used for ‘Voice’. +8 +8
N ¾¾® V H ¾¾® P
3. (a) A successful finish of ‘Education’ equips one with
22. (b) Fifth and third letters of the first term are first and
’Diploma’. Similary, a successful finish in ‘Sports’
second letters of the second term and first two letters
equips one with ‘Trophy’.
of the first term are third and fourth letters of the
4. (c) Reverse the first four letters of the given word. Now, second term.
all letters are coded as one place forward as in English
alphabet except the middle letter, which remains 23. (c) The relationship is (2x – 4) : x.
unchanged. 24. (d) As, Similarly,
5. (c) In each pair 2nd thing is made up of 1st thing while in +14 +14
option (c), the 1st thing is made up of 2nd thing. A ¾¾¾
®O S ¾¾¾
®G
6. (d) The clock makes a journey of time. +18 +18
C ¾¾¾
®U U ¾¾¾
®M
7. (c) Cure ensures removal of illness in the same way as
remedy insures removal of grief. +20 +20
F ¾¾¾
®Z X ¾¾¾
®R
8. (c) Jewellery consists of Necklace ie ‘Necklace’ is a kind
+0 +0
of ‘Jewellery’. Similarly, ‘Shirt’ is a kind of ‘Apparel’. J ¾¾® J B ¾¾® B
9. (d) Bouquet is a bunch of flowers. Similarly, Sentence is a 25. (d) Second term = 4 × First term
set of words that is complete in itself. \ Fourth term = 4 × Third term
10. (e) From SECTOR TO RTERBN; The second letter 26. (a) As medicine cures sickness, in the same way, books
becomes third, fourth becomes second, and last remove ignorance.
becomes first. Also, after subtracting one letter from
27. (c) Bank is the land beside a river.
the first, we get fourth, from third, we get fifth and from
fifth we get last. Similarly, coast is the land beside a sea.
28. (d) As supervisor supervises the worker, in the same
–4
BEAUTIF UL way, officer supervises the clerk.
11. (c)
–4 29. (c) As ‘Rain’ is followed by ‘Thunder’, similarly
‘Darkness’ is followed by ‘Night’.
12. (d) When Income is more than expenditure, it bears Profit. 30. (d) As, Similarly,
But when Expenditure is more than income, then loss
+7 +7
occurs. A ¾¾® H M ¾¾® T
13. (d) Wire is the medium to transmit Electricity. Similarly,
+6 +6
Pipe is the medium to carry Water. C ¾¾® T M ¾¾® U
14. (b) Here, the first is the working place of the second. +7 +7
15. (a) Words are arranged in alphabetical order but from right E ¾¾® L Q ¾¾® X
to left. If becomes UTSOMC. 31. (d) First two letters of the first term are in reverse order
16. (b) As ‘Bird’ flies, in the same way, ‘snails’ creeps. in the second term and so are the next two letters.
17. (d) First is an instrument to measure the second. 32. (a) 11 : 17 alternate prime number (skipping 13) 19 : 29
18. (a) As a man covers some distance after walking, in the alternate prime number (skipping 23)
same way, a fish covers some distance after 33. (d) First is the place where the second is imparted.
swimming. Hence the correct answer is (a). 34. (d) Second is more intense than the first.
19. (b) The words in each pair are antonyms. 35. (c) ‘Oxygen’ helps in burning while ‘carbon dioxide’
20. (b) First consists of the second. extinguishes fire.
21. (d) As Similarly, 36. (a) ‘Teheran’ is the capital of ‘Iran’ and ‘Beijing’ is the
+8 +8 capital of ‘China’.
P ¾¾® X J ¾¾® R
37. (d) Second term = (First term)3
+8 +8
R ¾¾® Z L ¾¾® T \ Fourth term = (Third term)3
A-26 ANALOGY & CLASSIFICATION
38. (c) Diseases are studied under Pathology. 47. (d) Except ‘mustard’ each belongs to the same category,
Similarly, planets are studied in Astronomy. viz food grains. Mustard is an oilseed.
39. (c) The words in each pair are synonyms. 48. (a) All others are held in the hand and not shot out.
40. (d) As, Similarly 49. (a) The rest are not squares of a number.
+2 50. (d) All others are the features of sense organes.
C ¾¾® E F®H
51. (e) Mustard is an oilseed while the rest are foodgrains.
+2 52. (c) ‘Large’ is an adjective whereas others are noun.
G ¾¾® I J®L
53. (c) 72 = 92 – 9
41. (c) All other numbers are in the form of n 2 + 4 where n is 42 = 72 – 7
a natural number. 152 = 122 + 8
42. (c) Except it others are birds whereas bat is a mammal 110 = 112 – 1l
43. (d) Lack of first one causes second one. 156 = 132 – 13
44. (a) All the rest are exclusive professions while a teacher Except 152, others show the trend x2 – x.
may, be there in any of these categories. 54. (d) Lichi has only one seed inside whereas others have
45. (a) The rest are based on the expression x2+ 1. many seeds.
But 126 = 112 + 5. 55. (c) All others are found in solid state while mercury is
46. (c) After a close look you will get that except 360 each found in liquid state.
number is one more than square of a natural number,
i.e., 226 = 152 + 1 ; 290 = 172 + 1 ; 170 = 132 + 1 ;
122 = 112 + 1
CODING-DECODING A-27

Coding-
Decoding
3 Chapter
In this segment of commonsense reasoning, secret messages or Pattern 3: Coding based on skipped sequence.
words have to be decoded. They are coded as per a definite Example: If the word ‘FACT’ is coded as ‘IDFW’; then how will
pattern/ rule which should be identified 1st. Then the same is you code ‘DEEP’?
applied to decode another coded word. Under this segment you
come across two types of coding letter coding and number coding. Explanation: Here, you see that 2 letters are omitted in alphabetic
Based on these two types of coding-decoding various types of sequence. The following diagram gives you the more clear picture :
problems come your way. This chapter makes you familiar with
F A C T
every types of problems based on coding-decoding.
+3 +3 +3 +3
TYPE I (CODING BY LETTER SHIFTING)
I D F W
Pattern 1: Coding in forward sequence
Example: If ‘GOOD’ is coded as ‘HPPE’, then how will you code Clearly, ‘F’ (skip 2 letters) ‘I’
‘BOLD’? ‘A’ (skip 2 letters) ‘D’
Explanation: Here,every letter of the word ‘Good’ shifts one place ‘C’ (skip 2 letters) ‘F’
in forward alphabetical sequence. Let us see: ‘T’ (skip 2 letters) ‘W’
G O O D Similarly, ‘DEEP’ can be coded. Let us see :
+1 +1 +1 +1
D E E P
H P P E +3 +3 +3 +3
Similarly, every letter in the word ‘BOLD’ will move one place in
G H H S
forward alphabetical sequence as given below:
\ Code for ‘DEEP’ will be ‘GHHS’.
B O L D
+1 +1 +1 +1 TYPE II (CODING BY ANALOGY)
Example: If ‘RPTFA’ stands for ‘BLADE’, how will you code
C P M E
‘BALE’.
\ Code for ‘BOLD’ will be ‘CPME’.
Explanation: Here, ‘BLADE’ has been coded as ‘RPTFA’. You
Pattern 2: Coding in backward sequence. will see that all the letters in the word ‘BALE’, which have to be
Example: If ‘NAME’ is coded as ‘MZLD’, then how will code coded, are also there in the word ‘BLADE’. Hence, all that needs
SAME? to be done is to choose the relevant code letters from the code
Explanation: Here, every letter of the word ‘MZLD’ moves one word ‘RPTFA’. Therefore, B becomes R, A becomes T, L becomes
place in backward alphabet sequence. Let us see: P, and E becomes A. Therefore, ‘BALE’ will be coded as ‘RTPA’.
N A M E \ Correct answer is ‘RPTA’.
–1 –1 –1 –1 TYPE III (CODING BY REVERSING LETTERS)
M Z L D Example: If ‘TEMPERATURE’ is coded as ‘ERUTAREPMET’,
Similarly, every letter of the word ‘SAME’ will move one place in then how will you code ‘EDUCATION’ following the same scheme.
backward alphabet sequence. Let us see : Explanation: Here, the word ‘TEMPERATURE’ has been reversed.
Hence, the code for ‘education’ will be ‘NOITACUDE’.
S A M E
–1 –1 –1 –1 TYPE IV (CODING IN FICTIONS LANGUAGE)
In some cases of coding-decoding, fictions language is used to
R Z L D
code some words. In such questions, the codes for a group of
\ Code for ‘SAME’ will be ‘RZLD’. words is given. In such types of problems, codes for each word
can be found by eliminating the common words.
A-28 CODING-DECODING
Example: In a certain code language ‘over and above’ is written The fact that the code for ‘TALE’ is 38, gives you a clue that the
as ‘da pa ta’ and ‘old and beautiful’ is written as ‘Sa na pa’. How code is probably obtained by performing an arithmatical
is ‘over’ written in that code language? operations of the numbers of each other. Let us see :
20 + 1 + 12 + 5 = 38
Explanation: Over and above = da Pa ta Thus, the code for ‘CAME’ is
C A M E
Old and beautiful = Sa na Pa
3 + 1 + 13 + 5 = 22
Clearly, ‘and’ is common in both and a common code is ‘Pa’. \ Code for ‘CAME’ = 22
\ Code for ‘and’ must be ‘Pa’.
TYPE VII : CODING LETTER OF WORD
Code for ‘over’ = ‘da’ or ‘ta’.
Directions: These questions are based on code language which
Code for above = ‘da’ or ‘ta’.
utilizes letters in the English alphabet. In each question, there
Code for old = ‘Sa’ or ‘n’ is a word written in capital letter, with one letter underlined.
Code for beautiful = ‘Sa’ or ‘na’ For each letter in that word there is a code written in small
\ We can’t certainly say what will be exact code for ‘over’. But letters. That code is denoted by 1, 2, 3, 4, and 5 not in the same
it is sure that code for ‘over’ must be either ‘da’ or ‘ta’. order. You have to find out the exact code for the underlined
letter in the word. The number of that code is the answer. Please
TYPE V (CODING BASED ON NUMBERS) note that the same letter appearing in other word(s) may be
Pattern 1: When numerical values are given to words. coded differently.
Example: If in a certain language A is coded as 1, B is coded as 2. 1. QUIT E
C is coded as 3 and so on, then find the code for AEECD. (a) hj (b) su
Explanation: As given the letters are coded as below: (c) tv (d) pr
A B C D E F G H I (e) df
1 2 3 4 5 6 7 8 9 2. PRISM
(a) R (b) O
A E E C D (c) H (d) Q
Now, 1 5 5 3 4 (e) I
3. BEAST
\ Code for AEECD = 15534
(a) c (b) w
Pattern 2: When alphabetical code value are given for numbers.
(c) d (d) h
Example: In a certain code 3 is coded as ‘R’, 4 is coded as ‘D’, 5
(e) v
is coded as ‘N’, 6 is coded as ‘P’, then find the code for ‘53446’.
Solution:
Explanation: As per the given condition
1. (d) is the correct answer.
3 4 5 6 Explanation Each single letter is expressed as two letters,
R D N P one behind and the other ahead of the given letter. Therefore,
A becomes zb, B comes ac and so on.
5 3 4 4 6 2. (d) is the correct answer.
Now, N R D D P Explanation All the letters of the word are coded as one
letter behind.
\ Code for 53446 = NRDDP. 3. (b) is the correct answer.
TYPE VI (MATHEMATICAL OPERATIONS WITH THE Explanation All the letters of the word are coded as three
POSITION NUMBERS OF LETTERS) letters ahead.
Example: In a certain code, if ‘TALE’ is written as 38, then how Now, you must have been aware of the various kind of coding-
will you code ‘CAME’ using the same coding scheme? decoding patterns. Point to be noted that the patterns discussed
Explanation : Look at the numbered alphabet and write down the under this chapter are commonly known pattern/ basic patterns.
number corresponding to the letters of the word ‘TALE’. So, if you practice hard, you find that after some times you become
T A L E competent enough to solve coding-decoding problems even if
certain changes are made in such problems to surprise or puzzle
20 1 12 5
you.
CODING-DECODING A-29

EXERCISE
1. In a certain code language BEAM is written as 5 % * K and Word Coded Form
COME is written as $ 7 K %. How is BOMB written in that SEAT : [5] [15] [15] [5]
code? CUT : [5] [10] [5]
(a) 5 % K5 (b) 5 7 K5 ONE : [0] [5] [0]
(c) $ 7 K $ (d) 5$ % 5 DEEP : [5] [20] [20] [5]
(e) None these POUR : [5] [15] [15] [5]
2. In a certain code PATHOLOGIST is written as PIN : [5] [10] [5]
PIUBQKSRHFN. How is CONTROVERSY written in that NONE : [5] [25] [5] [25]
code? BOOK : [5] [20] [20] [5]
(a) SUOPDNXRQDU (b) SUOPDNZTSFW OPEN : [30] [5] [30] [5]
(c) QSMNBPXRQDU (d) QSMNBPZTSFW ATE : [0] [5] [0]
(e) None of these PAGE : [5] [25] [5] [25]
Directions (Qs.3-7): In a certain code language meanings of some UNIT : [30] [5] [30] [5]
words are as follows: Directions (Qs. 8-12): Find out the coded form of each of
(i) ‘pit na sa’ mean ‘you are welcome’. the words printed in bold.
(ii) ‘na ho pa la’ means ‘they are very good’. 8. DOSE
(iii) ‘ka da la’ means ‘who is good’? (a) [5] [15] [5] [15] (b) [5] [10] [5] [30]
(iv) ‘od ho pit la’ means ‘they welcome good people’. (c) [5] [30] [5] [30] (d) [5] [0] [5] [15]
3. Which of the following means ‘people’ in that code (e) None of these
language? 9. SIP
(a) ho (b) pit (a) [5] [0] [5] (b) [0] [5] [0]
(c) la (d) od (c) [5] [5] [5] (d) [5] [10] [5]
(e) Data inadequate (e) None of these
4. Which of the following means ‘very’ in that code language? 10. AGED
(a) na (b) da (a) [0] [5] [0] [5] (b) [30] [10] [30] [10]
(c) pa (d) Data inadequate (c) [30] [5] [30] [5] (d) [25] [5] [25] [5]
(e) None of these (e) None of these
5. Which of the following statements is/are redundant to 11. DATA
(a) [5] [30] [5] [30] (b) [5] [25] [5] [25]
answer the above two questions?
(c) [5] [15] [5] [15] (d) [5] [10] [5] [10]
(a) None (b) (i) and (iii)
(e) None of these
(c) (ii) or (iv) (d) (i) or (iv)
12. EVE
(e) None of these (a) [0] [5] [0] (b) [0] [15] [0]
6. In a certain code language NATIONALISM is written as (c) [15] [15] [15] (d) [0] [10] [10]
OINTANMSAIL. How is DEPARTMENTS written in that (e) None of these
code? Directions (Qs. 13-17): Which of the words denoted by (A), (B),
(a) RADEPTSTMNE (b) RADPETSTMNE & (C) can be the correct words (s) for the codes given against
(c) RADPESTMTNE (d) RADPETSTNME each questions number?
(e) None of these 13. [5] [25] [5] [25]
7. In a certain code language OUTCOME is written as (A) TRAP
OQWWEQOE. How is REFRACT written in that code? (B) DRAW
(a) RTGITCET (b) RTGTICET (C) BOAT
(c) RTGITECT (c) RTGICTET (a) A and B only (b) B and C only
(e) None of these (c) A and C only (d) All the three
Directons (Qs. 8-17): In a certain code, letters of English alphabet (e) None of these
(consonants and vowels) are coded as given for some a words. 14. [5] [20] [20] [5]
(A) DOLL
The numeric code for each letter is given in bracket under coded
(B) MOOD
form and corresponds to the letter in the word in the same serial
(C) BEEP
order. Study the coded forms of the given words and find out the (a) A and B only (b) B and C only
rules for their codification. Applying those rules, answer the (c) A and C only (d) All the three
questions that follow in the two sets. (e) None of these
A-30 CODING-DECODING
15. [5] [10] [5] 24. In a certain code language STREAMLING is written as
(A) MAN CGTVUHOJMN. How will the word PERIODICAL be written
(B) TOP in that language?
(C) CAT (a) PJSFQMNBJE (b) QKTGRMBDJE
(a) A and B only (b) B and C only (c) QKTGRMCEKF (d) PJSFQMBDJE
(c) A and C only (d) All the three (e) None of these
(e) None of these 25. If ‘green’ is called `white’, `white’ is called `yellow , ‘yellow’
16. [0] [5] [0] is called `red’, `red’ is called `orange’, then which of the
(A) ARE following represents the colour of sunflower?
(B) AND (a) red (b) yellow
(C) ORE (c) brown (d) indigo
(a) None (b) All the three (e) None of these
(c) A and B only (d) B and C only 26. In a certain code language GEOPHYSICS is written as
(e) A and C only IOPDHZRJBT. How is ALTIMETE` written in that code’?
17. [30] [5] [30] [5] (a) NHULBFSDQT (b) NIUKBFSDQT
(A) ARID (c) NHUKCFSDQT (d) NHUKBFSEQT
(B) EVIL (e) None of these
(C) OURS 27. In a certain code BROUGHT is written as SGFVAQN. How
(a) A and B only (b) B and C only is SUPREME written in that code?
(c) A and C only (d) All the three (a) FNFSRTO (b) RTOSDLD
(e) None of these (c) DLDSRTO (d) DLDSTVQ
18. A trader in order to code the prices of article used the letters (e) None of these
of PSICHOLAZY in the form of ‘0 to 9’ respectively. Which 28. If W means White, Y means Yellow, B means Black, G means
of the following code stands for ` 875.50? Green, R means Red, which of the following will come next
(a) AIL.HP (b) AIL.HS in the sequence given below?
(c) ZYA.HO (d) ZCA.OP WW YWYBWYBGWYBGRWWYWYBWYB
(e) None of these (a) Red (b) White
19. If B is coded as 8, F is coded as 6, Q is coded as 4, D is (c) Green (d) Yellow
coded as 7, T is coded as 2, M is coded as 3, and K is coded (e) None of these
as 5, then what is the coded form of QKTBFM? 29. In a certain code ‘CLOUD’ is written as ‘GTRKF’. How is
(a) 452683 (b) 472683 SIGHT written in that code?
(c) 452783 (d) 425783 (a) WGJHV (b) UGHHT
(e) None of these (c) UHJFW (d) WFJGV
20. If in a certain code language ‘pen pencil’ is written as ‘$ £’, (e) None of these
‘eraser sharpener’ is written as @ #’, and ‘pencil eraser’ is 30. In a certain code CHAIR is written as # * • ÷ % and HIDE is
written as ‘$ @’, then what is the code for ‘pen’? written as * ÷ + $. How is DEAR written in that code?
(a) £ (b) @ (a) $ + • % 2 (b) + $ ÷ %
(c) $ (d) # (c) $ + % ÷ 4 (d) + # • % 5
(e) None of these (e) None of these
21. In a certain code language ‘Infinite’ means ‘Size’. What is 31. In a certain code AROMATIC is written as BQPLBSJB. How
the meaning of ‘Indefinite’ in that code language? is BRAIN written in that code?
(a) Time (b) Day (a) CQBJO (b) CSBJO
(c) Meaning (d) Shape (c) CQBHO (d) CSBHO
(e) None of these (e) None of these
22. In a certain code language GAME is written as ‘$ ÷ * %’ and 32. If ‘yellow’ means ‘green’, ‘green’ means ‘white’, white means
BEAD is written as ‘# % ÷ ×’. How will the word MADE be ‘red’, ‘red’ means ‘black’, ‘black’ means ‘blue’ and ‘blue’
written in that code language? means ‘violet’, which of the following represents the colour
(a) $ ÷ × % (b) * ÷ $ % of human blood?
(c) * ÷ × % (d) # ÷ × % (a) black (b) violet
(e) None of these (c) red (d) blue
23. In a certain code language BORN is written as APQON and (e) None of these
LACK is written as KBBLK. How will the word GRID be 33. In a certain code ‘FEAR’ is written as ‘ + × ÷ * ’ and ‘READ’
written in that code language? is written as ‘*× ÷ $ ’. How is ‘FADE’ written in that code?
(a) FQHCD (b) FSHED (a) + ÷ $ × (b) × ÷ + $
(c) HSJED (d) FSHCD (c) $ ÷ + * (d) ÷ $ + ×
(e) None of these (e) None of these
CODING-DECODING A-31
34. In a certain code BREAK is written as ASDBJ. How is SOLAR (a) Red (b) Blue
written in that code? (c) Cloud (d) Sky
(a) RPKBS (b) TPMBS (e) None of these
(c) RPKBQ (d) TPKBQ 43. In a certain code language ‘POETRY’ is written as
(e) None of these ‘QONDSQX’ and ‘OVER’ is written as ‘PNUDQ’. How is
35. In a certain code language EMPHASIS is written as ‘MORE’ written in that code?
NDIOBRJR. How will CREATURE be written in that code (a) NNNQD (b) NLPQD
language? (c) NLNQD (d) LNNQD
(a) SBBDUTSD (b) QBBDTUSD (e) None of these
(c) DSDBSTSF (d) SBDBUTDS 44. If water is called air, air is called green, green is called brown,
(e) None of these brown is called steel, steel is called red, red is called rain,
36. In a code language “1357” means “We are very happy”, rain is called tree and tree is called road, what is the colour
“2639” means “They are extremely lucky”, and “794” means of human blood?
“Happy and lucky”. Which digit in that code language (a) Red (b) Water
stands for “very”? (c) Road (d) Tree
(a) 1 (b) 5 (e) Rain
(c) 7 (d) Data inadequate 45. In a certain code language ‘MOTHERS’ is written as
(e) None of these ‘OMVGGPU’. How is ‘BROUGHT’ written in that code?
37. In a certain code language ‘CREATIVE’ is written as (a) CPRTIEV (b) DPQSIFV
‘BDSBFUJS’. How is ‘TRIANGLE’ written in that code? (c) DPRTIDV (d) DPQTIFV
(a) BSHSFHKM (b) BHSSMHHF (e) None of these
(c) BSSHFMKH (d) BHSSFKHM 46. In a certain code ‘PENCIL’ is written as ‘RCTAMJ’ then in
(e) None of these that code ‘BROKEN’ is written as
38. In a certain code ‘BROTHER’ is written as‘$%53#4%’ and (a) SPFLIM (b) SVFLIN
‘DREAM’ is written as ‘9%47 ’. How is ‘THREAD’ written (c) FVSMGL (d) FPSMIL
in that code? (e) None of these
(a) #3%479 (b) 3#%479 47. In a certain code language the word FUTILE is written as
(c) 3$%479 (d) 3#% 79 HYVMNI. How will the word PENCIL be written in that
(e) None of these language?
39. In a certain code language ‘allow children to play on the (a) OIFRLT (b) OIFRLS
ground’ is written as ‘play allow on children the to ground’ (b) OLFRIT (c) OIRFLT
then how will ‘the do open not electric touch wires’ be (e) None of these
decoded from that language? 48. In a certain code language the word ‘NUMBER’ is written
(a) not the electric do touch open wries as ‘UMHTEL’. How will the word ‘SECOND’ be written in
(b) do not touch the electric open wires that language?
(c) do not touch the open electric wires (a) CTQDRB (b) GRQDRB
(d) not the do electric touch open wires (c) CTQFRB (d) GRQFRB
(e) None of these (e) None of these
40. In a certain code OVER is written as ‘PWFSQ’ and BARE 49. If the sentence “you must go early to catch the train” is
is written as ‘CBSFD’. How is OPEN written in that code? coded as “early catch train must to go the you”, what will
(a) PQFOM (b) NODMO be code for the sentence “morning exercise will help you to
(c) PQFOO (d) POFMM keep fit”?
(e) None of these (a) help to fit you exercise will keep morning
41. In a certain code language ‘OMNIPRESENT’ is written as (b) help to fit exercise you will keep morning
‘QJONPTSMDRD’. How is ‘CREDIBILITY’ written in that (c) will help to fit you exercise keep morning
code? (d) will fit to exercise you help keep morning
(a) JEFSDCXSHKH (b) JEFSDDXSHKH (e) None of these
(c) DSFEJDDXSHKH (d) JEFSDXDSHKH 50. In a certain code ‘SENSITIVE’ is written as ‘QHLVGWGYC’.
(e) None of these How is ‘MICROSOFT’ written in that code?
42. If ‘white’ is called ‘rain’, ‘rain’ is called ‘green’, ‘green’ is (a) KGAPMQMDT (b) QKETQUQHV
called blue’, ‘blue’, is called ‘cloud’, ‘cloud’ is called ‘red’, (c) KLAUMVMIR (d) LKBTNUNHS
‘red’ is called ‘sky’, ‘sky’ is called ‘yellow’ and ‘yellow’ is (e) None of these
called’ ‘black’, what is the colour of ‘blood’?
A-32 CODING-DECODING

ANSWER KEY
1 (b) 7 (a) 13 (e) 19 (e) 25 (a) 31 (c) 37 (d) 43 (c) 49 (b)
2 (a) 8 (e) 14 (b) 20 (a) 26 (e) 32 (e) 38 (b) 44 (e) 50 (c)
3 (d) 9 (d) 15 (d) 21 (a) 27 (c) 33 (a) 39 (c) 45 (d)
4 (c) 10 (c) 16 (e) 22 (c) 28 (c) 34 (c) 40 (a) 46 (e)
5 (e) 11 (b) 17 (a) 23 (b) 29 (a) 35 (a) 41 (b) 47 (e)
6 (b) 12 (a) 18 (e) 24 (b) 30 (e) 36 (d) 42 (d) 48 (b)

Answers &
Explanations
1. (b) Here, B Þ 5, E Þ %, A Þ *, 8-17: Coding of letters is based on the structure of the word.
M Þ K, C Þ $,O Þ 7 Logic of coding is very simple. Observe the word and
Therefore, BOMB Þ 57K5 the codes. It gives us following information:
2. (a) If the word consists of four letters,
(i) and consonants occupy places at extreme ends only,
P A T H O L O G I S T then those consonants are replaced by [5] while vowels
are replaced by either [20] (if both the vowels are same)
-1
or [15] (if both the vowels are different).
K (ii) and the vowels occupy the second and the fourth
P I U B Q S R H F N
places only then those vowels are replaced by [25],
Five letters of the word PATHOLOGIST are reversed
while the consonants are replaced by [5].
first and then coded as one place forward. Similarly,
(iii) and the vowels occupy the first and third places only
the last five letters of the word are reversed then coded
then those vowels are replaced by [30], while the
as one place backward. Middle letter is coded as one
consonants are replaced by [5].
place backward.
If the word consists of three letters,
Hence, CONTROVERSY will be written as
SUOPDNXRQDU. (i) and the vowels occupy places at extreme ends only
3-7 pit na sa Þ you are welcome… (i) then those vowels are replaced by [0], while the
consonant is replaced by [5].
na ho pa la Þ they are very good… (ii)
(ii) and the consonants occupy places at extreme ends
ka da la Þ who is good… (iii)
only then those consonants are replaced by [5], while
od ho pit la Þ they welcome good people… (iv) the vowel is replaced by [10].
Code for Thus, in each and every case consonants are coded as [5]
(a) ‘good’ is la [ from (ii) and (iv)]. but numeric value assigned to vowels varies according to
(b) ‘they’ is ho [ from (ii), (iv) and (a)]. the position of vowels in a particular word.
(c) ‘welcome’ is pit [ from (i) and (iv)]. 8. (e) D O S E
(d) ‘people’ is od [ by elimination in (iv)]. Consonant – Vowel – Consonant – Vowel
(e) ‘are’ is na [ from (i) and (ii)]. Code for DOSE will be same as the codes for PAGE
(f). ‘very’ is pa [ by elimination in (ii)]. and NONE, i.e., [5] [25] [5] [25].
5. (e) Only iii is redundant 9. (d) S I P
6. (b) Divide the word into two groups of five letters each. Consonant – Vowel – Consonant
The first five letters are in group I and the last five Code for SIP will be same as the codes for PIN and
letters are in group II. Now, for its coding the middle CUT, i.e., [5] [10] [5].
letters remain unchanged. While the letters in each 10. (c) A G E D
group change their position as 1 ® 3, 2 ® 5, 3 ® 4, Vowel – Consonant – Vowel – Consonant
4 ® 2 and 5 ® 1. Code for AGED will be same as the codes for UNIT
7. (a) The first letter is coded as two letters: the first remains and OPEN, i.e., [30] [5] [30] [5].
unchanged and the second two letters forward as in 11. (b) D A T A
English alphabet. The second, fourth, fifth and sixth Consonant – Vowel – Consonant – Vowel
letters are coded as two letters forward while the third Code for DATA will be same as the codes for PAGE
letter is coded as three letters forward as in English and NONE, i.e., [5] [25] [5] [25].
alphabet. The last letter remains unchanged.
CODING-DECODING A-33
12. (a) E V E 26. (e) Divide the word into two halves. Now, reverse the order
Vowel – Consonant – Vowel of the letters of the first half and replace odd positioned
Code for EVE will be same as the codes for ONE and letters with one letter forward and even positioned
ATE, i.e., [0] [5] [0]. letter with one letter backward as in English alphabet.
13. (e) [5] [25] [5] [25] is the coded form of NONE and PAGE. For the second half letters, the odd-positioned letters
Among the given words TRAP, DRAW and BOAT, no are coded as one letter forward and even-positioned
word has similar stucture as compared with the words
letters are coded as one letter backward’as in English
NONE and PAGE.
alphabet.
14. (b) [5] [20] [20] [5] is the coded form of DEEP and BOOK.
27. (c) Here the given word is BROUGHT. Reversing the order
Among the given words DOLL, MOOD and BEEP, the
of the letters, it becomes THGUORB. Now, write each
last two words have a sturcture similar to the words
DEEP and BOOK. letter one place backward except the middle letter (write
15. (d) [5] [10] [5] is the coded form of PIN and CUT. Among middle letter one place forward). It becomes SGFVNQA.
the given words MAN, TOP and CAT, all the words Now, reverse the order of the last three letters and it
have the same sturucture as the words PIN and CUT. becomes SGFVAQN.
16. (e) [0] [5] [0] is the coded form of ONE and ATE. Among Similarly,
the given words ARE, AND and ORE, the two words SUPREME ® EMERPUS ® DLDSOTR ® DLDSRTO
ARE and ORE have the same structure as the words 28. (c) The series is W/WY/WYB/WYBG/WYBGR
ONE and ATE 29. (a) Here, each letter of the word CLOUD is written as three
17. (a) [30] [5] [30] [5] is the coded form of OPEN and UNIT. letters forward and one letter backward alternately.
Among the given words ARID and EVIL have the same Following this CLOUD becomes FKRTG. After that,
structure as OPEN and UNIT. reverse the order of the result obtained in the previous
18. (e) P S I C H O L A Z Y operation. Thus, FKRTG becomes GTRKF.
0123456789
Similarly, SIGHT will change its form as follows:
875.50 = ZAO.OP
SIGHT ® VHJGW ® WGJHV
19. (e) Q K T B F M = 4 5 2 8 6 3
30. (e)
20. (a) Pen pencil = $£ ...(i)
Eraser sharpner = @# ...(ii) Letter: # * • ¸ % + $
Pencil eraser = $@ ...(iii) Code: C H A I R D E
From (i) and (iii), the code for ‘pencil’ is $. Therefore, code for DEAR = + $ • %
Hence, from (i), the code for ‘pen’ is £. 31. (c) A R O M A T I C
22. (c) G($), A(÷), M(*), E(%) B(#), E(%), A(÷), D(×) MADE = +1 – 1 +1 – I +1 –1 +1 –1
*÷×% B Q P L B S J B
23. (b) B O R N Similarly, B R A I N
–1 +1 –1 +1 +1 – 1 +1 – 1 +1
A P Q O N C Q B H O
L A C K 32. (e) The colour of human blood is red. Here white means
–1 +1 –1 +1 red. Therefore white is our answer.
K B B L K Do not opt for black because red means black implies
Similarly, that black is called red.
G R I D 33. (a) It is clear that F ® +, A ® ¸ , D ® $ and E ® ×
–1 +1 –1 +1 \ FADE ® + ¸ $ ×
F S H E D 34. (c) The odd-positioned letters are coded as one position
24. (b) Split the word STREAMLING into two groups backward and the even-positioned letters are coded
consisting of equal letters. You get STREA and MLING. as one position forward as in English alphabet.
Now, reverse both the groups. You get AERTS and 35. (a)
GNILM. Now, write each letter of first group two places
E+1 M P H A S I S
forward. You get CGTVU. Write each letter of second +1 +1 –1 +1 –1

group one place forward. You get HOJMN. Now, join –1 –1

both the groups without changing the order of letters. N D I O B R J R


You get CGTVUHOJMN.
Similarly, PERIODICAL is coded as 37. (d) C R E A T I V E
PERIODICAL ® OIREPLACID ® QKTGRMBDJE When the letters in both the halves are reversed, we
25. (a) The colour of sunflower is yellow and yellow is called get
‘red’. Hence sunflower is red. A E R C E V I T
A-34 CODING-DECODING
+1 –1 +1 –1 +1 –1 +1–1 Similarly, MORE will be coded as follows:
B D S B F U J S M O R E
Next, the letters have been written as one place forward

+1
| –1| –1| –1| –1
and one place backward alternately. N L N Q D
Similarly, TRIANGLE is coded as follows:
T R I A N G L E 44. (e) The colour of human blood is red. But here red is called
A I RT E L G N rain.
+1 –1 +1 –1 +1 –1 +1 –1 45. (d) M O T H E R S
B H S S F K H M +2 –2 +2 –1 +2 –2 +2
Hence, code for TRIANGLE is BHSSFKHM O M VG G P U
38. (b) Here code for BROTHER is $%53#4% and code for Similarly, BROUGHT be coded as follows:
DREAM is 9%47*. Now, see the position of two Rs in B R O U G H T
the word BROTHER. Each ‘R’ has been replaced by +2 –2 +2 –1 +2 –2 +2
‘%’. This is also true for the word DREAM. Again, see D P Q T I F V
the position of E in both the words. Each ‘E’ has been 46. (e) The first three letters of the word are reversed. Thus
replaced by ‘4’. This implies that the elements used as PENCIL becomes NEPCIL. Now add 4 to odd-
the code for letters are in the same order as the letters positioned letters and subtract 2 from even-positioned
of the word. It represent codes of respective letters. ones. Similarly, BROKEN becomes ORBKEN. Then we
Hence, code for THREAD Þ 3#%479. do the calculations: O + 4, R – 2, B + 4, K – 2, E + 4,
39. (c) Play allow on children the ground : children. The N – 2, i.e. SPFIIL.
ground Similarly, the do open not electric touch wires 47. (e) Odd-placed letters are coded as two places forward
: do not touch the open electric wires. and even-placed letters are coded as four places
40. (a) OV ER O PE N forward as in English alphabet.
+ 1 + 1 + 1 + 1 – 1 Similarly, + 1 + 1 + 1 + 1– 1 48. (b) A real tough one! If we number the letters of the word
P W F S Q P Q F OM from 1 to 6, first rearrange the letters in the order 615243.
41. (b) Split the word OMNIPRESENT in three parts OMNIP. Next, to this reversed order of letters, apply the
R, and ESENT. Reverse the order of the letters of the following alternately: move three letters ahead; go one
first part. It becomes PINMO. Now write each letter of letter backward.
this part one place forward. i.e., PINMO becomes Thus NUMBER first becomes RNEUBM. Then
QJONP. Write the letter of the second part two places R + 3 = U, N – 1 = M,
forward. i.e. R becomes T. Again, reverse the order of E + 3 = H, U – 1 = T, B + 3 = E, M – 1 = L. So the final
the letters of the third part, i.e., ESENT becomes code is UMHTEL.
TNESE. Now write each letter one place backward, i.e. Similarly, SECOND® DSNEOC ® GRQDRB
TNESE becomes SMDRD. Hence, OMNIPRESENT is 49. (b) you must go early to catch the train
coded as QJONPTSMDRD. 1 2 3 4 5 6 7 8
Similarly, CREDIBILITY will be written as early catchtrain must to go the you
JEFSDDXSHKH. 4 6 8 2 5 3 7 1
42. (d) We know colour of blood is red. Here, red is called sky. Similarly,
Therefore, our correct answer is ‘sky’. morning exercise will help you to keep fit
1 2 3 4 5 6 7 8
P O E T R Y 4 6 8 2 5 3 7 1
+1

43. (c) | –1| –1| –1| –1| –1| –1 help to fit exercise you will keep morning
Q O N D S Q X 50. (c) The letters at odd-numbered positions (1st, 3rd, ...)
Similarly, move two letters backward. While those at even
numbered positions (2nd, 4th, ...) move three letters
O V E R
forward.
+1

| –1| –1| –1| –1


P N U D Q
DIRECTION & DISTANCE A-35

Direction &
Distance
4 Chapter
This part of reasoning comes under the category of common Concept of turn
sense reasoning. In fact this segment gauge the sense of direction Let turn = clockwise turn
of a candidate. In every objective competitive examinations this Right turn = Anticlockwise turn.
type of questions are asked. Particularly, in banking job exams, Let us understand it through pictorial representation:
these questions can be seen in every question papers. This is the Right turn Left turn
reason, examinees are required to pay special attention towards

Right turn

Left turn
Right turn

Left turn
such questions.
Concept of direction.
In our day to day life, we make our concept of direction after
seeing the position of sun. In fact, this is a truth that sun rises in Right turn Left turn
the East and goes down in the west. Thus when we stand facing (i) (ii)
sunrise, then our front is called East while our back is called
West. At this position our left hand is in the Northward and the
right hand is in the Southward. Let us see the following direction
map that will make your concept more clear: Right turn Left turn
Direction Map:
North
North-West North-East (iii) (iv)
Important point regarding direction
(1) If our face is towards North, than after left than our face will
West East be it towards West while after right turn it will be towards
East.
(2) If our face is towards South, then after left turn our face will
South-West South-East be towards East and after right turn it will be towards West.
South (3) If our face is towards East, then after left turn our face will
Note: be forwards North and after right turn it will be towards
On paper North is always on top be while South is always in South.
bottom. (4) If our face is towards West, then after left turn our face will
Concept of Degree be towards South and after right turn it will be towards
Let us see the following picture: North.
(5) If our face is towards North-West, then after left turn our
360º 360º
face will be towards South-West and after right turn it will
0º 0º be towards North-East.
315º
Anti clockwise (ACW)

45º 45º 315º (6) If our face is towards South-West, then after left turn our
Clockwise (CW)

face will be towards South-East and after right turn it will be


270º 90º 90º 270º towards North-West.
(7) If our face is towards South-East, then after left turn our
face will be towards North-East and after right turn it will be
225º 135º 135º 225º
towards South-West.
180º 180º
(8) If our face is towards North-East, then after left turn our
face will be towards North-East and after right-turn it will be
towards South-East.
A-36 DIRECTION & DISTANCE
Concept of minimum distance A EXAMPLE 2. Rashmi walks 10 km towards North. She walks
Minimum distance between initial and last point
6 km towards South then. From here she moves 3 km towards
h2 = b2 + p2, where
East. How far and in which direction is she with reference to her
h = Hypotenuse
starting point?
b = Base (a) 6 km West (b) 7 km East
P = Perpendicular P h
(c) 8 km North (d) 5 km North-East.
Remember this important rule is (e) None of these.
known as ‘Pythogoras Theorem’ Sol. It is clear, Rashmi moves from A 10 km Northwards upto B,
then moves 6 km Southwards upto C, then turns towards
B b C East and walks 3 km upto D.
EXAMPLE 1. Raman walked 2 km West from his office and Then, AC = (AB – BC) = 10 – 6 = 4 km
then turned South covering 4 km. Finally, he waked 3 km towards CD = 3km.
East and again move 1 km West. How far is Raman from his
initial position. B
(a) 4 km (b) 8 km 6 km
(c) 10 km (d) 7 km 3 km
(e) None of these D
C
Sol. Raman starts from his office A, moves 2 km West upto B,
then 4 km to the South upto C, 3 km East upto D and finally
1 km West upto E, Thus his distance from the initial position 10 km
A = AE = BC = 4 km.
Hence option (a) is the correct answer.
B 2 km A
A

\ Rashmi’s distance from starting point A

1 km = AD = AC 2 + CD 2 = 4 2 + 3 2 = 16 + 9 = 25 = 5 km .
D
C 2 km E From figure, D is to the North-East of A, Hence (d) is the
correct option

EXERCISE
1. Q travels towards East. M travels towards North. S and T 4. M is to the East of D, F is to the South of D and K is to the
travel in opposite directions. T travels towards right of Q. West of F. M is in which direction with respect to K?
Which of the following is definitely true? (a) South-West (b) North-West
(a) M and S travel in the opposite directions. (c) North-East (d) South-East
(b) S travels towards West. (e) None of these
(c) T travels towards North. 5. After 4 pm on a sunny day when Ramesh was returning
(d) M and S travel in the same direction. from his school, he saw his uncle coming in the opposite
(e) None of these direction. His uncle talked to him for some time. Ramesh
2. P, Q, R, S and T are sitting around a circular table. R is to the saw that the shadow of his uncle was to his right side.
right of P and is second to the left of S. T is not between P Which direction was his uncle facing during their talk?
and S. Who is second to the left of R? (a) North (b) South
(a) S (b) T (c) East (d) Data inadequate
(c) Q (d) Data inadequate (e) None of these
(e) None of these 6. A and B arestanding at a distanceof 20 km from each other
3. Of the five villages P, Q, R, S and T situated close to each on a straight East-West road. A and B start walking
other, P is to west of Q, R is to the south of P, T is to the
simultaneously, eastwardsand westwardsrespectively, and
north of Q, and S is to the east of T. Then, R is in which
both cover a distance of 5 km. Then A turns to his left and
direction with respect to S?
walks10 km. ‘B’ turnsto hisright and walks10 kmand at the
(a) North-West (b) South-East
(c) South-West (d) Data Inadequate
samespeed. Then both turn to their left and cover adistance
(e) None of these of 5 km at thesamespeed. What will bethedistancebetween
them?
DIRECTION & DISTANCE A-37
(a) 10 km (b) 5 km (a) North (b) South
(c) 20 km (d) 25 km (c) East (d) West
(e) None of these (e) None of these
7. Alok walked 30 metres towards east and took a right turn 16. A rat runs 20' towards east and turns to right, runs 10' and
and walked 40 metres. He again took a right turn and walked turns to right, runs 9' and again turns to left, runs 5' and
50 metres. Towards which direction is he from his starting then to left, runs 12' and finally turns to left and runs 6'.
point? Now, which direction is the rat facing?
(a) South (b) West (a) East (b) West
(c) South-West (d) South-East (c) North (d) South
(e) None of these (e) None of these
8. Ten boys are standing in a row facing the same direction. 17. If South-east becomes North, North-east becomes West
Abhijit, who is seventh from the left end of the row, is to the and so on, what will West become?
immediate right of Sushant, who is fifth from the right end (a) North-east (b) North-west
of the row. Sushant is third to the right of Rupin. How many (c) South-east (d) South-west
children are there between Abhijit and Rupin? (e) None of these
(a) One (b) Two 18. P, Q, R and S are playing a game of carrom. P, R and S, Q are
(c) Three (d) Data inadequate partners. S is to the right of R who is facing west. Then, Q
(e) None of these is facing
9. Y is to the East of X, which is to the North of Z. If P is to the (a) North (b) South
South of Z, then P is in which direction with respect to Y? (c) East (d) West
(a) North (b) South (e) None of these
(c) South-East (d) North-East 19. A and B start walking, from a point, in opposite directions.
(e) None of these A covers 3 km and B covers 4 km. Then A turns right and
10. One afternoon, Manisha and Madhuri were talking to each walks 4 km while B turns left and walks 3 km. How far is each
other face to face in Bhopal on M.G. Road. If Manisha’s from the starting point ?
shadow was exactly to the left of Madhuri, which direction (a) 5 km (b) 4 km
was Manisha facing? (c) 10 km (d) 8 km
(a) North (b) South (e) None of these
(c) East (d) Data inadequate 20. Anuj started walking positioning his back towards the sun.
(e) None of these After sometime, he turned left, then turned right and then
11. ‘X’ started walking straight towards South. He walked a towards the left again. In which direction is he going now?
distance of 5 metres and then took a left turn and walked a (a) North or South (b) East or West
distance of 3 metres. Then he took a right turn and walked (c) North or West (d) South or West
a distance of 5 metres again. ‘X’ is facing which direction (e) None of these
now? 21. From her home, Prerna wishes to go to school. From home,
(a) North-East (b) South she goes towards North and then turns left and then turns
(c) North (d) South-West right, and finally she turns left and reaches school. In which
(e) None of these direction her school is situated with respect to her home?
12. If A is to the south of B and C is to the east of B, in what (a) North - East (b) North - West
direction is A with respect to C? (c) South - East (d) South - West
(a) North-east (b) North- west (e) None of these
(c) South-east (d) South-west 22. One day, Ravi left home and cycled 10 km southwards,
(e) None of these turned right and cycled 5 km and turned right and cycled 10
13. One morning after sunrise, Gopal was facing a pole. The km and turned left and cycled 10 km. How many kilometres
shadow of the pole fell exactly to his right. Which direction will he have to cycle to reach his home straight?
was he facing? (a) 10 km (b) 15 km
(a) South (b) East (c) 20 km (d) 25 km
(c) West (d) Data inadequate (e) None of these
(e) None of these 23. Rasik walks 20 m North. Then, he turns right and walks 30
14. A boy rode his bicycle northwards, then turned left and m. Then he turns right and walks 35 m. Then he turns left
rode one km and again turned left and rode 2 km. He found and walks 15 m. Then he again turns left and walks 15 m. In
himself exactly one km west of his starting point. How far which direction and how many metres away is he from his
did he ride northwards initially? original position?
(a) 1 km (b) 2 km (a) 15 metres West (b) 30 metres East
(c) 3 km (d) 5 km (c) 30 metres West (d) 45 metres East
(e) None of these (e) None of these
15. Ravi wants to go to the university. He starts from his home 24. From his house, Lokesh went 15 km to the North. Then he
which is in the East and come to a crossing. The road to the turned West and covered 10 km. Then, he turned South and
left ends is a theatre, straight ahead is the hospital. In which covered 5 km. Finally , turning to East, he covered 10 km. In
direction is the university? which direction is he from his house?
A-38 DIRECTION & DISTANCE
(a) East (b) West (a) 35 metres East (b) 35 metres North
(c) North (d) South (c) 40 metres East (d) 60 metres East
(e) None of these (e) None of these
25. Kailash faces towards north. Turnings to his right, he walks 29. One morning after sunrise, Reeta and Kavita were talking to
25 metres. He then turns to his left and walks 30 metres. each other face to face at Tilak Square. If Kavita’s shadow was
Next, he moves 25 metres to his right. He then turns to the exactly to the right to Reeta, which direction Kavita was facing?
right again and walks 55 metres. Finally, he turns to the (a) North (b) South
right and moves 40 metres. In which direction is he now (c) East (d) Data inadequate
from his starting point ? (e) None of these
(a) South-West (b) South 30. I am facing east. I turn 100° in the clockwise direction and
(c) North-West (d) South-East then 145° in the anticlockwise direction. Which direction
(e) None of these am I facing now?
26. A clock is so placed that at 12 noon its minute hand points (a) East (b) North-east
towards north-east. In which direction does its hour hand (c) North (d) South-west
point at 1: 30 pm ?
(e) None of these
(a) North (b) South
31. A man is facing north-west. He turns 90° in the clockwise
(c) East (d) West
direction, then 180° in the anticlockwise direction and then
(e) None of these
27. One evening before sunset two friends Sumit and Mohit another 90° in the same direction. Which direction is he
were talking to each other face to face. If Mohit’s shadow facing now?
was exactly to his right side, which direction was Sumit facing? (a) South (b) South-west
(a) North (b) South (c) West (d) South-east
(c) West (d) Data inadequate (e) None of these
(e) None of these 32. A man is facing west. He runs 45° in the clockwise direction
28. Rohit walked 25 metres towards South. Then he turned to and then another 180° in the same direction and then 270° in
his left and walked 20 metres. He then turned to his left and the anticlockwise direction. Which direction is he facing now?
walked 25 metres. He again turned to his right and walked (a) South (b) North-west
15 metres. At what distance is he from the starting point (c) West (d) South-west
and in which direction? (e) None of these

ANSWER KEY
1 (d) 5 (b) 9 (e) 13 (a) 17 (c) 21 (b) 25 (d) 29 (a)
2 (c) 6 (a) 10 (a) 14 (b) 18 (a) 22 (b) 26 (c) 30 (b)
3 (c) 7 (c) 11 (b) 15 (a) 19 (a) 23 (d) 27 (b) 31 (d)
4 (c) 8 (c) 12 (d) 16 (c) 20 (a) 24 (c) 28 (a) 32 (d)

Answers &
Explanations
1. (d) We have been given that Q travels towards East and T
3. (c)
M travels towards North. Now, T travels towards right
of Q implies that T travels towards South. Hence, S P S
travels towards North (because S and T travel in
opposite directions). Therefore, it is definitely true that Q
M and S travel in the same direction i.e., North. R
Hence, R is to the South-West with respect to S.
2. (c) 4. (c)
D M

Q is second to the left of R.

K F
M is to the North-East of K.
DIRECTION & DISTANCE A-39
5. (b) After 4 pm the shadow will be towards East. Now, East 12. (d) Clearly, comparing the direction of A w.r.t. C in the
is to the right of Ramesh. So Ramesh faces North. And second diagram with that in the first diagram, A will be
his uncle, who is opposite him, faces South. south-west of C.
5 km B 5 km
6. (a) A N
B C

W E
10 km
SW A
S

13. (a) The Sun rises in the east. So, in morning, the shadow
A B
5 km 5 km
falls towards the west. Now, shadow of pole falls to
20 km the right of Gopal.Therefore, Gopal’s right side is the
west. So, he is facing South.
C 1km B
14. (b) Clearly, the boy rode from A to B,
Starting point then to C and finally up to D. Since 2km
7. (c) N D lies to the west of A, so required
30 m
distance = AB = CD = 2 km. D 1km A
W E 15. (a) Starting from his house in the East, Ravi moves
40 m
westwards. Then, the theatre, which is to the left, will be
S in the South. The hospital, which is straight ahead, will
be to the West. So, the University will be to the North.
50 m
N University
8. (c) only three students
W E
Hospital Home
R S A
9. (e) Theatre
S
N
X Y 16. (c) The movements of rat are as shown in figure. Clearly, it
is finally walking in the direction FG i.e. North.
W E
Z 20' B
A
SW 10'
P S D 9' C G

10. (a) In the afternoon the sun is in the west. Hence the 5' 6'
shadow is in the east. Now, east is to the left of
E 12' F
Madhuri. So, Madhuri is facing south. Therefore,
Manisha, who is face to face with Madhuri, is facing
17. (c) Here, each direction moves 90° + 45° = 135°
north.

N SW
S N
5
NW NE
11. (b) W E 3 SE NW
W E
5 SE
SW S
E
S
NE
A-40 DIRECTION & DISTANCE

18. (a) Here, R faces towards West. S is to the right of R. So,


S is facing towards South. Thus, Q who is the partner 10 km
D A (Ravi)
of S, will face towards North. E

S N 10 km 10 km

P R W E C B
5 km

Q S 23. (d) The movements of Rasik from A to F are as shown in


figure.
19. (a) Here, O is the starting point. Since CD = AB + EF, so F lies in line with A.
Rasik’s distance from original position A = AF
= (AG + GF) = (BC + DE) = (30 + 15) m = 45m.
Also, F lies to the east of A.
B 30 m
C
4
20 m
G F
3 (Rasik) A 35 m 15 m
A 3 O 4 B
D 15 m E
24. (c) The movements of Lokesh are as shown in figure. (A
to B, B to C, C to D to E). Clearly, his final position is E
Both A and B are 32 + 42 = 5 km from the starting which is to the North of his house A.
point.
C 10 km B
20. (a) Clearly, there are two possible movements of Anuj as

15 km
shown below: 5 km
E
D 10 km
North

Sun A (Lokesh)

25. (d)

Sun South 25 m
N

21. (b) 30 m
N
NW NE School
W E
25 m 55 m
W E

SE
Sou

SW NW Home
S S
th - E
ast

It is clear from the diagram that school is in North-west


40 m
direction with respect to home.
22. (b) Here, Ravi starts from home at A, moves 10 km End point
southwards up to B, turns right and moves 10 km up to
C, turns right again and moves 10 km up to D and
26. (c) The positions of the minute and hour hands at 12 noon
finally turns left and moves 10 km up to E.
and 1:30 p.m. are as shown in the diagram. Comparing
Thus, his distance from initial position A = AE with direction figure, we see that the hour hand at
= AD + DE 1:30 p.m. points towards the East.
= BC + DE = (5 + 10) km = 15 km.
DIRECTION & DISTANCE A-41
30. (b) As shown in figure, the man initially faces towards
NE east i.e., in the direction OA. On moving 100° clockwise,
N E he faces in the direction OB. On further moving 145°
11 12 1 anticlockwise, he facings the direction OC. Clearly, OC
10 2
NW SE makes an angle of (145° – 100°) i.e. 45° with OA and
9 3
so, the man faces in the direction North-east.
8 4
7 6 5 W S C
SW

O A
100°
145°
11 12 1
10 2
9 3 B
8 4
7 6 5
31. (d) As shown in figure, the man initially faces in the
direction OP. On moving 90° clockwise, he faces in the
direction OX. On further moving 180° anticlockwise,
27. (b) In the evening, sun is in the west and so the shadows he faces in the direction OY. Finally, on moving 90°
fall towards east. So, Mohit’s shadow fell towards east. anticlockwise, he faces in the direction OZ, which is
Now, since Mohit’s shadow fell towards right, therefore, South-east.
Mohit is facing North. So Sumit, standing face to face N
X
NW NE P
with Mohit, was facing South. 180°
28. (a) The movements of Rohit are as shown in figure. 90°
W E
O
Rohit’s distance from starting point A = AE 90°
= (AD + DE) = (BC + DE) = (20 + 15) m = 35 m. SW SE
S Y Z
Also, E is to the East of A.
32. (d) Clearly, the man initially faces in the direction OA. On
moving 45° clockwise, he faces in the direction OB. On
(Rohit) 15 m further moving 180° clockwise, he faces in the direction
A E
D OC. Finally, on moving 270° anticlockwise, he faces in
the direction OD, which is South-west. Hence, the
25 m 25 m
answer is (d)

B 20 m C B
270°
29. (a) In morning, sun rise in the east so shadow of a object 180°
45°
O
falls towards the west. Now, Kavita’s shadow falls to A
the rights of Reeta. Hence, Reeta is facing South and 90°
Kavita is facing North.

N D C
Reeta

W E

Kavita's Kavita
Shadow S
A-42 BLOOD RELATION

Blood
5 Chapter Relation
Problems based on blood relations are very important segment given indirect relation. It does mean questions are in the
of analytical reasoning. The question papers of almost every form of indirect relation & one has to convert this indirect
competitive exams of objective type include 4–5 questions based relation into direct relation. For example “only son of my
on blood relation. Particularly for getting jobs in banking sectors, father” does mean ‘me’ (myself). Here in place of ‘me’ indirect
one has a good skill of solving such questions. In this chapter, relation has been given in form of “only son of my father”.
we are giving quicker approach to crack problems based on blood Similarly, “the only daughter of the parents in laws of the
relation. husband of Vandana” does mean ‘Vandana’ herself. In this
example also the sentence “the only daughter of the parents
MEANING OF BLOOD RELATION in laws of the husband of ‘Vandana’ has been given in the
Blood relation does mean biological relation. Remember a wife form of indirect relation. Below are given some indirect
and husband are met biologically related but they are biological relation in the form of a list. Examinees are required to learn
parents of their own children. Similarly, brother, sister, paternal them by heart. If are keeps this list in one’s mind, he/she will
grandfather, paternal grandmother maternal grandfather, maternal find it very easy to solve problems based on blood relations.
grandmother, grandson, granddaughter, niece, cousin etc. are 1. Son of father or mother : Brother
our blood relatives.
2. Daughter of father or mother : Sister
TYPES OF BLOOD RELATIONS 3. Brother of father : Uncle
There are mainly two types of blood relatives: 4. Brother of mother : Maternal uncle
(i) Blood relation from paternal side 5. Sister of father : Aunt
(ii) Blood relation from maternal side
6. Sister of mother : Aunt
Now, we will discuss both kind of relations one-by one.
(i) Blood relation from paternal side : This type of blood 7. Father of father : Grandfather
relation can be further subdivided into three types: 8. Father of father of father : Great grand father
(a) Past generations of father : Great grandfather, great 9. Father of grandfather : Great grandfather
grandmother, grandfather, grandmother etc.
10. Mother of father : Grandmother
(b) Parallel generations of father: Uncles (Brothers of father).
aunts (sisters of father) etc. 11. Mother of mother of father : Great grandmother
(c) Future generations of father: Sons, daughters, grandsons, 12. Mother of grandmother : Great grandmother
granddaughters etc. 13. Father of mother : Maternal grandfather
(ii) Blood relation from maternal side: This type of blood
14. Father of father of mother : Great maternal grand
relations can also be subdivided into three types:
father
(a) Past generations of mother: Maternal great grandfather,
maternal great grandmother, maternal grandfather, maternal 15. Father of maternal : Great maternal
grandmother etc. grandfather grandfather
(b) Parallel generations of mother: Maternal uncles, maternal 16. Mother of mother : Maternal grandmother
aunts etc. 17. Mother of mother of mother : Great maternal
(c) Future generations of mother: Sons, daughters, grandmother
grandsons, granddaughters etc. 18. Mother of maternal : Great maternal
In the examinations, the questions are given in complicated grandmother grandmother
way. In other words, in the given questions, the easy
relationship takes the complicated form and examinees are 19. Wife of father : Mother
expected to solve this complication in order to find out the 20. Husband of mother : Father
correct answer. How does an examinee get aid of this 21. Wife of Grandfather : Grandmother
complication? For this, an examinee sees the given data in 22. Husband of Grandmother : Grandfather
the question with a serious eye; then tries to establish
relation among elements of given data on the basis of certain 23. Wife of son : Daughter-in-law
logic and finally finds out the required answer. In fact 24. Husband of daughter : Son-in-law
complications in the asked question occur because of the 25. Brother of Husband : Brother-in-law
BLOOD RELATION A-43
26. Brother of wife : Brother-in-law Remember: Solution Tips
27. Sister of Husband : Sister-in-law (a) While solving blood relation based question, first of all
find out that two persons between whom a relationship has
28. Sister of wife : Sister-in-law
to be established.
29. Son of brother : Nephew (b) Next, try to find out middle relation
30. Daughter of brother : Niece (c) Finally findout the relationship between two persons to be
31. Wife of brother : Sister-in-law identified for this purpose.
32. Husband of sister : Brother-in-law Type of problems
(1) General problems of blood relation
33. Son of sister : Nephew
(2) Blood relation based on family tree
34. Daughter of sister : Niece (3) Coded blood relationship.
35. Wife of uncle : Aunt Now, we will discuss all the three types of problems one by one
36. Wife of maternal uncle : Aunt (1) General problem of blood relation
37. Son/daughter of uncle/Aunt : Cousin Sample Q: Pointiry towards a photograph, Mr. Sharma said, “She
38. Son/daughter of maternal : Cousin is the only daughter of mother of my brother’s sister.” How
uncle/maternal aunt is Mr. Sharma related to the lady in the photograph?
(a) Cousin (b) Sister
39. Son/daughter of sister : Cousin
(c) Aunt (d) Daughter in law
of Father (e) None of these
40. Son/daughter of sister : Cousin Ans. Here we have to find relationship between Mr. Sharma &
of Mother the lady in the photograph.
Mother of my brother’s sister does mean my (Mr. Sharma’s)
41. Only son of grandfather : Father
mother. Only daughter of Mr. Sharma’s mother does mean
42. Only daughter of maternal : Mother “sister of Mr. Sharma”. Hence option (b) is the correct
grandfather answer.
43. Daughter of grandfather : Aunt (2) Blood relation based on family tree
44. Sons of grandfather other : Uncle Sample Q: Q is the brother of C and C is the sister of Q. R and D
than father are brother and sister. R is the son of A while A & C are wife
45. Son of maternalgrandfather : Maternal Uncle. and husband. How is Q related with D.
Ans. For such type of question a family tree is made in which
/maternal grand mother
some symbols are used as below:
46. Only daughter in law of : Mother ‘Û’ is used for husband & wife.
grandfather/ grandmother ‘___’ is used for brother & sister
47. Daughters in law of : Aunt other than mother ‘ | ’ is used for parents (father or mother). Parents are put on
grandfather/ grandmother top while children are put at the botom.
‘–’ or minus sign is used for female
48. Daughters-in-law of : Aunt maternal
‘+’ or plus sign is used for male.
maternal grandfather/ grandmother Now adopting and using the above given symbols we can
49. Neither brother nor sister : Self make a family tree and solve the given problem, let us see
the family tree for sample question:
SOME IMPORTANT INFORMATION ABOUT BLOOD
Family + tree :
RELATION
A. Without the information of gender, no relationship can be – +
A+ C Q
established between two people. For example, If given that
R is the child of P & Q, then we can only say that P & Q are
the parents of R. But we can not find out:
(i) R is the son of P & Q or R is the daughter of P & Q.
(ii) Who is mother of R and who is father of R.
But if we have given that P is a male, Q is a female and R is R+ D–
male, then we can easily say that R is the son of P and Q.
Further we can also say that P is father of R and Q is mother As per the question Q is the brother of c and c is the sister of Q.
of R. Hence relation between C & Q has been presented as C — Q(
– +
)
B. Gender can not be decided on the basis of name. For example where ‘–’ sign above C makes it clear that C is a female and ‘+’
in Sikh community the names like Manjit, Sukhvinder etc. sign above ‘Q’ makes it clear that Q is a male. Similarly for R and
are the names of both male and female. Similarly, in the æ+ ö
Hindu Community ‘Suman’ is the name of both male and D. The presentation ç R — D- ÷ has been made. Further
è ø
female. according to the question.
A-44 BLOOD RELATION
A and C are having a husband and wife relationship and hence
æ+ ö N– K+
this has been presented as ç A Û C - ÷ . As it is already given
è ø
that C is the sister of Q and A and C are wife and husband, this
becomes clear that A is the male member of the family and this is
the reason A has ‘+’ as its gender sign. Lastly, the vertical line
gives father and son relationship and has been presented as
M
æ A+ ö
ç | ÷ . Now from this family tree it becomes clear that C is the can find out that this option can not give you the gender of M.
è R+ ø For this only a serious look at the option is enough.
mother of R and D and as Q is the brother of C, then Q will Presentation of option (2) [N× K ÷ M]
definitely be the maternal uncle of R & D. Hence we can say that
Q is the maternal uncle of D and this is the required answer for N+ K–
our sample question. This family tree presentation also does not give the
M
NOTE : Sample problem is the very easy type of blood relation gender of M. Like option (1) this option gives you a clear
problem. In this chapter we will solve only such type of problem indication, only by a serious look, that gender of M can not be
based on family tree. But we will solve more difficult problems of find out and for this making family tree is not necessary. Hence
such type in a different chapter “Problem solving”. option (2) is also rejected.
(3) Coded blood relationship. Presentation of option (3) [ N ÷ K × M]
Sample: Directions: Read the following informations carefully Like option (1) and (2), option (3) is also rejected and only a
to give the answers of following questions: serious look can make you clear that in this case also the gender
‘P × Q’ means P is the brother of Q of M can not be find out
‘P – Q’ means P is the sister of Q –
‘P + Q’ means P is the father of Q N
‘P ÷ Q’ means P is the mother of Q.
K+ M
Which of the following option is the presentation of M is the
nephew of N? Presentation of option (4) [ N – K + M × T]
(1) N – K + M This presentation makes it clear that N is the sister of K who is
(2) N × K ÷ M father of M. Here gender of M is clear that M is a male. Hence, M
(3) N ÷ K × M is clearly nephew of N.
(4) N – K + M × T
(5) None of these. N– K+
Solution: To solve it we will use the symbols of family tree in
place of mathematical signs ( + , –, × & ÷). Let us make family tree M+ T
presentation for every option: Hence for sample question option (4) is the correct answer. Now
Presentation for option (1) [N – K + M] we have come to the conclusion of this chapter. Readers are
Here gender of M can not be find out so, this option is rejected, advised to practice more and more to crack such questions quick
point to be noted that even without making a family tree. You as possible.

EXERCISE
1. Anil, introducing a girl in a party, said, she is the wife of the 4. Vinod is the brother of Bhaskar. Manohar is the sister of
grandson of my mother. How is Anil related to the girl? Vinod. Biswal is the brother of Preetam and Preetam is the
(a) Father (b) Grandfather daughter of Bhaskar. Who is the uncle of Biswal?
(c) Husband (d) Father-in-law (a) Bhaskar (b) Manohar
(e) None of these
2. A man said to a woman, “Your mother’s husband’s sister is (c) Vinod (d) Insufficient data
my aunt.” How is the woman related to the man ? (e) None of these
(a) Granddaughter (b) Daughter 5. A man said to a woman, “Your brother’s only sister is my
(c) Sister (d) Aunt mother.” What is the relation of the woman with the maternal
(e) None of these grandmother of that man?
3. Introducing Rajesh, Neha said, “His brother’s father is the
only son of my grand father”. How Neha is related to Rajesh? (a) Mother (b) Sister
(a) Sister (b) Daughter (c) Niece (d) Daughter
(c) Mother (d) Niece (e) None of these
(e) None of these
BLOOD RELATION A-45
6. Pointing to a photograph, a man said, “ I have no brother or 14. How is Vikash’s wife related to Neela?
sister but that man’s father is my father’s son.” Whose (a) Sister (b) Niece
photograph was it? (c) Sister-in-law (d) Data inadaequate
(a) His own (b) His son’s (e) None of these
(c) His father’s (d) His nephew’s 15. Pointing to a girl, Abhishek said, “She is daughter of the
(e) None of these only child of my father.” How is Abhishek’s wife related to
7. Pointing to a photograph, a lady tells Pramod, “I am the that girl?
only daughter of this lady and her son is your maternal
(a) Daughter (b) Mother
uncle,” How is the speaker related to Pramod’s father?
(c) Aunt (d) Sister
(a) Sister-in-law
(b) Wife (e) None of these
(c) Neither (a) nor (b) 16. Introducing Sarita, Meena said, “She is the only daughter
(d) Aunt of my father’s only daughter.” How is Meena related to
(e) None of these Sarita?
8. Introducing a man, a woman said, “His wife is the only (a) Niece (b) Cousin
daughter of my mother.” How is the woman related to that (c) Aunt (d) Data inadequate
man? (e) None of these
(a) Aunt (b) Wife Directions (Qs. 17-21): Each of the questions below consists of
(c) Mother-in-law (d) Maternal Aunt a question and two or three statements given below it. You have
(e) None of these to decide whether the data provided in the statements are sufficient
9. Deepak said to Nitin, “That boy playing with the football is to answer the question.
the younger of the two brothers of the daughter of my 17. Who is the uncle of L?
father’s wife.” How is the boy playing football related to A P, brother of M, is father of L; M is father of S.
Deepak? B R is father of L’s cousin.
(a) Son (b) Brother (a) A alone is sufficient
(c) Causin (d) Nephew
(b) B alone is sufficient
(e) None of these
(c) Either A alone or B alone is sufficient
10. A is the mother of B. C is the father of B and C has 3 children.
On the basis of this information, find out which of the (d) Both A and B together are not sufficient
following relations is correct : (e) Both A and B together are necessary
(a) C has three daughters. 18. How is A related to B?
(b) C has three sons. A P, the only son of A, has two sisters.
(c) B is the son. B A's son is the brother of the only sister of B.
(d) None of these. C B and P are children of A.
(e) B is the daughter (a) Both A and C are sufficient
11. A man pointing to a photograph says, “The lady in the (b) Only B
photograph is my nephew’s maternal grandmother.” How (c) Either A or B
is the lady in the photograph related to the man’s sister (d) Both B and C
who has no other sister?
(e) None of these
(a) Cousin (b) Sister-in-law
19. How many daughters does W have?
(c) Mother (d) Mother-in-law
(e) None of these A B and D are sisters of M.
12. A is the brother of B. A is the brother of C. To find what is B M's father T is the husband of W.
the relation between B and C. What minimum information C Out of three children which T has, only one is boy.
from the following is necessary? (a) Only A and C (b) All A, B and C
(i) Gender of C (ii) Gender of B (c) Only B and C (d) Only A and B
(a) Only (i) (b) Only (ii) (e) None of these
(c) Either (i) or (ii) (d) both (i) and (ii) 20. Is F grandaughter of B?
(e) None of these A B is father of M. M is the sister of T. T is the mother of
Directions (Qs.13-14): Study the information given below and F.
answer the questions following it: B S is the son of F. V is the daughter of F. R is the brother
Mohan is son of Arun’s father’s sister. Prakash is son of Reva, of T.
who is mother of Vikash and grandmother of Arun. Pranab is (a) A alone is sufficient
father of Neela and grandfather of Mohan. Reva is wife of Pranab.
(b) B alone is sufficient
13. How is Mohan related to Reva?
(c) Either A alone or B alone is sufficient
(a) Grandson (b) Son
(c) Nephew (d) Data inadaequate (d) Both A and B are not sufficient
(e) None of these (e) Both A and B together are necessary.
A-46 BLOOD RELATION
21. How is P related to J? Directions (Qs.29-30): Study the meaning of the given symbols
A M is the brother of P and T is the sister of P and answer the questions based on it.
B P's mother is married to J's husband, who has one son (i) ‘P × Q’ means ‘Q' is mother of P’.
and two daughters (ii) ‘P + Q’ means ‘P' is brother of Q’.
(a) A alone is sufficient (iii) ‘P – Q’ means ‘P' is sister of Q’.
(b) B alone is sufficient (iv) ‘P ÷ Q’ means ‘Q' is father of P’.
(c) Either A alone or B alone is sufficient 29. Which of the following definitely means R is grandson
of K?
(d) Both A and B are not sufficient
(a) R × T ÷ K (b) M + R × T ÷K
(e) Both A and B together are necessary.
(c) M – R × T ÷ K (d) Cannot be determined
22. Kalyani is mother-in-law of Veena who is Sister-in-law of
(e) None of these
Ashok. Dheeraj is father of Sudeep, the only brother of
30. Which of the following statements is superfluous to answer
Ashok. How is Kalyani related to Ashok?
the above question?
(a) Mother-in-law (b) Aunt
(a) None (b) (i) Only
(c) Wife (d) Cousin (c) (ii) Only (d) (iii) Only
(e) None of these (e) (iv) Only
23. If P $ Q means P is father of Q, P # Q means P is mother of Q, Directions (Qs. 31-32): Study the following information and
P * Q means P is sister of Q, then how is Q related to N in N answer the questions given below.
# L $ P * Q? (a) ‘P ÷ Q’ means ‘Q is father of P’
(a) Grandson (b) Granddaughter (b) ‘P × Q’ means `P is sister of Q’.
(c) Nephew (d) Data inadequate (c) P + Q’ means ‘P is brother of Q’.
(e) None of these (d) ‘P – Q’ means ‘Q is mother of P’.
Directions (Qs.24-25): Study the following information carefully 31. Which of the following means R is nephew of T?
and answer the given questions based on it: (a) R + N – Q × T (b) R – Q × N × T
(A) ‘P × Q’ means ‘Q is mother of P’. (c) R – N × T (d) T + M ÷ R
(B) ‘P + Q’ means ‘P is father of Q’. (e) T – Q ÷ R
(C) ‘P – Q’ means ‘P is brother of Q’. 32. Which of the following is/are redundant to answer the
(D) ‘P ÷ Q’ means ‘Q is sister of P’. above question?
(a) (ii) only
24. Which of the following means ‘M is niece of T’?
(b) (i) only
(a) M ÷ D + T × R (b) T – D + R ÷ M
(c) (i) and (iv) only
(c) T × D + R ÷ M (d) Cannot be determined
(d) Either (i) and (iii) or (ii) and (iv)
(e) None of these
(e) Either (i) and (ii) or (iii) and (iv)
25. Which of the following statements is redundant to answer
33. If ‘P $ Q’ means ‘P is brother of Q’, ‘P # Q’ means ‘P is
the question no. 24?
mother of Q’ and ‘P*Q’ means ‘P is daughter of Q’, then
(a) A only (b) B only
who is the father in ‘A # B $ C * D ’ ?
(c) Either A or B only (d) Either C or D only
(a) D (b) B
(e) All are required
(c) C (d) Data inadequate
26. Pointing to a boy in a photograph, Akhil says, "He is the
(e) None of these
son of my mother's only son." How is Akhil related to that
34. Pointing to a boy, Meena says, “He is the son of my
boy?
grandfather’s only son.’’ How is the boy’s mother related
(a) Uncle (b) Brother
to Meena?
(c) Father (d) Cousin (a) Mother (b) Aunt
(e) None of these (c) Sister (d) Data inadequate
27. Pointing to a boy, Namrata says, “He is the son of my (e) None of these
grandfather’s only child.” How is the boy related to Namrata? Directions (Qs. 35-36): Study the following information carefully
(a) Brother (b) Cousin and answer the given questions following it.
(c) Uncle (d) Data inadequate (i) ‘P × Q’ means ‘Q’ is the mother of ‘P’.
(e) None of these (ii) ‘P – Q’ means ‘P’ is the brother of ‘Q’.
28. Pointing to Kedar, Veena said, ‘His mother’s brother is the (iii) ‘P + Q’ means ‘P’ is the father of ‘Q’.
father of my son Nitin.’ How is Kedar related to Veena? (iv) ‘P ¸ Q’ means ‘Q’ is the sister of ‘P’.
(a) Niece (b) Aunt 35. Which of the following means M is the daughter of K?
(c) Nephew (d) Sister-in-law (a) K + R ¸ M (b) K ¸ M + R
(e) None of these (c) K × R ¸ M (d) K – R ´ M
(e) None of these
BLOOD RELATION A-47
36. Which of the following statement(s) is redundant to answer (a) P × Q ÷ R + S – T (b) P × Q ÷ S – R + T
the above question? (c) P × Q ÷ R – T + S (d) P × Q ÷ R – S + T
(a) Both (i) and (ii) (b) (i) only (d) None of these
(c) (ii) only (d) Either (i) or (iii) and (ii) 38. A + B means B is brother of A; A × B means B is husband of
(e) None of these A; A – B means A is mother of B; and A ÷ B means A is father
37. If ‘A + B’ means ‘A is brother of B’, ‘A – B’ means ‘A is sister of B. Then which of the following expressions indicates ‘P’
of B’, ‘A × B’ means ‘A is wife of B’, and ‘A ÷ B’ means ‘A is is grandmother of ‘T’?
father of B’, then which of the following indicates ‘S is son (a) Q – P + R ÷ T (b) P × Q ÷ R – T
of P’? (c) P × Q ÷ R + T (d) P + Q ÷ R – T
(e) None of these

ANSWER KEY
1 (d) 6 (b) 11 (c) 16 (e) 21 (e) 26 (c) 31 (a) 36 (a)
2 (c) 7 (b) 12 (d) 17 (c) 22 (e) 27 (a) 32 (b) 37 (d)
3 (a) 8 (b) 13 (a) 18 (e) 23 (d) 28 (c) 33 (a) 38 (b)
4 (c) 9 (b) 14 (c) 19 (c) 24 (b) 29 (e) 34 (a)
5 (d) 10 (d) 15 (b) 20 (d) 25 (a) 30 (a) 35 (a)

Answers &
Explanations
1. (d) Clearly, the grandson of Anil’s mother is son of Anil the man in the photograph, i.e. the man in the
and wife of Anil’s son is daughter in-law of Anil. Thus, photograph is his son.
Anil is the father-in-law of the girl. 7. (b) Clearly, the speaker’s brother is Pramod’s maternal
2. (c) Woman’s Mother’s husband uncle. So, the speaker is Pramod’s mother or his father’s
wife.
Woman’s father 8. (b) Clearly, only daughter of her mother is woman herself.
So, that woman is the wife of man.
Woman’s father’s sister ¾¾ ® Woman’s Aunt.
9. (b) Father’s wife — Mother; Mother’s daughter — Sister;
Since, woman’s aunt is man’s aunt
Sister’s younger brother — His brother. So, the boy is
\ woman is sister of man. Deepak’s brother.
3. (a) Father of Rajesh’s brother is the father of Rajesh.
Rajesh’s father is the only son of Neha’s grandfather. 10. (d) A C
Mother
Hence, Rajesh’s father is Neha’s father. So, Neha is Father
B
the sister of Rajesh.
Q C has three children but we can’t say that he has
4. (c) Vinod
Sister Manohar three daughters or three sons.
Brother U ncl So, options (a) and (b) are incorrect.
e
Bhaskar

Biswal

Also, we don’t know that B is a boy or girl.


D a ug
er

hter So, option (c) is also incorrect.


Broth

tam

11. (c) Clearly, the lady is the grandmother of man’s sister’s


Pree

son i.e., the mother of the mother of man’s sister’s son


i.e., the mother of man’s sister.
Thus, Vinod will be uncle of Biswal. So, the lady is man’s mother.
5. (d) The only sister of the brother of the woman will be the 12. (d) Without knowing the sex of C, we can’t be determined
woman herself and she is the mother of that man. Thus, whether B is sister of C or B is brother of C. Similarly
the woman is the daughter of the maternal without knowing the sex of B we can’t be determined
grandmother of that man. whether C is sister of B or C is brother of B. Therefore,
6. (b) Since the narrator has no brother, his father’s son is both (i) and (ii) are necessary.
he himself. So, the man who was talking is the father of
A-48 BLOOD RELATION

(13-14) : Pranab Û Reva 26. (c) Photograph is the son of Akhil’s mother’s only son.
or, Photograph is the son of Akhil.
(+) (-) or, Akhil is the father of the boy.
¯ ¯ ¯ 27. (a) Son of Namrata's grandfather's only child is Namrata's
Neela Prakash Vikash brother
(–) ( +)
** * Veena
¯ ( - ) – ( + ) Û ( -)
28. (c) | |
Mohan Arun
(+) Kedar(+ ) Nitin
15. (b) Girl is daughter of the only child of Abhishek’s father Hence, Kedar is Veena’s nephew.
or, Girl is daughter of Abhishek (29-30) :
Hence, girl is daughter of Abhisek’s wife. 29. (e) Reject1) because the equation does not tell about sex
17. (c) From (A) above of R. Reject 2) and 3) also on the same basis. In equation
2) M is the grandson of K. This implies that with the
(+)P — M(+)
| help of the given information in the direction, it is
|
L S possible to form an equation which can show that R is
the grandson of K. Hence, reject 4) also.
Hence, ‘M’ is uncle of ‘L’.
31. (a) If R is nephew of T then the required equation must
from (B) alone it is clear that
depict R as a male. Reject 2) and 3) because in these
'R' is uncle of 'L'
equations the symbol '—' denotes the sex of Q and N
Hence the statement can be answered by statement
respectively. The sex of R remains unknown. Reject 4)
(A) alone or statement (B) above.
and 5) because these equations depict R older then T.
20. (d) Using statement A :
Now, check 1)
+
B Q(–) — T
|
– –
R(+) — N
M T Obviously, R is the nephew of T.
32. (b) R +.N – Q × T is the required equation to answer the
F previous question. Here the used symbols are ‘+’, `–’
The gender of F is not known. So we can't say if F is and ‘×’. Therefore, redundant part of the symbols
granddaughter or grandson of B. given in the direction is statement (i).
Using statement B : 33. (a) Clearly, B and C are siblings. While A and D are
F parents. Now, A is the mother.,Hence, D must be the
father.
+
+ –
R T 34. (a) One’s grandfather’s only son Þ one’ss father. And the
S V son of one’s father Þ One’ss brother or oneself. Hence,
The name of B has not been mentioned. Using both the mother of the boy is Meena’s mother.
the statement together. Still the gender of F cannot be 37. (d) As ‘S’ is a female in options (a) and (b), they can be
determined. rejected directly. The sex of ‘S’ in option (c) is not
21. (e) Using statement A : known, hence, it can also be eliminated. Now, check
We cannot find the relation using this statement as J's option (d).
name is not even mentioned in it. (–) P « Q (+)
Using statement B : |
J

+ R (–) – S (+) – T
[Clearly, S is son of P.]
38. (b) (c) is ruled out because we need two generation-change
P
signs (– and ÷) between P and T. Same is the case with
We can say that J is the mother of P but cannot decide
(a) . Again, (d) is ruled out because P + Q does not
whether P is the son or daughter of J.
give the sex of P. Now, check (b)
Using both statements together :
P (–) « Q (+)
We find, P is the daughter of J.
|
22. (e) Kajyani (–) ¬ ® Dheeraj (+)
R (–)
(+) Ashok ‘Sudeep (+) ¬ ® Veena (–) |
Clearly, Kalyani is mother of Ashok. T
24. (b) T(+) – D(+)
|
R – M(–)
Time Sequence,
Number &
6 Chapter
Ranking Test
Time sequence EXAMPLE 1. Neena returned house after 3 days earlier
To solve problems related to time sequence, let us gather 1st the
than the time she had told her mother. Neena’s sister Veena
following informations :
reached five days later than the day Neena was supposed to return.
1 Minute = 60 seconds
If Neena returned on Thursday, on what day did Veena return ?
1 Hour = 60 minutes
1 Day = 24 hours (a) Friday (b) Saturday
1 Week = 7 days (c) Wednesday (d) Sunday
1 Month = 4 weeks (e) None of these
1 Year = 12 months Sol. Neena returned home on Thursday. Neena was supposed
1 Ordinary year = 365 days to return 3 days later, i.e., on Sunday.
1 Leap year =366 days Veena returned five days later from Sunday. i.e., on Friday.
1 Century = 100 years \ Option (a) is the correct option.
Other facts to be remembered
• A day is the period of the earth’s revolution on its axis. EXAMPLE 2. Vandana remembers that her father’s birthday
• A ‘Solar year’ is the time taken the earth to travel round the is between 15 th and 16 th of June. Whereas her brother
1 remembers that their Father’s birthday is between 14th and
sun. It is equal to 365 days, 5 hours, 48 minutes and 47 18 th of June. On which day is their Father’s birthday ?
2
seconds nearly. (a) 14 th June (b) 16 th June
• A ‘Lunar month’ is the time taken the moon to travel round (c) 15 th June (d) 18 th June
the earth. It is equal to nearly 28 days. (e) None of these
Leap Year Sol. According to Vandana her father’s birthday is on one of the
• If the number of a given year is divisible by 4, it is a leap days among 14 th and 15 th June. According to Vandana’s
year. Hence, the years like 1996, 2008, 2012 are leap years. brother, the father’s birthday is on one of the days among
But years like 1997, 1991, 2005, 2007 are not divisible by 4 15 th 16th and 17th June.
and therefore, such years are not leap years. It is obvious that the father’s birthday is on the day common
• In a leap year, February has 29 days. to both the above groups. The common day is 15th June.
• A leap year has 52 weeks and 2 days. Therefore, a leap year Hence, the father’s birthday falls on 15 th June.
has 2 odd days.
\ Option (c) is the correct option.
Ordinary year
• An ordinary year has 12 months. EXAMPLE 3. January 5, 1991 was a Saturday. What day of
• An ordinary year has 365 days. the week was March 4, 1992 ?
• An ordinary year has 52 weeks and 1 day. Therefore, an (a) Wednesday (b) Thursday
ordinary year has 1 odd day. (c) Saturday (d) Friday
Century (100 years) (e) None of these
• A century has 76 ordinary years and 24 leap years. Sol. Total number of days between Jan 5, 1991 and March 4,
• A century has 5 odd days. 1992
Odd days
= 360 days in 1991 + (31 + 29 + 4) days in 1994.
Odd days in an ordinary year = 1
=360 + 64 = 424
Odd days in a leap year = 2
= 60 weeks + 4 days = 4 odd days
Odd days in 100 years = 5
\ March 4, 1992 is 4 days beyond Saturday i.e., Wednesday
Odd days in 200 years = (5 × 2) = 1 week + 3 days = 3
\ Correct option is (a).
Odd days in 300 years = (5 × 3) = 2 weeks + 1 day = 1
Number Test
Odd days in 400 years = (5 × 4 + 1) = 21 days
In such test, generally you are given a long series of numbers.
= 3 weeks + 0 day = 0
The candidate is required to find out how many times a number
Similarly, each 800, 1600, 2000, 2004, etc. has 0 odd days.
satifying the conditions specified in the question occurs.
A-50 TIME SEQUENCE, NUMBER & RANKING TEST

EXAMPLE 4. How many 8s are there in the following number When arranged in descending order the numbers become
as follows :
sequence which are immediately preceded by 5 but not
immediately followed by 3? 936 715 523 297 148
38584583988588893 Now, the third number from top is 523. Hence, the last digit
of 523 is 3.
(a) One (b) 4
\ Option (d) is correct.
(c) 3 (d) 2
Ranking Test
(e) None of these
In such problems, the ranks of a person both from the top and
Sol. Let use see the following :
from the bottom are given and on the basis of this the total number
3 85 8 4 5 8 3 9 88 5 888 93 of persons is asked. Sometimes question is twisted also and
clearly, such 8 is only one. position of a particular person is asked.
\ Option (a) is correct.
EXAMPLE 6. Karishma ranks 10 th from the top and 15 th
EXAMPLE 5. What will be last digit of the 3rd number from from the bottom in an examination. Find the total number of
top when the numbers given below are arranged in descending students in Karishma’s class.
order after reversing the position of the digits within each (a) 35 (b) 31
number? (c) 28 (d) 30
517 325 639 841 792 (e) None of these
(a) 2 (b) 5
(c) 7 (d) 3 Sol. As per the question; the class has
(e) None of these (i) 15 students higher than Karishma
Sol. The given numbers are : (ii) 14 students lower than Karishma
517 325 639 841 792 (iii) Karishma
After reversing, the numbers becomes as follows : \ Total number of students = 15 + 14 + 1 = 30
715 523 936 148 297 Hence, option (d) is correct.

EXERCISE
1. Mohan and Suresh study in the same class. Mohan has 3. Rajnish is older than Rajesh and Raman. Ramesh is older
secured more marks than Suresh in the terminal examination. than Rajesh but younger than Rajeev. Raman is older than
Suresh’s rank is seventh from top among all the students in Rajeev. Who among them is oldest?
the class. Which of the following is definitely true? (a) Rajeev (b) Rajesh
(a) Mohan stood first in the terminal examination. (c) Rajnish (d) Ramesh
(b) There is at least one student between Mohan and (e) None of these
Suresh in the rank list. 4. If ‘P’ means ‘division’, ‘T’ means ‘addition’, ‘M’ means
(c) There are at the most five students between Mohan ‘subtraction’, and ‘D’ means ‘multiplication’ then what will
and Suresh in the rank list. be the value of the following expression?
(d) Suresh is five ranks lower than Mohan in the rank list. 12 M 45 D 28 P 7 T 15 = ?
(e) None of these (a) – 15 (b) 45
2. Fifteen children are standing in a row facing north. Ravi is (c) – 30 (d) 15
(e) None of these
to the immediate left of Prabha and is eighth from the left
5. If the positions of the first and the fifth digits of the number
end. Arjun is second from the right end. Which of the
83721569 are interchanged, similarly, the positions of the
following statements is not true?
second and the sixth digits are interchanged, and so on,
(a) Prabha is 7th from right end.
which of the following will be the third from the right end
(b) There are four children between Prabha and Arjun. after the rearrangement?
(c) There are five children between Ravi and Arjun. (a) 6 (b) 3
(d) Arjun is 13th from the left end. (c) 2 (d) 7
(e) Ravi is exactly in the middle. (e) None of these
TIME SEQUENCE, NUMBER & RANKING TEST A-51
6. In a class some students play cricket only, some other which of the following days was the case put up to the
head clerk by the senior clerk?
1
students play football only and remaining th students (a) Wednesday (b) Thursday
6
(c) Friday (d) Saturday
play both cricket and football. Which of the following (e) None of these
statements is definitely true?
13. “Jayant could not reach Pune from Mumbai on last Saturday
(a) Two-thirds of the students play cricket. day because of non-availability of tickets”. Which of the
(b) Three-fourths of the students play football only. following, if true, would support and strengthen this
(c) One-thirds of the students play football only. statement?
(d) Three-fourths of the students play cricket only. (i) Last Friday evening, he had booked a luxury car for 3
(e) None of these days for going to a picnic spot near Vasal for his boss.
(ii) He was seen at railway reservation counter requesting
7. If the positions of the first and the sixth digits of the group
for a ticket for Pune on Saturday morning.
of digits 5904627813 are interchanged, similarly, the
(iii) His secretary had contacted several travel agents to
positions of the second and the seventh are interchanged,
get a seat for Jayant on last Thursday, Friday and even
and so on, which of the following will be the fourth from the
Saturday morning.
right end after the rearrangement?
(iv) Jayant attended a dinner party last Saturday evening.
(a) 4 (b) 9
(v) Jayant’s wife was reluctant to go to Pune last week.
(c) 1 (d) 0 (a) Only (i), (ii) and (v) (b) Only (ii) and (iii)
(e) None of these (c) Only (iv) and (v) (d) Only (i), (iii) and (iv)
8. In a row of boys Akash is fifth from the left and Nikhil is (e) None of these
eleventh from the right. If Akash is twenty-fifth from the 14. If Nikhil is eleventh from the left in a row of boys, Rehaman
right then how many boys are there between Akash and is fourteenth from the right, how many boys are there in the
Nikhil? row?
(a) 14 (b) 13 (a) 25 (b) 23
(c) 15 (d) 12 (c) 36 (d) Data inadequate
(e) None of these (e) None of these
9. The positions of the first and the sixth digits in the number 15. If it is possible to make a number which is perfect square of
3597280164 are interchanged. Similarly, the positions of the a two-digit odd number with the second, the sixth and ninth
second and the seventh digits are interchanged, and so on. digits of the number 187642539. which of the following is
Which of the following will be the fourth digit from the right the digit in the unit’s place of that two-digit odd number ?
end after the rearrangement? (a) 1 (b) 7
(a) 5 (b) 3 (c) 9
(d) No such number can be made
(c) 9 (d) 4
(e) More than one such number can be made
(e) None of these
16. A, B, C, D and E, when arranged in descending order of
10. In a shop, there were 4 dolls of different heights M, N, O their weight from top, A becomes third, E is between D and
and P. ‘P’ is neither as tall as ‘M’ nor as short as ‘O’. ‘N’ is A, C and D are not at the top. Who among them is the
shorter than ‘P’ but taller than ‘O’. If Anvi wants to purchase second?
the tallest doll, which one should she purchase? (a) C (b) B
(a) Either M or P (b) Either P or N (c) E (d) Data inadequate
(c) Only P (d) Only M (e) None of these
(e) None of these 17. Vijay’s position is 14th from upwards in a class of 43
11. Ketan takes casual leave only on first working day of every students. What will be his position from downwards?
month. The office has weekly offs on Saturday and Sunday. (a) 30th (b) 28th
In a month of 30 days, the first working day happened to be (c) 29th (d) 31st
Tuesday. What will be the day for his next casual leave? (e) None of these
18. Rakesh is on 9th position from upwards and on 38th position
(a) Wednesday (b) Thursday
from downwards in a class. How many students are in class?
(c) Friday (d) Monday (a) 47 (b) 45
(e) None of these (c) 46 (d) 48
12. Abhay gave an application for a new ration card to the clerk (e) None of these
on Monday afternoon. Next day was a holiday. So the clerk 19. Sarita is on 11th place from upwards in a group of 45 girls. If
cleared the papers on the next working day on resumption we start counting from downwards, what will be her place?
of duty. The senior clerk checked it on the same day but (a) 36th (b) 34th
forwarded it to the head clerk on next day. The head clerk (c) 35th (d) Can not be determined
decided to dispose the case on the subsequent day. On (e) None of these
A-52 TIME SEQUENCE, NUMBER & RANKING TEST
20. Raman is 9th from downwards in a class of 31 students. downwards. How many students failed?
What will be his position from upwards? (a) 19 (b) 20
(a) 21st (b) 22nd (c) 15 (d) 18
(c) 23rd (d) 24th (e) None of these
(e) None of these 29. In a row at a bus stop, A is 7th from the left and B is 9 th from
21. Some boys are sitting in a line. Mahendra is on 17th place the right. Both of them interchange their positions and thus
from left and Surendra is on 18th place from right. There are A becomes 11th from the left. How many people are there in
8 boys in between them. How many boys are there in the that row?
line? (a) 18 (b) 19
(a) 43 (b) 42 (c) 20 (d) 21
(c) 41 (d) 44 (e) None of these
(e) None of these 30. In a row of boys facing the North, A is sixteenth from the
22. In a line of boys, Ganesh is 12th from the left and Rajan is left end and C is sixteenth from the right end. B, who is
15th from the right. They interchange their positions. Now, fourth to the right of A, is fifth to the left of C in the row.
Rajan is 20th from the right. What is the total no. of boys in How many boys are there in the row ?
the class? (a) 39 (b) 40
(a) 30 (b) 29 (c) 41 (d) 42
(c) 32 (d) 31 (e) None of these
(e) None of these 31. In a class of 60, where girls are twice that of boys, kamal
23. In a queue, Vijay is fourteenth from the front and Jack is ranked seventeenth from the top. If there are 9 girls ahead
seventeenth from the end, while Mary is in between Vijay of kamal, how many boys are after him in rank ?
and Jack. If Vijay be ahead of Jack and there be 48 persons (a) 3 (b) 7
in the queue, how many persons are there between Vijay (c) 12 (d) 23
and Mary? (e) None of these
(a) 8 (b) 7 32. Ravi is 7 ranks ahead of Sumit in a class of 39. If Sumit's rank
(c) 6 (d) 5
is seventeenth from the last, what is Ravi's rank from the
(e) None of these start?
24. Malay Pratap is on 13th position from the starting and on
(a) 14th (b) 15th
17th position from the end in his class. He is on 8th position
(c) 16th (d) 17th
from the starting and on 13th position from the end among
(e) None of these
the students who passed. How many students failed?
33. In a queue, A is eighteenth from the front while B is six-
(a) 7 (b) 8
teenth from the back. If C is twentieth from the front and is
(c) 9 (d) Can not be determined
exactly in the middle of A and B, then how many persons
(e) None of these
are there is the queue ?
25. In a row of students, Ramesh is 9th from the left and
(a) 45 (b) 46
Suman is 6th from the right. When they both interchange
(c) 47 (d) 48
their positions then Ramesh will be 15th from the left. What
will be the position of Suman from the right? (e) None of these
(a) 12th (b) 13th 34. In a row of 21 girls, when monika was shifted by four place
(c) 15th (d) 6th towards the right, she became 12 th from the left end. What
(e) None of these was her earlier positions from the right end of the row ?
26. In a row of children, Bhusan is seventh from the left and (a) 9th (b) 10th
Motilal is fourth from the right. When Bhusan and Motilal (c) 11th (d) 14 th
exchange positions, Bhusan will be fifteenth from the left. (e) None of these
Which will be Motilal’s position from the right ? 35. In a row of girls . Rita and monika occupy the ninth place
(a) Eighth (b) Fourth from the right end and tenth place from the left end respec-
(c) Eleventh (d) Twelfth tively. If the interchange their places, then Rita and monika
(e) None of these occupy seventh place from the right and eighteenth place
27. In a line of students Madhukar is on 15th position from from the left respectively How many girls are there in the
right and Dhirendra is on 18th position from left. When row ?
they both interchange their positions then Madhukar is on (a) 25 (b) 26
20th position from right. What will be the position of (c) 27 (d) Data inadequate
Dhirendra from left? (e) None of these
(a) 18th (b) 24th 36. Ram and Sham are ranked 13th and 14th respectively is a
(c) 23rd (d) 20th class of 23. What are their ranks from the last respectively?
(e) None of these (a) 10 th : 11th (b) 11 th; 12 th
28. In a class of 45 students, among those students who passed, (c) 11th ; 10 th (d) None of these
Anmol secured 11th position from upwards and 15th from (e) None of these
TIME SEQUENCE, NUMBER & RANKING TEST A-53

ANSWER KEY
1 (c) 5 (b) 9 (a) 13 (b) 17 (a) 21 (a) 25 (a) 29 (b) 33 (c)
2 (d) 6 (e) 10 (d) 14 (d) 18 (c) 22 (d) 26 (d) 30 (b) 34 (d)
3 (c) 7 (b) 11 (b) 15 (b) 19 (c) 23 (b) 27 (c) 31 (c) 35 (b)
4 (e) 8 (b) 12 (b) 16 (a) 20 (c) 24 (c) 28 (b) 32 (c) 36 (c)

Answers &
Explanations
2. (d) 8th 9th 14th supporting premise, what is the assumption and what
Ravi Prabha Arjun is the conclusion.
3. (c) Rajnish > Rajesh, Raman... (i) Step II: If a suggested statement supports any of the
Rajeev > Ramesh > Rajesh ... (ii) three it would be a strengthening statement. If it
Raman > Rajeev ... (iii) contradicts any of the three it would be a weakening
Combining all, we get statement.
Rajnish > Raman > Rajeev > Ramesh > Rajesh In order to decide which statement strengthens (or
4. (e) 12 – 45 × 28 ÷ 7 + 15 weakens) an argument the most, follow an additional
= 12 – 45 × 4 + 15 = 27 – 180 = – 153 step:
5. (b) New arrangement of numbers is as follows: 15698372 Step III: If a suggested statement supports (or
Hence, third number from right end is 3. weakens) the basic assumption of an argument, it
6. (e) We can’t find the proportion of those students out of would be the most strengthening (weakening)
the total students who play only cricket. Similarly, we argument.
can’t find the proportion of those students out of the
Or
total students who play only football. But 5/6th of the
total strength play either cricket only or football only. If a suggested statement provides a very strong proof
7. (b) In the original group of digits ‘7’ is fourth from the (or contradictory proof) in favour of (or in contradiction
right, which is interchanged with ‘9’. The new series is to) the conclusion of the argument, it would be the
2781359046. most strengthening (or weakening) statement.
8. (b) There are (25 – 11– 1 =) 13 boys between Akash and 14. (d) We do not have information regarding the number of
Nikhil. persons between Nikhil and Rehaman.
9. (a) After interchanging the number becomes as follows: 15. (b) The specified digits are 8, 2 and 9. Now, we know a
8 01 6 43 5972 perfect square number does not have 8 and 2 at unit’s
Hence, the fourth digit from the right end is 5. place. Therefore, we can make only two three-digit
10. (d) The correct order of dolls according to descending numbers from it, i.e., 829 and 289. Among these two
order of their heights are: numbers, 289 is a perfect square number, i.e., square of
M> P>N >O 17. Thus, unit’s digit is 7 and ten’s digit is 1.
Therefore, Anvi will purchase the doll M. 16. (a) ------
11. (b) If the first working day happened to be Tuesday then ----A----
8th, 15th, 22nd and 29th of the month will be Tuesday. DEA - - [It is not possible as D is not at the top.]
Hence, the last day of the month will be Wednesday --- AED
(since, number of days in the month is 30). Thus, the
BCAED
next casual leave will be on Thursday.
Hence, C is second among them.
12. (b) (i) Submitted application form : Monday
(ii) Holiday : Tuesday 17. (a) Vijay’s position from downwards
(iii) Clearance from clerk : Wednesday = [ Totalstudents –Vijay's position from upwards ] + 1
(iv) Clearance from senior : Wednesday = [43 – 14] + 1 = 30th
clerk 18. (c) Total students
(v) Submitted to the head clerk : Thursday = [Rakesh’s position from upwards + Rakesh’s position
13. (b) Here (i) weakens the statement whereas (ii) and (iii) from downwards] – 1
support and strengthen the statement. But (iv) and (v) = [9 + 38] – 1 = 46
neither weaken nor strengthen the statement. 19. (c) Sarita’s place from downwards
In order to decide whether a statement strengthens(or
= é Total -
Sarita 's place ù
weakens) an argument, follow these steps. + 1 = [45 – 11] + 1 = 35th
êë girls from upwards úû
Step I: Break up the argument mentally. See what is the
A-54 TIME SEQUENCE, NUMBER & RANKING TEST

20. (c) Raman’s position from upwards 29. (b) After interchanging their positions, position of A from
left = 11
é Raman 's ù then positions of A form right = 9.
= ê Total - position ú + 1
ê students ú \ The total no. of people in the row
from down ú
ëê û = (9 + 11) – 1 = 19.
= [31 – 9] + 1 = 23rd 30. (b)
21. (a) Total boys
N
é ù
ê Mahendra 's Surendra 's ú éê Boys between ùú A B C
=ê place + place ú + 15 3 4 15
from right ú êë them úû
ê from left Clearly, according to the given conditions, there are 15
ë û
boys to the left of A , as well as to the right of C. Also, B
= [17 + 18] + 8 = 43 lies between A and C such that there are 3 boys between
22. (d) Total students A and B and 4 boys between B and C. So, number of
= [First position of Ganesh + Second position of Rajan] boys in the row = (15 + 1 + 3 + 1 + 4 + 1 + 15) = 40.
–1 31. (c) Let the number of boys be x.
= [12 + 20] – 1 = 31 Then, number of girls = 2x.
23. (b) Number of persons between Vijay and Jack \ x + 2x = 60 or 3x = 60 or x = 20.
= 48 – (14 + 17) = 17
So, number of boys = 20 and number of girls = 40.
Now, Mary lies in middle of these 17 persons i.e., at the
Number of students behind Kamal in rank (60 – 17)
eighth position.
=43.
So, number of persons between Vijay and Mary = 7.
Number of girls ahead of Kamal in rank = 9.
24. (c) Total boys
Number of girls behind Kamal in rank = (40 – 9) = 31
= [Malay’s place from starting + Malay’s place from
end] –1 \ Number of boys behind Kamal in rank
= [13 + 17] – 1 = 29 = (43 – 31) = 12.
Number of passed students 32. (c) Sumit is 17 th from the last and Ravi is 7 ranks ahead of
= [Malay’s place from starting + Malay’s place from sumit. So, Ravi is 24 th from the last.
end] –1 Number of students ahead of Ravi in rank
= [8 + 13] – 1= 20 = (39 – 24) = 15.
\ Number of failed students = 29 – 20 = 9 So, Ravi is 16th from the start.
25. (a) Position of Suman from right 33. (c) A is 18th from front and C is 24th
Number of persons between A and C = 6.
é Difference of First position ù
= ê Ramesh 's position + ú Since C is exactly in middle of A and B, so number of
of Suman ú persons between C and B = 6.
ëê û
17 6 6 15
= [(15 – 9) + 6 ] = 12th
26. (d) After exchanging positions, Bhusan becomes fifteenth A C B
instead of seventh from the left, it means there are 7 \ Number of persons in the queue
students between them. So Motilal’s position from the = ( 17 + 1 + 6 + 1 + 6 + 1 + 15 ) = 47.
right will become twelfth. [i.e., (15 – 7) + 4 = 12] 34. (d) The change of place by Monika can be shown as under.
27. (c) Second place of Dhirendra from left 1 2 3 4 5 6 7 8 9 10 11 M 13 14 15 16 17 18 19 20 21

é ù
ê Difference of First place ú Clearly, Monika's earlier position was 8th from the left
= ê places of + ú and 14th from the right end.
ê Madhukar of Dhirendra ú 35. (b) Since Rita and Monika exchange places, so Rita's new
êë úû
position is the same as Monika's earlier positions .
= [(20 – 15) + 18] = 23rd This position is 17th from the right and 10th from the
left
28. (b) Failed Students
\ Number of girls in the row = (16 + 1 + 9) = 26.
= [Total students] – [(Anmol’s position from upwards)
+ (Anmol’s position from downwards) – 1] 36. (c) Rank of Ram from the last = 23 – 13 + 1 = 11
= 45 – [(11 + 15) – 1] = 20 and Rank of Shyam from the last = 23 – 14 + 1 = 10
Problem
7 Chapter
In this chapter you will see some typical problems in which you
Solving
L M N O P Q
would be given a series of interlinked information and on the
Historical × × × ×
basis of those informations you would be expected to reach certain
place
conclusions. Such questions are the essential part of examinations
like Bank PO, AAO (LIC & GIC), MBA entrance, etc. Industrial ×
TYPES OF INFORMATIONS IN A GIVEN PROBLEM city
1. Basic informations (Usefull secondary informations): It is Hill station ×
given in 1 st couple of sentences in the given data. For As above table gives definite informations about L, O. L is
example. neither a historical place nor a hill station. So, it must be an
There are six cities L, M, N, O, P and Q. (in example 1) industrial city. In the same manner O is neither a historical
(i) L is not a hill station nor an industrial city. So, O must be a hill station. Hence,
(ii) M and P are not historical places we put ‘P’ mark at the appropriate place which give the
(iii) O is not an industrial city table following look:-
(iv) L and O are not historical cities
(v) L and M are not alike. L M N O P Q
2. Actual informations: Whatever remains after the basic Historical × × × ×
informations are known as actual information. place
For example (i), (ii), (iii), (iv) and (v) are actual information. Industrial P ×
(in example 1) city
3. Negative informations: Actual informations having Hill station × P
negative sentences are called negative information. A
negative information does not inform us anything exactly Now, as per the condition (V) (L and M are not alike), M can
but it gives a chance to eliminate a possibility. not be an Industrial city. Also M is not a historical place
For example, A is not the brother C. either. Therefore, it is very obvious that M is a hill station.
In example, 1 all the actual informations (i), (ii), (iii), (iv) and (v) are Again, in the given problem there is no negative
negative sentences and hence they come under the category of information about N. Hence, we can assume that N is a hill
negative informations. station as well as a historical place and an industrial city.
Types of Problems Combining if these aspects, the following table will be
1. Simple problems (based on categorisation) prepared finally.
2. Problems based on arrangement.
L M N O P Q
3 Problems based on comparison.
Historical × × P × × P
4. Problems based on blood relations.
place
5. Blood relations and profession based problems.
6. Problems based on conditional selection. Industrial P × P × P P
7. Miscellaneous problems. city
Now, we will discuss all the types of problems one by one Hill × P P P P P
1. Simple Problems (Based on Categorisation) station
Ex. 1 (Problem format) is such type of problem and it can be Now, after analysing the given questions we get the
solved by preparing a table in the manner given below. following answer:-
L M N O P Q Q. (1) b Q. (2) c Q. (3) c Q. (4) a
Historical 2. Problems Based On Arrangement
place In such problems a group of people, objects, etc, may have
Industrial to be is arranged in a row or in a circle or any other way. Let
city us see the example given below:-
Hill station 1. Directions (questions 1 to 5): Just read the
EXAMPLE
(i), (ii), (iii), (iv) are negative informations. Therefore as per following informations carefully to answer the questions
such informations. We put ‘X’ (not) mark wherever given below it:
applicable. As a result the table looks like the one below.
A-56 PROBLEM SOLVING
Five friends P, Q, R, S, and T are sitting on a bench. Now, look at the given questions and check that you get
(i) P is sitting next to Q. the following answer:-
(ii) R is sitting next to S. Q. (1) e Q. (2) a Q. (3) d
(iii) S is not sitting with T. Q. (4) b Q. (5) d
(iv) T is on the last end of the bench. 3. Problems Based On Comparision
(v) R is on the 2nd position from the right. In such problems comparision of different objects or
(vi) P is on the right of Q and T. persons has to be made. Such comparisions are done on
(vii) P and R are sitting together. the basis of marks, ages heights, etc.
1. Where is A sitting? Let us see the following examples:-
(a) Between S and R (b) Between S and R EXAMPLE 2. Directions:- Read the informations given below
(c) Between T and S (d) Between S and T to answer the given questions:
(e) Between Q and R (i) 7 students A, B, C, D, E, F and G take a series of tests.
2. Who is sitting in the centre? (ii) No two students obtain the same marks.
(a) P (b) Q (c) R (iii) G always scores more than A.
(d) S (e) T (iv) A always scores more than B.
3. R is sitting between............... (v) Each time either C scores the highest and E gets the
(a) Q and S (b) P and T
least, or alternatively D scores the highest and F or B
(c) S and T (d) P and S
scores the least.
(e) P and Q
4. What is the position of S? Questions:
(a) Extreme left (b) Extreme right 1. If D is ranked 6th and B is ranked 5th, which of the following
(c) Third from left (d) Second from left can be true?
(e) None of these (a) G is ranked 1st or 4th (b) C is ranked 2nd or 3rd
(c) A is ranked 2 or 5nd th (d) F is ranked 3rd or 4th
5. What is the position of Q?
(a) 2nd from right (b) Centre th
(e) E is ranked 4 or 5 . th
(c) Extreme left (d) 2nd from left 2. If C gets most, G should be ranked not lower than -----
(e) None of these (a) 2nd (b) 3rd (c) 4th
Now, point to be noted that in arrangement problems the (d) 5 th (e) 6 . th
actual information can be classified into 2 categories:-
3. If C is ranked 2nd and Q is ranked 5th, which of the following
(a) Definite information: -
must be true?
A definite information is one when the place of object/man
is definitely mentioned. (a) D is ranked 3rd (b) E is ranked 6th
(b) Comparative information:- (c) A is ranked 6th (d) G is ranked 4th
(e) F is ranked 6 . th
In such information the place of object/man is not mentioned
definitely but only a comparative position is given. In other 4. If D is ranked 2nd, which of the following can be true?
words the positions of objects/men are given in comparision (a) F gets more than G (b) G gets more than D
to another objects/men. (c) A gets more than C (d) A gets more than G
Now, to solve the problem go as per the following steps:- (e) E gets more than B
Step I Sketch a diagram of empty places 5. If G is ranked 5th, which of the following must be true?
Step II. Fill up as many empty places as possible using
(a) D scores the highest (b) C is ranked 2nd
all the definite informations.
(c) E is ranked 3rd (d) B is ranked 4th
Step III. With the help of comparative information consider
all possibilities and select the possibilities which (e) F scores the least
does not violate any condition. Method to Solve
Now, we can solve the above example : If you give a serious look to the problem you will find that such
Here 4th and 5th sentences constitute definite information: problems are as same as the arrangement problems. Therefore,
Comparative informations are: 1st, 2nd, 6th and 7th sentences we have to go like arrangement problem while solving problems
while 3rd is a negative information. based on comparision.
Now, start with definite information, sketch the following Solution (Ex. 2)
arrangement:- In this case, we see there is no definite information. Sentence 5
T __ __ R __ gives a definite information but it is conditional. Still, we draw all
Now, this is the time to look for the comparative informations the possibilities based on sentence 5.
that tell about T and R. Such informations are 2nd, 6th and (1) C __ __ __ __ __ __ E
7th sentences. Take the 7th and the 1st sentence. If P and R
or, (2) D __ __ __ __ __ __ F
are together and also Q and P are together, then P must be
between Q and R. Now the arrangement take the form as:- or, (3) D __ __ __ __ __ __ B
T Q P R ____ We see that the two additional informations (iii) and (iv) are
By the virtue of the 2nd sentence: inadequate to reach a definite conclusion. Hence, keeping these
TQP RS in mind. We move on to the given questions.
PROBLEM SOLVING A-57
1. D is ranked 6th and B is 5th. This does mean that possibilities (d) Can’t be determined (e) None of these
(2) and (3) are violated. Hence, possibility (i) must be true. To solve such questions, remember the following point:-
Thus, we have: Draw a family tree using
C __ __ __ B D E (i) Vertical/diagonal lines to represent parent-child
Also by virtue of (iii) and (iv) we can have only one relationships
arrangement for G, A and B which is GAB. Accordingly, (ii) Single/double horizontal line like ( « / Û ) to
there are two possibilities:
F G AF B D E represent marriages
or, C F GAB D E (iii) a dashed lin e (—) for brother an d sister
relationship
\ Correct answer: d.
(iv) ‘+’ sign for male and ‘–’ sign for female
2. Just see the analysis of Q (1)
For example.
\ Correct answer: c.
3. G is ranked 2nd does mean possibility (1) is false. Therefore, A
+
B

possibilities (2) and (3) remain. Now, B is ranked 5th does


mean possibility (3) is false.
Hence, possibility (2) remains:
+ –
D C __ __ B __ F D E
Now, by virtue of (iii) and (iv), we must have G and A before
B in that order. Consequently the 6th place would go to the
only letter remaining that is E. G
– +
F
Hence, D C G A B E F or
\ Correct answer: b.
4. D is ranked 2nd does mean possibilities (2) and (3) are false. A
+ –
B
Hence possibility (1) is true. Now look at the analysis of
Question (1) and you will get the correct answer as: a.
5. If G is ranked 5th, we can not definitely say which among
the three possibilities (i), (2) and (3) are true or false. But
+ –
sentences (iii) and (iv) definitely imply that the position of D E
A will be 6th and that of B seventh. Now if B is 7th, it does
mean that possibility (3) is true. Hence, we have G
– +
F
D ? ? ? G A B.
\ Correct answer: a. or
4. Problems Based On Blood Relation + –
A B
Such problems involves analysis of certain blood relations.
Let us see the problems given below:-
EXAMPLE 3. (Directions): Read the following information
carefully and answer the questions given below: + –
There are 6 members in a family. They are M, N, O, P, Q, R D E
are travelling together. N is the son of O but O is not the

mother of N. M and O are a married couple. Q is the brother G F+
O. P is the daughter of M. R is the brother of N.
1. How many make members are there in the family? The above diagrams tells us:-
(a) A and B are couple; A is the husband while B is the
(a) 1 (b) 3 (c) 2
wife.
(d) 4 (e) 5
(b) D is son of A and B while E is daughter of A and B.
2. Who is the mother of N? (c) D is the brother of E and E is the sister of D.
(a) P (b) R (c) Q (d) D has a son F
(d) M (e) None of these (e) F and G are couple; F is the husband and G is the wife.
3. How many children does M have? (f) F is the grandson of A and B.
(a) 1 (b) 2 (c) 3 (g) G is the daughter in law of D.
(d) 4 (e) None of these (h) E is the aunt (Bua) of F
4. Who is the wife of Q? (i) There are 3 males (A, D and F) and 3 females (B, E, G)
(a) M (b) R (c) N Now that you have learnt how to make a family tree. Let us
(d) Can’t be determined (e) None of these see the actual method of solving the problem.
5. Which of the following is a pair of females? Solution to example: 3
(a) MQ (b) NP (c) PR Here all the sentences are actual information except the first
(d) MP (e) None of these out of these the 2nd and the fifth sentences give information
6. How is Q related to P? on parent child relationship. We can begin with either of
(a) Father (b) Brother the two. Let us begin with the 6th sentence. Our diagram
(c) Uncle will be as
A-58 PROBLEM SOLVING
M 3. Which of the following is definitely a group of male
(?) members?
(a) B, P, E (b) P, E
(c) B, P, A (d) B, P
(–) (e) None of these
P 4. Who is the sister of E?
As we do wet want to make many diagram and instead we (a) C (b) D
would prefer to only add to the existing diagrams. Therefore (c) A (d) Data inadequate
we should look for sentences that talk of M or P. The 3rd (e) None of these
sentence talks about M. Hence we add this information, 5. What is the profession of A?
that M and O are married couple in our diagram. (a) Housewife (b) Engineer
(c) Teacher (d) Engineer or Teacher
M O (e) Housewife or Teacher
(?) (?) Solution to Ex. 4
Here, (i), (ii), and (iv) are useful secondary informations.
(–) While (iii), (iv) and (v) are the actual informations. We start
P
with the 3rd sentence because it mentions a parent. Child
Now the 2nd sentence talks about O. It says that N is the relationship its diagram can be made as the following:-
son of O but O is not the mother of N. Obviously, O must be B
the father of N. This means O is a male and hence M must (+, Eng)
be a female. Now our diagram takes the form as following:-
(–)
M (+)
O B is an Engineer
and father of E

(–)
P N
(+)
E
(? ?)
Now, we add the two sentences ‘Q is the brother of O’ and Now, we move on to another sentence that involves either
‘R is the brother of N’ and we get the final diagram as below:- B or E. You see that the 5th sentence gives some information
(–)
M O
(+) (+)
Q about E. It says that D is the grandmother E. Point to be
noted that if D is the grandmother of E, then the son of D
must be father of E and hence B is the son of D. Now, the
diagram takes the following form.
(–)
P R
(+)
N
(+)
D
Now, you can read the questions to check your answer:- (–, Housewife)
Q. (1) d Q. (2) d Q. (3) c Q. (4) d
Q. (5) d Q. (6) c
5. Problems Based On Blood Relations And Profession:
Such problems are very much similar to the problems related B D is a housewife and
to blood relation. What makes it different is the addition of (+, Eng) grandmother E
new data:- the professions of family members. You will get
the more clear idea about this type of problem. Let us see
the example given below:-
EXAMPLE 4. Directions: Read the following information E
(?, ?)
carefully and answer the questions given below it: Now, the 4th sentence has the remaining information and
(i) A, B, C, D, E and P are members of a family. diagram for it is given below:-
(ii) There are two married couples. P
(iii) B is an engineer and the father of E (+, Lawyer)
(iv) P is the grandfather of C and is a lawyer.
(v) D is the grandmother of E and is a housewife. P is a lawyer and
(vi) There is one engineer, one lawyer, one teacher, one (?) grandfather of C
housewife and two students in the family.
1. Who is the husband of A?
(a) C (b) P (c) B C
(d) D (e) E (? ?)
2. Which of the following are two married couple? Now, we see that we have ended up with two different
(a) PD, BA (b) PD, BE component. Then how to resolve this deadlock? The answer
(c) PD, CA (d) ED, CP is simple: - to resolve it we make used of the given useful
(e) None of these secondary information (USI).
PROBLEM SOLVING A-59
“There are two married couple in the family.” Clearly, the 4. If 4 members including N, have to be boys, the members
two possible pairs are of grandfather, grandmother and other than N are ------
father, mother. Therefore, we combine the two diagrams into (a) JKLQR (b) JMOST
the following way. (c) KLOQR (d) JLMOQ
(e) None of these.
P D 5. If 4 members have to girls, the members of the team are ----
(+, Lawyer) (–, Housewife) (a) KLPQRS (b) KOPRST
(c) KLQRST (d) KLPQRT
(e) None of these
Solution to Ex- 5
Solving problems like example 6 is very easy. Make the
B A
(– ?) group of all the pairs that have to be together on one side
(+, Eng)
and the pairs that must not be together on the other side.
Next, read each of the questions and treat that as an
additional information. Finally analyse the possibilities and
E choose the possibilities that satisfies all the conditions. Let
C
(? ?) (? ?) us see the process below:-
1stly, we can summarise the conditions in the following
Point to be noted that the professions of A, E and C are yet way:-
unknown. However, with reasonable justification, we may
assume that the mother (A) should be the teacher and the J, M S, T
two children E and C should be students. But this conclusion ( +) (+ ) (- ) (-)
can be challenged and has no reason at all.
K, R L, Q ® Group ' must be together '.
Apart from that the sexes of E and C can not be determined.
Now, read the question and check your answer one by one:- ( + ) ( - 1) ( + ) ( - )
Q. (1) c Q. (2) a Q. (3) d Q. (4) d
Q. (5) c
L, S, K, N, M, P
6. Problems Based On Conditional Selection: (+) (–) (+) (+) (+) (– 1) Group never
In this type of problems, a group of objects/persons has to be together’
be selected from a given larger group, as per the given Now we move on to questions one by one.
restrictions. You will get the better idea of such type of 1. Here, number of boys are 5. We see than K and N can never
problem from the problem given below:- be together. Therefore, there are only two ways of selecting
5. Directions:- Study the following information 5 boys:- JKLMO and JNLMO. But the possiblity is not
EXAMPLE
possible because if K would go then R should also go, and
carefully and answer the questions given below:- if L goes than Q should also go. Hence, JNLMO is the only
From, amongst 6 boys J, K, L, M, N, and O and 5 girls P, Q, possibility in which L’s friend Q would be the lone girl
R, S and T, a team of 6 is to be selected under the following member.
conditions:- \ Correct answer choice is (b).
(i) J and M have to be together. 2. There are three girls including P. P is there, so M must not
(ii) L can not go with S. be there. If M is not there, J would not be there. So two
(iii) S and T have to be together. boys J and M are eliminated. Since, the team should have
only 6 members, hence there should be three boys. Two
(iv) K can not be teamed with N.
boys J and M are eliminated. Therefore, the possibilities of
(v) M cannot go with P. selecting three boys are :- KLN, KLO, KNO, LNO. But K
(vi) K and R have to be together. and N can’t be together. Hence the remaining possibilities
(vii) L and Q have to be together. are KLO and LNO. Now, K must be with R and L must be
1. If there be 5 boys in the team, the lone girl member is ------ with Q. Therefore, we have PKRLQO and PLQNO. To the
(a) P (b) Q (c) R 2nd possibility we need to add a girl. We can’t add R since R
can’t go without K. We can’t add T since T can’t go
(d) S (e) None of these
withouts. Conversely, we can’t add S either. Hence, this
2. If including P, the team has three girls, the members other possibility is also eliminated. This, the only possible choice
than P are ------ remains PKRLQO.
(a) K L O P Q (b) JMNST \ correct answer choice is (a).
(c) JMKST (d) KORST Quicker method:
(e) None of these Start with the answer choices. Choice (b) and choice (c)
3. If, the team including L consists of 4 boys the members of have M in them. M can’t go as P is there. Choice (d) is not
the team other than L are ------ correct as it has more than three girls including P. Hence,
the correct answer choice must be either (a) or (c). But on
(a) JMNPQ (b) JKMQR
verifying we see that a is indeed the correct choice as it
(c) MNOJQ (d) KNORQ does not violate any restriction.
(e) None of these \ Correct answer choice is (a).
A-60 PROBLEM SOLVING
3. There are 4 boys including L. So there must be two girls. Quicker method
Now if L is present, S can’t go and if S can’t go, T won’t go. Choice (a) is incorrect as it has M and P together. Choice (c)
Hence, three girls remain:- P; Q and R out of these, two can is incorrect as it has only one girl. Choice (d) is incorrect as
be selected in the ways given below:- it has K and N together. Hence, two choices (b) and (e)
PQ, PR, and QR. remain. On verifying we see that (b) is the correct answer
Now, if P is selected, M can’t go and if M can’t go, J will not choice.
go. In such case the team would have to include K and N as 4. Inclusion of N Þ Exclusion of k Þ Exclusion R. Four boys
4 boys hence to be selected. But K and N can’t be together. does mean there should be two girls. How do you select 2
This means that P should not be selected. Therefore, the girls out of P, Q, S and T if S and T have always to be
only possibility of selecting two girls is QR. But R means together? The only two possible way are:- P, Q, and S, T. If
the necessary inclusion of K, which in turn means we choose P we can’t select M, and hence we can’t select J
necessary inclusion of N. Hence, the possible combination either. This means the exclusion of J and M in addition to
is LKQR. To this we should add two boys out of J, M and that of K. Since, this is not possible in order to have four
O. The only possibility is adding J and M as neither of boys, we must not select P. Hence, we select S.T. Now,
these would go without the other. Hence, the team is selecting S means excluding L. Hence, K and L are excluded.
JMLKQR. The team would be: - JMNOST
\ Correct answer choice is (b). \ Correct answer choice is (b).

EXERCISE
Directions (Qs. 1-5): Study the following information carefully Directions (Qs. 6-10): Study the following information carefully
and answer the questions given below: and answer the questions given below:
A, B, C, D, E, F and G are seven persons who travel to office P, Q, R, S, T, V and W are travelling in three different vehicles.
everyday by a particular train which stops at five stations I, II, III, There are at least two passengers in each vehicle– I, II & III and
IV and V respectively after it leaves base station. only one of them is a male. There are two engineers, two doctors
(1) Three among them get in the train at the base station. and three teachers among them.
(2) D gets down at the next station at which F gets down. (i) R is a lady doctor and she does not travel with the pair of
(3) B does not get down either with A or E. sisters, P and V.
(4) G alone gets in at station III and gets down with C after (ii) Q, a male engineer, travels with only W, a teacher in vehicle
having passed one station. I.
(5) A travels between only two stations and gets down at (iii) S is a male doctor.
station V. (iv) Two persons belonging to the same profession do not travel
(6) None of them gets in at station II. in the same vehicle.
(7) C gets in with F but does not get in with either B or D. (v) P is no an engineer and travels in vehicle II.
(8) E gets in with two others and gets down alone after D. 6. What is V ’ s profession?
(9) B and D work in the same office and they get down together (a) Engineer (b) Teacher
at station III. (c) Doctor (d) Data inadequate
(10) None of them gets down at station I. (e) None of these
1. At which station does E get down? 7. In which vehicle does R travel?
(a) # II (b) # III (a) I (b) II
(c) # IV (d) Data inadequate (c) III (d) II or III
(e) None of these (e) None of these
2. At which station do C and F get in? 8. Which of the following represents the three teachers?
(a) # I (b) # II (a) WTV (b) WTP
(c) # III (d) Data inadequate (c) WTV or WTP (d) Data inadequate
(e) None of these (e) None of these
3. At which of the following stations do B and D get in? 9. Which of the following is not correct?
(a) # I (b) Base station (a) T-Male-Teacher (b) Q-Male-Engineer
(c) # III (d) Data inadequate (c) P-Female-Teacher (d) V-Female-Teacher
(e) None of these (e) W-Female-Teacher
4. After how many stations does E get down? 10. How many lady members are there among them?
(a) One (b) Two (a) Three (b) Four
(c) Three (d) Four (c) Three or Four (d) Data inadequate
(e) Five (e) None of these
5. E gets down after how many stations at which F gets down? Directions (Qs. 11-15): Study the following information and
(a) Next station (b) Two answer these questions :
(c) Three (d) Four (A) P, Q, R, S, T, U and V are sitting in a circle facing the
(e) None of these centre.
PROBLEM SOLVING A-61
(B) S, who is second to the right of R, is not to the immediate 19. How many boys are there in the group?
right of V. (a) Two (b) Three
(C) U is not between V and T. (c) Four (d) Data inadequate
(D) P is between R and Q. (e) None of these
11. Which of the following is wrong? 20. Which of the following combinations of student-subject is
(I) T is to the immediate left of R. correct?
(II) Q is to the immediate left of U. (a) D-Mathematics (b) F-English
(III) U, S and T are in a sequence, one after the other. (c) A-Sanskrit (d) B-Geography
(a) Only I (b) Only II (e) None of these
(c) Only III (d) Only I and II Directions (Qs. 21 -25): Study the following information carefully
(e) All I, II and III and answer the questions given below:
12. Which of the following are the two pairs of adjacent is There are seven students — P, Q, R, S, T, V and W — in a class.
members? Each of them has a different favourite subject, viz Science,
(a) VS and TR (b) SU and PQ Chemistry, Biology, French, English, Mathematics and Hindi. Each
(c) PR and TQ (d) VU and QR of the students secured different marks in his favourite subject. R
(e) None of these has secured second highest marks and neither Science nor French
13. What is the position of T? is his favourite subject. T secured the least marks and neither
(a) To the immediate left of R Mathematics nor Chemistry is his favourite subject. The favourite
(b) Second to the left of P subject of S is Biology and he secured more marks than marks of
(c) Fourth to the left of U Q and W but less than the marks of P. The favourite subject of Q
(d) Second to the left of V is English and his marks is more than the marks of T but less than
(e) None of these the marks of W.
14. Which of the following is correct? The favourite subject of P is Hindi and he has not secured the
(I) V is third to the left of R. highest marks. The student whose favourite subject is French,
(II) U is between S and V got the highest marks. The favourite subject of R is not Chemistry.
(III) Q is to the immediate left of P. 21. Which is the favourite subject of T?
(a) Only I (b) Only II (a) Chemistry (b) Science
(c) Only III (d) Only II and III (c) English (d) Data inadequate
(e) None of these (e) None of these
15. If Q and R interchange places so as T and V, then 22. What is the favourite subject of W?
(a) S is third to the right of R (a) Chemistry (b) Mathematics
(b) T is second to the left of R (c) Either Chemistry or Mathematics
(c) Q is fourth to the right of T (d) Data inadequate
(d) V is third to the right of U (e) None of these
(e) None of these 23. Who got the second lowest marks?
Directions (Qs. 16-20): Study the following information and (a) W (b) S
answer these questions: (c) R (d) Data inadequate
A, B, C, D, E, F and G are students of a class. Each of them has a (e) None of these
different favourite subject, viz. History, Mathematics, Geography, 24. French is the favourite subject of
Civics, English, Sanskrit and Marathi but not necessarily in the (a) R (b) P
same order. There are two such students whose one brother each (c) W (d) V
is there in the group. There is no other relation among the (e) None of these
students. No girl likes Mathematics or Sanskrit. D, who does not 25. When all the seven students are arranged according to
like Civics and English, is the sister of that student who likes their marks in their favourite subjects in descending order,
Marathi. The student who likes Civics is the brother of that girl what is the position of P from the top?
student who likes History. F is a girl student. B is brother of A. (a) Second (b) Fourth
16. Which of the following is a pair of brother - sister other (c) Third (d) Data inadequate
than A and B? (e) None of these
(a) D and G (b) D and C Directions (Qs. 26- 30): Study the following information carefully
(c) G and F (d) Data inadequate and answer the questions given below:
(e) None of these A, B, C, D, E, F and G are seven members of a family belonging to
17. Which of the following is true? three generations. There are two married couples—one each of
(a) D likes History. first and second generations respectively. They travel in three
(b) C, B and D are girl students. different cars P, Q and R so that no car has more than three
(c) E and F are boy students. members and there is at least one female in each car. C, who is a
(d) The number of girls is more than that of boys in the grand daughter, does not travel with her grandfather and grand
group. mother. B travels with his father E in car Q. F travels with her
(e) None of these grand daughter D in car P. A travels with her daughter in car R.
18. Who likes Sanskrit? 26. How many female members are there in the family?
(a) Only C (b) Only E (a) Three (b) Four
(c) Only B (d) Either C or E (c) Five (d) Data inadequate
(e) None of these (e) None of these
A-62 PROBLEM SOLVING
27. Which of the following is one of the married couples? 35. Which of the following is not the correct combination of
(a) DB (b) BC student and subject?
(c) EF (d) Data inadequate (a) P-metallurgy (b) Q-electrical
(e) None of these (c) U-electronics (d) S-civil
28. In which car are three members travelling? (e) All are correct
(a) P (b) Q 36. Which student is from Chennai?
(c) Either P or Q (d) R (a) R (b) U
(e) None of these (c) S (d) T
29. How is D related to E? (e) None of these
(a) Daughter (b) Niece 37. P is from which city?
(c) Grand daughter (d) Data inadequate (a) Chennai (b) Calcutta
(e) None of these (c) Hyderabad (d) Data inadequate
30. How is G related to A? (e) None of these
(a) Daughter (b) Sister
38. Which student is from Bangalore‘?
(c) Mother-in-law (d) Data inadequate
(a) T (b) Q
(e) None of these
(c) S (d) T or P
Directions (Qs. 31-34): Study the following information carefully
(e) None of these
and answer the questions given below:
39. R is studying which subject?
In a building there are thirteen flats on three floors — II, III and IV.
Five flats are unoccupied. Three managers, two teachers, two (a) Electronics (b) Mechanical
lawyers and one doctor occupy the remaining flats. There are at (c) Metallurgy (c) Data inadequate
least three flats on any floor and not more than six flats on any (e) None of these
floor. No two persons of the same profession stay on any floor. Directions (Qs. 40-44): Study the following information carefully
On the second floor, out of four flats, one occupant is the lawyer and answer the questions given below:
and has only one neighbour. One teacher lives one floor below (a) Seven persons A, B, C, D, E, F, and G are teaching seven
the other teacher. The doctor is not the neighbour of any of the subjects History, Geography, Physics, Chemistry, Maths,
lawyers. No flat is unoccupied on the third floor. Biology and English from Monday to Friday. Each person
31. How many flats are there on the third floor? teaches a different subject and not more than two subjects
(a) Three or Four (b) Four are taught on any one of the days.
(c) Five (d) Three (b) Chemistry is taught by B on Tuesday.
(e) None of these (c) D teaches on Friday but neither Geography nor Physics.
32. What is the combination of occupants on the second floor? (d) F teaches History but neither on Thursday nor on Friday.
(a) Lawyer, Manager (b) Teacher, Doctor (e) A teaches English on the day on which History is taught.
(c) Manager, Doctor (d) Manager, Teacher (f) C teaches Maths on Monday.
(e) None of these (g) Geography and Chemistry are taught on the same day.
33. Who among the following is the neighbour of the other (h) G teaches on Thursday.
lawyer? 40. English is taught on which day?
(a) Manager (a) Wednesday (b) Monday
(b) Teacher (c) Tuesday (d) Data inadequate
(c) Both the manager and the teacher (e) None of these
(d) Data inadequate 41. Which of the following subjects is taught by ‘G’?
(e) None of these (a) Biology (b) Geography
34. How many flats are occupied on the fourth floor? (c) Physics (d) Chemistry
(a) Two (b) Three (e) None of these
(c) Four (d) Data inadequate 42. Geography is taught on which day?
(e) None of these (a) Monday (b) Tuesday
Directions (Qs. 35-39): Study the following information carefully (c) Wednesday (d) Thursday
and answer the questions given below: (e) Friday
(i) Six students P, Q, R, S, T and U are in different branches of 43. Which subject is taught on Friday?
Engineering, viz. civil, mechanical M chemical, electrical, (a) Physics (b) History
metallurgy and electronics but not necessarily in the same (c) Geography (d) Biology
order. (e) None of these
(ii) Each of them is a resident of a different city viz Mumbai, 44. Which of the following pairs of persons teaches on
Calcutta, Chennai, Delhi, Hyderabad and Bangalore. R is Tuesday?
the resident of Delhi but he is not in chemical or electrical. T, (a) B and D (b) A and B
who is in mechanical, is not the resident of Mumbai or (c) B and F (d) B and C
Hyderabad. Q is from Calcutta and he is in electrical. The (e) None of these
student from Chennai is in electronics and S is from Mumbai. Directions (Qs. 45-47): Study the following information carefully
P is in metallurgy. and answer the questions given below:
PROBLEM SOLVING A-63
Five friends Yash, Neeraj, Mehul, Ram and Prakash are students Directions (Qs. 53-57) : Study the following information carefully
of five different disciplines Medical, Engineering, Architecture, and answer the questions given below:
Arts, Management and each plays a different musical instrument Seven specialist doctors B, M, K, P, D, F and H visit a polyclinic
Sitar, Tabla, Sarod, Guitar and Violin. on four days –– Tuesday, Wednesday, Friday and Saturday –– in
Mehul, a medical student, does not play Sarod or Sitar nor Guitar. a week. At least one doctor but not more than two doctors visits
Prakash is neither a student of Engineering nor Management. the polyclinic on each of these days. Each of them is specialist in
Ram, who plays tabla, is an Arts student. Neither Prakash nor different fields –– ENT, Orthopaedics, Paediatrics, Neurology,
Yash plays Sarod. Ophthalmology, Radiology and Onchology.
45. Who among the following plays Sarod? (i) P visits on Friday with Radiologist.
(a) Yash (b) Neeraj (ii) The Paediatrician does not visit on Saturday nor with D
(c) Prakash (d) Data inadequate and H.
(e) None of these (iii) The Oncologist F visits alone on Tuesday.
46. The guitarist is a student of which of the following (iv) M visits on Wednesday and he is not Paediatrician.
disciplines? (v) K visits on Wednesday. H is not Radiologist.
(a) Engineering (vi) The Paediatrician visits with the ENT specialist.
(b) Either Engineering or Management (vii) The Neurologist visits on Friday.
(c) Architecture (viii) B is neither Orthopaedician nor Radiologist.
(d) Data inadequate 53. What is the speciality of B?
(e) None of these (a) Ophthalmology (b) ENT
47. Who among the following plays Sitar? (c) Paediatrics (d) Data inadequate
(a) Yash (b) Neeraj (e) None of these
(c) Prakash (d) Data inadequate 54. On which day of the week does D visit?
(e) None of these (a) Wednesday (b) Saturday
Directions (Qs. 48-52): Study the following information carefully (c) Wednesday or Saturday (d) Friday
(e) None of these
and answer the questions given below:
55. Who among them visits the polyclinic along with B ?
(i) M, N, P, Q, S and T are six members of a group in which
(a) None (b) H
there are three female members. Females work in three
(c) D (d) P
departments —Accounts, Administration and Personnel —
(e) Either H or P
and sit on three different floors — Ist, IInd and IIIrd. Persons
56. On which of the following days do the specialists in
working in the same department are not on the same floor.
Orthopaedics and Ophthalmology visit ?
On each floor two persons work.
(a) Wednesday (b) Friday
(i) No two females work in the same department or on the same (c) Saturday (d) Data inadequate
floor. N and S work in the same department but not in (e) None of these
Personnel. Q works in administration. S and M are on the 57. What is P’s profession?
Ist and IIIrd floors respectively and work in the same (a) Paediatriccian (b) ENT
department. Q, a female, does not work on IInd floor. P, a (c) Ophthalmologist (d) Data inadequate
male, work on Ist floor. (e) None of these
48. Which of the following groups of persons are females? Directions (Qs. 58-59) : Read the following information carefully
(a) SQT (b) QMT and answer the questions given below:
(c) QPT (d) Data inadequate (i) Six books on different subjects, viz History, Geography,
(e) None of these English, Hindi, Economics and Psychology, are arranged in
49. Which of the following pairs of persons work in Adminis- a pile not necessarily in the same order. Each book belongs
tration? to different persons whose names are Dinesh, Harish,
(a) QP (b) QN Vishwas, Lalita, Sanjay and Neeta.
(c) SP (d) Data inadequate (ii) English book, which is kept on the top, belongs to Dinesh.
(e) None of these Psychology book, which is kept at the bottom, does not
50. T works in which department? belong to Sanjay or Lalita.
(a) Accounts (b) Administration (iii) Economics book belongs to Harish and is kept imme diately
(c) Personnel (d) Accounts or Personnel after Hindi.
(e) None of these (iv) History book is immediately above Geography and
51. Which of the following pairs works on IInd floor? immediately below Economics. Hindi book belongs to Neeta
(a) PT (b) SM and History book belongs to Lalita.
(c) QN (d) QT 58. Whom does the Psychology book belong to?
(e) None of these (a) Neeta (b) Vishwas
52. If T is transferred to Accounts and S is transferred to (c) Harish (d) Data inadequate
Administration, who is to be transferred to Personnel to (e) None of these
maintain the original distribution of females on each floor? 59. Which book is fourth from the bottom?
(a) P (b) Q (a) History (b) Geography
(c) N (d) Data inadequate (c) Economics (d) Hindi
(e) None of these (e) None of these
A-64 PROBLEM SOLVING
Directions (Qs. 60-63) : Read the following information carefully 67. Which of the following houses is located immediately next
and answer the questions given below : to the yellow house?
(a) An examiantion board has organised examination for ten (a) Red (b) Blue
subjects viz A, B, C, D, E, F, G, H, I and J on six days of the (c) Green (d) Data inadequate
week with a holiday on Sunday, not having more than two (e) None of these
papers on any of the days. 68. What is the colour of the tallest house?
(b) Exam begins on Wednesday with subject F. (a) Red (b) Blue
(c) D is accompained by some other subject but not on (c) Red or blue (d) Data inadequate
Thursday. A and G are on the same day immediately after (e) None of these
holiday. Directions (Q. 69-72): Read the following information carefully
(d) There is only one paper on last day and Saturday. B is and answer the questions given below.
immediately followed by H, which is immediately followed A, B, C, D, E, F and G are seven students in a class. They are
by I. sitting on three benches I, II and III in such way that there is at
(e) C is on Saturday. H is not on the same day as J. least two of them on each bench and there is at least one girl on
60. Examination for which of the following pairs of subjects is each bench. C, a girl student, does not sit with A, E and D. F, a
on Thursday? boy student, sits with only B. A sits with his best friend on bench
(a) HE (b) DB I. G sits on bench III. E is brother of C.
(c) FD (d) Data inadequate 69. How many girl students are there?
(e) None of these (a) 3 (b) 4
61. Examination for which of the following subjects is on the (c) 3 or 4 (d) Data inadequate
next day of D? (e) None of these
(a) B (b) C 70. Who sits with C?
(c) I (d) H (a) B (b) G
(e) None of these (c) D (d) E
62. Examination for which of the following subjects is on the (e) None of these
last day? 71. Which of the following is a group of girls?
(a) B (b) E (a) BAC (b) BFC
(c) J (d) Data inadequate (c) CDF (d) BCD
(e) None of these (e) None of these
63. Examination for subject F is on the same day as which of 72. On which bench do three students sit?
the following subjects? (a) II (b) III
(a) E (b) D (c) I (d) I or II
(c) I (d) B (e) None of these
(e) None of these
Directions (Qs. 64-68): Read the following information carefully Directions (Qs. 73-75): Read the following information carefully
and answer the questions given below: and answer the questions given below:
P, Q, R, S, T and M, six houses of different heights and different Six persons A, B, C, D, E and F took up a job with a firm in a week
colours of red, blue, white, orange, yellow and green, are located from Monday to Saturday. Each of them joined for different posts
on either sides of a road with three on each side. T, the tallest on different days. The post were of – Clerk, Officer, Technician,
house, is exactly opposite the red-coloured house. The shortest Manager, Supervisor, and Sales Executive, though not
house is exactly opposite the green-coloured house. M is the respectively.
orange-coloured house and is located between P and S. R, the F joined as a Manager on the first day. B joined as a Supervisor
yellow-coloured house, is exactly opposite P. Q, the green- but neither on Wednesday nor Friday. D joined as a Technician
coloured house. is exactly opposite M. P, the white-coloured on Thursday. Officer joined the firm on Wednesday. E joined as a
house, is taller than house R but shorter than houses S and Q. Clerk on Tuesday. A joined as a Sales Executive.
64. What is the colour of house S? 73. Who joined the firm on Wednesday?
(a) Red (b) Blue (a) B (b) C
(c) Red or blue (d) Data inadequate (c) B or C (d) Data inadequate
(e) None of these (e) None of these
65. What is the position of house P from top when the houses 74. Who was the last person to join the firm?
are arranged in descending order of their heights? (a) E (b) F
(a) Third (b) Second (c) A (d) B
(c) Fourth (d) Data inadequate (e) None of these
(e) None of these 75. On which of the following days did the Sales Executive
66. Which of the following is the second in height? join?
(a) S (b) Q (a) Tuesday (b) Thursday
(c) S or Q (d) Data inadequate (c) Saturday (d) Wednesday
(e) None of these (e) None of these
PROBLEM SOLVING A-65
Directions (Qs. 76-79): Read the following information carefully 82. Which of the lectures is to be organised immediately before
to answer the questions given below: the physics lecture?
(i) There are six different books on different subjects P, Q, R, (a) Maths (b) Zoology
S, T and U. These books are kept one above the other on a (c) Chemistry (d) Data indadquate
shelf. These books belong to six different persons - A, B, C, (e) None of these
D, E and F. It is not necessary that the orders of these Directions (Qs. 83-85) : Read the following in formation carefully
books and persons are the same. and answer the questions given below.
(ii) Only book of subject Q is kept between the books of subject (i) Five students Sujit, Randhir, Neena, Mihir and Vinay
P and T and only book of subject S is kept between books have total five books on subjects Physics, Chemistry, Maths,
of subject P and U. The book of subject R is immediately Biology and English written by authors Gupta, Khanna,
above the book of subject T. Harish, D’Souza and Edwin. Each student has only one
(iii) C’s book is kept on the top. A does not have books on book on one of the five subjects.
subjects T and S. The book on subject P belongs to F. The (ii) Gupta is the author of Physics book, which is not owned by
book on subject U belongs neither to B nor to A. D’s book Vinay or Sujit.
is kept at the bottom. (iii) Mihir owns the book written by Edwin.
76. The book on which of the following subjects belongs to A?
(iv) Neena owns Maths book. Vinay has English book, which is
(a) Q (b) S not written by Khanna. Biology book is written by D’Souza.
(c) P (d) T
83. Which of the following is the correct combination of subject,
(e) None of these student and author?
77. Who among the following possesses the book on subject (a) Maths-Neena-Harish (b) Physics-Mihir-Gupta
T?
(c) English-Vinay-Edwin (d) Biology-Sujit-D'Souza
(a) B (b) E
(e) None of these
(c) C or E (d) B or E
84. Who is the author of Chemistry book?
(e) None of these
78. Who owns the book on subject U? (a) Harish only (b) Edwin only
(a) B (b) E (c) Khanna or Harish (d) Edwin or Khanna
(c) D (d) C (e) Data inadequate
(e) None of these 85. Who is the owner of the book written by Harish?
79. The book on which of the following subjects is kept on the (a) Vinay (b) Sujit
top? (c) Randhir (d) Data inadequate
(a) T (b) R (e) None of these
(c) U (d) Data inadequate Directions (Qs. 86-89) : Read the following information carefully
(e) None of these to answer these questions.
Directions (Qs. 80-82): Read the following information carefully (i) In a family of six members A, B, C, D, E and F each one plays
and answer the questions given below: one game out of the six games Chess, Carrom, Table tennis,
(a) Maths, Physics, Chemistry, Botany, Zoology and Statistics Badminton, Bridge and Cricket.
are six subjects on which a series of lectures are to be (ii) Two are married couples.
organised on a day, though their order is not necessarily the (iii) B, who plays Carrom, is daughter-in-law of E.
same. (iv) A is father of D, the Table-tennis player, and D is father of C,
(b) The lectures on Zoology and Chemistry are to be organised who plays Cricket
either in the beginning or at the end: (v) F is brother of C.
(c) The lecture on Physics is to be organised immediately before (vi) Chess is not played by a female member.
that on Botany. The lecture on Statistics is to be organised (vii) E’s husband plays Badminton.
immediately after that on Botany: 86. Who among them plays Bridge?
(d) There will be a small break after the lecture on Physics and (a) E (b) F
each lecture will be of 45 minutes’ duration. (c) A (d) Data inadequate
(e) There will be only two lectures before Physics and the lecture
(e) None of these
on Chemistry is to be organised immediately before that on
87. How is F related to A?
Maths.
(a) Granddaughter (b) Grandson
80. In the series of lectures, the lecture on which of the following
subjects is to be organised immediately before the last lecture? (c) Son (d) Daughter
(a) Botany (b) Zoology (e) None of these
(c) Either Zoology or Botany 88. Who is husband of B?
(a) Data inadequate (b) A
(d) Data inadequate (e) None of these
(c) C (d) D
81. Which of the above given statements is not necessary to
(e) F
answer the questions?
89. How many male members are there in the family?
(a) V (b) IV
(a) Two only (b) Three only
(c) III (d) II (c) Four only (d) Data inadequate
(e) None of these (e) None of these
A-66 PROBLEM SOLVING
Directions (Qs. 90 - 93): Read the following information carefully 96. In which colour was the picture of Joker printed?
and answer the questions given below : (a) Data inadequate (b) Yellow
(i) There are five types of cards viz. A, B, C, D and E. There are (c) Red (d) Green
three cards of each type. These are to be inserted in (e) None of these
envelopes of three colours- red, yellow and brown. There 97. Picture of palace was printed on which of the following
are five envelopes of each colour. cards?
(ii) B, D and E type cards are to be inserted in red envelopes; A, (a) E (b) F
B and C type cards are to be inserted in yellow envelopes; (c) D (d) Either D or E
and C, D and E type cards are to be inserted in brown (e) None of these
envelopes. Directions (Qs. 98-99): After a cricket series, a panel judged 5
(iii) Two cards each of B and D type are inserted in red players / Pervez, Jatin, Robin, Dinkar and Rahul and gave them
envelopes. ranking for batting and bowling. The ranking was in descending
90. How many cards of E type are inserted in brown envelopes? order. Rahul, who was ranked first in batting, was last in bowling.
(a) Nil (b) One Robin had same ranking in both and was just above Rahul in
(c) Two (d) Three bowling. In batting, Pervez was just above Dinkar but in bowling
(e) Data inadequate he was in the middle after Jatin.
98. Who was ranked first as bowler?
91. Which of the following combinations of the type of cards
(a) Jatin (b) Rahul
and the number of cards is definitely correct in respect of
(c) Robin (d) Data inadequate
yellow-coloured envelopes?
(e) None of these
(a) A-2, B-1, C-2 (b) B-1, C-2, D-2
99. Who was ranked fifth in batting?
(c) A-2, E-1, D-2 (d) A-3, B-1, C-1 (a) Dinkar (b) Jatin
(e) None of these (c) Robin (d) Data inadequate
92. Which of the following combinations of types of cards and (e) None of these
the number of cards and colour of envelope is definitely Directions (Qs. 100 - 103): Read the following information and
correct? answer the given questions :
(a) C-2, D-1, E-2, Brown (b) C-l, D-2, E-2, Brown (i) Six friends Ramesh, Dinesh, Lokesh, Nilesh, Shailesh and
(c) B-2, D-2, A-1, Red (d) A-2, B-2, C-1, Yellow Hitesh work in different companies, namely ‘P’, ‘Q’, ‘R’, ‘S’,
(e) None of these ‘T’ and ‘U’, and each one wears company-sponsored different
93. Which of the following combinations of colour of the coloured tie, i.e. Blue, Green, Pink, Yellow, Purple and Red,
envelope and the number of cards is definitely correct in though not necessarily in the same order.
respect of E type cards? (ii) The one wearing Blue tie works in Company ‘S’ and the one
(a) Red-2, Brown-1 (b) Red-1, Yellow-2 wearing Green tie works in Company ‘P’.
(c) Red-2, Yellow-1 (d) Yellow-1, Brown-2 (iii) Hitesh does not work in Company ‘R’ or ‘T’.
(e) None of these (iv) Ramesh wears Pink tie and works in Company ‘Q’.
Directions (Qs. 94 - 97): Study the following information and (v) Nilesh does not work in Company ‘T’ and purple colour tie
answer the questions given below: is not sponsored by Company ‘R’.
(i) 6 picture cards A, B, C, D, E and F are printed in six different- (vi) Shailesh works in company ‘U’ and neither Nilesh nor
coloured inks / blue, red, green, grey, yellow and brown / Dinesh works in company ‘S’.
and are arranged from left to right (not necessarily in the (vii) Company ‘T’ does not sponsor Purple or Yellow coloured
same order and colour as given). tie and Lokesh works in company P.
(ii) The pictures were of king, princess, queen, palace, joker 100. Which colour tie is sponsored by Company ‘R’?
and prince. (a) It can not be ascertained
(iii) The picture of palace was in blue colour but it was not (b) Blue (c) Green
printed on card D. (d) Pink (e) None of these
(iv) Card ‘A’, which was bearing Queen’s picture printed in 101. Which of the following “colour of tie-company-person”
brown ink, was at the extreme right. combinations is correct?
(v) The picture of princess was neither on card D nor E and (a) Green-R-Nilesh (b) Blue-S-Lokesh
was not printed in either green or yellow ink card ‘C’ had (c) Red-T-Dinesh (d) Yellow-R-Shailesh
picture of King printed in ‘grey’ ink and it was fifth from (e) None of these
right and next to card B having picture of prince. 102. Which of the following is true?
94. If the Princess’s card is between the cards of the palace and (a) Company ‘U’ sponsors Green tie.
prince, then at what number the Joker’s card is placed from (b) Shailesh wears Red tie.
left? (c) Nilesh works in Company ‘T’.
(a) First (b) Fourth (d) Red colour is sponsored by Company ‘T’.
(c) Fifth (d) Second (e) None of these
(e) None of these 103. Which of the following sequence of companies represents
95. Which of the following combinations of card and colour is Ramesh, Dinesh, Lokesh, Nilesh, Shailesh and Hitesh in
TRUE for picture of princess? the same order?
(a) E-Yellow (b) F-Red (a) Q, P, T, R, U, S (b) Q, T, P, R, U, S
(c) Q, P, T, S, U, R (d) Q, T, S, U, R, P
(c) B-Green (d) Data inadequate
(e) None of these
(e) None of these
PROBLEM SOLVING A-67
Directions (Qs. 104-106): Read the following information Directions (Qs. 110-112: Read the following information carefully
carefully and answer the questions given below: to answer the questions given below:
(i) A, B, C, D, E, F, G and H are standing in a row facing North. The annual gathering of a school was organised on a day in the
(ii) B is not neighbour of G. morning hours. Six different items, viz. drama, singing, mimicry,
(iii) F is to the immediate right of G and neighbour of E. speech, story-telling and dance, are to be performed by six children
(iv) G is not at the extreme end. A, B, C, D, E and F not necessarily in the same order. The
(v) A is sixth to the left of E. programme begins with song not sung by B and ends with dance.
(vi) H is sixth to the right of C. C performs mimicry immediately after speech. E performs drama
104. Who among the following are neighbours? just before dance. D or F is not available for the last performance.
(a) AB (b) CG (c) FH (d) CA Speech is not given by A. An interval of 30 minutes is given
(e) None of these immediately after mimicry with three more items remaining to be
105. Which one among the following defines the position of D? performed. D performs immediately after interval.
(a) Fourth to the right of H(b) Third to the right of A 110. Which item is performed by F?
(c) Neighbour of B and F (d) To the immediate left of B (a) Drama (b) Song
(e) None of these (c) Speech (d) Story-telling
106. Which of the following is true? (e) None of these
(a) C is to the immediate left of A 111. Who performed dance?
(b) D is neighbour of B and F (a) A (b) B
(c) G is to the immediate right of D (c) F (d) Data inadequate
(d) A and E are at the extreme ends (e) None of these
(e) None of these 112. Who was the first performer?
Directions (Qs. 107-109): Study the following information (a) A (b) B
carefully and answer the questions given below: (c) C (d) Data inadequate
(i) Five courses A, B, C, D and E each of one month duration (e) None of these
are to be taught from January to May one after the other Directions (Qs. 113-114) : Study the following information
though not necessarily in the same order by lecturers P, Q, carefully and answer the questions given below:
R, S and T. (i) Six children B, D, C, M, J and K are split in two groups of
(ii) ‘P’ teaches course ‘B’ but not in the month of April or May. three each and are made to stand in two rows in such a way
(iii) ‘Q’ teaches course ‘A’ in the month of March. that a child in one row is exactly facing a child in the other
(iv) ‘R’ teaches in the month of January but does not teach row.
course ‘C’ or ‘D’. (ii) M is not at the ends of any row and is to the right of J, who
107. Which course is taught by ‘S’? is facing C. K is to the left of D, who is facing M.
(a) C (b) E (c) Either C or D (d) D 113. Which of the following groups of children are in the same
(e) None of these row?
108. Which lecturer’s course immediately follows after course (a) BMD (b) MJK
‘B’? (c) BDC (d) MJD
(a) Q (b) P (c) S (d) T (e) None of these
(e) None of these 114. Who is to the immediate left of B?
109. Which course is taught in the month of January? (a) M (b) D
(a) C (b) D (c) J (d) Data inadequate
(c) E (d) Data inadequate (e) None of these
(e) None of these

ANSWER KEY
1 (c) 14 (c) 27 (c) 40 (a) 53 (a) 66 (c) 79 (b) 92 (a) 105 (b)
2 (d) 15 (e) 28 (b) 41 (c) 54 (d) 67 (c) 80 (e) 93 (e) 106 (c)
3 (d) 16 (d) 29 (c) 42 (b) 55 (b) 68 (b) 81 (b) 94 (a) 107 (c)
4 (d) 17 (e) 30 (d) 43 (d) 56 (c) 69 (c) 82 (a) 95 (b) 108 (a)
5 (b) 18 (e) 31 (d) 44 (e) 57 (e) 70 (b) 83 (d) 96 (a) 109 (c)
6 (a) 19 (c) 32 (a) 45 (b) 58 (b) 71 (d) 84 (b) 97 (a) 110 (e)
7 (c) 20 (b) 33 (c) 46 (d) 59 (c) 72 (c) 85 (a) 98 (e) 111 (d)
8 (b) 21 (b) 34 (b) 47 (d) 60 (a) 73 (b) 86 (a) 99 (b) 112 (d)
9 (d) 22 (a) 35 (d) 48 (a) 61 (b) 74 (d) 87 (b) 100 (e) 113 (e)
10 (b) 23 (e) 36 (b) 49 (d) 62 (c) 75 (e) 88 (d) 101 (c) 114 (a)
11 (d) 24 (d) 37 (c) 50 (c) 63 (d) 76 (a) 89 (d) 102 (d)
12 (e) 25 (c) 38 (a) 51 (e) 64 (a) 77 (d) 90 (c) 103 (b)
13 (e) 26 (c) 39 (e) 52 (b) 65 (c) 78 (c) 91 (d) 104 (d)
A-68 PROBLEM SOLVING

Answers &
Explanations
1-5: Here, the persons who travel are: A, B, C, D, E, F, and G. Vehicle Person
Stations are: Base station, #I, #II, #III, #IV, and #V. Let us I. Q, W
proceed with the following information: (1), (4), (5), (6), (9), II. P,S,V
(8), and (10). III. T, R
These information give us the following table: Thus the obtained information can be summarised as below:
Station Get in Get down Person Profession Vehicle Sex
Base station ––– ×× × Q Engine er I Male
#I ×× × W Teache r I Female
# II ×× × P Teache r II Female
# III Only G B, D S Doctor II Male
# IV A Only E V Engine er II Female
#V ×× × A, G, C T Teache r III Male
Now, from clue (2), F gets down at # II. And he got in either R Doctor III Female
at base station or at # I. 6. (a) 7. (c) 8. (b) 9. (d) 10. (b)
Now, since F got down at #II and he had got in with C, it 11-15: These are the probable cases of the sitting
implies that both C and F got in either at base station or arrangements of P, Q, R, S, T, U and V:
at # I.
P P
Again, since B and D get down at # III this implies that they
R Q R Q
too got in either at base station or at # I.
It is given that E got in with two other persons i.e., in a
group of three persons. Obviously, E got in at base station.
Hence, once again the above information can be summarised U T T U
as :
Station Get in Get down S V S V
Base station E and (C, F) or (B, D) ×× × Case:I Case:II
#I (C,F) or (B, D) ×× × P P
R Q R Q
# II ×× × Only F
# III Only G B, D
# IV A Only E U V T V
#V ×× × A, G, C
1. (c) 2. (d) 3. (d) 4. (d) 5. (b) S T S U
6-10:Here the persons are P, Q, R, S, T, V and W and the vehicles Case:III Case:IV
are I, II and III. If there are at least two passengers in each 11. (d) We can’t say (iii) to be wrong because case III (as
vehicle and one of them is a male then, in the group there mentioned above) makes the statement true. While
are as least three males. case I and case II makes the statement wrong. Since,
Among them R is a female and she is a doctor. p and v are we are not certain about the positions of U, V and T.
also females. From clue (ii) we get W is a teacher. And q is a Thus, only (i) and (ii) are wrong.
male and ‘ he is an engineer. He travels with only W. This 12. (e) 13. (e) 14. (c) 15. (e) 16. (d)
implies W is a female. And both of them travel in vehicle I. 17. (e) 18. (e) 19. (c) B, C, E and G
From clue (iii), S is a male and he is a doctor. From clue (v),
20. (b)
P is not an engineer (and she can 't be a doctor because
(21-25):
there are only two doctors R and S). Hence, P is a teacher
and she travels in vehicle II. Student's Rank in descending
Now, see the bold parts. It says that there are four females Name Favourite subject order of performance
R, P, V and W. Hence the remaining persons must be males S C B F E M H
because in each vehicle there is at least one male. Hence, T P × × × × × × ü 3
is a male. This implies that S and T will occupy seats in two Q × × × × ü × × 6
different vehicles (II and III) because in vehicle I. Q travels R × × × × × ü × 2
with only W. S × × ü × × × × 4
Again since, R can travel neither with S (see clue iv) nor T ü × × × × × × 7
with P and V (see clue i). Thus, we get their sitting V × × × ü × × × 1
arrangement as follows: W × ü × × × × × 5
PROBLEM SOLVING A-69
S-Science, C-Chemistry, B-Biology, F-French, E-English, M- 31. (d) 32. (a) 33. (c) 34. (b)
Mathematics, H-Hindi. (35-39)
21. (b) 22. (a) 23. (e) 24. (d) 25. (c) Student State Branch
26-30: R Delhi Chem, Electrical (×)
On the basis of the given clues we ge the following information: T Mumbai, Hyder (×) Mech
Car Persons travelling in car Q Calcutta Electrical
P F (–), D (–) Chennai Electronics
Q B (+), E (+) S Mumbai Notital
R A (–), ? (–) P Metallurgy
Besides this, we also came to know C is a female but The following bold lette` can be filled easily with the
still we do not know about the sex of G. But it is given given information.
that there is at least one female in each car. The above R Delhi Civil
table shows that the car Q needs a female to fulfil the
T Bangalore Mechanicl
condition described above. Hence, the seventh person
is a female, i.e., G is a female. But still we do not know Q Calcutta Electrical
among the cars Q and R, in which car C and G are. But U Chennai Electronics
it is clear that both are not in the same car. Let us S Mumbai Chemical
proceed to draw family tree. P Hyderabad Metallurgy
35. (d) 36. (b) 37. (c) 38. (a) 39. (e)
E (+) Û F(-) 1. (40-44) Here
Seven persons are: A, B C, D, E, F and G. Seven subjects are:
B(+) Û A (-) History, Geography, Physics, Chemistry, Maths, Biology,
and English. Subjects are taught on: Monday, Tuesday,
C(-) D ( -) Wednesday, Thursday, and Friday.
Since, C does not travel with her grandfather and Now let us proceed with direct information.
grandmother, C is not in the car P and in Q. Hence, C is From clues (ii), (iv) and (viii), we get
in the car R and G is in the car Q. Still we are not Table 1
aware of G's position in the family tree.
26. (c) 27. (c) 28. (b) 29. (c) 30. (d) Pers on S ubject Day
31 - 34: B Ch emis try Tues day
No. of flats = 13 C M aths M on d ay
Unoccupied flats = 05
Occupied flats = 08 G — Thurs day
No. of flats on second floor = 04 Now, with the help of clues (iii), (iv), (v) and (vii) we get the
Second floor comprises four flats. One occupant is lawyer table in the following form:
and since he has only one neighbour, this imples that out of Table 2
four flats on second floor, two are unoccupied.
Again, since no flat is unoccupied on the third floor, it implies P er s on S ubjec t D ay
that there are three unoccupied flats on floor IV. Since, there
B Geo g ra p h y T u es d ay
are three managers, there will be a manager in each floor. It
is given that there are only two occupant in second floor. Ch e mis try T u es d ay
This im;ies others reside either on floor III or on IV. Again, C M a th s M o n d ay
since there are two teachers there will be a teacher each on G T h u rs d a y
floors III and IV. Again, doctor can't be neighbour of a lawyer. D Frid a y
Hence, the doctor and lawyer will not reside on same floor. F His to ry *
Therefore, doctor and lawyer reside on floor III and IV but A En g lis h *
not necessarily in the same order. Thus we get following
information based on the given clues. From, Table 2, it is obvious that E teaches Geography on
Thus, we get following information based on the given Tuesday.
clues. Now, History and English are not taught on Tuesday,
Floor Total Occupied Unoccupied Occupants Monday, Thursday and Friday. Therefore. History and
flats flats flats English are taught on Wednesday.
II 04 02 02 Lawyer, Manager Again, from Table 2, it is obvious that D teaches either
Physics or Biology. But, from clue (iii), D does not teach
III 03 03 00 Teacher, Manager, Physics. Hence, D teaches Biology. By elimination, G
Lawyer or Doctor
teaches Physics.
IV 06 03 03 Teacher, Manager, Hence, the obtained information can be summarised as
Doctor or Lawyer below:
A-70 PROBLEM SOLVING
Table 3 From clue (i), we get that the two persons who visit on
Person Subject Day Friday are P and the person who is Radiologist. Fill the
above information in the chart.
1. E Geography Tuesday Now, from clue (iii), we get that F is Oncologist and he
2. B Chemistry Tuesday visits alone on Tuesday.
3. C Maths Monday Again, from clue (vii), we get that Neurologist visits
on Friday. Thisimplies that P is Neurologist.
4. G Physics Thursday
Now, from clues (iv) and (v), we get M and K visit on
5. D Biology Friday Wednesday. Again, from clue (vi), we get Paediatrician
6. F History Wednesday visits with ENT specialist. This implies that they visit
7. A English Wednesday either on Wednesday or on Saturday. Again, from clue
(ii), the Paediatrician does not visit on Saturday.
40. (a) 41. (c) 42. (b) 43. (d) Therefore, Paediatrician and ENT specialist visit on
44. (e) B and E Wednesday.
|45-47: Now, using clue (iv), we get that M is ENT specialist
whereas K is the Paediatrician. Again, from clue (v),
Discipline Musical Instrument
Name Med Eng Arch Art Mgmt Sitar Tabla Sarod Guitar Violin
we get that H is not Radiologist. This implies H does
Neeraj × — × × — × × ü × ×
not visit on Friday. Hence, H visits on Saturday.
Yash × — × × — — × × — × Similarly, from clue (viii), we get that B also visits on
Mehul ü × × × × × × × × ü Saturday.
Ram × × × × × ü × × × Now, look at the chart. What do you observe? the
Prakash × × ü × × — × × — × only person left is D. Obviously, D is the Radiologist.
Now, use clue (viii). Since B is not in Orthopaedics, H
45. (b)
46. (d) Engineering or Architecture or Management. is in Orthopaedics and B is Ophthalmologist.
47. (d) Thus the whole information can be summarised as:
(48-52): N, S —Account/Administration ... (i)
Q(–)— Administration – I/III …(ii) Tuesday: F(Oncologist)
S works on Ist floor. M(ENT Specialist),
Wednesday:
M works on IIIrd floor. K(Paediatrician)
S and M work in the same department ...(iii) Friday: P(Neurologist), D(Radiologist)
P(+) – Ist floor
As only two persons work on each floor, Q and M work on H(Orthopaedician),
Saturday:
IIIrd floor. And N and T work on IInd floor. As notwo females B(Ophthalmologist)
work on the same floor, M is a male. Similarly, S is a female.
From (i) and (iii), it is clear that N, S and M work in the same 53. (a) 54. (d) 55. (b) 56. (c)
department. And as no two females work in the same 57. (e) Neurology 58. (b) 59. (c)
department, N is a male and T is a female. 60. (a) 61. (b) 62 (c) 63. (d)
As Q works in Administration, S works in Accounts with N 64-68:
and M. Houses are: P, Q, R, S, T and M.
Now, it is clear that T works in personnel. Now, the table will Colours are: red, blue, white, orange, yellow and green.
look as: Heights are: h 1 , h2, h3, h4, h5 and h6
Person Sex Floor Department where h1= highest house
P Male I Personnel/ h6 = lowest house
Administration Houses are located as shown below:
S Female I Accounts
N Male II Accounts
T Female II Personnel
Q Female III Administration
M Male III Accounts Use the direct informations first and then the secondary
48. (a) 49. (d) 50. (c) 51. (e) N and T informations.
52. (b) Since M is the orange-coloured house located between
53-57: Let us proceed with the following chart: P and S, positions of M, P and S will be as follows:
(a) · · ·
Days Person Specialisation P M S
(or)
Tuesday — —
(b) · · ·
Wednesday — — S M P
Now, since R is the yellow-coloured house exactly
Friday — —
opposite P and Q, the green-coloured house, is exactly
Saturday — — opposite M positions of the houses may be as follows:
PROBLEM SOLVING A-71

· · · · · · 69. (c) D, B, C are girls. Possibility of fourth girl still exists


R Q Q R because sex of G is not known.
(yellow) (green) (green) (yellow) 70. (b) 71. (d) 72. (c)
or (73 – 75) :
· · · · · · Person Posts Days
P M S S M P F Manager Monday
(orange) (orange) B Supervisor Saturday
Obviously, T is opposite S. Now, since T is opposite D Technician Thursday
red-coloured house, S is the red coloured house. It is C Officer Wednesday
given that P is the white-coloured house. Therefore, E Clerk Tuesday
by elimination, T is the blue-coloured house. A Sales executive Friday
Now height of T is h1 (given). The shortest house is The places of bold .letter/words can be filled easily.
M because it is opposite the green coloured house. 73. (b) 74. (d) 75. (e) Friday
Hence, height of M is h6 . (76 – 79) : From (ii) :
Since P is taller than house R but shorter than S and Q, P T
it implies that heights of R and P are hs and h 5 Q Q
respectively. But heights of Q and S are still not known. T P
Thus, the above information can be summarised as
below P U
House Colour Height S S
P White h4 U P
Q Green h 2/h 3 From the last sentence of (ii), only one possibility
R Yellow h5 remains:
R
S Red h 3/h 2 T
T Blue h1 Q
M Orange h6 P
S
Positions of the houses on either side of street may be
as follows: U
Now, using (iii) and the above derived result:
R Q T T Q R R C
T E/B
or
Q A
P M S S M P P F
64. (a) 65. (c) 66. (c) 67. (c) 68. (b) S B/E
U D
69-72: 76. (a) 77. (d) 78. (c) 79. (b)
We have been given that A and G sit on bench I and III (80 – 81) : Subject Sl. No.
respectively. From (2) :Zoology 1/6
Now, since F is a boy and sits with only B, this implies Chemistry 1/6
B is a female (because there is at least one girl on each From (3) :Physics X
bench). F and B sit on bench II. Botany X + 1
On the basis of above information we get From (5) :X = 3
Bench I. A – Chemistry Y
Bench II. F(+) B (–) Maths Y+1
Bench III. G – Now, it is clear that the lecture of Chemistry is not to be
(+) indicates male; (–) indicates female. organised at the end. Hence, from (2), Chemistry–1 and
Now, since C (a girl student) does not sit with A, E and Zoology–6. Hence, the arrangement will be : Chemistry–1,
D, it implies that C sits on bench III (because on bench Maths –2, Physics –3, (break), Botany–4, Statistics–5, Zoology
II only two persons sit). By elimination E and D sit on –6.
bench I. 80. (e) 81. (b) 82. (a)
Now, See the clue, “A sits with his best friend”. The 83 – 85: From the definite informations, we get the following
pronoun `his’ implies that A is a male. Again E is the table (say Table I):
brother of C implies that E is a male. By elimination D is
a female. But sex of G is still not known. Student Subject Author
Thus the information obtained above can be Phy Gupta
summarised as follows. Mihir Edwin
Bench I: A (+) D (–) E (+)
Bench II: F (+) B (–) Neena Maths
Bench III: G C (–) Vinay Eng
(+) indicates male; (–) indicate female. Bio D’Souza
A-72 PROBLEM SOLVING
Now, since Phy is not owned by Sujit, it must be owned by Princess - Red ... (v)
Randhir. The remaining pairs are Mihir-Chem and Sujit Again, from (ii), (v) and Table 1, we get
Bio. Again. Eng-Khanna (x). Combining this information Princess - Red - F ... (vi)
with the table above, we get Maths-Khanna and Eng-Harish. From (i), (vi) and Table 1, we get
So the final table is : Palace - Blue - E ... (vii)
From (iv), (vi), (vii) and Table 1, and then filling up the
Student Subject Author
remaining information, we get Table 2:
Randhir Phy Gupta Card Colour Picture Position
Mihir Chem Edwin
A Brown Queen 6
Neena Maths Khanna B Green/Yellow Prince
Vinay Eng Harish
C Grey King 2
Sujit Bio D’Souza
D Yellow/Green Joker
E Blue Palace
83. (d) 84. (b) 85. (a)
F Red Princess
86 - 89. (+, badminton) A E ( , bridge)
94. (a) The vacant positions are 1, 3, 4 and 5. The given
situation demands that three consecutive cards – 3, 4
(+ , TT) D B ( , carrom) and 5 – be assigned to these. So, Joker will be at No. I
from left.
95. (b)
(?, cricket) C F (+ , chess)
96. (a) Yellow or Green
86. (a) 87. (b) 88. (d) 97. (a)
89. (d) Gender of C is not known. 98-99: Batting Bowling
Peverz 2 3
(90 - 93): Two B-type and two D-type cards are inserted in red
Jatin 5 2
envelopes. Since there are five red envelopes, so only one E-type
Robin 4 4
card is put in red envelope. The two remaining E-type cards and
Dinkar 3 1
one D-type card are thus put in brown envelopes. The remaining Rahul 1 5
two cards in brown envelopes are C-type. The yellow envelopes 98. (e) : Dinkar 99. (b)
thus contain one B-type, one C-type and three A-type cards. (100 - 103) :
Tie Company Name
Envelope Cards
Blue S Hitesh
Red B B D D E
Green P Lokesh
Yellow A A A B C
Pink Q Ramesh
Brown C C D E E
Purple U Shailesh
90. (c) 91. (d) 92. (a) Yellow R Nilesh
93. (e) Brown-2, Red-1
Red T Dinesh
94-97: Let us first proceed with the definite informations in
(iv) and (v). We get the following table (say Table 1: 100. (e) yellow 101. (c)
102. (d) 103. (b)
Card Colour Picture Position 104-106
A Brown Queen 6 GFE ... (iii)
B Prince A ------------ E...(v)
C Grey King 2 C ------------ H... (vi)
D 1800 = 30T2
E Now (v) and (vi) may be combined as
F AC ----------- EH ... (vii) (a)
or CA --------- HE ... (vii) (b)
Note: The cards have been assigned position I to 6 from left to
But (vii) (b) is ruled out because of statement (iv).
right. Combining (iii) and (vii) (a), we get
Let us summarise the remaining information: AC - - GFEH ... (viii)
Palace - Blue - D (×) .....(i) Now, from (ii) and (viii), we get
Princess - D, E (×) .....(ii) ACB - GFEH ... (ix)
Princess - Green, Yellow (×) ... (iii) Now, the blank can be filled by ‘D’, hence the
B-2 or 4 ... (iv) arrangement will be ACBDGFEH
From (i), (iii) and Table 1, we get 104. (d) 105. (b) 106. (c)
PROBLEM SOLVING A-73
(107-109): Lecturers Courses Month (110-112): Programme its order Performer
P B Jan/Feb/Mar ... (ii) song1 B(x) ........ (i)
Q A Mar (iii) dance 6 ......... (ii)
R A/B/E Jan (iv) mimicry x=3 C...... (iii)
Now, from the table it is clear that P will teach in speech (x – 1) = 2 A(x) ..... (iv)
February and ‘R’ will definitely teach the course E. drama 5 E ...... (v)
Hence the table can be made as, [order can be determined with the help of II]
6 D/F(x) ...... (vi)
Lecturers Course Month 4 D ...... (vii)
P B Feb ‘x = 3’ is known by the secondlast line of the given
Q A Mar information:
R E Jan story telling 4 D
S C/D Apr/May drama 5 E
T D/C May/Apr mimicry 3 C
speech 2 B/F
107. (c) 108. (a) 109. (c) song 1 A/F
dance 6 A/B
110. (e) 111. (d) 112. (d)
113 - 114
C D K

J M B
113. (e) 114. (a)
Coded
8 Chapter
Inequalities
Questions related to coded inequalities are essential part of middle Now, we can say that the result of multiplication between 3 and 3
level competitive examinations such as PO, CPO, AAO, etc. Such is equal to 9. Therefore, 3 × 3 = 9 is a case of equality. But when we
problems are not very difficult and very easy for them who are multiply 3 × 4, we get 12 as a result of this multiplication. It does
even slightly comfortable with basis mathematics. But for those, mean that
who are not at case with maths may find it a bit difficult. This 3×4¹9
chapter would give you the basic idea of inequalities and methods As 3 × 4, is not equal to 9, it is a case of inequality.
to solve it in time saving way.
When, we come to know that one thing is not equal to another;
WHAT IS THE PROBLEM LIKE? (PROBLEM FORMAT) there can be only two possibilities:-
Sample Problem (i) One thing is greater than another thing.
(Q s 1-5). Directions: In the following questions, the or
symbols a, b, g, d and h are used with following meaning: (ii) One thing is less than the another thing.
A a B means A is greater than B. When, we denote (i) and (ii) mathematically, then we will write.
A b B means A is either greater than or equal to B. (i) One thing > another thing.
A g B means A is equal to B. or
A d B means A is smaller than B.
(ii) One thing < another thing.
A h B means A is either smaller than or equal to B.
Now, in each of the following questions, assuming the given where ‘>’ denotes ‘greater than’.
statements to be true, find which of the two conclusions I and II and ‘<’ denotes ‘less than’
given below them is/are definitely true. Give answer. Hence, you can write,
(a) If only conclusion I is true. 3×4>9
(b) If only conclusion II is true. 4×1<9
(c) If either I or II is true.
( 3 × 4 > 9)does mean ‘Product of 3 and 4 is greater than 9’.
(d) If neither I nor II is true; and
(e) If both I or II is true. (4 × 1 < 9) does mean ‘Product of 4 and 1 is less than 9’.
Sometimes we come across two numbers where, we do not know
1. Statements: P a N, L g P, O d N, L h K the exact state of inequality between them. For example, we may
Conclusions: I. P d K II. L a N have two numbers m and n and all that we know that ‘n’ is not
2. Statements: E g F, C d D, F b G, D a E less than m’. In such case m can be either greater than or equal to
Conclusions: I. E a G II. C g E n. This situation is represented as ³ sign. When we have to
3. Statements: T b M, O g N, T d H, M g O represent ‘m is less than or equal to n’ then we will use ‘£’ sign.
Conclusions: I. T g N II. T a N Let us see:-
4. Statements: R h Y, K g L, Y d X, R a K m ³ n does mean m is either greater than or equal to n.
Conclusions: I. Y a L II. Y g L m £ n does mean n is either less than or equal to m.
5. Statements: P d I, S g C, S b I, C a O Hence, we can summarise the signs to be used in inequalities as
below:
Conclusions: I. C d I II. S a P
It is clear from the given problem format that such ‘=’ is called equal to
problems involves essentially a combination of two ‘>’ is called greater than
elementary problems:- ‘³’ is called greater than or equal to
i. Inequalities ii. Coding ‘<’ is called less than
It is obvious, that the coding part is not a big challenge here as ‘£’ is called less than or equal to
the coding scheme is told entirely in advance. Hence, to decode
the inequalities in a given problem is not an uphill task. In fact, WHAT IS CHAIN OF INEQUALITIES?
you require only few seconds to decode the inequalities. As such Sometimes two or more inequalities are combined together to
problems based on inequalities, it is high time to get the concept create a single inequality having three or more terms. Such
of the basics of inequalities. combination is called chain of inequalities. For
WHAT IS INEQUALITY? example 24 > 20 and 20 > 16 can be combined as 24 > 20 > 16. In the
As we know, same way, 13 < 17; 17 < 31 and 31 < 38 may be combined as 13 <17
3×3=9 < 31 < 38.
CODED INEQUALITIES A-75
IMPORTANT you have to think a little bit more. Let us consider the
If you see the given problem format (sample problem). You will combined inequality given below:
find that your primarily task is to combine two or more inequalities m³l>n
to create a single inequality. Here, m is either greater than l or equal to l.
Hence, the minimum value for m is equal to l. But l is always
CONDITIONS FOR COMBINING TWO INEQUALITIES greater than n. Therefore, m is always greater than n.
Condition I: Two inequalities will be combined if and only if
they have a common term. \ Our conclusion is m>n
Condition II: Two inequalities will be combined if and only if
the common term is greater than (or ‘greater’ than (d) When, we have the following inequalities:-
or equal to’) one and less than (or ‘less than or m> l³n
equal to’) the other. In this case, m is always greater than l and l is either greater
For example : 14 > 13, 13 > 12 can be easily combined as than n or equal to it. When l is greater than n; m will
‘14 > 13 > 12’. obviously be greater than n. Even when l is equal to n; m
Coded Inequalities will be greater than n as m is always greater than l.
Here,
14 > 13 > 12 \ Our conclusion is m>n

Common term (e) When, we have combined inequality


Clearly, 14 > 13 and 13 > 12 have common term 13 and this common m³l³n
term is greater than 12 and less than 14. Hence, 14 > 13 and 13 > 12 Here, m is either greater than l or equal to l.
have been combined into 14 > 13 > 12 as per the conditions I and When m is greater than l; we have m > l ³ n, which gives the
II. conclusion.

For example : 17 < 19, and 19 < 20 can be easily combined as m>n — (A)
17 < 19 < 20. When m is equal to l; we have
m = l ³ n, which gives the conclusion
Here
m ³ n — (B)
17 < 19 < 20
Combining (A) and (B), we have the final conclusion as
Common term
m ³ n
Clearly, 17 < 19 and 19 < 20 have common term 19 and this common From (a), (b), (c), (d) and (e), we get a rule for deriving conclusions
term is greater than 17 and less than 20. Hence, 17 < 19 and 19 < 20 from a combined inequality, we may say it ‘Golden Rule’.
have been combined into 17 < 19 < 20 as per the conditions I and GOLDEN RULE
II.
Now, let us see some examples of inequalities which can not be The conclusion inequality will have an '³'
combined. Some such examples are given below: sign or a '£' sign if and only if both the
i. 14 > 12, 19 > 18 signs in the combined inequality are '³'
ii. 18 < 20, 22 < 25 or '£' sign
iii. 100 > 99, 80 > 77
Clearly, in (a), (b), (c), (d) and (e) only one inequality
iv. 100 < 115, 118 < 119 (e) (m ³ l ³ n) has ‘³’ as its both the sign.
Clearly, (i), (ii), (iii) and (iv) can not be combined as they do not Important Points to Remember
have any common term and therefore, they do not follow condition
I and condition II. If m > n , then n < m mus t be true.
How to derive conclusions from a combined inequalities?
If m < n , then n > m mus t be true.
To derive conclusion from a combined inequality, you have to
eliminate the common term. If m ³ n , then n £ m mus t be true.
For example, If m £ n , then n ³ m mus t be true.
(a) If we have
Either Choice Rules:
m>l>n
I. When your derived conclusion is of the type m ³ n
then, our conclusion is m>n (or m £ n) then check if the two conclusions are
(b) When, we have m > n and m = n (or, m < n and m = n). If yes, choice “either
follows” is true.
m<l<n
II. If neither of the given conclusions seems correct. Then try
then, our conclusion is m<n to check if the given conclusions form a complementary
pair. Given conclusions form a complementary pair in the 4
(c) When, we have ‘³’ signs in the combined inequalities then cases given below:-
A-76 CODED INEQUALITIES
(i) m ³ n and m < n (ii) m > n and m £ n Conclusion II: Relevant statements are P > N and
(iii) m £ n and m > n (iv) m < n and m ³ n L = P combining both the relevant
In such case, the choice “either follows” is correct. statement, we get L > N. Hence, only
Steps for Solving Problems: conclusion II follows.
Step I: Decode the given symbols like a, b, g, q, d, h, etc. \ Our correct answer choice is (2)
2. Conclusion I: Relevant statements = (E = F, F ³ G). Combining
Step II: Take one conclusion at a time and make an idea that
both the relevant statements, we get E ³ G.
which statements are relevant for evaluating it.
This does not match with the given
Step III: Use conditions I and II and the ‘Golden Rule’ to combine conclusion E > G.
the relevant statements and derive a conclusion from it. Conclusion II: Relevant statements are C < D and D > E.
They are: Combining both the relevant statements, we
Condition I: There must be a common term. get C < E. This does not match with C = E.
Condition II: The common term must be less than or equal to Hence, both conclusions are rejected. Now,
one term and greater than or equal to another. new steps I, II, III, IV as mentioned in the
Golden Rule: The conclusion — inequality is obtained by letting segment ‘Steps for solving problems’ also
the common term be eliminated and it has a ‘³’ or a does not work for this conclusion. Hence,
‘£’ sign if and only if both the inequalities in 2nd our correct answer choice is (4).
step had a ‘³’ or a ‘£’ sign. In all other cases, there 3. Conclusion I: Relevant statements = (O = N,
will be a ‘>’ or a ‘<’ sign in the conclusion. M = O, T ³ M). Combining the 1st two
After performing the above mentioned three steps, if a conclusion statements, we have M = N. Now, combining
is established and verified, it is well and good. But if does not M = N with T ³ M, we get T ³ N. Clearly,
happen so, then you have to perform 4 more new steps given conclusion I does not follow.
below: Conclusion II: We have already seen then T ³ N follows.
New Step I: Check if the given conclusion directly This is different from T > N. So, the
follows from anyone single statement. conclusion II does not follow. But, by virtue
New Step II: Check if the conclusion — inequality you get is of New step III. Choice (3) is our correct
essentially as same as the given conclusion but answer.
written differently (As discussed in important 4. Conclusion I: Conclusion I is Y > L. Now from the given
points to remember) statements Y and L do not appear separately
New Step III: Check if the derived conclusion follows ‘Either with a single common term. Y appears with R,
choice Rule I’. R with K and K with L. Hence, we will take
New Step IV: If neither of the conclusions has been proved these three statements as our relevant
correct till now, then check ‘Either choice Rule II’. statements. They are
R £ Y, R > K, K = L
Solution to Sample Problem (Problem Format)
Through this, we will demonstrate how to use the stepwise Combining R £ Y and R > K
method mentioned above to solve the real problem. (Just see ‘Golden Rule’), we get Y > K, now
Step I: We decode the symbols at this very 1st step. combining it with K = L; we get Y >L. Hence,
conclusion I follows.
(1) Statements: P > N, L = P, O < N, L £ K Conclusion II: Conclusion II is Y = L, which is not true as
Conclusions: I. P < K II. L > N > L has been proved.
\ Our correct answer choice is (1).
(2) Statements: E = F, C < D, F ³ G, D > E 5. Conclusion I: Conclusion I is C < I. C and I appear separately
Conclusions: I. E > G II. C = E with S in S = C and S ³ I. So, these two are
our relevant statements. Combining these
(3) Statements: T ³ M, O = N, T < H, M = O two relevant statements, we get: C > I. This
Conclusions: I. T = N II. T > N does mean conclusion I is not true.
Conclusion II: Conclusion II is S > P. Now, S and P appear
(4) Statements: R £ Y, K = L, Y < X, R > K separately with a common term I; in P > I and
Conclusions: I. Y > L II. Y = L S ³ I. So these two are our relevant
statements and combining them, we get: P <
(5) Statements: P < I, S = C, S ³ I, C > O S. By New step II, it is the same as S > P.
Conclusions: I. C < I II. S > P Therefore, conclusion II follows.
Next, we will take each of the questions separately and perform
\ Our correct answer choice is (2)
step II and step III for each of the conclusion.
1. Conclusion I: Relevant statements = (L = P, L £ K). In this type of questions, usual mathematical symbols (+, –, ÷, ×,
Combining both the relevant statements, we <, > etc.) are represented by symbols, different from the usual
get P £ K. This does not match to the given ones. To solve this type of questions, substitute the real signs in
conclusion the given expression and then solve the expression according to
P < K. the rule BODMAS.
CODED INEQUALITIES A-77

EXAMPLE 1 to 3: In the following questions, the symbols ©, So, either I or II is true.


@, =, * and $ are used with the following meanings : 2. Statements: D © F, F = S, S $ M
P © Q means ‘P is greater than Q’; Conclusions : I. D © M II. F @M
P @ Q means ‘P is greater than or equal to Q’; Sol. (d) Given statements : D > F, F = S, S £ M
P = Q means ‘P is equal to Q’; F = S, S £ M Þ F £ M
P * Q means ‘P is smaller than Q’; Therefore, II is not true.
P $ Q means ‘P is either smaller than or equal to Q’. Now D > F, F £ M
Now in each of following questions, assuming the given Þ nothing can be said about F and M.
statements to be true, find which of the two conclusions I and II So, I is not true.
given below them is/are definitely true. 3. Statements : J = V, V * N, R $ J
Give answer : Conclusions : I. R * N II. J @ N
(a) if only conclusion I is true; Sol. (a) Given statements: J = V, V < N, R £ J
(b) if only conclusion II is true; R £ J, J = V, V < N Þ R < N i.e R * N.
(c) if either I or II is true; So, I is true.
(d) if neither I nor II is true. Now, J = V, V< N Þ J < N
1. Statements : P © T, M $ K, T = K
So, J @ N i.e., J ³ N is not true.
Conclusions : I. T © M II. T = M
Thus, II is false.
Sol. (c) Given statements : P > T, M £ K, T = K.
T = K, K ³ M Þ T ³ M Þ T > M or T = M
Þ T © M or T = M

EXERCISE
Directions (Qs.1-5): In the following questions, the symbols @, I. L % F
#, S, * and % are used as illustrated below: II. F % A
‘P @ Q’ means ‘P is not smaller than Q’. 5. Statements:
‘P # Q’ means ‘P is neither greater than nor equal to Q’. B * D, D $ M, F % M
‘P $ Q’ means ‘P is neither smaller than nor greater than Q’. Conclusions:
‘P * Q’ means ‘P is not greater than Q’. I. B # M
‘P % Q’ means ‘P is neither smaller than nor equal to Q’. II. F % B
Now, in each of the following questions assuming the given Directions (Qs. 6-11): In the following questions the
statements to be true, find which of the two conclusions I and II symbols +, ×, ?, @ and $ are used with the following
given below them is/are definitely true? Give answer meanings:
(a) if only Conclusion I is true. P + Q means P is neither smaller nor greater than Q.
(b) if only Conclusion II is true. P × Q’means P is neither equal to nor smaller than Q.
(c) if either Conclusion I or II is true. P ? Q means P is neither greater than nor equal to Q.
(d) if neither Conclusion I nor II is true. P @ Q means P is either greater than or equal to Q.
(e) if both Conclusions I and II are true. P $ Q means P is not equal to Q.
1. Statements: Now, in each of the following questions assuming the given
M $ K, D * K, R # K statements to be true, find which of the two conclusions I and II
Conclusions: given below them is/are definitely true. Give answer
I. D $ M (a) if only conclusion I is true;
II. M % D (b) if only conclusion II is true;
2. Statements: (c) if either I or II is true:
F * M, M % R, E @ F (d) if neither I nor II is true; and
Conclusions: (e) if both I and II are true.
I. M % E 6. Statement:
II. R @ E P $ Q, Q × R, P + R
3. Statements: Conclusions:
H $ K, T # H, W * T I. Q × P
Conclusions: II. P ? Q
I. K % W 7. Statement:
II. T # K A + B, B $ C, C ? A
4. Statements: Conclusions:
N % A, A # L, F $ N I. C $ A
Conclusions: II. B + C
A-78 CODED INEQUALITIES
8. Statement: (b) if only conclusion II is true.
Y @ Z, Z × Q, Q $ P (c) if either conclusion I or II is true.
Conclusions: (d) if neither conclusion I nor II is true.
I. Y ? Q (e) if both conclusions I and II are true.
II. Y ? P 17. Statements:
9. Statement: Z # N, F © N, F « K
E × F, F @ L, L+ N Conclusion :
Conclusions: I. K $ N
I. N + F II. K @ Z
II. E × L 18. Statements:
10. Statement: D $ T, T © M, M # K
H @ J. J ? K, K × M Conclusion:
Conclusions: I. M $ D
I. H @ M II. D @ M
II. M × J 19. Statements:
11. Statement: W © A, B « A, B @ M
M @ T, T + V, V ? E Conclusions:
Conclusions: I. B # W
I. V + M II. W $ B
II. V ? M 20. Statements:
Directions (Qs. 12-16): In the following questions, the symbols J « M, M $ N, N # T
+, @, $, – and × are used as illustrated below: Conclusions:
‘A+B’ means ‘A is not greater than B’. I. T @ J
‘A@B’ means ‘A is not smaller than B’ II. T $ J
‘A$B’ means `A is neither greater than nor smaller than B’ 21. Statements:
‘A–B’ means `A is neither greater than nor equal to B’ V « F, F @ R, R © G
‘A×B’ means ‘A is neither smaller than nor equal to B’ Conclusions:
Now, in each of the following questions, find out which of the I. G # V
two conclusions I & II is/are definitely true. Give answer II. G @ V
(a) if only conclusion I is true. Directions (Qs. 22-26): In the following questions, the symbols
(b) if only conclusion II is true. #, $, @, * and © are used with the following meaning as illustrated
(c) if either conclusion I or II is true. below:
(d) if neither conclusion I nor II is true. ‘P # Q’ means ‘P is not smaller than Q’
(e) if both conclusions I and II are true. ‘P $ Q’ means `P is neither smaller than nor greater than Q’
12. Statements : L – U, U × G, G×S ‘P @ Q’ means `P is neither greater than nor equal to Q’
Conclusions : I.L × S ‘P * Q’ means `P is not greater than Q’
II.G – L ‘P © Q’ means ‘P is neither smaller than nor equal to Q’
13. Statements : A + U, U $ L, J –L Now in each of the following questions assuming the given
Conclusions : I. J × A statements to be true, find which of the two conclusions I
II. J + U and II given below them is/are definitely true. Give answer
14. Statements : C + S, S – D, D×M (a) if only Conclusion I is true.
Conclusions : I. D × C (b) if only Conclusion II is true.
II. S < M (c) if either Conclusion I or II is true.
15. Statements : Y – G, G @ H, H$ R (d) if neither Conclusion I nor II is true.
Conclusions : I. R $ G (e) if both Conclusions I and II are true.
II. R – G 22. Statements:
16. Statements : P @ Q, D × P, S$ D B $ K, K @ D, D # M
Conclusions : I. S $ P Conclusions:
II. S × P I. B $ M
Directions (Qs. 17 - 21): In the following questions, the symbols II. B @ M
@, ©, «, $ and # are used with the following meaning: 23. Statements:
‘P @ Q’ means ‘P is neither smaller than nor equal to Q’. H @ N, N © W, W # V
‘P © Q’ means ‘P is not smaller than Q’. Conclusions:
‘P«Q’ means ‘P is not greater than Q’. I. H @ V
‘P $ Q’ means ‘P is neither smaller than nor greater than Q’. II. V @ N
‘P # Q’ means ‘P is neither greater than nor equal to Q’. 24. Statements:
Now, in each of the following questions, assuming the given J * D, Q # D, Q @ M
statements to be true, find which of the two conclusions I Conclusions:
and II given below them is/are definitely true? Give answer I. Q © J
(a) if only conclusion I is true. II. Q $ J
CODED INEQUALITIES A-79
25. Statements: (a) if only conclusion I is true;
F # G, N $ G, N © T (b) if only conclusion II is true;
Conclusions: (c) if either I or II is true;
I. T © F (d) if neither I nor II is true; and
II. N * F (e) if both I and II are true.
26. Statements: 32. Statements:
M © R, R @ K , K $ T Z $ K, K × T, T © F
Conclusions: Conclusions:
I. T © R I. F # Z
II. T © M II. Z × T
Directions (Qs.27-31): In the following questions the symbols 33. Statements:
@, +, ©, $, and ? are used with the following meaning: K × B, B @ D, D # K
P Q means P is not equal to Q. Conclusions:
P @ Q means P is greater than Q. I. B @ K
P + Q means P is smaller than Q. II. B # K
P © Q means P is either greater than or equal to Q. 34. Statements:
P $ Q means P is either smaller than or equal to Q. N © R, R @ M, M $ J
P ? Q means P is equal to Q. Conclusions:
Now in each of the following questions assuming the given I. N © M
statements to be true, find which of the two conclusions I II. N # M
and II given below them is/are definitely true. Give answer 35. Statements:
(a) if only conclusion I is true. S $ T, T @ R, R # M
(b) if only conclusion II is true. Conclusions:
(c) if either conclusion I or II is true. I. M × T
(d) if neither conclusion I nor II is true. II. M© T
(e) if both conclusions I and II are true. 36. Statements:
27. Statements : H @ V, V © M, M × R
K © M, M R, R ? T Conclusions:
Conclusions: I. R × H
I. K © T II. H × R
II. M ? T Directions (Qs. 37-41): In these questions, certain symbols have
28. Statements: been used to indicate relationships between elements as follows:
B + D, D @ N, N $ H A B means A is either equal to or greater than B.
Conclusions: A $ B means A is equal to B.
I. M © D A £ B means A is either equal to or smaller than B.
II. H © N A & B means A is smaller than B.
29. Statements: A @ B means A is greater than B.
M © K, K@ P, P $ N In each question, three statements showing relationships
Conclusions: have been given, which are followed by two conclusions I
I. M @ N & II. Assuming that the given statements are true, find out
II. M ? N which conclusion(s) is/are definitely true.
30. Statements: Mark answer (a) if only conclusion I is true.
T $ M, M ? Q, Q + R Mark answer (b) if only conclusion II is true.
Conclusions: Mark answer (c) if either conclusion I or II is true.
I. Q @ T Mark answer (d) if neither I nor II is true.
II. Q ? T Mark answer (e) if both conclusions I and II are true.
31. Statements: 37. Statements:
D @ B, B $ T, T + M S K, T & K, K B
Conclusions: Conclusions:
I. M @ B I. S$B
II. T © B II. S @ B
Directions (Qs.32-36): In the following questions, the symbols 38. Statements:
$, ©, ×, @ and # are used with the following meanings: Y $ Z, H $ D, Z D
P $ Q means P is not smaller than Q. Conclusions:
P © Q means P is neither greater than nor smaller than Q. I. D£Y
P @ Q means P is not greater than Q. II. H £ Z
P × Q means P is neither smaller than nor equal to Q. 39. Statements:
P # Q means P is neither greater than nor equal to Q. M @ N, P @ R, P & N
Now in each of the following questions, assuming the given Conclusions:
statements to be true, find which of the two conclusions I I. P£M
and II given below them is/are definitely true. Give answer II. R & N
A-80 CODED INEQUALITIES
40. Statements: 46. Statements : W×M, M©F, D F
T & K, K B, S K Conclusions :
Conclusions: I. D@W
I. B T II. M©D
II. S £ T III. F@W
41. Statements: (a) None follows
P @ R, M @ N, P & N (b) Only I and II follow
Conclusions: (c) Only II and III follow
I. N @ R (d) Only I and III follow
II. P & M (e) All follow
Directions (Qs. 42-46): In the following questions, the symbols Directions (Qs. 47-54) : In the questions given below, certain
×, %, , @ and © are used with the following meanings as symbols are used with the following meanings:
illustrated below: P $ Q means P is neither equal to nor smaller than Q.
‘P @ Q’ means ‘P is neither greater than nor equal to Q’. P © Q means P is not smaller than Q.
‘P × Q’ means ‘P is not smaller than Q’. P * Q means P is neither greater nor smaller than Q.
‘P Q’ means ‘P is not greater than Q’. P # Q means P is neither greater than nor equal to Q.
‘P © Q’ means ‘P is neither smaller than nor equal to Q’. P @ Q means P is not greater than Q.
‘P % Q’ means ‘P is neither greater than nor smaller than Q’. Now in each of the following questions, assuming the given
Now in each of the following questions assuming the given statements to be true, find which of the two conclusions I
statements to be true, find which of the conclusions given and II given below them is/are definitely true. Give answer
below are definitely true. (a) if only conclusion I is true.
42. Statements : J©T, T B, B@R (b) if only conclusion II is true.
Conclusions : (c) if either conclusion I or II is true.
I. J@R (d) if neither conclusion I nor II is true.
II. R%T (e) if both conclusions I and II are true.
III. J@B 47. Statements :
(a) None follows M # K, K * D, D @ P
(b) Only I follows Conclusions :
(c) Only II follows I. M @ P
(d) Only II and III follow II. M *P
(e) Only I and II follow 48. Statements :
43. Statements : T M, K@M, K×Z, W © T, T $ M, B # M
Conclusions : Conclusions :
I. T@Z I. W $ B
II. Z@M II. M # W
III. M%Z 49. Statements :
(a) None follows H * D, D # R, R © N
(b) Only II follows Conclusions :
(c) Only either II or III follows I. N * H
(d) Only I follows II. N $ H
(e) All follow 50. Statements:
44. Statements : K N, N%T, R@T Z @ R, R © D, D # T
Conclusions : Conclusions:
I. K@R I. D # Z
II. T©K II. Z # T
III. R%K 51. Statements:
(a) All follow Q # P, P @ F, F * M
(b) Only II follows Conclusions:
(c) Only either I or III and II follow I. M $ P
(d) Only either I or II and III follow II. P*M
(e) None of these 52. Statements:
45. Statements : H©M, M×D, T@D
E $ J, J # H, H © M
Conclusions :
Conclusions:
I. T@M
I. E $ M
II. H©D
II. J $ M
III. H%D
53. Statements:
(a) All follow
R © P, P $ M, M @ D
(b) Only I and III follow
Conclusions:
(c) Only II and III follow
I. D $ R
(d) Only I and II follow
II. M # R
(e) None of these
CODED INEQUALITIES A-81
54. Statements: 59. Statements: V @ R, T « V, T $ E
F # K, K © D, N @ D Conclusion:
Conclusions: I. E« R
I. N * K II. N # K II. R%E
Directions (Qs. 55-59) : In the following questions, the symbols Directions (Qs. 60-64) : In the following questions the symbols
$, %, @, #, % and « are used with the following meanings: @, c, £, ? and $ are used with the following meanings:
‘P # Q’ means ‘P is neither smaller than nor equal to Q’. A @ B means A is neither equal to nor smaller than B.
‘P * Q’ means ‘P is neither smaller than nor greater than Q’. A c B means A is neither greater nor smaller than B.
‘P $ Q’ means ‘P is not greater than Q’. A £ B means A is not equal to B.
‘P % Q’ means ‘P is neither greater than nor equal to Q’. A ? B means A is neither greater than nor equal to B.
‘P @ Q’ means ‘P is not smaller than Q’. A $ B means A is either greater or equal to B.
Now in each of the following questions assuming the given Now, in each of the following questions, assuming the given
statements to be true, find which of the two conclusions I statements to be true, find which of the two conclusions I
and II given below them is/are definitely true. Give answer and II given below them is/are definitely true.
(a) if only conclusion I is true. Give answer (a) if only conclusion I is true
(b) if only conclusion II is true. Give answer (b) if only conclusion II is true
(c) if either conclusion I or II is true. Give answer (c) if either I or II is true
(d) if neither conclusion I nor II is true. Give answer (d) is neither I nor II is true
(e) if both conclusions I and II are true. Give answer (e) if both I and II are true
55. Statements: Z « D, F $ D, F # H 60. Statements : N ? S, S @ P, P £ M
Conclusions: Conclusions :
I. H%Z I. S @ M
II. Z # F II. P c N
56. Statements: M % N, N $ T, T # B 61. Statements : J c P, P $ N, J £ H
Conclusions: Conclusions :
I. T # M I. J c N II. H @ P
II. B # N 62. Statements : Z @ D, F c D F $ G
57. Statements: T @ R. K # R, M % K Conclusions :
Conclusions: I. D c G II. Z @ G
I. R # M 63. Statements : L @ T, P ? T, K $ L
II. M # T Conclusions :
58. Statements: M $ K, B % K, B # J I. L @ P
Conclusions: II. K @ T
I. M # J 64. Statements : R c U, U ? Q, W $ R
II. J%K Conclusions :
I. W c U
II. W @ U

ANSWER KEY
1 (c) 7 (a) 13 (e) 19 (c) 25 (b) 31 (e) 37 (c) 43 (b) 49 (d) 55 (a) 61 (d)
2 (d) 8 (d) 14 (a) 20 (a) 26 (a) 32 (e) 38 (e) 44 (e) 50 (d) 56 (a) 62 (b)
3 (e) 9 (b) 15 (c) 21 (d) 27 (d) 33 (b) 39 (b) 45 (d) 51 (c) 57 (d) 63 (e)
4 (b) 10 (d) 16 (b) 22 (d) 28 (b) 34 (c) 40 (d) 46 (e) 52 (d) 58 (b) 64 (c)
5 (b) 11 (c) 17 (b) 23 (b) 29 (d) 35 (a) 41 (e) 47 (d) 53 (b) 59 (c)
6 (e) 12 (d) 18 (c) 24 (c) 30 (c) 36 (d) 42 (a) 48 (e) 54 (c) 60 (d)

Answers &
Explanations
1. (c) M = K .…. (i); 2. (d) F £ M… (i); M > R… (ii); E ³ F… (iii)
D £K ….. (ii); From (i) and (iii), no specific relation can be obtained
R <K ..… (iii) between M and E. Similarly, no specific relation can be
From (i) and (ii), we get obtained between R and E.
M=K ³ D Þ M³D 3. (e) H = K… (i); T < H…(ii),
Hence, either M > D (conclusion II) or M = D W £ T …(iii)
(conclusion I) is true From (i), (ii) and (iii), we get
A-82 CODED INEQUALITIES
K = H > T ³ W Þ K > W (conclusion I) and between S and M. Hence,conclusion II is not true.
T < K (conclusion II). 15. (c) Y < G ...(i); G ³ H ...(ii); H = R ...(iii)
4. (b) N > A… (i), A < L… (ii), F = N…(iii) Combining (ii) and (iii), we get
From (i) and (iii), we get G ³ H = R Þ R = G or R < G
F = N > A Þ F > A (conclusion II). But no specific Hence, either conclusion I or conclusion II is true.
relation can be obtained between L and F. Hence, 16. (b) P ³ Q ...(i), D > P ...(ii); S = D ...(iii)
conclusion I is not necessarily true. Combining (ii) and (iii). we get
5. (b) B < D…(i); D = M…(ii);
S = D > P Þ S > P. Hence, conclusion II is true. But I
F > M …(iii)
From (i), (ii) and (iii), we get is not true.
17. (b) Z < N ....(i); F > N .... (ii); F < K ....(iii) Combining all, we
F > M = D > B Þ B < M and F > B (conclusion II).
get
Since, B < M, therefore, conclusion I is not necessarily K > F > N > Z Þ K > N and K > Z
true.
Hence, conclusion I (K = N) is not necessarily true but
6. (e) P ¹ Q ...(i), Q > R ... (ii), P = R ...(iii)
conclusion II (K > Z) is true.
From (ii) and (iii), we get Q > R = P Þ Q > P. Hence, 18. (c) D = T ....(i); T > M ....(ii); M < K ... (iii) Combining (i)
both I and II are true. and (ii), we get
7. (a) A = B ... (i), B ¹ C ... (ii), C < A ... (iii) From (iii), conclusion D = T > M Þ D > M Þ D = M or D > M
I is true. II contradicts statement (ii), hence, it is not Hence, either conclusion I (M = D) or conclusion II
true. (D > M) is true.
8. (d) Y ″ Z ...(i), Z > Q ... (ii), Q ÷ P ...(iii) 19. (c) W > A ...(i); B < A .... (ii); B > M ... (iii) Combining all,
From (i) and (ii), we get Y > Z > Q Þ Y > Q ... (A) we get
Hence, I is not true. From (iii), two possible relationships W>A> B> M ÞB<W
between P and Q are; Þ B < W or B = W
Case I: When P > Q Hence, either conclusion I or II is true.
Now, using (A), we get Y > Q < P Þ no conclusion. 20. (a) J < M ....(i); M = N ...(ii); N < T .... (iii)
Case II: When Q > P Combining all, we get
using (A), we get Y > Q > P Þ Y > P. Hence, II is not J < M = N< T ÞT> J
true. Hence, only conclusion I is true
9. (b) E > F .... (i), F > L ... (ii), L = N ...(iii) 21. (d) V < F .... (i); F > R .... (ii); R > G ....(iii)
From (ii) and (iii), we get F ″ L= N Þ F ″ N or N ′ F. Combining (ii) and (iii), we get F > R > G ....(iv)
Hence, I may be true but not necessarily so. Comparing (i) and (iv), we can’t get any specific
From (i) and (ii), we get E > F > L Þ E > L relationship between G and V. Hence, both conclusions
Hence, II is true. are not true.
10. (d) H ″ J ... (i), J < K ... (ii), K > M ... (iii) 22. (d) B = K ...(i);
K<D ...(ii);
From (ii) and (iii), we get J < K > M Þ no relationship D> M ...(iii)
between J and M can be established. Hence, II can’t From (i) and (ii), we get
be established. Again, combining all we can’t conclude D>K=B ...(iv)
the relationship between H and M. Hence, I is not true. From (iii) and (iv), no specific relation can be obtained
11. (c) M ″ T ... (i), T = V .... (ii), V < E ...(iii) between B and M. Therefore, B = M (Conclusion I)
From (i) and (ii), we get and B < M (Conclusion II) are not necessarily true.
M ″ T = V Þ M ″ V Þ either V = M or V < M is 23. (b) H < N ... (i)
true. N> W ...(ii);
12. (d) L < U ...(i); U > G ...(ii); G > S ...(III) W>V ...(iii)
Combining (ii) and (iii), we get From (ii) and (iii), we get
U > G ³ S ...(IV) N>W>V ...(iv)
Now, from (i) and (iv), we do not get any specific relation From (i) and (iv), no specific relation can be obtained
between L and S. Hence, conclusion I (L > S) is not between H and V. Hence, H < V (Conclusion I) is not
true. On a similar basis conclusion II (G < L) is also not necessarily true. But V < N (Conclusion II) follows
true. from equation (iv).
13. (e) A £ U ...(i) U = L ...(ii), J > L ...(iii) 24. (c) J < D ...(i);
Combining (i) , (ii) and (iii), we get Q>D ...(ii);
Q< M ...(iii)
J > U = L ³ A Þ J > A and J > U.
Combining (i) and (ii), we get
Hence, both the conclusions are true.
Q > D > J Þ Q > J (Conclusion I) or Q = J
14. (a) C £ S ...(i) S < D ...(ii); D > M ...(iii) (Conclusion II)
Combining (i) and (ii), we get Hence, either conclusion I or conclusion II is true.
D > S ³ C ...(iv) 25. (b) F > G ...(i);
From (iv), we get D > C. Hence, conclusion I is true. N=G ... (ii);
From (iii) and (iv), we do not get any specific relation N>T ... (iii)
CODED INEQUALITIES A-83
Combining all, we get 38. (e) Combining all the three statements, we get
F > G = N > T Þ N < F (Conclusion II) and T < F. Y = Z ³ D = H. Therefore D £ Y and H £ Z are true.
Hence, conclusion I (T > F) is not true but conclusion 39. (b) Combining all the three statements together we get
II is true.
26. (a) M>R ...(i); M > N > P > R. Therefore P £ M is not true, but
R<K ...(ii); R < N is true.
K=T ... (iii) 40. (d) Combining I and III, we get
Combining (ii) and (iii), we get S ³ K > T. Therefore S £ T is not true. We have no
K= T > R information about the relationship between B and T.
Þ T > R (Conclusion I). 41. (e) Combining all the three statements, we get
On the basis of the given information no specific M > N > P > R. Therefore N > R and P < M are true.
relation can be obtained between T and M. Hence, T > 42. (a) J > T ....(i)
M (Conclusion II) is not necessarily true. T £ B....(ii)
27. (d) K ³ M ... (i); M ¹ R ...(ii); R = T ...(iii) B < R ... (iii)
Combining all equations, we get From (ii) and (iii), we get
K ³ M ¹ R=T Þ M ¹ T R > B > T ... (iv)
From this we can’t get any specific relation between K Hence, no specific relation can be obtained between
and T. Hence, conclusion I is not true. Conclusion II is (i) J and R or (ii) J and B. Hence, neither I nor III follows.
false since M ¹ T.. From equation (iv) we get R > T. Therefore, conclusion
II does not follow.
28. (b) B < D ....(i); D > N ...(ii); N £ H ...(iii)
From equations (ii) and (iii), we can’t obtain any specific 43. (b) T £ M ... (i)
K< M ...(ii)
relation between H and D. Hence, conclusion I (H ³ D) K > Z ... (iii)
is not true. But conclusion II (H ³ N) follows from From (ii) and (iii), we get
equation (iii) M > K > Z ...(iv).
29. (d) M ³ K ... (i); K > P ...(ii); P £ N ...(iii) Þ M>Z
Combining (i) and (ii), we get Hence, conclusion II (Z < M) follows. But conclusion
M ³ K > P ...(iv) III (M = Z) does not follow. Again, no specific relation
From (iii) and (iv), no specific relation can be obtained can be obtained between T and Z. Hence, conclusion
between M and N. Hence, conclusion I (M > N) and I does not follow.
conclusion II (M = N) are not true. 44. (e) K £ N ...(i);
30. (c) T £ M ...(i); M = Q ...(ii); Q < R ...(iii) N = T ...(ii);
Combining (i) and (ii) we get R < T ...(iii)
M = Q ³ T Þ Q > T (Conclusion I) From (i) and (ii), we get
or Q = T (Conclusion II) T = N > K ...(iv)
31. (e) D > B ... (i); B £ T ...(ii); T < M ...(iii) From (ii) and (iii), we get
Combining (ii) and (iii), we get T = N > R ...(v)
M > T ³ B Þ M > B (Conclusion I) and From equation (iv), we get T ³ K. Hence, conclusion
T ³ B (Conclusion II). II (T > K) is not necessarily true.
32. (e) From equations (iv) and (v) we can’t obtain any specific
33. (b) K > B …(i), B < D…(ii), D < K…(iii) relation between K and R. There fore, conclusion I and
From (i), B < K. Hence II is true but I is not true. conclusion III do not follow. Thus, no conclusion
34. (c) N = R...(i),R < M...(ii), M > J...(iii) follows.
From (i) and (ii), we get N = R < M Þ N < M. Hence 45. (d) H > M ... (i); M ³ D ... (ii); T < D ... (iii)
either I or II is true. From (i), (ii) and (iii), we get
35. (a) S > T...(i),T < R... (ii), R < M ... (iii) H> M ³ D>T ...(iv)
From (ii) and (iii), we get T < R < M Þ T < M or M > T.. From equation (iv), we get H > T. This implies T < M.
Hence I is true and II is not true. Hence, conclusion I follows.
36. (d) H < V ... (i), V = M ... (ii), M > R... (iii) Again, H > D. Hence II follows but III does not.
Combining all, we get H < V = M > R Þ no relationship 46. (e) W ³ M... (i); M > F... (ii); D £ F...(iii)
between H and R can be established. Since conclusions From (i), (ii) and (iii), we get
I and II are not exhaustive, neither of them is true.
W ³ M > F ³ D...(iv)
37. (c) S ³ K... (i); T < K ... (ii); From (iv); we get W > D. Hence, conclusion I (D < W)
K ³ B ... (iii) follows. Again, from the equation (iv), we get M > D.
Combining (i) and (iii), we get Hence, conclusion II (M > D) follows. Again, from the
S ³ K ³ B. Hence, S > B or S = B. Therefore either equation (iv), we get W > F. Hence, conclusion III
conclusion I or II is true. (F < W) follows.
47. (d) M < K ... (i); K = D... (ii); D < P...(iii)
A-84 CODED INEQUALITIES
Combining all the equations, we get necessarily true.
P ³ D = K > M Þ P > M. Hence, conclusion I (M £ 56. (a) M < N ... (i); N < T ...(ii); T > B ...(iii)
P ) and conclusion II (M = P) are not true. From (i) and (ii), we get
48. (e) W ³ T... (i); T > M... (ii); B < M ...(iii) T > N > M Þ T > M (conclusion I)
Combining all, we get W ³ T > M > B But no relation can be obtained between B and N.
Þ W > B and W > M. Hence, both conclusions Hence, conclusion II does not follow.
(W > B, M < W) are true. 57. (d) T > R ....(i); K > R ...(ii); M < K ...(iii) From (ii) and (iii),
49. (d) H = D ...(i); D < R ...(ii) R ³ N ...(iii) we can’t compare R and M. Hence, I does not follow.
Combining (i) and (ii), we get On a similar basis, we can’t compare M and T. Hence,
R > H = D ...(iv) conclusion II also does not follow.
From (iii) and (iv), we cann’t get any specific relation 58. (b) M < K ...(i) B < K ...(ii) B > J ... (iii) Combining
between N and H. Therefore, conclusion I (N = H) and (ii) and (iii), we get
conclusion II (N > H) are not true. K > B > J ....(iv)
50. (d) Z £ R ...(i); R ³ D ...(ii); D < T ...(iii) From (i) and (iv) we can’t obtain any relation between
M and J. Hence, conclusion I does not follow. But
With these equations no relation can be established
conclusion II is true from (iv).
between D and Z, and Z and T.
59. (c) V > R .... (i); T = V ...(ii);
51. (c) Q < P ...(i), P £ F ...(ii); F = M ...(iii) T < E .... (iii)
Combining all the equations, we get From (i), (ii) and (iii), we get
F = M ³ P > Q Þ M ³ P, ie M > P or M = P E > T = V > R Þ E > R Þ E > R or E = R
Hence, either conclusion I or II is true. Hence, either conclusion I or conclusion II must be
52. (d) E > J ...(i); J < H ...(ii); H ³ M... (iii) ture
No relation can be established between E and M or (60-64): In these questions the negative meaning of these
between J and M. Hence, conclusion I (E > M) and symbols is given to establish the relation between A and B, which
conclusion II (J > M) are not true. can be easily converted to its simple and positive meaning.
53. (b) R ³ P ...(i); P > M ...(ii); M £ D ...(iii) Expressing them mathematically, we get, @ ↑ > (greater than),
Combining (i) and (ii), we get R ³ P > M ...(iv). c ↑ = (equal to), £ ↑ ÷ (means either greater than or smaller
From conclusion (iv), we get R > M. Hence, conclusion than, but no definite conclusion), ? ↑ < (smaller than), and $
II (M < R) is true. But we can’t get any specific relation 60. (d) N < S ... (i), S > P ... (ii), P # M ... (iii)
between D and R. Therefore conclusion I is not true. From (ii) and (iii), no definite relationship between S
54. (c) F < K ...(i); K ³ D ...(ii); N £ D ...(iii) and M can be established. Hence I is not true. From (i)
Combining the equations (ii) and (iii), we get and (ii), we get, N < S > P no conclusion. Hence, II is
K ³ D ³ N Þ K ³ N, i.e., K > N or K= N not true.
Hence, either conclusion I or conclusion II is true. 61. (d) J = P ... (i), P ″ N ... (ii), J ÷ H ... (iii) From (i) and (ii),
55. (a) Z = D ... (i); F < D ...(ii) F > H ...(iii) we get, J = P ″ N Þ J ″ N Þ I may be true but not
From (i), (ii) and (iii), we get necessarily so.
Z = D > F > H Þ H < Z (conclusion I) From (i) and (iii), P = J Þ H Þ P # H. Hence, again II
and Z > F may be true but H < P may be other possibility. Hence,
Hence, conclusion I is true but conclusion II does not II is not true.
9 Chapter
Syllogisms
Syllogism is a Greek word that does mean ‘inference’ or
‘deduction’. The problems of syllogism are based on two parts : Categorical proposition
1. Proposition / Propositions
2. Conclusion / Conclusions drawn from given proposition/
propositions Universal Particular

WHAT IS A PROPOSITION?
Just consider the sentences given below:
Positive Negative Positive Negative
(i) “All lions are pigs ”
All M are P No M are P Some M are P Some M are not P
Subject Predicate (A type) (E type) (I type) (O type)
Therefore, it is clear, that universal propositions either completely
(ii) “No cat is rat ” include the subject (A type) or completely exclude it (E type). On
the other hand, particular propositions either only partly include
Subject the subject (I type) or only partly exclude the subject (O type).
Predicate
Now we can summarise the four types of propositions to be used
(iii) “Some girls are beautiful ” while solving the problems of syllogism :
Format Type
All M are P – A
Subject Predicate No M are P – E
Some M are P – I
(iv) “Some kites are not birds ” Some M are not P – O

Venn Diagram:
Subject Predicate
All A, E, I and O type of propositions can also be represented in
All the sentences mentioned above give a relation between subject pictorial way and this method is known as Venn diagram.
and predicate. Here, it is clear from the sentences that a subject is (i) Representation of “All M are P” (A type):
the part of a sentence something is said about, while a predicate
is the term in a sentence which is related to the subject. P
Now, let us define the proposition :
A proposition is a sentence that makes a statement giving a relation M
between two terms. It has three parts :
(a) the subject
(b) the predicate Here, the whole circle denoting M (all M) lies inside the
(c) the relation between subject and predicate circle denoting P. The other possibility is as picture given
WHAT ISA CATEGORICAL PROPOSITION? below :
Let us see the sentences given below :
“All M are P” M, P
“No M are P”
“Some M are P”
“Some M are not P” (ii) Representation of “No M are P” (E type):
What we notice in all above-Mentioned sentences that they are
condition free. These type of sentences are called Categorical M P
Propositions. In other words a categorical proposition has no
condition attached with it and it makes direct assertion. It is Here, the circle denoting M and P do not intersect at all and
different from non-categorical proposition which is in the format therefore, truely represents
“If M then P” “No M are P”
Types of categorical proposition: (iii) Representation of “Some M are P” (I type):
It can be understood by the diagram given below : This representation will be in two ways :
A-86 SYLLOGISMS
Either (a): Or :
M
M P
P
Here it is clear from the picture that shaded part of M is Some M are P
some part of P and shaded part of P is some part of M. Thus [All P are M]
“Some M are P”. Similarly, unshaded part of M is not P and
Some M are not P (O type):
unshaded part of P is not M. Thus it represents “Some M
are not P”. Either:
Or (b):
M M P
P
Some M are not P
Here, only shaded part of M is P also. Thus we can say [Some M are P]
“Some M are P.” Or:
(iv) Representation of “Some M are not P” (O type):
This representation will be in three ways : P
Either (a):
M

M P Some M are not P


[All P are M]
Here, unshaded part of M is not a part of P. Thus it represents How to identify hidden propositions ?
“Some M are not P.” But the shaded part represents “Some (i) A type: Apart from ‘all’ it starts with every, each and any.
M are P”. EXAMPLE 1.
Or (b) :
Every girl is beautiful.
[All girls are beautiful.]
P Each of them is healthy.
[All (of them) are healthy.]
M Any one could kill the lion.
[All can kill the lion.]
Further, let us see the sentences given below :
Here, shaded part of M is not a part of P. Thus it represents
“Some M are not P” and the circle denoting P represents He should be amended Bharat Ratna
“All P are M”.
Or (c):
Subject Predicate
M P
Amitabh Bacchan is a great actor.
It is clear from this pictorial representation that this
represents “Some M are not P” and “No M are P” as well.
Now we can make a summary of Venn diagram: Subject Predicate
All M are P (A type): Thus, a positive sentence with a particular person as its
P subject is A type.
M, P Also, a sentence in the following format is A type :
and
M definite exception
[Possibility]
No M are P (E type): “All girls except Reeta are healthy.”

M P (ii) E type: Apart from ‘no’ this type of propositions starts from
‘no one’, ‘none’, ‘not a single’ etc.
Some M are P (I type): EXAMPLE 2.
Either:
No one (student) is studious.
[No student is studious]
M P None of the girl is beautiful.
[No girl is beautiful]
Some M are P Not a single girl is healthy.
[Some M are not P] [No girl is healthy].
SYLLOGISMS A-87
Further, let us see the sentences given below : EXAMPLE 4.
He does not deserve Bharat Ratna All girls are not beautiful.
[Some girls are not beautiful]
Every boy is not present.
Subject Predicate [Some boys are not present.]
Further, let us see the following sentences :
Amitabh Bacchan is not a great actor. Poor are usually not healthy.
[Some poor are not healthy]
Almost all the girls are not beautiful.
Subject Predicate
[Some girls are not beautiful.]
Thus, a negative sentence with a particular person as its subject Most of the garments are not handmade.
is E type proposition. [Some of the garments are not handmade.]
Also, sentences in following formats are E type : Girls are not frequently short tempered.
definite exception [Some girls are not short tempered].
Now, it is clear from the above mentioned examples that negative
propositions with words such as ‘almost’, ‘frequently’, ‘most’,
“No student except Reena has failed” ‘mostly’, ‘a few’, generally, etc. are to be reduced to the O–type
“Is there any truth left in the world” propositions.
[No truth is left in the world.] Again, positive propositions starting with words like ‘few’,
(iii) I type: Apart from some it also starts with words such as ‘scarcely’, ‘rarely’, ‘little’, ‘seldom’ etc. are said to be O–type.
often, frequently, almost, generally, mostly, a few, most etc. EXAMPLE 5.
EXAMPLE 3. Seldom are women jealous.
Almost all the girls are beautiful. [Some women are not jealous]
[Some girls are beautiful]. Few girls are beautiful.
Most of the garments are handmade. [Some girls are beautiful]
[Some of the garments are handmade]. Rarely is a wealthy person worried.
Usually girls are beautiful. [Some wealthy person are not worried.]
[Some girls are beautiful.] Also, see the following formates :
A few money are left in my wallet.
[Some money are left in my wallet]. No definite exception as name of
Further, let us see the sentences given below : girls are not given.
Few girls are not studious.
[Some girls are studious.] No girls except three are beautiful.
Rarely is a girl not beautiful.
[Some girls are beautiful]. [Some girls are not beautiful.]
Seldom are women not housewife.
[Some women are housewife]. No definite exception as name of
It is clear from the above examples that negative sentences women are not given.
begining with words like ‘few’, ‘rarely’, ‘seldom’, etc. (Also
‘hardly’, ‘scarcely’, ‘little’ etc.) are to be reduced to I type.
No women except a few are housewife.
Just see the other formates given below :
Therefore, a negative proposition with an indefinite exception, is
Not a definite exception as name of reduced to O type.
girls are not given.
IDENTIFYING EXCLUSIVE PROPOSITIONS :
Such propositions start with ‘only’, ‘alone’, ‘none else but’, ‘none
All girls except a few are beautiful. but’ etc. and they can be reduced to either A or E or I format.
EXAMPLE 6.
[Some girls are beautiful]
Only graduates are Probationary Officers.
Not a definite exception as name of Þ No graduate is Probationary Officer (E type)
girls are not given. Þ All Probationary Officers are graduates. (A type)
Þ Some graduates are Probationary Officers (I type)
General format of sentences given in the examinations :
All girls except 5 have passed All M are P (A type)
No M are P (E type)
[Some girls have passed] Some M are P (I type)
Therefore, a positive proposition with an indefinite exception Some M are not P (O type)
is reduced to I type. NOTE : General format given above are frequently asked formats
(iv) O type : Apart from “Some ....... not’ this type of statements in the examinations. But students must be ready for other hidden
start with words like ‘all’, ‘every’, ‘any’, ‘each’, etc. formates of A, E, I and O types of propositions as problems in
A-88 SYLLOGISMS
hidden formates can also be given in question papers. conclusion (in form of proposition) we get out of two propositions.
Conversion of propositions : To understand it, let us see the following conclusion table :
Before solving the problems of syllogism it is must to know the Conclusion Table
conversion rules of all A, E, O, and I types of propositions : I Proposition II Proposition Conclusion
Conversion of A type : A A A
Subject Predicate A E E
E A (O)R
E I (O)R
“All M are P ” (A type)
I A I
After conversion it becomes. I E O
Subject Predicate Note :
(a) Apart from above 6 pairs of propositions, no other pair will
give any conclusion.
“Some P are M ” (I type) (b) The conclusion drawn out of two propositions is itself a
Therefore, it is clear that A type of propositions get converted proposition and its subject is the subject of the Ist statement
into I type. while its predicate is the predicate of the 2nd statement.
Conversion of E type : The common term get disappeared.
Subject Predicate (c) (O)R does mean that the conclusion is O type but is in
reverse order. In this case, the subject of the inference or
conclusion is the predicate of the 2nd proposition and the
“No M are P ”(E type)
predicate of the conclusion is the subject of the Ist sentence
After conversion it becomes or statement.
Subject Predicate (d) The conclusion table gives correct conclusions or inference
if and only if the two propositions are aligned properly.
WHAT IS ALIGNING ?
“ No P are M ” (E type)
Let us see the following examples :
Therefore, E get converted into E. “sss” EXAMPLE 7.
Conversion of I type : Statements :
Subject Predicate
I. All girls are beautiful.

“Some M are P ” (I type) II. Some girls are Indian.


After conversion it becomes EXAMPLE 8.
Subject Predicate Statements :
I. No pen is chair..
“Some P are M ” (I type)
II. Some tables are pen .
Therefore, I get converted into I.
Conversion of O type : EXAMPLE 9.
O type of proposition can’t be converted. Statements :
Note : In each conversion, subject becomes predicate and
predicate become subject. I. Some women are men .
In fact, conversion is an immediate inference that is drawn from a
single proposition while inference drawn from two propositions II. No men is chair..
are called mediate inference. In all the above mentioned example, we notice that in two
Now we can make a short table of conversion to remember. statements of every example, there is a common term. In
Table of conversion : example 1 the word ‘girl’ is common; in example 2 the word
Type of proposition Get converted into ‘pen’ is common while in example 3 the word ‘men’ is
A .................................. I common.
E .................................. E Now, the aligning of the two statements (propositions) does
I .................................. I mean that the pair of statements must be written in such a
O .................................. Never get converted way that the common term is the predicate of the 1st sentence
Rule to draw conclusion : and the subject of the 2nd.
After knowing conversion of propositions, we must learn the Just think over the following examples :
rules to draw conclusions. In problems of syllogism, conclusions Statements :
are drawn either from single propositions or from two proposition
or from both. But a conclusion from single proposition is just a I. Some girls are cute .
conversion of that proposition while to get conclusion from two
propositions a certain table is used that tells us what type of II. All cute are tall.
SYLLOGISMS A-89
Here, the common term cute is the predicate of the I EXAMPLE 10.
statement and subject of the 2nd statement. Therefore, the Statements :
two statements (I & II) are properly aligned. I. All rats are cats.
But see another example. II. All rats are men.
Statements : When aligned it takes the form as
I. Some bats are chairs. I. Some cats are rats [I type]
II. Some cats are bats . II. All rats are men [A type]
Here, the sentences are not aligned as the predicate of the Now we use the conclusion table given in this chapter that
1st statement is not the subject of the 2nd. says
Then how to align it ? In such type of cases we change the I + A = I type of conclusion.
order of sentences. In another words we put I sentence in Therefore, the drawn conclusion must be
place of II and II in place of I : “Some cats are men”
II. Some cats are bats . It is clear that the conclusion drawn “Some cats are men” is
a mediate inference as it is the result of two propositions.
I. Some bats are chairs. But in actual problem immediate inferences are also given
in conclusion part and that format is given below :
Let us consider another pair of statements.
I. All bats are chair. EXAMPLE 11.
II. All bats are cats. Statements :
Then how to align it ? In fact, in such cases we do alignment I. All rats are cats.
in two ways : II. All rats are men.
(a) by converting statement I as Conclusion :
(i) Some cats are men.
I. Some chair are bats . (ii) Some men are cats.
(iii) Some rats are cats.
II. All bats are cats. (iv) Some cats are rats.
and (v) Some rats are men.
(b) by changing the order of the sentences and then (vi) Some men are rats.
converting the statement II. Answer options :
Now 1st change the order as : (a) only (iii) follows
II. All bats are cats. (b) only (i), (ii) and (iii) follow
I. All bats are chair. (c) only (iv) follows
Again we do conversion for II and the aligned pair (d) all follow
takes the form as (e) none of these
Here, the correct option is (d).
II. Some cats are bats . Conclusion (i) follows because it is the mediate inference of
statements I & II.
I. All bats are chair.. Conclusion (ii) is the conversion of conclusion (i).
Therefore, as per the requirement and nature of the Conclusion (iii) is the immediate inference (conversion) of
sentence the alignment is done. statement I while conclusion (iv) is the conversion of
(i) only by changing the order of sentences. conclusion (iii).
or Conclusion (v) is the immediate inference (conversion) of
(ii) only by converting of the sentences. statement II while conclusion (vi) is the conversion of
or conclusion (v).
(iii) By changing the order of the statements and then Further, in some problems complementary pairs are also
converting one of the sentences. seen in the conclusion part in the forms of sentence given
IEA Rule : below :
Alignment must be done in IEA order. It does mean that if the two (i) Some cats are rats.
statements are I & E then the conversion must be done for I and I-O pair
(ii) Some cats are not rats.
for E & I it will be done for E.
After discussing all the minute things about this chapter, now we (i) All cats are rats.
A-O pair
have come at the position of solving the problems of syllogism. (ii) Some cats are not rats.
This chapter suggests two methods:
(i) Some cats are rats.
(1) By Analytical Method I-E pair
(2) By Venn Diagram (ii) No cats are rats.
(1) Analytical method : Apart from I-O, A-O and I-E pair the two sentences must
This method has two main steps : have same subject and predicate as are the above mentioned
(a) Aligning the pair of sentences. pairs. For these pairs we write the form
(b) Using conclusion table to draw conclusion. Either (i) or (ii) follows
A-90 SYLLOGISMS
For example, see the following format : Here, 1A and 2A are representations for statement A while
EXAMPLE 12. 1B and 2B are representations for statement B. In these
representations
Statements :
b = books
I. Some dogs are cats. II. Some cats are rats.
c = chairs
Conclusions : t = ties
(i) Some cats are dogs. 2nd step :
(ii) Some rats are cats. Let us combine all the possible pairs of this pictorial
(iii) All cats are rats. representations :
(iv) Some dogs are rats.
(v) Some dogs are not rats.
Answer options : b c t c t
b
(a) All follow.
(b) Only (i) follows.
(c) Only (ii) and (iii) follow.
(d) Either (iv) or (v) and (i) & (ii) follow. (1A + 1B) (1A + 2B)
Here, option (d) is correct because conclusion (i) is the
immediate inference (conversion) of statement I while
t c, b c, b t
conclusion (ii) is the immediate inference of II. Conclusion
(iv) & (v) make complementary pair of I-O type.
Conclusion (iii) is not correct because I and II are I type of
statements and I + I does not give any conclusion. Further, (2A + 1B) (2A + 2B)
A type of conclusion can not be find from the immediate 3rd step :
inferences (conversion) of I type of statements as I & II are. When we interpret the pictures in step II, we find that all the
Now, the complete process of solving syllogism problems pictures support both the conclusions. Therefore,
can be summaried as below : conclusion I :
(a) 1st step is aligning the sentences. “Some ties are books” and
(b) 2nd step is using conclusion table. conclusion II.
(c) 3rd step is checking immediate inferences. “Some ties are chairs”
(d) 4th step is checking through the conversion of both are true.
immediate inferences & mediate inferences. Important Note : In the Venn diagram method, any conclusion
(e) Checking the complementary pairs. given with any problem will be true if and only if it is supported
(2) Venn diagram method for solving problems : by all the combined pictorial representations through 2nd step.
Students will have to adopt three steps to solve the If any pictorial representation contradicts the given conclusion,
syllogism problems through Venn diagram method : it will be put in the category of incorrect or wrong conclusion.
(a) 1st step is sketching all possible pictorial Now take another problem :
representation for the statements separately. EXAMPLE 14.
(b) 2nd step is combining possible pairs of these
Statements :
representations of all the statements into one.
A. Some tigers are wolves.
(c) 3rd and final step is making interpretation of this
B. Some wolves are lions.
combined figure. Conclusions are true if they are
Conclusion :
supported by all the combined figures in 2nd step. I. Some tigers are lions.
Now let us solve a problem. II. Some tiger are not lions.
EXAMPLE 13. Sol. 1st step :
Statements :
A. All chairs are books. B. All books are ties. t w w l
Conclusions :
I. Some ties are books. II. Some ties are chairs.
1st Step : 1A 1B

b c t b w t l w

2A 2B
1A 1B 2nd step :
l
c, b b, t t w l or t w

2A 2B (1A + 1B)
SYLLOGISMS A-91
3rd step :
When we interpret the combined pictures given in 2nd steps
w t l or w t l we find that the conclusion I “Some tigers are lions”, is
verified by all the pictures except.
(2A + 1B) Combined pictures of (1A +1B) and (2A + 1B). Hence,
conclusion I is not true and is rejected. Conclusion II “
l Some tigers are not lions” s incorrect because only combined
w t or l t w pictures of (1A + 1B) and (2A + 1B) support is while all other
pictures given in the 2nd step reject it.
(1A + 2B) But here we come across another fact that conclusions I &
II make complementary pain. Therefore one of the two
choices must be true. Hence, either I or II follows.
l w t

(2A + 2B)

EXERCISE
Directions (Qs.1-5): In each of the questions below are given Conclusions:
three statements followed by four conclusions numbered I, II, III I. Some windows are fans.
and IV. You have to take the given statements to be true even if II. Some windows are rooms.
they seem to be at variance with commonly known facts. Read all III. Some fans are green.
the conclusions and then decide which of the given conclusions IV. No green is fan.
logically follows from the given statements disregarding (a) Only I follows (b) Only III follows
commonly known facts. (c) Only IV follows (d) Only II and IV follow
1. Statements: (e) All follow
Some boxes are trees. 4. Statements:
Some trees are horses. Some tablets are rains.
All horses are fruits. All dogs are rains.
Conclusions: All rains are chairs.
I. Some fruits are boxes. Conclusions:
II. Some fruits are trees. I. Some chairs are tablets.
III. Some horses are boxes II. All dogs are chairs.
IV No fruits are boxes. III. Some tablets are dogs.
(a) None follows IV. Some tablets are chairs.
(a) All follow
(b) Only either II or IV follows
(b) Only, I, II and III follow
(c) Only either I or IV and II follow
(c) Only II, III and IV follow
(d) Only either I or III and IV follow (d) Only III and IV follow
(e) None of these (e) None of these
2. Statements: 5. Statements:
All flowers are buses. No man is sky.
Some buses are cats. No sky is road.
All cats are tigers. Some men are roads.
Conclusions: Conclusions:
I. Some tigers are buses. I. No road is man.
II. Some tigers are flowers. II. No road is sky.
III. Some cats are flowers. III. Some skies are men.
IV. Some buses are tigers. IV. All roads are men.
(a) None follows (b) Only I and II follow (a) None follows (b) Only I follows
(c) Only III and IV follow (d) Only I and IV follow (c) Only I and III follow (d) Only II and III follows
(e) Only II and III follow (e) None of these
3. Statements: 6. Statement:
All fans are rooms. Some pages are papers.
No room is green. Some papers are magazines.
Some windows are green. All magazines are books.
A-92 SYLLOGISMS
Conclusions: 11. Statements:
I. Some books are magazines. All biscuits are chocolates.
II. Some books are papers. Some chocolates are breads.
III. Some magazines are pages. All breads are pastries.
IV. Some pages are magazines. Conclusions:
(a) All follow (b) Only I and III follow I. Some biscuits are pastries.
(c) Only II and IV follow (d) Only I and IV follow II. Some pastries are chocolates.
(e) None of these III. Some biscuits are not pastries.
7. Statements: IV. All pastries are breads.
All windows are doors. (a) Only I and II follow
No door is a lock. (b) Only I, lI and III follow
Some locks are keys. (c) Only either I or III and II follow
Conclusions: (d) Only either I or III and IV follow
I. Some locks are doors. (e) None of these
II. Some locks are not doors. 12. Statements:
III. Some keys are not doors. All birds are fruits.
IV. All doors are windows. Some fruits are towers.
(a) All follow. All towers are windows.
(b) Only either I or II follows Conclusions:
(c) Only either I or II and III follow I. Some birds are towers.
(d) None follows II. Some windows are birds.
(e) None of these III. Some windows are fruits.
8. Statements: IV. Some towers are birds.
Some shirts are coats. (a) None follows (b) Only I and II follow
All coats are jackets. (c) Only II and III follow (d) Only III follows
Some jackets are trousers. (e) All follow
Conclusions: 13. Statements:
I. Some shirts are jackets. Some buses are trains.
II. Some jackets are shirts. No train is a dog.
III. All trousers are jackets. All dogs are parrots.
IV. Some trousers are jackets. Conclusions:
(a) All follow I. No bus is a parrot.
(b) Only I, II and III follow II. Some parrots are trains.
(c) Only I, II and IV follow III. Some parrots are buses.
(d) Only II, III and IV follow IV. No dog is a bus.
(e) None of these (a) Only either I or III follows
9. Statements: (b) Only II follows (c) Only IV follows
All bikes are scooters. . (d) Only I and III follow (e) None of these
All scooters are scooties. 14. Statements:
All scooties are mopeds. Some cups are flowers.
Conclusions: Some flowers are boxes.
I. All mopeds are scooties. All boxes are tigers.
II. All scooties are scooters. Conclusions:
III. All scooters are bikes. I. Some tigers are cups.
IV. All bikes are mopeds. II. Some tigers are flowers.
(a) None follows (b) All follow III. Some boxes are cups.
(c) Only III and IV follow (d) Only IV follows IV. No tiger is a flower.
(e) None of these (a) None follows
10. Statements: (b) Only either II or IV follows
Some pots are buckets. (c) Only III follows
Some buckets are bags. (d) Only either I or III follows
Some bags are purses. (e) None of these
Conclusions: 15. Statements:
I. Some purses are buckets. Some cats are lions.
II. Some bags are pots. All lions are hares.
III. Some purses are pots. All hares are horses.
IV. Some pots are bags. Conclusions:
(a) All follow (b) None follows I. Some cats are horses.
(c) Only I and III follow (d) Only II and IV follow II. Some horses are lions.
(e) None of these III. Some hares are cats.
SYLLOGISMS A-93
IV. Some cats are hares. II. All dogs are doors.
(a) Only I and II follow (b) Only III and IV follow III. Some cats are windows.
(c) Only I and III follow (d) All follow IV. Some dogs are windows.
(e) None of these (a) Only I and II follow (b) Only II and III follow
16. Statements: (c) Only I and IV follow (d) All follow
All boats are rivers. (e) None of these
All rivers are mangoes. 21. Statements:
All mangoes are apples. All buses are trains.
Conclusions: All trains are rickshaws.
I. All apples are boats. All rickshaws are cycles.
II. All mangoes are boats. Conclusions:
III. All rivers are apples. I. All cycles are buses.
IV. All boats are apples. II. All rickshaws are buses.
(a) All follow (b) Only I and II follow III. All buses are rickshaws.
(c) Only I, III and IV follow (d) Only II, III and IV follow IV. All trains are cycles.
(e) None of these (a) All follow (b) None follows
17. Statements: (c) Only.I and II follow (d) Only II and III follow
All glasses are roads. (e) None of these
No road is a stick. 22. Statements:
Some sticks are pens. Some candles are houses.
Conclusions: Some houses are trains.
I. Some glasses are sticks. Some trains are roads.
II. Some pens are sticks. Conclusions:
III. Some roads are sticks. I. Some roads are candles.
IV. No glass is a stick. II. Some trains are candles.
(a) None follows III. Some roads are houses.
(b) Only I or IV and II follow IV. Some candles are roads.
(c) Only either I or III or II follows (a) None follows (b) All follow
(d) Only either I or II and IV follow (c) Only I and II follow (d) Only II and III follow
(e) None of these (e) Only III and IV follow
18. Statements: 23. Statements:
Some lions are goats. No tree is fruit.
Some goats are horses. All fruits are stones.
Some horses are flowers. All stones are rains.
Conclusions: Conclusions:
I. Some lions are horses. I. No stone is tree.
II. Some goats are flowers. II. No rain is tree.
III. Some lions are flowers. III. Some rains are fruits.
IV. Some horses are lions. IV. Some rains are trees.
(a) None follows (b) All follow (a) None follows
(c) Only I and IV follow (d) Only II and III follow (b) Only either II or IV and III follow
(e) None of these (c) Only either II or III and I follow
19. Statements: (d) All follow
All trees are books. (e) None of these
Some books are tables. 24. Statements:
All tables are pencils. All books are stars.
Conclusions: Some stars are windows.
I. Some pencils are tables. All windows are hills.
II. Some books are trees. Conclusions:
III. Some tables are trees. I. Some windows are books.
IV. Some pencils are trees. II. Some hills are stars.
(a) None follows (b) All follow III. Some hills are books.
(c) Only either I or III follows (d) Only either I or II follows IV. Some stars are books.
(e) None of these (a) None follows (b) Only I and III follow
20. Statements: (c) All follows (d) Only II and IV follow
Some doors are windows. (e) None of these
All windows are dogs. 25. Statements:
Some dogs are cats. Some cats are rats.
Conclusions: All rats are bats.
I. Some dogs are doors. Some bats are jungles.
A-94 SYLLOGISMS
Conclusions: III. No banana is hotel.
I. Some jungles are cats. (a) Only I and II (b) Only II and III
II. Some bats are cats. (c) Only I follows (d) Either I or III and II follows
III. Some jungles are rats. (e) All follow
IV. No jungles is cat. 31. Statements:
(a) None follows Some books are lamps.
(b) Only III follows Some lamps are rods.
(c) Only either I or IV and III follow Conclusions:
(d) Only either I or IV and II follow I. Some books are rods.
(e) None of these II. No rod is either book or lamp.
26. Statements: III. All rods are lamps
All flowers are clouds.
(a) Only either I or II follows
No cloud is sky.
(b) Only I and III follow
All skies are tigers.
(c) Only II and III follow
Conclusions:
I. Some clouds are flowers. (d) None follows
II. All clouds are flowers. (e) All follow
III. Some tigers are skies. 32. Statements:
IV. All tigers are skies. Some spectacles are boxes.
(a) Only II and IV follow No bat is a ball.
(b) Only either I or II follows Some boxes are balls.
(c) Only either III or IV follows Conclusions:
(d) All follow I. Some boxes are not bats.
(e) None of these II. Some bats are spectacles.
27. Statements: III. No bat is a box.
All bulbs are tables. IV. No ball is a spectacle.
Some bulbs are pots. (a) Only I follows (b) Only I and III follow
Conclusions: (c) Only II and III follow (d) Only IV follows
I. All pots are tables. (e) None of these
II. No pot is table. 33. Statements:
III. Some pots are tables. All fans are tubelights.
(a) Only I (b) Only II No pen is a bulb.
(c) Only III (d) Only I and II Some bulbs are fans.
(e) None of these Conclusions:
28. Statements: I. Some pens are tubelights.
All rats are bells.
II. No pens are tubelights.
All bells are cars.
III. Some tubelights are fans.
Conclusions:
IV. All tubelights are fans.
I. All bells are rats.
II. Some cars are neither bells nor rats. (a) Only I and II follow
III. No car is rat. (b) Only I, II and III follow
(a) Only I (b) Only II (c) Either I or II and III follow
(c) Only III (d) Only II and III (d) Only III and IV follow
(e) None follow (e) None of these
29. Statements: 34. Statements:
All roads are trees. Some fruits are vegetables.
No tree is soap. All liquids are drinks.
Conclusions: All drinks are fruits.
I. No soap is road. Conclusions:
II. Some trees are roads. I. Some drinks are vegetables..
III. No road is soap. II. Some fruits are liquids.
(a) Only I (b) Only II III. All liquids are fruits.
(c) Only III (d) None follows IV. No liquids are vegetables.
(e) All follow (a) Only I and II follow (b) Only II and III follow
30. Statements: (c) Only III and IV follow (d) Only I, II and III follow
Some hotels are bricks. (e) None of these
All bananas are bricks. 35. Statements:
Conclusions: All shirts are trousers.
I. Some bananas are hotels. Some socks are shoes.
II. Some bricks are hotels. All shoes are shirts.
SYLLOGISMS A-95
Conclusions: All cars are trains.
I. Some socks are shirts. Conclusions:
II. Some socks are trousers. I. All trains are trucks.
III. All shoes are trousers. II. All cars are trucks.
IV. All shoes are socks. III. All trucks are trains.
(a) Only I and II follow (b) Only I or II or III follows IV. All vans are trains.
(c) Only II and IV follow (d) Only III and IV follow (a) All follow (b) Only I and II follow
(e) None of these (c) Only II and III follow (d) Only II and IV follow
36. Statements: (e) None of these
Some books are papers. 41. Statements:
All plates are records. No, table is fruit.
Some records are books. No fruit is window.
Conclusions: All windows are chairs.
I. Some plates are books. Conclusions:
II. Some records are papers. I. No window is table.
III. Some papers are plates. II. No chair is fruit.
IV. Some books are records. III. No chair is table.
(a) Only I follows (b) Only II and III follow IV. All chairs are windows.
(c) Only I and Ill follow (d) Only IV follows (a) None follows (b) All follow
(e) None of these (c) Only I and II follow (d) Only III and IV follow
37. Statements: (e) None of these
Some bricks are trees. Directions (Qs. 42-46) : In each of the following questions there
All trees are pens. are three items. These three items may or may not be related with
All pens are boats. one another. Each group of items may fit into one of the diagrams
Conclusions: (a), (b), (c), (d) and (e). You have to decide in which of the following
diagrams and groups of items may fit. The number of that diagram
I. Some boats are bricks.
is the answer.
II. Some pens are bricks.
Give answer (a) If only conclusion I follows.
III. Some trees are bricks.
Give answer (b) if only conclusion II follows.
IV. Some bricks are boats.
Give answer (c) if either I or II follows.
(a) None follows (b) All follow
Give answer (d) if neither I nor II follows.
(c) Only I and II follow (d) Only III and IV follow
Give answer (e) if both I and II follow.
(e) None of these 42. Statements:
38. Statements: All leaders are good team workers.
All cups are tables. All good team workers are good orators.
No table is water. Conclusions:
Some waters are clothes. I. Some good team workers are leaders.
Conclusions: II. All good orators are leaders.
I. No cloth is cup. 43. Statements:
II. No cloth is table. All terrorists are human.
III. Some clothes are waters. All humans are bad.
IV. Some waters are cups. Conclusions:
(a) None follows (b) All follow I. All terrorists are bad.
(c) Only III follows (d) Only I and II follow II. No human can be a terrorist.
(e) None of these 44. Statements:
39. Statements: Some teachers are followers.
Some flowers are rods. Some followers are famous.
Some rods are doors. Conclusions:
Some doors are houses. I. Some teachers are famous.
Conclusions: II. Some followers are teachers.
I. Some houses are flowers. 45. Statements:
II. Some doors are flowers. Some books are pens.
III. Some flowers are doors. No pen is pencil.
IV. No house is flower. Conclusions:
(a) Only I and IV follow I. Some books are pencils.
(b) Only II and III follow II. No book is pencil.
(c) Only either I or II follows 46. Statements:
(d) Only either I or IV follows Some dedicated sourls are angles
(e) None of these All social workers are angles.
40. Statements: Conclusions:
All trucks are vans. I. Some dedicated souls are social workers
All vans are cars. II. Some social workers are dedicated souls
A-96 SYLLOGISMS

ANSWER KEY
1 (c) 6 (e) 11 (c) 16 (e) 21 (e) 26 (e) 31 (d) 36 (d) 41 (a) 46 (d)
2 (d) 7 (e) 12 (d) 17 (e) 22 (a) 27 (c) 32 (a) 37 (b) 42 (a)
3 (c) 8 (c) 13 (a) 18 (a) 23 (b) 28 (e) 33 (c) 38 (c) 43 (a)
4 (e) 9 (d) 14 (e) 19 (e) 24 (d) 29 (e) 34 (b) 39 (d) 44 (b)
5 (e) 10 (b) 15 (d) 20 (c) 25 (d) 30 (d) 35 (e) 40 (e) 45 (c)

Answers &
Explanations
1. (c) Conclusion II follows from conversion of the 9. (d) I does not follow from the last statement, on conversion.
conclusion obtained from statement (b) and statement II does not follow from the second statement, on
(c) [ Q I + A = I]. Conclusion I, III and IV do not follow conversion. III does not follow from the first statement,
because statement (a) + statement (b) gives no on conversion. 1st + 2nd + 3rd statement gives
conclusion. But the conclusion I and IV make a conclusion IV.
complementary pair IE-type. Hence, either of the two 10. (b) As all the statements are I-type, no conclusion is
follows. possible from their combinations. Hence, none follows.
2. (d) Conclusion IV follows from statement (b) and statement 11. (c) Some chocolates are breads + All breads are pastries
(c). [Q I +A = 1]. Conclusion I follows from conversion Þ Some chocolates are pastries ® on conversion ®
of conclusion IV. Statement (a) and statement (b) give Some pastries are chocolates. Hence, II follows. 1st
no conclusion [ Q A + I = no conclusion ]. Therefore, statement + Some chocolates are pastries gives no
conclusion II and III do not follow. conclusion. Hence, I and III do not follow but they
3. (c) Statement (a) + statement (b) gives the conclusion “No make a complementary (1-O) pair. Hence, either I or III
fans are green” [say (d)] [Q A + E = E]. Now, conversion follows. IV does not follow from the last statement.
of statement (d) gives conclusion IV. Now statement 12. (d) Some fruits are towers (I) + All towers are windows
(c) + conclusion IV gives the conclusion “Some (A) Þ Some fruits are windows (I) (Q I + A = I). Now,,
windows are not fans”. Hence, I does not follow. conversion of “Some fruits are windows” gives
Conclusion III does not follow because conclusion IV conclusion III.
follows. 13. (a) Conclusions I and III make a complementary pair.
Again, statement (b) + conversion of statement (c) Hence, either I or III follows. Conclusion II does not
gives the conclusion “Some windows are not rooms”.
follow since “No train is a dog” (E) + “All dogs are
Hence, conclusion II does not follow.
parrots” (A) gives the conclusion “Some parrots are
4. (e) Only I, II and IV follow. Statement (a) + statement (c)
not trains” [Q E + A O «]. Conclusion IV does not
gives conclusion IV [Q I + A = I]. Statement (b) +
follow because “Some buses are trains” + “No train is
statement (c) gives conclusion II [Q A + A = A]. Now,,
statement (b) + conversion of statement (a) gives no a dog” gives the conclusion “Some buses are not
conclusion. Hence, conclusion III does not follow. dogs” [ Q I + E = O].
Conversion of conclusion IV gives conclusion I. 14. (e) “Some flowers are boxes” (I) + “All boxes are tigers”
5. (e) Only II follows. Statement (a) + statement (b) gives no (A) gives conclusion “Some flowers are tigers” (I)
conclusion [Q E + E = no conclusion]. Hence, I does [Q I + A = I]. On conversion, we get “some tigers are
not follow. Note that I does not follow from statement flowers”. Hence, conclusion II follows but IV does not
(c) either. Conclusion II follows from conversion of follow. “Some cups are flowers” (I) + “Some flowers
statement (b). Conclusion III does not follow from are boxes” (I) gives no conclusion [ Q I + I= No
statement (a). Conclusion IV does not follow from conclusion]. Hence, III does not follow. No relation is
statement (c). given between tigers and cups and hence, I does not
6. (e) Some pages are papers + Some papers are magazines follow.
= no conclusion. Hence, IV and III do not follow. In all 15. (d) “Some cats are lions” (I) + “All lions are hares” (A)
the given options either III or IV is present. So, we do Þ “Some cats are hares” (I) [Q I + A = I]. Hence,
not go further and choose (e) as our answer. conclusion IV follows. Conversion of conclusion IV
7. (e) No door is a lock ® on conversion ® No lock is a gives conclusion III. Now, conclusion IV + “All hares
door ® implication ® Some locks are not doors. are horses" (A) gives conclusion I [ Q I + A = I]. “All
Hence, II follows and I does not. No door is a lock + lions are hares" (A) + "All hares are horses" (A) gives
Some locks are keys = Some keys are not doors. Hence, conclusion "All lions are horses" (A) [ Q A + A = A].
III follows. IV does not follow from the first statement. Now, "All lions are horses" implies "Some horses are
8. (c) First + second statement gives conclusion I. lions." Hence, conclusion II also follows.
Conclusion II follows as conversion of conclusion I. 16. (e) Only III and IV follow
Third statement, on conversion, gives conclusion IV 17. (e) Only II and IV follows.
but not conclusion III.
SYLLOGISMS A-97
19. (e) I follows from the last statement, on conversion. (b) gives conclusion III[Q A + E = E]. Hence, con-
Similarly, II follows from the first statement, on clusion III follows. Conclusion I follows from
conversion. Conclusions III and IV do not follow. conversion of conclusion III. Hence, All follow.
20. (c) IV follows from the second statement, on conversion. 30. (d) Conclusion II follows from conversion of statement
Some doors are windows + All windows are dogs = (a). Now, statement (b) + conversion of statement
Some doors are dogs ® on conversion ® Some dogs (a) gives no conclusion [Q A + I = no conclusion].
are doors. Hence, I follows and II does not. III does Hence, conclusions I and III do not follow. But
not follow from the last two statements. conclusion I and conclusion III make an IE-type
21. (e) 1st + 2nd statements gives conclusion III. 2nd + 3rd complementary pair. Hence either conclusion I or
statements gives conclusion IV. I and II do not follow. conclusion III follows.
22. (a) Statement (a) + Statement (b) gives no conclusion 31. (d) Statement (a) + Statement (b) gives no conclusion [ Q I
[\ I + I = no conclusion]. Hence, conclusion II does + I = on conclusion]. Therefore, conclusion I does not
not follow. Conclusions I and IV do not follow because follow. Again conversion of statement (b) gives the
no conclusion can be obtained regarding candles and conclusion “Some rods are lamps”. Hence, conclusions
trains. Statement (b) + Statement (c) gives no II and III do not follow.
conclusion [\ I + I = no conclusion]. Hence, conclusion 32. (a) Some boxes are balls + No ball is a bat (conversion of
III does not follow. 2nd statement) = Some boxes are not bats [I + E = 0].
23. (b) Statement (a) + Statement (b) gives the conclusion Hence, I follows and III does not. Now, some spectacles
“Some stones are not trees.” [Q E + A = O«]. Hence, are boxes and conclusion I give no conclusion. Hence,
conclusion I does not follow. Statement (b) + Statement II and IV do not follow.
(c) gives the conclusion “All fruits are rains”. On 33. (c) Some bulbs are fans + All fans are tubelights = Some
conversions it gives conclusion III. Now, statement bulbs are tubelights ... (a) [I + A = I]. Now, statement (b)
(a) + “All fruits are rains” gives the conclusion “Some + (a) gives: .Some tubelights are not pens. Hence,
rains are not trees” [ Q E + A = O«]. Conclusions II conclusions I and II can’t be established. III follows
and IV do not follow but these two conclusions make from first statement on conversion. But IV does not.
a complementary pair (El-type). Hence, either
But I and II make a complementary pair [I – E pair].
conclusion II or conclusion IV follows.
Hence, either I or II follows.
24. (d) Statement (a) + Statement (b) gives no conclusion
36. (d) Only IV follows from statement (c) on conversion.
[Q A + I = no conclusion]. Hence, conclusion I does
37. (b) III follows from the first statement, on conversion. 1st
not follow. Statement (b) + Statement (c) gives the
statement + 2nd statement gives: Some bricks are
conclusion “Some stars are hills” [ Q I + A = I].
pens...(A) ® on conversion ® Some pens are bricks.
Conversion of “Some stars are hills” gives conclusion
Hence II follows. (A) + 3rd statement gives conclusion
II. Conclusion III does not follow because statement
(a) + statement (b) gives no conclusion. Conclusion IV, which on conversion, give conclusion I.
IV follows from statement (a). 38. (c) III follows from the last statement, on conversion. 1st
25. (d) Statement (a) + Statement (b) gives the conclusion statement + 2nd statement gives: No cup is water ...
“Some cats are bats” [ Q I + A = I]. “Some cats are (A). Hence IV does not follow from A, on conversion.
bats” ® on conversion ® “Some bats are cats”. (A) + last statement gives: Some clothes are not cups.
Hence, conclusion II follows. Conclusion III does not Hence, I does not follow. 2nd statement + last statement
follow because statement (b) + Statement (c) gives no gives: Some clothes are not tables. Hence II does not
conclusion [ Q A + I = no conclusion]. Conclusions I follow.
and IV do not follow because statement (b) + statement 39. (d) As all the statements are I-type, hence no conclusion
(c) gives no conclusions. But conclusions I and IV follows from their combinations. But I and IV make a
make a complementary pair (IE-type). Hence, either I complementary pair, hence either I or IV follows.
or II follows. 40. (e) 1st statement + 2nd statement gives: All trucks are
26. (e) Only I and III follow. Conclusions I and III follow from cars ... (A). Hence II does not follow, on conversion.
statement (a) and statement (c) respectively [on (A) + last statement gives conclusion III. III, on
conversion]. But conclusions II and IV do not follow conversion, gives: Some trains are trucks. Hence I does
because A-type statement can’t be converted into A- not follow. The last two statements gives conclusion
type. IV.
27. (c) Conversion of statement (b) + Statement (a) gives 41. (a) 1st + 2nd statement gives no conclusion. Hence I does
conclusion III [ Q I + A = I). not follow. 2nd + last statement gives: Some chairs are
Hence, III follows but conclusions I and II do not follow. not fruits. Hence II does not follow. III does not follow
28. (e) Statement (a) + Statement (b) gives the conclusion from combining all. IV does not follow from the last
“All rats are cars” [ Q A + A = A] Þ “Some cars are statement, on conversion.
rats”. Hence neither conclusion II nor conclusion III 42. (a) Conclusion I is the conversion of first statement, hence
follows. Conclusion I does not follow from statement I I follows. But II does not follow because A + A = A i.e.
since conversion of statement (a) will give the con All leaders are good orators but not vice versa.
clusion “Some bells are rats”. 43. (a) A + A = A; i.e. All terrorists are human.
29. (e) Conversion of statement (a) gives conclusion II. Hence, 44. (b) I does not follow. But II follows because it is conversion
conclusion II follows. Again statement (a) + statement of the first statement.
A-98 DATA SUFFICIENCY

Data
10 Chapter
Sufficiency
Data sufficiency is not a new kind of problem. It is just a way to EXAMPLE 2. What is the date of birth of Rama?
check your known reasoning ability in new format. In fact, in Statements:
such problems 2 statements are given from different part of I. Veena remembers that Rama’s date of birth is between
reasoning like coding, decoding. Problem solving, blood relation, 17th June and 21st June.
etc, and the examinee is required to find out if each statement II. Surbhi says that Rama’s date of birth is after 19 th June
alone/combinedly sufficient to answer the question. Let us the but before 23rd June.
format of the problem given below:- Sol. From I, we conclude that the possible answers are 18th
PROBLEM FORMAT June, 19th June, and 20th June. From II we come to the
Directions: The problem(s) below consist of a question/questions conclusion that 18th June and 19th June are ruled out. Hence,
followed by two statements labelled I and II. You have to decide 20th June must be the answer clearly, both the statements
if these statements are sufficient to answer the question. are needed to answer the question but none of the two
Mark Answer: statements alone is sufficient to got the answer.
(a) If statement I alone is sufficient to answer the
question but statement II alone is not sufficient to answer EXAMPLE 3. Who is the heaviest among L, M, N and O?
the question. Statements:
(b) If only statement II is sufficient to answer the I. M is heavier than L, but lighter than O.
question but statement I is not sufficient to answer the II. N is lighter than M.
question. Sol. Write statement I as
(c) If both statements I and II are together sufficient to answer O > M > L (‘>’ means heavier than) Write II as
the question although neither statement sufficies by itself. M>N
(d) If both the statements are sufficient to answer the question Now the two inequalities can be combined as
independently and separately.
O > M > N > L or O > M > L > N
(e) It both the statements are not sufficient but still more data
is needed to answer the questions. But in either case O is the heaviest. Hence, I and II are
together needed to answer the question but neither of the
EXAMPLE 1. What is the age of x?
two statement alone can give the answer.
Statements: I The age of y is 50 years.
II x is older than y. Now, from above solved example you must have got the
After seeing the sample problem, you must have got the clear concept about data sufficiency and in a
idea of what is the problem all about. But before solving the position to solve the sample problem also.
sample problem, we must solve some other problems related Solution to sample problem (Problem Format)
to this segment. Only the solution of some problems will Answer choice 5 will be our correct answer as the given
give you the clear concept about this chapter. Let us see information is not sufficient. x can be of any age greater
some examples of solutions given below:- than 50 years.
Thus, it is clear to you that while solving problems related
to data sufficiency, the following methodis used :-
Step I — Check statement I
Step II — Check statement II
Step III — Check both statement I and II if required.
DATA SUFFICIENCY A-99

EXERCISE
Directions (Qs. 1-89) : Each of the questions below consists of 9. Is D brother of J ?
a questions and two statements numbered I and II given below it. I. J is the sister of M and K.
You have to decide whether the data provided in the statements II. K is the brother of D.
are sufficient to answer the question. 10. Which direction is John facing?
Read both the statements and Give answer I. Alok is to the right of John.
(a) if the data in statement I alone are sufficient to answer II. Aman is sitting opposite of Alok facing north.
the question, while the data in statement II alone are 11. Who is the tallest among A, B, C, D and E?
not sufficient to answer the question. I. C is taller than B and E.
(b) if the data in statement II alone are sufficient to answer II. E is taller than D and A and D is taller than B and C.
the question, while the data in statement I alone are 12. A box contains oranges, bananas and apples. How many
not sufficient to answer the question. apples are there in the box?
(c) if the data either in statement I alone or in statement II I. Of the hundred fruits in the box, fifty per cent constitute
alone are sufficient to answer the question. oranges and bananas together.
(d) if the data in both the statements I and II together are II. Fifty per cent of the fruits in the box are apples.
not sufficient to answer the question. 13. A, B,C and D made their project presentation, one on each
(e) if the data in both the statements I and II together are day, on four consecutive days but not necessarily in that
necessary to answer the question. order. On which day did ‘C’ make his presentation?
1. What does ‘pit’ mean in a certain code language? I. The first presentations was made on 23rd,Tuesday and
I. ‘ja na pit sod’ means ‘beautiful bunch of flowers’ in was followed by ‘D’s presentation.
that code language. II. 'A’ did not make his presentation on 25th and one of
II. ‘na sod pa tok’ means ‘huge-bunch of twigs’ in that them made his presentation, between A’s and B’s.
code language. 14. In a certain code “al ed nop” mens “We play chess”. Which
code word means “chess”?
2. Towards which direction is P from R?
I. “id nim nop’ means “We are honest”.
I. S is towards west of M and north-east of R.
II. “gob ots al” means “They play cricket”.
II. P is towards south of S.
15. “You must submit your application within 10 days from the
3. How is M related to R? date of release of this advertisement.” What is exact date
I. P and R are children of K, who is wife of M. before which the application must be submitted?
II. N’s sister M is married to R’s father. I. The advertisement was released on 18th February.
4. Among Q, R, S, T and V who is third from the top when they II. It was a leap year.
are arranged in ascending order of their heights? 16. Kiran is older than Manoj and Dilip is older than Neelam.
I. T is taller than Q and V but shorter than R. Who among them is the youngest?
II. R and S are taller than T and Q is shorter than T but I. Kiran is older than Neelam.
taller than V. II. Manoj is younger than Dilip.
5. When was the election of the president of the society held? 17. ‘B’ is the sister of ‘A’. How is ‘A’ related to ‘B’?
I. Suresh submitted his nomination for the election on I. ‘A’ is the brother of ‘C’.
13th and left on 17th for Delhi the day after he won II. ‘A’ is the uncle of ‘D’.
the election. 18. Brinda’s merit rank is 17th in her class. What is her rank
II. The nominations were scrutinised on 14th and the from the last?
ballot papers were prepared on the following day. I. There are 70 students in her class.
6. What is the meaning of “nic” in a certain code language? II. Nisha who ranks 20th in Brinda’s class is 51st from the
I. In that code language “pat nic no ran” means “what is last.
your name”? 19. Mandar is taller than Sunil and Raghu is shorter than
II. In that code language “nic sa ran ja” means “my name Abhishek. Who among them is the shortest?
is Shambhu”. I. Raghu is shorter than Mandar.
7. How many daughters does K have’? II. Abhishek is shorter than Sunil.
I. L and N are sisters of M. 20. How is ‘go’ written in a certain language?
II. N’s mother is K who has only one son. I. ‘you may come’ is written as ‘pic na ta’ in that code
8. How is S related to R? language.
I. R’s sister is the mother of N, who is daughter of S. II. ‘he may go’ is written as ‘ja ho pic’ in that code
II. P is the sister of S. language.
A-100 DATA SUFFICIENCY
21. Among P, Q, R, S, T and V, who is the heaviest? 37. How many boys are there in the class?
I. P and S are heavier than Q, T and V but none of them I. Mita’s rank among girls is 5th from the top and her
is the heaviest. rank in the class is 9th from the bottom.
II. P is heavier than S but lighter than R. II. No. of boys in the class is twice the number of girls.
22. A, B, C, D and F are seated around a circular table facing at 38. Who is to the immediate right of P among five persons P, Q,
the center. Who is on the immediate right of B? R, S and T facing North?
I. D is between A and F. I. R is third to the left of Q; P is second to the right of R.
II. C is between B and F. II. Q is to the immediate left of T, who is second to the
23. What is the relation between M and F? right of P.
I. M has two sons, one of whom is B. 39. Z is in which direction with respect of X?
II. The mother of F has two sons D and B. I. Y is to the South of X and Z is to the East of P, which is
24. H is in which direction with respect to V? to the North of Y.
I. S is to the south of K, who is to the west of V. II. P is to the South of X.
II. M is to the north of H, who is to the east of V. 40. How is P related to N?
25. 48 children of a class were asked to sit in rows and columns. I. N is sister of M, who is son of Q, whose wife is P.
How many children are seated in each row? II. M is brother of N and son of Q, whose wife is P.
I. The number of columns is more than the number of 41. What is Sunil’s position in a row of forty students?
rows. I. There are sixteen students towards the left of Sunil.
II. The number of rows is 3/4 of the number of columns. II. There are twenty-three students towards the right of
26. Who among M, T, R, K and Q is the tallest’? Sunil.
I. T is taller than R, M and Q but shorter than K. 42. On which date in April was Varun born?
II. R, T and M are shorter than K but taller than Q I. Varun’s mother remembers that Varun was born before
27. In which month of the year was Mohan born? nineteenth but after fifteenth.
II. Varun’s sister remembers that Varun was born before
I. Mohan was born in winter.
II. Mohan was born exactly fourteen months after his seventeenth but after twelfth.
elder sister, who was born in October. 43. How is ‘go’ written in a code language?
I. ‘you may go’ is written as ‘pit. ja ho’ in that code
28. D is in which direction of P?
language.
I. S is to the south of P, which is to the west of D.
II. ‘he may come’ is written as, ‘ja da na’ in that language.
II. P and R are in a straight line and R is to the south of D.
44. How is D related to M?
29. How is P related to M?
I. M has two sisters K and R.
I. P is brother of K and T.
II. D’s mother is sister of K’s father.
II. T is daughter of Q and sister of M’s daughter.
45. Who among M, T, R, J and K is the lightest?
30. In a certain code language what does ‘come’ mean?
I. R is heavier than T and K but lighter than J.
I. ‘pit na ja’ means ‘come and go’ in the code language.
II. J is not the heaviest.
II. ‘na dik sa’ means ‘you may go’ in the code language.
46. How is ‘M’ related to ‘N’?
31. What is Meena’s rank from top in a class of twenty students?
I. ‘P’ is the daughter of ‘M’ and mother of ‘S’?
I. Rama is fifth from the top and two ranks above Meena.
II. ‘T’ is the son of ‘P’ and husband of ‘N’.
II. Ashok is tenth from the bottom and three ranks below
47. On which date of a particular year was Aryabhatta
Meena.
commissioned into the Earth’s orbit?
32. Who among P, Q, S, T, V and W is the shortest?
I. China’s secret services claim that it was between 7th
I. S is taller than T, P and W and is not the tallest.
and 10th of May.
II. T is shorter than Q but is not the shortest.
II. The Japan’s space research scientists claim that it was
33. Which of the following means ‘very’ in a certain code
language? between 5th and 10th of May.
I. ‘pit jo ha’ means ‘very good boy’ in that code 48. How is ‘A’ related to ‘D’?
language. I. ‘C’ is the daughter of A and sister of B.
II. ‘jo na pa’ means ‘she is good’ in that code language. II. ‘D’ is the son of F who is C’s grandfather.
34. On which day of the week was Pramod born? 49. In a row of five buildings A, B, C, D and E, which building is
I. Pramod’s sister was born on Wednesday? in the middle?
II. Pramod’s birthday was after his brother’s birthday but I. Buildings D and B are at the two extreme ends of the
before his sister’s birthday. row.
35. How many sisters does P have? II. Building E is to the right of building C.
I. M and T are sister of K. 50. Which codeword stands for ‘good’ in the coded sentence
II. D is husband of B, who is mother of K and P. ‘sin co bye’ which means ‘He is good’?
36. Who scored highest among A, B, C, D, and E? I. In the same code language ‘co mot det’ means ‘They
I. B scored more than D, but not as much as C. are good’.
II. E scored more than C but not more than A. II. In the same code language ‘sin mic bye’ means ‘He is
honest’.
DATA SUFFICIENCY A-101
51. Among five colleagues, A, B, C, D and E who is the highest 63. P, Q, R, S and T are sitting around a circular table facing the
salary earner? centre. Who is on the immediate right of R?
I. B’s salary is less than the sum of the salaries of A and I. P and T are on the either sides of S.
C, but more than the sum of salaries of E and D. II. Q is on the immediate left of T.
II. A’s salary is more than that of both E and D but less 64. How is M related to D?
than that of C who ranks second in the descending I. D says I have only one brother.
order of their salaries. II. M says I have only one sister.
52. How many students are there in the school? 65. How is ‘over’ written in a code language?
I. The number of boys is 90 more than that of girls.
I. ‘go over there’ is written as ‘na ho ja’ in that code
II. The percentage of boys to the percentage of girls is
language.
145.
II. ‘over and again’ is written as ‘pit tak na’ in that code
53. How is ‘flower’ written in a code language?
I. ‘it is a beautiful flower’ is written as ‘ho na ta ja pa’ in language
that code language. 66. B is F’s brother. K is mother of F. How is F related to B?
II. ‘this is a beautiful place’ is written as ‘ko ja ta po na’ in I. K has only one son and one daughter.
that code language. II. B is the only son of M, who has two children.
54. K is in which direction of T? 67. Among M, T, R, D, B, each one of them having different
I. P is towards South of T and towards East of N. weight, who is the third from top when they are arranged in
II. M is towards North of T and towards West of K. descending order of their weights?
55. How many children are there between M and P in a row of I. R is heavier than M and T but lighter than B.
children? II. M is lighter than R but heavier than T.
I. M is fifteenth from the left in the row. 68. Which village is to the North-East of village A’?
II. P is exactly in the middle and there are ten children I. Village ‘B’ is to the North of village A’, and village `C’
towards his right. and `D’ are to the East and West of village `B’,
56. P, Q, R, S and T are sitting in a circle, facing towards the respectively.
centre of the circle. Who is second to the right of P? II. Village ‘P’ is to the South of village ‘A’, and village ‘E’
I. R is on the immediate left of T and second to the right is to the East of village ‘P’, village ‘K’ is to the North of
of S. village ‘P’.
II. Q is on the immediate right of S and third to the left of P. 69. Can Rohan retire from office ‘X’ in January 2000 with full
57. Among M, K, B, D and W, who is the youngest? pension benefits?
I. B is younger than D. I. Rohan will complete 30 years of service in office ‘X’ in
II. W is younger than K but older than M. April 2000 and desires to retire.
58. What does ‘Ne’ stands for in the code language? II. As per office ‘X’ rules, an employee has to complete
I. ‘Na Ni Nok Ne’ means ‘I will tell you’ and ‘Ni Nok Ne minimum 30 years of service and attain age of 60. Rohan
Nam’ means ‘he will tell you’ in that code language. has 3 years to complete age of 60.
II. ‘Ni Ne Mo Nam’ means ‘will he call you’ and ‘Ne Mok 70. Among five friends P, Q, R, S and T, who ranks third in
Sac Ni’ means ‘how will you go’ in that code language. terms of salary obtained by them?
59. Who amongst P, Q, R, S, T and U is the tallest? I. T’s salary. is more than P and Q but not more than S.
I. P is taller than R and T but not as tall as U, who is taller
II. R’s salary is lowest among them.
than Q and S.
71. How is P related to Q?
II. R is the third in height in the ascending order and not
I. J has two daughters, one of them ‘R’ is married to ‘P’
as tall as U, P and Q, Q being taller than P but not the
tallest. II. Q is the mother of ‘S’, the younger sister of ‘R’
60. Who among A, B, C, D, E & F read the book last? 72. Which word in the code language means ‘flower’?
I. F, who gave the book to B after reading, was third to I. ‘dem fu la pane’ means ‘rose flower is beautiful’ and ‘la
read the same. quiz’ means 'beautiful tree’.
II. C, who read the book after A, was the third person to II. ‘dem fu chin’ means ‘red rose flower’ and ‘pa chin’
read the book before it reached E. means ‘red tea’.
61. Who is paternal uncle of P? 73. In an examination ‘X’ four tests P, ‘Q, R and S are given.
I. P is brother of L, who is daughter of Q, who is sister of Which is the easiest one?
N, who is brother of S. I. Most of the examinees attempted test,’Q’ first. While
II. M is brother of K, who is husband of L, who is mother ‘P’ was left incomplete by many.
of G, who is sister of P. II. Test ‘R’ is found easier than test ‘S’ by all the
62. What is Sudin’s rank in the class of 44 students? examinees.
I. Ramesh, whose rank is 17th in the class, is ahead of Shyam 74. In a row of five A, B, C, D and E, who is standing in the
by 6 ranks, Shyam being 7 ranks ahead of Sudin. middle?
II. Suketu is 26 ranks ahead of Sudin and Shyamala is 6 I. D is to the immediate right of E and B is to the immediate
ranks behind Sudin while Savita stands exactly in the
left of E.
middle of Shyamala and Suketu in ranks, her rank being
II. B is at the extreme left of the row.
17.
A-102 DATA SUFFICIENCY
75. What is the distance between villages ‘X’ and ‘Y’ by the 82. Mahesh’s flat is on which floor of the five-floor apartment?
shortest route? I. His flat is exactly above Ganesh’s flat whose flat is
I. Village ‘X’ is to the North of village ‘Z’ at a distance of exactly above Nitin’s first-floor flat.
35 km. II. Jeevan’s flat, which is adjacent to Mahesh’s flat, is
II. Village ‘Y’ is to the west of village ‘Z’ at a distance of exactly below Ahmed’s flat, who is on fourth floor.
20 km. 83. How many stations are there while going from station ‘X’ to
76. How is Sushma related to Nandini? station ‘Y’?
I. Sushma’s husband is the only son of Nandini’s mother. I. Station ‘G’ precedes station ‘Y’ and station ‘K’ is next
II. Sushma’s brother and Nandini’s husband are cousins. station after station ‘X’.
77. How many candidates were interviewed everyday by the II. Station ‘M’ is third from ‘K’ and there are 4 stations
panel ‘A’ out of the three panels A, B and C? between M and Y.
I. The three panels on an average can interview 15 84. If the first day of a month is Thursday, how many days were
candidates every day. there in that month?
II. Out of a total of 45 candidates interviewed everyday I. The fourth Sunday happened to be on 25th.
by the three panels, the no. of candidates interviewed II. The last day of the month was the fifth Saturday of
by panel ‘A’ is less by 2 than the candidates interviewed that month.
by panel ‘C’ and is less by 1 than the candidates 85. How many girls are taller than Samir in his class?
interviewed by panel ‘B’. I. When students of Samir ’s class are ranked in
78. Which direction is Shashidhar facing? descending order of their height, Samir’s rank is 17th
Statements : from the top among all the students and 12th among
I. In the early morning Shashidhar was standing in front boys.
of a puppet and the shadow of the puppet was falling II. Samir’s rank from the bottom on the basis of height
to the right of Shashidhar. among boys is 18th and among all students is 29th.
II. In the early morning Shashidhar was standing on the 86. Among Nitin, Amit, Sudesh, Rekha and Sujata, who came
ground. His shadow was falling behind him when he last for the programme?
turned to his left. I. Nitin came after Amit but not after Sujata.
79. Who among A, B, C, D and E teaches History? II. Rekha came after Sujata but not after Sudesh.
Statements : 87. (b) Out of the four teams A, B, C and D which team is not
I. Each one of them teaches only one subject. B teaches likely to win as per the opinion poll?
Mathematics, while E teaches Science. A or C does not I. As per the opinion poll, chances of team C’s winning
teach Geography. A or D does not teach English. are more than that of team A but not as much as that of
II. C and E are teachers of English and Science team B, whose chances of winning are more than that
respectively and A is the teacher of Mathematics. of team A.
80. In a row of boys facing South who is immediate left to II. As per the opinion poll team C’s chances of winning
Ramakant? are less than that of team B but not less than that of
Statements : team D, whose chances of winning are more than that
I. Suresh is immediate right to Chandrakant, who is fourth of team A.
to the right of Ramakant. 88. How is Pratibha related to Suresh?
II. Suresh is third to the right of Ramakant and Naresh is I. Suresh’s mother is Pratibha’s mother-in-law.
second to the right of Suresh. II. Suresh is the only son of Sushila, who is Pratibha’s
81. Who has secured the maximum marks among six friends A, mother-in-law.
B, C, D, E and F ? 89. Five friends P, Q, R, S and T are standing in a row facing
Statements : East. Who is standing at the extreme right end?
I. B secured less marks than A and F but not less than C, I. Only P is between S and T ; R is to the immediate right
D and E. of T.
II. F secured more marks than B but not as much as A. II. R is between T and Q.
DATA SUFFICIENCY A-103

ANSWER KEY
1 (d) 11 (b) 21 (a) 31 (c) 41 (c) 51 (b) 61 (b) 71 (e) 81 (e)
2 (d) 12 (a) 22 (d) 32 (d) 42 (c) 52 (e) 62 (c) 72 (d) 82 (c)
3 (c) 13 (e) 23 (d) 33 (d) 43 (d) 53 (d) 63 (e) 73 (c) 83 (e)
4 (b) 14 (e) 24 (b) 34 (d) 44 (e) 54 (b) 64 (d) 74 (e) 84 (b)
5 (e) 15 (a) 25 (b) 35 (d) 45 (d) 55 (e) 65 (e) 75 (e) 85 (a)
6 (d) 16 (d) 26 (c) 36 (e) 46 (d) 56 (b) 66 (d) 76 (c) 86 (e)
7 (e) 17 (c) 27 (b) 37 (d) 47 (d) 57 (d) 67 (d) 77 (b) 87 (b)
8 (d) 18 (c) 28 (a) 38 (c) 48 (d) 58 (d) 68 (a) 78 (c) 88 (b)
9 (d) 19 (b) 29 (e) 39 (e) 49 (d) 59 (c) 69 (d) 79 (a) 89 (e)
10 (d) 20 (d) 30 (d) 40 (c) 50 (c) 60 (d) 70 (d) 80 (d)

Answers &
Explanations
1. (d) From I: ja na pit sod = beautiful bunch of flowers 5. (e) From I: The election was held some day between 14th
From II: na sod pa tok = huge bunch of twigs and 16th.
Even using I & II together, we can’t determine the From II: The election was held after 15th.
code of pit. It may be beautiful or flowers. Using I & II we get election was held on 16th.
2. (d) From I: S M From II: S 6. (d) pat nic no ran = what is your name nic sa ran ja = my
R name is Shambhu ‘nic’ and ‘ran’ is common in both
Using I & II, we get sentences. Hence `nic’ means `is’ or `name’.
7. (e) From I:
S M S M L (–) – (–) N – M
From II:
R P R K (–)
|
P N (–)
Now, combining I and II, we get K (–)
Hence, ‘P’ may be east of ‘R’, or ‘P’ may be southeast of ‘R’, L(–) – N(–) – M (+)
or ‘P’ may be north-east of ‘R’. K has two daughters.
3. (c) From I: (+) M « K (–) 8. (d) From I, S is either R’s sister or brother-in-law. II is no
P R help either.
Clearly, ‘M’ is father of R. 9. (d) From I:
From II: J (-) – M – K
N – M(–) « From II:
K(+) – D
R Combining all, we get, J(-) – M – K(+) – D Hence, ‘D’ may be
Clearly ‘M’ is mother of R. either brother or sister of J.
4. (b) From I: R > T > Q, V 10. (d)
The position of S is not known. Hence I alone is not 11. (b) From I:
sufficient. C > B, E.
From II: R, S > T > Q > V From II:
It is clear that when they are arranged in ascending E > D, A ...(i) D > B, C ... (ii)
order, T is third from the top. From (i) and (ii), E is the tallest.
A-104 DATA SUFFICIENCY
12. (a) From I: Total fruits = 100 the codes used for words of the statement are in the
50% of = 50 same order as the words are.
Hence, number of oranges and bananas is 50. Now, when we use the data given in both statement,
Therefore, number of apples = 100 – 50 = 50. Hence we can get code for ‘may’ only, i.e., ‘pic’.
only statement I alone is sufficient. Statement II does 21. (a) From I: P, S > Q, T, V
not give any absolute figure of fruits or apples. Since none of them is the heaviest. Hence, R is the
13. (e) From I: We get heaviest among them.
First presentation : 23rd (Tuesday) From II: R > P > S
Second presentation : 24th (Wednesday) : D 22. (d) From I:
Third presentation : 25th (Thursday)
Fourth presentation : 26th (Friday)
F A
From II: A did not make his presentation on 25th and
one of them made his presentation between A and B.
From I and II: ‘A’ did not make his presentation on
A D F D
26th also, because ‘D’ made presentation on 24th.
Hence, ‘A’ made presentation on 23rd, ‘B’ on 25th and From II:
‘C’ on 26th. C C
14. (e) From I: code for ‘We’ is ‘nop’.
From II: code for ‘play’ is ‘al’. B F F B
Hence, from I and II together code for ‘chess’ is ‘ed’.
15. (a) Within ten days means before 28th Feb because
advertisement was released on 18th February.
16. (d) We have been given From I and II:
Kiran > Manoj ...(i)
Dilip > Neelam ...(ii) C B
Now, who is the youngest? We need information by B C
which the above equations can be combined into a F A
single equation.
From I: If Kiran > Neelam then either
D D
Manoj or Neelam will be the youngest. A F
From II: If Dilip > Manoj then either Manoj or Neelam (i) (ii)
be the youngest. The person who is immediate right of B is either A or C
Hence, neither I nor II is sufficient. 23. (d) From I : M
17. (c) We have been given ‘B’ is the sister of ‘A’. To answer
‘How is ‘A’ related to ‘B” we need information
regarding the sex of ‘A’. B(+) (+)
From I: If ‘A’ is the brother of ‘C’, obviously ‘A’ is a From II:
male. mother (–)
From II: if ‘A’ is the uncle of ‘D’, obviously ‘A’ is a
male.
Hence, either I or II is sufficient.
F(–) D(+) B(+)
18. (c) From I: Required rank = 70 – 17 + 1 = 54
From II: Total students in Bindra’s class Both the statements I and II are not independently
= 51 + 20 – 1 = 70 sufficient because statement I does not say about F
Now, required rank = 70 – 17 + 1 = 54 and statement II does not say about M.
19. (b) We have been given From statement I and II together, F is the daughter of
Mandar > Sunil ....(i) M. But we don’t know the sex of M.
Abhishek > Raghu ....(ii) 24. (b) From I:
From I: We get either Raghu or Sunil is the shortest. N
From II: Mandar > Sunil > Abhishek > Raghu. K V
Hence, Raghu is the shortest. W E
20. (d) I alone is not sufficient because the word ‘go’ is absent S S
in statement I. Statement II alone is not sufficient
because there is no such indication by which we can [Information regarding H is absent. Hence, I alone is
find out code for ‘go’. Note that you can’t assume that not sufficient]
DATA SUFFICIENCY A-105
From II: From II: Q > T And at least one person is shorter than T.
M From I and II even together, we get no clue regarding
the shortest person.
33. (d) From I: ‘pit jo ha’ Þ very good boy ... (i)
V H From II: ‘jo na pa’ Þ she is good ... (ii)
From I and II, we get ‘jo’ means ‘good’. But still we do
Hence, H is to the east of V
not know whether ‘pit’ or ‘ha’ means ‘very’.
25. (b) From statement (II) alone,
34. (d) The information given in both the statements I and II
let the number of columns be C.
together gives no clue about which day of the week
3 Pramod was born on.
\ No. of rows = C
4 35. (d) Both the statements I and II even together can’t
specify the sex of K.
3 2 36. (e) From I: C > B > D From II: A > E > C
\ No. of children = C
4 Combining both, we get, A > E > C > B > D
3 2 Hence, both statements together are necessary.
\ C = 48 37. (d) From I, it can’t be determined how many girls are there
4
behind Mita, and hence, total no. of girls can’t be
Þ C=8 found out. Hence, II has no use.
The number of children seated in each row is equal to 38. (c) From I: R – PQ (Hence, Q is immediate right of P.)
the number of columns = 8 From II: PQT (Hence, Q is immediate right of P.)
Hence, statement (II) alone is sufficient. 39. (e) From I:
26. (c) From I: X P ,Z
K > T > R, M, Q | |
Hence, ‘K’ is tallest among them. Y Y
From II: (i) (ii)
K > R, T, M > Q
Hence, ‘K’ is tallest among them. Combining (i) & (ii),
27. (b) From II:
X P ,Z
(October + 12 + 2 =) December. |
Hence, Mohan was born in December. |
28. (a) From I: P,Z or X
|
|
P D Y
Y
S
(South-East) (North-East)
From II: (A) (B)
From II:
D D D X
RP PR R |
P P
D is east of P. Now, using I & II together, ‘A’ will be valid.
Many arrangements are possible. 40. (c) From I:
Hence, we can’t determine P « Q (∗)
29. (e) From I: | (P is mother of N)
P (+) — K — T ( n) N , M (∗)
From II:
Q (–) « M(+) From II. Q « P (–)
| |
T (–) Using I and II together, we set M (∗) , N (P is mother of N)
Q (–) « M (+) 41. (c) From I: If there are sixteen students towards left of
| Sunil then Sunil is at (16 + 1 =) 17th position from the
T (–) — K — P (+) left end and (40 – 17 + 1 =) 24th position from the right
Hence, ‘P’ is son of ‘M’. end.
31. (c) From I: Meena’s rank is (5 + 2 =) 7th from the top. From II: If there are twenty-three students towards
From II: Meena’s rank is (11– 3 =) 8th from the top. right of Sunil then Sunil is at (23 + 1=) 24th position
32. (d) From I: S > T, P and W. And at least one person is taller from the right end and (40 – 24 + 1 = ) 17th position
than S. from the left end.
A-106 DATA SUFFICIENCY
42. (c) From I: Varun’s birthday may be 16th, 17th or 18th of boys and girls (145 : 100). Now combining I and II,
April. we get 90 = 45% of total girls.
From II: Varun’s birthday may be 13th, 14th, 15th or Obviously, total strength = 245% of total girls
16th April. Now, 90
= ´ 245 = 490
From I and II: Varun’s birthday is on 16th April. 45
43. (d) We do not know whether the codes of the given words Thus, both the statements are necessary.
are in the same order as the order of the words. 53. (d) From I and II : We get code for ‘flower’ is either ho or
pa. Still, we need some more information to answer the
Therefore, statement I alone is not sufficient. Again,
question.
statement II does not consist of the word ‘go’. Hence, 54. (b) Statement I alone is not sufficient because the
statement 11 alone is not sufficient. Even statements I statement mentions nothing about K. Now, from II we
and II together give the code for ‘may’ only. get.
M–K
44. (e) From I: K and R are sisters of M.
|
From II: * (Father of K) – # (Mother of D) T
| | Thus, K is towards north-east of T.
K D 55. (e) Statement I alone is not sufficient because it mentions
# only M’s position in the row. Whereas statement II
hints only the position of P in the row, ie (10 + 1 =) 11.
From I and II: D Thus from I and II, we get required number of children
M K(–) R(–)
= (15 – 1) – 3 = 11
Thus, D is the cousin of M. 56. (b) From I : We get
45. (d) From I: J > R > T or K S
From II: Atleast one person is heavier than J.
From I and II: We get
M > J > R > T or K
Hence, both the statements I and II together are T
R
insufficient to answer who among T or K is the lightest.
46. (d) Neither statements talk about the sex of M. Since position of P still not clear, statement I alone is
47. (d) From statement I: not sufficient.
From II: We get
Probable dates are: 8th or 9th May
From statement II: S
Probable dates are: 6th, 7th, 8th or 9th May. Q P
From I and II together:
Probable dates are: 8th or 9th May.
48. (d) Neither statements talk about the sex of A.
49. (d)
50. (c) Given: ‘sin co bye’ means ‘He is good’ Obviously, Q is second to the right of P. Hence,
From I: ‘co mot det’ means ‘They are good’ statement II alone is sufficient.
From II: ‘sin mic bye’ means “He is Honest’ 57. (d) From I and II: We get
After a comparison between the given information and D > B...(i)
statement I we get ‘co’ is the code for ‘good’. Similarly, K > W > M... (ii)
after a comparison between the given information and Still, we lack some clue as to whether B or M is the
statement II we get ‘sin’ and ‘bye’ as the codes for ‘He’ youngest. Hence, both statements I and II even
and ‘is’. Thus ‘co’ is the code for ‘good’. together are not sufficient.
51. (b) From I: (A + C) > B > (E + D) We can’t answer the 58. (d) From I: Na Ni Nok Ne ®I will tell you ... (i)
Ni Nok Ne Nam ® he will tell you ... (ii)
question on the basis of statement I. We need some
From (i) & (ii) Na ® I and Nam = he
more information.
From II: Ni Ne Mo Nam ® will he call you ... (iii) Ne
From II: C > A > (E + D) And ‘C’ has the second position
Mok Sac Ni ® how will you go ... (iv)
in descending order of their salaries. Hence B is the
Ne Ni is common in all the four statements. Exact
highest salary earner. transformation of Ne can’t be determined.
52. (e) I alone is not sufficient because we do not know about 59. (c) From I : P > R, P > T, U > P, U > Q, U > S
the number of girls. Similarly, II alone is not sufficient ® U is tallest. [Since U is taller than P, Q & S and P is
because the given information merely gives the ratio taller than R and T]
DATA SUFFICIENCY A-107
From II : R < U, P & Q...(i); Q > P ... (ii)
From (i) and (ii) R P Q U B
Hence U is tallest. |
61. (b) From I : Q(–) — N (+) — S R
|
|
P (+) – L(–)
N is maternal uncle of P. M
From II : (+) M – (+)K – L(–) |
| T
G (–)
M is the paternal uncle of P But information regarding D is necessary to answer
62. (c) From I : Ramesh = 17th the question.
\ Shyam = (17 + 6 =) 23th 68. (a) From I : D B C
Sudin = (23 + 7 =) 30th A
From II: Savita = 17th Hence, village ‘C’ is North-East of village 'A'.
From II : A
K
Suketu ® 25— ® Sudin— 5 — Shyamala P E
69. (d) No statements discuss about pension benefits.
63. (e) 70. (d) From I : S > T > P, Q
From II and using I : S > T > P, Q > R
R P Hence either P or Q ranks third in terms of salary.
71. (e) From I :
Hence P is the son-in-law of Q.
From II :
Q S
J
T
(–) (–) R P
64. (d) The given statements do not give any clue to answer Q (–)
the question.
65. (e) From I: go over there Þ na ho ja ... (i) (–)S – R
From II: over and again Þ pit tak na ... (ii)
J (+) Q (–)
Only ‘over is common in both (i) and (ii). Thus code
Using both I and II, we get,
for ‘over’ is ‘na’. (–) S R(–) P (+)
66. (d) In question part we have the following information: Hence P is the son-in-law of Q.
K(–) 72. (d) From I : dem fu la pane = rose flower is beautiful... (i)
| la quiz = beautiful tree ....(ii)
B (+) – F
\ la = beautiful
We need to know only the sex of F to answer the From II : dem fu chin = red rose flower .... (iii)
question. pa chin = red tea .... (iv)
From I : K has one son (B) and one daughter. Implies From (i) & (iii), dem fu = rose flower
that F is a female. Hence, F is the sister of B. But there we get stuck.
From II : B is the only son of M who has two children
73. (c) From I, P is easier than Q. From II, R is easier than S.
implies that F is a female. Hence, F is the sister of B.
But what about relationships like that between P and
67. (d) From I : B
R?
|
74. (e) From I, we get BED as a sequence. Now, II tells us that
R
B is at the extreme left. Clearly then, D is third from left,
|
i.e. in the middle of the five.
M T
75. (e) Taking Z as the reference point and using both the
From II: R
equations, we can get the distance between X and Y.
|
76. (c) From I: Sushma’s husband is Nandini’s brother (only
M
son of mother) Sushma is Nandini’s sister-in-law.
|
T From II: Sushma’s brother = cousin of Nandini’s
From I and II : husband
Þ Sushma = cousin of Nandini’s husband
= Nandini’s cousin-in-law
A-108 DATA SUFFICIENCY
77. (b) I is not sufficient. Average leads us nowhere when it 83. (e) XK...,....GY...... (I)
comes to specifics. K_ _ M; M _ _ _ _ Y ... (II)
From II: A = C – 2 = B – l and A + B + C = 45. Combing I and II, we get XK _ _ M_ _ _GY.
Solving these, we can get the value of A. Thus, there are eight stations between X and Y.
78. (c) From I: It means the sun is to the left of Shashidhar 84. (b) (I) gives no additional information; it may be
calculated from the statement itself.
and since it is morning, the left of Shashidhar is East.
(II) leads us to an answer. Thursday is 1st day (given
Hence, Shashidhar is facing South.
in the statement). So first Saturday will be the 3rd day.
From II: Sun is to the left of Shashidhar. Hence, he is
And 5th Saturday = 3 + 4 × 7 = 31st day. Since, this is
facing South [Since it is morning]. the last day, there were 31 days in the month.
79. (a) From I: A teaches History among A, B, C, D and E [The 85. (a) From I: (17–1 2) = 5 girl students are taller than Samir
name of other four subjects is given in the statement From II: (29 – 18) = 11 girl students are shorter than
and A teaches none of them.] Samir. But, from II alone it is not known how many girls
From II: Either B or D teaches History. are there in the class.
80. (d) From I: Ramakant _ _ _ Chandrakant Suresh 86. (e) From I: A > N > S From II: S > R > Sud
Hence, cannot be determined. From I and II, we get A > N > S > R > Sud
From II: Ramakant _ _ Suresh _ Naresh 87. (b) From (I), B > C > A
Hence, cannot be determined. D is not frosent here, so no conclusion.
The combination of both statements is not possible. isom (II),
81. (e) From I: A, F > B > C, D, E B> C> D >A
Either A or F has secured maximum marks.
So, A is not likely to win.
From II: A > F > B
88. (b)
From I and II, A secured the maximum marks.
89. (e) From I: S P T R From II: T R Q East
82. (c) From I, Mahesh’s flat is on the 3rd floor. Combining I & II, we get, S P T R Q
From II also, his flat is on the 3rd floor. Hence, Q is at extreme right
Section B : TEST OF NUMERICAL ABILITY

Number System &


Simplifcation
1 Chapter
The ten symbols 0, 1, 2, 3, 4, 5, 6, 7, 8, 9 are called digits, which can Co-primes or Relatively prime numbers: A pair of numbers not
represent any number. having any common factors other than 1 or –1. (Or alternatively
Real numbers: Real numbers comprise the full spectrum of their greatest common factor is 1 or –1)
numbers. They can take on any form – fractions or whole numbers, For Example: 15 and 28 are co-prime, because the factors of 15
decimal points or no decimal points. The full range of real numbers (1,3,5,15), and the factors of 28 (1,2,4,7,14,28) are not in common
includes decimals that can go on forever and ever without end. (except for 1).
3 Twin Primes: A pair of prime numbers that differ by 2
For Example: 8, 6, 2 + 3 , etc. (successive odd numbers that are both Prime numbers).
5
Natural numbers: A natural number is a number that comes For Example: (3,5), (5,7), (11,13), ...
naturally. Natural Numbers are counting numbers from 1, 2, 3, Place value : Place value is a positional system of notation in
4, 5, ........ which the position of a number with respect to a point determines
Whole numbers: Whole numbers are just all the natural numbers its value. In the decimal system, the value of the digits is based
plus zero. on the number ten.
Each position in a decimal number has a value that is a power of
For Example: 0, 1, 2, 3, 4, 5, and so on upto infinity.
10. A decimal point separates the non-negative powers of 10,
Integers: Integers incorporate all the qualities of whole numbers (10)0=1, (10)1=10, (10)2=100, (10)3 =1000, etc.) on the left from the
and their opposites (or additive inverses of the whole numbers).
Integers can be described as being positive and negative whole 1 1 1
negative powers of 10, (10)–1 = , (10)–2 = , (10)–3 = ,
numbers. 10 100 1000
For Example: … –3, –2, –1, 0, 1, 2, 3, . . . etc.) on the right.
p Face value : The face value of a number is the value of the number
Rational numbers: All numbers of the form where p and q are without regard to where it is in another number. So 4 7 always has
q
integers (q ¹ 0) are called Rational numbers. a face value of 7. However the place value includes the position
of the number in another number. So in the number 4,732, the 7
For Example: 4, 3 , 0, .... has a place value of 700, but has a face value of just 7.
4
Irrational numbers: Irrational numbers are the opposite of rational Example 1 : Place and face values of the digits in the number 495,
numbers. An irrational number cannot be written as a fraction, 784:
and decimal values for irrational numbers never end and do not Number Digit Place value Face value
have a repeating pattern in them. 'pi' with its never-ending decimal
places, is irrational. 495,784 4 400000 4
For Example: 7, 5, 2 + 2, p,..... 9 90000 9

Even numbers: An even number is one that can be divided evenly 5 5000 5
by two leaving no remainder, such as 2, 4, 6, and 8. 7 700 7
Odd numbers: An odd number is one that does not divide evenly 8 80 8
by two, such as 1, 3, 5, and 7. 4 4 4
Prime numbers: A prime number is a number which can be divided
only by 1 and itself. The prime number has only two factors, 1 FRACTIONS
and itself. A fraction is known as a rational number and written in the form
For example: 2, 3, 5, 7, 11, 13, 17, .... are prime numbers. p
of where p and q are integers and q ¹ 0. The lower number
Composite Number: A Composite Number is a number which can q
be divided into a number of factors other than 1 and itself . Any 'q' is known as denominator and the upper number 'p' is known
composite number has additional factors than 1 and itself. as numerator.
For example: 4, 6, 8, 9, 10 .....
B-2 NUMBER SYSTEM & SIMPLIFICATION
TYPE OF FRACTIONS : 2nd rational number (i.e., between 3 and 4)
Proper Fraction: The fraction in which numerator is less than the 3+ 4 7
= =
denominator is called a proper fraction. 2 2
3rd rational number (i.e., between 4 and 5)
2 5 10
For Example: , , etc. 4+5 9
3 6 11 = = .
Improper fraction : The fraction in which numerator is greater 2 2
than the denominator is called improper fraction. Both rational and irrational numbers can be represented in
number line. Thus real numbers is the set of the union of
3 6 8 rational and irrational numbers.
For Example : . , , etc
2 5 7 R =Q ÈQ'
Mixed fraction : Mixed fraction is a composite of a fraction and a Every real number is either rational or irrational.
whole number.
1 3 6 Equivalent fractions/Equal fractions : Fractions with same
For example: 2 , 3 ,5 etc. value.
2 4 7
Complex fraction: A complex fraction is that fraction in which 2 4 6 8 æ 2ö
For example : , , , ç= ÷ .
numerator or denominator or both are fractions. 3 6 9 12 è 3 ø
2 2 3 Like fractions: Fractions with same denominators.

For Example: 3 , 5 , 7 , etc. For example :


2 3 9 11
, , ,
4 6 5 7 7 7 7
7 6 Unlike fractions : Fractions with different denominators.
Decimal fraction: The fraction whose denominator is 10 or its
2 4 9 9
higher power, is called a decimal fraction. For example : , , ,
5 7 8 2
7 11 12 Simple fraction : Numerator and denominator are integers.
For Example: , ,
10 100 1000
3 2
Continued fraction: Fractions which contain addition or For example : and .
subtraction of fractions or a series of fractions generally in 7 5
denominator (sometimes in numerator also) are called continued Vulgar fraction : Denominators are not the power of 10.
fractions. 3 9 5
For example : , , .
Formulae to Remember 7 2 193
Example 3 : Write 2.73 as a fraction.
n(n + 1)
v Sum of first n natural numbers = Sol. 2.73 =
273
2
100
v Sum of first n even numbers = n(n + 1)
2
v Sum of first n odd numbers = n2 Example 4 : Express as a decimal fraction.
5
Composite Numbers : It is a natural number that has atleast one 2 2´ 2 4
Sol. = =
divisor different from unity and itself. 5 5 ´ 2 10
Every composite number can be factorised into its prime factors. Example 5 : After doing 3/5 of the Biology homework on Monday
For Example : 24 = 2 × 2 × 2 × 3. Hence, 24 is a composite number. night, Sanjay did 1/3 of the remaining homework on Tuesday
The smallest composite number is 4. night. What fraction of the original homework would Sanjay have
Whole Numbers : The natural numbers along with zero (0), form to do on Wednesday night to complete the Biology assignment ?
the system of whole numbers. (a) 1/15 (b) 2/15
It is denoted by W. (c) 4/15 (d) 2/5
There is no largest whole number and Sol. (c) Remaining homework on Monday night
The smallest whole number is 0.
3 2
The number line : The number line is a straight line between =1– =
negative infinity on the left to positive infinity on the right. 5 5
1 2 2
-4 -3 -2 -1 0 1 2 3 4 of =
Work done on Tuesday night =
3 5 15
Example 2 : Find three rational numbers between 3 and 5.
Remaining homework to complete the biology
3+ 5 8
Sol. 1st rational number = = =4 2 2 6-2 4
2 2 assignment = - = =
5 15 15 15
NUMBER SYSTEM & SIMPLIFICATION B-3
Example 6 : Example 8 : Is 473312 divisible by 7?
(a) Write 21.3751 upto two places of decimal. Sol. 47331 – 2 × 2 = 47327
(b) Write 3.27645 upto three places of decimal. 4732 – 2 × 7 = 4718
471 – 2 × 8 = 455
Sol. (a) 21.3751 = 21.38
45 – 2 × 5 = 35
(b) 3.27645 = 3.276
35 is divisible by 7, therefore, 473312 is divisible by 7.
DE CIMA LS Example 9 : What is the value of M and N respectively if
1. The decimal expansion of a rational number is either M39048458N is divisible by 8 and 11, where M and N are single
terminating or non-terminating recurring. More over , a digit integers?
number whose decimal expansion is terminating or non- (a) 7, 4 (b) 8, 6
terminating recurring is rational. (c) 6, 4 (d) 3, 2
2. The decimal expansion of an irrational number is non- Sol. (c) A number is divisible by 8 if the number formed by the
terminating non recurring. Moreover, a number whose last three digits is divisible by 8.
decimal expansion is non-terminating non recurring is i.e., 58N is divisible by 8.
irrational. Clearly,N = 4
Again, a number is divisible by 11 if the difference
For example : 2 = 1.41421356237309504880... between the sum of digits at even places and sum of
p = 3.1415926535897932384626433... digits at the odd places is either 0 or is divisible by 11.
1 2 i.e. (M + 9 + 4 + 4 + 8) - (3 + 0 + 8 + 5 + N)
Example 7 : Find an irrational number between and .
7 7 = M + 25 - (16 + N)
1 2 = M - N + 9 must be zero or it must be divisible by 11
Sol. We find by dividing, = 0.142857 and = 0.285714 .
7 7 i.e. M - N = 2
Þ M = 2+4 = 6
DIVISIBILIT Y RULES
Hence, M = 6, N = 4
Divisibility by 2 : A number is divisible by 2 if its unit’s digit is
DIVISIONALGORITHM :
even or 0.
Dividend = (Divisor × Quotient) + Remainder
Divisibility by 3 : A number is divisible by 3 if the sum of its digits
where, Dividend = The number which is being divided
are divisible by 3.
Divisor = The number which performs the division process
Divisibility by 4 : A number is divisible by 4 if the last 2 digits are Quotient = Greatest possible integer as a result of division
divisible by 4, or if the last two digits are 0’s. Remainder = Rest part of dividend which cannot be further divided
Divisibility by 5 : A number is divisible by 5 if its unit’s digit is by the divisor.
5 or 0.
Divisibility by 6 : A number is divisible by 6 if it is simultaneously Example 10 : A certain number when divided by 899 leaves the
divisible by 2 and 3. remainder 63. Find the remainder when the same number is divided
Divisiblity by 7 : A number is divisible by 7 if unit’s place digit is by 29.
multiplied by 2 and subtracted from the remaining digits and the (a) 5 (b) 4
number obtained is divisible by 7. (c) 1 (d) Cannotbe determined
Divisibility by 8 : A number is divisible by 8 if the last 3 digits of Sol. (a) Number = 899Q + 63, where Q is quotient
the number are divisible by 8, or if the last three digits of a number = 31 × 29 Q + (58 + 5) = 29 [ 31Q + 2] + 5
are zeros. \ Remainder = 5
Divisibility by 9 : A number is divisible by 9 if the sum of its digits
Important Algebraic formulae
is divisible by 9.
Divisibility by 10 : A number is divisible by 10 if its unit’s digit is 0. 1. (a + b)2 = a2 + 2ab + b2
Divisibility by 11 : A number is divisible by 11 if the sum of digits 2. (a – b)2 = a2 – 2ab + b2
at odd and even places are equal or differ by a number divisible 3. (a – b) (a + b) = a2 – b2
by 11. 4. (a + b)2 + (a – b)2 = 2(a2 + b2)
Divisibility by 12 : A number is divisible by 12 if the number is 5. (a + b)2 – (a – b)2 = 4ab
divisible by both 4 and 3. 6. (a + b)3 = a3 + b3 + 3ab (a + b)
Divisibility by 13 : A number is divisible by 13 if its unit’s place = a3 + 3a2b + 3ab2 + b3
digit is multiplied by 4 and added to the remaining digits and the 7. (a – b) = a3 – 3a2b + 3 ab2 – b3
3
number obtained is divisible by 13. = a3 – b3 – 3ab (a – b)
Divisibility by 14 : A number is divisible by 14 if the number is 8. a + b = (a + b) (a2 – ab + b2)
3 3
divisible by both 2 and 7. 9. a3 – b3 = (a – b) (a2 + ab + b2)
Divisibility by 15 : A number is divisible by 15 if the number is 10. a3 + b3 + c3 – 3abc = (a + b + c) (a2 + b2 + c2 – ab – bc – ca)
divisible by both 3 and 5. 11. a3 + b3 + c3 = 3abc, if a + b + c = 0
B-4 NUMBER SYSTEM & SIMPLIFICATION
Simplification : Algebraic expressions contain alphabetic Example 12 : What value should come in place of the question
symbols as well as numbers. The process to find the value of mark (?) in the following question ?
given expression is called simplification.
2 4
Rule of Simplification : To solve the expression we use a rule ‘V- 6 ÷ 4 =?
BODMAS’. 3 5
1 7
Symbol Full name Sign of the symbol (a) 1 (b) 1
3 18
V Viniculam – (Bar)
12 5
B Bracket ( ), { }, [ ] (c) 1 (d) 1
O Of (as multiply) 19 8
D Division ÷ 20 24 = 20 ´ 5 = 25 = 1 7
Sol. (b) ?= ¸
M Multiplication × 3 5 3 24 18 18
A Addition + Example 13 : What value should come in place of the question
S Subtraction –
mark (?) in the following question ?
Before applying the ‘BODMAS’ rule, we simplify the expression
under the viniculam. After removing the brackets, we must the 3 5 2
of of of 1680 = ?
operations in the given order. 8 7 5
LAW OF SURDS: (a) 150 (b) 180
æ 1ö
n (c) 210 (d) 240
1 1 1
v çan ÷ = a v 2 5 3
çè ÷ø a n b n = (ab) n Sol. (b) ? = 1680 ´ ´ ´ =180
5 7 8
1 4´ 2 + 6
æ 1n ö m 1
Example 14 : =?
v a = a mn 5 ´ 16 - 2
çè ÷ø
16
LAW OF INDICES: (a) 5 (b)
35
v am × an = am+n v am ÷ an = am–n 1 7
1 (c) (d)
(am)n = amn 5 39
v v am =
m
a
1 4´ 2+ 6 8 + 6 14 7
v a–m = m v am / n = n
am
Sol. (d) ? = = =
5 ´ 16 - 2 80 - 2 78 39
=
a
v a0 = 1
0.827 ´ 0.827 - 0.173 ´ 0.173
Example 15 : The solution of is
Addition and subtraction of Surds 0.654
Example: 5 2 + 20 2 - 3 2 = 22 2 (a) 1 (b) 2
(c) 3 (d) 4
Example: 45 - 3 20 + 4 5 = 3 5 - 6 5 + 4 5 = 5
0.827 ´ 0.827 - 0.173 ´ 0.173
Example 11 : What value should come in place of the question Sol. (a)
0.654
mark (?) in the following question ?
432×66 –1562 = ? (0.827)2 - (0.173)2
Þ
(a) 23450 (b) 24360 0.654
(c) 25890 (d) 26950 (0.827 + 0.173) (0.827 - 0.173)
Þ
0.654
Sol. (d) ? = 432 × 66 – 1562 = 28512 – 1562 = 26950
[\ a 2 - b2 =(a + b) (a - b)]

(1) (0.654)
Þ =1
0.654
NUMBER SYSTEM & SIMPLIFICATION B-5

EXERCISE
Directions (Qs. 1 - 10): What will come in place of the question 11. What should come in the place of question mark (?) in the
mark (?) in the following questions? following equation?
1. 49 × 49 × 49 × 49 = 7? 8265 + 2736 + 41320 = ?
(a) 4 (b) 7 (a) 51321 (b) 52231
(c) 8 (d) 16 (c) 52321 (d) 52311
(e) None of these (e) None of these
2. (500 + 200) × 4 × (3 + 2) = ? 12. What should come in place of the question mark (?) in the
(a) 2902 (b) 14000 following equation?
(c) 8402 (d) 16800
(e) None of these (7´?) 2
< 81
3. 16.02 × 0.001 = ? 49
(a) 0.1602 (b) 0.001602 (a) 9 (b) 2
(c) 1.6021 (d) 0.01602 (c) 3 (d) 4
(e) None of these (e) None of these
? 60.5 13. What approximate value will come in place of the question
4. =
50 ? mark (?) in the following equation?
(a) 55 (b) 1512.5 625.04 ´16.96 + 136.001 ¸ 17 = ?
(c) 52.5 (d) 57.5 (a) 418 (b) 441
(e) None of these (c) 425 (d) 433
5. 5400 ÷ 9 ÷ 3 = ? (e) 449
(a) 1800 (b) 900
14. What approximate value should come in place of the
(c) 450 (d) 300
question mark?
(e) None of these
48.25 × 150 + 32 × 16.5 – 125 × 10.5 = ?
1 1 2 (a) 6200 (b) 7500
6. + + =?
5 7 3 (c) 6450 (d) 7100
(e) 6700
1 104
(a) 1 (b) 15. What approximate value should come in place of the
105 105 question mark (?).
1 36.0001 ÷ 5.9998 × ? = 108.0005
(c) 1 (d) 1 2
7 105 (a) 325 (b) 316
(e) None of these (c) 256 (d) 16
(e) 18
7. 10150 ÷ 10146 = ? 16. What should come in place of question mark (?)?
(a) 106 (b) 100000 138.009 + 341.981 – 146.305 = 123.6 + ?
(c) 1000 (d) 10000 (a) 210.85 (b) 120.85
(e) None of these (c) 220.085 (d) 120.085
8. 200 + 5 × 4 = ? (e) None of these
(a) 820 (b) 202 17. What should come in place of question mark (?) in the
(c) 420 (d) 209 following equation ?
(e) None of these
197 × ? + 16 2 = 2620
9. ?% of 360 = 129.6
(a) 22 (b) 12
(a) 277 (b) 36
(c) 14 (d) 16
(c) 64 (d) 72
(e) None of these
(e) None of these
18. What approximate value should come in place of question
( )
2
10. 49 =? mark (?) in the following equation
287.532 + 1894.029 – 657.48 = 743.095 + ?
(a) 7 (b) 14 (a) 870 (b) 790
(c) 49 (d) 21
(c) 780 (d) 770
(e) None of these
(e) 890
B-6 NUMBER SYSTEM & SIMPLIFICATION
19. What should come in place of question mark (?) in the 26. What should come in place of the question mark(?) in the
following equation ? following equation?
3 2 5 6 1 1 1 2
27 + 118 – 32 = 11 + ? 4 ´ 4 -8 ¸5 = ?
11 5 22 11 2 3 3 3

9 9 1
(a) 113 (b) 111 (a) 8 (b) 18
10 11 34
33 7
9 9 (c) 1 (d)
(c) 90 (d) 101 34 17
10 11
(e) None of these
(e) None of these
27. What approximate value should come in place of the
20. What should come in the place of the question mark(?) in
question mark (?) in the following equation?
the following equation?
85.147 + 34.912 × 6.2 + ? = 802.293
21 9 5 10 (a) 400 (b) 450
¸ ´ ¸ =? (c) 550 (d) 600
25 20 12 17
77 9 (e) 500
(a) 7 (b) 11 28. What should come in place of the question mark (?) in the
125 10
following equation?
119 29 9548 + 7314 = 8362 + ?
(c) (d) 1
450 90 (a) 8230 (b) 8500
(e) None of these (c) 8410 (d) 8600
21. What should come in the place of the question mark(?) in (e) None of these
the following equation? 29. What approximate value should come in place of the
69012 – 20167 + (51246 ¸ 6) = ? question mark (?) in the following equation?
(a) 57385 (b) 57286 248.251 ÷ 12.62 × 20.52 = ?
(c) 57476 (d) 57368 (a) 400 (b) 450
(e) None of these (c) 600 (d) 350
(e) 375
22. What approximate value should come in the place of
30. What proximate value should come in place of the question
question mark(?) in the following equation?
mark (?) in the following question?
98.98 ÷ 11.03 + 7.014 × 15.99 = (?)2
6.595 × 1084 + 2568.34 – 1708.34 = ?
(a) 131 (b) 144 (a) 6,000 (b) 12,000
(c) 12 (d) 121 (c) 10,000 (d) 8,000
(e) 11 (e) 9,000
23. What approximate value should come in place of the 31. What should come in place of the question mark (?) in the
question mark (?) in the following equation? following equation?
39.05 × 14.95 – 27.99 × 10.12 = (36 + ?) × 5 5679 + 1438 – 2015 = ?
(a) 22 (b) 29 (a) 5192 (b) 5012
(c) 34 (d) 32 (c) 5102 (d) 5002
(e) 25 (e) None of these
24. What approximate value will come in place of the question 32. What should come in place of the question mark (‘?) in the
mark(?) in the following equation? following equation?
2070.50 ÷15.004 + 39.001 × (4.999)2 =? 2
18 of 150.8 +? = 8697.32 – 3058.16
(a) 1005 (b) 997 5
(c) 1049 (d) 1213 (a) 2764.44 (b) 2864.34
(e) 1113 (c) 1864.44 (d) 2684.44
25. What should come in the place of the question mark(?) in (e) None of these
the following equation? 33. What should come in place of the question mark in the
following questions?
452 ´ 272
=? ? 72
1352 <
24 ?
(a) 81 (b) 1 (a) 12 (b) 16
(c) 243 (d) 9 (c) 114 (d) 144
(e) None of these (e) None of these
NUMBER SYSTEM & SIMPLIFICATION B-7
34. What should come in place of the question mark (?) in the 42. What should come in place of question mark (?) in the
following equation? following equation?
5 1 2 1 3 11 1
6 × 5 + 17 ´ 4 = ? 5 ¸3 +5 =?
6 3 3 2 5 15 2

1 1
(a) 112 (b) 663 (a) 7 (b) 8
3 2

2 1 1
(c) 7 (d) 6
(c) 240 (d) 116 2 2
3
(e) None of these
(e) None of these
35. What approximate value should come in place of the
43. What should come in place of the question mark (?) in the
question mark (?) in the following equation?
following equation?
35% of 1478 + 29% of 3214 = ?
5978 + 6134 + 7014 = ?
(a) 1600 (b) 1250
(a) 19226 (b) 16226
(c) 1300 (d) 1450
(c) 19216 (d) 19126
(e) 1500
(e) None of these
36. What approximate value should come in place of the
question mark (?) in the following equation ? 44. What should come in place of the question mark (?) in the
following equation?
5 9568 – 6548 – 1024 = ?
of 1596 + 3015 = ? – 2150
7 (a) 2086 (b) 1996
(a) 7200 (b) 48000 (c) 2293 (d) 1896
(c) 5300 (d) 58000 (e) None of these
(e) 6300 45. What approximate value should come in place of the
37. In the following equation what value would come in place question mark (?) in the following equation?
of question mark (?)? 1.542 × 2408.69 + 1134.632 = ?
5798 – ? = 7385 – 4632 (a) 4600 (b) 4800
(a) 3225 (b) 2595 (c) 5200 (d) 6400
(c) 2775 (d) 3045 (e) 3600
(e) None of these above 46. What approximate value should come in place of the
38. What approximate value should come in place of the question mark (?) in the following equation?
question mark (?) in the following equation? 8.539 + 16.84 × 6.5 ÷ 4.2 = ?
152 ? + 795 = 8226 – 3486 (a) 25 (b) 42
(a) 425 (b) 985 (c) 44 (d) 35 (e) None of these
(c) 1225 (d) 1025 47. What should come in place of the question mark (?) in the
(e) 675 following equation?
39. What approximate value should come in place of the 2 1
question mark(?) in the following equation? 17 of 180 + of 480 = ?
3 4
2 (a) 3180 (b) 3420
6.39 × 15.266 + 115.8 of =?
5 (c) 3200 (d) 3300
(a) 145 (b) 165 (e) None of these
(c) 180 (d) 130 48. What approximate value should come in the place of
(e) 135 question mark (?) in the following equation?
40. What should come in place of question mark(?) in the 3
following equation? 1325 17 + 508.24 of 20% – 85.39 of 4 = ?
8597 – ? = 7429 – 4358
(a) 5500 (b) 5200
(a) 5706 (b) 5526
(c) 5426 (d) 5626 (c) 5800 (d) 4900
(e) None of these (e) 5900
41. What approximate value should come in place of question 49. What should come in place of question mark (?) in the
mark (?) in the following equation? following equation?
of 857 of 14% – 5.6 × 12.128 = ? 45% of 1500 + 35% of 1700 = ?% of 3175.
(a) 48 (b) 36 (a) 50 (b) 45
(c) 60 (d) 52 (c) 30 (d) 35
(e) 46 (e) None of these
B-8 NUMBER SYSTEM & SIMPLIFICATION
50. What approximate value should come in place of the 57. Four of the following five numbered parts (a), (b), (c), (d)
question mark (?) in the following equation’? and (e) are equal. The number of the part which is not equal
to the other four parts is the answer.
3
3 of 157.85 + 39% of 1847 = ? – 447.30 (a) 115 × 8 ÷ 10 + 8 (b) 425 ÷ 17 × 4
5
(c) 36 × 5 ÷ 6 + 17 × 4 (d) 26 + 256 + 20
(a) 1200 (b) 1500
(e) 35 × 12 ÷ 14 + 14 × 5
(c) 1600 (d) 1800
58. Four of the five parts numbered (a), (b), (c), (d) and (e) in the
(e) 2100
following equation are exactly equal. Which of the parts is
51. What should come in place of the question mark ( ?) in the not equal to the other four? The number of that part is the
following equation? answer.
477.5 ¸ 473/ 2 ´ 47,3 < ( 47) ? (a) 45 × 120 + 52 × 10
(b) 113 × 25 × 2
1 (c) 27 × 25 × 8 + 15 × 6 + 4 × 40
(a) 3 (b) 2 (d) 226 × 5 + 113 × 45 =
2
(e) 502 × 2 + 13 × 50
(c) 6 (d) 3.5
59. Four of the following five parts numbered (a), (b), (c), (d)
(e) None of these
and (e) are exactly equal. The number of the part which is
52. What approximate value should come in place of question not equal to the other four parts is your answer.
mark (?) in the following equation? (a) 85 ¸ 17 ´110 (b) 45 × 6 + 75 × 4
1 (c) 175 ¸ 25 ´ 75 + 52 (d) 36 × 4 + 212 – 7 × 5
33 % of 768.9 + 25% of 161.2 – 68.12 = ?
3 (e) 65 × 12 – 46 × 5
(a) 230 (b) 225 60. Four of the five parts numbered (a), (b), (c), (d) and (e) in the
(c) 235 (d) 220 following sequence are exactly equal. Which part is not
(e) 240 equal to the other four? The number of that part is the
53. Four of the following five parts (a), (b), (c), (d) and (e) are answer.
exactly equal. The number of the part which is not equal to 2
the other four parts is your answer. (a) 120 × 12 – 22 × 20 (b) 10% of 5000 + of 1200
5
(a) 30 × 14 ÷ 7 × 5 (b) 103 – 100 × 7
(c) 80 × 40 – 20 × 110 (d) 8640 ÷ 60 + 53.5 × 16
(c) 5 ´ 3600 (d) 450 ÷ 50 × 50 – 5 × 30 (e) 5314 – 3029 – 1285
(e) 10 × 3 + 120 × 2 61. Four of the five parts numbered (a), (b), (c), (d) and (e) are
54. Four of the five parts numbered a, b, c, d and e in the exactly equal. Which of the parts is not equal to the other
four? The number of that part is the answer.
following equation are exactly equal. Which of the parts is
not equal to the other four? The number of that part is your (a) 16.80 × 4.50 + 4.4 (b) 1600 ÷ 40 + 16 × 2.5
answer. (c) 5.5 × 8.4 + 34.6 (d) 1620 ÷ 20 – 1
(e) 1856.95 – 1680.65 – 96.3
(a) 10.36 + 69.802 + 24.938
62. Four of the five parts numbered (a), (b), (c), (d) and (e) are
2 exactly equal. Which of the parts is not equal to the other
(b) 2207.1 ÷ 21 (c) 16 % of 630.6 four? The number of that part is the answer.
3
1
1 1 (a) 40% of 160 + of 240
(d) 32.84375 × 3.2 (e) of of 4729.4 3
5 9
(b) 120% of 1200
55. Four of the following five parts numbered (a), (b), (c), (d) (c) 38 × 12 – 39 × 8
and (e) are exactly equal. The number of the part which is (d) 1648 – 938 – 566
not equal to the other four parts is your answer.
(a) 2x2 – 3xy + y2 (b) (2x + y) (x – y) 1
(e) 6 of 140 – 2.5 × 306.4
2
(c) (x – y) + x(x – y) (d) (2x – y)2 – x(2x – y) 2
(e) (2x – y)2 – 2x2 + xy 63. Four of the five parts numbered (a), (b), (c), (d) and (e) are
56. Four of the following five parts numbered (a), (b), (c), (d) exactly equal. Which of the parts is not equal to the other
and (e) are exactly equal. The number of the part which is four? The number of that part is the answer.
not equal to the other four parts is your answer. (a) 732.534 + 412.256 – 544.29
(b) 1256.214 – 355.514 – 300.2
(a) 75 × 8 ÷ 6 (b) 98 ÷ 2.5 + 15.2 × 4
(c) 246.86 + 439.38– 80.74
(c) 225 ´ 23 – 5 × 22 (d) 76 × 1.5 – 5.5 × 2.6 (d) 1415.329 + 532.4 – 1347.229
(e) 48 × 1.2 + 127.2 ÷ 3 (e) 398.14 – 239.39 + 441.75
NUMBER SYSTEM & SIMPLIFICATION B-9
64. Four of the five parts numbered (a), (b), (c), (d) and (e) in the 71. 18 × 3 ÷ 2 + 3 < 27?
following equation are exactly equal. Which of the parts is (a) s followed by p (b) p followed by v
not equal to the other four? The number of that part is the (c) s followed by v (d) s followed by w
answer. (e) None of these
(a) 371.587 + 46.32 – 217.907 72. (48 + 9) ÷ 19 × 2 = 12?
(b) 4 × 125 – 75 × 4 (a) p followed by w (b) q followed by t
(c) 58.25 × 4.5 – 65.875 (c) r followed by p (d) p followed by s
(d) 25 × 12 – 2 × 5 × 10 (e) None of these
(e) 121 × 3.5 – 2 × 111.75 73. What will come in place of the question mark (?) in the
65. Four of the five parts numbered (a) , (b) , (c) , (d) and (e) in following equation?
the following equation are exactly, equal. Which of the parts 257.5 × 52.5 ÷ 1251.5 = 5?
is not equal to the other four? The number of that part is the (a) 16 (b) 17.5
answer. (c) 8.5 (d) 13
(a) 375.789 + 41.28 – 115.249 (e) None of these
(b) 6.45 × 120.8 – 477.34 74. Four of the five parts numbered (a), (b), (c), (d) and (e) in the
(c) 1015.71 – 738.416 + 24.526 following equation are exactly equal. You have to find out
(d) 853.12 + 109.73 – 661.03 the part that is not equal to the other four. The number of
(e) 132.8 × 3.5 – 152.98 that part is the answer.
66. Which of the following is the highest fraction? (a) 7529.0 – 6 (1110.555) (b) 593.27 – 167.20 + 439.60
(c) 490.92 + 439.65 – 64.9 (d) (7189.3 – 2860.93) + 5
5 3 (e) 2 (269.40 + 163.435)
(a) (b)
7 4 75. Four of the following five parts numbered (a), (b), (c), (d)
and (e) in the following equation are exactly equal. Which
2 6
(c) (d) of the parts is not equal to the other four? The number of
3 7 that part is the answer.
7 (a) xy2 – x2y + 2x2y2
(e) (b) xy2 (1 – 2x) + x2y (2y – 1)
8
(c) xy2 (1 + x) – x2y (1 – y)
4 3 5 (d) xy [y (1 + x) – x (1 – y)]
67. If of of of a number is 45, what is the number?
9 10 8 (e) xy (y – x + 2xy)
(a) 450 (b) 550 76. Four of the five parts numbered (a), (b), (c), (d) and (e) in the
(c) 560 (d) 650 following equation are exactly equal. Which of the parts is
not equal to the other four? The number of that part is the
(e) None of these
answer.
Directions (Qs. 68 -72) : Following p,q,r,s,t,u,v,w are combinations
(a) (a – b)2 + 4ab (b) a (a + b) + b (a + b)
of an operation and an operand.
(c) (a2 – b2) + 2b (b + a) (d) (a – b) b + (a + b) 2b
(p) means ÷ 3 (q) means × 3
(e) (a + b)2 – 2a (a – b) + a (2a – 2b)
(r) means – 3 (s) means + 3
77. Four of the five parts numbered as (a), (b), (c), (d) and (e)
(t) means ÷ 2 (u) means × 2
in the following equations are exactly equal. You have to
(v) means – 2 (w) means + 2 find out the part that is not equal to the other parts. The
You have been given one or more of these as answer choices number of that part is the answer.
for the following questions. Select the appropriate choice (a) a2 + 5ab + 6b2
to replace the question mark in the equations.
(b) (a + 3b) (a + 2b)
68. 42 × 21 – 12 ? = 880 (c) a(a + 3b) + 2b(a + 3b)
(a) p (b) u (d) (3a + b) (a + 2b)
(c) v (d) r (e) a(a + 2b) + 3b(a + 2b)
(e) None of these 78. Four of the five parts numbered (a) , (b) , (c) (d) and (e) in
69. 36 + 12 ? = 48 the following equation are exactly equal. Which of the parts
(a) p followed by u (b) p followed by q is not equal to the other four? The number of that part is the
(c) q followed by u (d) r followed by p answer.
(e) None of these (a) 2a2 – ab + 3b2
70. 48 ? + 12 × 4 = 80 (b) (a – b) (2a – 3b) + 4ab
(a) t followed by q (b) s followed by p (c) (a – b)2 + (a2 + ab + 2b2)
(c) w followed by p (d) q followed by u (d) (a + b)2 + (a2 + 2b2 – 3ab)
(e) None of these (e) (a + b) (a – b) + (2a2 – ab + 4b2)
B-10 NUMBER SYSTEM & SIMPLIFICATION
79. What approximate value should come in place of the 82. Four of the five parts numbered (a), (b), (c), (d) and (e) in the
question mark (?) in the following equation? following equation are exactly equal. Which part is not equal
to the other four ? The number of that part is the answer.
16 524 ∗1492 , 250.0521 < ?
(a) (x + 2y) (3x + 2y)
(a) 1600 (b) 1800 (b) (x + 2y)2 + 2x (x + 2y)
(c) 1900 (d) 2400 (c) (3x + 2y)2 – 2x (3x + 2y)
(e) 1400 (d) 3x (x + 2y) + 2y (x + 2y)
80. Four of the following five parts lettered a, b, c, d and e are (e) x (3x + 2y) – 2y (3x – 2y)
exactly equal. Which of the following is not equal to the
83. Four of the five parts numbered (a), (b), (c), (d) and (e) are
other four?
exactly equal. Which of the parts is not equal to the other
(a) 242 – 122 + 112 ÷ 14 (b) 17 × 12 + 59 × 4 four? The number of that part is the answer.
(c) 15 × 28 + 20 (d) 27 × 16 + 56 ÷ 8 (a) (a + b) (a – 2b) (b) a2 – b (a + 2b)
(e) 185 × 6 ÷ 2 – 23 × 5 (c) a2 + b2– ab – 3b2 (d) (a + b) (a–b) + ab
81. Four of the five parts numbered (a), (b), (c), (d) and (e) are (e) (a – b)2 + b (a – 3b)
exactly equal. Which of the parts is not equal to the other
84. Four of the five parts numbered (a), (b), (c), (d) and (e) in the
four? The number of that part is the answer.
following equation are exactly equal. Which part is not equal
(a) (2x + 3y)2 to the other four? The number of that part is the answer.
(b) (2y – x)2 + y (16x + 5y) (a) x (x + y)2 – 2x2y (b) x (x + y)2 + 2x2y
(c) 4x (x + 2y) + y (4x + 9y) (c) x (x2 – y2 ) (d) x [(x + y)2 – 2xy]
(d) (2x + 2y)2 + y (4x + 5y) (e) x [(x + y)2 – 2xy2]
(e) (2x – y)2 + 8y (2x + y) 85. Which of the following has fractions in ascending order?

2 3 1 4 5 6 1 2 3 5 4 6
(a) , , , , , (b) , , , , ,
5 5 3 7 6 7 3 5 5 6 7 7

1 2 3 4 5 6 1 2 4 3 5 6
(c) , , , , , (d) , , , , ,
3 5 5 7 6 7 3 5 7 5 6 7
(e) None of these

ANSWER KEY
1 (c) 10 (c) 19 (e) 28 (b) 37 (d) 46 (d) 55 (b) 64 (c) 73 (d) 82 (e)
2 (b) 11 (c) 20 (d) 29 (a) 38 (e) 47 (d) 56 (d) 65 (e) 74 (d) 83 (d)
3 (d) 12 (c) 21 (e) 30 (d) 39 (a) 48 (a) 57 (c) 66 (e) 75 (b) 84 (e)
4 (a) 13 (d) 22 (e) 31 (c) 40 (b) 49 (e) 58 (d) 67 (e) 76 (d) 85 (d)
5 (e) 14 (c) 23 (e) 32 (e) 41 (d) 50 (d) 59 (b) 68 (e) 77 (d)
6 (a) 15 (e) 24 (e) 33 (d) 42 (a) 51 (c) 60 (b) 69 (b) 78 (e)
7 (d) 16 (e) 25 (a) 34 (e) 43 (d) 52 (a) 61 (c) 70 (c) 79 (a)
8 (e) 17 (b) 26 (b) 35 (d) 44 (b) 53 (e) 62 (b) 71 (d) 80 (d)
9 (b) 18 (c) 27 (e) 36 (e) 45 (b) 54 (e) 63 (c) 72 (a) 81 (b)
NUMBER SYSTEM & SIMPLIFICATION B-11

Answers &
Explanations
1. (c) 49 × 49 × 49 × 49 = 7?
2620 - 256
or, 72 × 72 × 72 × 72 = 7? 17. (b) ? = = 12
or 78 = 7? or, ? = 8 197
2. (b) (500 + 200) × 4 × (3 + 2) = ? 18. (c) ? = 285 + 1895 – 655 – 745 or, ? = 780
or, ? = 700 × 4 × 5 = 14000
æ 3 2 5 6ö
3. (d) 16.02 × 0.001 = ? 19. (e) ? = (27 + 118 – 32 – 11) + ç + - - ÷
è 11 5 22 11ø
or, ? = 0.01602
? 60.5 æ 30 + 44 - 25 - 60 ö
4. (a) = or, ? = 102 + ç ÷ø
50 ? è 110
or, ? = 50 × 60.5 or, ?2 = 3025
2
1 9
or, ? = 3025 = 55 or, ? = 102 – = 101
10 10
5. (e) 5400 ÷ 9 ÷ 3 = ?
21 20 5 17 119 29
1 20. (d) ?= ´ ´ ´ < <1
or, ? = 5400 ´ = 200 25 9 12 10 90 90
9´3
51246
1 1 2 21. (e) ? = 48845 + = 48845 + 8541 = 57386
6. (a) + + =? 6
5 7 3
22. (e) 98.98 ¸ 11.03 + 7.014 × 15.99 = (?)2
21 + 15 + 70 106 1 Suppose ? = x
or, ? = = =1
105 105 105 Then 99 ÷ 11 + 7 × 16 + » 121
7. (d) 10150 ÷ 10146 = ? (taking approximate value)
10150 \ x = 11
or, ? = = 10150–146 i.e., 23. (e) Solve using approximation
146
10
24. (e) 2070.50 ¸ 15.004 + 39.001 × (4.999)2 = ?
é mx x- y
ù or ? » 2070 ¸ 15 + 39 × 5 × 5
êQ y = m ú
= 138 + 975 = 1113
ëê m ûú
= 104 = 10000 45´ 45´ 27 ´ 27
25. (a) ? = = 81
8. (e) 200 + 5 × 4 = ? 135´135
or, ? = 200 + 20 = 220
9. (b) ?% of 360 = 129.6 9 13 25 3
26. (b) ? = ´ , ´
2 3 3 17
129.6 ´ 100
or, ? = = 36
360 39 25 663 , 50 613 1
< , < < < 18
2 17 34 34 34
( )
2
10. (c) 49 = ? or, ? = 49
27. (e) 85.147 + 34.912 × 6.2 + ? = 802.293
13. (d) ? = 625.04 ´16.96 + 136.001 ¸ 17 » 25 ´ 17 + 8 or, ? = 802.293 – 85.147 – 34.912 × 6.2
= 425 + 8 = 433 » 800 – 85 – 35 × 6 » 500
28. (b) 9548 + 7314 = 8362 + ?
36 or, ? = 9548 + 7314 – 8362 = 8500
15. (e) Using approximation, we get ´ ? = 108
6 29. (a) 248.251 ÷ 12.62 × 20.52 = ?
108 or, ? » 240 ÷ 12 × 20 = 20 × 20 = 400
or, ?=
6 30. (d) ? » 6.6 × 1080 + 2560 –1700 » 7128 + 860 » 8000
31. (c) ?=5679 + 1438 – 2015 = 5102
or, ? = 18
or, ? = 324 » 325. 92
32. (e) ? = 8697.32 – 3058.16 – ´ 150.8
16. (e) ? = 138.009 + 341.981 – 146.305 – 123.60 5
\ ? = 210.085 = 5639.16 – 2774.72 = 2864.44
B-12 NUMBER SYSTEM & SIMPLIFICATION
49. (e) 45% of 1500 + 35% of 1700 = ? % of 3175
33. (d) ? × ? < 24´72 Squaring both the sides,
(? ×? ) = ?3 = (8 × (3) × (8 × 3) × (8 × 9) × (8 × 9)
2 127000
\ ?= < 40
= 238393 3175
\ ? = 2 × 8 × 9 = 144
18 40
50. (d) ? » ×160 + ×1850 + 450
41 16 53 9 41 ´ 16 + 53 ´ 9 ´ 3 5 100
34. (e) ?= ´ + ´ =
6 3 3 2 6´3 » 576 + 740 + 450 » 1760 » 1800
51. (c) (47)3 = [(47)1/2]6
656 + 1431 2087 17
= = = 115 1
18 18 18 52. (a) ? = 33 % of 768.9 + 25% of 161.2 – 68.12
3
35. (d) ? = 35% of 1478 + 29% of 3214
» 35% of (1500 – 22) + (30% – 1%) of 3200 1 1
= of 768.9 + of 161.2 – 68.12
» 35 × 15 – 8 + 30 × 32 – 32 3 4
= 525 + 960 – 40 = 525 + 920
= 1445 » 1450 = 256.3 + 40.3 – 68.12 » 230
36. (e) ? » 5 × 230 + 3000 + 2150 53. (e) Other parts are equal to 300.
= 1150 + 3000 + 2150 = 6300 54. (e) The other parts are equal to 105.10.
37. (d) ? = 5798 + 4632 – 7385 = 3045 55. (b) Other parts are equal to
2x2 + y2 – 3xy.
38. (e) 152 ? » 8200 – 3500 – 800 = 3900 56. (d) Check all options one by one.
3900 75 ´ 8
\ ? = (a) = 100
152 6

æ 4000 ö 98
= slightly less than çè = ÷ 26.67 i.e., 26 (b) + (15.2 × 4) = 100
150 ø 2.5
(c) (15 × 8) – (5 × 4) = 100
\ ? = (26)2 = 676 » 675 (d) (76 × 1.5) – (5.5 × 2.6) = 99.7
2 127.2
39. (a) ? = 6.39 ×15. 266 + 115.8 of (e) (48 × 1.2) + = 100
5 3
» 6.50 × 15 + 115 × 0.4 = 97.50 + 46 » 145 57. (c) All others are equal to 100. But option (c) gives a
40. (b) 8597 – ? = 7429 – 4358 value equal to 98.
58. (d) Others equal 5650.
\ ? = 5526
59. (b) Except it, others are equal to 550.
41. (d) ? = 857of 14% – 5.6 × 12.128
60. (b) The other parts are equal to 1000.
» 857of 14% – 5.6 × 12 » 120 – 67 » 52
3 11 1 28 56 11 61. (c) Others equal 80 whereas (c) equals 80.8.
42. (a) ? = 5 ¸ 3 ∗ 5 < ¸ ∗ 62. (b) Others are equal to 144 whereas (b) equals 1440.
5 15 2 5 15 2
63. (c) (c) = 605.5 whereas the other parts are equal to 600.5
28 15 11 3 11 14 64. (c) The other parts are equal to 200.
= ´ ∗ < ∗ < <7 65. (e) Others are equal to 301.82.
5 56 2 2 2 2
66. (e) Note: If the difference between the numerator and
43. (d) ? = 5978 + 6134 + 7014 = 19126 the denominator is constant, then the fraction which
44. (b) ? = 9568 – 6548 – 1024 = 1996. has larger value in the numerator is the larger fraction.
45. (b) ? » 3700 + 1100 = 4800 Using the above, among the options (a), (c), (d) and
46. (d) ? » 8.6 + 4 × 6.5 » 35 (e), (e) has the larger value. Now, compared to option
(a), (e) has the larger value.
2 1
47. (d) 17 of 180 + of 480 = ?
3 4 45 ´ 9 ´ 10 ´ 8
67. (e) Number = Þ 540
4´ 3´ 5
53 1 68. (e) 42 × 21 ? = 880
or, ? = of 180 + of 480 = 3180 + 120 = 3300
3 4 or, (42 × 21 = 882 – 880 =) 2 = 12?
3 Now, by trial, (1) ® 12 ÷ 3 = 4 ¹ 2
48. (a) ? = 1325 17 + 508.24 of 20% – 85.39 of (b) ® 12 × 2 = 24 ¹ 2;
4
(c) ® 12 – 2 = 10 ¹ 2
= 1325 17 + 500 of 20% – 85 × 0.75 (d) ® 12 + 3 = 15 ¹ 2;
= 5460 + 100 – 60 = 5500 \ answer is (e).
NUMBER SYSTEM & SIMPLIFICATION B-13
69. (b) 36 + 12? = 48 74. (d) Others are equal to 865.67.
Now, by trial 75. (b) The other parts are equal to xy2 – x2y + 2x2y2.
1 ® 36 + 12 ÷ 3 × 2 = 44 ¹ 48 76. (d) The other parts are equal to (a + b)2.
2 ® 36 + 12 ÷ 3 × 2 = 48 77. (d) The other parts are equal to a2 + 5ab + 6b2.
70. (c) 48? + 12 × 4 = 80 78. (e) The other parts are equal to 34.
Now, by trial 79. (a) ? » 1 6 × 23 + 1490 – 250 » 1600
1 ® 48 ÷ 2 × 3 + 48 = 120 ¹ 80 80. (d) Other parts are equal to 440.
2 ® 48 + 3 ÷ 3 + 48 = 97 ¹ 80 81 (b) Others are requal to 4x2 + 9y2 + 12xy.
3 ® 48 × 2 ÷ 3 + 48 = 120 82. (e) Others are equal to 3x2 + 8xy + 4y2
72. (a) (48 + 9) ÷ 19 × 2 = 12? 83. (d) Others equal a2 – ab – 2b2 whereas (d) equals
or, 57 ÷ 19 × 2 = 12? a2 + ab – b2 .
or 3 × 2 = 12? = 12 ÷ 3 + 2 = 6 84. (e) Others are x3 + xy2
73. (d) 25 × 52.5 ÷ 1251.5 = 5?
7.5
1 2 4 3
or, 52×7.5 × 52.5 ÷ 53 × 1.5 = 5? 85. (d) = 0.33; = 0.4; = 0.57 ; = 0.6;
3 5 7 5
1 5 6
or, 515 ´ 52.5 ´ 4.5
= 5? = 0.83and = 0.85
5 6 7
or, 513 = 5? or, ? = 13
B-14 AVERAGE AND PROBLEM ON AGES

Average and
Problem
2 Chapter on Ages
AV ERAG E Total age of 20 students
In the pure mathematical sense, an average is a calculated value, = 13.1 × 20 = 262 years
obtained by adding values and dividing the sum by the number \ Average age of 30 students
of values. In English, this value is called the mean. 125 + 262 387
= = = 12.9 years
sum of elements 30 30
Hence, average =
number of elements Example 5: The average weight of 24 students in a class is 35 kg.
Example 1: To find the average of 3, 5 and 7. if the weight of the teacher is included, the average weight rises
Sol. Step 1: Find the sum of the numbers. by 400 gms. Find the weight of the teacher.
3 + 5 + 7 = 15 Sol. Total weight of 24 students
Step 2: Calculate the total number. = (24 × 35) kg = 840 kg
There are 3 numbers. Total weight of 24 students and the teacher
Step 3: Finding average. = (25 × 35.4) kg = 885 kg
\ Weight of teacher
15 = (885 – 840) kg = 45 kg
=5
3
Fo rmul as to Re memb er
QUICKER METHOD T O SOLVE QU ESTIONS ON
n +1
AV ERAG E v The average of first n natural numbers =
2
v Sum of elements = average × no. of elements
v The average of the first n consecutive even numbers
Example 2: The average of marks obtained by 4 students in a
= (n + 1)
class is 65. Find the sum of marks obtained?
Sol. Here, number of students = 4 v The average of the first n consecutive odd numbers = n

Average = 65 v The average of the squares of the first n natural numbers


\ sum of marks obtained = 65 × 4 = 260 1
Example 3: If the sum of elements and average are respectively can be shown to be (n + 1) (2n + 1)
6
65 and 13, then find the number of elements.
x ( x + 1)
v The average of first n multiples of x is
.
Sum of elements 2
Sol. Number of elements = v The average of odd numbers from 1 to n is
Average
é Last odd number + 1 ù
65 êë úû
= =5 2
13
v The average of even numbers from 1 to n is
Example 4: There are 30 students in a class. The average age of
the first 10 students is 12.5 years. The average age of the next 20 é Last even number + 2 ù
êë 2 úû
students is 13.1 years. Find the average age of the whole class.
Sol. Total age of 10 students v If average of n 1 observations is x 1 and average of
n2 observations is x2, then average of total observations
= 12.5 × 10 = 125 years
n1x1 + n 2 x 2
(n1 + n2) will be .
n1 + n 2
AVERAGE AND PROBLEM ON AGES B-15

EXERCISE
1. The average of two numbers is XY. If one number is X, then 10. The average of Suresh’s marks in English and History is 55.
the other number is His average of marks in English and Science is 65. What is
Y the difference between the marks which he obtained in
(a) Y (b) History and Science?
2
(c) 2XY – X (d) X (Y – 1) (a) 40 (b) 60
(e) None of these (c) 20 (d) Data inadequate
2. The average of four consecutive odd numbers is 12.Which (e) None of the above
is the lowest odd number? 11. The average of four consecutive even numbers is one-fourth
(a) 9 (b) 3 of the sum of these numbers. What is the difference between
(c) 5 (d) Cannot be determined the first and the last number?
(e) None of these
(a) 4 (b) 6
3. The average marks scored by Ganesh in English, Science,
Mathematics and History is less than 15 from that scored (c) 2 (d) Cannot be determind
by him in English, History, Geography and Mathematics. (e) None of these
What is the difference of marks in Science and Geography 12. The average of three consecutive odd numbers is 14 more
scored by him? than one-third of the first of these numbers, what is the last
(a) 40 (b) 50 of these numbers?
(c) 60 (d) Data inadequate (a) 17 (b) 19
(e) None of these (c) 15 (d) Data inadequate
4. The average age of 24 students and the class teacher is 16
(e) None of these
years. If the class teacher’s age is excluded, the average
reduces by one year. What is the age of the class teacher? 13. A mathematics teacher tabulated the marks secured by 35
(a) 50 years (b) 45 years students of 8th class. The average of their marks was 72. If
(c) 40 years (d) Data inadequate the marks secured by Reema was written as 36 instead of 86
(e) None of these then find the correct average marks upto two decimal places.
5. There are 50 boys in a class. Their average weight is 45 kg. (a) 73.41 (b) 74.31
When one boy leaves the class, the average reduces (c) 72.43 (d) 73.43
by 100 g. Find the weight of the boy who left the class. (e) Cannot be determined
(a) 40.9 kg (b) 42.9 kg 14. The average of five consecutive odd numbers is 61. What
(c) 49.9 kg (d) 39.9 kg
is the difference between the highest and the lowest number?
6. The average age of 36 students in a group is 14 years.
When teacher’s age is included to it, the average increases (a) 8 (b) 2
by one. What is the teacher’s age in years? (c) 5 (d) Cannot be determined
(a) 31 (b) 36 (e) None of these
(c) 51 (d) cannot be determined 15. In a coconut grove, (x + 2) trees yield 60 nuts per year, x
7. The average weight of 8 persons increases by 1.5 kg. If a trees yield 120 nuts per year and (x – 2) trees yield 180 nuts
person weighing 65 kg is replaced by a new person, what per year. If the average yield per year per tree be 100, find x.
could be the weight of the new person? (a) 3 (b) 4
(a) 76 kg (b). 77 kg
(c) 5 (d) 6
(c) 76.5 kg (d) Data inadequate
(e) None of these (e) None of the above
8. N number of persons decide to raise ` 3 lakhs by equal 16. 30 pens and 75 pencils were purchased for ` 510. If the
contributions from each. If they contributed ` 50 each extra, average price of a pencil was ` 2.00, find the average price
the contribution would be ` 3.25 lakhs. How many persons of a pen.
are there? (a) ` 10 (b) ` 11
(a) 600 (b) 400 (c) ` 12 (d) cannot be determined
(c) 450 (d) Cannot be determined (e) None of the above
(e) None of these
17. A school has 4 section of Chemistry in Class X having 40,
9. The average of the first and the second of three numbers is
35, 45 and 42 students. The mean marks obtained in
15 more than the average of the second and the third of
these numbers. What is the difference between the first Chemistry test are 50, 60, 55 and 45 respectively for the 4
and the third of these three numbers? sections. Determine the overall average of marks per student
(a) 15 (b) 45 (a) 50.25 (b) 52.25
(c) 60 (d) Data inadequate (c) 51.25 (d) 53.25
(e) None of these (e) None of the above
B-16 AVERAGE AND PROBLEM ON AGES
18. The average of 20 numbers is zero. Of them, at the most, 27. The average number of printing error per page in a book of
how many may be greater than zero? 512 pages is 4. If the total number of printing error in the
(a) 0 (b) 1 first 302 pages is 1,208, the average number of printing errors
(c) 10 (d) 19 per page in the remaining pages is
(e) None of the above (a) 0 (b) 4
19. The average of six numbers is 3.95. The average of two of (c) 840 (d) 90
them is 3.4, while the average of the other two is 3.85. What (e) None of the above
is the average of the remaining two numbers? 28. The average attendance in a school for the first 4 days of
(a) 4.5 (b) 4.6 the week is 30 and for the first 5 days of the week is 32. The
(c) 4.7 (d) 4.8 attendance on the fifth day is
(e) None of the above (a) 32 (b) 40
20. Nine persons went to a hotel for taking their meals. Eight (c) 38 (d) 36
of them spent ` 12 each on their meals and the ninth spend (e) None of the above
` 8 more than the average expenditure of all the nine. What 29. The average expenditure of a labourer for 6 months was
was the total money spent by them? ` 85 and he fell into debt. In the next 4 months by reducing
(a) ` 115 (b) ` 117 his monthly expenses to ` 60 he not only cleared off his
(c) ` 119 (d) ` 122 debt but also saved ` 30. His monthly income is
(e) None of the above (a) ` 70 (b) ` 72
21. The average age of A and B is 20 years. If C were to replace (c) ` 75 (d) ` 78
A, the average would be 19 and if C were to replace B, the (e) None of the above
averge would be 21. What are the age of A, B and C? 30. The average of a batsman for 40 innings is 50 runs. His
(a) 22, 18, 20 (b) 20, 20, 18 highest score exceeds his lowest score by 172 runs. If these
(c) 18, 22, 20 (d) 21,20,19 two innings are excluded, his average drops by 2 runs. Find
(e) None of the above his highest score.
22. 3 years ago the average age of a family of 5 members was 17 (a) 172 (b) 173
years. With the birth of a new baby, the average age of six (c) 174 (d) 175
members remains the same even today. Find the age of the
(e) None of the above
new baby.
31. Last year, a Home Appliance Store sold an average
(a) 1 year (b) 2 years
(arithmetic mean) of 42 microwave ovens per month. In the
1 first 10 months of this year, the store has sold an average
(c) 1 years (d) cannot be determined
2 (arithmetic mean) of only 20 microwave ovens per month.
(e) None of the above
What was the average number of microwave ovens sold
23. The average age of a group of person going for picnic is 16 per month during the entire 22 months period ?
years. Twenty new persons with an average age of 15 years
(a) 21 (b) 30
join the group on the spot due to which their average
(c) 31 (d) 32
becomes 15.5 years. Find the number of persons initially
going for picnic. (e) None of the above
(a) 20 (b) 18 32. The captain of a cricket team of 11 players is 25 years old
(c) 22 (d) 19 and the wicket-keeper is 3 years older. If the age of these
(e) None of the above two players are replaced by that of another two players, the
24. A batsman in his 12th innings makes a score of 65 and average of the cricket team drops by 2 years. Find the
thereby increases his average by 2 runs. What is his average average age of these two players.
after the 12th innings if he had never been ‘not out’? (a) 15 years (b) 15.5 years
(a) 42 (b) 43 (c) 17 years (d) 16.5 years
(c) 44 (d) 45 (e) None of the above
(e) None of the above 33. A batsman makes a score of 87 runs in the 17th inning and
25. A pupil’s marks were wrongly entered as 83 instead of 63. thus increases his average by 3. Find his average after 17th
Due to that the average marks for the class got increased inning.
by half. The number of pupils in the class is: (a) 36 (b) 39
(a) 10 (b) 20 (c) 42 (d) 45
(c) 40 (d) 73 (e) None of the above
(e) None of the above 34. Nine men went to a hotel. 8 of them spent ` 3 each over their
26. In the first 10 overs of a cricket game, the run rate was only meals and the ninth spent ` 2 more than the average
3.2. What should be the run rate in the remaining 40 overs expenditure of all the nine. The total money spent by all of
to reach a target of 282 runs ? them was
(a) 6.25 (b) 6.50 (a) ` 26 (b) ` 40
(c) 6.75 (d) 7.00 (c) ` 29.25 (d) ` 27
(e) None of the above (e) None of the above
AVERAGE AND PROBLEM ON AGES B-17
35. The mean of 30 values was 150. It was detected on 41. A family consists of grandparents, parents and three
rechecking that one value 165 was wrongly copied as 135 grandchildren. The average age of the grandparents is 67
for the computation of the mean. Find the correct mean. years, that of the parents is 35 years and that of the
(a) 151 (b) 149 grandchildren is 6 years. What is the average age of the
family?
(c) 152 (d) 148
4 5
(e) None of the above (a) 28 years (b) 31 years
7 7
36. A cricketer whose bowling average is 12.4 runs per wicket
takes 5 wickets for 26 runs and thereby decreases his 1 1
(c) 32 years (d) 27 years
average by 0.4. The number of wickets taken by him till the 7 2
last match was: (e) None of the above
(a) 64 (b) 72 42. In Arun’s opinion, his weight is greater than 65 kg but less
(c) 80 (d) 85 than 72 kg. His brother does not agree with Arun and he
(e) None of the above thinks that Arun’s weight is greater than 60 kg but less than
70 kg. His mother’s view is that his weight cannot be greater
37. In an examination, a pupil’s average marks were 63 per paper.
than 68 kg. If all of them are correct in their estimation, what
If he had obtained 20 more marks for his Geography paper is the average of different probable weights of Arun?
and 2 more marks for his History paper, his average per (a) 67 kg (b) 68 kg
paper would have been 65. How many papers were there in (c) 69 kg (d) 66.5 kg
the examination? (e) None of the above
(a) 8 (b) 9 43. The average age of a board of 8 functional directors in a
(c) 10 (d) 11 company is the same as it was 3 years ago, a younger man
(e) None of the above having been substituted for one of the directors. How much
38. A car owner buys petrol at ` 7.50, ` 8.00 and ` 8.50 per litre younger was the new man than the director whose place he
for three successive years. What approximately is his took.
(a) 24 years (b) 26 years
average cost per litre of petrol if he spends ` 4000 each
(c) 28 years (d) 27 years
year?
(e) None of the above
(a) ` 8 (b) ` 9 44. A batsman makes a scores of 98 runs in his 19th inning and
(c) ` 7.98 (d) ` 8.50 thus increases his average by 4. What is his average after
(e) None of the above 19th inning ?
39 A batsman has scored an average of 46 runs for a certain (a) 22 (b) 24
number of innings played in England. When he came back (c) 28 (d) 26
to India, he played another two test matches of two innings (e) None of the above
each and scored at an average of 55 runs. For the innings in 45. The average weight of 45 students in a class is 52 kg. 5 of
England and in India taken together, he has improved his them whose average weight is 48 kg leave the class and
average by 2 runs over the matches played in England. other 5 students whose average weight is 54 kg join the
Find the number of innings played in England. class. What is the new average weight (in kg) of the class ?
(a) 12 (b) 13 1 2
(a) 51 (b) 52
(c) 14 (d) 15 3 3
(e) None of these 1
(c) 52 (d) 43.42
40 There were 35 students in a hostel. Due to the admission of 3
7 new students, the expenses of mess were increased by (e) None of the above
` 42 per day while the average expenditure per head 46. The average of 10 numbers is 40.2. Later it is found that two
diminished by ` 1. What was the original expenditure of the numbers have been wrongly copied. The first is 18 greater
mess? than the actual number and the second number added is 13
(a) ` 400 (b) ` 420 instead of 31. Find the correct average.
(c) ` 445 (d) ` 465 (a) 40.2 (b) 40.4
(e) None of the above (c) 40.6 (d) 40.8
(e) None of the above
B-18 AVERAGE AND PROBLEM ON AGES

ANSWER KEY
1 (c) 6 (c) 11 (b) 16 (c) 21 (a) 26 (a) 31 (d) 36 (d) 41 (b) 46 (a)
2 (a) 7 (b) 12 (d) 17 (b) 22 (b) 27 (b) 32 (b) 37 (d) 42 (d)
3 (c) 8 (e) 13 (d) 18 (d) 23 (a) 28 (b) 33 (b) 38 (c) 43 (a)
4 (c) 9 (e) 14 (a) 19 (b) 24 (b) 29 (d) 34 (c) 39 (c) 44 (d)
5 (c) 10 (c) 15 (b) 20 (b) 25 (c) 30 (c) 35 (a) 40 (b) 45 (b)

Answers &
Explanations
1. (c) Let the other number is N. 12. (d) Let the three consecutive odd numbers be
x – 2, x, x + 2 respectively.
X+N
Then, = XY Þ N = 2XY – X According to question,
2
2. (a) Let the four consecutive odd nos. be x-2
x= + 14
2x – 3, 2x – 1, 2x + 1 and 2x + 3. 3
Now, 2x = 12 or, x = 6 \ 3x – x + 2 = 42 Þ 2x = 40
Lowest odd no. = 2 × 6 – 3 = 9 \ x = 20 = an even number, which goes against our
supposition.
E+H +G+M E+S+M +H
3. (c) – = 15
4 4 35 ´ 72 + (86 - 36)
13. (d) Correct average =
Þ G – S = 60 35
4. (c) Age of the CT = 25 × 16 – 24 × 15 = 400 – 360 = 40 yrs. » 72 + 1.43 = 73.43
5. (c) Here one boy is excluded and final average of the group
14. (a) Suppose the consecutive odd numbers are : x, x + 2,
decreases.
x + 4, x + 6 and x + 8
\ change in average is (–)ve = – 0.1 kg.
Therefore, the required difference = x + 8 – x = 8
Using the formula
Note that answering the above question does not
Sum of the quantities excluded
require the average of the five consecutive odd
numbers.
æ Change in no. of quantities ö æ Change in average ö (x + 2) ´ 60 + x ´ 120 + (x - 2) ´ 180
=ç × ÷+ç × ÷ 15. (b) = 100
ç ÷ ç Final no. of quantities ÷ (x + 2) + x + (x - 2)
è Original average ø è ø
360x - 240
Þ weight of the boy who left = (1 × 45) – (– 0.1 × 49) Þ = 100
= 49.9 kg 3x
\ weight of the boy who left the class is 49.9 kg. Þ 60x = 240 Þ x = 4
6. (c) Age of the teacher = (37 × 15 – 36 × 14) years 16. (c) Since average price of a pencil = ` 2
= 51 years. \ Price of 75 pencils = ` 150
7. (b) total weight increases = 8 ´ 1.5 \ Price of 30 pens = ` (510 – 150) = ` 360
= 12 kg 360
so the weight of new person = 65 + 12 = 77 kg \ Average price of a pen = = ` 12
30
325000 - 300000 17. (b) Required average marks
8. (e) Required persons = = 500
50
40 ´ 50 + 35 ´ 60 + 45 ´ 55 + 42 ´ 45
9. (e) Set the first, second and third no be F, S and T =
40 + 35 + 45 + 42
F+S S+T
Respectively = +15 . Solving, we get F – T 2000 + 2100 + 2475 + 1890
2 2 =
162
= 30.
10. (c) E + H = (55 × 2 =) 110; 8465
E + S = (65 × 2 =) 130 = = 52.25
162
\ Reqd difference = 130 – 110 = 20
18. (d) Average of 20 numbers = 0.
11. (b) Let the four consecutive even nos. be
\ Sum of 20 numbers = (0 × 20) = 0.
2x, 2x + 2, 2x + 4 and 2x + 6 respectively.
It is quite possible that 19 of these numbers may be
Reqd difference = 2x + 6 – 2x = 6
positive and if their sum is a, then 20th number is (– a).
AVERAGE AND PROBLEM ON AGES B-19
19. (b) Sum of the remaining two numbers 30. (c) Total runs = 40 × 50 = 2000
= (3.95 × 6) – [(3.4 × 2) + (3.85 × 2)] Let his highest score be =x
= 23.70 – (6.8 + 7.7) = 23.70 – 14.5 = 9.20 Then his lowest score = x – 172
æ 9.2 ö 2000 – x - (x - 172)
\ Required average = ç ÷ = 4.6. Now = 48
è 2 ø 38
20. (b) Let the average expenditure of all the nine be ` x. Þ 2x = 2172 – 1824
Then, 12 × 8 + (x + 8) = 9x or 8x = 104 or x = 13. Þ x = 174
\ Total money spent = 9x = ` (9 × 13) = ` 117. 42 ´ 12 + 20 ´ 10 504 + 200 704
21. (a) Given A + B = 40 …(i) 31. (d) = = = 32
12 + 10 22 22
C + B = 38 … (ii)
32. (b) Let average of team = x years
A + C = 42 … (iii) Then, 25 + 28 + S9 = 11x … (i)
(i) + (ii) + (iii) Þ A + B + C = 60 …(iv) where S9 is the sum of ages of remaining players
from (i) and (iv), we get Also, Np + S9 = 11(x – 2) , … (ii)
C = 20 years where Np is the sum of ages of new players
\ B = 18 years and A = 22 years (i) – (ii) Þ 53 – Np = 22
22. (b) Sum of present ages of the six members Þ Np = 31
= (17 × 6) years = 102 years.
Sum of present ages of the 5 members (excluding baby) 31
\ Average age of new two players = = 15.5 years
= 5 × (17 + 3) years = 100 years. 2
\ Age of the baby = 102 – 100 = 2 years 33. (b) Let the average after 17th inning = x.
23. (a) Let the number of persons, initially going for Then, average after 16th inning = (x – 3).
Picnic = x \ 16(x – 3) + 87 = 17x or x = (87 – 48) = 39.
\ Sum of their ages = 16x 34. (c) Let the average expenditure of all the ninte be ` x
Then, 3 × 8 + x +2 = 9x Þ x = 3.25
16x + 15 ´ 20 \ Total money spent = 9x = 9 × 3.25 = ` 29.25
Also, = 15.5
x + 20
35. (a) Corrected mean = 150 ´ 30 – 135 + 165
Þ 0.5x = 10 Þ x = 20 years 30
24. (b) Let ‘x’ be the average score after 12 th innings 4500 – 135 + 165 4530
Þ 12 x = 11 × (x – 2) + 65 = = = 151
30 30
\ x = 43
36. (d) Let the number of wickets taken till the last match be x.
25. (c) Let there be x pupils in the class.
Then,
æ 1ö x 12.4x + 26
Total increase in marks = ç x ´ ÷ = . = 12 Þ 12.4x + 26 = 12x + 60
è 2ø 2 x+5
x x 34 340
\ = (83 - 63) Þ = 20 Þ x = 40. Þ 0.4x = 34 Þ x = = = 85.
2 2 0.4 4
26. (a) Total runs in the first 10 overs = 10 × 3.2 = 32 37. (d) Let the number of papers be x. Then, 63x + 20 + 2 = 65x
Run rate required in the remaining 40 overs or 2x = 22 or x = 11.
282 – 32 250 38. (c) Let average cost of petrol per litre be ` x
= = = 6.25 runs per over
40 40 12000
Then x =
27. (b) Remaining pages = 512 – 302 = 210 4000 4000 4000
+ +
Let average printing error in remaining pages = x 7.5 8 8.5

1208 + 210 ´ x 3 6120


Then, =4 = = = Rs 7.98 per litre
512 2 1 2 767
+ +
Þ 210x = 840 Þ x = 4 15 8 17
28. (b) Attendance on the fifth day = 32 × 5 – 30 × 4 39. (c) Let the number of innings played in England be x.
= 160 – 120 = 40 \ Total runs scord in England = 46 x
29. (d) Income of 6 months = ` (6 × 85) – debt Total runs scored for innings played in India
= ` 510 – debt = 55 × 4 = 220
Income of the man for next 4 months (Q the number of innings played in India = 4)
= ` 4 × 60 + debt + ` 30
46x + 220
= ` 270 + debt Also, = 48
\ Income of 10 months = ` 780 x+4
Average monthly income = ` 780 ÷ 10 = ` 78 Þ 46x + 220 = 48 x + 192
Þ 2x = 28 Þ x = 14
B-20 AVERAGE AND PROBLEM ON AGES
40. (b) Let the original average expenditure be ` x. Then, S - 24 S - x
Now, =
42(x – 1) – 35x = 42 Û 7x = 84 Þ x = 12. 8 8
\ Original expenditure = ` (35 × 12) = ` 420. Þ x = 24 years
Shortcut Method : If the new young director would
æ 67 ´ 2 + 35 ´ 2 + 6 ´ 3 ö
41. (b) Required average =ç ÷ have been not substituted, then total age would have
è 2+ 2+3 ø increased at present by 8 × 3 = 24 years.
æ 134 + 70 + 18 ö 222 Therefore, the new man is 24 years younger keeping
=ç ÷= the average at present same as 3 years ago.
è 7 ø 7
44. (d) Let the average score of 19 innings be x.
5
= 31 years. Then, 18x + 98 = x + 4
7 19
42 (d) Let Arun’s weight be X kg. The average score after 20th innings
According to Arun, 65 < X < 72. = x + 4 = 22 + 4 = 26
According to Arun’s brother, 60 < X < 70. 45. (b) Total weight of 45 students
According to Arun’s mother, X < 68. = 45 × 52 = 2340 kg
The values satisfying all the above conditions are 66 Total weight of 5 students who leave
and 67. = 5 × 48 = 240 kg
Total weight of 5 students who join
æ 66 + 67 ö æ 133 ö
\ Required average = ç ÷=ç ÷ = 66.5 kg. = 5 × 54 = 270 kg
è 2 ø è 2 ø
Therefore, new total weight of 45 students
43. (a) Let the new man was younger than the director = x years = 2340 – 240 + 270 = 2370
and 3 years ago, the sum of ages of board of directors
= S – 8 × 3 = S – 24 2370 2
Þ New average weight = = 52 kg
Then, 3 years ago, average age of board of directors 45 3
S - 24 46. (a) Sum of 10 numbers = 402
= Corrected sum of 10 numbers
8
= 402 – 13 + 31 – 18 = 402

Hence, new average = 402 = 40.2


10
Ratio & Proportion

3 Chapter
RATIO PROPERTIES
Ratio is strictly a mathematical term to compare two similar
quantities expressed in the same units. a c b d
1. If = then = , i.e., the inverse ratios of two equal
The ratio of two terms ‘x’ and ‘y’ is denoted by x : y. b d a c
In general, the ratio of a number x to a number y is defined as the ratios are equal. The property is called Invertendo.
quotient of the numbers x and y.
a c a b
The numerator of the ratio is called the antecedent (x) and the 2. If = then = , i.e., the ratio of antecedents and
b d c d
denominator is called the consequent (y) of the ratio.
consequents of two equal ratios are equal. This property
COMPARISON OF TWO OR MORE RATIOS is called Alternendo.
Two or more ratios may be compared by reducing the equivalent a c a +b c+d
fractions to a common denominator and then comparing the 3. If = , th en = . This property is
b d b d
magnitudes of their numerator. Thus, suppose 2 : 5, 4 : 3 and 4 : 5 called Componendo.
2 4 4
are three ratios to be compared then the fractions , and a c a -b c-d
5 3 5 4. If = , th en = . This property is
b d b d
are reduced to equivalent fractions with a common denominator.
For this, the denominator of each is changed to 15 equal to the called Dividendo.
L.C.M. their denominators Hence the given ratios are expressed
a c a +b c+d
5. If = , th en = . This property is
6 20 12 b d a -b c-d
, and or 2 : 5, 4 : 3, 4 : 5 according to magnitude.
15 15 15 called Componendo – Dividendo.

a c e
6. If = = = .............. . Then,
b d f
ê The two quantities must be of the same kind and in same
unit. sum of Numerators
Each ratio = sum of denominators
ê The ratio is a pure number, i.e., without any unit of
a c
measurement. i.e. = = a + c + e + ...
ê The ratio would stay unaltered even if both the antecedent b d b + d + f + ...
and the consequent are multiplied or divided by the same 7. If we have two equations containing three unknowns as
number.
a1x + b1y + c1z = 0 and ... (i)
Compound ratio : Ratios are compounded by multiplying a2x + b2y + c2z = 0 ... (ii)
together the antecedents for a new antecedent and the then, the values of x, y and z cannot be resolved without
consequents for a new consequent. having a third equation.
a´c However, in the absence of a third equation, we can find
The compound of a : b and c : d is , i.e., ac : bd.
b´d the proportion x : y : z.
The duplicate ratio of x : y is x2 : y2. This will be given by
The triplicate ratio of x : y is x3 : y3. b1c2 – b2c1 : c1a2 – c2a1 : a1b2 – a2b1.
The subduplicate ratio of x : y is x : y. 8. To find the ratio of the two variables of a homogeneous
3 equation of second degree.
The subtriplicate ratio of x : y is x : 3 y.
For this all the terms of the homogeneous equation are
1 1 taken on one side and factorized into linear facto` A linear
Reciprocal ratio of a : b is : or b : a
a b equation is formed from each of the factors and the ratio of
Inverse ratio the variables obtained
Inverse ratio of x : y is y : x.
B-22 RATIO & PROPORTION

If the ratio between the first and the second quantities is


a : b and the ratio between the second and third quantities
is c : d, then the ratio among first, second and third ê If four quantities are in proportion, the product of the
quantities is given by ac : bc : bd extremes is equal to the product of the means.
If the ratio between the first and the second quantities is Let a, b, c, d be in proportion, then
a : b and the ratio between the second and third quantities a c
= Þ ad = bc.
is c : d and the ratio between the third and fourth quantities b d
is e : f, then the ratio among first, second, third and fourth ê If three quantities a, b and c are in continued proportion,
quantities is given by ace : bce : bde : bdf then a : b = b : c
To divide a given quantity into a given ratio. \ ac = b2
Suppose any given quantity a, is to be divided in the ratio b is called mean proportional.
m : n. ê If three quantities are proportionals, then first is to the
Let one part of the given quantity be x then the other part third is the duplicate ratio of the first is to the second.
If a : b :: b : c then a : c = a2 : b2
will be a – x.
x m Direct Proportion :
\ = or nx = ma – mx or (m + n) x
a-x n If on the increase of one quantity, the other quantity increases
= ma to the same extent or on the decrease of one, the other decreases
ma to the same extent, then we say that the given two quantities
\ one part is and the other part will be
m+n are directly proportional. If A and B are directly proportional
ma na then we denote it by A µ B.
a- = Also, A = kB, k is constant
m+n m+n
A
In any 2-dimensional figures, if the corresponding sides Þ =k
B
are in the ratio x : y, then their areas are in the ratio
If b1 and b2 are the values of B corresponding to the values
x2 : y2.
a1, a2 of A respectively, then
a1 a 2
In any two 3-dimensional figures, if the corresponding =
b1 b2
sides are in the ratio x : y, then their volumes are in the Some Examples :
ratio x3 : y3. If the ratio between two numbers is a : b and 1. Work done µ number of men
if each number is increased by x, the ratio becomes c : d. 2. Cost µ number of Articles
Then 3. Work µ wages
4. Working hour of a machine µ fuel consumed
x (a + b) (c - d)
Sum of the two numbers = 5. Speed µ distance to be covered
ad - bc Indirect Proportion (or inverse proportion) :
x (a - b) (c - d) If on the increase of one quantity, the other quantity decreases
Difference of the two numbers =
ad - bc to the same extent or vice versa, then we say that the given
two quantities are indirectly proportional. If A and B are
xa (c - d) xb (c - d)
Two numbers are given as and 1
ad - bc ad - bc indirectly proportional then we denote it by A µ .
B
If the sum of two numbers is A and their difference is
k
a, then the ratio of numbers is given by A + a : A – a. Also, A = (k is a constant)
B
PROPORTION Þ AB = k
If b1, b2 are the values of B corresponding to the values a1,
When two ratios are equal, the four quantities composing them
a2 of A respectively, then
are said to be in proportion. a1b1 = a2b2
Some Examples :
a c
If = , then a, b, c, d are in proportions. 1. More men, less time
b d 2. Less men, more hours
This is expressed by saying that ‘a’ is to ‘b’ as ‘c’ is to ‘d’ and 3. More speed, less taken time to be covered distance
the proportion is written as PARTNERSHIP
a : b :: c : d or a : b = c : d A partnership is an association of two or more persons who
invest their money in order to carry on a certain business.
The terms a and d are called the extremes while the terms b and
A partner who manages the business is called the working
c are called the means.
partner and the one who simply invests the money is called
the sleeping partner.
RATIO & PROPORTION B-23
Partnership is of two kinds : MIXTURE
(i) Simple (ii) Compound.
Simple Mixture : When two different ingredients are mixed
Simple partnership : If the capitals is of the partners are
together, it is known as a simple mixture.
invested for the same period, the partnership is called simple.
Compound partnership : If the capitals of the partners are Compound Mixture : When two or more simple mixtures are
invested for different lengths of time, the partnership is called mixed together to form another mixture, it is known as a
compound. compound mixture.
If the period of investment is the same for each partner, Alligation : Alligation is nothing but a faster technique of
then the profit or loss is divided in the ratio of their solving problems based on the weighted average situation as
investments. applied to the case of two groups being mixed together.
If A and B are partners in a business, then The word ‘Alligation’ literally means ‘linking’.
Investment of A Pr ofit of A Loss of A
= or = Alligation rule : It states that when different quantities
Investment of B Pr ofit of B Loss of B
of the same or different ingredients of different costs are
If A, B and C are partners in a business, then mixed together to produce a mixture of a mean cost, the
Investment of A : Investment of B : Investment of C ratio of their quantities is inversely proportional to the
= Profit of A : Profit of B : Profit of C, or difference in their cost from the mean cost.
= Loss of A : Loss of B : Loss of C
Quantity of Cheaper Price of Dearer - Mean Price
=
MONTHLY EQUIVALENT INVESTMENT Quantity of Dearer Mean Price - Price of Cheaper
It is the product of the capital invested and the period for Graphical representation of Alligation Rule :
which it is invested.
If the period of investment is different, then the profit or loss Quantity Quantity
a b
is divided in the ratio of their Monthly Equivalent Investment.
Monthly Equivalent Investment of A Mean
Monthly Equivalent Investment of B
Average (d)

Pr ofit of A Loss ofA


= or d–a
Pr ofit of B Loss of B b–d

Investment of A ´ Period of Investment of A Quantity of a b - d


i.e., =
Investment of B ´ Period of Investment of B Quantity of b d - a

Pr ofit of A Loss ofA Applications of Alligation Rule :


= or
Pr ofit of B Loss of B (i) To find the mean value of a mixture when the prices of
If A, B and C are partners in a business, then two or more ingredients, which are mixed together and the
Monthly Equivalent Investment of A : Monthly Equivalent proportion in which they are mixed are given.
Investment of B : Monthly Equivalent Investment of C (ii) To find the proportion in which the ingredients at given
= Profit of A : Profit of B : Profit of C. prices must be mixed to produce a mixture at a given
= Loss of A : Loss of B : Loss of C. price.
B-24 RATIO & PROPORTION

EXERCISE
1. Salaries of A, B and C were in the ratio of 3 : 5 : 7 respectively. 8. Seats for Maths, Physics and Biology are in the ratio of
If their salaries were increased by 50%, 60% and 50% 5 : 7 : 8 respectively. There is a proposal to increase these
respectively, what will be the new ratio of their respective seats by 40%, 50% and 75% respectively. What will be the
salaries? respective ratio of increased seats?
(a) 3 : 6 : 7 (b) 4 : 5 : 7 (a) 2 : 3 : 4 (b) 6 : 7 : 8
(c) 4 : 5 : 8 (d) Data inadequate (c) 6 : 8 : 9 (d) Cannot be determined
(e) None of these (e) None of these
2. If 40% of a number is equal to two-thirds of another number, 9. Mr Sharad started a business investing ` 50000. Four
what is the ratio of the first number to the second? months later Mr Praveen joined the business by investing
(a) 7 : 3 (b) 3 : 7 ` 90000. If the profit in the business at the end of the year
(c) 2 : 5 (d) 5 : 3 was ` 22000 how much amount would Mr Praveen have
(e) None of these received as the profit?
3. Radha started a business, investing ` 75,000. After 3 months, (a) ` 16000 (b) ` 14000
Sunidhi joined her with an amount of ` 1,25,000 and after (c) ` 12000 (d) ` 11000
another six months Neha joined them with an amount of (e) None of these
` 1,50,000. Profit earned at the end of three years from when 10. The ratio of Gomati’s and Rashmi’s ages is 3 : 5 respectively.
Radha started the business should be distributed in what After ten years this ratio will become 2 : 3. What is Rashmi’s
ratio among Radha, Sunidhi and Neha respectively? age in years?
(a) 36 : 55 : 54 (b) 18 : 28 : 27 (a) 50 (b) 40
(c) 35 : 54 : 55 (d) Cannot be determined (c) 60 (d) Cannot be determined
(e) None of these (e) None of these
4. What should come in place of the question mark(?) in the 11. Salaries of Rajesh and Sunil are in the ratio of 2 :3. If the
28 ? salary of each one is increased by ` 4000 the new ratio
following equation? <
? 112 becomes 40 : 57. What is Sunil’s present salary?
(a) 70 (b) 56 (a) ` 17000 (b) ` 20000
(c) 48 (d) 64 (c) ` 25500 (d) Csannot be determined
(e) None of these (e) None of these
5. An empty fuel tank to a car was filled with A type of petrol. 12. The numbers of students speaking English and Hindi are in
When the tank was half empty, it was filled with B type of the ratio of 4:5. If the number of students speaking English
petrol. Again when the tank was half empty, it was filled increased by 35% and that speaking Hindi increased by
with A type of petrol. When the tank was half empty again, 20%, what would be the new respective ratio?
it was filled with B type of petrol. At this time, what was the (a) 19 : 20 (b) 7 : 8
percentage of A type of petrol in the tank? (c) 8 : 9 (d) Cannot be determined
(a) 50% (b) 40% (e) None of these
(c) 33.5% (d) 37.5% 13. Abhijit started a business investing ` 70000. Anuja joined
(e) None of these him after six months with an amount of ` 105000 and Sunil
6. The ratio of A’s and B’s salary is 9 : 4. If A’s salary is joined them with ` 1.4 lakhs after another six months. The
increased by 15%, then his total salary becomes ` 5175. amount of profit earned should be distributed in what ratio
What is the salary of B? among Abhijit, Anuja and Sunil respectively, three years
(a) ` 2,000 (b) ` 4,000 after Abhijit started the business?
(c) ` 4,500 (d) ` 2,500 (a) 42 : 45 : 56 (b) 7 : 6 : 10
(e) None of these (c) 12 : 15 : 16 (d) Cannot be determined
7. Three friends A, B and C started a business by investing a (e) None of these
sum of money in the ratio of 5 : 7 : 6. After 6 months C 14. The ratio of males and females in a city is 7 : 8 and the
withdraws half of his capital. If the sum invested by ‘A’ is percentage of children among males and females is 25%
` 40,000, out of a total annual profit of ` 33,000, C’s share and 20% respectively. If the number of adult females in the
will be city is 156800 what is the total population?
(a) ` 9,000 (b) ` 12,000 (a) 245000 (b) 367500
(c) ` 11,000 (d) ` 10,000 (c) 196000 (d) 171500
(e) None of these (e) None of these
RATIO & PROPORTION B-25
15. Hariprasad and Madhusudan started a business, investing 22. Abhishek started a business investing ` 50,000. After one
sums in the ratio of 2 : 3. If Hariprasad had invested an year he invested another ` 30,000 and Sudin also joined him
additional amount of ` 10,000 the ratio of Hariprasad's with a capital of ` 70,000. If the profit earned in three years
investment to Madhusudan's investment would have been from the starting of business was ` 87,500, then find the
3 : 2. What was the amount invested by Hariprasad? share of Sudin in the profit.
(a) ` 8000 (b) ` 12000
(a) ` 37,500 (b) ` 32,500
(c) ` 9000 (c) Data inadequate
(c) ` 38,281 (d) ` 52,500
(e) None of these
(e) None of these
16. The ratio of the present ages of a son and his father is 1 : 5
and that of his mother and father is 4 : 5. After 2 years the 23. Weights of two friends Ram and Shyam are in the ratio of
ratio of the age of the son to that of his mother becomes 4 : 5. Ram’s weight increases by 10% and the total weight of
3 : 10. What is the present age of the father? Ram and Shyam together becomes 82.8 kg, with an increase
(a) 30 years (b) 28 years of 15%. By what per cent did the weight of Shyam increase?
(c) 37 years (d) Data inadequate (a) 12.5% (b) 17.5%
(e) None of these (c) 19% (d) 21%
17. The ratio of the number of students appearing for (e) None of these
examination in the year 1998 in the states A, B and C was 24. When 50% of one number is added to a second number, the
3 : 5 : 6. Next year if the number of students in these states second number increases to its four-thirds. What is the
increases by 20%, 10% and 20% respectively, the ratio in ratio between the first number and the second number?
states A and C would be 1 : 2. What was the number of (a) 3 : 2 (b) 3 : 4
students who appeared for the examination in the state A in
(c) 2 : 3 (d) Data inadequate
1998?
(e) None of these
(a) 7200 (b) 6000
25. The ratio of present ages of Nisha and Shilpa is 7:8
(c) 7500 (d) Data inadequate
(e) None of these respectively. Four years hence this ratio becomes 9:10
18. A man spends ` 1810 for buying bedsheets at ` 200 each respectively. What is Nisha’s present age in years?
and pillows at ` 70 each. What will be the ratio of bedsheets (a) 18 (b) 14
to pillows when maximum number of bedsheets are bought? (c) 17 (d) Data inadequate
(a) 3:8 (b) 8:3 (e) None of these
(c) 9:1 (d) 1:9 26. When a number is added to another number the total
(e) None of these
19. Mr Shivkumar started a business, investing ` 25000 in 1996. 1
becomes 33 per cent of the second number. What is the
In 1997 he invested an additional amount of ` 10000 and 3
Mr Rakesh joined him with an amount of ` 35000. In 1998, ratio between the first and the second number?
Mr Shivkumar invested another additional amount of ` 10000 (a) 3 : 7 (b) 7 : 4
and Mr Suresh joined them with an amount of ` 35000. (c) 7 : 3 (d) Data inadequate
What will be Rakesh’s share in the profit of ` 150000 earned (e) None of these
at the end of three years from the start of the business in 27. The ratio between the present ages of P and Q is 5 : 8. After
1996? four years, the ratio between their ages will be 2 : 3. What is
(a) ` 70000 (b) ` 50000
Q’s age at present?
(c) ` 45000 (d) ` 75000
(a) 36 years (b) 20 years
(e) None of these
20. Incomes of two companies A and B are in the ratio of 5 : 8. (c) 24 years (d) Data inadequate
Had the income of company A been more by ` 25 lakh, the (e) None of these
ratio of their incomes would have been 5 : 4. What is the 28. Jaydeep purchased 25 kg of rice at the rate of ` 16.50 per kg
income of company B? and 35 kg of rice at the rate of ` 24.50 per kg. He mixed the
(a) ` 80 lakh (b) ` 50 lakh two and sold the mixture. Approximately, at what price per
(c) ` 40 lakh (d) ` 60 lakh kg did he sell the mixture to make 25 per cent profit?
(e) None of these (a) ` 26.50 (b) ` 27.50
21. The ratio of number of students studying Arts, Commerce (c) ` 28.50 (d) ` 30.00
and Science in a College is 3 : 5 : 8. What is the new ratio of (e) ` 29.00
the number of students studying Arts, Commerce and 29. In 1 kg mixture of sand and iron, 20% is iron. How much
Science respectively if there is an increase of 20%, 40% and sand should be added so that the proportion of iron
25% in the number of students studying Arts, Commerce
becomes 10%?
and Science?
(a) 1 kg (b) 200 gms
(a) 18:35:50 (b) 3:10:10
(c) 800 gms (d) 1.8 kg
(c) 4:8:5 (d) 32:35:25
(e) None of these (e) None of these
B-26 RATIO & PROPORTION
30. The ratio of P's and Q's ages is 5 : 7. If the difference between 39. An amount of money is to be divided among P, Q and R in
the present age of Q and the age of P six years hence is 2 the ratio of 4 : 9 : 16. If R gets 4 times more than P, what is Q’s
then what is the total of present ages of P and Q? share in it?
(a) 52 years (b) 48 years (a) ` 1,800 (b) ` 2,700
(c) 56 years (d) Data inadequate (c) ` 3,600 (d) Data inadequate
(e) None of these (e) None of these
31. There is a ratio of 5 :4 between two numbers. If forty percent 40. Jagtap purchases 30 kg of wheat at the rate of ` 11.50 per kg
of the first number is 12 then what would be the 50 percent and 20 kg of wheat at the rate of ` 14.25 per kg. He mixed the
of the second number? two and sold the mixture. Approximately at what price per
(a) 12 (b) 24 kg should he sell the mixture to make 30 per cent profit?
(c) 18 (a) ` 16.30 (b) ` 18.20
(d) Data inadequate (c) ` 15.60 (d) ` 14.80
(e) None of the above (e) ` 15.40
32. An amount of money is to be distributed among P, Q and R 41. Mr. Gangadhar, Mr. Ramesh and Mr. Shridhar together
in the ratio of 5 : 8 : 12 respectively. If the total share of Q earned ` 19800. The ratio of earnings between Mr. Gangadhar
and R is four times that of P, what is definitely P’s share? and Mr. Ramesh is 2 : 1 while that between Mr. Ramesh and
(a) ` 3,000 (b) ` 5,000 Mr. Shridhar is 3 : 2. How much did Mr. Ramesh earn?
(c) ` 8,000 (d) Data inadequate (a) ` 3600 (b) ` 5400
(e) None of these (c) ` 1800 (d) ` 6300
33. When 30 per cent of a number is added to another number (e) None of these
the second number increases to its 140 per cent. What is 42. Mr. Kutty has only hens and sheep. If the total number of
the ratio between the first and the second number? their heads is 38 and the total number of legs is 100 then
(a) 3 : 4 (b) 4 : 3 what is the ratio between the numbers of hens and sheep?
(c) 3 : 2 (d) Data inadequate (a) 2 : 1 (b) 1 : 2
(e) None of these (c) 6 : 13 (d) 13 : 6
34. If 25% of a number is subtracted from a second number the (e) None of these
second number reduces to its five-sixths. What is the ratio A B C
between the first number and the second number? 43. If A : B : C = 2 : 3 : 4, then : : is equal to :
B C A
(a) 2 : 3 (b) 3 : 2
(c) 1 : 3 (d) Data inadequate (a) 4 : 9 : 16 (b) 8 : 9 : 12
(e) None of these (c) 8 : 9 : 16 (d) 8 : 9 : 24
35. Two friends P & Q started a business investing amounts in 44. A sum of money is to be distributed among A, B, C, D in the
the ratio of 5 : 6. R joined them after six months investing an proportion of 5 : 2 : 4 : 3. If C gets ` 1000 more than D, what
amount equal to that of Q’s amount. At the end of the year is B’s share?
20% profit was earned which was equal to ` 98,000. What (a) ` 500 (b) ` 1500
was the amount invested by R? (c) ` 2000 (d) ` 1400
(a) ` 2,10,000 (b) ` 1,05,000 (e) None of these
(c) ` 1,75,000 (d) Data inadequate 45. The sum of three numbers is 98. If the ratio of the first to the
(e) None of these second is 2 : 3 and that of the second to the third is 5 : 8,
36. One year ago the ratio of Yamini’s and Gamini’s ages was then the second number is :
6 : 7 respectively. Four years hence this ratio would become (a) 20 (b) 30
7 : 8. How old is Gamini? (c) 38 (d) 48
(a) 35 years (b) 30 years (e) None of these
(c) 31 years (d) Cannot be determined 46. The ratio of number of ladies to gents at a party was 1 : 2,
(e) None of these but when 2 ladies and 2 gents left, the ratio became 1 : 3.
37. Ratio of present age of P and Q is 7 : 3. After four years their How many people were originally present at the party?
ages are in the ratio of 2 : 1. What is the present age of P ? (a) 6 (b) 9
(a) 24 years (b) 28 years (c) 12 (d) 10
(c) 32 years (d) Data inadequate (e) None of these
(e) None of these 47. A man divides his property so that his son’s share to his
38. If 40 per cent of a number is added to an other number then wife’s and the wife’s share to his daughter are both in the
it becomes 125 per cent of itself. What will be the ratio of ratio 3 : 1. If the daughter gets ` 10,000 less than the son,
first and second numbers? find the total worth of the property.
(a) 8 : 5 (b) 5 : 7 (a) ` 16,200 (b) ` 16,250
(c) 5 : 8 (d) Data inadequate (c) ` 16,500 (d) ` 15,300
(e) None of these (e) None of these
RATIO & PROPORTION B-27

48. A bag contains an equal number of one rupee, 50 paise and 55. A, B and C start a business each investing ` 20,000. After 5
25 paise coins respectively. If the total value is ` 35, how months A withdrew ` 5000, B withdrew ` 4000 and C invests
many coins of each type are there? ` 6000 more. At the end of the year, a total profit of ` 69,900
(a) 20 coins (b) 30 coins was recorded. Find the share of B.
(c) 28 coins (d) 25 coins (a) ` 20,000 (b) ` 21,200
(e) None of these (c) ` 28,200 (d) ` 20,500
49. The salaries of A,B,C are in the ratio 2 : 3 : 5. If the increments (e) None of these
of 15%, 10% and 20% are allowed respectively in their 56. A is a working partner and B is a sleeping partner in a
salaries, then what will be the new ratio of their salaries? business. A puts in ` 50,000 and B ` 60,000. A gets 12.5% of
(a) 3 : 3 : 10 (b) 10 : 11 : 20 the profit for managing the business, and the rest is divided
(c) 23 : 33 : 60 (d) Cannot be determined in proportion to their capitals. Find the share of A in profit
(e) None of these of ` 8800.
50. In an express train, the passengers travelling in A.C. sleeper (a) ` 3500 (b) ` 4600
class, First class and Sleeper class are in the ratio 1:2:7, and (c) ` 5400 (d) ` 4800
rate for each class is in the ratio 5 : 4 : 2. If the total income (e) None of these
from this train is ` 54, 000, find the income of Indian Railways 57. A began business with ` 12500 and is joined afterwards by
from A.C. sleeper class. B with ` 37500. When did B join, if the profits at the end of
(a) ` 12,000 (b) ` 20,000 the year are divided equally?
(d) ` 22,000 (d) ` 10,000 (a) 8 months (b) 9 months
(e) None of these (c) 10 months (d) 7 months
51. What is the ratio whose terms differ by 40 and the measure (e) None of these
2 58. A began business with ` 45,000 and was later joined by B
of which is ? with ` 54,000. When did B join if the profit at the end of the
7
(a) 16 : 56 (b) 14 : 56 year were divided in the ratio 2 : 1?
(c) 15 : 56 (d) 16 : 72 (a) 5 months after (b) 10 months after
(e) None of these (c) 7 months after (d) 12 months after
52. The average age of three boys is 25 years and their ages are (e) None of these
in the proportion 3: 5 : 7. The age of the youngest boy is: 59. A and B enter into partnership with capitals in the ratio 3 : 4.
(a) 21 years (b) 18 years At the end of 10 months A withdraws, and the profits now
(c) 15 years (d) 9 years are divided in the ratio of 5 : 6. Find how long B remained in
(e) None of these the business?
(a) 9 months (b) 8 months
1" 7"
53. A photograph measuring 2 ´ 1 is to be enlarged so (c) 6 months (d) 7 months
2 8
(e) None of these
that the length will be 4”. How many inches will the enlarged
60. A and B invest ` 3,000 and ` 4,000 in a business. A receives
breadth be?
` 10 per month out of the profit as a remuneration for running
1 1 the business and the rest of profit is divided in proportion
(a) 1 (b) 2
2 8 to the investments. If in a year ‘A’ totally receives ` 390,
3 what does B receive?
(c) 3 (d) 3
8 (a) ` 375 (b) ` 360
(e) None of these (c) ` 350 (d) ` 260
1 1 (e) None of these
54. In a partnership, A invests of the capital for of the time,
6 6 61. A started a business with ` 4500 and another person B
1 1 joined after some period with ` 3000. Determine this period
B invests of the capital for of the time and C, the rest of after B joined the business if the profit at the end of the year
3 3
the capital for whole time. Find A’s share of the total profit is divided in the ratio 2 : 1
of ` 2,300. (a) After 3 months (b) After 4 months
(a) ` 100 (b) ` 200 1
(c) After 6 months (d) After 2 months
(c) ` 300 (d) ` 400 2
(e) None of these (e) None of these
B-28 RATIO & PROPORTION

62. A and B entered into a partnership with capitals in the ratio 69. One litre of water was mixed to 3 litres of sugar solution
1 containing 4% of sugar. What is the percentage of sugar in
of 4 : 5. After 3 months, A withdrew of his capital and B
4 the solution?
1 (a) 3 (b) 4
withdrew of his capital. The gain at the end of 10 months
5 (c) 6 (d) Insuffficient data
was ` 760. Find the profit of B.
(e) None of these
(a) ` 450 (b) ` 430
70. How much water must be added to 60 litres of milk at
(c) ` 410 (d) ` 340
(e) None of these 1 2
1 litres for ` 20 so as to have a mixture worth ` 10 a
63. A and B rent a pasture for 10 months; A puts in 80 cows for 2 3
7 months. How many can B put in for the remaining 3 months,
litre?
if he pays half as much again as A?
(a) 10 litres (b) 12 litres
(a) 120 (b) 180
(c) 15 litres (d) 18 litres
(c) 200 (d) 280
(e) None of these
(e) None of these
71. How many kg of salt at 42 paise per kg must a man mix with
2
64. In a partnership between X and Y, X’s capital is of total 25 kg of salt at 24 paise per kg so that he may, on selling the
5
mixture at 40 paise per kg gain 25% on the outlay?
2 4
and is invested for year. If his share of the profit is of (a) 15 kg (b) 18 kg
3 7
the total, for how long is Y’s capital in the business? (c) 20 kg (d) 24 kg
1 (e) None of these
(a) 1 year (b) years 72. A trader mixes 80 kg of tea at `15 per kg with 20 kg of tea at
8
cost price of ` 20 per kg. In order to earn a profit of 25%,
1 1
(c) years (d) years what should be the sale price of the mixed tea?
3 4
(a) ` 23.75 (b) ` 22
(e) None of these
(c) ` 20 (d) ` 19.20
65. X and Y put in ` 3,000 and ` 4,000 respectively into a
(e) None of these
business. X reinvests into the business his share of the
73. A company blends two varieties of tea from two different
first year’s profit of ` 2,100 whereas Y does not reinvest. In
what ratio should they share the second year’s profit? tea gardens, one variety costing ` 20 per kg and other ` 25
(a) 39 : 40 (b) 3 : 4 per kg, in the ratio 5 : 4. He sells the blended tea at ` 23 per
(c) 3 : 7 (d) 40 : 79 kg. Find his profit percent :
(e) None of these (a) 5% profit (b) 3.5% loss
66. Gold is 19 times as heavy as water and copper is 9 times (c) 3.5% profit (d) No profit, no loss
heavy. In what ratio must these metals be mixed so that the (e) None of these
mixture may be 15 times as heavy as water? 74. Alcohol cost ` 3.50 per litre and kerosene oil cost ` 2.50 per
(a) 2 : 3 (b) 3 : 2 litre. In what proportion these should be mixed so that the
(c) 1 : 3 (d) 2 : 1
resulting mixture may be ` 2.75 per litre?
(e) None of these
(a) 2 : 5 (b) 1 : 3
67. Six litres of a 20% solution of alcohol in water are mixed with
4 litres of a 60% solution of alcohol in water. The % alcoholic (c) 4 : 7 (d) 2 : 3
strength of the mixture is (e) None of these
(a) 80 (b) 40 75. Pure milk costs ` 3.60 per litre. A milkman adds water to 25
(c) 36 (d) 48 litres of pure milk and sells the mixture at ` 3 per litre. How
(e) None of these many litres of water does he add?
68. A merchant lent out ` 1,000 in two parts, one at 8% and the (a) 2 litres (b) 5 litres
other at 10% interest. The yearly average comes out to be
(c) 7 litres (d) 11 litres
9.2%. Find the amount lent in two parts.
(e) None of these
(a) ` 400, ` 600 (b) ` 500, ` 500
(c) ` 300, ` 700 (d) cannot be determined
(e) None of these
RATIO & PROPORTION B-29

ANSWER KEY
1 (e) 9 (c) 17 (d) 25 (b) 33 (b) 41 (b) 49 (c) 57 (a) 65 (a) 73 (c)
2 (d) 10 (a) 18 (b) 26 (c) 34 (a) 42 (d) 50 (d) 58 (c) 66 (b) 74 (b)
3 (a) 11 (e) 19 (b) 27 (e) 35 (b) 43 (d) 51 (a) 59 (a) 67 (c) 75 (b)
4 (b) 12 (e) 20 (c) 28 (a) 36 (e) 44 (c) 52 (c) 60 (b) 68 (a)
5 (d) 13 (c) 21 (a) 29 (a) 37 (b) 45 (b) 53 (c) 61 (a) 69 (a)
6 (a) 14 (b) 22 (e) 30 (d) 38 (e) 46 (c) 54 (a) 62 (b) 70 (c)
7 (a) 15 (a) 23 (c) 31 (a) 39 (d) 47 (b) 55 (c) 63 (d) 71 (c)
8 (a) 16 (e) 24 (c) 32 (d) 40 (a) 48 (a) 56 (b) 64 (c) 72 (c)

Answers &
Explanations
1. (e) Suppose the salaries of A, B and C were 300k, 500k and 6. (a) Let the salaries of A and B be 9x and 4x.
700k respectively.
115
After increment salary of 9x ´ < 5175
A = 300k + 50% of 300k = 450k 100
B = 500k + 60% of 500k = 800k [ x = 500
C = 700k + 50% of 700k = 1050k [ salary of B = 500 × 4 = ` 2000
Hence, new ratio of the respective salaries of A, B and 7. (a) Sum invested by A, B and C is
C = 450k : 800k : 1050k = 9 : 16 : 21 5 × 12 : 7 × 12 : 6 × 6 + 3 × 6
2. (d) Suppose the first number is x and the second number y. or, 60 : 84 : 54 or, 10 : 14 : 9
2 9
Therefore, 40% of x =
3
of y [ Share of C = ´ 33, 000 <` 9, 000
33
x 2 100 5 140 150 175
\ = ´ = 8. (a) Reqd ratio 5 ´ :7´ : 8´
y 3 40 3 100 100 100
3. (a) Ratio of their profits (Radha’s : Sunidhi’s : Neha’s) = 5 × 140 : 7 × 150 : 8 × 175 = 2 : 3 : 4
= 75 × 36 : 125 × 33 : 150 × 27 9. (c) Ratio of their investment
= 3 × 36 : 5 × 33 : 6 × 27 = 50000 × 12 : 90000 × 8 = 5 : 6
= 3 × 12 : 5 × 11 : 6 × 9
= 36 : 55 : 54 6
[ Amount received by Praveen = ´ 22, 000
11
28 ?
4. (b) < [ ? < 28´112 < 56 = ` 12000
? 112
10. (a) Quicker Method : After 10 years,the ratio of Gomati
5. (d) Petrol Petrol and Rashmi’s ages is 2 : 3. We can also write 2 : 3 as
A B 4 : 6. Now, difference in the ratio is I in both the cases.
I: A 0
Therefore, 1 = 10 \ 5 ® 5 × 10 = 50 years.
A B
II: 11. (e) Let the salaries of Rajesh and Sunil be ` 2x and ` 3x
2 2 respectively.
A A B 2x + 4000 40
III: + Then, =
4 2 4 3x + 4000 57
A A B B or, 114x + 228000 = 120x + 160000
IV: + +
8 4 8 2 or, 6x = 68000
or, 3x = ` 34000
A A 3A
Now, amount of petrol A = + = 4 ´ 135
4 8 8 12. (e) Reqd ratio = = 9 :10
5 ´ 120
3A
\ required % = × l00 = 37.50% 13. (c) Ratio of their investments
8´ A
= 70 × 36 : 105 × 30 : 140 × 24 = 12 : 15 : 16
B-30 RATIO & PROPORTION
21. (a) Suppose the number of students studying Arts,
100
14. (b) Number of females = 156800 × = 196000 Commerce and Science be 3x, 5x and 8x respectively.
80 When their numbers are increased by 20%, 40% and
7 25% respectively the new ratio becomes
[ Number of males =× 196000 = 171500
8 3x × 120% : 5x × 140% :
[ Total population = 196000 + 171500 = 367500
8x × 125% = 36 : 70 : 100
15. (a) Let the initial investments of Hariprasad and
=18 : 35 : 50
Madhusudan be 2x and 3x, respectively.
22. (e) Ratio of Abhishek and Sudin for one month
From the question,
= (50,000 × 36) + (30,000 × 24) : (70,000 × 24)
2 x ∗ 10000 3 = (18,00,000 + 7,20,000) : 16,80,000 = 3 : 2
<
3x 2 Hence share of Sudin in the profit earned from the
or, 4x + 20000 = 9x business.
[ x = 4000
87,500
[ Amount invested by Hariprasad = 2x = ` 8000 = × 2 = ` 35,000.
(3 + 2)
S 1
16. (e) = Þ F = 5S 23. (c) Let the weights of Ram and Shyam be 4x and 5x. Now,
F 5 according to question,
M 4 4 4 x ´ 110
= ÞM = F
F 5 5 + Shyam’s new wt = 82.8 ........ (i)
100
S+2 3 9 x ´ 115
= and (4 x + 5 x ) = = 82.8 ........ (ii)
M + 2 10 100
From (ii), x = 8
4
Þ 10S + 20 = 30 M + 6 = 3 ´ ´5S+ 6 = 12S + 6 Putting in (i), we get
5 Shyam’s new wt = (82.8 – 35.2) = 47.6
[ 2 S = 14 Þ S = 7 years
[ F = 5S = 35 years æ 47.6 - 40 ö
% increase in Shyam’s wt = ç ´100 ÷ =19%
17. (d) Let the number of students appearing for examination è 40 ø
in the year 1998 in the states A, B and C be 3x, 5x and 24. (c) Let the numbers be y and x respectively
6x respectively.
According to the question, 4x y 4x
x + 50% of y = or, = –x
3 2 3
120
3x ´ y x y 2
100 < 1 or, = or, =
120 2 2 3 x 3
6x ´
100 25. (b) Nisha Shilpa
Hence data inadequate. 7 8
18. (b) The man can’t purchase more than 8 bedsheets 4 9 10
\ cost of 8 bedsheets = 8 × 200 = ` 1600 4 ´ (10 - 9)
Remaining amount = 1810 – 1600 = ` 210 Present age of Nisha = 9 ´ 8 - 10 ´ 7 ´ 7

And in ` 210, the man can purchase = 14 years


26. (c) Let the first and second numbers be x and y
210
= 3 pillows respectively.
70
Reqd ratio = 8 : 3 10
Then, x + y = y
19. (b) Ratio of their investments = 25,000 × 1 + 35000 × 1 + 3
45000 × 1 : 35000 × 2 : 35000 × 1 = 3 : 2 : 1.
7
2 y
or, x =
\ Rakesh’s share = 6 ×150000 = ` 50000 3
\ x:y=7:3
20. (c) Let the incomes of two companies A and B be 5x and
8x respectively. P 5 5Q
27. (e) = or P = ....(i)
From the question, Q 8 8
5x + 25 5 P+4 2
= Þ 20x + 100 = 40x \ x = 5 =
8x 4 Q+4 3
\ Income of company B = 8x = ` 40 lakh or, 3P + 12 = 2Q + 8 or, 2Q – 3P = 4 ... (ii)
RATIO & PROPORTION B-31
Putting value of P from eq. (i), 34. (a) Let the first and second number be x and y respectively.
5 25 5
2Q – 3 × Q = 4 Þ Q = 32 x, y ´ < y´
8 100 6
28. (a) CP = 25 × 16.50 + 35 × 24.50 = ` 1270 x 5
or, y , < y
125 4 6
SP = 1270 × = ` 1587.50
100 1 x
or, y<
1587.50 6 4
Price per kg = » 26.50 [ x:y=2:3
60
35. (b) Ratio for amount invested by P, Q & R
29. (a) In 1 kg mixture quantity of iron = 200 gm
= 5x × 12 : 6x × 12 : 6x × 6
Let x gm sand should be added, then = 60x : 72x : 36x
10% of (1000 + x) = 200 = 5x : 6x : 3x
[ x = 1000 gm = 1 kg Profit = 98000 = 20% of T
where, T = Total amount
P 5
30. (d) = T = ` 490000
Q 7 Amount received by
7 3x
or, Q= P R= (490000)
5 3x + 6 x + 5 x
Case I : Q – (P + 6) = 2 = ` 105000
or, Q = P + 8 36. (e) Y : G = 6 : 7; One year before
Y : G = 7 : 8; After 4 years
7 Now, 1 ® 5 \ 7 ® 35
\ P =P+8
5 Therefore, the age of Gamini now is 35 + 1 = 36 years
or, 7P = 5P + 40 P 7 3
37. (b) = ÞQ= P ....(i)
40 7 Q 3 7
\ P= = 20 and Q = ´ 20 = 28
2 5 also, 2Q + 8 = P + 4
\ P + Q = 20 + 28 = 48 or, 2Q – P = – 4 ...(ii)
Combining equations (i) and (ii), we get
Case II : (P + 6) – Q = 2
P = 28 years
7 x 125x x 20
or, P+ 6– P =2
5 38. (e) 40 +y= Þ =
100 100 y 17
or, P = – 10 and Q = 14
39. (d) Here, neither the total amount nor the individual
\ P + Q = 10 + 14 = 24 years amount is given. So the share of Q cannot be
a 5 4 2 determined.
31. (a) = , b = a Given (40% of a =) a = 12 40. (a) CP = 30 × 11.50 + 20 × 14.25 = ` 630
b 4 5 5
130 1
4 SP = 630 ´ ´ » `16.30
\ a = 5 × 6 and b = × 5 × 6 = 24 100 50
5 41. (b) Ratio of investment = 6 : 3 : 2
24 3
\ 50% of b = = 12 \ Share of Mr. Ramesh = ´ 19800 = ` 5400
2 11
32. (d) P : Q : R = 5 : 8 : 12 42. (d) Let the total number of hens and sheep be x and y
respectively.
Total share of Q and R 8 + 12 20
= == 4 i.e., x + y = 38 and 2x + 4y = 100 \ Ratio = 13 : 6
share of P 5 5 43. (d) Let A = 2x, B = 3x and C = 4x. Then,
So, we see that no new information has been given in A 2x 2 B 3x 3 C 4x 2
the question and P’s share can’t be determined. = = , = = and = =
B 3x 3 C 4x 4 A 2x 1
33. (b) Let the first and the second numbers be x and y respect
then A B C 2 3 2
Þ : : = : : = 8 : 9 : 24.
y + 30% of x = 140% of y B C A 3 4 1
or, y + 0.3x = 1.4y 44. (c) Let the shares of A, B, C and D be ` 5x, ` 2x, ` 4x,
or, 0.3x = 0.4y ` 3x respectively.
\ x : y = 0.4 : 0.3 = 4 : 3 Then, 4x – 3x = 1000 Þ x = 1000
\ B’s Share = ` 2x = ` 2000
B-32 RATIO & PROPORTION
45. (b) A : B = 2 : 3 = 2 × 5 : 3 × 5 = 10 : 15 53. (c) Let enlarged breadth be x inches. Then,
and B : C = 5 : 8 = 5 × 3 : 8 × 3 = 15 : 24 5 15
Therefore, A : B : C = 10 : 15 : 24 : 4 :: : x
2 8
\ A : B : C = 10 : 15 : 24
5 15
Let the number be 10x, 15x and 24x. Þ x = 4 ´ Þ x = 3 inches
Then, 10x + 15x + 24x = 98 2 8
or 49x = 98 or x = 2
(a) Remaining capital = 1 - æç + ö÷ =
1 1 1
54.
Þ Second number = 15x = 15 × 2 = 30 è 6 3 ø 2
46. (c) Let number of ladies = x
Ratio of their profit
then, number of gents = 2x
x-2 1 1 é1 ù 1 é1 ù 1
Now, = Þ 3x - 6 = 2x - 2 = ´ ê ´ 12ú : ´ ê ´12ú : ´12
2x - 2 3 6 ë6 û 3 ë3 û 2
Þx=4
1 4
\ Total number of people originally present = : : 6 = 1: 4 :18
= 4 + 8 = 12 3 3
47. (b) Let Son’s share = ` S; 1
Daughter’s share = ` D; \ A’s share = ´ 2300 = ` 100
1 + 4 + 18
and Wife’s share = ` W.
Also, S : W = W : D = 3 : 1 55. (c) Ratio of the capitals of A, B and C
\ S:W: D =9:3 :1 = 20000 ×5 + 15000 × 7 : 20000 × 5 + 16000 × 7 : 20000 ×
then S = 9x , D = x 5 + 26000 × 7
and 9x – x = 10,000 Þ x = ` 1250 = 205000 : 212000 : 282000 = 205 : 212 : 282.
\ Total worth of the property = (9 + 3 + 1) x = 13x æ 212 ö
B’s share = ` ç 69900 ´ = ` 21200;
= 13 × 1250 = ` 16,250 è 699 ÷ø
48. (a) Let number of each type of coin = x. Then,
56. (b) The amount A gets for managing
1 × x + .50 × x + .25 x = 35
Þ 1.75x = 35 Þ x = 20 coins = 12.5% of Rs. 8800 = ` 1100
49. (c) Let A = 2k, B = 3k and C = 5k. Remaining profit = ` 8800 – ` 1100 = ` 7700
This is to be divided in the ratio 5 : 6.
A’s new salary = 115 of 2k = æç 115 ´ 2k ö÷ = 23 k Share of A = 5/11 of ` 7700 = ` 3500
100 è 100 ø 10 Þ Total share of A = ` 3500 + ` 1100 = ` 4600.
B’s new salary = 110 of 3k = æ 110 ´ 3k ö = 33 k 57. (a) Let B join after x months of the start of the business so
ç ÷ that B’s money is invested for (12–x) months.
100 è 100 ø 10
\ Profit ratio is 12 × 12500 : (12 – x) × 37500
120 æ 120 ö or 12 : 3(12 – x)
C’s new salary = of 5k = ç ´ 5k ÷ = 6k
100 è 100 ø Since profit is equally divided so
23k 33k 12 = 3(12 – x) or x = 8. Thus B joined after 8 months.
\ New ratio = : : 6k = 23: 33: 60.
10 10 58. (c) Let B join after ‘x’ month of the start of the business.
50. (d) Let number of passengers = x, 2x, 7x Þ (45,000 × 12) : 54,000 × (12 – x) = 2 : 1
and Rate = 5y, 4y, 2y \ (45,000 × 12) × 1 = 54,000 × (12 – x) × 2
Now, since income = Rate × Number of passengers Þ x=7
\ Income = 5xy, 8xy, 14 xy 59. (a) Initially A’s investment = 3x and B’s investment = 4x
\ Income in ratio = 5 : 8 : 14 Let B remain in the business for ‘n’ months.
5 Þ 3x × 10 : 4x × n = 5 : 6
\ Income from A.C. sleeper class = ´ 54, 000
5 + 8 + 14 \ 3x ×10 × 6 = 4x × n × 5 Þ n = 9
= ` 10, 000 60. (b) In a year, for A, total amount as a remuneration
51. (a) Let the ratio be x : (x + 40). Then, = 10 ×12 = ` 120
x 2 \ Amount of A’s profit = 390 – 120 = ` 270
= Þ 7 x = 2 x + 80 Þ x = 16. Ratio of investment = 3 : 4
(x + 40) 7
Let total profit = ` x
\ Required ratio = 16 : 56.
Then, B’s profit = ` (x – 270)
52. (c) Total age of 3 boys = (25 × 3) years = 75 years. Ratio of
their ages = 3 : 5 : 7. 3
\ ´ x = 270 Þ x = 630
æ 3ö 3+ 4
Age of the youngest = ç 75 ´ ÷ years = 15 years.
è 15 ø \ B’s profit = 630 – 270 = ` 360
RATIO & PROPORTION B-33
61. (a) Let B joined after x months.
Then, 4500 × 12 : 3000 (12 – x) = 2 : 1 67. (c) % alcoholic strength in mixture = 6 ´ 20 + 4 ´ 60 = 36%
Ratio of their investments 6+4

4500 ´12 2 Quantity lent at 8% 10 – 9.2 0.8 2


= = Þ x=3 68. (a) = = =
3000(12 - x ) 1 Quantity lent at 10% 9.2 - 8 1.2 3
62. (b) Let A’s capital = ` 4x and B’s capital = ` 5x \ Quantity of money lent at 8%
\ Ratio of profit
2
3 4 = ´ 1000 = ` 400
= 4x × 3 + (4x) ´ 7 : 5x ´ 3 + (5x) ´ 7 2+3
4 5 and quantity of money lent at 10%
= 33 : 43
3
43 = ´ 1000 = ` 600
\ Profit of B = ´ 760 = ` 430 2+3
33 + 43
69. (a) New % of sugar in (3 + 1) litre solution
63. (d) Let B puts = x cows
0.04 ´ 3
3 = = 0.03 = 3%
then amount paid by B = ´ amount paid by A . (3 + 1)
2
80 ´ 7 amount paid by A æ 2ö 40
\ = 70. (c) C.P. of 1 litre of milk = ` ç 20 ´ ÷ = ` .
x ´ 3 3/2 × amount paid by A è 3ø 3

80 ´ 7 ´ 3
Þx= = 280 cows C.P. of 1 litre C.P. of 1 litre
3´ 2 of water of milk
40
2 0 Mean `
64. (c) Q X’s capital = of total price 3
5
32
2 3 `
\ Y’s capital = 1 - = of total 3
5 5
Let Y invested capital for t years. 40 32 8 32 32
– = –0 =
2 2 3 3 3 3 3 3
\ Ratio of profit = ´ : ´t
5 3 5
8 32
4 3t \ Ratio of water and milk = : = 8 : 32 = 1 : 4.
= : … (i) 3 3
15 5
\ Quantity of water to be added to 60 litres of milk
4 3
Share of Y’s profit = 1 - = of the total æ1 ö
7 7 = ç ´ 60 ÷ litres = 15 litres.
è 4 ø
Actual ratio of profit = 4 : 3
4 /15 4 1 100
\ By (i), = Þ t = year. 71. (c) Here, cost price of mixture = 40 ´ = 32 paise
3t / 5 3 3 100 + 25
65. (a) For first year, ratio of profit = 3 : 4
q1 32 - 24 8 4
\ q = 42 - 32 = 10 = 5
X’s profit of first year = 3 ´ 2100 = ` 900 2
7
Now, for second year, 4
and hence q1 = ´ 25 = 20 kg
ratio of profit = 3000 × 12 + 900 × 12 : 4000 × 12 5
= 46800 : 48000 = 39 : 40
66. (b) By the rule of alligation, we have 80 ´ 15 + 20 ´ 20
72. (c) C.P. of mixture = = ` 16
80 + 20
Gold Copper
19 times 9 times (100 + 25)
\ S.P. = ´ 16 = ` 20
100
15 times 73. (c) Let the quantity of two varieties of tea be 5x kg and 4x
kg, respectively.
Now, SP = 23 × 9x = 207x
15 – 9 =6 19 – 15 =4 and CP = 20 × 5x + 25 × 4x = 200x
7x
\ Required ratio = 6 = 3 : 2 Profit % = ´ 100 = 3.5%
4 200x
B-34 RATIO & PROPORTION
74. (b) By the rule of alligation, we have
0.25 1
Cost of Alcohol Cost of Kerosene Oil \ Required ratio = = i.e. 1: 3
0.75 3
` 3.50 ` 2.50
75. (b) In mixture,

cost of Quantity of pure milk 3- 0 3 5


= = =
mixture Quantity of water 3.6 - 3 0.6 1
` 2.75
Since in every 5 litres of milk, he adds 1 litre of water.
\ in every 25 litres of milk, he adds 5 litres of water.

2.75 – 2.50 = 0.25 3.30 – 2.75 = 0.75


Percentage
4 Chapter

The word “per cent” is derived from the latin words “per centum”,
which means “per hundred”. Decrease value
Decrease % = ´ 100
A percentage is a fraction with denominator hundred. Original value
It is denoted by the symbol %. If the price of a commodity increases by r %, then reduction
Numerator of the fraction is called the rate per cent. in consumption, so as not to increase the expenditure is

Simple Fraction Their Percentage æ r ö


çè ´ 100÷ % .
100 + r ø
1 100% If the price of a commodity decreases by r %, then the
increase in consumption so as not to decrease the
1
50%
2 æ r ö
expenditure is ç ´100 ÷ %
è 100 - r ø
1
33.3% Population Formula : If the original population of a town is
3 P, and the annual increase is r %, then the population after
n
1 æ r ö
25% n years is P ç1 + ÷ and population before n years
4 è 100 ø
P
1 = n
20% æ r ö
5 çè 1 + ÷
100 ø
1 If the annual decrease be r %, then the population after n
16.67% n
6 æ r ö
years is P ç1 - ÷ and
è 100 ø
1
14.28% P
7 population before n years = n
æ r ö
çè 1 - ÷
1 100 ø
12.5% First Increase and then decrease : If the value is first
8
increased by x % and then decreased by y% then there is
1 æ xy ö
11.11% çx -y- ÷ % increase or decrease, according to the +ve
9 è 100 ø
or –ve sign respectively.
1
10% Successive increase or decrease
10
If the value is increased successively by x % and y % then
the final increase is given by
1
9.09%
11 æ xy ö
çx + y+ ÷%
è 100 ø
1
8.33% If the value is decreased successively by x % and y % then
12
the final decrease is given by
Increase value
Increase % = ´ 100 æ xy ö
Original value ç -x - y + ÷%
è 100 ø
B-36 PERCENTAGE
If A’s income is r % more than that of B, then B’s income is Example 3: The population of a certain town increased at a certain
less than that of A by rate per cent per annum. Now it is 456976. Four years ago, it was
390625. What will it be 2 years hence ?
æ r ö
ç ´ 100 ÷ % Sol. Suppose the population increases at r% per annum. Then,
è 100 + r ø 4
æ r ö
390625 ç1 + = 456976
è 100 ÷ø
If A’s income is r % less than that of B, then B’s income is
more than that of A by
2
æ r ö 456976 676
æ r ö \ ç1 + ÷ = =
ç ´100 ÷ % è 100 ø 390625 625
è 100 - r ø 2
æ r ö
Due to increase/decrease the price x%, A man purchase a Population 2 years hence = 456976 ç 1 + ÷
è 100 ø
kg more in ` y, then
676
æ xy ö = 456976 × = 494265 approximately.
625
Per kg increase or decrease = ç ÷
è 100 ´ a ø Example 4: Vishal requires 40% to pass. If he gets 185 marks, falls
xy short by 15 marks, what was the maximum he could have got ?
Per kg starting price = ` 100 ± x a Sol. If Vishal has 15 marks more, he could have scored 40%
( ) marks.
11 Now, 15 marks more than 185 is 185 + 15 = 200
Example 1: Express the fraction into the per cent.
12 Let the maximum marks be x, then
11 2 40% of x = 200
´ 100 91
11 12 2
Sol. = = 3 = 91 % 40 200 ´100
12 100 100 3 Þ ´ x = 200 Þ x = = 500
100 40
a
To express % equivalent to fraction : a% = Thus, maximum marks = 500
100
Quicker method :
Example 2: Rent of the house is increased from ` 7000 to ` 7700.
Express the increase in price as a percentage of the original rent. 100(185 + 15) 100 ´ 200
Maximum marks = = = 500 .
Sol. Increase value = ` 7700 – ` 7000 = `700 40 40

Increase value 700 Example 5: If the radius of a circle is increased by 10 %, what is


Increase % = ´ 100 = × 100= 10 the percentage increase in its area ?
Original value 7000
Sol. Radius is increased by 10%.
\ Percentage rise = 10 %.
10(10 + 200)
So, Area is increased by = 21 %.
100
PERCENTAGE B-37

EXERCISE
1. The income of a company increases 20% per annum. If its 8. Mr X, a businessman, had income in the year 1995 such
income is ` 2664000 in the year 1999 what was its income in that he earned a profit of 20% on his investment in the
the year 1997? business. In the year 1996 his investment was less by `
(a) ` 2220000 (b) ` 2850000 5000 but still had the same income (Income = Investment +
(c) ` 2121000 (d) ` 1855000 Profit) as that in 1995. Thus the per cent profit earned in
(e) None of these 1996 increased by 6%. What was his investment in 1995?
2. If the growth in production of company A from 1994 to 1995 (a) ` 100000 (b) ` 100500
was 25% and that from 1995 to 1996 was 60%, then what (c) ` 105000 (d) Data inadequate
percentage growth took place from 1994 to 1996? (e) None of these
(a) 85% (b) 75% 9. The strength of a school increases and decreases every
(c) 200% (d) 100% alternate year. It starts with increase by 10% and there-after
(e) None of these the percentage of increase/decrease is the same. Which of
3. A shopkeeper employed a servant at a monthly salary of the following is definitely true about the strength of the
` 1500. In addition to it, he agreed to pay him a commission school in 2001as compared to that in 1996?
of 15% on the monthly sale. How much sale in Rupees (a) Increase approximately by 2%
should the servant do if he wants his monthly income as (b) Decrease approximately by 2%
` 6000? (c) Increase approximately by 20%
(a) ` 30000 (b) ` 415000 (d) Decrease approximately by 20%
(c) ` 31500 (d) ` 50000 (e) None of these
(e) None of these 10. Milk contains 5% water. What quantity of pure milk should
4. Mr Yadav spends 60% of his monthly salary on consumable be added to 10 litres of milk to reduce this to 2% ?
items and 50% of the remaining on clothes and transport. (a) 5 litres (b) 7 litres
(c) 15 litres (d) Cannot be determined
He saves the remaining amount. If his savings at the end of
(e) None of these
the year were ` 48456, how much amount per month would
11. A = 10% of x, B = 10% of y, C = 10% of x + 10% of y. On the
he have spent on clothes and transport?
basis of the above equalities, what is true in the following?
(a) ` 4038 (b) ` 8076
(a) A is equal to B
(c) ` 9691.20 (d) `4845.60
(b) A is greater than B
(e) None of these
(c) B is greater than A
5. In a class of 60 children, 30% children can speak only
(d) Relation cannot be established between A and B
English, 20 % Hindi and English both and the rest of the
(e) None of these
children can speak only Hindi. How many children can speak
12. If inflation increases at a rate of 8 p.c.p.a. what will a ` 20
Hindi?
article cost at the end of two years?
(a) 42 (b) 36
(a) Between ` 20 and ` 21
(c) 30 (d) 48
(b) Between ` 21 and ` 22
(e) None of these
(c) Between ` 22 and ` 23
6. Somesh bought a microwave oven and paid 10% less than (d) Between ` 23 and ` 24
its original price. He sold it at 30% profit on the price he had (e) None of these
paid. What percentage of profit did Somesh earn on the 13. In a recent survey 40% houses contained two or more
original price? people. Of those houses containing only one person 25%
(a) 32% (b) 11% were having only a male. What is the percentage of all
(c) 20% (d) 17% houses which contain exactly one female and no males?
(e) None of these (a) 75 (b) 40
7. The price of commodity X increases by 40 paise every year, (c) 15 (d) Cannot be determined
while the price of commodity Y increases by 15 paise every (e) None of these
year. If in 1988, the price of commodity X was ` 4.20 and that 14. Sumitra has an average of 56% on her first 7 examinations.
of Y was ` 6.30, in which year will commodity X cost 40 How much should she make on her eighth exami nation to
paise more than commodity Y? obtain an average of 60% on 8 examinations?
(a) 1997 (b) 1998 (a) 88% (b) 78%
(c) 1999 (d) 2000 (c) 98% (d) Cannot be determined
(e) None of these (e) None of these
15. If 3x + 7 = x2 + M = 7x + 5, what is the value of 120% of M?
B-38 PERCENTAGE
(a) 8.90 (b) 9.90 24. Out of a total 85 children playing badminton or table tennis
(c) 9.98 (d) Cannot be determined or both, total number of girls in the group is 70% of the total
(e) None of these number of boys in the group. The number of boys playing
16. Four-fifths of three-eighths of a number is 24. What is 250 only badminton is 50% of the number of boys and the total
per cent of that number? number of boys playing badminton is 60% of the total
number of boys. The number of children playing only table
(a) 100 (b) 160
tennis is 40% of the total number of children and a total of
(c) 120 (d) 200 12 children play badminton and table tennis both. What is
(e) None of these the number of girls playing only badminton?
17. What approximate value should come in place of the (a) 16 (b) 14
question mark (?) in the following equation? (c) 17 (d) Data inadequate
159% of 6531.8 + 5.5 × 1015.2 = ? + 5964.9 (e) None of these
(a) 10,000 (b) 10,900 25. When the price of a product was increased by 15%, the
(c) 11,000 (d) 10,600 number sold was decreased by 20%. What was the net
(e) 12,000 effect?
18. When 35 per cent of a number is added to another number, (a) 8% gain (b) 5% loss
the second number increases by its 20 per cent. What is the (c) 8% loss (d) Cannot be determined
(e) None of these
ratio between the second number and the first number?
(a) 4 : 7 (b) 7 : 4 1 2 4
26. If of of of a number is 12 then 30 per cent of the
(c) 8 : 5 (d) Data inadequate 8 3 5
(e) None of these number will be
19. Two-fifths of thirty per cent of one-fourth of a number is (a) 48 (b) 64
15. What is 20 per cent of that number’? (c) 54 (d) 42
(a ) 90 (b) 150 (e) None of these
(c) 100 (d) 120 27. Venkat purchased twenty dozens of toys at the rate of
` 375 per dozen. He sold each one of them at the rate of
(e) None of these
` 33. What was his percentage profit?
20. Jeevan bought an article with 30 per cent discount on the
(a) 6.5 (b) 5.6
labelled price. He sold the article with 12 per cent profit on (c) 3.5 (d) 4.5
the labelled price. What was his per cent profit on the price (e) None of these
he bought? 28. A speaks truth in 75% and B in 80% cases. In what
(a) 40 (b) 50 percentage of cases are they likely to contradict each other
(c) 60 (d) Data inadequate when narrating the same incident?
(e) None of these (a) 35 (b) 30
21. Naresh’s monthly income is 30% more than that of Raghu. (c) 25 (d) 20
Raghu’s monthly income is 20% less than that of Vishal. If (e) None of these
the difference between the monthly incomes of Naresh and 29. Pradip spends 40 per cent of his monthly income on food
Vishal is ` 800, what is the monthly income of Raghu? items, and 50 per cent of the remaining on clothes and
(a) ` 16,000 (b) ` 20,000 conveyance. He saves one-third of the remaining amount
(c) ` 12,000 (d) Data inadequate after spending on food, clothes and conveyance. If he saves
` 19,200 every year, what is his monthly income?
(e) None of these
(a) ` 24,000 (b) ` 12,000
22. In a certain year, the population of a certain town was 9000.
(c) ` 16,000 (d) ` 20,000
If in the next year the population of males increases by 5% (e) None of these
and that of the females by 8% and the total population 30. When 30 per cent of a number is added to another number
increases to 9600, then what was the ratio of population of the second number increases by its 20 per cent. What is the
males and females in that given year? ratio between the first and the second number?
(a) 4:5 (b) 5:4 (a) 3 : 2 (b) 2 : 3
(c) 2:3 (d) Data inadequate (c) 2 : 5 (d) Data inadequate
(e) None of these (e) None of these
23. A petrol pump owner mixed leaded and unleaded petrol in 31. Rajesh solved 80 per cent of the questions in an examination
such a way that the mixture contains 10% unleaded petrol. correctly. If out of 41 questions solved by Rajesh 37
What quantity of leaded petrol should be added to 1 ltr questions are correct and of the remaining questions out of
mixture so that the percentage of unleaded petrol becomes 8 questions 5 questions have been solved by Rajesh
5%? correctly then find the total number of questions asked in
(a) 1000 ml (b) 900 ml the examination.
(c) 1900 ml (d) 1800 ml (a) 75 (b) 65
(c) 60 (d) Cannot be determined
(e) None of these
(e) None of these
PERCENTAGE B-39
32. 10% of the inhabitants of a village having died of cholera, a
1 1
panic set in, during which 25% of the remaining inhabitants (a) 11 % and 10% (b) 10% and 11 %
left the village. The population is then reduced to 4050. 9 9
Find the number of original inhabitants.
1
(a) 5000 (b) 6000 (c) Both 10% (d) Both 11 %
(c) 7000 (d) 8000 9
(e) None of these (e) None of these
33. Chunilal invests 65% in machinery 20% in raw material and 39. 405 sweets were distributed equally among children in such
still has ` 1,305 cash with him. Find his total investment. a way that the number of sweets received by each child is
(a) ` 6,500 (b) ` 7, 225 20% of the total number of children. How many sweets did
(c) ` 8,500 (d) ` 7, 395 each child receive?
(e) None of these (a) 7 (b) 9
34. When the price of a pressure cooker was increased by 15%, (c) 18 (d) 45
the sale of pressure cookers decreased by 15%. What was (e) None of these
the net effect on the sales? 40. There is an increase of 30% in the production of milk
(a) 15% decrease (b) no effect chocolates in Amul Dairy in one month. If now it is 9,100
(c) 2.25% increase (d) 2.25% decrease milk chcolates per month, what was it one month ago?
(e) None of these
(a) 10,000 chocolates (b) 9000 chocolates
35. When the price of a radio was reduced by 20%, its sale
(c) 8000 chocolates (d) 7000 chocolates
increased by 80%. What was the net effect on the sale?
(e) None of these
(a) 44% increase (b) 44% decrease
(c) 66% increase (d) 75% increase 41. In a college election between two rivals, a candidate who
(e) None of these got 40% of the total votes polled, was defeated by his rival
36. When the price of sugar was increased by 32%, a family by 160 votes. The total number of votes polled was
reduced its consumption in such a way that the expenditure (a) 900 (b) 800
on sugar was only 10% more than before. If 30 kg were (c) 700 (d) 600
consumed per month before, find the new monthly (e) None of these
consumption. 42. A scooter costs ` 25, 000 when it is brand new. At the end of
(a) 20 kg (b) 25 kg each year, its value is only 80% of what it was at the
(c) 30 kg (d) 35 kg beginning of the year. What is the value of the scooter at
(e) None of these the end of 3 years?
37. If 10 % of an electricity bill is deducted, ` 45 is still to be (a) ` 10,000 (b) ` 12,500
paid. How much was the bill? (c) ` 12,800 (d) ` 12,000
(a) ` 50 (b) ` 60 (e) None of these
(c) ` 55 (d) ` 70 43. A number is increased by 11% and then reduced by 10%.
(e) None of these After these operations, the number :
38. The ratio of salary of a worker in July to that in June was (a) does not change (b) decreases by 1%
1 1 (c) increases by 1% (d) increases by 0.1%
2 : 2 , by what % the salary of July more than salary of (e) None of these
2 4
June. Also find by what %, salary of June was less than that
of July.

ANSWER KEY
1 (e) 6 (d) 11 (d) 16 (d) 21 (a) 26 (c) 31 (b) 36 (b) 41 (b)
2 (d) 7 (b) 12 (d) 17 (a) 22 (a) 27 (b) 32 (b) 37 (a) 42 (c)
3 (a) 8 (c) 13 (e) 18 (b) 23 (a) 28 (a) 33 (d) 38 (a) 43 (b)
4 (a) 9 (e) 14 (d) 19 (c) 24 (b) 29 (c) 34 (d) 39 (b)
5 (a) 10 (c) 15 (b) 20 (c) 25 (c) 30 (b) 35 (a) 40 (d)
B-40 PERCENTAGE

Answers &
Explanations
1. (e) Let income in 1997 = x Investment of company X in 1996 would be (x – 5000) From
According to the question, the question,
x 6x 126 6
Income in 1998 < x ∗ < (x – 5000) ´ = x Þ x = ` 105000
5 5 100 5
9. (e) Let the strength of school was x in 1998
6x 6x 36x
Income in 1999 < ∗ < \ strength in 2001 will be
5 25 25
But given, income in 1999 = 2664000 110 90 110 90 110
=x ´ ´ ´ ´ = 1.07811 x
100 100 100 100 100
36x
\ = 2664000 Þ x = 1850000 \ increment = 1.07811x – x = 0.07811 x
25
\ % increase = 7.811 » 8%
2. (d) Percentage growth from 1994 to 1996
10 ´ (5 - 2)
25 ´ 60 10. (c) Here required quantity of pure milk =
= +25 + 60 + = + 100% 2
100
3. (a) Servant’s commission amount 10 ´ 3
= = 15 litres
= 6000 – 1500 = ` 4500 2
i.e., 15% = 4500 11. (d) The given information gives no indication regarding
the comparison of x and y.
4500
or, 100% = ×100 = ` 30000 12. (d) Required sum
15
4. (a) Q Amount, he have spent in 1 month on clothes æ 8 ö÷
2
20 ≥ 27≥ 27
ç
= 20 çç1 ∗ ÷ < < 23.3
transport = Amount spent on saving per month è 100 ø÷ 25≥ 25
[ Amount, spent on clothes and transport
13. (e) Houses containing only one person = 100 – 40 = 60%
48456
= = ` 4038 25
12 Houses containing only a male = 60 ´ = 15%
100
5. (a) Number of students who speak only English
= 30% of 60 = 18 \ Houses containing only one female = 60 – 15 = 45%
14. (d) Since the weightage of eighth examination is not
[ Number of students who speak Hindi and English
= 20% of 60 = 12 known, hence can not be determined.
Number of students who speak only Hindi 15. (b) If 3x + 7 = x2 + M = 7x + 5
= (60 – 30 =) 30 ie, 3x + 7 = 7x + 5
[ No. of students who speak Hindi = 30 + 12 = 42 1
6. (d) Quicker Method: or, 4x = 2 \ x =
2
% profit which Somesh gets 2
and 3x + 7 = x + M
10 ´ 30
= –10 + 30 – = +17% 1 3 1 1
100 or, + M = + 7 Þ M + = 8+
4 2 4 2
7. (b) Suppose in ‘n’ years the price of comodity X will be
more by 40 paise than that of commodity Y. 1
\ M =8
\ 420 + 40n – 630 – 15n = 40 4
or, 25n – 210 = 40 16. (d) Let the number = x.
or, 25n = 250
4 3 24 ´ 2 ´ 5
250 ´ x = 24 or x = = 80
or, n= = 10 years 5 8 3
25
250
8. (c) Let the investment of X in 1995 be ` x. \ 250 per cent of the number = ´ 80 = 200
100
x æ xö 6 17. (a) ? » 160% of 6530 + 5.5 × 1010 – 5965
\ Profit = ` \ Income = ` çè x + ÷ø = ` x » 10448 + 5555 – 5965 » 10,000
5 5 5
PERCENTAGE B-41

120 15 ´ 20
18. (b) 35% of x + y = y 25. (c) Net effect = + 15 – 20 – = – 8%
100 100
–ve sign indicates loss.
35 x + 100 y 120 y
or = Þ 35x = 20y 26. (c) Let the number be x.
100 100
1 2 4
y 35 and of of × x = 12
= =7:4 8 3 5
x 20
19. (c) Let the number = x 3x
[ = 54
10
2 30 x 15 ´ 5 ´ 100
´ ´ = 15 or, x = = 500 27. (b) Cost price of 20 dozen toys = 20 × 375 = ` 7,500
5 100 4 2 ´ 30
Selling price of 20 dozen toys = 20 × 33 × 12 = ` 7,920
20% of 500 = 100
20. (c) Let the labelled price of the article = ` 100 then 7,920 , 7, 500
Profit percentage = ≥100 < 5.6%
CP = ` 70 and SP = ` 112. 7, 500
112 - 70 28. (a) Let the truth spoken by A and B be pl and p2
\ Reqd profit percent = ´ 100
70 3 4
respectively, i.e., p1 = and p2 =
42 4 5
= ´ 10 = 60
7 They will contradict each other only when one speaks
21. (a) N = R + 30% of R = 1.3 R truth and the other is lying.
R = V – 20% of V = 80% of V = 0.8 V 3 1 4 1 3 4 7 35
\ N = 1.3 × 0.8V = 1.04 V i.e., ≥ ∗ ≥ < ∗ < < i.e., 35%
4 5 5 4 20 20 20 100
Now, N – V = 1.04 V – V = 0.04 V = ` 800 (given)
29. (c) Food items = 40%
\ V = ` 20000
\ R = 0.8 × 20000 = `. 16000 1
Clothes + conveyance = of 60% = 30%
22. (a) Let the population of males = x; then the population of 2
females = 9000 – x
Now, 5% of x + 8% of (9000 – x) 1 19, 200
of 30% = Þ 10% = 1600
= (9600 – 9000 ) = 600 3 12
or 0.05 x + 720 – 0.08x = 600 \ 100% = ` 16,000
or 720 – 600 = 0.08x – 0.05x 120
or, 120 = 0.03x 30. (b) 30% of I + II = II ´
100
\ x = 4000
\ Reqd ratio of population of males and females 3 2
or, I= II Þ I : II = 2 : 3.
4000 4000 10 10
= = = 4:5 31. (b) Suppose there are 8x questions apart from the 41
9000 - 4000 5000
questions.
23. (a) In 1 lit mixture quantity of unleaded petrol = 100 ml
Let x ml leaded petrol be added, then 37 + 5 x 4
Then, = 80% =
5% of (1000 + x) = 100 ml 41 + 8 x 5
or, 5 (1000 + x) = 100 × 100 Þ 185 + 25x = 164 + 32x Þ 7x = 21Þ x = 3
5000 \ Total no. of questions = 41 + 8x = 65
Þx< < 1000 ml
5 32. (b) Let the total number of original inhabitants be x. Then,
(100 – 25)% of (100 –10)% of x = 4050
24. (b) Let the number of boys = x
7x æ 75 90 ö 27
Þç ´ ´ x ÷ = 4050 Þ x = 4050
then x ∗ < 85 or x = 50 è 100 100 ø 40
10
No. of girls = 85 – 50 = 35 æ 4050 ´ 40 ö
Þ x =ç ÷ = 6000.
è 27 ø
Badminton
\ Number of original inhabitants = 6000.
20 boys
25 boys 5 boys 33. (d) Let he had originally ` x. Then
TT
7 girls
65% of x + 20 % of x + 1305 = x
14 girls 0.65x + 0.2 x + 1305 = x
14 girls
Þ 0.15 x = 1305 Þ x = ` 8700
B-42 PERCENTAGE
\ His total investment = 65% of 8700 + 20% of 8700 Required percentages
= 85% of 700 = ` 7395
5 9 5 9
x- x x- x
(common % change)2
34. (d) Net effect on sale = – = 2 4 ´ 100 and 2 4 ´100
100 9 5
x x
4 2
-(15) 2
= = 2.25% decrease
100 100 100 1
= % and % = 11 % and 10%
35. (a) Let the original price be x and sale be of y units. 9 10 9
Then, the revenue collected initially = x × y 39. (b) Let the total number of children be x.
Now, new price = 0.8x, new sale = 1.8 y Then, x × (20 % of x) = 405
Then, new revenue collected = 1.44xy
1 2
Û x = 405 Û x 2 = 2025 Û x = 45
0.44xy 5
% increase in revenue = ´ 100
xy \ Number of sweets received by each child
= 44% increase = 20% of 45 = 9
36. (b) Since, expenditure = price × consumption 40. (d) Let one month ago, production be x chocolates.
Then, 130 % of x = 9100
132
\ 110% of 30 = ´ new consumption
9100 ´100
100 Þx= = 7000 chocolates
130
110 132
Þ ´ 30 = ´ new consumption 41. (b) Let total number of votes polled be x.
100 100 Then, votes polled by other candidate
Þ New consumption = 25 kg = (100 – 40)% of x = 60% of x
37. (a) Let the bill be ` x. Then Now 60% of x – 40% of x = 160
90% of x = 45
20x
45 ´ 100 Þ = 160 Þ x = 800 votes
Þ x= = ` 50 100
90 42. (c) After first year, the value of the scooter = ` 20,000
5 After second year, the value of scooter = ` 16,000
38. (a) Let the salary of July be ` x After third year, the value of scooter = ` 12,800
2
43. (b) Let the original number be 100.
9 Then, the new number = 100 × 1.1 × 0.9 = 99
and the salary of June be ` x.
4 i.e. the number decreases by 1%.
Profit and
5 Chapter
Loss
Cost Price : The amount paid to purchase an article or the price
Example 1: A cycle was purchased for ` 1600 and sold for ` 1400.
at which an article is made, is known as its cost price.
Find the loss and loss %.
The cost price is abbreviated as C.P. Sol. C.P. of the cycle = ` 1600
Selling Price : The price at which an article is sold, is known as S.P. of the cycle = ` 1400
its selling price. Since S.P < C.P, so there is a loss.
The selling price is abbreviated as S.P. Loss = C.P. – S.P.
Profit : If the selling price (S.P.) of an article is greater than the
= ` 1600 – ` 1400 = ` 200.
cost price (C.P.), then the difference between the selling price and
Loss 200 1
cost price is called profit. Loss % = ´ 100 = ´ 100 = 12 %
Thus, If S.P. > C.P., then C.P. 1600 2
Profit = S.P. – C.P. Example 2: If the C.P. of 15 tables be equal to the S.P. of 20 tables,
find the loss per cent.
Þ S.P. = C.P. + Profit
Sol. By direct method,
Þ C.P. = S.P. – Profit.
Loss : If the selling price (S.P.) of an article is less than the cost -5
Profit/Loss% = ´ 100 = 25% loss, since it is –ve.
price (C.P.), then the difference between the cost price (C.P.) and 20
the selling price (S.P.) is called loss. Example 3: A shopkeeper sold two commodies for ` 800 earn and
Thus, if S.P. < C.P., then on one he gained 10% while on other he losses 10% how much
Loss = C.P. – S.P. does he gain or loss in whole transaction
Þ C.P. = S.P. + Loss (a) 2% (b) 4%
Þ S.P. = C.P. – Loss (c) 6% (d) 1%
2 2
Remember æ common loss and gain ö æ 10 ö
Sol. (d) loss% = ç ÷ = ç ÷ = 1%
C.P. × Profit % è 10 ø è 10 ø
ê Profit = Example 4. By selling 33 metres of cloth, a man gains the sale
100
price of 11 metres. The gain % is
C.P. × Loss % (a) 50% (b) 25%
ê Loss =
100
1
æ 100 + Profit% ö (c)33 % (d) 20%
S.P. = ç 3
ê ÷ × C.P.
è 100 ø Sol. (a) Gain = S.P. of 33 metres – C.P. of 33 metres
æ 100 - Loss% ö = S.P. of 11 metres
ê S.P. = ç ÷ × C.P.
è 100 ø Þ S.P. of 22 metres = C.P. of 33 metres
100 ´ S.P. gain
ê C.P. = \ % gain = ´100
100 + Profit % C.P.of metres
100 ´ S.P. S.P.of 11 metres
ê C.P. = = ´ 100
100 - Loss % C.P. of 33 metres
S.P.of 11 metres 11
• If CP of x things = SP of y things, then = ´ 100 = ´ 100 = 50%
S.P. of 22 metres 22
éx - y ù Shortcut method :
Profit/Loss = ê ´ 100 ú %
ë y û If on selling ‘x’ articles a man gains equal to the S.P. of y
If +ve, Profit; articles. Then,
If –ve, Loss
B-44 PROFIT AND LOSS

y 11 11 Sol. Error = 1 kg – 960 g


% gain = ´100 = ´100 = ´100 = 50%
= 1000 g – 960 g = 40 g.
x-y 33 - 11 22
40
\ Gain % = ´ 100
Marked Price : The price on the lable is called the marked price 1000 - 40
or list price. 40 1
= ´ 100 = 4 %
The marked price is abbreviated as M.P. 960 6
Discount : The reduction made on the ‘marked price’ of an article Goods passing through successive hands
is called the discount. When there are two successive profits of a% and b%, then
the resultant profit per cent is given by
NOTE : When no discount is given, ‘selling price’ is the same as
‘marked price’. æ ab ö
ça + b + ÷%
• Discount = Marked price × Rate of discount. è 100 ø
• S.P. = M.P. – Discount. When there is a profit of a% and loss by b% in a transaction,
then the resultant profit or loss per cent is given by
Discount
• Discount % = ´ 100 .
M.P. æ ab ö
• Buy x get y free i.e., if x + y articles are sold at cost price of ça -b - ÷ % , according to the + ve or – ve sign
è 100 ø
y respectively.
x articles, then the percentage discount = × 100.
x+y • When cost price and selling price are reduced by the same
Example 5. A dishonest dealer professes to sell his goods at cost amount (A) and profit increases then cost price (C.P.)
price, but he uses a weight of 960 g for the kg weight. Find his
[Initial profit % + Increase in profit %] ´ A
gain per cent. =
Increase in profit %

EXERCISE
1. If by selling twelve note-books, the seller earns profit equal 4. A shopkeeper sold a TV set for ` 17940, at a discount of 8%
to the selling price of two note-books, what is his percentage and gained 19.6%. If no discount is allowed, what will be
profit? his gain per cent?
(a) 20% (b) 25% (a) 25% (b) 36.4%
(c) 24.8% (d) Can’t be determined
2 (e) None of these
(c) 16 % (d) Data inadequate
3 5. Deepa bought a calculator at 30% discount on the listed
(e) None of these price. Had she not got the discount, she would have paid
2. A grocer purchased 20 kg of rice at the rate of ` 15 per kg ` 82.50 extra. At what price did she buy the calculator ?
and 30 kg of rice at the rate of ` 13 per kg. At what price per (a) ` 192.50 (b) ` 275
(c) ` 117.85 (d) Cannot be determined
1
kg should he sell the mixture to earn 33 % profit on the (e) None of these
3 6. A shopkeeper sells a TV set for ` 16560 at 10% discount on
cost price? its marked price and earns 15% profit. If no discount is
(a) ` 28.00 (b) ` 20.00 offered, then what will be his present per cent profit?
(c) ` 18.40 (d) ` 17.40 7 7
(e) None of these (a ) 27 (b) 22
9 9
3. By selling an article for ` 96, double profit is obtained than
the profit that would have been obtained by selling it for 7
(c) 25 (d) Data inadequate
` 84. What is the cost price of the article? 9
(a) ` 72.00 (b) ` 75.00 (e) None of these
(c) ` 70.00 (d) ` 68.00 7. A builder purchased a plot of land for ` 80 lakh and
(e) None of these constructed a five-storey building inclusive of ground floor
PROFIT AND LOSS B-45
on it. How much should he charge for each flat to make 25% 15. A shopkeeper sold an article offering a discount of 5% and
profit on his investment on land, if there are five flats on earned a profit of 23.5%. What would have been the
each storey? percentage of profit earned if no discount had been offered?
(a) ` 50000 (b) ` 100000 (a) 28.5 (b) 27.675
(c) ` 500000 (d) ` 2000000 (c) 30 (d) Data inadequate
(e) None of these (e) None of these
8. A trader purchased on old bicycle for ` 480. He spent 20% 16. A shopkeeper sold sarees at ` 266 each after giving 5%
of the cost on its repair. If he wants to earn ` 144 as net discount on labelled price. Had he not given the discount,
profit on it, how much percentage should he add to the he would have earned a profit of 12% on the cost price.
purchase price of the bicycle? What was the cost price of each saree?
(a) 50% (b) 48% (a) ` 280 (b) ` 260
(c) 96% (d) 100% (c) ` 38 mph (d) Data inadequate
(e) None of these (e) None of these
9. The price of 2 sarees and 4 shirts is ` 16000. With the same 17. The profit earned by selling an article for ` 832 is equal to
money one can buy 1 saree and 6 shirts. If one wants to buy the loss incurred when the same article is sold for ` 448.
12 shirts, how much shall one have to pay? What should be the sale price of the article for making 50
(a) ` 2,400 (b) ` 4,800 per cent profit?
(c) ` 1,200 (d) Cannot be determined
(a) ` 960 (b) ` 1060
(e) None of these
(c) ` 1,200 (d) ` 920
10. A shopkeeper bought 150 calculators at the rate of ` 250
(e) None of these
per calculator. He spent ` 2500 on transportation and
packing. If the marked price of calculator is ` 320 per 18. Prabhu purchased 30 kg of rice at the rate of ` 17.50 per kg
calculator and the shopkeeper gives a discount of 5% on and another 30 kg rice at a certain rate. He mixed the two
the marked price then what will be the percentage profit and sold the entire quantity at the rate of ` 18.60 per kg and
gained by the shopkeeper? made 20 per cent overall profit. At what price per kg did he
(a) 20% (b) 14% purchase the lot of another 30 kg rice?
(c) 15% (d) 16% (a) ` 14.50 (b) ` 12.50
(e) None of these (c) ` 15.50 (d) ` 13.50
11. A garment company declared 15% discount for whole sale (e) None of these
buyers. Mr Sachdev bought garments from the company 19. An article when sold for ` 200 fetches 25 per cent profit.
for ` 25,000 after getting discount. He fixed up the selling What would be the percentage profit/loss if 6 such articles
price of garments in such a way that he earned a profit of are sold for ` 1,056?
8% on original company price. What is the approximate (a) 10 pre cent loss (b) 10 per cent profit
total selling price? (c) 5 per cent loss (d) 5 per cent profit
(a) ` 28,000 (b) ` 29,000 (e) None of these
(c) ` 32,000 (d) ` 28,500 20. A shopkeeper gave an additional 20 per cent concession
(e) ` 29,500 on the reduced price after giving 30 per cent standard
12. A shopkeeper sold an article for ` 720 after giving 10% concession on an article. If Arun bought that article for `
discount on the labelled price and made 20% profit on the 1,120, what was the original price?
cost price. What would have been the percentage profit, (a) ` 3,000 (b) ` 4,000
had he not given the discount’? (c) ` 2,400 (d) ` 2,000
(a) 25% (b) 30% (e) None of these
(c) 23% (d) 28% 21. What per cent of selling price would be 34% of cost price if
(e) None of these gross profit is 26% of the selling price?
13. The difference between a discount of 35% and two (a) 17.16 (b) 74.00
successive discounts of 20% and 20% on a certain bill was (c) 25.16 (d) 88.40
` 22. Find the amount of the bill. (e) None of these
(a) ` 1,100 (b) ` 200 22. A man sold 10 eggs for 5 rupees and gained 20%.How many
(c) ` 2,200 (d) Data inadequate
eggs did he buy for 5 rupees?
(e) None of these
(a) 10 eggs (b) 12 eggs
14. A shopkeeper labels the price of articles 20% above the
(c) 14 eggs (d) 16 eggs
cost price. If he allows ` 31.20 off on a bill of ` 312, find his
profit per cent on the article? (e) None of these
23. A person sells 36 oranges per rupee and suffers a loss of
1 4%. Find how many oranges per rupee to be sold to have a
(a) 8 (b) 12
3 gain of 8%?
(a) 30 (b) 31
2 1
(c) 11 (d) 8 (c) 32 (d) 33
3 3
(e) None of these
(e) None of these
B-46 PROFIT AND LOSS
24. Coconuts were purchased at ` 150 per hundred and sold at 33. A dishonest dealer sells his goods at the cost price but still
` 2 per coconut. If 2000 coconuts were sold, what was the earns a profit of 25% by underweighing. What weight does
total profit made? he use for a kg?
(a) ` 500 (b) ` 1000 (a) 750 g (b) 800 g
(c) ` 1500 (d) ` 2000 (c) 825 g (d) 850 g
(e) None of these (e) None of these
25. A shopkeeper’s price is 50% above the cost price. If he 34. A shopkeeper marks up his goods to gain 35%. But he
allows his customer a discount of 30% what profit does he allows 10% discount for cash payment. His profit on the
make? cash transaction therefore, in percentage, is
(a) 5% (b) 10%
(c) 15% (d) 20% (a) 13 1 (b) 25
(e) None of these 2
26. A shopkeeper purchases 10 kg of rice at ` 600 and sells at a 1 1
(c) 21 (d) 31
loss as much the selling price of 2 kg of rice . Find the sale 2 2
rate of rice/ kg. (e) None of these
(a) ` 60 per kg (b) ` 50 per kg 35. A man sold two steel chairs for ` 500 each. On one he gains
(c) ` 80 per kg (d) ` 70 per kg 20% and on other, he loses 12%. How much does he gain or
(e) None of these lose in the whole transaction?
27 If 15 oranges are bought for a rupee, how many must be (a) 1.5% gain (b) 2% gain
sold for a rupee to gain 25%?
(c) 1.5% loss (d) 2% loss
(a) 12 (b) 10
(e) None of these
(c) 20 (d) 18
(e) None of these 36. A firm of readymade garments makes both men’s and
28. A man buys milk at ` 6 per litre and adds one third of water women’s shirts. Its average profit is 6% of the sales. Its
to it and sells mixture at ` 7.20 per litre. The gain is profit in men’s shirts average 8% of the sales and women’s
(a) 40% (b) 80% shirts comprise 60% of the output. The average profit per
sale rupee in women shirts is
(c) 60% (d) 25%
(e) None of these (a) 0.0466 (b) 0.0666
29. A milk man makes a profit of 20% on the sale of milk. If he (c) 0.0166 (d) 0.0366
were to add 10% water to the milk, by what %would his (e) None of these
profit increase? 37. A man purchases two watches at ` 560. He sells one at 15%
profit and other at 10% loss. Then he neither gains nor lose.
40
(a) 30 (b) Find the cost price of each watch.
3 (a) ` 224, ` 300 (b) ` 200, ` 300
(c) 22 (d) 10 (c) ` 224, ` 336 (d) ` 200, ` 336
(e) None of these (e) None of these
30. A grocer purchased 80 kg of sugar at ` 13.50 per kg and
38. A man bought a horse and a carriage for ` 3000. He sold the
mixed it with 120 kg sugar at ` 16 per kg. At what rate should
horse at a gain of 20% and the carriage at a loss 10%, thereby
he sell the mixture to gain 16%?
gaining 2% on the whole. Find the cost of the horse.
(a) ` 17 per kg (b) ` 17.40 per kg
(a) ` 1000 (b) ` 1200
(c) ` 16.5 per kg (d) ` 16 per kg
(e) None of these (c) ` 1500 (d) ` 1700
31. A dishonest fruit seller professes to sell his goods at the (e) None of these
cost price but weighs 800 grams for a kg weight. Find his 39. Two electronic musical instruments were purchased for
gain percent. ` 8000. The first was sold at a profit of 40% and the second
(a) 100% (b) 150% at loss of 40%. If the sale price was the same in both the
(c) 50% (d) 200% cases, what was the cost price of two electronic musical
(e) None of these instruments?
32. A shopkeeper purchased 150 identical pieces of calculators (a) ` 2000, ` 5000 (b) ` 2200, ` 5500
at the rate of ` 250 each. He spent an amount of ` 2500 on (c) ` 2400, ` 5000 (d) ` 2400, ` 5600
transport and packing. He fixed the labelled price of each (e) None of these
calculator at ` 320. However, he decided to give a discount 40. A man sells an article at a gain 15%. If he had bought it at
of 5% on the labelled price. What is the percentage profit 10% less and sold it for ` 4 less, he would have gained
earned by him ? 25%. Find the cost price of the article.
(a) 14 % (b) 15 % (a) ` 150 (b) ` 160
(c) 16 % (d) 20 % (c) ` 170 (d) ` 180
(e) None of these (e) None of these
PROFIT AND LOSS B-47
41. A man sells an article at 5% profit. If he had bought it at 5%
15 2
less and sold it for ` 1 less, he would have gained 10%. The (c) 5 (d) 8
cost price of the article is : 17 3
(a) ` 200 (b) ` 150 (e) None of these
(c) ` 240 (d) ` 280 44. The cost price of 20 articles is equal to the selling price of
(e) None of these 25 articles. The loss percent in the transaction is
42. Five kg of butter was bought by a shopkeeper for ` 300. (a) 5 (b) 20
One kg becomes unsaleable. He sells the remaining in such (c) 25 (d) 30
a way that on the whole he incurs a loss of 10%. At what (e) None of these
price per kg was the butter sold? 45. By selling 66 metres of cloth a person gains the cost price
(a) ` 67.50 (b) ` 52.50 of 22 metres. Find the gain per cent.
(c) ` 60 (d) ` 72.50 1
(a) 22% (b) 22 %
(e) None of these 2
43. A fruitseller sells 8 oranges at a cost price of 9. The profit 1
per cent is (c) 33% (d) 33 %
3
1 1 (e) None of these
(a) 12 (b) 11
2 9

ANSWER KEY
1 (a) 7 (e) 13 (c) 19 (b) 25 (a) 31 (a) 37 (c) 43 (a)
2 (c) 8 (a) 14 (a) 20 (d) 26 (b) 32 (a) 38 (b) 44 (c)
3 (a) 9 (a) 15 (c) 21 (c) 27 (a) 33 (b) 39 (d) 45 (d)
4 (e) 10 (b) 16 (e) 22 (b) 28 (b) 34 (c) 40 (b)
5 (a) 11 (c) 17 (a) 23 (c) 29 (b) 35 (a) 41 (a)
6 (a) 12 (e) 18 (d) 24 (b) 30 (b) 36 (a) 42 (a)

Answers &
Explanations
5. (a) List price of calculator
2
1. (a) Percentage profit = ´100 < 20% 82.50
12 – 2 = ´ 100 = ` 275
30
2. (c) CP = 20 × 15 + 30 × 13 = ` 690
Deepa bought calculator in
4 1 275 ´ 0.70 ` 192.50
[ SP = of 690 × = ` 18.40
3 50 6. (a) Original selling price of TV set
3. (a) Let the cost price of the article be ` x. 16560´100
=` = `18400
Then, 2(84 – x) = 96 – x 90
l68 – 2x = 96 – x [ x = `72 16560 ´100
4. (e) If no discount is given, selling price of TV Cost price of the TV < = `14400
115
100 at no discount, % profit will be
= 17940 ´ = `19500
92 ∋18400 ,14400(´100 7
< < 27 %
14400 9
100
Cost price of TV = 17940 ´ = ` 15000 7. (e) We do not know the total investment of builder,
119.60
because in the question construction cost is not given.
Hence, ‘None of these’ is the answer.
19500 - 15000
Gain % = ´ 100 = 30% 8. (a) Purchase price = ` 480
15000 Repair cost = 20% of ` 480 = ` 96
B-48 PROFIT AND LOSS

\ Total cost = ` 480 + ` 96 = ` 576


100 ´ (100 + 23.5)
Net profit = ` 144 Then printed price =
\ Selling price = Total cost + Net profit (100 - 5)
= ` 576 + 144 = ` 720 100 ´ 247
Now, Selling price - Purchase price = = ` 130
190
= ` 720 – ` 480 = ` 240
Hence, required % Profit = 130 – 100 = 30%
` 240 100
\ Reqd percentage = ` 480 × 100 = 50% 16. (e) Marked price = 266 × ` 280
95
9. (a) Let the price of one saree and one shirt be ` x and ` y
100
respectively. Cost price = 280´ = ` 250
112
Then, 2x + 4y = 1600
17. (a) Let the profit or loss be ` x
or, x + 2y = 800 ... (i)
Also, x + 6y = 1600 ... (ii) 384
and 832 – x = 448 + x or, x = = ` 192
Solving equations (i) and (ii), we get 2
4y = 800 or, y = 200 [ Cost price of the article = 832 – x = 448 + x = ` 640
\ cost of 12 shirts = 12 × 200 = ` 2400 150
10. (b) CP of 150 calculators = 150 × 250 = ` 37,500. [ SP of the article = 640 × < ` 960
100
[ total CP = 37,500 + 2500 = ` 40,000
18. (d) Let he purchase of ` x/kg.
Marked price of 150 calculators = 150 × 320 = ` 48,000
120
95 \ (525 + 30 x) ´ = 60 ´ 18.60
Selling price after discount = 48000 × 100
100 Þ x = ` 13.5 / kg.
= ` 45,600
200
19. (b) CP = ´ 100 = ` 160
45, 600 , 40, 000 125
[ Percentage profit = ´100 < 14%
40, 000 \ CP of 6 articles = 6 × 160 = ` 960
\ profit = 1056 – 960 = 96
25000 ´ 100
11. (c) Original S.P. of company = = ` 29411.8 96
85 Percentage profit = ´ 100 = 10%
960
29411.8 ´ 108
\ Approximate total S.P. = ; ` 32000 100 100
100 20. (d) Original price < 1120´ ´ < ` 2000
70 80
720 ´ 100 21. (c) Let the selling price of the article be ` 100.
12. (e) Cost price = = `600; \ Profit = ` 26
120
\ Cost price of the article = 100 – 26 = ` 74
720 ´ 100
S.P. at no discount = = `800 \ 34 ´ 74
90 Reqd. % = = 25.16%
100
200 ´ 100 1
\ % profit = = 33 % 5 1
600 3 22. (b) S.P. for 1 egg = ` = `
10 2
20 ´80
13. (c) Successive discount = 20% + 100 1 5
100 \ C. P. for 1 egg = ´ =`
(100 + 20) 2 12
= 20 + 16 = 36%
Þ He bought 12 eggs for 5 rupees.
Difference in discount = 36 – 35 = 1%
[ Bill amount = 22 × 100 = ` 2200 1
23. (c) : (100 - 4) :: x : (100 + 8)
14. (a) Let the CP = ` 100 Marked price = ` 120 36

312 1 108 1
Discount = ´ < 10% = ` 12 Þ x= =
31.2 100 96 ´ 36 32
He sells 32 oranges per rupee.
SP = 120 – 12 = ` 108 Profit% = 108 – 100 = 8% 1 50 3
15. (c) Giving no discount to customer implies selling the 24. (b) C.P. for one coconut = ` = `
1 00 2
product on printed price. Suppose the cost price of S.P. for one coconut = ` 2
the article is ` 100.
PROFIT AND LOSS B-49

3 1
30. (b) C.P. of 200 kg of mixture = `(80 × 13.50 + 120 × 16)
Profit on one coconut = 2 - = ` = ` 3000.
2 2
æ 116 ö
1 S.P. = 116% of `3000 = ` ç ´ 3000 ÷ = ` 3480.
\ Profit on 2000 coconut = ´ 2000 = ` 1000 è 100 ø
2
25. (a) Let C.P. = `100, then M. P. = ` 150 æ 3480 ö
\ Rate of S.P. of the mixture = ` ç per kg
S.P. = 70% of 150 = ` 105 è 200 ÷ø

105 - 100 = ` 17.40 per kg.


\ % profit = ´ 100 = 5% 31. (a) He gives 800 grams but charges the price of 1000 grams
100
(1 kg) Þ on every 800 grams, he gains (1000 – 800)
26. (b) Let S. P. = ` x per kg grams i.e. 200 grams.
\ S.P. of 2 kg of rice = ` 2x = Loss
200
Now, Loss = C.P. – S.P. \ His gain % = ´100% = 25%
800
2x = 600 – 10 x
Þ x = ` 50 per kg error
Short cut : Gain % = = 100%
true weight – error
1
27. (a) C.P. for one orange = `
15 32. (a) C.P. of 150 calculators
= 150 × 250 + 2500 = 37500 + 2500 = ` 40000
(100 + 25) 1 125 1 Labelled price of 150 calculators
Then S.P. = ´ = =
100 15 100 ´ 15 12 = 150 × 320 = `48000
Discount allowed = 5%
1 \ S.P. of 150 calculators
Hence S.P. for one orange = `
12 = 48000 – 5% = ` 45600
\ 12 oranges must be sold for a rupee 5600
28. (b) C.P. of one litre = ` 6 \ Profit % = ´ 100 = 14
40000
After adding water to it
True weight 100 + gain%
2 33. (b) =
One has to pay ` 7.20 for litre of milk. False weight 100 + x
3
Here S.P. = C. P. \ x = 0
2
So S.P. of litre of milk = ` 7.20 1000×100
3 Þ False weight= = 800 gm
125
7.20 ´ 3 34. (c) Let cost Price = ` 100
Þ S.P. of 1 litre of milk = ` = ` 10.80
2 \ Marked price = ` 135
After discount, selling price = 135 – 13.5 = 121.5
Q S.P. > C.P.
\ Profit% = 121.5 – 100 = 21.5%
10.80 - 6 4.80 35. (a) S.P. of the 1st chair = ` 500
Hence gain = ´ 100 = ´ 100 Gain = 20%
6 6
= 0.80 × 100 = 80% 500 ´ 100 500 ´ 100
\ C.P. of the 1st chair = =
29. (b) Let profit per litre = ` 20 100 + 20 120
So, C.P. / litre = ` 100 1250
S.P. / litre = ` 120 =
3
On adding 10% water to the milk S.P. of the 2nd chair = ` 500
9 Loss = 12%
C.P. / litre = ` 100
10 500 ´ 100 500 ´ 100
\ C.P. of the 2nd chair = =
9 100 - 12 88
S.P. / litre = ` 120
10 500 ´ 25 250 ´ 25 6250
= = =
22 11 11
120 ´ 10 400
S.P. / litre = ` =` Now S.P. of both the chairs = ` 1000
9 3
C.P. of both the chairs
400 1250 6250 13750 + 18750 32500
Þ Profit / litre = - 100 = 33.33 = + = =
3 3 11 33 33
% by which profit increases = 33.33 – 20 = 13.3
B-50 PROFIT AND LOSS
Difference of two selling prices is ` 115 – ` 112.50
\ Net gain = 1000 – 32500 = 500 = ` 2.50 and C.P. of the article is ` 100
33 33
But actual difference is ` 4.
500 33 500
Þ Gain % = ´ 100 = ´ 100 100
32500 33 32500 \ C.P. = ´ ` 4 = ` 160.
2.50
100 20 41. (a) Let the CP of the article be ` x.
= = = 1.5% (To one place of decimal)
65 13 105x
Then, SP = `
36. (a) Women's shirt comprise 60% of the output. 100
\ Men's shirts comprise (100 – 60) = 40% of the
out put. 95x 105x
Now, new CP = ` and new SP = -1
\ Average profit from men's shirt = 8% of 40 100 100
= 3.2 out of 40 According to the question
Overall average profit = 6 out of 100
105x 95 10 ´ 95x
\ Average profit from women's shirts = 2.8 out of 60 -1- =
100 100 100 ´100
i.e. 0.0466 out of each shirt.
37. (c) Here, in whole transaction, there is neither gains nor \ x = ` 200
loss, therefore, 42. (a) Let S.P. = ` x per kg
Amount of gain in one watch \ S.P. of 4 kg = ` 4x
= Amount of loss in other watch 100 – 40
\ 4x = ´ 300
Þ 0.15 ´ CP1 = 0.10 ´ CP2 100

CP1 0.10 2 270


Þ = = Þ x= = ` 67.50
CP2 0.15 3 4
Also CP1 + CP2 = 560 43. (a) Let C.P. of one orange = ` 1
Then C.P. of 8 oranges = ` 8
2 S.P of 8 oranges = ` 9
\ CP1 = (2 + 3) ´ 560 = ` 224
9-8 100 1
and CP2 = 560 – 224 = ` 336 \ Gain % = ´ 100 = = 12 %
8 8 2
38. (b) Let the C.P. of horse = ` x
44. (c) Let C.P. of 1 article = ` 1
Then the C.P. of carriage = ` (3000 – x)
then C.P. of 25 articles = ` 25
20% of x – 10% of (3000 – x) = 2% of 3000
and S.P. of 25 articles = ` 20
x (3000 – x)
Þ - = 60 \ loss % = 25 - 20 ´ 100 = 25%
5 10 20
Þ 2x - 3000 + x = 600 45. (d) Let C.P. of one metre of cloth = ` 1
Þ 3x = 3600 Þ x = 1200 then C.P. of 66 metres of cloth = ` 66
39. (d) Here, SP1 = SP2 Gain = C.P. of 22 metres = ` 22

CP1 6 3 22 1
Þ 140 CP1 = 60CP2 Þ = = % gain = ´ 100 = 33 %
CP2 14 7 66 3
Shortcut method :
3
\ CP1 = ´ 8000 = ` 2400 If on selling ‘x’ articles, a man gains equal to the C.P. of
(3 + 7)
and CP2 = 8000 – 2400 = ` 5600 y
‘y’ articles, then % gain = ´100
40. (b) Let the C.P. be ` 100 x
First S.P. = `115 22 1
Second C.P. = ` 90 \ % gain = ´ 100 = 33 %
66 3
Second S.P = 125% of ` 90 = ` 112.50
Simple and
Compound
6 Chapter Interest
INTEREST R = 25%
Interest is the fixed amount paid on borrowed money. 100 ´ S.I. 100 ´ 2P
\ T = = = 8 years
The sum lent is called the Principal. P´R P ´ 25
The sum of the principal and interest is called the Amount. (ii) Compound Interest : Money is said to be lent at compound
Interest is of two kinds : interest when at the end of a year or other fixed period, the
(i) Simple Interest : When interest is calculated on the original interest that has become due is not paid to the lender, but is
principal for any length of time, it is called simple interest. added to the sum lent, and the amount thus obtained becomes
Principal ´ Time ´ Rate the principal in the next year or period. The process is repeated
Simple interest =
100 until the amount for the last period has been found. Hence,
When the interest charged after a certain specified time
P´ R´T period is added to form new principal for the next time period,
i.e. S.I. =
100 the interest is said to be compounded and the total interest
Amount = Principal + Interest accrued is compound interest.
PRT æ RT ö éæ r ö
n ù
i.e. A= P+ I = P + = P ç1 + ÷ C.I. = P êç1 + ÷ –1ú ;
100 è 100 ø êëè 100 ø úû
100 ´ S.I. n
Amount (A) = P æç1 + ö
Principal (P) = r
R´T è 100 ÷ø
If rate of compound interest differs from year to year, then
100 ´ S.I.
Rate (R) =
T´P æ r öæ r öæ r ö
Amount = P ç1 + 1 ÷ ç1 + 2 ÷ ç1 + 3 ÷ .......
è 100 ø è 100 ø è 100 ø
100 ´ S.I.
Time (T) =
P´R
If rate of simple interest differs from year to year, then Since r is calculated half-yearly therefore the rate per cent will
become half and the time period will become twice, i.e.,
(R1 + R 2 + R 3 + .....)
S.I. = P ´ 2n
100 æ r ö
A = P ç1 + ÷
Example 1 : Find the interest to be paid on a loan of `6000 at 5% è 2 ´ 100 ø
per year for 5 years
Compound interest – when interest is calculated quarterly
Sol. P = `6000, R = 5% and T = 5 years
Since 1 year has 4 quarters, therefore rate of interest will become
P ´ R ´ T 6000 ´ 5 ´ 5
S.I. = = = `1500 1
th of the rate of interest per annum, and the time period will be
100 100 4
Example 2 : Find the amount to be paid back on a loan of 4 times the time given in years
`18,000 at 5.5% per annum for 3 years Hence, for quarterly interest
Sol. P = `18000, R = 5.5%, T = 3 years
4´n 4n
æ r/4ö æ r ö
P ´ R ´ T 18000 ´ 5.5 ´ 3 A = P ç1 + ÷ = P ç1 + ÷
S.I. = = = `2970 è 100 ø è 400 ø
100 100
Amount = P + I = 18000 + 2970 = `20970 Example 4 : Find the compound interest on `25625 for 12 months
Example 3 : In how many years will a sum of money triple itself, at at 16% per annum, compounded quarterly.
25% per annum simple interest. Sol. Principal (P) = `25625
Sol. Let the sum of money be ` P. So, A = 3P
16
and S.I. = A – P = 3P – P = 2P Rate (r) = 16% = % = 4%
4
B-52 SIMPLE AND COMPOUND INTEREST
Time = 12 months = 4 quarters Example 5 : The difference between compound interest and simple
æ 4 ö
4
æ 26 ö
4 interest on a certain amount of money at 5% per annum for 2
A = 25625 ç1 + ÷ = 25625 ç ÷ years is `15. Find the sum :
è 100 ø è 25 ø
(a) `4500 (b) `7500
26 26 26 26
25625 ´ ´ ´ ´ = `29977.62 (c) `5000 (d) `6000
25 25 25 25
Sol. (d) Let the sum be `100.
C.I. = A – P = 29977.62 – 25625 = `4352.62
Therefore, SI = 100 ´ 5 ´ 2 = Rs 10
Difference between Compound Interest and Simple Interest 100
When T = 2 2
æ 5 ö
2 and CI = 100ç1 + ÷ - 100
æ R ö è 100 ø
(i) C.I. – S.I. = P ç ÷
è 100 ø
21´ 21 41
R ´ S.I. = 100 ´ - 100 = Rs
(ii) C.I. – S.I. = 20 ´ 20 4
2 ´ 100 41 1
When T = 3 Difference of CI and SI = - 10 =
4 4
PR 2 æ 300 + R ö 1
(i) C.I. – S.I. = ç ÷ If the difference is , the sum = 100
104 è 100 ø 4
Þ If the difference is `15, the sum
S.I. éæ R ö æ R öù
2
= 400 × 15 = `6000
(ii) C.I. – S.I. = 3 êç 100 ÷ + 3 ç 100 ÷ ú
êëè ø è ø úû OR
15 ´ (100)2 15 ´100 ´100
The sum is = = = `6000.
2 25
(5)

EXERCISE
1. Arun borrowed a sum of money from Jayant at the rate of (a) 7 p.c.p.a. (b) 8 p.c.p.a.
8% per annum simple interest for the first four years, 10% (c) 9 p.c.p.a. (d) Data inadequate
per annum for the next six years and 12% per annum for the
(e) None of these
period beyond ten years. If he pays a total of ` 12,160 as
5. Ashok borrowed some money at the rate of 6% p.a. for the
interest only at the end of 15 years, how much money did
he borrow? first two years, at the rate of 9% p.a. for the next three years
and at the rate of 14% p.a. for the period beyond five years.
(a) ` 8000 (b) ` 10,000
If he pays a total interest of ` 11,400/ - at the end of nine
(c) ` 12,000 (d) ` 9,000
years, how much money did he borrow?
(e) None of these
(a) ` 16,060 (b) ` 14,000
2. A sum fetched total simple interest of ` 4016.25 at the rate
(c) ` 18,000 (d) ` 12,000
of 9 p.c.p.a. in 5 years. What is the sum?
(a) ` 8925 (b) ` 8032.50 (e) None of these
(c) `4462.50 (d) ` 8900 6. A certain amount earns simple interest of `1750 after 7
(e) None of these years. Had the interest been 2% more, how much more
3. At a simple interest ` 800 becomes ` 956 in three years. If interest would it have earned?
the interest rate, is increased by 3%, how much would ` 800 (a) ` 35 (b) ` 350
become in three years? (c) ` 245 (d) Cannot be determined
(a) ` 1020.80 (b) ` 1004 (e) None of these
(c) ` 1028 (d) Data inadequate 7. What will be the difference in simple and compound interest
(e) None of these on ` 2000 after three years at the rate of 10 percent per
4. On ` 3,000 invested at a simple interest rate 6 p.c.p.a, annum?
` 900 is obtained as interest in certain years. In order to (a) ` 160 (b) `42
earn ` 1,600 as interest on ` 4,000 in the same number of (c) ` 62 (d) ` 20
years, what should be the rate of simple interest? (e) None of these
SIMPLE AND COMPOUND INTEREST B-53
8. Nikhilesh invested certain amount in three different schemes (a) `15,000 (b) ` 13,500
A, B and C with the rate of interest 10 p.c.p.a., 12 p.c.p.a. (c) ` 12,000 (d) Cannot be determined
and 15 p.c.p.a. respectively. If the total interest accrued in (e) None of these
one year was ` 3200 and the amount invested in scheme C 16. Seema invested an amount of ` 16000 for two years at
was 150% of the amount invested in scheme A and 240% of compound interest and received an amount of ` 17640 on
the amount invested in scheme B, what was the amount maturity. What is the rate of interest?
invested in scheme B? (a) 8 pcpa (b) 5 pcpa
(a) ` 8000 (b) ` 5000 (c) 4 pcpa (d) Data inadequate
(c) ` 6500 (d) Cannot be determined (e) None of these
(e) None of these 17. Amit Kumar invested an amount of ` 15,000 at compound
9. Aniket deposited two parts of a sum of ` 25000 in different interest rate of 10 pcpa for a period of two years. What
banks at the rates of 15% per annum and 18% per annum amount will he receive at the end of two years?
respectively. In one year he got ` 4050 as the total interest. (a) ` 18,000 (b) ` 18,500
What was the amount deposited at the rate of 18% per (c) ` 17,000 (d) ` 17,500
annum? (e) None of these
(a) ` 9000 (b) ` 18000 18. In a business A and C invested amounts in the ratio 2:1.
(c) ` 15000 (d) Data inadequate Whereas the ratio between amounts invested by A and B
(e) None of these was 3:2. If ` 1,57,300 was their profit, how much amount did
10. Mr X invested an amount for 2 years @ 15 p.c.p.a at simple B receive?
interest. Had the interest been compounded, he would have (a) ` 72,600 (b) ` 48,400
earned ` 450/- more as interest. What was the amount (c) ` 36,300 (d) ` 24,200
invested? (e) None of these
(a) ` 22000 (b) ` 24000 19. Mr. Sane invested a total amount of ` 16,500 for two years
(c) ` 25000 (d) Data inadequate in two schemes A and B with rate of simple interest 10
p.c.p.a. and 12 p.c.p.a. respectively. If the total amount of
(e) None of these
interest earned was ` 3,620, what was the amount invested
11. Difference between the compound interest and the simple
in scheme B?
interest accrued on an amount of ` 18000, in two years was
(a) ` 8,000 (b) ` 8,600
` 405. What was the rate of interest p.c.p.a?
(c) ` 8,150 (d) Data inadequate
(a) 16 (b) 12
(e) None of these
(c) 15 (d) Cannot be determined 20. The difference between the simple and the compound
(e) None of these interest
12. Anish borrowed ` 15000 at the rate of 12% and an other compounded every six months at the rate of 10% p.a. at the
amount at the rate of 15% for two years. The total interest end of two years is `. 124.05. What is the sum?
paid by him was ` 9000. How much did he borrow? (a) ` 10,000 (b) ` 6,000
(a) ` 32,000 (b) ` 33,000 (c) ` 12,000 (d) ` 8,000
(c) ` 30,000 (d) ` 35,000 (e) None of these
(e) None of these 21. Parameshwaran invested an amount of ` 12,000 at the simple
13. The compound interest on any sum at the rate of 5% for interest rate of 10 pcpa and another amount at the simple
two years is ` 512.50. Find the sum. interest rate of 20 pcpa. The total interest earned at the end
(a) ` 5200 (b) ` 4800 of one year on the total amount invested became 14 pcpa.
(c) ` 5000 (d) ` 5500 Find the total amount invested.
(e) None of these (a) ` 22,000 (b) ` 25,000
14. Mr Amin borrowed some money from Mr Vishwas. The rate (c) ` 20,000 (d) ` 24,000
of interest for first two years is 8% p.a., for the next three (e) None of these
years is 11 % p.a. and for the period beyond 5 years 14% 22. Raviraj invested an amount of ` 10,000 at compound interest
p.a. Mr Vishwas got an amount of `10920 as an interest at rate of 10 pcpa for a period of three years. How much amount
the end of eight years. Then what amount was borrowed by will Raviraj get after three years?
Mr Amin’? (a) ` 12,310 (b) ` 13,210
(a) ` 12000 (b) ` 15000 (c) ` 13,320 (d) ` 13,120
(c) ` 1400 (d) Data inadequate (e) None of these
(e) None of these 23. Nelson borrowed some money at the rate of 6 p.c.p.a. for
15. Mr Sridharan invested money in two schemes A and B, the first three years, 9 p.c.p.a. for the next five years and 13
offering compound interest @ 8 p.c.p.a. and 9 p.c.p.a. p.c.p.a. for the period beyond eight years. If the total interest
respectively. If the total amount of interest accrued through paid by him at the end of eleven years is ` 8,160, how much
the two schemes together in two years was ` 4818.30 and money did he borrow?
the total amount invested was ` 27,000, what was the (a) ` 12,000 (b) ` 10,000
amount invested in Scheme A? (c) ` 8,000 (d) Data inadequate
(e) None of these
B-54 SIMPLE AND COMPOUND INTEREST
24. Amal borrowed a sum of money with simple interest as per 33. A milk man borrowed ` 2,500 from two money lenders. For
the following rate structure: one loan, he paid 5% p.a. and for the other, he paid 7% p.a.
(a) 6 p.c. p.a. for the first three years The total interest paid for two years was ` 275. How much
(b) 8 p.c. p.a. for the next five years did he borrow at 7% rate?
(c) 12 p.c. p.a. for the next eight years (a) ` 600 (b) ` 625
If he paid a total of ` 5,040 as interest at the end of twelve (c) ` 650 (d) ` 675
years, how much money did he borrow? (e) None of these
(a) ` 8,000 (b) ` 10,000 34. What annual instalment will discharge a debt of ` 4,200 due
(c) ` 12,000 (d) ` 6,000 in 5 years at 10% simple interest?
(e) None of these (a) ` 500 per year (b) ` 600 per year
25. The simple interest in 14 months on a certain sum at the rate (c) ` 700 per year (d) ` 800 per year
of 6 per cent per annum is ` 250 more than the interest on (e) None of these
the same sum at the rate of 8 per cent in 8 months. How 35. Adam borrowed some money at the rate of 6% p.a. for the
much amount was borrowed? first two years, at the rate of 9% p.a. for the next three years,
(a) ` 15000 (b) ` 25000 and at the rate of 14% p.a. for the period beyond five years.
(c) ` 7500 (d) ` 14500 If he pays a total interest of ` 11,400 at the end of nine
(e) None of these years, how much money did he borrow?
26. On retirement, a person gets 1.53 lakhs of his provident (a) ` 10,000 (b) ` 12,000
fund which he invests in a scheme at 20% p.a. His monthly
(c) ` 14,000 (d) ` 16,000
income from this scheme will be
(e) None of these
(a) ` 2, 450 (b) ` 2,500
36. A person borrows ` 5000 for 2 years at 4% p.a. simple
(c) ` 2, 550 (d) ` 2, 600
(e) None of these 1
27. A sum was put at simple interest at a certain rate for 4 years interest. He immediately lends it to another person at 6 %
4
Had it been put at 2% higher rate, it would have fetched ` p.a. for 2 years. Find his gain in the transaction per year.
56 more. Find the sum. (a) ` 112.50 (b) ` 125
(a) ` 500 (b) ` 600 (c) ` 150 (d) ` 167.50
(c) ` 700 (d) ` 800 (e) None of these
(e) None of these 37. A certain amount earns simple interest of ` 1750 after 7
28. Simple interest on a certain sum is 16 over 25 of the sum.
years Had the interest been 2% more, how much more
Find the rate per cent and time, if both are equal.
interest would it have earned?
(a) 8% and 8 years (b) 6% and 6 years
(a) ` 35 (b) ` 245
(c) 10% and 10 years (d) 12 % and 12 years
(c) ` 350 (d) Cannot be determined
(e) None of these
(e) None of these
29. The simple interest on ` 200 for 7 months at 5 paise per
rupee per month is 38. What will be the ratio of simple interest earned by certain
(a) ` 70 (b) ` 7 amount at the same rate of interest for 6 years and that for
(c) ` 35 (d) ` 30.50 9 years?
(e) None of these (a) 1 : 3 (b) 1 : 4
30. A father left a will of ` 68,000 to be divided between his two (c) 2 : 3 (d) Data inadequate
sons aged 10 years and 12 years such that they may get (e) None of these
equal amount when each attains the age of 18 years If the 39. Two equal sums of money were invested, one at 4% and
money is reckoned at 10% p.a., find how much each gets at the other at 4.5%. At the end of 7 years, the simple interest
the time of the will. received from the latter exceeded to that received from the
(a) ` 30,000, ` 38,000 (b) ` 28,000, ` 40,000 former by ` 31.50. Each sum was :
(c) ` 32,000, ` 36,000 (d) Cannot be determined. (a) ` 1,200 (b) ` 600
(e) None of these (c) ` 750 (d) ` 900
31. If there are three sum of money P,Q and R so that P is the (e) None of these
simple interest on Q and Q is the simple interest of R, rate % 40. Nitin borrowed some money at the rate of 6% p.a. for the
and time are same in each case, then the relation of P, Q and first three years, 9% p.a. for the next five years and 13% p.a.
R is given by for the period beyond eight years If the total interest paid
(a) P2 = QR (b) Q2 = PR by him at the end of eleven years is ` 8160, how much
2
(c) R = PQ (d) PQR = 100 money did he borrow?
(e) None of these (a) ` 8000 (b) ` 10,000
32. In how many minimum number of complete years, the (c) ` 12,000 (d) Data inadequate
interest on ` 212.50 P at 3% per annum will be in exact (e) None of these
number of rupees? 41. An automobile financier claims to be lending money at simple
(a) 6 (b) 8 interest, but he includes the interest every six months for
(c) 9 (d) 7 calculating the principal. If he is charging an interest of
(e) None of these 10%, the effective rate of interest becomes :
SIMPLE AND COMPOUND INTEREST B-55
(a) 10% (b) 10.25% 47. Mr. Thomas invested an amount of ` 13,900 divided in two
(c) 10.5% (d) None of these different schemes A and B at the simple interest rate of 14%
(e) None of these p.a. and 11% p.a. respectively. If the total amount of simple
42. A lent ` 5000 to B for 2 years and ` 3000 to C for 4 years on interest earned in 2 years be ` 3508, what was the amount
simple interest at the same rate of interest and received invested in Scheme B?
` 2200 in all from both of them as interest. The rate of interest (a) ` 6400 (b) ` 6500
per annum is: (c) ` 7200 (d) ` 7500
(a) 5% (b) 7% (e) None of these
48. An amount of ` 1,00,000 is invested in two types of shares.
1
(c) 7 % (d) 10% The first yields an interest of 9% p.a. and the second, 11%
8
(e) None of these 3
p.a. If the total interest at the end of one year is 9 % , then
43. A sum of ` 725 is lent in the beginning of a year at a certain 4
rate of interest. After 8 months, a sum of ` 362.50 more is the amount invested in each share was:
lent but at the rate twice the former. At the end of the year, (a) ` 52,500; ` 47,500 (b) ` 62, 500; ` 37,500
` 33.50 is earned as interest from both the loans. What was (c) ` 72,500: ` 27,500 (d) ` 82, 500; ` 17,500
the original rate of interest? (e) None of these
(a) 3.6% (b) 4.5% 49. David invested certain amount in three different schemes
(c) 5% (d) 3.46% A, B and C with the rate of interest 10% p.a., 12% p.a. and
(e) None of these 15% p.a. respectively. If the the total interest accrued in
44. The difference between the simple interest received from one year was ` 3200 and the amount invested in Scheme C
two different sources on ` 1500 for 3 years is ` 13.50. The was 150 % of the amount invested in Scheme A and 240% of
difference between their rates of interest is: the amount invested in Scheme B, what was the amount
(a) 0.1% (b) 0.2% invested in Scheme B?
(c) 0.3% (d) 0.4% (a) ` 5000 (b) ` 6500
(e) None of these (c) ` 8000 (d) cannot be determined
45. The rates of simple interest in two banks A and B are in the (e) None of these
ratio 5 : 4. A person wants to deposit his total savings in 50. A person invested in all ` 2600 at 4%, 6% and 8% per annum
two banks in such a way that he received equal half-yearly simple interest. At the end of the year, he got the same
interest from both. He should deposit the savings in banks interest in all the three cases. The money invested at 4% is:
A and B in the ratio. (a) ` 200 (b) ` 600
(a) 2 : 5 (b) 4 : 5 (c) ` 800 (d) ` 1200
(c) 5 : 2 (d) 5 : 4 (e) None of these
(e) None of these 51. Divide ` 2379 into 3 parts so that their amounts after 2, 3
46. The price of a T.V. set worth ` 20,000 is to paid in 20 and 4 years respectively may be equal, the rate of interest
instalments of ` 1000 each. If the rate of interest be 6% per being 5% per annum at simple interest. The first part is:
annum, and the first instalment be paid at the time of (a) 759 (b) 792
purchase, then the value of the last instalment covering the (c) 818 (d) 828
interest as well will be : (e) None of these
(a) ` 1050 (b) ` 2050
(c) ` 3000 (d) None of these
(e) ` 2020

ANSWER KEY
1 (a) 7 (c) 13 (c) 19 (a) 25 (a) 31 (b) 37 (d) 43 (d) 49 (a)
2 (a) 8 (b) 14 (a) 20 (d) 26 (b) 32 (b) 38 (c) 44 (c) 50 (d)
3 (c) 9 (e) 15 (c) 21 (c) 27 (c) 33 (b) 39 (d) 45 (b) 51 (d)
4 (b) 10 (e) 16 (b) 22 (e) 28 (a) 34 (c) 40 (a) 46 (d)
5 (d) 11 (c) 17 (e) 23 (c) 29 (a) 35 (b) 41 (b) 47 (a)
6 (d) 12 (b) 18 (b) 24 (e) 30 (c) 36 (a) 42 (d) 48 (b)
B-56 SIMPLE AND COMPOUND INTEREST

Answers &
Explanations
8. (b) Ratio of Nikhilesh’s investments in different schemes
1. (a) Let the Principal = P
P ´ 8 ´ 4 P ´ 10 ´ 6 P ´ 12 ´ 5 150 ´ 100
+ + = 100 : :150 = 8 : 5 :12
Then = 12160 240
100 100 100
Þ 152P = 12160 ×100 Now, according to the question,
12160 ´ 100 8k ´10 5k ´12 12 k ´15
or = ` 8000 + + = 3200
152 100 100 100
2. (a) Let the sums be ` P.
Now, 45% of P = 4016.25 or, 80k + 60k + 180k = 3200 × 100
or, P = ` 8925 or, 320k = 3200 × 100 or, k = 1000
956 , 800 \ amount invested in scheme B willl be
3. (c) Rate of interest = ´100 < 6.50% = 1000 × 5 = ` 5000
3´800
9. (e) Let the amount deposited at the rate of 15% per annum
800 ´ 9.5 ´3 be ` x.
[ Amount = 800 +
100 15% of x + 18% of (25000 – x) = 4050
= 800 + 228 = `1028 or, 15% of x + 18% of 25000 – 18% of x = 4050
900 ´100 or, 3% of x = 4500 – 4050 = 450 Þ x = ` 15000
4. (b) Time = = 5 years
3000 ´6 [ Amount deposited at 18%
1600 ´100 = (25000 – 15000 =) ` 10000
Rate = < 8%
5´ 4000 30p é æ 15 ö÷ ù
10. (e) ∗ 450 < ê p çç1 ∗ ÷
÷ , pú
p´r´t 100 êë èç 100 ø úû
5. (d) We have, SI =
100
Þ p = ` 20,000.
p ´ 6 ´ 2 p ´ 9 ´ 3 p ´ 14 ´ 4
\ 11400 = + + 405 ´ 100 ´ 100
100 100 100 11. (c) Rate % = = 15%
or, 12p + 27p + 56p = 11400 × 100 18000
or, 95p = 11400 × 100 12. (b) Let x be the other amount
\ p = ` 12000
6. (d) Let p and r be the principal amount and rate of interest 3x
respectively. [ + 3600 = 900 Þ x = `18000
10
p´r´7 [ total borrowed sum = 33000
Then, = 1750
100 13. (c) Let the sum be ` x.
or, pr = 25000
éæ ù
5 ö÷ æ 441 , 400 ö÷
2
p ´ (r + 2)´ 7 512.50 = x êêçç1 ∗ ÷ ,1úú < x çç ÷
Now, SI = ç
è ÷
ø èç 400 ø÷
100 êë 100 úû
We have to find the value of
p ´ (r + 2) ´ 7 p ´ r ´ 7 512.50´ 400
- = M – 1750 [ x< = ` 5000
100 100 41
M = SI when the rate of interest is 2% more. When we solve 14. (a) Let ‘x’ be the amount borrowed by Mr Amin.
this equation, we find that we have two variables and one
equation. Therefore, can’t be determined the correct answer. x ´ 2 ´ 8 x ´ 3 ´ 11 x ´ 3 ´ 14
\ + + = 10920
7. (c) For 3 years: 100 100 100
Sum ´ (rate) 2 (300 + rate) 91 10920´100
Diff. = x = 10920 or x = < 12000
(100)3 or,
100 91
2000 ´ 10 ´ 10 ´ 310 15. (c) Let, in scheme A, Sridharan invest ` x.
= = ` 62 Then, his investment in scheme B = ` (27000 – x).
100 ´ 100 ´ 100
SIMPLE AND COMPOUND INTEREST B-57
Now, 21. (c) Let the amount invested at 20% rate be ` x. According
2 2 to the question,
æ 8 ö æ 9 ö
x ç1 + + (27000 – x) ç 1 +
è 100 ÷ø è 100 ø
÷
12000 ´
10
∗ x´
20
< (12000 ∗ x ) ´
14
100 100 100
– 27000 = 4818.30
or, x(1.08)2 + (27000 – x)(1.09)2 = 31818.30 x 7
or, 1.1664x + 32078.7 – 1.1881x = 31818.30 or, 1200 ∗ < 1680 ∗ x
5 50
or, 0.0217x = 260.4
x 7
260.4 or, , x < 480
or, x= = `12000 5 50
0.0217
3
A æ r ö
t
17640 æ r ö
2 or, x < 480
16. (b) = ç1 + = ç1 + 50
÷ or, ÷
P è 100 ø 16000 è 100 ø
[ x = ` 8000
21 r [ Total amount invested ` = (12000 + 8000)
= 1+ = ` 20000
20 100
Þ r = 5% æ 10 ö÷3
22. (e) Amount = =10000 ççç1+ ÷
2 è 100 ø÷
æ 10 ö
17. (e) Amount = 15000 ç 1 +
è 100 ÷ø
11 11 11
< 10000 ´ ´ ´ × = ` 13310
11 11 10 10 10
= 15000 ´ ´ = ` 18150 23. (c) Let ‘x’ be the amount borrowed by Nelson
10 10
18. (b) Ratio A : B = 3 : 2 and A : C = 2 : 1 x ´ 6 ´ 3 x ´ 9 ´ 5 x ´ 13 ´ 3
and + + = 8160
[ A: B:C= 6:4:3 100 100 100
Profit share of B = 4 × 1,57,300 = ` 48400
19. (a) % interest on total amount per annum 102
Þ x = 8,160
100
3620 ´ 100 362
= = %
16500 ´ 2 33 8,160 ´ 100
\ x= = ` 8, 000
Now, use Alligation method. 102
24. (e) Let x be the amount Amal borrowed.
Scheme Scheme \ 18% of x + 40% of x + 48% of x = 5040
A B or, 106% of x = 5040
10% 12%
5040
\ x= ´ 100 = ` 4754.71
362 106
%
33 25. (a) Let the amount be ` x.
x ´ 14 ´ 6 x ´ 8 ´ 8
æ 362 ö æ 362 ö From the question, - = 250
ç12 - ÷ ç 33 - 10 ÷ 1200 1200
è 33 ø è ø
\ x = ` 15000
Hence, ratio of amount invested in schemes A and B 26. (b) Let S.I. = ` x

æ 362 ö æ 362 ö 1.53 ´105 ´ 20


= çè 12 - - 10÷ = 17 :16 =
÷ :ç 100
33 ø è 33 ø

16 ´ 16500 27. (c) Difference in S.I. = P ´ T (R1 - R 2 )


Hence, amount invested in B = = ` 8000 100
(17 + 16)
20. (d) Let the sum be ` x. P´4´2
Þ 56 = (Q R1 – R2 = 2)
100
é æ 5 ö ù é x ´ 10 ´ 2 ù
4
Then, ê x ç 1 + ÷ - xú - ê ú = 124.05 56 ´ 100
êë è 100 ø úû ë 100 û Þ P= = ` 700
4´2
Solving the above eqns, we get x = ` 8,000.
B-58 SIMPLE AND COMPOUND INTEREST
35. (b) Let the sum borrowed be x. Then,
16 P´R ´R
28. (a) P=
25 100 æ x ´ 6 ´ 2 ö æ x ´ 9 ´ 3 ö æ x ´ 14 ´ 4 ö
ç 100 ÷ + ç 100 ÷ + ç 100 ÷ = 11400
1600 40 è ø è ø è ø
Þ R2 = ÞR= = 8%
25 5 æ 3x 27x 14x ö 95x
Also, time = 8 years Þç + + ÷ = 11400 Þ = 11400
è 25 100 25 ø 100
29. (a) Q Rate = 5 paise per rupee = 5%
200 ´ 5 ´ 7 æ 11400 ´ 100 ö
\ S.I. = = ` 70 Þ x =ç ÷ = 12000.
100 è 95 ø
30. (c) Let one gets = ` x Hence, sum borrowed = ` 12,000.
then, second gets = ` (68,000 – x) 36. (a) Gain in 2 years
Given : A1 = A2
éæ 25 2 ö æ 5000 ´ 4 ´ 2 ö ù
x ´10 ´ 8 (68000 - x) ´10 ´ 6 = ` êç 5000 ´ ´ ÷-ç ÷ú
x+ = (68, 000 - x) + ëè 4 100 ø è 100 øû
100 100
= ` (625 – 400) = ` 225.
Þ x[100 + 80] = (68, 000 - x)[100 + 60]
æ 225 ö
\ Gain in 1 year = ` ç ÷ = ` 112.50
180x è 2 ø
Þ = 68, 000 – x
160
37. (d) We need to know the S.I., principal and time to find the
Þ 34x = 68000 ´16 Þ x = ` 32, 000 rate. Since the principal is not given, so data is
\ second gets = ` 36,000 inadequate.
38. (c) Let the principal be P and rate of interest be R%.
Q´r´ t R´r´t
31. (b) P= and Q =
100 100 éæ P´ R ´6 öù
ê ç 100 ÷ ú 6PR 6
P Q r´t è øú =
Þ = = \ Required ratio = ê = = 2 : 3.
Q R 100 ê æ P ´ R ´ 9 ö ú 9PR 9
ê çè 100 ÷ø ú
\ Q2 = PR. ë û
3 51
32. (b) Interest for one year = ` 212.50 ´ ´1 = ` 39. (d) Difference of S.I. = ` 31.50
100 8
Let each sum be ` x. Then
Thus in 8 years, the interest is ` 51.
33. (b) Let he borrowed at 5% = ` x 1
\ he borrowed at 7% = ` (2500 – x) x´4 ´7
2 x´4´7
Now I1 + I2 = 275 - = 31.50
100 100
x ´ 5 ´ 2 (2500 - x) ´ 7 ´ 2
+ = 275 7 x 1 63
100 100 or ´ =
100 2 2
Þ 10x + 14 (2500 – x) = 27500
Þ 4x = 35000 – 27500 = 7500 or x = ` 900
Þ x = Rs 1875 40. (a) Let the sum be ` x. Then,
\ Sum borrowed at 7% rate = 2500 – 1875 = ` 625 æ x ´ 6 ´ 3 ö æ x ´ 9 ´ 5 ö æ x ´ 13 ´ 3 ö
34. (c) Shortcut method : ç 100 ÷ + ç 100 ÷ + ç 100 ÷ = 8160
è ø è ø è ø
If borrowed amount be ` M and it is to be paid in equal
instalments, then Þ 18 x + 45x + 39x = (8160 × 100) Þ102x = 816000
Þx = 8000.
ra n(n - 1)
M = na + ´ 41. (b) Let the sum be Rs 100. Then,
100 ´ Y 2
where Y = no. of instalments per annum æ 100 ´ 10 ´1 ö
S.I. for first 6 months = ` ç ÷ = ` 5.
a = annual instalment è 100 ´ 2 ø
Here, M = 4200, y = 1, r = 10, n = 5, a = ?
æ 105 ´10 ´1 ö
10a 5(5 - 1) S.I. for last 6 months = ` ç ÷ = ` 5.25.
4200 = 5a + ´ è 100 ´ 2 ø
100 2
So, amount at the end of 1 year = ` (100 + 5 + 5.25)
Þ 4200 = a [5 + 1] Þ 6a = 4200 = ` 110.25.
Þ a = ` 700 \ Effective rate = ` (110.25 – 100) = 10.25%.
SIMPLE AND COMPOUND INTEREST B-59

42. (d) Let the rate be R% p.a. Then, 49. (a) Let x, y and z be the amounts invested in schemes A, B
and C respectively. Then,
æ 5000 ´ R ´ 2 ö æ 3000 ´ R ´ 4 ö
ç ÷+ç ÷ = 2200 æ x ´10 ´ 1 ö æ y ´12 ´1 ö æ z ´15 ´ 1 ö
è 100 ø è 100 ø ç 100 ÷ + ç 100 ÷ + ç 100 ÷ = 3200
è ø è ø è ø
æ 2200 ö Þ 10x + 12y + 15z = 320000 ..... (i)
Þ 100R + 120R = 2200 Þ R = ç ÷ = 10.
è 220 ø 12
Now, z = 240% of y = y ..... (ii)
43. (d) Let the original rate be R%. Then, new rate = (2R)%. 5
3
And, z = 150% of x = x
æ 725 ´ R ´1 ö æ 362.50 ´ 2R ´ 1 ö 2
\ç ÷+ç ÷ = 33.50
è 100 ø è 100 ´ 3 ø 2 æ 2 12 ö 8
Þ x = z = ç ´ ÷ y = y ..... (iii)
Þ (2175 + 725)R = 33.50 ´ 100 ´ 3 = 10050 3 è3 5 ø 5
From (i), (ii) and (iii), we have :
10050 16y + 12y + 36y = 320000 Þ 64y = 320000 Þ y = 5000.
ÞR = = 3.46% \ Sum invested in Scheme B = ` 5000.
2900
50. (d) Let the parts be x, y and [2600 – (x + y)]. Then,
æ 1500 ´ R1 ´ 3 ö æ 1500 ´ R 2 ´ 3 ö x ´ 4 ´ 1 y ´ 6 ´1 [2600 - (x + y)] ´ 8 ´1
44. (c) ç ÷-ç ÷ = 13.50 = =
è 100 ø è 100 ø 100 100 100
y 4 2 2
1350 \ = = or y = x.
Þ 4500(R 1 - R 2 ) = 1350 Þ R 1 - R 2 = = 0.3% x 6 3 3
4500
45. (b) Let the savings be X and Y and the rates of simple æ 5 ö
x ´ 4 ´ 1 ç 2600 - x ÷ ´ 8
3 ø
interest be 5x and 4x respectively. So, =è
100 100
1 1 1 1 X 4
Then, X ´ 5x ´ ´ = Y ´ 4x ´ ´ or = , (7800 - 5x) ´ 8
2 100 2 100 Y 5 Þ 4x = Þ 52x = (7800 ´ 8)
3
i.e., X : Y = 4 : 5.
46. (d) Money paid in cash = ` 1000. æ 7800 ´ 8 ö
Þ x =ç ÷ = 1200.
Balance payment = ` (20000 – 1000) = ` 19000. è 52 ø
47. (a) Let the sum invested in Scheme A be ` x and that in \ Money invested at 4% = ` 1200.
Scheme B be ` (13900 – x). 51. (d) Let the parts be x, y and [2379 – (x + y)].

x ´14 ´ 2 ö é (13900 - x) ´11´ 2 ù æ 5 ö æ 5 ö


Then, æç ÷+ x + ç x ´ 2´ ÷ = y + ç y ´ 3´
è 100 ø êë 100 úû = 3508 è 100 ø è
÷
100 ø

Þ 28x – 22x = 350800 – (13900 × 22) Þ 6x = 45000 æ 5 ö


= z + çz ´ 4´ ÷
Þ x = 7500. è 100 ø
So, sum invested in Scheme B = ` (13900 – 7500)
= ` 6400.
11x 23y 6z 10k 20k 5k
48. (b) Let the sum invested at 9% be ` x and that invested at Þ = = =kÞ x = ,y= ,z=
10 20 5 11 23 6
11% be ` (100000 – x).
But x + y +z = 2379.
Then,
10k 20k 5k
æ x ´ 9 ´1 ö é (100000 - x) ´11´1 ù Þ + + = 2379
ç ÷+ê ú 11 23 6
è 100 ø ë 100 û Þ 1380 k + 1320 k + 1256 k = 2379 × 11 × 23 × 6
æ 39 1 ö 2379 ´ 11 ´ 23 ´ 6 3 ´11´ 23 ´ 6
= ç100000 ´ ´ ÷ Þk= =
è 4 100 ø 3965 5
9x + 1100000 - 11x 39000 æ 10 3 ´11´ 23 ´ 6 ö
Þ = = 9750 \ x =ç ´ ÷ = 828.
100 4 è 11 5 ø
Þ 2x = (1100000 – 975000) = 125000 Þ x = 62500. Hence, the first part is ` 828.
\ Sum invested at 9% = ` 62500.
Sum invested at 11% = ` (100000 – 62500) = ` 37500.
B-60 TIME & WORK / PIPES & CISTERNS

Time & Work /


Pipes & Cisterns
7 Chapter

In this chaper we will study the following two topics


v If an inlet pipe can fill a cistern in A hours, the part filled in
(i) Work and Time
1
(ii) Pipes and Cisterns 1 hour = (same as work and time fundamentals)
A
v If a person completes a job in n days then he will complete v If pipe A is 'x' times bigger than pipe B , then pipe A will
1 1
th part in one day.. take of the time taken by pipe B to fill the cistern.
n x th
Above fact is clear from the followong examples: Example 4 : It takes 4 hrs for pipe A to empty a 100 liter tank; if
Example 1 : Ram is twice as good as Shyam in work; Shyam will another pipe B which is double the size of pipe A is employed,
do a piece of work in 30 days; in how many days Ram will do the how long will it take to empty the tank?
work? Sol. Since the Pipe is double the size,
Sol. Since Ram is twice as good, 1
it will take time of the time taken by the smaller pipe
30 2
he will do the work in = 15 days
2 1
Therefore ´ 4 = 2 hrs
2
v If A and B can do a piece of work in X and Y days respectively
v If x and y fill/empty a cistern in 'm' and 'n' hours, then
xy together they will take
while working alone, they will together take days
( x + y)
to complete it. æ mn ö
ç ÷ hours to fill/empty the cistern
Example 2 : Shyam will do a piece of work in 30 days; Ram can do è m+nø
same work in 15 days; in how many days can both do the work
together? n+m
and in one hour n m th part of the cistern will be
Sol. As per the formula, required days
30 ´15 450 filled/ emptied. (same as time and work)
= = = 10 days
30 + 15 45 Example 5 : There are two pipes (inlet and outlet) attached with a
v If A , B , C can do a piece of work in X , Y , Z days respectively tank of 1000 litres. The inlet pipe can fill the tank in 2 hrs. The
outlet pipe can empty the tank in 4 hrs. What is the time required
xyz
while working alone , they will together take to fill the tank in case both are open? In one hour what part of the
[ xy + yz + zx ]
tank will be filled?
days to finish it Sol. For Inlet pipe, time to fill the tank = 2 hrs
Example 3 : Shyam will do a piece of work in 30 days; Ram can do For Outlet pipe, time to empty the tank = 4 hrs
same work in 15 days, Bhuvan can do the same work in 10 days; Time to fill the tank
in how many days can all three do the work together?
2´4 8
Sol. As per the formula, required days = = = 4 hrs
4-2 2
30 ´ 15 ´10
=
[30 ´15 + 15 ´10 + 30 ´10] 1 1 1
Net part filled/emptied in one hour = - = th
4500 2 4 4
= = 5 days
900 part of the tank, which is obvious from the earlier result.
TIME & WORK / PIPES & CISTERNS B-61

EXERCISE
1. 12 men take 18 days to complete a job whereas 12 women in (a) 4 (b) 5
(c) 6 (d) 12
3
18 days can complete of the same job. How many days (e) None of these
4 7. 10 men and 15 women finish a work in 6 days. One man
will 10 men and 8 women together take to complete the alone finishes that work in 100 days. In how many days will
same job? a woman finish the work?
1 (a) 125 days (b) 150 days
(a) 6 (b) 13
2 (c) 90 days (d) 225 days
(c) 12 (d) Data inadequate (e) None of these
(e) None of these 8. A tank is filled in 5 hours by three pipes A, B and C. The
2. Seven men and four boys can complete a work in 6 days. A pipe C is twice as fast as B and B is twice as fast as A. How
man completes double the work than a boy. In how many much time will pipe A alone take to fill the tank?
days will 5 men and 4 boys complete the work? (a) 35 hours (b) 25 hours
(a) 5 (b) 4 (c) 20 hours (d) Cannot be determined
(c) 6 (d) Cannot be determined (e) None of these
(e) None of these 9. 24 men working 8 hours a day can finish a work in 10 days.
3. The work done by a woman in 8 hours is equal to the work Working at the rate of 10 hours a day, the number of men
done by a man in 6 hours and by a boy in 12 hours. If required to finish the same work in 6 days is :
working 6 hours per day, 9 men can complete a work in 6 (a) 30 (b) 32
days then in how many days can 12 men, 12 women and 12 (c) 34 (d) 36
boys together finish the same work by working 8 hours per (e) None of these
day?
2
10. A water tank is th full. Pipe A can fill the tank in 10 minutes
(a) 1 1 days (b) 3 2 days 5
3 3 and the pipe B can empty it in 6 minutes. If both the pipes
1
(c) 3 days (d) 1 days are open, how long will it take to empty or fill the tank
2 completely?
(e) None of these (a) 6 minutes to empty (b) 6 minutes to fill
4. Tap ‘A’ can fill a water tank in 25 minutes, tap ‘B’ can fill the (c) 9 minutes to empty (d) 9 minutes to fill
same tank in 40 minutes and tap ‘C’ can empty that tank in (e) None of these
30 minutes. If all the three taps are opened together, in how 11. A water tank has three taps A, B and C. Tap A, when opened,
many minutes will the tank be completely filled up or can fill the water tank alone in 4 hours. Tap B, when opened,
emptied? can fill the water tank alone in 6 hours and tap C, when
opened, can empty the water tank alone in 3 hours. If taps
(a) 2 (b) 15 5
3 A, B and C are opened simultaneously, how long will it take
13 13 to fill the tank completely?
2 (a) 10 hours (b) 8 hours
(c) 8 (d) 31 11 (c) 18 hours (d) 12 hours
13 19
(e) None of these
(e) None of these 12. Twenty-four men can complete a work in sixteen days.
5. Machine A can print one lakh books in 8 hours. Machine B Thirty-two women can complete the same work in twenty-
can do the same job in 10 hours. Machine C can do the four days. Sixteen men and sixteen women started working
same job in 12 hours. All the three machines start job at 9.00 and worked for twelve days. How many more men are to be
am. A breaks down at 11.00 am and the other two machines added to complete the remaining work in 2 days?
finish the job. Approximately at what time will the job be (a) 48 (b) 24
finished? (c) 36 (d) 16
(a) 12.00 noon (b) 1.30 pm (e) None ofthe above
(c) 1.00 pm (d) 11.30 am 13. The total monthly income of four men and two women is
(e) None of these ` 46,000. If every woman earns ` 500 more than a man then
6. Suresh can complete a job in 15 hours. Ashutosh alone can what is the monthly income of a woman?
complete the same job in 10 hours. Suresh works for 9 hours (a) ` 7,500 (b) ` 8,000
and then the remaining job is completed by Ashutosh. How (c) ` 9,000 (d) ` 6,500
many hours will it take Ashutosh to complete the remaining (e) None of these
job alone?
B-62 TIME & WORK / PIPES & CISTERNS
14. 10 men can complete a piece of work in 15 days and 15 22. A and B can finish a work in 10 days while B and C can do
women can complete the same work in 12 days. If all the 10 it in 18 days. A started the work, worked for 5 days, then B
men and 15 women work together, in how many days will worked for 10 days and the remaining work was finished by
the work get completed? C in 15 days. In how many days could C alone have finished
the whole work ?
2
(a) 6 (b) 7 (a) 30 days (b) 15 days
3
(c) 45 days (d) 24 days
2 (e) None of these
(c) 6 (d) 6 1 23. 12 men complete a work in 18 days. Six days after they had
3 3
started working, 4 men joined them. How many days will all
(e) None of these
of them take to complete the remaining work ?
15. ‘A’ completes a work in 12 days. ‘B’ completes the same
(a) 10 days (b) 12 days
work in 15 days. ‘A’ started working alone and after 3 days
(c) 15 days (d) 9 days
B joined him. How many days will they now take together
to complete the remaining work? (e) None of these
(a) 5 (b) 8 24. A tyre has two punctures. The first puncture along would
have made the tyre flat in 9 minutes and the second alone
(c) 6 (d) 4
would have done it in 6 minutes. If air leaks out at a constant
(e) None of these
rate, how long does it take both the punctures together to
16. Rajani has to read a book of 445 pages. She has already
make it flat?
read the first 157 pages of the book and if she reads 24
pages of the book everyday then how long will she take 1 1
(a) 1 minutes (b) 3 minutes
now to complete the book? 2 2
(a) 25 days (b) 20 days 3 1
(c) 3 minutes (d) 4 minutes
(c) 46 days (d) 21 days 5 4
(e) None of these (e) None of these
17. 24 men working 8 hours a day can finish a work in 10 days. 25. A man, a woman or a boy can do a job in 20 days, 30 days or
Working at the rate of 10 hours a day, the number of men 60 days respectively. How many boys must assist 2 men
required to finish the same work in 6 days is and 8 women to do the work in 2 days?
(a) 30 (b) 36 (a) 15 boys (b) 8 boys
(c) 34 (d) 32 (c) 10 boys (d) 11 boys
(e) None of these (e) None of these
18. X and Y can do job in 25 days and 30 days respectively. 26. A can do 50% more work as B can do in the same time. B
They work together for 5 days and then X leaves. Y will alone can do a piece of work in 20 hours. A, with help of B,
finish the rest of the work in how many days? can finish the same work in how many hours ?
(a) 18 days (b) 19days (a) 12 (b) 8
(c) 20days (d) 21 days
1
(e) None of these (c) 13 1 (d) 5
19. A and B can do a job is 16 days and 12 days respectively. 4 3 2
(e) None of these
days before finishing the job, A joins B. B has started the 27. If 15 women or 10 men can complete a project in 55 days, in
work alone. Find how many days B has worked alone? how many days will 5 women and 4 men working together
(a) 6 days (b) 4 days complete the same project ?
(c) 5 days (d) 7 days (a) 75 (b) 8
(e) None of these (c) 9 (d) 85
20. A contractor undertakes to built a walls in 50 days. He (e) None of these
employs 50 peoples for the same. However after 25 days he 28. A machine P can print one lakh books in 8 hours, machine Q
finds that only 40% of the work is complete. How many can print the same number of books in 10 hours while
more man need to be employed to complete the work in machine R can print them in 12 hours. All the machines are
time? started at 9 a.m. while machine P is closed at 11 a.m. and the
(a) 25 (b) 30 remaining two machines complete the work. Approximately
(c) 35 (d) 20 at what time will the work be finished?
(e) None of these (a) 11 : 30 am (b) 12 noon
21. A is 30% more efficient than B. How much time will they, (c) 12 : 30 pm (d) 1 pm
working together, take to complete a job which A along (e) None of these
could have done in 23 days? 29. 10 men can complete a piece of work in 15 days and 15
(a) 11 days (b) 13 days women can complete the same work in 12 days. If all the 10
3 men and 15 women work together, in how many days will
(c) 20 days (d) 12 days
17 the work get completed?
(e) None of these
TIME & WORK / PIPES & CISTERNS B-63

1 36. A and B can do a job in 15 days and 10 days, respectively.


(a) 6 (b) 6 They began the work together but A leaves after some days
3
and B finished the remaining job in 5 days. After how many
2 2 days did A leave?
(c) 6 (d) 7
3 3 (a) 2 days (b) 3 days
(e) None of these (c) 1 day (d) 4 days
30. A can do a piece of work in 10 days, while B alone can do it (e) None of these
in 15 days. They work together for 5 days and the rest of 37. Mr. Suresh is on tour and he has ` 360 for his expenses. If
the work is done by C in 2 days. If they get ` 450 for the he exceeds his tour by 4 days he must cut down daily
whole work, how should they divide the money ? expenses by ` 3. The number of days of Mr. Suresh’s tour
programme is :
(a) ` 225, ` 150, ` 75 (b) ` 250, ` 100, ` 100
(a) 20 days (b) 24 days
(c) ` 200, ` 150, ` 100 (d) ` 175, ` 175, ` 100
(c) 40 days (d) 42 days
(e) None of these
(e) None of these
31. A alone would take 8 days more to complete the job than if
38. A can knit a pair of socks in 3 days. B can knit the same
both A and B would together. If B worked alone, he took
thing in 6 days. If they are knitting together, in how many
1 days will they knit two pairs of socks?
4 days more to complete the job than A and B worked
2 (a) 4 days (b) 2 days
together. What time would they take if both A and B worked 1
together? (c) 4 days (d) 3 days
2
(a) 7 days (b) 5 days
(e) None of these
(c) 4 days (d) 6 days 39. A can do a job in 3 days less time than B. A works at it alone
(e) None of these for 4 days and then B takes over and completes it. If
32. 10 men and 15 women together can complete a work in 6 altogether 14 days were required to finish the job, how many
days. It takes 100 days for one man alone to complete the days would each of them take alone to finish it?
same work. How many days will be required for one woman (a) 17 days, 20 days (b) 12 days, 15 days
alone to complete the same work? (c) 13 days, 16 days (d) 14 days, 11 days
(a) 90 (b) 125 (e) None of these
(c) 145 (d) 225 40. Two workers A and B working together completed a job in 5
(e) None of these days. If A worked twice as efficiently as he actually did and
33. A contractor undertook to do a piece of work in 9 days. He
1
employed certain number of laboures but 6 of them were B worked as efficiently as he actually did, the work would
absent from the very first day and the rest could finish the 3
work in only 15 days. Find the number of men originally have completed in 3 days. Find the time for A to complete
employed . the job alone.
(a) 15 (b) 6 1 3
(a) 6 days (b) 5 days
(c) 13 (d) 9 4 4
(e) None of these (c) 5 days (d) 3 days
1 (e) None of these
34. After working for 8 days, Anil finds that only of the work 41. X can do a piece of work in 15 days. If he is joined by Y who
3
is 50% more efficient, in what time will X and Y together
has been done. He employs Rakesh who is 60 % efficient as
finish the work?
Anil. How many more days will Anil take to complete the
(a) 10 days (b) 6 days
job?
(c) 18 days (d) Data insufficient
(a) 15 days (b) 12 days
(e) None of these
(c) 10 days (d) 8 days
42. A can build up a wall in 8 days while B can break it in 3 days.
(e) None of these A has worked for 4 days and then B joined to work with A
35. A sum of ` 25 was paid for a work which A can do in 32 for another 2 days only. In how many days will A alone
days, B in 20 days, B and C in 12 days and D in 24 days. build up the remaining part of wall?
How much did C receive if all the four work together ?
1 1
14 16 (a) 13 days (b) 7 days
(a) ` (b) ` 3 3
3 3
1
(c) 6 days (d) 7 days
15 17 3
(c) ` (d) `
3 3 (e) None of these
(e) None of these
B-64 TIME & WORK / PIPES & CISTERNS
43. Sakshi can do a piece of work in 20 days. Tanya is 25% more 47. Two pipes can fill a cistern in 14 and 16 hours respectively.
efficient than Sakshi. The number of days taken by Tanya The pipes are opened simultaneously and it is found that
to do the same piece of work is: due to leakage in the bottom, 32 minutes extra are taken for
(a) 15 (b) 16 the cistern to be filled up. If the cistern is full, in what time
(c) 18 (d) 25 would the leak empty it ?
(e) None of these (a) 110 hours (b) 112 hours
44. Two taps can fill a tank in 12 and 18 minutes respectively. (c) 115 hours (d) 100 hours
Both are kept open for 2 minutes and the first is turned off. (e) None of these
In how many minutes more will the tank be filled ? 48. Two pipes A and B can fill a cistern in 10 and 15 minutes
(a) 15 min. respectively. Both fill pipes are opened together, but at the
(b) 20 min. end of 3 minutes, ‘B’ is turned off. How much time will the
(c) 11 min. cistern take to fill ?
(d) 13 min. (a) 6 min (b) 8 min
(e) None of these (c) 10 min (d) 12 min
45. A cistern normally takes 6 hours to be filled by a tap but (e) None of these
because of a leak, 2 hours more. In how many hours will the 49. A cistern has two taps which fill it in 12 minutes and 15
leak empty a full cistern ? minutes respectively. There is also a waste pipe in the cistern.
(a) 20 hours When all the three are opened, the empty cistern is full in 20
(b) 24 hours minutes. How long will the waste pipe take to empty the full
(c) 26 hours cistern ?
(d) 18 hours (a) 10 min (b) 12 min
(e) None of these (c) 15 min (d) 9 min
46. One fill pipe A is 3 times faster than second fill pipe B and (e) None of these
takes 10 minutes less time to fill a cistern than B takes. Find 50. Two taps A and B can fill a cistern in 12 minutes and 18
when the cistern will be full if fill pipe B is only opened. minutes respectively. They are turned on at the same time.
(a) 20 min If the tap A is turned off after 4 minutes, how long will tap
(b) 18 min B take to fill the rest of the cistern ?
(c) 15 min (a) 8 min. (b) 9 min.
(d) 10 min (c) 10 min. (d) 7 min.
(e) None of these (e) None of these

ANSWER KEY
1 (b) 7 (d) 13 (b) 19 (c) 25 (b) 31 (d) 37 (a) 43 (b) 49 (a)
2 (e) 8 (a) 14 (c) 20 (a) 26 (b) 32 (d) 38 (a) 44 (d) 50 (a)
3 (d) 9 (b) 15 (a) 21 (b) 27 (a) 33 (a) 39 (b) 45 (b)
4 (d) 10 (a) 16 (e) 22 (c) 28 (d) 34 (c) 40 (a) 46 (c)
5 (c) 11 (d) 17 (d) 23 (d) 29 (c) 35 (b) 41 (b) 47 (b)
6 (a) 12 (b) 18 (b) 24 (c) 30 (a) 36 (b) 42 (b) 48 (b)
TIME & WORK / PIPES & CISTERNS B-65

Answers &
Explanations
4
Therefore, remaining part can be printed by
1. (b) 12 M × 18 = 12 W × 18 ×
3 10 ´ 12 23
B and C in ´ » 2 hrs
3 22 60
[ W= M
4 Hence, the job will be finished at
3 9 am + 2 + 2 = 1.00 p.m.
10M + 8W = 10M + 8 × M = 16 M
4 6. (a) The part of job that Suresh completes in 9 hours
[ 16 men can complete the same work 9 3
= =
12 ≥18 27 1 15 5
in < < 13 days
16 2 2 3 2
Remaining job = 1 - =
2. (e) M = 2B 5 5
\ 7M + 4B = 14B + 4B = 18B Remaining job can be done by Ashutosh in
5M + 4B = 10B + 4B = 14B 2
Q 18 boys complete the work in 6 days. × 10 = 4 hours
5
\ 14 boys complete the work in
6 ´ 18 5 7. (d) 15 women's work of a day = 1 – 1 Þ 1 part
= 7 days. 6 10 15
14 7
Note: 7 men and 4 boys complete the work in 6 days. We \ for 1 whole part a woman will take
have to find out the no. of days in which 5 men and 4 boys = 15 × 15 = 225 days.
complete the work. Here, we see that 4 boys are common in 8. (a) Here ratio of efficiencies of pipes A, B and C are as
both the cases, therefore, 5 men will take more time to follows:
complete the work, i.e., more than 6 days, which is not given C B A
in any options. Therefore, without calculating we can say 2 1
that our answer is (e). 2 1
3. (d) 8W = 6M = 12B 4 : 2 : 1
12M + 12W + 12B Þ 12M + 9M + 6M = 27M Suppose the efficiencies of pipes C, B and A are 4K,
[ 9 men can complete the work by working 1 hour per 2K and K.
day in 6 × 6 days Since, the tank is filled in 5 hours by the three pipes
6≥ 6≥ 9 1 having combined efficiency equal to 7K, the time
[ 27 men working 8 hours per day = < 1 days.
27 ≥8 2 7K ´ 5
required to fill the tank by A alone = = 35 hours
K
1 1 1
4. (d) Tank filled in 1 minute = ∗ , part
25 40 30 9. (b) m1 ´ d1 ´ t1 ´ w 2 = m 2 ´ d 2 ´ t 2 ´ w1
24 × 10 × 8 × 1 = m2 × 6 × 10 × 1
24 ∗ 15 , 20 19
< < part
600 600 24 ´ 10 ´ 8
Þ m2 = = 32 men
\ tank will be filled complete in minutes 6 ´ 10
10. (a)
600 11
< < 31 Q Pipe A in 1 minute fills 1/10 part and Pipe B in 1 min.
19 19
5. (c) Part of print done by A, B and C in 2 1
empties part
6
æ 1 1 1 ö 37
hours = 2 ç + + ÷ =
è 8 10 12 ø 60 1 1 ,1
[ Pipe A + B in 1 min = , <
10 6 15
37 23
Remaining = 1 - =
1
60 60
Q part gets emptied in 1 min
If B and C print together, then they can print 15

10 ´ 12 2 2
in hrs. [ part is emptied in 15 × min = 6 min
10 + 12 5 5
B-66 TIME & WORK / PIPES & CISTERNS
11. (d) Required time to fill the tank 17. (d) m1 ´ d1 ´ t1 ´ w 2 = m 2 ´ d 2 ´ t 2 ´ w1
1 1 1 24 × 10 × 8 × 1 = m2 × 6 × 10 × 1
= = = = 12 h
æ1 1ö 1 5 1 1
çè + ÷ø - - 24 ´ 10 ´ 8
4 6 3 12 3 12 Þ m2 = = 32 men
6 ´ 10
12. (b) 24 men complete the work in 16 days
1
\ 16 men complete æç ´ ö1
16 12 18. (b) X’s one day’s work = th part of whole work.
= ÷ part of work in 25
è 24 16 ø2
12 days Y’s one day’s work = 1 th part of whole work.
32 women complete the work in 24 days 30
16 14 7 1 1 1
\ 16 women complete ´ = part of work Their one day’s work = + = th
32 24 24 25 30 150
in (12 + 2 =) 14 days part of whole work.
So, the remaining part of the work which is done by
sixteen men + sixteen women and the reqd additional 11 11
Now, work is done in 5 days = ´5 = th
no. of men in 2 days 150 30
æ1 7 ö 1 7 5 of whole work
= 1- ç + ÷ = - = (part)
è 2 24 ø 2 24 24 11 19
\ Remaining work = 1 - = th of whole work
30 30
5
Now, in 2 days part of the work is done by
24 1
Now, th work is done by Y in one day..
30
16 5
24 ´ ´ = 40 men
2 24 19 1 19
\ th work is done by Y in ´ = 19 days
Hence, the reqd. additional no. of men 30 1/ 30 30
= 40 – 16 = 24 men.
13. (b) 4 M + 2 W = 46000; 1
19. (c) A’s one day’s work = th work
Again, W = M + 500 16
or, M = W – 500
1
\ 4 (W – 500) + 2W = 46000 B’s one day’s work = th work
or, 6W = 46000 + 2000 = 48000 12
\ W = ` 8000 Let B has worked alone = x days. Then,
A’s amount of work + B’s amount of work = 1
1 1 9
14. (c) 10 men + 15 women in 1 day do ∗ < work
15 12 60 æ1 ö æ 1ö
Þ 4 ç ÷ + (x + 4) ç ÷ = 1
è 16 ø è 12 ø
60 2
[ Time taken = days = 6 days 1 x+4 3
9 3 Þ + = 1 Þ x = ´ 12 - 4
15. (a) Work done by ‘A’ in 3 days 4 12 4

1 1 Þ x = 5 days
= ´3 =
12 4 20. (a) 50 men complete 0.4 work in 25 days.

1 3 Applying the work rule, m1 ´ d1 ´ w 2 = m 2 ´ d 2 ´ w1


\ Remaining work = 1 - = we have,
4 4
50 ´ 25 ´ 0.6 = m 2 ´ 25 ´ 0.4
12 ´ 15 20
Work done by A and B together = =
27 3 50 ´ 25 ´ 0.6
or m2 = = 75 men
\ Remaining work done by A and B together in 25 ´ 0.4
3 20 Number of additional men required = (75 – 50) = 25
= ´ = 5days
4 3 21. (b) Ratio of times taken by A and B = 100 : 130 = 10 : 13.
16. (e) Remaining pages to read = 445 – 157 = 288 Suppose B takes x days to do the work.

288 æ 23 ´ 13 ö 299
\ Reqd. number of days = = 12 Then, 10 : 13 : : 23 : x Þ x = ç ÷ Þx= .
24 è 10 ø 10
TIME & WORK / PIPES & CISTERNS B-67

1 10 26. (b) B alone can do a work in 20 hours.


A’s 1 day’s work = ; B’s 1 days work = .
23 299 3
\ A alone can do of the work in 20 hours.
æ 1 10 ö 23 1 2
(A + B)’s 1 day’s work = ç + ÷ = = .
è 23 299 ø 299 13 40
\ A and B together can complete the job in 13 days. i.e., A alone can do the same work in hours
3
22. (c) Let C completes the work in x days.
1 3 1 5 1
Work done by (A + B) in 1 day = \ (A + B)’s one hour’s work = + = =
10 40 20 40 8
Þ A and B together can finish the whole work in
1
Work done by (B +C) in 1 day = 8 hours.
18 27. (a) 15 W = 10 M
A’s 5 days’ work + B’s 10 days’ work + C’s 15 days’
work = 1 4 ´ 15
Now, 5W + 4M = 5W + W = 5W + 6W = 11 W
or (A + B)’s 5 days’ work + (B + C)’s 5 days’ work 10
+ C’s 10 days’ work = 1
If 15 women can complete the project in 55 days,
5 5 10 11 women can complete the same project in
or + + = 1 or x = 45 days
10 18 x
55 ´15
1 = 75 days
23. (d) In 1 day, work done by 12 men = 11
18
æ 1 1 1 ö 37
6 1 28. (d) (P + Q + R)’s 1 hour’s work = ç + + ÷ = .
In 6 days, work done by 12 men = = è 8 10 12 ø 120
18 3
2 æ 37 ö 37
Remaining work = Work done by P, Q and R in 2 hours = ç ´ 2÷ = .
3 è 120 ø 60
Now, m1 ´ d1 ´ w 2 = m 2 ´ d 2 ´ w1 æ 37 ö 23
Remaining work = ç1 - ÷ = .
2 è 60 ø 60
or 12 ´ 18 ´ = 16 ´ d 2 ´ 1
3
æ 1 1 ö 11
4 ´ 18 ´ 2 (Q + R)’s 1 hour’s work = ç + ÷ = .
or d2 = = 9 days è 10 12 ø 60
16
11
æ1 1ö 5 Now, work is done by Q and R in 1 hour..
24. (c) 1 minute’s work of both the punctures = ç + ÷ = . 60
è 9 6 ø 18
So, both the punctures will make the tyre flat in 23
So, work will be done by Q and R in
18 3 60
= 3 min.
5 5 æ 60 23 ö 23
ç 11 ´ 60 ÷ = 11 hours » 2 hours.
1 1 è ø
25. (b) Man’s two day’s work = 2 ´ th work = th work
20 10 So, the work will be finished approximately 2 hours
Woman’s two days’s work after 11 a.m., i.e., around 1 p.m.
1 1 1
= 2´ th work = th work 29. (c) 10 men’s 1 day’s work = ;
30 15 15
1 1 1
Boy’s two day’s work = 2 ´ th work = th work 15 women’s 1 day’s work = .
60 30 12
Now, let 2 men, 8 women and x boys can complete (10 men + 15 women)’s 1 day’s work
work in 2 days. Then ,
2 men’s work +8 women’s work + x boy’s work =1 æ 1 1ö 9 3
= ç + ÷= = .
æ 1ö æ1ö æ 1 ö è 15 12 ø 60 20
2ç ÷ + 8ç ÷ + x ç ÷ =1
è 10 ø è 15 ø è 30 ø \ 10 men and 15 women will complete the work in

æ 1 8ö 20 2
Þ x = ç1 - - ÷ ´ 30 Þ x = 8 boys = 6 days.
3 3
è 5 15 ø
B-68 TIME & WORK / PIPES & CISTERNS

æ 1 1ö 5 1
30. (a) Work done by A and B in 5 days = ç + ÷ ´ 5 = Now, th work is done by them in one day..
è 10 15 ø 6 15

5 1 2 2
Work remaining = 1 - = \ rd work is done by them in 15 ´ = 10 days
6 6 3 3
\ C alone can do the work in 6 × 2 = 12 days 1
35. (b) A's one day's work =
5 5 2 32
Ratio of their share work = : : = 3 : 2 :1
10 15 12 1
B's one day's work =
Share of wages = ` 225, ` 150, ` 75. 20
31. (d) Let if both A and B work together, they take x days.
1
1 (B + C)'s one day's work =
\ (A + B)’s 1 days’s work = th work . 12
x
1 1 1
\ C's one day's work = - =
1 12 20 30
A’s 1 day’s work = th work .
x +8
1
D's one day's work =
1 24
B’s 1 day’s work = th work .
x +9/2 \ (A + B + C + D)'s one day's work

1 2 1 1 1 1 1 75 + 120 + 80 + 100
Now, + = = + + + =
x + 8 2x + 9 x 32 20 30 24 2400

Þ x(2x + 9 + 2x + 16) = (x + 8)(2x + 9) 375 15 5


= = =
Þ 4x2 + 25x = 2x2 + 25x + 72 2400 96 32
Þ x2 = 36 Þ x = 6 days 5
1 \ Out of of work done,
32. (d) 1 man’s 1 day’s work = . 32
100
1
1 of the work is done by C.
(10 men + 15 women)’s 1 day’s work = . 30
6
Þ Out of ` 25 paid for the work, C will receive
15 women’s 1 day’s work
1/ 30 1 32 16
æ 1 10 ö æ 1 1 ö 1 ` ´ 25, i.e. ´ ´ 25, i.e. `
=ç - ÷=ç - ÷= . 5 / 32 30 5 3
è 6 100 ø è 6 10 ø 15
1
1 36. (b) A’s one day’s work = th work .
\ 1 woman’s 1 day’s work = . 15
225
\ 1 woman alone can complete the work in 225 days. 1
B’s one day’s work = th work .
33. (a) Let the number of men originally employed be x. 10
9x = 15(x – 6) 1 1 1
or x = 15 (A + B)’s one day’s work = + = th work.
15 10 6
1 Let A left after x days.
34. (c) In 8 days, Anil does = rd work .
3
x
\ (A +B)’s x days’ work = th work .
1 6
\ in 1 day, he does = th work.
24
x 6-x
\ Rakesh’s one day’s work Remaining work = 1 - = th work.
6 6
1 1
= 60% of = th work . 6-x
24 40 Now, in 5 days, work done by B = th work .
6
1 2
Remaining work = 1 - =
\ in 1 day work done by B =
6-x
th work
3 3
30
(Anil and Rakesh)’s one day’s work
6-x 1
1 1 1 and =
= + = th work 30 10
24 40 15
\ x = 3 days
TIME & WORK / PIPES & CISTERNS B-69
37. (a) Let Suresh undertakes a tour of x days.
æ1ö 1
360 Now , (2A) 's work + ç ÷ B's work = rd work
Then, expenses for each day = è 3ø 3
x
2 1æ x -5ö 1 25 1
360 360 Þ + ç ÷= Þx= = 6 days
Now, = -3 x 3 è 5x ø 3 4 4
x+4 x
1
æ1
360ç -
1 ö 41. (b) X’s one day’s work = th work .
or ÷ =3 15
è x x+4ø
1 1 1
or x 2 + 4 x - 480 = 0 or x = – 24 or x = 20 Y’s one day’s work = + 50% of = th work
15 15 10
Since, x ¹ -24 we have x = 20
1 1 1
38. (a) A’s one day’s work =
1
rd work . \ (X + Y)’s one day’s work = + = th work
15 10 6
3
Hence, they together finish the work in 6 days.
1
B’s one day’s work = th work . 1
6 42. (b) A’s one day’s work = th work
8
1 1 1
(A + B)’s one day’s work = + = nd work
3 6 2 B’s one day’s work = 1 rd work
\ A and B together can complete the work (knit a pair 3
of socks) in 2 days. 1 1
\ They together knit two pair of socks in 4 days. \ A’s 4 day’s work = 4 ´ = nd work
8 2
39. (b) Let B can finish the work in x days.
Then A can finish the work in (x – 3) days. 1 æ1ö æ1ö
\ In next two days, total wall = + 2 ç ÷ - 2 ç ÷
1 2 è8ø è 3ø
B’s one day’s work = th work
x
1
= th wall
1 12
A’s one day’s work = th work
x -3
1 11
4 Remaining wall = 1 - = th
th work 12 12
A’s 4 days’ work =
x -3
1
Now, th wall is built up by A in one day..
4 x-7 8
Remaining work = 1 - = th work
x -3 x -3
11 11 1
The remaining work done by B in 14 – 4 = 10 days. \ th wall is built up by A in 8 ´ = 7 days .
12 12 3
x -7
Now, in 10 days, work done by B = th work 1
x -3
43. (b) Sakshi’s one day’s work = th work
20
1 æ x -7 ö
\ in 1 day, work done by B = ç ÷ th work Tanya’s one day’s work
10 è x - 3 ø
1 1 1
1 æ x-7ö 1 = + 25% of = th work
and ç ÷= 20 20 16
10 è x - 3 ø x
Hence, Tanya takes 16 days to complete the work.
Þ x = 15days
\ B ® 15 days and A® 12 days 44. (d) Part filled by first tap in one min = 1 th
12
1
40. (a) (A + B)’s one day’s work = th work Part filled by second tap in one min = 1 th
5
18
Let A can do job in x days. Then,
Now, 2 éê + ùú + unfilled part = 1
1 1 1
A’s one day’s work = th work
ë 12 18 û
x
1 1 x -5 13
and B’s one day’s work = – = th work Þ unfilled part = th
5 x 5x 18
B-70 TIME & WORK / PIPES & CISTERNS
\ part of cistern emptied, due to leakage in one hour
1
Q th part of tank is filled by second tap in 1min.
18 15 1 1
= - = th
112 8 112
13
\ th part of tank is filled by second tap in 1 min. \ in 112hr, the leakage would empty the cistern.
18
1 1 1
13 48. (b) In one min, (A + B) fill the cistern = + = th
= 18 ´ min = 13 min. 10 15 6
18
45. (b) Q cistern fill in 6 hours. In 3 min, (A + B) fill the cistern = 3 = 1 th
6 2
\ in 1 hour, filled part = 1 th
6 1 1
Remaining part = 1 - =
2 2
1
Now, due to leakage, filled part in 1 hour = th
8 1
Q th part filled by A in one min.
Part of the cistern emptied, due to leakage in 1 hour 10

1 1 1 1 1
= – = th \ nd part filled by A in 10 ´ = 5 min .
6 8 24 2 2
\ The leakage will empty the full cistern in 24 hrs. \ Total time = 3 + 5 = 8 min.
49. (a) Work done by the waste pipe in 1 minutes
46. (c) Let B can fill the cistern in x min. Then, 1 æ1 1ö 1
= -ç + ÷ = - [–ve sign means emptying]
x 20 è 12 15 ø 10
then A can fill the cistern in min
3 \ Waste pipe will empty the full cistern in 10 minutes.
1 1 5
Given x - x = 10 Þ x = 15 min 50. (a) In one min, (A + B) fill the cistern = + = th
3 12 18 36
47. (b) Cistern filled by both pipes in one hour
5 5
In 4 min, (A + B) fill the cistern = ´ 4 = th
1 1 15 36 9
= + = th
14 16 112
5 4
112 Rest part = 1 - = th
\ Both pipes filled the cistern in hrs . 9 9
15
1
Now, due to leakage both pipes filled the cistern in Q th part is filled by B in one min.
18
112 32
+ = 8 hrs. 4
15 60 \ 4 th part is filled by B in 18 ´ = 8 min .
9 9
1
\ due to leakage, filled part in one hour =
8
Distance, Speed & Time
(Boats and Streams)

8 Chapter

The rate at which any moving body covers a particular distance Example 1 : A boy walking at a speed of 10 km/h reaches his
is called its speed. school 12 min late. Next time at a speed of 15 km/h reaches his
school 7 min late. Find the distance of his school from his house?
Distance Distance
Speed = ; Time = ; 5 1
Time Speed Sol. Difference between the time = 12 – 7 = 5 min = = hr
60 12
Distance = Speed × time
15 ´ 10 1 150 1
Unit : SI unit of speed is metre per second (mps). It is also Required distance = ´ = ´ = 2.5 km
15 - 10 12 5 12
measured in kilometers per hour (kmp) or miles per hour (mph).
Conversion of units :
A man leaves a point A at t1 and reaches the point B at t2.
(i) 1 hour = 60 minutes = 60 × 60 seconds.
Another man leaves the point B at t3 and reaches the point
1 km = 1000 m A at t4, then they will meet at
1 km = 0.6214 mile
Þ 1 mile = 1.609 km, i.e. 8 km = 5 miles t1 +
( t 2 – t1 )( t 4 – t1 )
1 yard = 3 feet ( t 2 – t1 ) + ( t 4 – t 3 )
1 foot = 12 inches Time taken by a train to cross a pole/a standing man
5 Length of train
1 km/h = m/sec, = .
18 Speed of train
18 Time taken by a train to cross platform/bridge etc. (i.e. a
1 m/sec = km/h
5 stationary object with some length)

22 length of train + length of platform/bridge etc.


1 miles/hr = ft/sec. = .
15 speed of train
When two trains with lengths L1 and L2 and with speeds S1
Total Distance
Average speed = and S2 respectively, then
Total time (a) When they are moving in the same direction, time taken
If a certain distance (d), say from A to B, is covered at ‘a’ by the faster train to cross the slower train
km/hr and the same distance is covered again say from B to L1 + L2
= .
A in ‘b’ km/hr, then the average speed during the whole difference of their speeds
journey is given by :
(b) When they are moving in the opposite direction, time
æ 2ab ö km taken by the trains to cross each other
Average speed = ç ÷
è a + b ø hr L1 + L 2
= .
... (which is the harmonic means of a and b sum of their speeds
Suppose two trains or two bodies are moving in the same
If a person with two different speeds U & V cover the same direction at u km/hr and v km/hr respectively such that u > v,
distance, then required distance then
their relative speed = (u – v) km/hr.
U´V If their lengths be x km and y km respectively, then time
= ´ Difference between arrival time
U-V taken by the faster train to cross the slower train (moving in

U´V æx+yö
Also, required distance = Total time taken ´ the same direction) = ç ÷ hrs.
U+V è u-vø
B-72 DISTANCE, SPEED & TIME (BOATS AND STREAMS)
Suppose two trains or two bodies are moving in opposite 12
directions at u km/hr and v km/hr, then their relative speed Upstream speed is = 2 kmph
6
= (u + v) km/hr.
If their lengths be x km & y km, then : Speed of Boat = 1 (Downstream + Upstream Speed)
2
æx+yö 1
time taken to cross each other = ç ÷ hrs. = [7 + 2] = 4.5 kmph
èu+vø 2
1
Example 2 : How long does a train 90 m long running at the rate Speed of current = (Downstream–Upstream speed)
2
of 54 km/h take to cross –
(b) a platform 120 m long? 1
= (7 – 2) = 2.5 kmph
(d) another train 160 m long running at 36 km/h in same 2
direction?
(f) a man running at 6 km/h in same direction? Example 4 : A man can row 6 km/h in still water. When the river is
Sol. (b) The platform is stationary of length = 120 m. running at 1.2 km/h, it takes him 1 hour to row to a place and back.
Length to be covered How far is the place?
= Length of the train + Length of the platform Sol. Man’s rate downstream = (6 + 1.2) = 7.2 km/h.
= 90 + 120 = 210 m Man’s rate upstream = (6 – 1.2) km/h = 4.8 km/h.
Let the required distance be x km.
\ Required time = 210 = 14 sec.
15 x x
Then + = 1 or 4.8x + 7.2x = 7.2 × 4.8
(d) Another train is moving in same direction. 7.2 4.8
Length to be covered
7.2 ´ 4.8
= Length of the train + length of the other train Þ x= = 2.88km
12
= 90 + 160 = 250 m
Relative speed = 54 – 36 = 18 kmph. By direct formula :

\ Required time =
250
5
= 50 sec.
Required distance =
(
1 ´ 6 2 – (1.2) 2 )
18 ´ 2´ 6
18
(f) The man is moving in same direction, 36 - 1.44 34.56
= = = 2.88 km
so Length to be covered = Length of the train, 12 12
and relative speed = speed of train – speed of man Example 5 : A man rows 27km with the stream and 15km against
90 the stream taking 4 hours each time. Find this rate per hour in still
\ Required time =
5 water and the rate at which the stream flows.
(54 – 6) ´
18
27 3
Sol. Speed with the stream = = 6 kmph
90 27 3 4 4
= ´3 = = 6 sec.
40 4 4
15 3
Speed of boat with the stream (or downstream or D/S) \ Speed against the stream = = 3 kmph.
= (X + Y) m/sec. 4 4
Speed of boat against the stream (or upstream or U/S) \ Speed of the man in still water
= (X – Y) m/sec. 1æ 3 3ö 1
= çè 6 + 3 ÷ø = 5 kmph
2 4 4 4
Example 3 : A boat is rowed down a river 28 km in 4 hours and up
a river 12 km in 6 hours. Find the speed of the boat and the river. 1æ 3 3ö
\ Speed of the stream = ç 6 - 3 ÷ø = 1.5 kmph
Sol. Downstream speed is 28 = 7 kmph 2è 4 4
4
DISTANCE, SPEED & TIME (BOATS ANDSTREAMS) B-73

EXERCISE
1. A car finishes a journey in ten hours at the speed of 80 8. A boat covers a distance of 30 km downstream in 2 hours
km/hr. If the same distance is to be covered in eight hours while it take 6 hours to cover the same distance upstream. If
how much more speed does the car have to gain? the speed of the current is half of the speed of the boat then
(a) 8 krn/hr (b) 10 km/hr what is the speed of the boat in km per hour?
(c) 12 km/hr (d) 16 krn/hr (a) 15 kmph (b) 5 kmph
(e) None of these (c) 10 kmph (d) Data inadequate
2. Two cars A and B are running towards each other from (e) None of these
different places 88 km apart. If the ratio of the speeds of the 9. A man starts walking. He walked 2 km in the first hour. Then
cars A and B is 5 : 6 and the speed of the car B is 90 km per he walked two-thirds of the distance of the previous hour
hour then after how long will the two meet each other? in each next hour. If he walked continuously then how long
could he walk maximum?
(a) 2 minutes (b) 24 minutes (a) 60 km (b) 6 km
26
3 (c) 12 km (d) 8 km
(c) 32 minutes (d) 36 minutes (e) None of these
(e) None of these 10. Starting with the initial speed of 30 km/hr, the speed is
3. Train ‘A’ leaves Mumbai Central for Lucknow at 11 am, increased by 4 km/hour every two hours. How many hours
running at the speed of 60 kmph. Train ‘B’ leaves Mumbai will it take to cover a distance of 288 km?
Central for Lucknow by the same route at 2 pm on the same (a) 4 (b) 6
day, running at the speed of 72 kmph. At what time will the (c) 12 (d) 8
two trains meet each other? (e) None of these
(a) 2 am on the next day 11. With a uniform speed a car covers a distance in 8 hours.
(b) 5 am on the next day Were the speed increased by 4 km/hr the same distance
(c) 5 pm on the next day
1
(d) 2 pm on the next day could be covered in 7 hours. What is the distance
(e) None of these 2
4. A motor starts with the speed of 70 kmph with its speed covered?
increasing every two hours by 10 kmph. In how many hours (a) 640 km (b) 480 km
will it cover 345 kms? (c) 420 km (d) Cannot be determined
(e) None of these
1 1 12. A 300-metre-long train crosses a platform in 39 seconds
(a) 2 hours (b) 4 hours
4 2 while it crosses a signal pole in 18 seconds. What is the
(c) 4 hours 5 minutes (d) Cannot be determined length of the platform?
(e) None of these (a) 320 metres (b) 650 metres
5. A boat takes 3 hours to travel from place M to N downstream (c) 350 metres (d) Data inadequate
and back from N to M upstream. If the speed of the boat in (e) None of these
still water is 4 km/hr, what is the distance between the two 13. A 260-metre-long train crosses a 120-metre-long wall in 19
places? seconds. What is the speed of the train?
(a) 8 km (b) 12 km (a) 27 km/hr (b) 49 km/hr
(c) 6 km (d) Data inadequate (c) 72 km/hr (d) 70 km/hr
(e) None of these (e) None of these
6. A boat has to travel upstream 20 km distance from point X 14. A 270-metre-long train running at the speed of 120 kmph
of a river to point Y. The total time taken by boat in travelling crosses another train running in opposite direction at the
from point X to Y and Y to X is 41 minutes 40 seconds. What speed of 80 kmph in 9 secs. What is the length of the other
is the speed of the boat? train?
(a) 66 km/hr (b) 72 km/hr (a) 240 metres (b) 320 metres
(c) 48 km/hr (d) Data inadequate (c) 260 metres (d) 230 metres
(e) None of these (e) None of these
7. The speed of a car increases by 2 km after every hour. If the 15. A monkey ascends a greased pole 12 metres high. He
distance travelled in the first hour was 35 km, what was the ascends 2 metres in first minute and slips down 1 metre in
total distance travelled in 12 hours? the alternate minute. In which minute, he reaches the top ?
(a) 522 km (b) 456 km (a) 21st (b) 22nd
(c) 556 km (d) 482 km (c) 23rd (d) 24th
(e) None of these (e) None of these
B-74 DISTANCE, SPEED & TIME (BOATS AND STREAMS)
1 24. A train leaves station X at 5 a.m. and reaches station Y at 9
16. A man walks a certain distance and rides back in 6 h. He
4 a.m. Another train leaves station Y at 7 a.m. and reaches
3 station X at 10: 30 a.m. At what time do the two trains cross
can walk both ways in 7 h. How long it would take to ride
4 each other ?
both ways ?
(a) 7 : 36 am (b) 7 : 56 am
1
(a) 5 hours (b) 4 hours (c) 8 : 36 am (d) 8 : 56 am
2
(e) None of these
3
(c) 4 hours (d) 6 hours 25. Cars C1 and C2 travel to a place at a speed of 30 and 45 km/
4
(e) None of these 1
h respectively. If car C2 takes 2 hours less time than C1
17. There are 20 poles with a constant distance between each 2
pole. A car takes 24 seconds to reach the 12th pole . How for the journey, the distance of the place is
much time will it take to reach the last pole? (a) 300 km (b) 400 km
(a) 25.25 s (b) 17.45 s (c) 350 km (d) 225 km
(c) 35.75 s (d) 41.45 s (e) None of these
(e) None of these 26. If I walk at 4 km/h, I miss the bus by 10 minutes. If I walk at
18. A man is walking at a speed of 10 km per hour. After every 5 km/h, I reach 5 minutes before the arrival of the bus. How
kilometre, he takes rest for 5 minutes. How much time will be far I walk to reach the bus stand ?
take to cover a distance of 5 kilometres?
(a) 5 km (b) 4.5 km
(a) 48 min. (b) 50 min.
1
(c) 45 min. (d) 55 min. (c) 5 km / h (d) Cannot be determined
(e) None of these 4
19. On a journey across Bombay, a tourist bus averages 10 km/h (e) None of these
for 20% of the distance, 30 km/h for 60% of it and 20 km/h for 27. A goods train leaves a station at a certain time and at a fixed
the remainder. The average speed for the whole journey was speed. After 6 hours, an express train leaves the same station
(a) 10 km/h (b) 30 km/h and moves in the same direction at a uniform speed of 90
(c) 5 km/h (d) 20 km/h kmph. This train catches up the goods train in 4 hours. Find
the speed of the goods train.
(e) None of these
20. In a 800 m race around a stadium having the circumference (a) 36 kmph (b) 40 kmph
of 200 m, the top runner meets the last runner on the 5th (c) 30 kmph (d) 42 kmph
minute of the race. If the top runner runs at twice the speed (e) None of these
of the last runner, what is the time taken by the top runner 28. Without stoppages, a train travels certain distance with an
to finish the race ? average speed of 80 km/h, and with stoppages, it covers
(a) 20 min (b) 15 min the same distance with an average speed of 60 km/h. How
(c) 10 min (d) 5 min many minutes per hour the train stops ?
(e) None of these (a) 15 (b) 18
21. A man walks half of the journey at 4 km/h by cycle does one (c) 10 (d) 16
third of journey at 12 km/h and rides the remainder journey (e) None of these
in a horse cart at 9 km/h, thus completing the whole journey 29. If a man walks to his office at 3/4 of his usual rate, he reaches
in 6 hours and 12 minutes. The length of the journey is
office 1/3 of an hour later than usual. What is his usual time
1332 to reach office.
(a) 36 km (b) km
67
1
(c) 40 km (d) 28 km (a) hr (b) 1 hr
(e) None of these 2
22. R and S start walking each other at 10 AM at the speeds of 3
3 km/h and 4 km/h respectively. They were initially 17.5 km (c) hr (d) 2 hrs
4
apart. At what time do they meet?
(a) 2 : 30 PM (b) 11 : 30 AM (e) None of these
(c) 1 : 30 PM (d) 12 : 30 PM 30. If a man walks to his office at 5/4 of his usual rate, he reaches
(e) None of these office 30 minutes early than usual. What is his usual time to
23. A train does a journey without stoppage in 8 hours, if it had reach office.
travelled 5 km/h faster, it would have done the journey in 6 1
hours 40 minutes. Find its original speed. (a) 2 hrs (b) 2 hr s
2
(a) 25 km/h (b) 40 km/h
(c) 1 hr 50 min (d) 2 hrs 15 min
(c) 45 km/h (d) 36.5 km/h
(e) None of these
(e) None of these
DISTANCE, SPEED & TIME (BOATS ANDSTREAMS) B-75
31. A train running between two stations A and B arrives at 38. A jogger running at 9 kmph alongside a railway track is 240
its destination 10 minutes late when its speed is 50 km/h metres ahead of the engine of a 120 metre long train running
and 50 minutes late when its speed is 30km/h. What is the at 45 kmph in the same direction. In how much time will the
distance between the stations A and B ? train pass the jogger?
(a) 40 km (b) 50 km (a) 3.6 sec (b) 18 sec
(c) 60 km (d) 70 km (c) 36 sec (d) 72 sec
(e) None of these (e) None of these
32. A car travels 25 km an hour faster than a bus for a journey of 39. Two trains are running at 40 km/h and 20 km/h respectively
500 km. If the bus takes 10 hours more than the car, then the in the same direction. Fast train completely passes a man
speeds of the bus and the car are sitting in the slower train in 5 seconds. What is the length
(a) 25 km/h and 40 km/h respectively of the fast train?
(b) 25 km/h and 60 km/h respectively 2
(a) 23 m (b) 23 m
(c) 25 km/h and 50 km/h respectively 9
(d) 25 km/h and 70 km/h respectively 7
(c) 27 m (d) 27 m
(e) None of these 9
33. A train consists of 12 boggies, each boggy 15 metres long. (e) None of these
The train crosses a telegraph post in 18 seconds. Due to 40. Two trains, 130 and 110 metres long, while going in the
some problem, two boggies were detached. The train now same direction, the faster train takes one minute to pass the
crosses a telegraph post in other completely. If they are moving in opposite direction,
(a) 18 sec (b) 12 sec they pass each other completely in 3 seconds. Find the
(c) 15 sec (d) 20 sec speed of trains.
(e) None of these (a) 30 m/s, 40 m/s (b) 32 m/s, 48 m/s
34. A man started running at a distance of 225 metre from the (c) 40 m/s, 44 m/s (d) 38 m/s, 42 m/s
train. If the speed of the man is 6 km/h, then how much time (e) None of these
should the train wait so that the man will be just able to 41. A train overtakes two person who are walking in the same
catch it ? direction in which the train is going , at the rate of 2 kmph
and 4 kmph and passes them completely in 9 and 10 seconds
1
(a) 2 min (b) 3 min respectively. The length of the train is:
4
(a) 45 m (b) 50 m
1 1
(c) 4 min (d) 4 min (c) 54 m (d) 72 m
4 2 (e) None of these
(e) None of these 42. Local trains leave from a station at an interval of 15 minutes
35. A man sitting in a train which is travelling at 50 kmph observes at a speed of 16 km/h. A man moving from opposite side
that a goods train, travelling in opposite direction, takes 9 meets the trains at an interval of 12 minutes. Find the speed
seconds to pass him. If the goods train is 280 m long, find of the man.
its speed. (a) 4 km/h (b) 3.5 km/h
(a) 62 kmph (b) 58 kmph (c) 4.5 km/h (d) 3 km/h
(c) 52 kmph (d) 50 kmph (e) None of these
(e) None of these 43. Local trains leave from a station at an interval of 14 minutes
36. Two trains, one from Howrah to Patna and the other from at a speed of 36 km/h. A man moving in the same direction
Patna to Howrah, start simultaneously. After they meet, the along the road meets the trains at an interval of 18 minutes.
trains reach their destinations after 9 hours and 16 hours Find the speed of the man.
respectively. The ratio of their speeds is: (a) 8 km/h (b) 7 km/h
(a) 2 : 3 (b) 4 : 3 (c) 6 km/h (d) 5.8 km/h
(c) 6 : 7 (d) 9 : 16 (e) None of these
(e) None of these 44. A train overtakes two persons walking along a railway track.
37. A train 75 metres long overtook a man who was walking at The first one walks at 4.5 km/h. The other one walks at 5.4
the rate of 6 km/h and passed him in 18 seconds. Again, the km/h. The train needs 8.4 and 8.5 seconds respectively to
train overtook a second person in 15 seconds. At what rate overtake them. What is the speed of the train if both the
was the second person travelling ? persons are walking in the same direction as the train?
(a) 3 km/h (b) 2.5 km/h (a) 66 km/h (b) 72 km/h
(c) 4 km/h (d) 1.5 km/h (c) 78 km/h (d) 81 km/h
(e) None of these (e) None of these
B-76 DISTANCE, SPEED & TIME (BOATS AND STREAMS)
45. Two trains running in opposite directions cross a man Directions (Qs. 47-48): Answer the following questions on the
standing on the platform in 27 seconds and 17 seconds basis of the information given below:
respectively and they cross each other in 23 seconds. The (i) Trains A and B are travelling on the same route heading
ratio of their speeds is: towards the same destination. Train B has already
(a) 1 : 3 covered a distance of 220 km before train A started.
(b) 3 : 2 (ii) The two trains meet each other 11 hours after the start
(c) 3 : 4 of train A.
(d) 2 : 1 (iii) Had the trains been travelling towards each other (from
(e) None of these a distance of 220 km), they would have met after one
46. Two trains each of 120 m in length, run in opposite directions hour.
with a velocity of 40 m/s and 20 m/s respectively. How long 47. What is the speed of train B in kmph?
will it take for the tail ends of the two trains to meet each (a) 100 (b) 180
other during the course of their journey : (c) 116 (d) Data inadequate
(a) 20 s (e) None of these
(b) 3 s 48. What is the speed of train A in kmph?
(c) 4 s (a) 102 (b) 80.5
(d) 5 s (c) 118 (d) Data inadequate
(e) None of these (e) None of these

ANSWER KEY
1 (e) 6 (d) 11 (b) 16 (c) 21 (a) 26 (a) 31 (b) 36 (b) 41 (b) 46 (c)
2 (c) 7 (e) 12 (c) 17 (d) 22 (d) 27 (a) 32 (c) 37 (a) 42 (a) 47 (a)
3 (b) 8 (c) 13 (c) 18 (b) 23 (a) 28 (a) 33 (c) 38 (c) 43 (a) 48 (e)
4 (b) 9 (b) 14 (d) 19 (d) 24 (b) 29 (b) 34 (a) 39 (d) 44 (d)
5 (d) 10 (d) 15 (b) 20 (c) 25 (d) 30 (b) 35 (a) 40 (d) 45 (b)

Answers &
Explanations
1. (e) Distance covered by the car = 80 × 10 = 800 km 45 1
\ Time taken = = hour
800 90 2
[ Speed = < 100 km / hr
8 1
[ Speed gain = 100 – 80 = 20 km/hr Total time = 4 + 1/2 = 4 hours.
2
5 5. (d) Let the distance between M and N and the speed of
2. (c) Speed of the car A < ´90 < 75 km/hr current in still water be d km and x km/hr respectively.
6
d d
88 According to the question, ∗ <3
[ Reqd. time = ´ 60 = 32 minutes 4∗ x 4, x
90 + 75
In the above equation we have only one equation but
3. (b) Distance covered by train A before the train B leaves
two variables, hence, can’t be determined.
Mumbai Central = 60 × 3 = 180 km
6. (d) Let x be the speed of the boat.
180 and y the speed of the current.
[ Time taken to cross each other = = 15 hours
12
20 20 25
[ Reqd time = 2 pm + 15 = 5 am on the next day [ ∗ <
x , y x ∗ y 36
4. (b) Distance covered in first two hours = 70 × 2 = 140 km
Distance covered in next two hours = 80 × 2 = 160 km In this equation there are two variables, but only one
Distance covered in first four hours equation, so, the value of ‘x’ cannot be determined.
140 + 160 = 300 km 7. (e) This is the problem of arithmetic progression (AP) with
Remaining distance = 345 – 300 = 45 km. Now, this the first term (a) = 35, common difference (d) = 2 and
distance will be covered at the speed of 90 km/hr. total no. of terms (n) = 12. The sum of this series will be
total distance travelled.
DISTANCE, SPEED & TIME (BOATS ANDSTREAMS) B-77

n
15. (b) In 2 minutes, he ascends = 1 metre
12
Sum (Sn) = {2a + (n – l)d} = {70 + 11× 2} \ 10 metres, he ascends in 20 minutes.
2 2 \ He reaches the top in 21st minute.
12 ´ 92 16. (c) We know that, the relation in time taken with two
= = 552 km different modes of transport is
2
8. (c) Here downstream speed = 15 km/hr and upstream speed twalk both + tride both = 2 (twalk + t ride)
= 5 km/hr 31 25
+ t ride both = 2 ´
15 + 5 4 4
\ Speed of the boat = = 10 km/h
2 25 31 19 3
9. (b) Required distance Þ t ride both = – = = 4 hrs
2 4 4 4
é 2 æ 2 ö 2 æ 2 ö3 ù 17. (d) Let the distance between each pole be x m.
= ê1 + + ç ÷ + ç ÷ + ...ú
2 Then, the distance up to 12th pole = 11 x m
êë 3 è 3 ø è 3 ø úû
11x
Speed = m/s
1 24
= 2´ = 2 ´ 3 = 6 km. Time taken to covers the total distance of 19x
2
1-
3 19 x ´ 24
= = 41.45 s
10. (d) Hour Speed (km/h) Distance travelled (in km) 11x
18. (b) Rest time = Number of rest × Time for each rest
2 30 60 = 4 × 5 = 20 minutes
2 34 68 Total time to cover 5 km
2 38 76 æ5 ö
2 42 84 = ç ´ 60 ÷ minutes + 20 minutes = 50 minutes.
è 10 ø
8 288 19. (d) Let the average speed be x km/h.
Hence, the required time = 8 hours and Total distance = y km. Then,

D D 0.2 0.6 0.2 y


11. (b) Here - =4 y+ y+ y=
7.5 8 10 30 20 x
(where D is the distance in km) 1
Þx= = 20km / h
Þ 0.5 D = 4 × 8 × 7.5 0.05
Þ D = 2 × 4 × 8 × 7.5 = 480 km 20. (c) After 5 minutes (before meeting), the top runner covers
12. (c) When a train crosses a platform, it crosses a distance 2 rounds i.e., 400 m and the last runner covers 1 round
equal to the sum of the length of the platform and that i.e., 200 m.
of the train. But when a train crosses a signal pole, it \ Top runner covers 800 m race in 10 minutes.
crosses the distance equal to its length only. 21. (a) Let the length of the journey = x km.
Here, time taken by the train to cross a signal
\ Journey rides by horse cart = x æç1 - - ö÷
pole = 18 seconds 1 1
è 2 3ø
300
Hence, speed of the train = m/sec 1
18
= x km.
The train takes 21(= 39 – 18) seconds extra in order to 6
cross the platform.
31
21´ 300 Then, total time taken to complete journey = hr
Hence, length of platform = = 350m 5
18
31
260 + 120 18 Þ t1 + t 2 + t 3 =
13. (c) Speed of train = ´ = 72 km/hr. 5
19 5
x 1 x 1 x 31
5 Þ ´ + ´ + =
14. (d) Relative speed = 120 + 80 kmph = 200 × m/sec 2 4 3 12 6 ´ 9 5
18
31 216
Þx= ´ = 36.2km » 36km
Distance (270 ∗ x ) ´ 9 5 37
t= =
Speed 500 22. (d) Let after t hours they meet then,
3t + 4t = 17.5 Þ t = 25
9×500
or 270 + x = x = 500 – 270 = 230 m \ Time = 10 am + 2.5 h = 12 : 30am
9
B-78 DISTANCE, SPEED & TIME (BOATS AND STREAMS)
23. (a) Let original speed = S km/h
a
Here, distance to be covered is constant 29. (b) If new speed is of original speed, then
b
æ 20 ö
\ S ´ 8 = (S + 5) ç ÷ æb ö
è 3 ø usual time × ç – 1 ÷ = change in time
èa ø
20 100 100
Þ 8S - S= ÞS= = 25 km / h æ4 ö 1
3 3 4 \ usual time × ç - 1÷ =
è3 ø 3
24. (b) Let the distance between X and Y be x km. Then, the
1
x 2x Þ usual time = ´ 3 = 1 hr
speed of A is km/h and that of B is km/h. 3
4 7
æ 4 ö -30
2x
km / h
x km x
km / h 30. (b) usual time × ç –1 ÷ =
7 X Y 4 è 5 ø 60
Relative speeds of the trains
1 1
Þ usual time = ´ 5 = 2 hrs
æ x 2x ö 15x 2 2
= ç + ÷= km / h
è 4 7 ø 28 31. (b) Let the distance between the two stations be x km.
Therefore the distance between the trains at 7 a.m. x 10 x 50
Then, - = -
x x 50 60 30 60
= x - = km
2 2
x 1 x 5
Hence, time taken to cross each other Þ - = -
50 6 30 6
x
x x 2
2 = x ´ 28 = 14 ´ 60 = 56 min or - = or x = 50 km
= 30 50 3
15x 2 15x 15
28 Thus distance between the station A and B = 50 km
Thus, both of them meet at 7 : 56 a.m. 32. (c) Let the speed of the bus be x km / h.
25. (d) Let C1 takes t hrs. Then, then speed of the car = (x + 25) km / h
Q Distance is same. 500 500
\ = + 10
x x + 25
\ 30t = 45 æç t - ö÷
5
è 2ø Þ x2 + 25x – 1250 = 0 Þ x = 25
Thus speed of the bus = 25 km/h
15 Speed of the car = 50 km/h
Þt= hrs
2 Alternative:
15 Difference in speeds 25 km / hr is in only option (c).
\ Distance = 30× = 225km
2 33. (c) Length of train = 12 × 15 = 180 m.
é difference of time ù 180
26. (a) d = product of speed ê ú Then, speed of train = = 10 m / s
ë difference of speed û 18
Now, length of train = 10 ´15 = 150m
4 ´ 5 é10 – (–5) ù [Here, –ve sign indicates
d=
60 êë 5 – 4 úû before the schedule time] 150
= 15sec.
\ Required time =
Þ d = 5 km 10
27. (a) Let the speed of the goods train be x kmph.
225 1
Distance covered by goods train in 10 hours 34. (a) Time = = 135sec = 2 min .
5 4
= Distance covered by express train in 4 hours. 6´
18
\ 10x = 4 × 90 or x = 36.
So, speed of goods train = 36 kmph. æ 280 ö æ 280 18 ö
28. (a) Due to stoppages, it covers 20 km less . 35. (a) Relative speed = ç ÷ m / sec = ç ´ ÷ kmph
è 9 ø è 9 5ø
Time taken to cover 20km = 20 h = 1 h = 112 kmph.
80 4 \ Speed of goods train = (112 – 50) kmph = 62 kmph.
36. (b) Let us name the trains as A and B. Then,
1
= ´ 60 min = 15 min
4 (A's speed) : (B's speed) = b : a = 16 : 9 = 4 : 3.
DISTANCE, SPEED & TIME (BOATS ANDSTREAMS) B-79

37. (a) Let actual speed of train = ST km / h. 42. (a) Let speed of man = S km/h. Then,
Distance covered in 15 min = Distance covered in 12
75 18 min
Then, ST - 6 = ´ = 15
18 5 15 12
16 ´ = [16 + S]
Þ ST = 21 km/h 60 60
Now, let speed of second man = Sm Þ 16 + S = 20 Þ S = 4 km/h
75 18 43. (a) Let speed of man = S km/h. Then,
21 – Sm = ´ = 18
15 5 14 18
36 ´ = [36 - S] Þ 36 – S = 28 Þ S = 8 km/h.
Þ Sm = 3km / h 60 60
38. (c) Speed of train relative to jogger æ 5ö
= (45 – 9) km/h = 36 km/h 44. (d) 4.5 km/h = ç 4.5 ´ ÷ m/sec = 1.25 m/sec,
è 18 ø
æ 5ö
= ç 36 ´ ÷ m / sec = 10 m / sec. æ 5ö
è 18 ø & 5.4 km/h = ç 5.4 ´ ÷ m/sec = 1.5 m/sec.
è 18 ø
Distance to be covered = (240 + 120) m = 360 m.
Let the speed of the train be S m/sec.
æ 360 ö
\ Time taken = ç ÷ sec = 36 sec. Then, (S – 1.25) × 8.4 = (S – 1.5) × 8.5
è 10 ø Þ 8.4S – 10.5 = 8.5S – 12.75 Þ 0.1S = 2.25 Þ S = 22.5.
39. (d) Relative speed = (40 – 20) km/h
\ Speed of the train = æç 22.5 ´ ö÷ km/h = 81 km/h.
18
æ 5ö æ 50 ö
= ç 20 ´ ÷ m / sec = ç ÷ m / sec. è 5ø
è 18 ø è 9 ø
45. (b) Let the speeds of the two trains be S1 m/sec and S2 m/
Length of faster train
sec respectively. Then, length of the first train = 27S1
æ 50 ö 250 7 metres, and length of the second train = 17S2 metres.
= ç ´ 5÷ m = m = 27 m .
è 9 ø 9 9
27S1 + 17S2
40. (d) Let speed of trains are S1 m/s and S2 m/s. \ = 23 Þ 27S1 + 17S2 = 23S1 + 23S2
S1 + S2
130 + 110
Then, S1 – S2 = =4 … (i)
60 S1 3
Þ 4S1 = 6S2 Þ = .
130 + 110 S2 2
and S1 + S2 = = 80 … (ii)
3 46. (c) Relative speed of the trains = (40 + 20) = 60 m/s
on solving (i) and (ii), we get Distance = (120 + 120) = 240 m
S1 = 42 m/s , S2 = 38 m/s Time taken by trains to cross each other completely
41. (b) Let actual speed of train = S m /sec
240
and length of train = L m. = = 4s
60
2´5 L
Then, S – = (47-48)
18 9
Let the speeds of train A and B be VA and VB respectively.
Þ 9S = L + 5 … (i)
220 220
5 L VA – VB = Þ Þ VA - VB = 20 .... (i)
and S - 4 ´ = 11 11
18 10
220
Þ 90S = 9L + 100 … (ii) VA + VB = Þ VA + VB = 220 .... (ii)
1
By (i) & (ii), we get L = 50 m. Solving the equations (i) and (ii), we get VA = 120 km/hr
and VB = 100 km/hr
47. (a) 48. (e)
B-80 MENSURATION

Mensuration
9 Chapter

Name Figure Perimeter Area Nomenclature


B

a c
h 1
Triangle a + b + c = 2s 1. .b.h b = base, h = height
2

C A
b

2. s(s - a)(s - b)(s - c) s = semi perimeter

p h
1
Right angled b+ h +p bh p = b2 + h2
2
triangle
b

a a
1
Equilateral 3a 1. ah a = side
2
triangle
a

3 2 3
2. a h = altitude = a
4 2

b
Rectangle 2(a + b) ab a = Length

a
b = Breadth

a
Square 4a a2 a = side

a
MENSURATION B-81

Name Figure Perimeter Area Nomenclature

a h a
Parallelogram 2(a + b) bh a, b = sides

b h = height

d1
a 1
Rhombus d2 4a d1d 2 a = side
2
d1, d2 are diagonals

p q
h1
d
1
Quadrilateral h2 sum of four sides (AC) (h1 + h2) AC = diagonal
s r 2
h1, h2 are altitude on AC

Circle 2pr = pd pr 2 r = radius


r

d = diameter

1 2
Semicircle pr + 2r pr r = radius
2
r

O
q r
pr 2 1
Sector of a circle l + 2r or lr l = length of arc
360 2
A B
l Segment

prd
or + 2r q = angle of the sector
180

R2
Ring R1 ...................... p (R22 – R12) R2 = Outer radius

R1 = Inner radius
B-82 MENSURATION

Name Figure Lateral Curved Total surface Volume Nomenclature


Surface Area Area

a
Cube 4a2 6a2 a3 a = Edge
a
a

h
Cuboid 2(l + b) h 2(lb + bh + hl) lbh l = Length

l b
b = Breadth
h = Height

1
Pyramid (Perimeter of base) × (Area of base) + (Area of base)
3
(slant height) Lateral surface × height
area

h
Right circular 2prh 2pr(h + r) pr 2 h r = Radius
cylinder r
h = Height

Right prism Perimeter of base 2 (area of base) + Area of base ×


× (height) lateral surface area height

l
h 1 2
Cone prl pr(l + r) pr h h = Height
3
r

R = Radius
MENSURATION B-83

Name Figure Lateral Curved Total surface Volume Nomenclature


Surface Area Area

4 3
Sphere — 4pr2 pr r = Radius
3

r
æ2ö 3
Hemisphere r 2pr2 3pr2 ç ÷ pr r = Radius
è3ø

r1

h
1
l = h 2 + ( r1 - r2 )
2
Frustum pl (r 1 + r2) pl (r 1 + r2) + ph (r12 + r22
3
r2 pr12 + pr22 + r1r2)
B-84 MENSURATION

EXERCISE
1. The area of rectangular field is 460 square metres. If the 9. The length of one pair of opposite sides of a square is
length is 15 per cent more than the breadth ,what is the increased by 5 cm on each side the ratio of the length and
breadth of the rectangular field? the breadth of the newly formed rectangle becomes 3 : 2.
(a) 15 metres (b) 26 metres What is the area of the original square?
(a) 25 sq. cm (b) 81 sq. cm
(c) 34.5 metres (d) Cannot be determined
(c) 120 sq. cm (d) 225 sq. cm
(e) None of these (e) None of these
2. What will be the cost of gardening 1-metre – broad boundary 10. The length and the breadth of a rectangle are in the ratio of
around a rectangular plot having perimeter of 340 metres at 3 : 2 respectively. If the sides of the rectangle are extended
the rate of ` 10 per square metre? on each side by 1 metre, the ratio of length to breadth
(a) ` 3400 (b) ` 1700 becomes 10 : 7. Find the area of the original rectangle in
(c) ` 3440 (d) Cannot be determined square metres.
(a) 256 (b) 150
(e) None of these
(c) 280 (d) Data inadequate
3. The cost of paint is ` 60 per kilograme. A kilogram paint (e) None of these
covers 20 square feet. How much will it cost to paint the 11. A right circular cone is exactly fitted inside a cube in such a
outside of a cube having each side 10 feet? way that the edges of the base of the cone are touching the
(a) ` 3000 (b) ` 900 edges of one of the faces of the cube and the vertex is on
(c) ` 1800 (d) ` 360 the opposite face of the cube. If the volume of the cube is
(e) None of these 343 cc, what approximately is the volume of the cone?
4. 20 buckets of water fill a tank when the capacity of each (a) 80 cc (b) 90 cc
bucket is 13.5 litres. How many buckets will be required to (c) 110 cc (d) 105 cc
(e) 100 cc
fill the same tank if the capacity of each bucket is 9 litres?
12. If the length of a rectangle is increased by 20% and the
(a) 30 (b) 32 breadth is reduced by 20%, what will be the effect on its
(c) 60 (d) Data inadequate area?
(e) None of these (a) 4% increase (b) 6% increase
5. The breadth of a rectangular hall is two-thirds of its length. (c) 5% decrease (d) 4% decrease
If the area of the hall is 2400 sq metres, what is the length in (e) None of these
metres? 13. The ratio between the length and the breadth of a
(a) 120 (b) 80 rectangular plot is 7 : 5. If the perimeter of the plot is 144
metres, what is its area?
(c) 60 (d) 40
(a) 1320 sq. metres (b) 1260 sq. metres
(e) None of these (c) 1280 sq. metres (d) 1380 sq. metres
6. If a pair of opposite sides of a square is increased by 5 cm (e) None of these
each, then the ratio of the sides of the new figure is 3 : 2. 14. The perimeter of a rectangle is equal to the perimeter of a
What is the original area of the square? right-angled triangle of height 12 cm. If the base of the
(a) 125 cm2 (b) 225 cm2 triangle is equal to the breadth of the rectangle, what is the
(c) 81 cm 2 (d) 100 cm2 length of the rectangle”
(a) 18 cm (b) 24
(e) None of these
(c) 22 cm (d) Data inadequate
7. An equilateral triangle, a square and a circle have equal (e) None of these
perimeters. If T denotes the area of the triangle, S, the area 15. The squared value of the diagonal of a rectangle is
of the square and C, the area of the circle, then : (64 + B2) sq cm, where B is less than 8 cm. What is the
(a) S > T > C (b) T > C > S breadth of that rectangle?
(c) T > S > C (d) C > S > T (a) 6 cm (b) 10 cm
(e) None of these (c) 8 cm (d) Data inadequate
(e) None of these
8. The capacity of a cylindrical tank is 246.4 litres. If the height
16. If the height of a triangle is decreased by 40%, land its base
is 4 metres, what is the diameter of the base?
is increased by 40%, what will be the effect on its area?
(a) 1.4 metres (b) 2.8 metres (a) No change (b) 16% increase
(c) 28 metres (d) 14 metres (c) 8% decrease (d) 16% decrease
(e) none of these (e) None of these
MENSURATION B-85
17. A circular ground whose diameter is 35 metres, has a 1.4 26. A rectangular garden has a 5-metre-wide road outside around
metre-broad garden around it. What is the area of the garden all the four sides. The area of the road is 600 square metres.
in square metres? What is the ratio between the length and the breadth of
(a) 160.16 (b) 6.16 that plot?
(c) 122.66 (d) Data inadeuate (a) 3 : 2 (b) 4 : 3
(e) None of these (c) 5 : 4 (d) Data inadequate
18. The length of a rectangular plot is 20 metres more than its (e) None of these
breadth. If the cost of fencing the plot at the rate of 27. Four sheets of 50 cm × 5 cm are to be arranged in such a
` 26.50 per metre is ` 5,300, what is the length of the plot (in manner that a square could be formed. What will be the area
metres)? of inner part of the square so formed?
(a) 2000 cm2 (b) 1600 cm2
(a) 40 (b) 120 2
(c) 1800 cm (d) 2500 cm2
(c) 50 (d) Data inadequate
(e) None of these
(e) None of these
28. In order to fence a square Manish fixed 48 poles. If the
19. A rectangular plate is of 6 in breadth and 12 in length. Two distance between two poles, is 5 metres then what will be
apertures of 2 in diameter each and one aperture of 1 in the area of the square so formed?
diameter have been made with the help of a gas cutter. What (a) Cannot be determined (b) 2600 cm2
is the area of the remaining portion of the plate? (c) 2500 cm2 (d) 3025 cm2
(a) 62.5 sq. in (b) 68.5 sq. in (e) None of these
(c) 64.5 sq. in (d) 66.5 sq. in 29. The area of a side of a box is 120 sq cm. The area of the other
(e) None of these side of the box is 72 sq cm. If the area of the upper surface
20. What would be the length of the diagonal of a square plot of the box is 60 sq cm then find the volume of the box.
whose area is equal to the area of a rectangular plot of 45 m (a) 259200 cm3 (b) 86400 cm3
length and 40 m width? (c) 720 cm 3 (d) Cannot be determined
(a) 42.5 m (b) 60 m (e) None of these
(c) 4800 m (d) Data inadequate 30. A circle and a rectangle have the same perimeter. The sides
(e) None of these of the rectangle are 18 cm and 26 cm. What is the area of the
21. What will be the ratio between the area of a rectangle and circle ?
the area of a triangle with one of the sides of rectangle as (a) 88 cm2 (b) 154 cm2
(c) 1250 cm 2 (d) 616 cm2
base and a vertex on the opposite side of rectangle.
(a) 1 : 2 (b) 2 : 1 (e) None of these
(c) 3 : 1 (d) Data inadequate 31. The cost of carpeting a room 18m long with a carpet 75 cm
(e) None of these wide at `. 4.50 per metre is `. 810. The breadth of the room is:
22. Two roads XY and YZ of 15 metres and 20 metres length (a) 7 m (b) 7.5 m
respectively are perpendicular to each other. What is the (c) 8 m (d) 8.5 m
distance between X & Z by the shortest route? (e) None of these
32. If the perimeter and diagonal of a rectangle are 14 and 5 cms
(a) 35 metres (b) 30 metres
respectively, find its area.
(c) 24 metres (d) 25 metres
(a) 12 cm2 (b) 16 cm2
(e) None of these 2
(c) 20 cm (d) 24 cm2
23. What will be the area of a semi-circle of 14 metres diameter? (e) None of these
(a) 154 sq metres (b) 77 sq metres 33. In an isoscele right angled triangle, the perimeter is 20 metre.
(c) 308 sq metres (d) 22 sq metres Find its area.
(e) None of these (a) 9,320 m2 (b) 8,750 m2
24. The area of a right-angled triangle is two-thirds of the area (c) 7,980 m 2 (d) 6,890 m2
of a rectangle. The base of the triangle is 80 percent of the (e) None of these
breadth of the rectangle. If the perimeter of the rectangle is 34. When the circumference and area of a circle are numerically
200 cm, what is the height of the triangle? equal, then the diameter is numerically equal to
(a) 20 cm (b) 30 cm (a) area (b) circumference
(c) 15 cm (d) Data inadequate (c) 4 (d) 2 p
(e) None of these (e) None of these
25. The area of a rectangular plot is 15 times its breadth. If the 35. In a parallelogram, the length of one diagonal and the
difference between the length and the breadth is 10 metres, perpendicular dropped on that diagonal are 30 and 20 metres
what is its breadth? respectively. Find its area.
(a) 10 metres (b) 5 metres (a) 600 m2 (b) 540 m2
(c) 680 m 2 (d) 574 m2
(c) 7.5 metres (d) Data inadequate
(e) None of these (e) None of these
B-86 MENSURATION
36. The diameter of a garden roller is 1.4 m and it is 2 m long. (a) 4 m (b) 3 m
(c) 2 m (d) 1 m
æ 22 ö
How much area will it cover in 5 revolutions ? ç use p =
è 7 ÷ø (e) None of these
44. If the area of a circle decreases by 36%, then the radius of a
(a) 40 m2 (b) 44 m2 circle decreases by
(c) 48 m2 (d) 36 m2 (a) 20% (b) 18%
(e) None of these
(c) 36% (d) 64%
37. The area of a triangle is 615 m2. If one of its sides is 123
(e) None of these
metre, find the length of the perpendicular dropped on that
45. The floor of a rectangular room is 15 m long and 12 m wide.
side from opposite vertex.
The room is surrounded by a verandah of width 2 m on all
(a) 15 metres (b) 12 metres
its sides. The area of the verandah is :
(c) 10 metres (d) 9 metres
(a) 124 m2 (b) 120 m2
(e) None of these
(c) 108 m2 (d) 58 m2
38. A horse is tethered to one corner of a rectangular grassy
(e) None of these
field 40 m by 24 m with a rope 14 m long. Over how much
area of the field can it graze? 46. A typist uses a paper 12" by 5" length wise and leaves a
margin of 1" at the top and the bottom and a margin of
(a) 154 cm2 (b) 308 m2
½"on either side. What fractional part of the paper is
(c) 150 m2 (d) 407 m2 available to him for typing ?
(e) None of these
39. How many plants will be there in a circular bed whose outer 2 1
(a) (b)
edge measure 30 cms, allowing 4 cm2 for each plant ? 3 2
(a) 18 (b) 750 1 5
(c) 24 (d) 120 (c) (d)
3 7
(e) None of these (e) None of these
40. From a square piece of a paper having each side equal to 10 47. A rectangular lawn 70 m × 30 m has two roads each 5 metres
cm, the largest possible circle is being cut out. The ratio of wide, running in the middle of it, one parallel to the length
the area of the circle to the area of the original square is and the other parallel to the breadth. Find the cost of
nearly : gravelling the road at the rate of ` 4 per square metre.
4 3 (a) ` 2,000 (b) ` 1,800
(a) (b)
5 5 (c) ` 1,900 (d) ` 1,700
(e) None of these
5 6
(c) (d) 48. A circular grass lawn of 35 metres in radius has a path 7
6 7
metres wide running around it on the outside. Find the area
(e) None of these
of path.
41. A square carpet with an area 169 m2 must have 2 metres cut-
(a) 1694 m2 (b) 1700 m2
off one of its edges in order to be a perfect fit for a
(c) 1598 m2 (d) 1500 m2
rectangualar room. What is the area of rectangular room?
(a) 180 m2 (b) 164 m2 (e) None of these
(c) 152 m2 (d) 143 m2 49. A cylindrical bucket of height 36 cm and radius 21 cm is
filled with sand. The bucket is emptied on the ground and a
(e) None of these
conical heap of sand is formed, the height of the heap being
42. A picture 30" × 20" has a frame 2½" wide. The area of the
12 cm. The radius of the heap at the base is :
picture is approximately how many times the area of the
(a) 63 cm (b) 53 cm
frame?
(c) 56 cm (d) 66 cm
1 (e) None of these
(a) 4 (b) 2
2 50. The radius of the wheel of a bus is 70 cms and the speed of
(c) 2 (d) 5 the bus is 66 km/h, then the r.p.m. (revolutions per minutes)
(e) None of these of the wheel is
43. A rectangular plot 15 m ×10 m, has a path of grass outside it. (a) 200 (b) 250
If the area of grassy pathway is 54 m 2, find the width of the (c) 300 (d) 330
path. (e) None of these
MENSURATION B-87

ANSWER KEY
1 (e) 8 (e) 15 (a) 22 (d) 29 (c) 36 (b) 43 (c) 50 (b) 57 (b)
2 (c) 9 (e) 16 (d) 23 (b) 30 (d) 37 (c) 44 (a) 51 (b) 58 (c)
3 (c) 10 (e) 17 (a) 24 (d) 31 (b) 38 (a) 45 (a) 52 (a) 59 (a)
4 (a) 11 (b) 18 (e) 25 (b) 32 (a) 39 (a) 46 (a) 53 (a) 60 (a)
5 (c) 12 (d) 19 (e) 26 (d) 33 (a) 40 (a) 47 (c) 54 (c) 61 (b)
6 (d) 13 (b) 20 (b) 27 (e) 34 (c) 41 (d) 48 (a) 55 (c)
7 (d) 14 (d) 21 (b) 28 (e) 35 (a) 42 (a) 49 (a) 56 (c)

Answers &
Explanations
1. (e) Let the breadth of the rectangular field be ‘x’ m. Then, 5. (c) Let the length of the rectangular hall be ‘x’ m, then the
length of the field will be
2x
breadth of the rectangular hall = m.
x ´ 15 23 x 3
x+ =
100 20
2x 2x2
23 x Area of hall = ´x=
Now, x ´ = 460 3 3
20
2x2
or, 23 x 2 = 460 ´ 20 or, = 2400 or, x = 60 m
3
or, x 2 = 20 ´ 20 6. (d) Let the original side of the square = x cm
or, x = 20 m
x ∗5 3
2. (c) Let l and b be the length and breadth of rectangular < or 2 x ∗ 10 < 3 x
x 2
plot respectively.
[ x = 10 cm
\ According to the question, we have
[ original area = (10)2 = 100 cm2
2 (l + b) = 340 Þ l + b = 170 7. (d) Let the perimeter of each be a.
Now, (l + 2) and (b + 2) be the length and breadth of
a
plot with boundary. Then, side of the equilateral triangle = ; side of the
3
\ Required area = (l + 2) (b + 2) – lb
= lb + 2l + 2b + 4 – lb a a
square = ; radius of the circle = .
= 2 (l + b) + 4 = 344 4 2p
\ Required cost = 344 ´ 10 = 3440 2 2
3 æaö 3a 2 æaö a2
3. (c) Area of cube \T= ´ç ÷ = ; S =ç ÷ = ;
4 è3ø 36 è4ø 16
= 6 × (side)2 = 6 × 10 × 10 = 600 square feet.
2
600 æ a ö a2 7 a 2
Cost to paint outside of the cube = ´ 60 C = p´ ç ÷ = = .
20 è 2p ø 4p 88
= ` 1800 So, C > S > T.
4. (a) Capacity of the tank = 20 × 13.5 = 270 litres 8. (e) Capacity (volume) of a cylindrical tank = pr2h
When the capacity of each bucket = 9 litres, then the (Here r = radius and h = height of the tank)
required no. of buckets
22 2
Now, from the question,246.4 × 0.001 = ´r ´4
270 7
= = 30
9 [Q 1 litre = 1000 cm3 = 0.001 m3]
B-88 MENSURATION
18. (e) Perimeter of the rectangular plot = [(b + 20) + b]× 2
0.2464 ´ 7
or, = r2
22 ´ 4 5300
= = 200
or, r = 0.14 m 26.5
or, diameter = 2r = 0.28m \ (2b + 20)2 = 200
9. (e) Let original length of each side = x cm. Þ b = 40
Then, its area = (x2) cm2. Þ l = 40 + 20 = 60 m
Length of rectangle formed = (x + 5) cm
ìï 2
1 üï
2
(e) Required area = 6 × 12 – í 2 ´ p æç ö÷ + p æç ö÷ ý
and its breadth = x cm. 2
19.
ïî è 2ø è 2ø ï
x+5 3 þ
\ = Û 2x + 10 = 3x Û x = 10
x 2 pö
æ 9p 9 22
\ Original length of each side = 10 cm = 72 – ç 2p + ÷ = 72 – = 72 - ´
è 4ø 4 4 7
and its area = 100 cm2
10. (e) Let the length and breadth be l and b respectively. æ 99
= 72 – ç
ö
=÷ 7.07 = 64.94 sq in.
è 14 ø
l 3 3
= or, l = b ........ (i) 20. (b) a2 = 45 × 40 = 1800
b 2 2
\ a = 1800 = 30 2
l + 1 10
= or, 7 l – 10b = 3 ........ (ii) \ Diagonal of the square = 2a= 2 × 30 2
b +1 7
= 30 × 2 = 60 m
From eq. (i)
10.5b – l 0b = 6 or, 0.5b = 3 or, b = 6 and l = 9 21. (b) A E D
Area = l × b = 6× 9 = 54m2
3
11. (b) Edge of the cube = 343 = 7 cm
\ Radius of cone = 3.5 cm
height = 7 cm
1 2 B F C
volume of cone = pr h
3
1
Area of D EBC = × BC × EF
1 2 1 22 1 2
pr h = ´ ´ (3.5) 2 ´ 7 = ´ 22 ´ 12.25 » 90cc
3 3 7 3
1
= × BC × AB [Since, EF = AB]
xy 2
12. (d) Percentage change = x - y -
100
1
Area of D EBC = × area of DABCD
20 ´ 20 2
= 20 – 20 – = -4% = 4% decrease
100 [ Required ratio = 2 : 1.
13. (b) Let the length and breadth be 7x and 5x respectively. 22. (d) XYZ is a right-angled triangle
Then, P = 2(7x + 5x) = 144 Þ x = 6 X
Area = 7 × 6 × 5 × 6 = 1260sq.m.
14. (d) P = 2 (l + b) = L + B + h = L + b + 12.
Data inadequate. 15 m
15. (a) Diagonal2 = 64 + B2 or, 102 = 64 + 62
é 40 ´40 ù Z
16. (d) Regd effect = ê∗ 40 , 40 , ú % < ,16% Y 20 m
êë 100 úû
i.e., the area will decrease by 16% XZ = 15 2 ∗ 20 2 = 625 = 25 m
17. (a) Req. area = p [(17.5 + 1.4)2 – (17.5)2] 1 2
23. (b) Area of semicircle = pr
22 2
= × (36.4 × 1.4) [since a2 – b2 = (a + b) (a – b)]
7 1 22
= 22 × 36.4 × 0.2 = 160.16 sq m = ´ ´ 7 ´ 7 = 77m 2
2 7
MENSURATION B-89
24. (d) Let the base and height of triangle, and length and Area of the room = Area of the carpet
breadth of rectangle be L and h, and L1 and b1
æ 75 ö 2 2
1 2 = ç 180 ´ ÷ m = 135 m .
respectively. Then ´ L ´ h = ´ L1 ´ b1 ...(i) è 100 ø
2 3
4 æ Area ö æ 135 ö
L = b1 ....(ii) and L1+ b1 = 100 .....(iii) \ Breadth of the room = ç ÷ =ç ÷m
5 è Length ø è 18 ø
In the above we have three equations and four = 7.5 m.
unknowns. Hence the value of ‘h’ can’t be determined. 32. (a) In a rectangle,
25. (b) L × B = 15 × B
\ L = 15 m (perimeter) 2
= (diagonal) 2 + 2 ´ area
and L – B = 10 4
\ B = 15 – 10 = 5 m
(14)2
5m Þ = 52 + 2 ´ area
4
49 = 25 + 2 × area
49 - 25 24
\ Area = = = 12cm 2
26. (d) 2 2
b
l 33. (a) In an isoscele right angled triangle,
Area = 23.3 × perimeter2
= 23.3 × 202 = 9320 m2
Area of shaded portion = 600 m.
\ 34. (c) According to question, circumference of circle
(l + 10) (b + 10) – lb = 600
or, lb + 10b + 10l + 100 – lb = 600 = Area of circle
or, 10 (b + (l) = 500 2
\ ædö
b + l = 50 or πd = π ç ÷ [where d = diameter]
From this equation we can’t get the required ratio. è2ø
\d=4
27. (e)
35. (a) In a parallelogram.
Area = Diagonal × length of perpendicular on it.
= 30 × 20 = 600 m2
36. (b) Required area covered in 5 revolutions
22
= 5 × 2prh = 5 × 2 × × 0.7 × 2 = 44 m2
7
The four sheets are BMRN, AMQL, NSKC and DLPK 37. (c) In a triangle,
\ Side of the new square sheet = 50 + 5 = 55 cm and
the side of the inner part of the square (55 – 10) = 45 cm 1
Area = ´ length of perpendicular × base
Hence, area = (45)2 = 2025 sq. cm. 2
28. (e) Let the side of the square be x m.
1
\ Perimeter of the square = 48 × 5 = 4x \ x = 60 m or 615 = ´ length of perpendicular × 123
2
\ Area = (60)2 = 3600 m2
3 615 ´ 2
29. (c) Volume of the box = 120 ´ 72 ´ 60 = 720cm \ Length of perpendicular = = 10 m.
123
30. (d) Perimeter of the circle = 2pr = 2(18 + 26)
38. (a) 14 m
22 D C
Þ 2 ´ ´ r = 88 Þ r = 14
7
14 m
\ Area of the circle
24 m
2 22 2
= pr = ´ 14 ´14 = 616 cm .
7
40 m
A B
æ Total cost ö
31. (b) Length of the carpet = ç ÷
è Rate / m ø Area of the shaded portion
1
æ 8100 ö = ´ p(14 )2 = 154 m2
=ç ÷ m = 180 m. 4
è 45 ø
B-90 MENSURATION
39. (a) Circumference of circular bed = 30 cm
43. (c)
2
(30)
Area of circular bed =
4π 10 w
15
Space for each plant = 4 cm2
\ Required number of plants
Let the width of the path = W m
(30)2 then, length of plot with path = (15 + 2W) m
= ¸ 4 = 17.89 = 18 (Approx)
4π and breadth of plot with path = (10 + 2 W) m
40. (a) Area of the square = (10)2 = 100 cm2 Therefore, Area of rectangular plot (wihout path)
The largest possible circle would be as shown in the = 15 × 10 = 150 m2
figure below : and Area of rectangular plot (with path)
= 150 + 54 = 204 m2
S R
Hence, (15 + 2W) × (10 + 2W) = 204
Þ 4W2 + 50 W – 54 = 0
Þ 2W2 + 25 W – 27 = 0
5 cm
10 cm Þ (W – 2) (W + 27) = 0
Thus W = 2 or –27
\ with of the path = 2 m
P Q
10 cm 44. (a) If area of a circle decreased by x % then the radius of a
circle decreases by
22
´ (5)2 =
22 ´ 25 (100 - 10 100 - x )% = (100 - 10 100 - 36)%
Area of the circle =
7 7
= (100 - 10 64)%
22 ´ 25 22 11
Required ratio = = =
= 100 - 80 = 20%
7 ´ 100 28 14
45. (a) Area of the outer rectangle = 19 × 16 = 304 m2
4
= 0.785 » 0.8 =
5
2m
41. (d) Side of square carpet = Area = 169 = 13m
After cutting of one side,
Measure of one side = 13 – 2 = 11 m
and other side = 13 m (remain same) 12
2m 2m
\ Area of rectangular room = 13 × 11 = 143 m2
15
42. (a) 2m
2.5''
2.5''

20'' Area of the inner rectangle = 15 × 12 = 180 m2


Required area = (304 – 180) = 124 m2
46. (a) Area of paper = 12 × 5 = 60 sq. inch
30''
1
Length of frame = 30 + 2.5 × 2 = 35 inch Area of typing part = (12 – 1 ´ 2) ´ (5 - ´ 2)
2
Breadth of frame = 20 + 2.5 × 2 = 25 inch = (12 – 2) × (5 – 1)
Now, area of picture = 30 × 20 = 600 sq. inch = (10 × 4) sq. inch
Area of frame = (35 × 2.5) + (25 × 2.5)
\ Area of frame = 875 – 600 = 275 sq. inch 40 2
\ Required fraction = =
60 3
MENSURATION B-91
\ Area of path = p(42)2 – p(35)2
47. (c) = p(422 – 352)
5m
= p( 42 + 35) (42 –35)

5m 30 m 22
= p × 77 × 7 = ´ 77 ´ 7 = 1694 m 2
7
49. (a) Volume of the bucket = volume of the sand emptied
Volume of sand = p (21)2 × 36
70 m
Let r be the radius of the conical heap.
1 2
Total area of road Then, pr ´ 12 = p(21)2 ´ 36
3
= Area of road which parallel to length + Area of road
or r2 = (21)2 × 9 or r = 21 × 3 = 63
which parallel to breadth – overlapped road 50. (b) Radius of the wheel of bus = 70 cm. Then,
= 70 × 5 + 30 × 5 – 5 × 5 circumference of wheel = 2pr = 140 p = 440 cm
= 350 + 150 – 25 = 500 – 25 = 475 m2 Distance covered by bus in 1 minute
\ Cost of gravelling the road = 475 × 4 = ` 1900 66
= ´ 1000 ´100 cms
48. (a) Radius of a circular grass lawn (without path) = 35 m 60
\ Area = pr2 = p (35)2 Distance covered by one revolution of wheel
Radius of a circular grass lawn ( with path) = circumference of wheel = 440 cm
= 35 + 7 = 42 m 6600000
\ Revolutions per minute = = 250
\ Area = pr2 = p(42)2 60 ´ 440
B-92 SERIES

Series
10 Chapter
A series is a sequence of numbers/alphabetical letters or both which follow a particular rule. Each element of series is called ‘term’.
We have to analyse the pattern and find the missing term or next term to continue the pattern.
Types of series are explained in the following chart :

SERIES

Number Alphabet Alpha-numeric Mixed Letter Correspondence


series series series series series series

A series that is A series that is A series in A series which A series of letters, A series consists of
made by only made by only which both is created by which follow a three sequence with
alphabets and the combination certain pattern, is three different
number or digit alphabetic letters of two or more
numbers are given with four / elements (for ex.
than two series five times blank capital letters,
used
1. Ascending series spaces in between. numbers and small
The order of letters). An element
2. Descending series
missing letters of each sequence is
3. Oscillating series correspond to the
is correct answer.
element of other
sequence on the
basis of the similarity
in position.
In number series, relationship between the terms is of any kind. Sol. (b) 4 6 9 13 18 Correct answer
For example.
(1) Consecutive even nunbers +2 +3 +4 +5
(2) Consecutive odd numbers EXAMPLE 2. Choose the next term of series given below.
(3) Consecutive prime numbers 64, 32, 16, 8, ?
(a) 0 (b) 1 (c) 2 (d) 4
(4) Square of numbers
Sol. (d) Each number is half of its previous number.
(5) Cubes of numbers
(6) Square root of numbers (II) To find the missing number of series :
(7) Omission of certain number of letter in any consecutive
order EXAMPLE 3. What will come in place of question mark in
(8) Addition /subtraction/ multiplication/ division by some the following series?
number ( For Ex. A.P & G.P) or any other relation. 79, 87, ? , 89, 83
TYPES OF QUESTIONS : (a) 80, (b) 81 (c) 82 (d) 88
(I) Complete the series
(II) Find Missing number of the series Sol. (b) 79 87 81 89 83
(III) Find Wrong number of the series +8 -6 +8 -6
EXAMPLES ON NUMBER SERIES EXAMPLE 4. What will come in place of question mark in
(I) Complete the series
the following series?
EXAMPLE 1. Which of the following is the next term of 37, 41, ? , 47, 53
series given below ?
(a) 42 (b) 43 (c) 46 (d) 44
4, 6, 9, 13, ....
(a) 17 (b) 18 (c) 19 (d) 20 Sol. (b) Consecutive prime numbers.
SERIES B-93

EXAMPLE 5. What will come in place of question mark in Sol. (b) Alphabets follow the sequence
the following series? K M P T Y
21, 34, ? , 89, 144
+2 +3 +4 +5
(a) 43 (b) 55 (c) 64 (d) 71
Sol. (b) Each number is the sum of the two preceding And numbers are increasing by 2
numbers. EXAMPLE 11. What will come in place of question mark
21 + 34 = 55 in the following series?
34 + 55 = 89 2 Z 5, 7 Y 7, 14 X 9, 23 W 11, 34 V 13, ?
55 + 89 = 144 Sol. First number is the sum of the number of the preceeding
term.
(III) To find the wrong term in the series :
Middle letter is moving one step backward.
EXAMPLE 6. Find the wrong term in the series Third number in a term is a series of odd numbers.
3, 8, 15, 24, 34, 48, 63. \ 6th term = 47 U 15.
(a) 15 (b) 15 (c) 34 (d) 63 EXAMPLES ON MIXED SERIES
Sol. (c) 2 – 1, 3 – 1, 4 – 1, 5 – 1, 62 – 1
2 2 2 2 EXAMPLE 12. Complete the series
EXAMPLES ON ALPHABETIC SERIES Z, L, X, J, V, H, T, F, __, __
(a) D, R (b) R, D (c) D, D (d) R, R
EXAMPLE 7. What will come in place of question mark in
Sol. (b) The given sequence consists of two series
the following series?
(i) Z, X, V, T, __
G, H, J, M, ?
(ii) L, J, H, F, __. Both consisting of alternate letters in
(a) R (b) S (c) Q (d) P the reverse order.
Sol. (c) G H J Q \ Next term of (i) series = R, and
M
Next term of (ii) series = D
+1 +2 +3 +4
EXAMPLE 13. What will come in place of question mark
EXAMPLE 8. What will come in place of question mark in in the following series?
the following series? 7, 5, 26, 17, 63, 37, 124, 65, ?, ?
BF, CH, ? , HO, LT (a) 101, 215 (b) 101, 101
(a) FG (b) EK (c) CE (d) FJ (c) 215, 101 (d) 215, 215
Sol. (c) The given series consists of two series
+2 +3 +4 +5 (i) 7, 26, 63, 124 .....
Sol. (b) BF CH EK HO LT (ii) 5, 17, 37, 65 .....
In the first series,
+1 +2 +3 +4 7 = 23 – 1, 26 = 33 – 1, 63 = 43 – 1 ,
124 = 53 – 1, \ 63 – 1 = 215
EXAMPLE 9. What will come in place of question mark in
and in the second series.
the following series?
5 = 22 + 1, 17 = 42 + 1,
DCXW, FEVU, HGTS, ? 37 = 62 + 1, 65 = 82 + 1,
(a) LKPO (b) ABYZ \ 102 + 1 = 101
(c) JIRQ (d) LMRS
EXAMPLES ON LETTER SERIES
Sol. (c) JIRQ
EXAMPLE 14. Which sequence of letters when placed at
–2 –2 –2
+2 +2 +2
the blanks one after another will complete the given letter
series?
D C X W F E V U H G T S J I R Q ba ab–a ba– bba– –
(a) bbaa (b) aaaa (c) abab (d) baba
+2 +2 +2
–2 –2 –2 Sol. (d) b a a b b a / b a a b b a / b a.
EXAMPLES ON ALPHA-NUMERIC SERIES
EXAMPLE 10. What will come in place of question mark
in the following series?
K 1, M 3, P 5, T 7, ?
(a) Y 9 (b) Y 11 (c) V 9 (d) V 11
B-94 SERIES

EXERCISE
1. In the following number series a wrong number is given. 15. 3 4 12 45 190 1005 6066
Find out the wrong number. (a) 98 (b) 96 (c) 384 (d) 386
3 10 35 172 885 5346 37471 (e) None of these
(a) 10 (b) 5346 (c) 885 (d) 35 16. 11 18 39 97.5 295.5 1037.5
(e) 172 (a) 122 (b) 122.5 (c) 123 (d) 124
2. In the following number series a wrong number is given. (e) None of these
Find out the wrong number. 17. 2 7 19 43 99 209 431
318 158 76 38 18 8 3 (a) 181 (b) 183 (c) 87 (d) 85
(a) 38 (b) 18 (c) 158 (d) 318 (e) None of these
(e) 76 Directions (Qs.18-22): In each of the following questions a
Directions (Qs. 3-7): In each of these questions a number series number series is given with one wrong number. Find that wrong
is given. Only one number is wrong in each series. You have to number.
find out the wrong number.
18. 2 3 6 15 45 156.5 630
3. 10 15 24 35 54 75 100
(a) 3 (b) 45 (c) 15 (d) 6
(a) 35 (b) 75 (c) 24 (d) 15
(e) 54 (e) 156.5
4. 1 3 4 7 11 18 27 47 19. 36 20 12 8 6 5.5 4.5
(a) 4 (b) 11 (c) 18 (d) 7 (a) 5.5 (b) 6 (c) 12 (d) 20
(e) 27 (e) 8
5. 3 2 3 6 12 37.5 115.5 20. 1 3 9 31 128 651 3913
(a) 37.5 (b) 3 (c) 6 (d) 2 (a) 651 (b) 128 (c) 31 (d) 9
(e) 12 (e) 3
6. 2 8 32 148 765 4626 32431 21. 2 3 10 40 172 885 5346
(a) 765 (b) 148 (c) 8 (d) 32 (a) 3 (b) 855 (c) 40 (d) 172
(e) 4626 (e) 10
7. 2 3 11 38 102 229 443 22. 5 8 16 26 50 98 194
(a) 11 (b) 229 (c) 120 (d) 38 (a) 8 (b) 26 (c) 50 (d) 16
(e) 3 (e) 98
Directions (Qs. 8-12): In each of the following number series, a Directions (Qs. 23 -27) : In each of the following questions a
wrong number is given. Find out that number.
number series is given. Only one number is wrong in each series.
8. 5 10 17 27 37 50 65
Find out that wrong number, and taking this wrong number as the
(a) 10 (b) 17 (c) 27 (d) 37
(e) 50 first term of the second series formed following the same logic,
9. 108 54 36 18 9 6 4 find out the third term of the second series.
(a) 54 (b) 36 (c) 18 (d) 9 23. 1 2 8 21 88 445
(e) 6 (a) 24.5 (b) 25 (c) 25.5 (d) 25
10. 2 3 5 8 14 23 41 69 (e) None of these
(a) 5 (b) 8 (c) 14 (d) 41 24. 6 7 18 63 265 1365
(e) 6 9 (a) 530 (b) 534 (c) 526 (d) 562
11. 0 1 9 36 99 225 441 (e) None of these
(a) 9 (b) 36 (c) 99 (d) 225 25. 7 23 58 127 269 555
(e) 441 (a) 263 (b) 261 (c) 299 (d) 286
12. 3 7. 5 15 37.5 75 167.5 375 (e) None of these
(a) 167.5 (b) 75 (c) 37.5 (d) 15 26. 5 4 9 18 66 195
(e) 7.5 (a) 12 (b) 25 (c) 20 (d) 18
Directions (Qs. 13-17): In each of the following questions, a (e) None of these
number series is given in which one number is wrong. You have
27. 2 7 28 146 877 6140
to find out that number and have to follow the new series which
(a) 242 (b) 246 (c) 252 (d) 341
will be started by that number. By following this, which will be the
third number of the new series? (e) None of these
13. 1 2 6 33 148 765 4626 Directions (Qs. 28-32): In each of these questions a number
(a) 46 (b) 124 (c) 18 (d) 82 series is given. Only one number is wrong in each series. You
(e) None of these have to find out the wrong number.
14. 2 9 5 36 125 648 3861 28. 1 2 4.5 11 30 92.5 329
(a) 12 (b) 11 (c) 75 (d) 72 (a) 92.5 (b) 4.5 (c) 11 (d) 2
(e) None of these (e) 30
SERIES B-95

29. 2 5 7 12 19 32 50 Directions (Qs. 43-47) : In each of the following questions, a


(a) 7 (b) 12 (c) 32 (d) 19 number series is given. After the series, a number is given followed
(e) 5 by (a), (b), (c), (d) and (e). You have to complete the series starting
30. 2 13 65 271 817 1639 1645 with the number given, following the sequence of the given series.
(a) 13 (b) 65 (c) 271 (d) 817 43. 15 16 25 50 189
(e) 1639 (a) (b) (c) (d) (e)
31. 3 4 16 75 366 1945 11886 Which of the following numbers will come in place of (e)?
(a) 16 (b) 75 (c) 366 (d) 1945 (a) 354 (b) 273 (c) 394 (d) 426
(e) 4 (e) None of these
32. 2 14 91 546 3002 15015 44. 6 3.5 4.5 8.25 40
(a) 15015 (b) 91 (c) 14 (d) 3002 (a) (b) (c) (d) (e)
(e) 546 Which of the following numbers will come in place of (c)?
Directions (Qs.33-37): In the following number series, a wrong (a) 20.5 (b) 21.5 (c) 33.75 (d) 69.5
number is given. Find out that wrong number. (e) None of these
33. 2 11 38 197 1172 8227 65806 45. 9 10 22 69 5
(a) 11 (b) 38 (c) 197 (d) 1172 (a) (b) (c) (d) (e)
(e) 8227 Which of the following numbers will come in place of (b)?
34. 16 19 21 30 46 71 107 (a) 15 (b) 28 (c) 14 (d) 45
(a) 19 (b) 21 (c) 30 (d) 46 (e) None of these
(e) 71 46. 2 10 27 60 5
35. 7 9 16 25 41 68 107 173 (a) (b) (c) (d) (e)
(a) 107 (b) 16 (c) 41 (d) 68 Which of the following numbers will come in place of (b)?
(e) 25 (a) 39 (b) 13 (c) 34 (d) 38
36. 42 3.5 7.5 26.25 118.125 (e) None of these
(a) 118.125 (b) 26.25 (c) 3.5 (d) 2 47. 5 149 49 113 146
(e) 7.5 (a) (b) (c) (d) (e)
37. 16 4 2 1.5 1.75 1.875 Which of the following numbers will come in place of (d)?
(a) 1.875 (b) 1.75 (c) 1.5 (d) 2 (a) 290 (b) 234 (c) 254 (d) 218
(e) 4 (e) None of these
Directions (Qs. 38-42) : In each of the following questions a Directions (Qs. 48-52): In each of the following questions a
number series is given. A number is given after the series and number series is given. After the series a number is given followed
then (a), (b), (c), (d) and (e) are given. According to the given by (a), (b), (c), (d) and (e).You have to complete the series starting
series, you have to form a new series which begins with the given with the number given and following the sequence of the original
number, and then answer the question asked. series. Answer the questions that follow the series.
38. 6 3.0 4.5 2.25 48. 5 6 16 57 244 1245
40 (a) (b) (c) (d) (e) 2 (a) (b) (c) (d) (e)
What will come in place of (d)?
Which of the following numbers will come in place of (c)?
(a) 46 (b) 39 (c) 156 (d) 173
(a) 20.5 (b) 21.5 (c) 33.75 (d) 69.5
(e) None of these
(e) 15
49. 3 5 9 17 33 65
39. 5 9 26 90
7 (a) (b) (c) (d) (e)
13 (a) (b) (c) (d) (e)
What will come in place of (d)?
Which of the following numbers will come in place of (e)?
(a) 95 (b) 51 (c) 99 (d) 49
(a) 2880 (b) 2292 (c) 1716 (d) 3432
(e) None of these
(e) None of these
50. 7 4 5 9 20 52.5
40. 4 9 25 103
3 (a) (b) (c) (d) (e)
3 (a) (b) (c) (d) (e)
What will come in place of (c)?
Which of the following numbers will come in place of (c)?
(a) 4.5 (b) 2 (c) 6 (d) 7
(a) 391 (b) 81 (c) 91 (d) 79
(e) None of these
(e) None of these
51. 3 10 32 111 460 2315
41. 6 10 32 126
2 (a) (b) (c) (d) (e)
2 (a) (b) (c) (d) (e)
What will come in place of (b)?
Which of the following numbers will come in place of (a)?
(a) 29 (b) 30 (c) 26 (d) 28
(a) 4 (b) 6 (c) 2 (d) 3
(e) None of these
(e) None of these 52. 5 8 6 10 7 12
42. 1260 628 312 154 7 (a) (b) (c) (d) (e)
788 (a) (b) (c) (d) (e) What will come in place of (c)?
Which of the following numbers will come in place of (d)? (a) 14 (b) 16 (c) 9 (d) 11
(a) 194 (b) 45.5 (c) 48 (d) 72.5 (e) None of these
(e) None of these
B-96 SERIES
Directions (Qs. 53-57): In each of the following questions a 61. 12 26 11 36 9
number series is given. After the series, a number is given followed 7 (a) (b) (c) (d) (e)
by (a), (b), (c), (d) and (e). You have to complete the series starting What would come in place of (c)?
with the number given following the sequence of the given series (a) 7 (b) 21 (b) 4 (d) 11
and answer the question given below the series. (e) None of these
53. 3 12 30 66 138 282 62. 2 3 6 15 45
7 (a) (b) (c) (d) (e) 16 (a) (b) (c) (d) (e)
What will come in place of (b)? What would come in place of (d)?
(a) 34 (b) 70 (c) 46 (d) 62 (a) 360 (b) 120 (c) 300 (d) 240
(e) None of these (e) None of these
Directions (Qs. 63-67) : In each of the following questions a
54. 2 3 10 39 172 885
number series is given. After the series, a number is given followed
5 (a) (b) (c) (d) (e) by (a), (b), (c), (d) and (e). You have to complete the series starting
What will come in place of (d)? with the number given following the sequence of the given series.
(a) 244 (b) 175 (c) 208 (d) 196 Then answer the question given below it.
(e) None of these 63. 9 19.5 41 84.5
55. 3 5 22 13.5 35 19 12 (a) (b) (c) (d) (e)
1 (a) (b) (c) (d) (e) Which of the following numbers will come in place of (c)?
What will come in place of (a)? (a) 111.5 (b) 118.5 (c) 108.25 (d) 106.75
(a) 3 (b) 2 (c) 5 (d) 4 (e) None of these
64. 4 5 22 201
(e) None of these
7 (a) (b) (c) (d) (e)
56. 2 3 7 25 121 721 Which of the following numbers will come in place of (d)?
3 (a) (b) (c) (d) (e) (a) 4948 (b) 4840 (c) 4048 (d) 4984
What will come in place of (c)? (e) None of these
(a) 31 (b) 49 (c) 45 (d) 39 65. 5 5.25 11.5 36.75
(e) None of these 3 (a) (b) (c) (d) (e)
57. 4 2 3 7.5 26.25 118.125 Which of the following numbers will come in place of (c)?
6 (a) (b) (c) (d) (e) (a) 34.75 (b) 24.75 (c) 24.5 (d) 34.5
What will come in place of (c)? (e) None of these
(a) 12.25 (b) 11.5 (c) 12.5 (d) 11.125 66. 38 19 28.5 71.25
18 (a) (b) (c) (d) (e)
(e) None of these
Which of the following numbers will come in place of (d)?
Directions (Qs. 58 - 62): A number series is given in each of the
(a) 118.75 (b) 118.25 (c) 108.25 (d) 118.125
following questions. A number is given after the series followed
(e) None of these
by ( a), (b), (c), (d) and (e). First you have to understand how the
67. 25 146 65 114
series is formed and then starting with that number you have to
39 (a) (b) (c) (d) (e)
complete the second series. Now, answer the given questions.
Which of the following numbers will come in place of (e)?
(a) 122. (b). 119 (c) 112 (d) 94
58. 13 14 5 18 0.5 19
(e) None of these
(a) (b) (c) (d) (e)
Directions (Qs. 68-72) : In each of the following questions a
What would come in place of (e) ? number series is given. A number in the series is suppressed by
(a) 13.75 (b) 27 (c) 18.75 (d) 6.75 letter ‘A’. You have to find out the number in the place of ‘A’ and
(e) None of these use this number to find out the value in the place of the question
59. 17 21.5 30.5 44 62 mark in the equation following the series.
21 (a) (b) (c) (d) (e) 68. 36 216 64.8 388.8 A 699.84 209.952
What would come in place of (e)’? A ÷ 36 = ?
(a) 61.39 (b) 0.324 (c) 3.24 (d) 6.139
(a) 84.5 (b) 88.5 (c) 86 (d) 88
(e) 32.4
(e) None of these
69. 42 62 92 132 A 242 312
60. 1 8 10 35 136
A + 14 = ? × 14
2 (a) (b) (c) (d) (e)
What would come in place of (c)’? 6 5 1
(a) 11 (b) 14 (c) 12 (d) 12
(a) 40 (b) 42 (c) 51 (d) 49 7 7 2
(e) None of these 1
(e) 12
6
SERIES B-97
70. 4 7 12 19 28 A 52 Directions (Qs. 78-82) : One number is wrong in each of the
A2 – 4 = ? number series given in each of the following questions. You have
(a) 1365 (b) 1353 (c) 1505 (d) 1435 to identify that number and assuming that a new series starts
with that number following the same logic as in the given series,
(e) 1517
which of the numbers given in (a), (b), (c), (d) and (e) given below
71. 18 24 A 51 72 98 129 each series will be the third number in the new series?
3 4 78. 3 5 12 38 154 914 4634
A´ ´ =?
7 5 (a) 1636 (b) 1222 (c) 1834 (d) 3312
(e) 1488
23 12 2
(a) 12 (b) 11 (c) 12 (d) 14 79. 3 4 10 34 136 685 4116
35 35 5
(a) 22 (b) 276 (c) 72 (d) 1374
2 (e) 12
(e) 10
7 80. 214 18 162 62 143 90 106
(a) 34 (b) 110 (c) 10 (d) 91
3 3 9 9 27 27 (e) 38
72. A
8 4 16 8 32 16 81. 160 80 120 180 1050 4725 25987.5
A= ? (a) 60 (b) 90 (c) 3564 (d) 787.5
(e) 135
2 6 6 3 82. 2 3 7 13 26 47 78
(a) (b) (c) (d)
3 8 4 4 (a) 11 (b) 13 (c) 15 (d) 18
9 (e) 20
(e) 83. What should come in the place of question mark (?) in the
8
following letter sequence?
Directions (Qs. 73-77) : In each of the following questions, a AZB, CYD, EXF, ?, IVJ, KUL
number series is given. After the series, below it, a number is
(a) FYH (b) GWG (c) HWH (d) GUF
given followed by (a), (b), (c), (d) and (e). You have to complete
(e) None of these
the series starting with the given number following the sequence
of the given series. Then answer the questions given below it. 84. What should come in the place of question-mark (?) in the
following series?
73. 11 15 38 126
ATTRIBUTION, TTRIBUTIO, RIBUTIO, IBUTI,?
7 (a) (b) (c) (d) (e)
(a) BUT (b) UTI (c) UT (d) IBU
Which of the following will come in place of (c)?
(e) None of these
(a) 102 (b) 30 (c) 2140 (d) 80
85. What will come in place of the question mark (?) in the
(e) 424 following sequence?
74. 2 3 8 27 SEQUENTIAL EQUENTIAL EQUENTI ? QUEN
5 (a) (b) (c) (d) (e) (a) QUENTI (b) QUENT (c) UENTI (d) EQUENT
Which of the following will come in place of (e)? (e) None of these
(a) 184 (b) 6 (c) 925 (d) 45 86. Which pair of English letters should be the next in the series
(e) 14 of pairs of letters of English alphabet given below? BE GK
75. 2 3 9 40.5 MR TZ BI
4 (a) (b) (c) (d) (e) (a) JR (b) KS (c) JS (d) KR
Which of the following will come in place of (b)? (e) JQ
(a) 486 (b) 81 (c) 3645 (d) 18 87. Choose the missing term out of the given alternatives :
(e) 6 J2Z, K4X, I7V, ?, H16R, M22P
76. 12 28 64 140 (a) I11T (b) L11S (c) L12T (d) G11T
37 (a) (b) (c) (d) (e) (e) None of these
Which of the following will come in place of (e)? 88. Find the next triplet of alphabets in the following series :
(a) 1412 (b) 164 (c) 696 (d) 78 ABD, DGK, HMS, MTB, SBL, ..... ?
(e) 340 (a) ZKU (b) ZKW (c) XKW (d) ZAB
77. 5 12 60 340 (e) None of these
7 (a) (b) (c) (d) (e) 89. Which sequence of letters when placed at the blanks one
Which of the following will come in place of (d)? after another will complete the given letter series ?
(a) 172 (b) 5044 (c) 1012 (d) 20164 ba _ b _ aab _ a _ b
(a) abaa (b) abba (c) baab (d) babb
(e) 28
(e) None of these
B-98 SERIES
90. Which sequence of letters when placed at the blanks one 95. Find the missing letter in the series : C4X, F9U, I16R, ?
after another will complete the given letter series ? (a) L25P (b) L25O (c) L27P (d) L260
c _ bba _ cab _ ac _ ab _ ac (e) None of these
(a) abcbc (b) acbcb (c) babcc (d) bcacb 96. Which sequence of letters when placed at the blanks one
(e) None of these after another will complete the given letter series ?
91. Which one number does not belong to theseries ? abca — bcaab — ca — bbc — a
3, 5, 8, 11, 17, 23 (a) ccaa (b) bbaa (c) abac (d) abba
(a) 8 (b) 11 (c) 17 (d) 23 (e) None of these
(e) None of these 97. What will be the next term in : BDF, CFI, DHL, ?
92. Which one number does not belong to the series ? (a) CJM (b) EIM (c) EJO (d) EMI
905, 180, 175, 35, 30, 6, 1 (e) None of these
(a) 6 (b) 1 (c) 175 (d) 905 98. Complete the given series.
(e) None of these LXF, MTJ, NPN, OLR, ?
93. 1 C V, 5 F U, 9 I T, 15 L S, 17 O R (a) PHV (b) PIU (c) PJW (d) PKX
(a) 9 JT (b) 15 LS (c) 5 FU (d) 17 OR (e) None of these
94. B 0 R, G 3 U, E 3 P, J 7 S, H 9 N 99. Which sequence of letters when placed at the blanks one
(a) E 3 P (b) H 9 A (c) J 7 S (d) G 3 U after another will complete the given letter series ?
(e) None of these b — b — bb — bbb — bb — b
(a) bbbbba (b) bbaabb (c) ababab (d) aabaab
(e) None of these

ANSWER KEY
1 (d) 13 (c) 25 (b) 37 (b) 49 (e) 61 (b) 73 (a) 85 (a) 97 (c)
2 (e) 14 (e) 26 (d) 38 (e) 50 (c) 62 (a) 74 (c) 86 (b) 98 (a)
3 (a) 15 (d) 27 (d) 39 (e) 51 (b) 63 (e) 75 (d) 87 (d) 99 (c)
4 (e) 16 (a) 28 (a) 40 (d) 52 (a) 64 (a) 76 (a) 88 (b)
5 (e) 17 (b) 29 (c) 41 (c) 53 (c) 65 (b) 77 (b) 89 (b)
6 (d) 18 (e) 30 (b) 42 (b) 54 (a) 66 (d) 78 (c) 90 (b)
7 (b) 19 (a) 31 (c) 43 (a) 55 (d) 67 (c) 79 (c) 91 (b)
8 (c) 20 (b) 32 (d) 44 (c) 56 (b) 68 (c) 80 (d) 92 (d)
9 (d) 21 (c) 33 (d) 45 (c) 57 (e) 69 (b) 81 (e) 93 (b)
10 (e) 22 (d) 34 (a) 46 (a) 58 (c) 70 (e) 82 (a) 94 (c)
11 (c) 23 (e) 35 (d) 47 (d) 59 (b) 71 (a) 83 (e) 95 (b)
12 (a) 24 (e) 36 (c) 48 (e) 60 (e) 72 (e) 84 (b) 96 (c)
SERIES B-99

Answers &
Explanations
1. (d) Series is × 2 + 22, × 3 + 32, × 4 + 42, ... 37. (b) The series is × 0.25, × 0.5, × 0.75, × 1,...
2. (e) The series is ÷ 2 – 1 in each term. 38. (e) The series is ¸ 2, × l .5 ...
3. (a) The series is +5, +9, +13, +17 .... The difference in 39. (e) Ans = 2860. The series is × 1 + 4, × 2 + 8, × 3 + 12,...
successive nos. 9 – 5 = 13 – 9 = 17 – 13 = .... = 4. Hence, 40. (d) The series is × 2 + 1, × 3 – 2, × 4 + 3, × 5 – 4 ...
35 is wrong. It should be 37. 41. (c) The series is × 2 – 2, × 3 + 2, × 4 – 2, ...
4. (e) The sum of the first two nos. is the third no. Hence, 27 42. (b) The series is ¸ 2 – 2 in each steps.
is wrong. It should be 29. 43. (a) The series is +12, + 32, + 52, + 72 ......
5. (e) The series is × 0.5 + 0.5, × 1 + 1, × 1.5 + 1.5 .... Hence, 12 44. (c) The series is × 0.5 + 0.5, × 1 + 1, × 1.5 + 1.5, × 2 + 2...
is wrong. It should be 14. 45. (c) The series is × 1 + 1, × 2 + 2, × 3 + 3 ......
6. (d) The series is × 2 + 22, × 3 + 32, × 4 + 42, × 5 + 52 ..... 46. (a) The series is × 2 + 6, × 2 + 7, × 2 + 6 ......
Hence, 32 is wrong. It should be 33. 47. (d) The series is + (12) 2, – (10)2, + (8)2, – (6)2 ......
7. (b) The series is + l3, + 23, + 33, + 43 ..... Hence, 229 is 48. (e) Ans = 172. The series is × l + 12, × 2 + 22, × 3 + 32, ....
wrong. It should be 227. 49. (e) Ans = 97. The series is × 2 – 1 in each term.
8. (c) The series is + 5, + 7, + 9, + 11, ... 50. (c) Ans = 6. The series is × 0.5 + 0.5, × l + 1,× 1.5 + 1.5, × 2
9. (d) The series is ¸ 2, ¸ 1.5 alternately.. + 2, ....
10. (e) The series is an alternate series, having 51. (b) Ans = 30. The series is : 3 × 1+ 7 × 1= 10; 10 × 2 + 6 × 2
S1 = 2 5 14 41; × 3 – 1 in each term = 32; 32 × 3 + 5 × 3 = 111; 111 × 4 + 4 × 4 = 460; 460 × 5
S2 = 3 8 23 69: × 3 – 1 in each term + 3 × 5 = 2315 ......
11. (c) The differences are 1– 0 = 1 = 13 ; 9 – 1= 8 52. (a) Ans = 14. The series is ×2 – 2, ÷ 2 + 2, × 2 – 2, ÷ 2 + 2, ......
= 23; 36 – 9 = 27 = 33; 99 – 36 = 63 ¹ 43, 53. (c) The series is × 2 + 6 in each term.
but 100 – 36 = 64 = 43; 225 – 100 = 125 = 53 ; 54. (a) The series is × l + 12, × 2 + 22, × 3 + 32,.....
441– 225 = 216 = 63 55. (d) The series is ¸ 2 + 3.5, × 2 + 12, ¸ 2 + 2.5,× 2 + 8,....
12. (a) The series is × 2.5,× 2 alternately. 56. (b) The series is × 2 –1, × 3 – 2, × 4 – 3,.....
13. (c) The series is ×1 + 12, × 2 + 22, × 3 + 32, × 4 + 42.... 57. (e) The series is × 0.5, × 1.5, × 2.5,....
14. (e) The series is × 1 + 7, × 2 – 11, × 3 + 15.... 58. (c) The series is × l + 1, ÷ 2 – 2, × 3 + 3, ...
15. (d) The series is ×1 + 12, × 2 + 22, × 3 + 32, × 4 + 42.... 59. (b) The series is + 4.5, + 9, + 13.5,...
16. (a) The series is × 1.5 + 1.5, × 2 + 2, × 2.5 + 2.5, × 3 + 3 .... 60. (e) The series is × 1 + 7, × 2 – 6, × 3 + 5, × 4 – 4,...
17. (b) The series is × 2 + 3, × 2 + 5, × 2 + 7, × 2 + 9.... 61. (b) The series is × 2 + 2, ÷ 2 – 2, × 3 + 3, ÷ 3 – 3,...
18. (e) The series is × 1.5, × 2, × 2.5, × 3 and so on. 62. (a) The series is ×1.5, × 2, × 2.5, × 3...
19. (a) The series is – 16, – 8, – 4, – 2, – 1, 0.5, and so on. 63. (e) The series is × 2 + 1.5, × 2 + 2, × 2 + 2.5 ......
20. (b) The series is × 1 + 2, × 2 + 3, × 3 + 4, and so on. So, 108.5 should come in place of (c).
21. (c) The series is × 1 + 12, × 2 + 22, × 3 + 32 and so on. 64. (a) The series is × 12 + 1, × 22 + 2, × 32 + 3, × 42 + 4, ........
22. (d) The series is × 2 – 2 65. (b) The series is × 1 + 0.25 × 1, × 2 + 0.25 × 4, × 3 + 0.25
23. (e) The series is × l + 1, × 2 + 2, × 3 + 3, ... So 8 is wrong. × 9 ...... So 24.75 should come in place of (c).
Beginning with 8, we get 20 as third term. 66. (d) The series is × 0.5, × 1.5, × 2.5 ...... So 118.125 should
24. (e) The series is × l + 12, × 2 + 22, × 3 + 32 .... come in place of (d).
25. (b) The series is × 2 + 9, × 2 + 11, × 2 + 13 .... 67. (c) The series is + 112, – 92, +72, – 52, ..... So 112 should
26. (d) The series is × 1 – 1, × 2 + 2, × 2 – 2, × 3 + 3, .... come in place of (e).
27. (d) The series is × 3 + 1, × 4 + 1, × 5 + 1, ....
28. (a) The series is × l + l, × l . 5 + 1.5, × 2 + 2, × 2.5 + 2.5,... 3
68. (c) The series is ´6,´ alternately. So, 116.64 will
29. (c) The series is 2 + 5 = 7; 7 + 5 = 12; 12 + 7 = 19;... 10
30. (b) The series is × 6 + 1, × 5 + 2, × 4 + 3, × 3 + 4, ... come in place of A. 116.64 ¸ 36 = 3.24
31. (c) The series is 69. (b) The series is +20, + 30, + 40... So 182 will come in
3, 3 3
´ 1 + 1 ´ 2 + 2 , ´ 3 + 3 , ´ 4 + 4 , ... 3 182 ∗ 14
place of A. ? = < 14
32. (d) The series is × 7, × 6.5, × 6, × 5.5,... 14
33. (d) The series × 3 + 5,× 4 – 6, × 5 + 7,× 6 – 8.... 70. (e) The series is + 3, + 5 +7, +9 ... So 39 will come
34. (a) The series is + 12, + 22, + 32, + 42,... in place of A. ? = 392 – 4 = 1517
35. (d) The series is 7 + 9 = 16; 16 + 9 = 25; 71. (a) The series is + 6, +11, +16, +21 So 35 will come
25 + 16 = 41;41+25 = 66; 66 + 41= 107... 3 4
36. (c) The series is × 0.5, × 1.5, × 2.5,×3.5,... in place of A. ? = 35 × ´ < 12
7 5
B-100 SERIES
88. (b) The first letters of the triplets move 3, 4, 5, 6, 7 .... steps
3 81
72. (e) The series is × 2 and × alternately. So will come forward.
4 64 The second letters of the triplets move 5, 6, 7, 8, 9 ...
steps forward.
81 9
in place of A. ? = < The third letters of the triplets move 7, 8, 9, 10 .... steps
64 8 forward.
73. (a) The series is × 1 + 4, × 2 + 8, × 3 + 12 ... Hence, the next triplet of alphabets is ZKW.
74. (c) The series is × l + 1, × 2 + 2, × 3 + 3,... 89. (b) baab/baab/baab
75. (d) The series is × 1.5, × 3, × 4.5, ... 90. (b) cabbac/cabbac/cabbac.
76. (a) The series is × 2 + 4, × 2 + 8, × 2 + 12. .. 91. (b) Differences between two consecutive terms are 2, 3, 4,
77. (b) The series is: × 8 – 28, × 7 – 24, ×6 – 20 , ... 5 and 6 respectively.
78. (c) The series is × 1 + 2, × 2 + 2, × 3 + 2, × 4 + 2, × 5 + 2, × 92. (d) The sequence followed in the series is ÷ 5, –5 which is
6 + 2. 914 is incorrect. It should be 772. The new series repeated.
begins with 914. 900 180 175 35 30 6 1
79. (c) The series is × 1 + 1, × 2 + 2, × 3 + 3, × 4 + 4, × 5 + 5, ×
6 + 6. 34 should be 33 and thus the new series starts ¸5 –5 ¸5 –5 ¸5 –5
with 34. \ 900 should be in place of 905.
80. (d) The series is – (14)2, + (12)2, – (10)2, + (8)2, – (6)2 and 93. (b) The numerical parts moves with a difference of + 4.
so on. Thus, the correct sequence of the numerical
components would be 1, 5, 9, 13, 17 Therefore, 15 LS
1 3 5 7 9 11 does not fit in the series.
81. (e) The series is ´ , ´ , ´ , ´ , ´ , ´ and so
2 2 2 2 2 2 94. (c) There are two series :
on. I. B0R E3P H9N
82. (a) The series is First letter moves + 3 steps forward. The middle
+ 12 – 0, + 22 – 1, + 32 – 2, + 42 – 3, + 52 – 4, + 62 – 5. numerical component moves + 3, + 6, + 9 ........... and
Thus, 7 is the wrong number. the letter in the third position moves 2 steps backwards
(– 2).
83. (e) Observe the first letter of each group. These letters are II. G3U, J7S
alternate letters starting from A. Hence, the first letter The same pattern follows in this series.
of the missing group will be G. Similarly, observe second Hence, J7S does not fit.
and third letters of each group. Hence, the required 95. (b) C is the 3rd letter, F sixth, I ninth so next letter will be
group is GWH. 12th, i.e. L.
84. (b) A TTRIBUTIO N Þ TTR BUTIO The middle numerics are the squares of 2, 3, 4 and so
on. So next numeric would be 25.
T T RIBUTIO Þ RIBUTIO The last letter follow the order : U is 3rd letter after R, X
is 3rd after U. So, R would be 3rd letter after ‘O’.
R IBUTI O Þ IBUTI \ Missing term = L25O.
96. (c) The series is abc/aabc/aabbc/aabbcc/a.
I B UTI Þ UTI
97. (c) Clearly, the first, second and third letters of each term
85. (a) SEQUENTIAL, (S)EQUENTIAL, EQUENTI(A)(L), are respectively moved one, two and three steps
(E)QUENTI QUEN(T)(I) forward to obtain the corresponding letters of the next
In the given sequence one letter and two letters are term. So, the next term is EJO.
lost alternately. 98. (a) The first letter of each term is moved one step forward,
86. (b) First letters of each group are in the order + 5, + 6, +7 the second letter is moved four steps backward and
and so on, whereas the last letters of each group are in the third letter is moved four steps forward to obtain
the order + 6, + 7, + 8 and so on. the corresponding letters of the next term.
87. (d) The first letters in odd numbered terms form series J, I, 99. (c) The series is babb/bbab/bbba/bbbb.
H and in even numbered terms form the series K, L, M. Thus, in each sequence, ‘a’ moves one step forward
The sequence followed by the numbers is + 2, + 3, + 4, and ‘b’ takes its place and finally in the fourth
+ 5, + 6. The third letter of each term is moved two sequence, it is eliminated.
steps backward to obtain the third letter of the next
term.
Section C : TEST OF ENGLISH LANGUAGE

Vocabulary
1 Chapter

WORD LIST · Brittle : Frail, fragile


· Brutal : Animal, savage, beastly, cruel
Given below is a list of words placed in alphabetical order. Each · Burglar : Thief, bandit, brigand, stealer
word is followed by a few of its synonyms. Note these words · Bystander : Spectator, onlooker, beholder
whenever you come across them. You should be familiar with · Calculate : Estimate, count, reckon, compute
most of the words for which synonyms are given if you have done · Callous : Hard, indifferent, cold-blooded
all the exercises till this point thoroughly. So, this list will be · Calm : Cool, confident, quiet, serene, tranquil
giving you synonyms for the words which you know. Thus · Cancel : Annual, withdraw, revoke, delete
learning will be easier. · Candid : Sincere, straightforward. frank
· Abandon : Leave, desert, forsake · Captive : Prisoner, confined, jailed, bonded
· Abase : Degrade, disgrace, humiliate · Cause : Make, originate, induce, generate,
· Abhor : Hate, loathe, detest create
· Abridge : Shorten, abbreviate · Censor : Cut off, prohibit, ban
· Absolute : Unalterable, unrestricted, unconditional · Censure : Blame, condemn, reprove, reprimand
· Absurd : Ridiculous, silly, foolish · Character : Letter, emblem, type, OR nature,
· Abundant : Ample, plentiful disposition, quality
· Accessory : Additional, auxiliary, subsidiary · Charity : Philanthropy, benevolence
· Adept : Proficient, skilled, expert · Chaste : Pure, immaculate, virgin, refined
· Adherent : Follower, stickler · Chatter : Babble, ramble, talk, discourse
· Adhesive : Sticky, glue, gum · Cheat : Defraud, gull, outwit, dupe
· Admire : Praise, adore, esteem · Cite : Quote, mention, name, adduce
· Adore : Respect, idolise, worship, admire · Clothes : Apparel, attire, dress, garb
· Adversity : Misery, misfortune · Colossal : Huge, gigantic, enormous, big
· Affliction : Distress, sorrow, sadness · Commence : Begin, start
· Alien : Foreign, stranger, unknown · Commensurate : Equivalent, suitable, applicable,
· Alive : Lively, vivacious, living proportionate
· Alleviate : Relieve, lighten, ease · Conceal : Hide, cover, shelter, disguise
· Alms : Gratuity, donation, grant · Confess : Admit, acknowledge, reveal, agree
· Amend : Improve, change, emend · Confuse or
· Amicable : Suitable, friendly, lovable, amiable confound : Mix, perplex, astonish, Amaze, bewilder
· Anxiety : Eagerness, misgiving, worry · Consequent : Following, resultant, outcome
· Apathy : Indifference, neutrality · Conspiracy : Plot, intrigue, treason
· Appalling : Terrific, terrifying, dreadful, horrible · Convict : Felon, culprit, criminal, guilty
· Apposite : Apt, suitable, well chosen
· Cowardly : Craven, dastardly, fearful, poltroon
· Appraise : Evaluate, estimate
· Coy : Modest, shy, reserved
· Apprehend : Seize, fear, arrest
· Crafty : Artful, adroit, dexetrous, cunning,
· Arbitrary : Despotic, wayward
deceitful
· Assent : Agree, consent, acquiesce
· Crazy : Mad, insane, silly
· Astonish : Astound, surprise, amaze, bewilder
· Credence : Belief, faith, trust, confidence
· Audacious : Bold, courageous, daring
· Crisis : Turning point, emergency, decisive
· Aversion : Dislike, detestation, hostility, hatred
moment
· Base : Mean, low, ignoble
· Criterion : Test, touchstone, standard, yardstick
· Beg : Implore, ask, beseech, solicit
· Criticism : Analysis, review, stricture
· Behaviour : Conduct, deportment, way, demeanour
· Cruel : Brutal, unmerciful, beastly, savage
· Brave : Courageous, intrepid, bold, daring, · Cynical : Captious, incredulous, sarcastic,
valiant morose
· Brisk : Active, fast, quick, busy, alert · Danger : Hazard, risk, peril
C-2 VOCABULARY
· Dash : Run, rush, fly · Diffident : Hesitating, doubtful, distrusting
· Dastardly : Cowardly, invaliant, afraid, fearful · Digression : Excursion, deviation, misguidance
· Dawn : Daybreak, appear, (sunrise), begin · Diligence : Care, industry, effort
· Deadly : Fatal, lethal, destructive · Dire : Terrible, awful, horrible; miserable
· Dearth : Scarcity, lack, want · Disapprove : Condemn, reject, disallow
· Debase : Degrade, defame, disparage, · Disavow : Deny, refuse
humiliate · Disciple : Follower, learner, student
· Decay : Decompose, rot, decline in power, · Disclose : Reveal, tell, uncover, divulge
wealth, waste, wither, fade · Disconsolate : Sad, cheerless, miserable
· Decease : Death, demise, end · Discredit : Disbelieve, doubt, disgrace
· Deceit : Fraud, cheating, forgery · Disgust : Abhorrence, dislike, detestation
· Decipher : Translate, interpret, solve, explain · Dismay : Disappointment,
· Decorum : Decency, etiquette, propriety, gravity discouragement
· Decree : Law, edict, ordinance, mandate, · Disorder : Disease, illness, OR untidiness,
judgement uncleanliness
· Defamation : Calumny, disparagement, debasement · Disown : Disclaim, deny, renounce
· Defection : Abandonment, desertion · Disparage : Debase, decry, defame
· Defer : Postpone, delay · Dispose : Adjust, arrange, incline
· Deference : Respect, reverence, honour · Dispute : Argument, controversy, altercation
· Deformity : Disfigurement, · Disregard : Neglect, overlook,
malformation, ugliness disrespect
· Dejected : Depressed, distressed, downhearted, · Dissolute : Corrupt, mean, lax, licentious
downcast · Distaste : Abhorrence, dislike, detestation
· Delectable : Charming, delightful, · Distorted : Blurred, maligned, changed, disguised,
pleasant deformed, misrepresented
· Delegate : Commission, depute,
· Distress : Affliction, depression, misery
authorise
· Divert : Turn aside, deflect, deviate
· Deliberate : Knowingly done,
· Divine : Heavenly, metaphysical, godlike
intentional, forcible
· Divulge : Reveal, uncover, disclose
· Delicacy : Softness, nicety,
· Docile : Amenable, tractable, submissive
slenderness, refinement, purity
· Doctrine : Precept, principle, teaching
· Delusion : Illusion, fancy, error, false belief
· Dogmatic : Categorical, authoritative, firm, preachy
· Demeanour : Behaviour, conduct, bearing
· Dolt : Blockhead, stupid, fool, idiot, dullard
· Demise : Death, decease
· Demolish : Break, destroy, annihilate · Domicile : Dwelling, home, residence
· Demure : Modest, coy, humane · Dotage : Senility, imebcility
· Denomination : Name, appellation, designation · Downright : Simple, unquestionable, blunt, frank
· Denounce : Accuse, malign, criticise, defame, · Dread : Apprehend, fear
condemn · Drench : Soak, wet
· Deny : Contradict, refuse, disavow, withhold · Drowsy : Sleepy, comatose, lazy, lethargic
· Deride : Ridicule, mock, taunt · Dubious : Suspicious, doubtful, unreliable
· Descant : Discourse, expatiate, enlarge · Ductile : Pliant, yielding, flexible
· Desire : Wish, long for, crave, covet · Dupe : Cheat, befool, steal
· Desolate : Lonely, deserted, solitary, devastated · Dwindle : Shrink, diminish, decrease
· Despise : Condemn, dislike · Earnest : Eager, ardent, intent,anxious, sincere
· Despondency : Despair, dejection, · Eccentric : Irregular, anomalous, abnormal, odd
hopelessness · Economise : Save, retrench
· Despotic : Arbitrary, tyrannical, illegal · Ecstasy : Trance, enchantment, rapture
· Destitute : Needy, poor, miserable, indigent · Efface : Blot out, obliterate, destroy
· Destruction : Ruin, demolition, ravage · Effeminate : Womanly, weak, unmanly
· Detain : Lock in, arrest, hold, custody · Efficacy : Energy, virtue, potence
· Detest : Despise, abhor, dislike · Egotistic : Self-centered, egoist, self-conceited
· Dethrone : Depose, remove (from office) · Egregious : Conspicuously bad, sinful, monstrous,
· Devastate : Ruin, demolition, ravage shocking
· Devoid : Lacking, empty, vacant · Elaborate : Explain, discuss, elucidate
· Devout : Religious, reverent · Elevated : Elated, promoted, upgraded, risen
· Dexterity : Adroitness, cleverness, skill · Eliminate : Remove, replace, dismiss, discard
· Diabolical : Fiendish, devilish, wicked · Eloquence : Oratory, rhetoric, finery (of speech)
· Diatribe : Tirade, denunciation fluency of expression
· Dictatorial : Tyrannical, arbitrary, despotic · Emanate : Originate, proceed, spring, issue
VOCABULARY C-3
· Emancipate : Free, deliver, liberate · Fastidious : Particular, over-nice, squeamish
· Embarrass : Vex, confuse, entangle · Fatal : Deadly, lethal, mortal
· Embezzle : Steal, peculate, cheat · Fatigue : Weakness, exhaustion, tiredness
· Embody : Incorporate, include, comprise · Feeble : Weak, frail, dim
· Emolument : Salary, wage, remuneration · Felicitate : Congratulate, compliment
· Emulate : Compete, rival, vie against, copy · Felicity : Joy, happiness, good luck
· Enchant : Charm, bewitch, hypnotise · Felon : Criminal, sinner, guilty, bandit
· Encompass : Surround, encircle · Ferment : Excite, agitate
· Encounter : Come across, combat, fight · Ferocity : Fierceness, vehemence, fanaticism
· Encroach : Trespass, intrude, invade · Fervent : Glowing, heated, impassioned
· Endeavour : Attempt, effort, aspiration
· Fervour : Warmth, glow, vehemence
· Endorse : Back, approve, ratify
· Festivity : Gaiety, merry-making
· Endurance : Patience, continuance, fortitude
· Fetter : Shackle, bind, imprison
· Enfranchise : Liberate, free, (also: give right to vote)
· Enlighten : Illuminate, edify, elaborate · Feud : Dispute, broil, strife
· Enmity : Hostility, hatred, animosity · Fickle : Changeable, vacillating, varying
· Enormous : Big, huge, colossal, gigantic · Fiendish : Devilish, diabolical, malignant
· Enrage : Infuriate, madden, incense, irritate · Figurative : Typical, imaginative, emblematic,
· Ensue : Succeed, follow, result metaphorical
· Entangle : Ravel, involve, perplex · Filthy : Dirty, foul, nasty
· Enterprise : Undertaking, venture, endeavour · Fissure : Crevice, rift, narrow opening
· Enthusiasm : Zeal, ardour, interest · Flaccid : Soft, loose, weak
· Entice : Allure, tempt, seduce, attract · Flatter : Adore, please, praise
· Entreat : Beseech, implore, beg · Fleece : Rob, despoil, cheat
· Entwine : Encircle, surround, encompass · Flexible : Variable, pliable, changeable
· Enumerate : Count, number one by one · Flimsy : Transparent, thin, trivial, tenuous
· Enunciate : Declare, publish, propound, reveal · Flounder : Stumble, falter, wallow, struggle
· Envoy : Legate, messenger, ambassador · Fluctuate : Undulate, waver, vacillate
· Epoch : Era, time, age · Flutter : Flip, quiver, ruffle, agitate
· Equivocal : Doubtful, Ambiguous, uncertain · Folly : Absurdity, silliness, imprudence,
· Eradicate : Root out, extirpate, annihilate foolishness
· Erroneous : Wrong, false · Fondle : Caress, touch, rub
· Erudite : Learned, scholarly, lettered · Foray : Incursion, inroad, venture
· Esteem : Admire, appreciate, adore, respect · Forbearance : Abstaining, refraining
· Eulogy : Laudation, praise, extolling, felicitation
· Forbid : Prohibit, disallow, debar
· Evidence : Testimony, proof, witness
· Forebode : Betoken, indicate, augur
· Evince : Show, manifest, demonstrate
· Forlorn : Disconsolate, cheerless, distressed,
· Exact : Extort, oppress, loot
· Exaggerate : Amplify, overstate abandoned, lonely
· Excerpt : Extract, quotations · Formidable : Dreadful, difficult, hard to overcome
· Exile : Expulsion, banishment, expatriation · Fragile : Weak, feeble, slender, delicate
· Exonerate : Acquit, absolve, release · Fragrant : Odorous, balmy, soothing
· Exorbitant : Excessive, too much, very high · Frailty : Weakness, delicacy,
· Extinguish : Quench, terminate, destroy, put out fragileness
· Extravagant : Excessive, lavish, stylish · Frantic : Frenzied, mad, distracted
· Exuberant : Abundant, plentiful · Fraudulent : Dishonest, cheating, deceitful
· Exult : Triumph, rejoice, delight · Fray : Battlefield, combat, brawl
· Fable : Story, legend, myth, fiction · Frisk : Skip, dance, caper, frolic
· Fabricate : Construct, forge, invent · Frivolous : Vain, foolish, trivial
· Fabulous : Fictitious, mythical, exaggerated · Frugal : Economical, thrifty
· Facile : Fluent, ready, glib (of writing), pliable, · Futile : Useless, hopeless, in vain
docile, tractable · Gaiety : Hilarity, jollity, festivity
· Faction : Clique, cabal, discord, section · Gainsay : Contradict, dispute, controvert, deny
· Fallacy : Deception, illusion, mistake · Gallantry : Courage, bravery, heroism
· Falter : Waver, hesitate, delay, flounder · Garbage : Filth, waste, useless, throwaway, trash
· Famine : Hunger, starvation, scarcity of food · Garner : Accumulate, collect, gather
· Fanatical : Bigoted, enthusiastic · Garrulous : Prattling, chattering
· Fancy : Liking, conception, craving, whim · Gawky : Awkward, clumsy
· Farcical : Droll, comic, extravagant · Gay : Happy, merry, joyous
· Fascinate : Charm, bewitch, attract · Generous : Noble, magnanimous, kind, liberal
C-4 VOCABULARY
· Genteel : Well-bred, well-cultured, polite, refined · Immortal : Everlasting, undying, endless
· Genuine : True, authentic, real · Impassioned : Fervent, frenzied, fanatical
· Ghastly : Horrible, horrifying, horrific · Impeachment : Imputation, accusation
· Ghost : Phantom, spirit, spectre · Implicit : Implied, assumed, inferred
· Glimmer : Shine, flash, gleam · Impostor : Cheat, conman, charlatan
· Glimpse : Glance, (quick) look, (brief) view · Impracticable : Impossible, (merely) theoretical
· Glisten : Shine, beam, glow · Impressive : Striking, affecting, extraordinary
· Gloom : Depression, melancholy, loneliness · Impunity : Exemption (from punishment)
· Glutinous : Sticky, viscous · Inane : Empty, silly, idiotic
· Gluttonous : Greedy, gorging, voracious · Inarticulate : Voiceless, indistinct, inexpressive
· Grandeur : Splendour, magnificence, glory · Incense : Infuriate, enrage, anger
· Grapple : Grasp, clutch, seize · Incessant : Unceasing, continual
· Gratification : Satisfaction, enjoyment · Incognito : (Identity) Concealed, secretly, stealthily
· Gravity : Seriousness, importance, calmness · Inculcate : Instil, enforce, pass on, generate
· Greed : Avarice, ravenous, envious, covetous · Inculpate : Blame, incriminate
· Grievance : Hardship, complaint, trouble · Incumbent : Compulsory, obligatory, binding
· Grotesque : Horrifying, contorted, bizzare, whimsical · Incursion : Inroad, foray, venture
· Grudge : Grievance, begrudge, resent · Indefatigable : Tireless, assiduous
· Gullible : Simple, easy, pliable, credulous · Indict : Accuse, charge
· Hallucination : Delusion, illusion, nightmare · Indiscriminate : Mixed, undistinguished, confused,
· Hamper : Impede, block, fetter, bind wanton
· Hapless : Unfortunate, unlucky · Indolence : Apathy, inactivity, laziness,
· Harangue : a lengthy speech, oration sluggishness, lethargy
· Hardship : Trouble, adversity, difficulty · Industrious : Busy, active, tireless
· Haughty : Arrogant, overbearing, imperious · Inevitable : Unavoidable, certain, sure
· Havoc : Devastation, destruction, ruin · Inexorable : Relentless, indefatigable
· Hearty : Sincere, warm, ardent · Infallible : Unfailing, unerring, certain
· Heave : Raise, lift · Infectious : Catching, contaminating, corrupting,
· Hedge : Fence, hem transmittable
· Heed : Advise, note, consider, mind · Infer : Gather, conclude, deduce
· Heinous : Atrocious, odious, wicked · Influence : Authority, effect, power
· Hem : Border, edge, fringe, outskirt · Infringe : Break, violate, transgress, encroach
· Herculean : Colossal, laborious, excessive · Ingredient : Constituent, component, element
· Heterogeneous : Dissimilar, unlike, different, diverse, · Inherent : Inborn, innate, built-in
varied · Inhibition : Restraint, check
· Hideous : Terrific, horrible, filthy · Iniquitous : Unjust, wrong, unfair
· Hilarious : Exceedingly, funny, boisterously merry, · Initiate : Start, begin, inchoate
amusing, joyous · Innocuous : Harmless, mild, innocent
· Hindrance : Impediment, hampering, obstruction, · Insanity : Madness, lunacy, mania
obstacle · Insidious : Deceitful, treacherous
· Histrionic : Theatrical, dramatic · Instantaneous : Immediate, Sudden, quick
· Hoax or · Instigate : Arouse, misguide, provoke
Humbug : Dupe, cheat, befool · Intact : Untouched, unscathed, whole,
· Homage : Deference, salute, worshipping unbroken, undamaged, unimpaired,
· Horizon : Verge, limit, skyline entire
· Hostile : Adverse, opposing, inimical · Integrity : Oneness, entirety, completeness,
· Hover : remain in one place in the air, remain honesty, wholeness, soundness
suspended, linger · Intellectual : Intelligent, rational, learned
· Humane : Compassionate, caring, bvenevolent · Intentional : Deliberate, intended,
· Humiliate : Abase, insult, mock, defy · Intercourse : Intimacy, association, communication
· Humorous : Amusing, laughable, comical · Intermission : Suspension, stoppage, pause,
· Hypocrisy : Pretence, imposture, deceit cessation, interval
· Idolise : Adore, worship, admire · Intervene : Interpose, mediate
· Illegal or Illicit : Unlawful, prohibited · Intimate : Close, tender, friendly or informal,
· Illogical : Inconsequent, irrational advertise, inform
· Imitate : Copy, follow, mimic, emulate · Intolerable : Unendurable, unbearable
· Immaculate : Spotless, stainless, perfect · Intransigent : Uncompromising, inflexible
· Immature : Crude, childish, unseasoned · Intrepid : Brave, fearless
· Imminent : Impending, near, due, threatening · Intricate : Complex, difficult, complicated
· Immodest : Indecent, indelicate, unchaste · Intuition : Insight, premonition, instinct
VOCABULARY C-5
· Inveterate : Habitual, deep-rooted · Noxious : Hurtful.
· Involuntary : Compulsory, unwilled, reflex · Nugatory : Having no power or force.
· Irksome : Annoying, disagreeable, tedious · Obdurate : Impassive to feelings of humanity or
· Irresolute : Wavering, confused, vacillating pity.
· Itinerant : Travelling (on a circuit), wandering, · Obfuscate : To darken; to obscure.
nomadic · Oblique : Slanting; said of lines.
· Jaded : Tired, weary · Obstreperous : Boisterous.
· Jargon : cant, technical language, gibberish · Odious : Hateful.
· Jocose : Jocular, humorous · Odium : A feeling of extreme repugnance, or of
· Jocular : Inclined to joke. dislike and disgust.
· Jovial : Merry. · Ominous : Portentous.
· Judicious : Prudent. · Onerous : Burdensome or oppressive.
· Juxtaposition : Closeness, nearness · Onus : A burden or responsibility.
· Kiosk : Stand (open on one side), booth · Palate : The roof of the mouth.
· Knead : Work up (into dough), press, squeeze · Palatial : Magnificent.
· Lacerate : Tear (tissue toughly), mangle · Palliate : To cause to appear less guilty.
· Lachrymose : Given to shedding tears. · Palpable : Perceptible by feeling or touch.
· Lackadaisical : Listless. · Panoply : A full set of armor.
· Laconic : (Briskly) Short, concise, pithy · Paragon : A model of excellence.
· Languid : Listless, spiritless · Pariah : A member of a degraded class; a social
· Languish : Die (with hunger or desire), weaken, outcast.
droop · Paroxysm : A sudden outburst (of any kind of
· Lascivious : Lustful. activity).
· Lassitude : Weariness, tiredness · Paucity : Fewness.
· Latent : Hidden, concealed · Pellucid : Translucent.
· Laudable : Praiseworthy. · Penchant : A bias in favor of something.
· Legacy : A bequest. · Penurious : Excessively sparing in the use of
· Legitimate : Genuine, lawful money.
· Levity : Frivolity. · Penury : Indigence.
· Libel : Defamation. · Peremptory : Precluding question or appeal.
· Licentious : Wanton. · Perfidy : Treachery.
· Lithe : Supple. · Perfunctory : Half-hearted.
· Loquacious : Talkative. · Peripatetic : Walking about.
· Lustrous : Shining. · Perjury : A solemn assertion of a falsity.
· Malaise : A condition of uneasiness or ill-being. · Permeate : To pervade.
· Malevolence : Ill will. · Pernicious : Tending to kill or hurt.
· Malleable : Pliant. · Persiflage : Banter.
· Mawkish : Sickening or insipid. · Perspicacity : Acuteness or discernment.
· Mellifluous : Sweetly or smoothly flowing. · Perturbation : Mental excitement or confusion.
· Mendacious : Untrue. · Petulant : Displaying impatience.
· Mendicant : A beggar. · Phlegmatic : Not easily roused to feeling or action.
· Mesmerize : To hypnotize. · Pique : To excite a slight degree of anger in.
· Meticulous : Over-cautious. · Plea : An argument (to obtain some desired
· Mettle : Courage. action).
· Mien : The external appearance or manner of a · Plenary : Entire.
person. · Plethora : Excess; superabundance.
· Moderation : Temperance. · Poignant : Severely painful or acute to the spirit.
· Modicum : A small or token amount. · Polyglot : Speaking several tongues.
· Mollify : To soothe. · Ponderous : Unusually weighty or forcible.
· Mordant : Biting. · Portent : Anything that indicates what is to
· Moribund : On the point of dying. happen.
· Morose : Gloomy. · Pragmatic : Practical(values), empirical
· Multifarious : Having great diversity or variety. · Precarious : Critical, dangerous
· Mundane : Worldly, as opposed to spiritual or · Preclude : To prevent.
celestial. · Precocious : Advan ced (in development),
· Munificent : Extraordinarily generous. overforward, premature
· Myriad : A vast indefinite number. · Predilection : Preference, partiality, inclination
· Nadir : The lowest point. · Predominate : To be chief in importance, quantity, or
· Nefarious : Wicked in the extreme. degree.
· Negligent : Apt to omit what ought to be done. · Preposterous : (Very) Absurd, ridiculous
· Neophyte : Having the character of a beginner. · Prerogative : (Special) Right, privilege
C-6 VOCABULARY
· Presage : To foretell. · Regale : To give unusual pleasure.
· Prescience : Knowledge of events before they take · Regicide : The killing of a king or sovereign.
place. · Reiterate : To say or do again and again.
· Preternatural : Extraordinary. · Relapse : To suffer a return of a disease after
· Prim : Stiffly proper. partial recovery.
· Pristine : Primitive. · Relegate : Assign a lower position, banish, demote
· Probity : Virtue or integrity tested and confirmed. · Repast : A meal; figuratively, any refreshment.
· Proclivity : A natural inclination. · Repine : To indulge in fretfulness and
· Procrastination : Delay. faultfinding.
· Prodigious : Large, immense · Reprisal : (Injury in) Return, retaliation, revenge
· Profligate : Immoral, wanton, reckless, dissolute, · Reprobate : One abandoned to depravity and sin.
licentious · Repudiate : Disavow, disclaim
· Profuse : Produced or displayed in overabundance. · Resilience : The power of springing back to a former
· Prolix : Verbose. position
· Promiscuous : Indiscriminate, impure, casual · Resonance : Able to reinforce sound by sympathetic
· Propinquity : Nearness, proximity vibrations.
· Propitious : Kindly disposed. · Respite : Interval of rest.
· Prosaic : Unimaginative.
· Restive : Resisting control.
· Protagonist : Leading character), hero(ine)
· Revere : To regard with worshipful veneration.
· Providential : Fortunate, lucky
· Reverent : Humble.
· Prudence : Caution.
· Rotund : Round from fullness or plumpness.
· Puerile : Childish.
· Pugnacious : Quarrelsome. · Ruffian : A lawless or recklessly brutal fellow.
· Puissant : Powerful, influential, mighty · Ruminate : To chew over again, as food previously
· Punctilious : Exact (in formalities), ceremonious, swallowed and regurgitated.
conscientious · Sagacious : Wise, shrewd, astute
· Pungency : The quality of affecting the sense of · Salacious : Obscene, foul, indecent, lecherous
smell. · Salubrious : Healthful; promoting health.
· Pusillanimous : Without spirit or bravery. · Salutary : (Morally) Healthy, salubrious,
· Putrefy : Decay, rot, decompose beneficial
· Pyre : A heap of combustibles arranged for · Sanguine : Ardent, confident, optimistic
burning a dead body. · Sardonic : Ironical, scornful, derisive
· Qualm : A fit of nausea. · Satiate : Gratify (fully), surfeit, saturate
· Quandary : Doubt, dilemma, plight · Satyr : A very lascivious person.
· Quibble : An utterly trivial distinction or · Savor : To perceive by taste or smell.
objection. · Schism : Disjunction, split
· Quiescence : Being quiet, still, or at rest; inactive · Scribble : Hasty, careless writing.
· Quirk : Twist, quibble, deviation · Sedition : Plotting (against government),
· Quixotic : (Foolishly) Chivalrous, unrealistic, incitement, insurgence
whimsical · Sedulous : Persevering in effort or endeavor.
· Rabble : Th rong (of th e vulgar ), crowd, · Severance : Separation.
proletariat · Sinecure : Any position (having emoluments with
· Rabid : Furious, mad, fanatical few or no duties).
· Raconteur : A person skilled in telling stories. · Sinuous : Curving in and out.
· Raillery : Jesting (language), banter, ridicule · Sluggard : A person habitually lazy or idle.
· Ramify : To divide or subdivide into branches · Solace : Comfort in grief, trouble, or calamity.
or subdivisions. · Solvent : Having sufficient funds to pay all debts.
· Rampant : Rife, widespread · Somniferous : Tending to produce sleep.
· Ramshackle : Dilapidated, tumbledown, rickety · Somnolent : Sleepy.
· Rapacious : Sieze by force, avaricious · Soporific : Causing sleep; also, something that
· Raucous : Harsh. causes sleep.
· Recalcitrant : Stubborn, refractory · Sordid : Filthy, morally degraded
· Recluse : One who lives in r etirement or · Specious : Plausible.
seclusion. · Spurious : Not genuine.
· Recondite : Incomprehensible to one of ordinary · Squalid : Having a dirty, mean, poverty-stricken
understanding. appearance.
· Recuperate : To recover. · Stanch : To stop the flowing of; to check.
· Redoubtable : Formidable. · Stingy : Cheap, unwilling to spend money.
· Redundant : Wordy, repetitious, super fluous, · Stolid : Expressing no power of feeling or
needless perceiving.
· Refractory : Not amenable to control. · Submerge : To place or plunge under water.
VOCABULARY C-7
· Subterfuge : Evasion. · Veracious : Habitually disposed to speak the truth.
· Succinct : Concise. · Veracity : Truthfulness.
· Sumptuous : Rich and costly. · Verbiage : Use of many words without necessity.
· Supercilious : Exhibiting haughty and careless · Verbose : Wordy.
contempt. · Verdant : Green with vegetation.
· Superfluous : Being more than is needed. · Veritable : Real; true; genuine.
· Supernumerary: Superfluous. · Vestige : (A visible) trace, mark, or impression
· Supersede : To displace. (of something absent, lost, or gone).
· Supine : Lying on the back. · Virago : Loud talkative women, strong statured
· Supplicate : To beg. women
· Suppress : To prevent from being disclosed or · Virtu : Rare, curious, or beautiful quality.
punished. · Visage : The face, countenance, or look of a
· Surcharge : An additional amount charged. person.
· Surfeit : To feed to fullness or to satiety. · Vitiate : To contaminate.
· Susceptibility : A specific capability of feeling or · Vituperate : To overwhelm with wordy abuse.
emotion. · Vivify : To endue with life.
· Taciturn : Disinclined to conversation. · Vociferous : Making a loud outcry.
· Taut : Stretched tight. · Volatile : Changeable.
· Temerity : Foolhardy disregard of danger;
· Voluble : Having great fluency in speaking.
recklessness.
· Whimsical : Capricious.
· Terse : Pithy.
· Winsome : Attractive.
· Timorous : Lacking courage.
· Torpid : Dull; sluggish; inactive.
· Torrid : Excessively hot. WORD USAGE
· Tortuous : Abounding in irregular bends or turns. Fill in the blanks or sentence completion:
· Tractable : Easily led or controlled.
· Transgress : To break a law. Sentence completion questions test your ability to use your
· Transitory : Existing for a short time only. vocabulary and recognise logical consistency among the elements
· Travail : Hard or agonizing labor. in a sentence. You need to know more than the dictionary
· Travesty : A grotesque imitation. definitions of the words involved. You need to know how the
· Trenchant : Cutting deeply and quickly. words fit together to make logical and stylistic sense.
· Trepidation : Nervous uncertainty of feeling. Sentence completion questions actually measure one part of
· Trite : Made commonplace by frequent reading comprehension. If you can recognise how the different
repetition. parts of a sentence affect one another, you should do well at
· Truculence : Ferocity. choosing the answer that best completes the meaning of the
· Truculent : Having the character or the spirit of a sentence or provides a clear, logical statement of fact. The ability
savage. to recognise irony and humour will also stand you in good stead,
· Turbid : In a state of turmoil; muddled as will the ability to recognise figurative language and to
· Turgid : Swollen. distinguish between formal and informal levels of speech.
· Turpitude : Depravity.
Because the sentence completion questions contain many clues
· Ubiquitous : Being present everywhere.
that help you to answer them correctly (far more clues than the
· Umbrage : A sense of injury.
antonyms provide, for example), and because analysing them
· Unctuous : Oily.
helps you warm up for the reading passages later on in the test,
· Undulate : To move like a wave or in waves.
· Untoward : Causing annoyance or hindrance. on the paper-and-pencil test, answer them first. Then go on to
· Upbraid : To reproach as deserving blame. tackle the analogies, the antonyms, and, finally, the time-
· Vagary : A sudden desire or action consuming reading comprehension section.
· Vainglory : Excessive, pretentious, and Sentence completion questions may come from any of a number
demonstrative vanity. of different fields-art, literature, history, philosophy, botany,
· Valorous : Courageous. astronomy, geology, and so on. You cannot predict what subject
· Vapid : Having lost sparkling quality and matter the sentences on your test will involve.
flavor. What makes the hard questions hard?
· Variegated : Having marks or patches of different 1. Vocabulary Level: Sentences contain words like
colors; also, varied. intransigence, nonplussed, harbingers. Answer choices
· Vehement : Very eager or urgent. include words like penchant, abeyance, and eclectic.
· Venal : Mercenary, corrupt. 2. Grammatical Complexity . Sentences combine the entire
· Veneer : Outside show or elegance. range of grammatical possibilities : adverbial clauses,
· Venial : That may be pardoned or forgiven, a relative clauses, prepositional phrases, gerunds, infinitives,
forgivable sin. and so on in convoluted ways. The more complex the
C-8 VOCABULARY
sentence, the more difficult it is for you to spot the key See how this Funda helps you deal with another question.
words that can unlock its meaning. 1. The evil of class and race hatred must be eliminated while it
3. Tone. Sentences reflect the writer’s attitude towards the is still in an .......................state; otherwise it may grow to
subject matter. It is simple to comprehend material that is dangerous proportions.
presented neutrally. It is far more difficult to comprehend (A) Amorphous (B) overt
material that is ironic, condescending, playful, sombre, or (C) uncultivated (D) embryonic
otherwise complex in tone. (E) independent
4. Style. Ideas may be expressed in different manners ornately On the basis of a loose sense of this sentence’s meaning, you
or sparely, poetically or prosaically, formally or informally, might be tempted to select Choice A. After all, this sentence
journalistically or academically, originally or imitatively. basically tells you that you should wipe out hatred before it gets
An author’s style depends on such details as word choice, too dangerous. Clearly, if hatred is vague or amorphous, it is less
imagery, repetition, rhythm, sentence structure and length. formidable than if it is well defined. However, this reading of the
Work through the following Fundas and learn techniques sentence is inadequate: it fails to take into account the sentence’s
that will help you with vocabulary, grammatical complexity, key phrase.
tone, and style. The key phrase here is ‘grow to dangerous proportions’. The
Funda 1: Before You Look at the Choices, Read the Sentence writer fears that class and race hatred may grow large enough to
and Think of a Word That Makes Sense endanger society. He wants us to wipe out this hatred before it is
Your problem is to find the word that best completes the sentence fully-grown. Examine each answer choice, eliminating those
in both thought and style. Before you look at the answer choices, answers that carry no suggestion that something lacks its full
see if you can come up with a word that makes logical sense in growth. Does overt suggest that something isn’t fully-grown?
the context. Then look at all five choices. If the word you thought No, it suggests that something is obvious or evident. Does
of is one of your five choices, select that as your answer. If the uncultivated suggest that something isn’t fully grown? No, it
word you thought of is not one of your five choices, look for a suggests that something is unrefined or growing without proper
synonym of that word. Select the synonym as your answer. care or training. Does independent suggest that something isn’t
This Funda is helpful because it enables you to get a sense of the fully-grown? No, it suggests that something is free and
sentence as a whole without being distracted by any misleading unconstrained. Only one word suggests a lack of full growth:
answers among the answer choices. You are free to concentrate embryonic (at a rudimentary, early stage of development). The
on spotting key words or phrases in the body of the sentence correct answer is Choice D.
and to call on your own “writer’s intuition” in arriving at a Funda 3: In Double-Blank Sentences, Go Through the Answers,
stylistically apt choice of word. Testing the First Word in Each Choice (and Eliminating Those
See how the process works in a typical model question. That Don’t Fit)
1. Because experience had convinced her that he was both
In a sentence completion question with two blanks, read through
self-seeking and avaricious, she rejected the likelihood that
the entire sentence to get a sense of it as a whole. Then insert the
his donation had been.................
first word of each answer pair in the sentence’s first blank. Ask
(A) redundant (B) frivolous yourself whether this particular word makes sense in this blank.
(C) inexpensive (D) ephemeral If the initial word of an answer pair makes no sense in the sentence,
(E) altruistic you can eliminate that answer pair.
This sentence presents a simple case of cause and effect. The (Note: Occasionally this Funda will not work. In some questions,
key phrase here is self-seeking and avaricious. The woman has for example, the first words of all five answer pairs may be near-
found the man to be selfish and greedy. Therefore, she refuses to synonyms. However, the Funda frequently pays off, as it does in
believe he can do something.............. What words immediately the following example.)
come to mind? Selfless, generous, charitable? The missing word
1. Critics of the movie version of The Colour Purple ...........
is, of course, altruistic. The woman expects selfishness (self-
its saccharine, overoptimistic mood at odds with the
seeking) and greediness (avaricious), not altruism (magnanimity).
novel’s more .....................tone.
The correct answer is Choice E.
(A) applauded...sombre
Practice of Funda 1 extensively develops your intuitive sense of
(B) condemned...hopeful
just the exactly right word. However, do not rely on Funda 1
(C) acclaimed...positive
alone. On the test, always follow up Funda 1 with Funda 2.
(D) denounced...sanguine
Funda 2: Look at All the Possible Answers Before You Make (E) decried...acerbic
Your Final Choice For a quick, general sense of the opening clause, break it up.
Never decide on an answer before you have read all the choices. What does it say? Critics……….the movie’s sugary sweet mood.
You are looking for the word that best fits the meaning of the How would critics react to something sugary sweet and over-
sentence as a whole. In order to be sure you have not been hasty hopeful? They would disapprove. Your first missing word must
in making your decision, substitute all the answer choices for the be a synonym for disapprove.
Now eliminate the misfits. Choices A and C fail to meet the test:
missing word. Do not spend a lot of time doing so, but do try
applauded and acclaimed signify approval, not disapproval.
them all. That way you can satisfy yourself that you have come
Choice B, condemned, Choice D, denounced and Choice E,
up with the best answer.
VOCABULARY C-9
decried, however, all disapprobation; they require a second look. surprise surprising surprisingly
To decide among Choices B, D, and E, consider the second blank. unexpected unexpectedly
The movie’s sugary, overly hopeful mood is at odds with the Note the function of such a contrast signal word in the following
novel’s tone: the two moods disagree. Therefore, the novel’s question.
tone is not hopeful or sugary sweet. It is instead on the bitter or 1. Paradoxically, the more ........... the details this artist
sour side; in a word, acerbic The correct answer is clearly Choice E. chooses, the better able she is to depict her fantastic,
Remember that, in double-blank sentences, the right answer must otherworldly landscapes.
correctly fill both blanks. A wrong answer choice often includes (A) ethereal (B) realistic
one correct and one incorrect answer. ALWAYS test both words. (C) fanciful (D) extravagant
Funda 4: Watch for Signal Words That Link One Part of the (E) sublime
Sentence to Another The artist creates imaginary landscapes that do not seem to belong
Writers use transitions to link their ideas logically. These to this world. We normally would expect the details comprising
these landscapes to be as fantastic and supernatural as the
transitions or signal words are clues that can help you figure out
landscapes themselves. But the truth of the matter, however, is
what the sentence actually means. Sentences often contain several
paradoxical: it contradicts what we expect. The details she chooses
signal words, combining them in complex ways.
are realistic, and the more realistic they are, the more fantastic the
1. Cause and Effect Signals
paintings become. The correct answer is Choice B.
Look for words or phrases explicitly indicating that one
thing causes another or logically determines another. Funda 5: Use Your Knowledge of Word Parts and Parts of Speech
Cause and Effect signal Words to figure out the meanings of Unfamiliar Words
Accordingly in order to Because If a word used by the author is unfamiliar, or if an answer choice
so...that Consequently Therefore is unknown to you, two approaches are helpful.
Given thus Hence 1. Break up the word into its component parts - prefixes,
when….then If….then suffixes, and roots - to see whether they provide a clue
Look for words or phrases explicitly indicating that the to its meaning. For example, in the preceding list of Implicit
omitted portion of the sentence supports or continues a Contrast Signal Words, the word incongruous contains
thought developed elsewhere in the sentence. In such cases, three major word parts, in- here means not; con- means
a synonym or near-synonym for another word in the together; gru- means to move or conic. Incongruous
sentence may provide the correct answer. behaviour, therefore, is behaviour that does not go together
Support Signal Words or agree with someone’s usual behaviour; it is unexpected.
Additionally furthermore Also 2. Change the unfamiliar word from one part of speech to
indeed And Likewise another. If the adjective embryonic is unfamiliar to you,
as well moreover besides cut off its adjective suffix -nic and recognise the familiar
too word embryo. If the noun precocity is unfamiliar to you
2. Contrast Signals (Explicit) cut off its noun suffix -ity and visualise it with different
Look for functional words or phrases (conjunctions, endings. You may think of the adjective precocious
adverbs, etc.) that explicitly indicate a contrast between (maturing early). If the verb appropriate is unfamiliar to
one idea and another, setting up a reversal of a thought. In you, by adding a word part or two, you may come up with
such cases, an antonym or near-antonym for another word the common noun appropriation or the still more common
in the sentence may provide the correct answer. noun misappropriation (as in the misappropriation of
Explicit Contrast Signal Words funds).
Albeit nevertheless Although Note the application of this Funda in the following typical
nonetheless But example.
Notwithstanding despite 1. This island is a colony; however, in most matters, it is
on the contrary even though ........ and receives no orders from the mother country.
on the other hand however (A) dichotomous (B) methodical
rather than in contrast still (C) heretical (D) autonomous
in spite of while instead of yet (E) disinterested
3. Contrast Signals (Implicit) First, eliminate any answer choices that are obviously incorrect.
Look for content words whose meanings inherently indicate If a colony receives no orders from its mother country, it is
a contrast. These words can turn a situation on its head. essentially self-governing. It is not necessarily methodical or
They indicate that something unexpected, possibly even systematic nor is it by definition heretical (unorthodox) or
unwanted, has occurred. disinterested (impartial). Thus, you may rule out Choices B, C,
Implicit Contrast Signal Words and E.
anomaly Anomalous anomalously The two answer choices remaining may be unfamiliar to you.
illogically illogical Analyse them, using what you know of related words. Choice A,
incongruity incongruous incongruously dichotomous, is related to the noun dichotomy, a division into
irony ironic ironically two parts, as in the dichotomy between good and evil. Though
paradox paradoxical paradoxically the island colony may be separated from the mother country by
C-10 VOCABULARY
distance, that has nothing to do with how the colony governs In calling the smuggled urn a "hot pot'',. Hoving is not necessarily
itself. Choice D, autonomous, comes from the prefix auto-(self) speaking characteristically or redundantly or cheerfully. He is
and the root nom-(law). An autonomous nation is independent; it speaking either informally or colloquially. (Hot here is a slang
rules itself. Thus, the correct answer is autonomous, Choice D. term meaning stolen or illegally obtained.) Its costliness is not
Funda 6 : Break Up Complex Sentences into Simpler Components being questioned. However, because the urn has been smuggled
into the country, there clearly are unresolved questions about
In analysing long, complex sentence completion items, you may
how it got here, in other words, about its manner of acquisition.
find it useful to simplify the sentences by breaking them up.
The correct answer is Choice C.
Rephrase dependent clauses and long participal phrases, turning
Note that in sentence completion questions a choice may he
them into simple sentences.
complicated by an unusual word order, such as:
See how this Funda helps you to analyse the following sentence.
1. placing the subject after the verb: To the complaints
1. Museum director Hoving ........... refers to the smuggled
window strode the angry customer.
Greek urn as the “hot pot;” not because there are doubts
2. placing the subject after an auxiliary of the verb: Only by
about its authenticity or even great reservations as to its
unending search could some few Havana cigars be found.
price, but because its ........... of acquisition is open to
3. inverting the subject and verb to give the sense of “if”:
question.
Were defeat to befall him today’s dear friends would be
(A) informally…..costliness
tomorrow’s acquaintances, and next week’s strangers.
(B) characteristically…..date
4 placing a negative word or phrase first, which usually
(C) colloquially…..manner
requires at least part of the verb to follow: Never have I
(D) repeatedly…..swiftness
encountered so demanding a test!
(E) cheerfully.....m ode
In all these instances, rephrase the sentence to make it more
What do we know?
straightforward. For example :
1. The urn has been smuggled.
1. The angry customer strode to the complaints window.
2. Hoving calls it a “hot pot.”
2. Some few Havana cigars could be found only by unending
3. It is genuine. (There are no doubts about its authenticity.)
search.
4. It did not cost too much. (There are no great reservations
3. If defeat were to befall him, today’s dear friends would be
as to its price.)
tomorrow’s acquaintances, and next week’s strangers.
4. I have never encountered so demanding a test!
VOCABULARY C-11

EXERCISE
Directions (Qs. 1-150): Pick out the most effective pair of 9. Management can be defined as a process of..............
words from the given pair of words to make the sentence/ organizational goals by working with and through human
sentences meaningfully complete. and non-human resources to .............. improve value
1. The teacher must..............the unique style of a learner in added to the world.
order to .............. it to the desired knowledge. (a) getting, deliberately (b) managing, purposefully
(a) advocate, direct (b) perpetuate, develop (c) targeting, critically (d) realizing, dialectically
(c) appreciate, focus (d) absorb, maintain (e) reaching, continuously
(e) discover, harness 10. If you are an introvert, you .............. to prefer working
2. Not all countries benefit .............. from liberalisation. The
alone and, if possible ,will .............. towards projects where
benefits tend to .............. first to the advantaged and to
you can work by yourself or with as few people as
those with the right education to be able to benefit from
the opportunities presented. possible.
(a) equally, generate (b) richly, downgrade . (a) like, depart (b) advocate, move
(c) suitably, ascribe (d) uniformly, percolate (c) tend, gravitate (d) express, attract
(e) judiciously, facilitate (e) feel, follow
3. He has a.................sense of words. Therefore, the sentences 11. The..............violence that our town is witnessing has turned
he constructs are always .............. with rich meaning. it into the most .............. town of our country.
(a) profound, pregnant (b) distinguished, loaded (a) huge, offensive
(c) terrific, tempted (d) meaningful, full (b) unrestrained, devalued
(e) outstanding, consistent (c) perplexing, repulsive
4. He was an .................musician, had been awarded the (d) unprecedented, degenerate
George Medal during the Second World War and .............. (e) mammoth, wanted
with the title of Rai Bahadur. 12. Although religion does not ..............the acquisition of
(a) outstanding, popularised wealth,
(b) underestimated, declared the tenor of its teaching is to..............an attitude of
(c) accomplished, honoured indifference to worldly things.
(d) impressive, assigned (a) proclaim, prohibit (b) inhibit, induce
(e) obdurate, proclaimed
(c) manifest, proud (d) delink, develop
5. Whether it be shallow or not, commitment is the ..............,
(e) allow, criticise
the bedrock of any .............. loving relationship.
(a) expression, perfunctory 13. Students should be .............. to realise that the real goods
(b) foundation, genuinely of life are spiritual, love of things of the spirit and ..............
(c) manifestation, deep of their fellowmen.
(d) key, alarmingly (a) made, service (b) allowed, needs
(e) basis, absorbing (c) believed, service (d) made, devotion
6. Many people take spirituality very seriously and .............. (e) professed, involvement
about those who don’t, worrying about them and .............. 14. Extreme poverty is as .............. to stagnation and
them to believe. impoverishment as ..............wealth.
(a) think, criticizing (b) pride, appraising (a) dangerous, restrained (b) provocative, permissible
(c) rationalize, enabling (d) wonder, pressing (c) supportive, foul (d) stupendous, corrupt
(e) ponder, venturing (e) liable, excessive
7. Unless new reserves are found soon ,the world’s supply 15. It is accurate to refer to poets as dreamers but it is
of coal is being .............. in such a way that with demand not.............. to infer that the dreams of poets have no
continuing to grow at present rates, reserves will be practical value beyond the..............of literary diversion.
.............. by the year 2050. (a) possible, shadow (b) sensible, object
(a) consumed, completed (b) depleted, exhausted (c) discerning, realm (d) valuable, field
(c) reduced, argument (d) burnt, destroyed
(e) comparable, circle
(e) utilized, perished
16. Every ..............step man takes in any field of life, is first
8. If you are .............. you tend to respond to stressful
situations in a calm, secure, steady and .............. way. taken along the dreamy ..............of imagination.
(a) resilient, rational (b) obdurate, manageable (a) calculated, vision (b) outward, base
(c) propitious, stable (d) delectable, flexible (c) forward, path (d) initial, thought
(e) supportive, positive. (e) parallel, extent
C-12 VOCABULARY
17. At the dawn of history India ..............on her unending (d) threatened, manifested
quest and trackless centuries are ..............with her striving (e) projected, discussed
and the grandeur of her success and her failures. 26. When interpersonal problems.............. but are not dealt
(a) started, filled (b) marched, evolved with, the organisation’s productivity inevitably..............
(c) proceeded, pulled (d) started, vanished (a) surface, develops (b) focus, increases
(e) marched, filled (c) establish, projects (d) develop, exhibits
18. It is height of ..............for men who fully appreciate in their (e) exist, diminishes
own case the great advantages of a good education 27. Participative management, in which everyone has a
but..............these advantages to women. .............. in a decision that a leader then makes, is a
(a) arrogance, abstain (b) propensity, bestow mechanism for .............. employees.
(c) proliferation, advance (d) selfishness, deny (a) share, protecting (b) value, thwarting
(e) stupidity, prohibit (c) motivation, involving (d) reward, stimulating
19. As everybody is .............. attached to his motherland it is (e) input, empowering
difficult to find a man who is ..............of love for his 28. Lack of.............. is basic to good teamwork but our ability
motherland. to work with others depends on our..............
(a) constantly, disposed (b) courageously, suspect (a) rigidity, compatibility
(c) profoundly, deprived (d) greatly, connected (b) dogmatism, motivation
(e) deeply, devoid (c) professionalism, vulnerability
20. If we study the lives of great men we are.............. that we (d) positivism, flexibility
too can achieve greatness and when we die, we leave (e) consideration, acumen
behind our ............... 29. Complete and constant openness is a notion that can
(a) convinced, children (b) reminded, footmarks be.............. to absurdity. Am I.............. to stop everyone
(c) conveyed, followers (d) commended, belongings on the street and tell them my reaction to their appearance?
(e) proclaimed, memories (a) consigned, communicated
21. Handicrafts constitute an important ..............of the (b) reduced, required
decentralised sector of India’s economy and .............. (c) attributed, requested
employment to over six million artisans. (d) projected, destined
(a) factors , aims (b) extension, plans (e) subjected, confined
(c) segment, provides (d) supplier, gives 30. When organizations.............. creativity and risk-taking, the
(e) period, projects usual method of maintaining order and..............are indeed
22. Among a number of hobbies that one can .............., hobby shaken.
of reading is the most useful and ............... (a) encourage, decorum (b) exhibit, durability
(a) develops, accumulative (c) propose, humility (d) enhance, supply
(b) cultivates, enlightened (e) propagate, production
(c) nourish, cumulative 31. When I am an autocrat, I am.............. in the extreme. My
(d) absorbs, durable direct method is to.............. power and control.
(e) considers, appreciative (a) perfectionist, explore (b) autocratic, engulf
23. The quality of .............. between individuals and the (c) dominating, seek (d) possessive, reject
organisation for which they work can be .............. to the (e) elaborate, develop
benefit of both the parties. 32. The fear.............. with feeling incompetent is the fear of
(a) services, evaluated being humilitated, embarrassed and..............
(b) interaction, improved (a) endowed, criticized (b) afflicted, downtrodden
(c) sophistication, developed (c) consistent, damaged (d) associated, vulnerable
(d) work, appreciated (e) imbued, exposed
(e) life, conceptualised 33. Gopal was frustrated with Sundar, who would not..............
24. Part of the confusion in our societies .............. from our himself to a deadline. Sundar claimed he was working well
pursuit of efficiency and economic growth, in the .............. without a deadline, but Gopal .............. and finally
that these are the necessary ingredients of progress. prevailed.
(a) stems, conviction (a) encourage, enforced (b) inculcate, ordered
(b) derives, evaluation (c) cooperate, stipulated (d) commit, persisted
(e) declared, pressurised
(c) emerges, consideration
34. I am not easily.............. by pressure that would interfere
(d) obtains, exploration
with accomplishing the goals of my unit. I stick to
(e) extends, planning
my..............
25. The problem of housing shortage .............. with the
(a) pessimistic, views
population explosion has also been .............. by this policy.
(b) swayed, convictions
(a) coped, highlighted
(c) discouraged, achievements
(b) dispensed, acknowledged
(d) empowered, organization
(c) compounded, addressed
(e) demurred, projections
VOCABULARY C-13
35. Coercion sometimes leads to the.............. of short-term 44. It is a noble quality to.............. the good and..............the
goals, but its drawbacks far.............. its advantages. bad.
(a) realization, damage (a) retain, preserve (b) imitate, pretend
(b) appreciation, percolate (c) appreciate, condemn (d) criticize, ape
(c) accomplishment, outweigh (e) ignore, eliminate
(d) achievement, crumble 45. Using..............designs to..............the importance of others
(e) destination, magnify is not a healthy act.
36. In our country, the .............. use of radio as an instrument (a) nefarious, undermine (b) architectural, elevate
of man’s education has not yet been.............. (c) fictitious, enhance (d) brittle, activate
(a) fullest, exploited (b) ultimate, used (e) proper, shrink
46. Professionals focus their ..............on fulfilling their
(c) meaningful, explored (d) resultant, tried
responsibilities and achieving results, not on..............a
(e) present, demonstrated
particular image.
37. The modern world is fast.............. itself into rival camps,
(a) planning, devising (b) leadership, attributing
armed to the teeth with the sophisticated and (c) abilities, contributing (d) energies, portraying
deadly.............. of destruction. (e) skills, obtaining
(a) making, sources 47. “Patriotism is the last refuge of the scoundrel,” says
(b) changing, factors Johnson. In the modern world where the cunning selfish
(c) developing, procedures people..............and the hardworking conscientious people
(d) parting, way ..............the quotation holds good.
(e) resolving, weapons (a) dominate, suppress (b) thrive, suffer
38. The great writer..............art out of facts of life, the (c) enjoy, mutilate (d) empower, subjected
significant.............. with which the pattern of life has been (e) harass, abdicate
woven. 48. We must rest assured that our sorrows, if any, are short
(a) decides, factor (b) selects, element – lived and a period of joy..............us. Such thinking will
(c) practises, boundary (d) creating, disposition always maintain our ..............and peace of mind.
(e) chooses, part (a) begets, dream (b) befits, body
39. We mean by civilization a way of life in which the (c) bemoans, skills (d) merits, life
wilder..............of humanity are restrained, the nobler (e) awaits, equipoise
instincts and inclinations are.............. and allowed to 49. When you are living with your..............values and
principles, you can be straightforward, honest and ...............
prevail.
(a) core, upfront (b) inherited, distinct
(a) dimensions, created (b) qualities, discouraged
(c) innate, durable (d) cultural, perceptive
(c) passions, developed (d) desires, inhibited
(e) inborn , vehement
(e) aspects, encouraged
50. In the role of a counsellor, you are an authority figure
40. We are said to be superstitious when we.............. ourselves whose objective is to ..............attentively and sensitively
to fanciful causes for happenings that seem to be.............. to employees who..............you with their feelings.
(a) bind, unwarranted (b) project, heavenly (a) project, focus (b) manage, direct
(c) torture, harmful (d) subject, inexplicable (c) listen, trust (d) concentrate, believe
(e) treat, harrowing (e) consider, explain
41. Despite..............knowledgeable, he remained..............all 51. If a junior executive neglects his professional development
through. and ..............education, he can easily and quickly become
(a) having, ignorant (b) of, doubtful obsolete in a world changing at..............rates.
(c) owing, through (d) having, enriched (a) management, voluminous
(e) being, poor (b) higher, vulnerable
42. He used to.............. the point that victory in any field (c) better, supreme
needs..............courage. (d) continuing, dizzying
(a) emphasise, little (b) stress, exemplary (e) value, profound
(c) refute, no (d) dismiss, formidable 52. Harassment is just plain............... There is never a valid
(e) distract, enormous reason or a good ..............for it.
43. We will call back the agitation if the Chairman gives (a) wrong, excuse (b) murder, command
an..............that there will be no..............of workers. (d) falsehood, remedy (d) killing, magic
(a) inkling, harassment (e) offence, strategy
(b) assurance, need 53. Man is .............. He likes to know how things work. The
(c) explanation, demand search for understanding is ..............in its own right.
(d) opportunity, place (a) evolving, prophetic
(b) inquisitive, legitimate
(e) undertaking, retrenchment
(d) appreciative, fundamental
C-14 VOCABULARY
(d) curious, philosophical 63. The .............. growth of Indian agriculture in the last three
(e) social, judgmental decades has earned .............. from other countries.
54. Leadership is one of the world’s oldest............... The (a) pervasive, reputation
understanding of leadership has figured strongly in the (b) significant, deliverance
..............for knowledge. (c) superior, regard
(a) cultures, desire (d) dynamic, accolades
(b) institutions, passion (e) distinctive, encouragement
(c) pre-occupations, quest 64. College-going students should .............. the spirit of service
(d) subjects, preference from the great men of ..............
(e) undertakings, conviction (a) inculcate, power (b) develop, possession
55. Our..............to understand the process of learning (c) invent, wisdom (d) analyze, distinction
underlying behaviour change are..............by the fact that (e) imbibe, yore
any given behaviour is determined jointly by many 65. The planting of trees on the .............. of towns and villages
processes. helps the ..............of a country.
(a) nature, determined (b) scope, preceded (a) surface, beautification
(c) implications, followed (d) limitations, moderated (b) periphery, output
(e) attempts, complicated (c) joints, production
56. This book is readable, clear and..............researched with (d) vicinity, cultivation
..............detailed references. (e) outskirts, afforestation
(a) fabulously, intricate (b) meticulously, extensive 66. The human infant is a lifelong bundle of energy with a
(c) leisurely, complete (d) hardly, notional ..............array of potentialities, and many...............
(e) closely, scattered (a) marvellous, vulnerabilities
57. The interest has ... innovation related to several sectors (b) peculiar, opportunities
of the... pharmaceutical business and industry. (c) critical, competencies
(a) accepted, skeletal (b) insured, fateful (d) vocational, strengths
(c) affected, plummeting (d) restricted, grown (e) perfect, peculiarities
(e) accelerated, burgeoning 67. His presentation was so lengthy and ..............that it was
58. The Indian hospitality industry, which has been..............a difficult for us to find out the real .............. in it.
prolonged slump, is now entering a new..............phase (a) boring, planning (b) tedious, skill
ready to enhance profitability. (c) verbose, content (d) laborious, coverage
(a) witnessing, ambitious (e) simple, meaning
(b) observing, listless 68. There is no ..............the fact that a man of knowledge
(c) demonstrating, efficient ..............great power.
(d) recovering, debt (a) justifying, acknowledges
(e) succumbing, lean (b) clarifying, exhibits
59. The new policy seeks to..............a process of widespread (c) advocating, projects
participation of people..............the way for more effective (d) denying, wields
implementation. (e) proclaiming, develops
(a) amend, opening (b) halt, propelling 69. The..............of opinion which emerged at a recently
(c) generate, paving (d) diagnose, resulting concluded seminar was that the problem of dowry cannot
(e) identify, dithering be .............. unless the law against it is made more stringent.
60. Journalism is a ..............for truth, a crusade to .............. (a) divergence, managed
injustice and wrong-doing. (b) sympathy, projected
(a) product, limit (b) boon, justify (c) consensus, tackled
(c) travesty, attack (d) quest, expose (d) similarity, curbed
(e) search, optimise (e) convergence ..... appreciated
61. The society provides the individual security of life, 70. Leisure must be spent carefully and ..............only, otherwise
..............of thought and sustenance for action. Every the devil will get the ..............of you.
individual who ..............from society is indebted to the (a) positively, care (b) constructively, better
society. (c) proactively, though (d) objectively, energy
(a) serenity, gains (b) prosperity, benefits (e) purposefully, measure
(c) objectivity, profits (d) seriousness, derives 71. The .......... manner of the officer ..........all his fears.
(e) semblance, evolves
(a) haughty, aggravated (b) officious, levelled
62. A hobby is an activity of interest ..............for pleasure. It
(c) amiable, concentrated (d) fickle, reduced
helps to break the monotony and tedium of our ..............
(e) genial, dispelled
routine.
72. To .......... his arguments, he ..........several references.
(a) developed, interesting (b) pursued, humdrum
(a) vindicate, refer (b) press, announced
(c) cultivated, developed (d) regularized, cultivated
(c) substantiate, cited (d) reveal, declared
(e) arranged, pursued
(e) clarify, averted
VOCABULARY C-15
73. As censorship was ..........people were ..........the furious 84. The handwriting on the letter was highly..............and none
battle being fought. could.............. it.
(a) clammed, oblivious of (a) legible, write (b) illegible, read
(b) revoked, indifferent to (c) original, recognize (d) faint, notice
(c) imposed, unaware of (e) shabby, identify
(d) dispelled, ignorant of 85. The delay in completing the task ..............can be attributed
(e) levelled, unmindful of to their ..............
74. He was..........by letter that he had .......... selected. (a) expeditiously, lethargy (b) fully, punctuality
(a) informed, been (b) told, not (c) economically, rigidity (d) religiously, strictness
(c) stated, finally (d) offered, been (e) carefully, perfectionism
(e) communicated, not 86. He ..............for having caused..............to the residents.
75. Giving unfair .......... to one's relatives in the matter of
(a) questioned, authenticity
appointments and such other benefits is ..........
(b) apologized, impatience
(a) punishments, parochialism
(c) regretted, inconvenience
(b) advantage, nepotism
(d) wondered, disappointment
(c) benefits, pragmatism
(e) pressurised, discomfort
(d) preferences, chauvinism
(e) leverage, communalism 87. The new ..............in the defence field are quite ..............and
76. The Vidhan Sabha voted to ..........the ban .......... drinking. hence appreciable.
(a) invoke, for (b) revoke, of (a) ideas, novel (b) studies, substandard
(c) prohibit, prevented (d) repeal, on (c) incentives, diplomatic (d) appointments, sensitive
(e) refrain, from (e) inventions,. admirable
77. Our army is .......... ..........the border of the neighbouring 88. He admitted having ..............about the ..............of the
country. documents.
(a) posted, with (b) accumulated, for (a) questioned, authenticity
(c) assembled, at (d) deployed, along (b) regard, truth
(e) fighting, on (c) aware, possession
78. The teacher ..........his students .......... being late to school. (d) predicted, correctness
(a) shouted, at (b) reprimanded, for (e) enquired, preparation
(c) reminded, with (d) narrated, of 89. Despite ..............of resources, the financially
(e) taught, for underprivileged students .............. in their endeavour.
79. The dimensions of the .......... are known and the solution (a) plenty, failed (b) availability, gave
is .......... on. (c) want, surrendered (d) lack, succeeded
(a) problem, agreed (b) theory, dealt (e) extremity, excelled
(c) measures, decided (d) risks, tempered 90. The work assigned to me is not..............though it is
(e) danger, looked very..............
80. There is so much .......... between the two sisters that it (a) voluminous, careful
is .......... to know one from the other.
(b) challenging, easy
(a) resemblance, difficult (b) identity, impossible
(c) impossible, stupendous
(c) equality, easy (d) disparity, undesirable
(d) exceptional, ordinary
(e) similarity, obvious
(e) meagre, difficult
81. How people would..............in a particular situation cannot
be..............with perfect accuracy. 91. Nothing is impossible in the world of politics. States
(a) think, observed (b) act, expected which were ..............foes and had their deadly missiles
(c) behave, predicted (d) react, analyzed pointed at each other find themselves ..............in military
(e) feel, resolved alliances.
82. When the mob turned ..............the police opened fire to (a) implacable, partners (b) intense, joining
control the ..............situation. (c) deadly, approaching (d) known, soliciting
(a) away, burning (b) up, untoward (e) enviable, grouping
(c) around, devastating (d) quietly, aggravate 92. ..............of whether leaders are born or made, it is
(e) hostile, riotous ..............clear that leaders are not like other people.
83. The Government ..............down the opposition ..............for (a) Pursuant, manifestly
a joint parliamentary committee probe. (b) Sequel, amply
(a) boiled, proposal (b) shot, demand (c) Regardless, unequivocally
(c) put, leaders (d) whipped, portfolio (d) Instead, purely
(e) struck, party (e) In spite, normally
C-16 VOCABULARY
93. There is a common talk today that women have made 102. The textile industry in India has..............rough weather in
the.............. in many professions; that they have total recent times. The textile mill is one of the few companies
freedom of opportunity. But the majority of women are to have..............this storm.
still left .............. at the unbreakable glass ceiling. (a) overcome, empowered
(a) entry, inward (b) grade, gazing (b) managed, absorbed
(c) progress, trying (d) mark, projecting (c) protested, fought
(e) achievement, wondering (d) withstood, survived
94. Inferring attitudes from expressed opinion has (e) ventured, managed
many..............People may..............their attitude and express 103. With large classes, it is difficult for teachers to..............
socially acceptable opinions. regular essay-type questions for homework
(a) limitations, conceal (b) advantages, show because..............long answers would take too much time.
(c) drawbacks, support (d) benefits, avoid (a) consider, writting (b) revalue, concise
(e) reasons, acknowledge (c) pursue, feeling (d) handle, weighing
95. We should move towards a system where the banks can (e) give, marking
..............capital in the market with..............safeguards so 104. Ours is a democracy and any..............or use of force is out
that they continue to be public sector banks. of question. Methods of..............and education are best
(a) improve, proper (b) strengthen, durable suited to a democratic regime.
(c) raise, adequate (d) stimulate, effective (a) attempt, coercion
(e) provide, delicate (b) compulsion, persuasion
96. Human Resource Management is an..............of mind rather (c) judgement, prayer
than a.............. of techniques. (d) inhuman, apprehension
(a) organisation, quality (b) attempt, mix (e) implied, technology
(c) evolution, measure (d) attitude, set 105. Despite being the_______partner in the relationship, the
(e) expertise, collection franchiser doesn’t always have all the________.
97. Statistics in an.............. tool for researchers (a) sincere ......limitations (b) vulnerable........powers
that..............them to make inferences or generalisations
(c) active ....losses (d) dominant......advantages
about populations from their observations of the
(e) authoritative ......legalities
characteristics of samples
106. India’s _____over the past half century since
(a) outstanding, proposes
independence has been unique and_______in many ways.
(b) invaluable, proceeds
(c) invaluable, enables (a) thinking ......feeling
(d) important, proclaims (b) development ..........commendable
(e) indispensable, enables (c) victory..... ...celebrating
98. The Dalits have never had a.............. of freedom in the (d) crash .......overbearing
suffocating society. They are a wounded people.............. (e) regress .......praiseworthy
and broken. 107. The so-called civilised human race has_________and
(a) glimpse, mitigated (b) sigh, rejected illtreated small and large animals in an attempt to prove
(c) moment, criticised (d) satisfaction, prohibited his __________.
(e) breath, battered (a) abused ...... supremacy
99. His vision could be the.............. that the policy-makers (b) misuse ...... power .
use to.............. the banking sector. (c) cruelty ...... altruism
(a) roadmap, restructure (b) manner, shape (d) advocated ...... worthlessness
(c) blueprint, plan (d) remedy, revise (e) beaten ...... generosity
(e) approach, represent 108. He objected to the proposal because it was founded on
100. A person’s formal educational background may.............. a ________ principle and also was ________ at times.
rich but complex in formation. To some degree (a) faulty - desirable
education.............. a person’s knowledge and skill base. (b) imperative - reasonable
(a) reveal, advocates (b) yield, indicates (c) wrong - inconvenient
(c) exhibit, develops (d) cover, evolves (d) sound - acceptable
(e) surmount, shows (e) conforming - deplorable
101. The RBI in consultation with Government of India 109. The criterion for ________ a player should be based on
has..............a working group to suggest measures his recent performance; but unfortunately, the journalists
for..............of weak public sector banks. are ________ to be carried away by earlier successes.
(a) commissioned, appreciating
(a) condemning - satisfying
(b) established, accommodation
(b) judging - prone
(c) reshaped, merger
(c) revealing - reluctant
(d) constituted, revival
(d) eager - acclaiming
(e) organised, development
(e) criticising - clean
VOCABULARY C-17
110. For the last half century he ________ himself to 120. Problems of the country can only be aggravated by
public affairs ________ taking a holiday. ...........and ........... citizens.
(a) by - committed (b) after - offered (a) careful, unscrupulous
(c) devoted - without (d) sacrificed - after (b) extraordinary, uncompromising
(e) prepared - before (c) dedicated, patriotic
111. You will see signs of ________ everywhere, which speak (d) selfish, irresponsible
well for the ________ of these people. (e) independent, practical
(a) decoration - senses (b) clear - debris 121. The ...........with which he is able to wield the paintbrush
(c) beauty - careful (d) industry - prosperity is really ...........
(e) repairs - extravaganza (a) practice, good (b) majesty, royal
(c) sweep, fine (d) energy, unnecessary
112. The police arrested Ramesh on a ________ of theft but
(e) ease, remarkable
for lack of evidence ________him.
122. If you kindly bear ............. me for some time, I shall
(a) crime - imprisoned (b) punished - complaint
.............the whole issue.
(c) left - condition (d) tip - absconding (a) out, understand (b) on, solve
(e) charge - released (c) for, know (d) with, clarify
113. People ________ to work fast if you ________ certain (e) at, inform
conditions on them. 123. The feeling of the people is not entirely...........or ...........
(a) decide, negotiate (b) try, thrust (a) sound, unfounded
(c) plan, invoke (d) hesitate, impose (b) unreasonable, imaginary
(e) volunteer, place (c) baseless, unjustified
114. The recent ________ in oil prices has given an (d) cohesive, irrational
unexpected additional ________ to the cost-spiral. (e) unpasdonable, unimaginative
(a) slump, drawback (b) cut, blow 124. People who are .............can ............. their things in an
(c) rise, twist (d) development, out orderly manner.
(e) deterioration, impetus (a) upright, keep (b) thrifty, perform
115. ________ your colleagues for important decision-making (c) punctual, manage (d) indecisive, do
activities ensures their ________ cooperation. (e) dependable, accumulate
(a) Counselling, whole-hearted 125. The WHO has .............the government for taking prompt
(b) Helping, occasional ............. action.
(c) Guiding, meagre (a) lauded, preventive (b) criticised, obligatory
(d) Neglecting, enthusiastic (c) blamed, ineffective (d) appreciated, hectic
(e) Dominating, unstinted (e) admired, unreasonable
116. The only way to ensure best output from your vehicle is 126. Though the hospital is extremely rich, the conditions
to provide it a ________ and ________ maintenance. there are ............. ..............
(a) nurturing, expensive (b) proper, timely (a) equally, rich
(c) careful, costly (d) trouble-free, everlasting (b) appallingly, poor
(e) precious, healthy (c) admirably, comfortable
117. The issues could be ________ amicably only because (d) extremely, conducive
of his ________ handling of the situation. (e) frequently, tolerable
(a) dropped, haphazard 127. Nine members have ............. about the decision, but the
(b) raised, careful tenth one views it .............
(c) discussed, enthusiastic (a) solution, critically
(d) suppressed, emphatic (b) consensus, similarly
(e) resolved, tactful (c) disagreement, collectively
118. Punishment is ...........and ...........a lot of wrong doings. (d) grievance, grudgingly
(a) a deterrent, prevents (e) agreement, differently
(b) inevitable, encourages 128. Man needs food not .............. for the body but for the
(c) mandatory, ensures soul also. The satisfaction of his physical wants does not
(d) an evil, prohibits imply his.............. .
(e) essential, nullifies (a) merely, contentment (b) properly, superiority
119. The account of the crime given by the accused was (c) only, spirituality (d) necessarily, commitment
...........and was ........... . (e) certainly, entitlement
(a) dependable, uncognizable 129. It is said that knowledge is power. The hunger for power
(b) fictitious, unreliable is ..............and therefore most difficult to ...............
(c) false, appreciable (a) accumulative, subsume (b) enormous, apply
(d) complete, imaginary (c) empowering, delegate (d) insatiable, contain
(e) exaggerated, unpardonable (e) evolutionary, rationalize
C-18 VOCABULARY
130. Nothing undermines the communication of a changed 140. It would be impossible for us to continue living in this
vision more than .............. on the part of key.............. that world if each of us.............. exactly what fate had in ..............
seems inconsistent with the vision. for him.
(a) anything , issues (b) behaviour, players (a) follow, plan (b) appreciate, strategy
(c) advocacy, managers (d) something, personnel (c) design, anticipation (d) visualize, hidden
(e) philosophy, problems (e) knew, store
131. Mountains and hills are a .............. sight. I have always 141. It is the .............. of selfishness for men, who fully ..............
.............. to see them. in their own case the great advantages of good education,
(a) extraordinary, advocated to deny these advantages to women.
(b) stupendous, encouraged
(a) parody, demand (b) height, appreciate
(c) loving, prepared
(c) height, assimilate (d) degree, appreciate
(d) joyful, imagined
(e) fascinating, longed (e) level, advance
132. Poetry is the language of the imagination and the ............... 142. The learner should be .............. to take a small first step
It relates to whatever gives .............. pleasure or pain to one that will provide immediate success and .............. the
the human mind. learning.
(a) thinking, permanent (b) analysis, temporary (a) encouraged, reinforce (b) forced, organise
(c) passions, immediate (d) circumspection, sporadic (c) directed, reorganise (d) cautioned, reinforce
(e) visualization, constant (e) encouraged, acknowledge
133. Success in business requires two things: a winning 143. His death .............. more tributes than have been paid at
competitive ..............and superb organizational ............... the ..............of any other human being in history.
(a) advantage, satisfaction (a) brought, passing
(b) planning, advantage (b) directed, helm
(c) strategy, execution (c) delivered, description
(d) phisolophy, motivation
(d) invited, living
(e) marketing, lstrategy
(e) acknowledged, perpetuation
134. To .............. in today's rapidly changing environment
144. Only with executive .............. can the organisation
corporations need to .............. their learning capability.
concentrate its energies on .............. competitive advantage
(a) develop, enlarge (b) surpass, align
over time.
(c) project, assimilate (d) service, mitigate
(a) position, embarking (b) deployment, directing
(e) compete, strengthen
135. A ..............reader gets much greater pleasure from reading (c) contingent, fabricating (d) commitment, sustaining
books than a miser gets in ..............money. (e) satisfaction, moulding
(a) powerful-accumulating(b) voracious-hoarding 145. All the performances of human art, at which we look with
(c) desirous-stocking (d) proverbial-spending praise or wonder, are .............. of the restless ..............of
(e) profound-lavishing perseverance.
136. Vision is usually ..............most effectively when many (a) manifestations, pronouncement
different ..............are used. (b) projections, component
(a) developed, manifestations (c) instances, force
(b) adapted, organisations (d) proofs, humanity
(c) communicated, vehicles (e) visions, future
(d) exhibited, forms 146. He found the jewellery box empty. All the ornaments,
(e) described, thought jewels, etc, were ..............
137. People who have been through difficult, painful and not (a) protected (b) seized
very .............. change efforts often end up ..............both (c) sold (d) melted
pessimistic and angry conclusions. (e) stolen
(a) successful, drawing (b) meaningful, projecting 147. Ill health cannot dampen his spirits. He was found quite
(c) reliable, evolving (d) strong, following ..............at the ceremony.
(e) challenging, lamenting (a) spiritual (b) effervescent
138. The human mind is never ..............; it advances or it .............. (c) subdued (d) uncomfortable
(a) absolute, diminishes (b) dynamic, stops (e) nervous
(c) perfect, disintegrates (d) stationary, retrogrades
148. There is a growing indifference among the villagers.
(e) happy, decomposes
..............of them bothered to extinguish the fire which
139. If misery is the effect of ill fortune, it ought to be pitied, broke out in the hutments.
if of .............. to be ...............
(a) Most (b) Many
(a) virtue, criticised (b) calamity, reverenced
(c) Few (d) None
(c) virtue, protected (d) vice, reverenced
(e) Everyone
(e) virtue, reverenced
VOCABULARY C-19
149. We spared no efforts to win, but the opposite side did 160. (A) Punishment (B) Divergence
better. The result was obviously ..............us. (C) Confluence (D) Confidence
(a) in favour of (b) rejoicing (a) B-C (B) B-D
(c) against (d) grudging (c) C-D (d) A-B
(e) introspecting (e) A-C
150. The musician had a sore throat. Despite that, performance 161. (A) Audacious (B) Venturous
at the concert was ............... (C) Abstruse (D) Silent
(a) outstanding (b) sub-standard (a) A-C (b) B-C
(c) C-D (d) A- B
(c) undesirable (d) excellence
(e) B-D
(e) happy
162. (A) Encomium (B) Extol
Directions (Qs. 151 -175): In each of the following questions (C) Eulogise (D) Euphemise
four words are given of which two words are most nearly the (a) A-B (b) B-C
same or opposite in meaning. Find the two words which are (c) B-D (d) A-D
most nearly the same or opposite in meaning. (e) C-D
151 (A) Prolixity (B) Brevity 163. (A) Recluse (B) Pandemic
(C) Agreement (D) Proposition (C) Transparent (D) Opaque
(a) A - B (b) B - C (a) A-B (b) C-D
(c) C - D (d) A - C (c) A-C (d) A-D
(e) A - D (e) B-D
152. (A) Suffuse (B) Deplete 164. (A) Diminutive (B) Intelligent
(C) Fight (D) Delay (C) Large (D) Prolific
(a) B - C (b) C - D (a) B-D (b) C-D
(c) A - C (d) A - D (c) A-C (d) A-B
(e) A - B (e) C-D
153. (A) Forensic (B) Delectable 165. (A) Enormous (B) Malign
(C) Leaf like (D) Charming (C) Absorb (D) Slander
(a) A - C (b) B - D (a) A-C (b) B-C
(c) A - D (d) A - C (c) C-D (d) B-D
(e) A - B (e) A-D
154. (A) Benevolent (B) Alarming 166. (A) Withstand (B) Climate
(C) Charitable (D) Stupendous (C) Hot (D) Surrender
(a) A - B (b) B - C (a) A-B (b) B-C
(c) C - D (d) A - C (c) A-D (d) B-D
(e) B - D (e) C-D
155 (A) Convenient (B) Intolerant 167. (A) Perky (B) Lively
(C) Enduring (D) Protestant (C) Honest (D) Kind
(a) A - B (b) A - C (a) A-B (b) B-C
(c) B - C (d) B - D (c) C-D (d) B-D
(e) C - D (e) A-C
156. (A) Eject (B) Spread 168. (A) Reverie (B) Stirring
(C) Mark (D) Spout (C) Serene (D) Fascination
(a) B - D (b) A - C (a) A-D (b) B-D
(c) B - C (d) A - B (c) C-D (d) A-B
(e) A - D (e) B-C
157 (A) Push (B) Thrive 169. (A) Pandemonium (B) Scramble
(C) Flourish (D) Arrange (C) Wriggle (D) Order
(a) A - C (b) A - D (a) A-B (b) B-C
(c) C - D (d) B - C (c) C-D (d) A-D
(e) B - D (e) A-C
158. (A) Refuse (B) Discourage 170. (A) Stimulate (B) Comprehend
(C) Lurk (D) Hide (C) Facilitate (D) Understand
(a) A - C (b) C - D (a) A-B (b) B-C
(c) B - D (d) B - C (c) A-C (d) B-D
(e) B - D (e) C-D
159. (A) Delirious (B) Confluent 171. (A) Dense (B) Graze
(C) Curt (D) Gracious (C) Pristine (D) Fresh
(a) A - B (b) B - C (a) B-C (b) C-D
(c) C - D (d) B - D (c) B-A (d) A-C
(e) A - D (e) B-D
C-20 VOCABULARY
172. (A) Enthralling (B) Respecting 174. (A) Concise (B) Elegant
(C) Projecting (D) Alluring (C) Indifferent (D) Indecorous
(a) A-B (b) B - C (a) B-C (b) A-C
(c) C-D (d) A - D (c) A-B (d) C-D
(e) B-D (e) B-D
173. (A) Swoop (B) Perturb 175. (A) Acquit (B) Defend
(C) Plunge (D) Boil (C) Forbid (D) Condemn
(a) A-D (b) B-C (a) B-C (b) A-C
(c) A-C (d) B-D (c) C-D (d) B-D
(e) C-D (e) A-D

ANSWER KEY
1 (e) 21 (c) 41 (e) 61 (a) 81 (c) 101 (d) 121 (e) 141 (b) 161 (d)
2 (d) 22 (c) 42 (b) 62 (b) 82 (e) 102 (d) 122 (d) 142 (a) 162 (b)
3 (a) 23 (b) 43 (e) 63 (d) 83 (b) 103 (e) 123 (b) 143 (a) 163 (b)
4 (c) 24 (a) 44 (c) 64 (e) 84 (b) 104 (b) 124 (c) 144 (d) 164 (c)
5 (b) 25 (c) 45 (a) 65 (e) 85 (a) 105 (d) 125 (a) 145 (c) 165 (d)
6 (d) 26 (e) 46 (d) 66 (e) 86 (c) 106 (b) 126 (b) 146 (e) 166 (c)
7 (b) 27 (a) 47 (b) 67 (c) 87 (e) 107 (a) 127 (e) 147 (b) 167 (a)
8 (a) 28 (a) 48 (e) 68 (d) 88 (e) 108 (c) 128 (a) 148 (c) 168 (e)
9 (e) 29 (b) 49 (a) 69 (c) 89 (d) 109 (b) 129 (d) 149 (c) 169 (d)
10 (c) 30 (a) 50 (c) 70 (b) 90 (c) 110 (c) 130 (b) 150 (a) 170 (d)
11 (d) 31 (c) 51 (d) 71 (e) 91 (a) 111 (d) 131 (e) 151 (a) 171 (b)
12 (b) 32 (d) 52 (a) 72 (c) 92 (c) 112 (e) 132 (c) 152 (e) 172 (d)
13 (a) 33 (d) 53 (d) 73 (c) 93 (b) 113 (d) 133 (c) 153 (b) 173 (c)
14 (e) 34 (b) 54 (c) 74 (a) 94 (a) 114 (c) 134 (e) 154 (d) 174 (e)
15 (b) 35 (c) 55 (e) 75 (b) 95 (c) 115 (a) 135 (b) 155 (c) 175 (d)
16 (c) 36 (a) 56 (b) 76 (d) 96 (d) 116 (b) 136 (d) 156 (e)
17 (a) 37 (e) 57 (e) 77 (d) 97 (e) 117 (e) 137 (a) 157 (d)
18 (d) 38 (b) 58 (a) 78 (b) 98 (e) 118 (a) 138 (d) 158 (b)
19 (e) 39 (c) 59 (c) 79 (a) 99 (a) 119 (b) 139 (e) 159 (c)
20 (b) 40 (d) 60 (d) 80 (a) 100 (b) 120 (d) 140 (e) 160 (a)
Grammar
2 Chapter

Grammar forms an important part in the English section of any For example :
competitive examination. The typical kinds of questions can be a) A deer was caught
categorized as follows: a) Fill in the blanks b) Identifying errors in b) Deer were caught.
sentences and c) Correcting the sentences. The questions can Here, the singular and plural form of the noun Deer is
be handled easily and you can score well if your basics are clear. same. Like Deer there are other nouns that have the same
Many of us can communicate well verbally but when it comes to form in singular as well as plural form. For example:
answering grammar-based questions, we commit mistakes. Grammar sheep, apparatus, species, series, hundred, dozen, hair
is not a set of rules but in reality a mere description of the language
etc. Preceding adjectives and articles decide whether the
used by all of us.
word is used in the singular form or plural form.
For example :
HOW THIS BOOK WILL HELP YOU
This book will help you to understand how language and a) He paid eight hundred rupees for this pair of shoes.
components of language work. It is oriented towards making you b) India again lost the series.
more confident users of English by giving you an insight into 3. Nouns denoting large numbers are used both in singular
correct usage. The material provided is user friendly with adequate and plural form
examples and ‘practice exercises’. For example:
If you make a concentrated effort, it will not only prepare you for a) Three hundred people attended the function.
the forthcoming competitive exams but also fine-tune your b) Hundreds of people attended the party.
communication skills. In sentence a), ‘hundred’ is preceded by number ‘three’.
READING: To supplement your efforts, you should build up So ‘hundred’ will take no plural form. Word ‘three hundred’
reading habits. This can be of any kind - magazines, newspapers indicates plurality . But in sentence b), ‘hundred’ is not
or novels. But, one should consciously look at the usage. Good preceded by any number. So to indicate plurality , we will
reading habits will definitely build up your understanding of write ‘hundreds’. So, rule is that when words like hundred,
grammatical usage and help you in being successful in competitive dozen, thousand, pair, score are not preceded by any word
exams. denoting number then they take the plural form, otherwise
not.
NOUNS Consider some more examples :
A Noun is a word used as a name of a person, place or thing. a) Coca-Cola paid lakhs of rupees to Aamir Khan for
There are five kinds of Noun – promoting their product.
(a) Proper Noun (b) Common Noun b) I brought two dozen bananas.
(c) Collective Noun (d) Abstract Noun 4. Tell which sentence is correct:
(e) Material Noun. a) Since long no news has been heard.
b) Since long no news have been heard.
FOLLOWING ARE CERTAIN RULES OF GRAMMAR Sentence a is correct. The reason is that some nouns are
REGARDING NOUNS THAT WOULD BE USEFUL IN A always used as singular though they look like plural
COMPETITIVE EXAM : nouns. That’s why we should never use the plural verb
1. Proper nouns are sometimes used as common nouns. with these words. Other similar words are politics,
For example : mathematics, physics, gallows, means, billiards, ethics,
a) Amitabh is Gandhiji of our class. (Incorrect) summons, innings.
b) Amitabh is the Gandhiji of our class, (Correct) For example :
Here Gandhiji does not mean Mahatma Gandhi. The word a) Politics is not my cup of tea.
here stands for the possessor of the qualities that Gandhiji b) I received summons.
is most known for - truth and non-violence. Thus Gandhiji c) Sachin once again played a superb innings
is being used as a metaphorical common noun. 5. Tell which sentence is correct:
a) The spectacles that you are wearing are really nice.
FOLLOWINGARE RULES REGARDING THE NUMBER OF
b) The spectacles that you are wearing is really nice.
THE NOUN :
2. Some nouns have the same form both in singular as well Sentence a) is correct. The reason being that some noun
words are always used in the plural form.
as in plural .
C-22 GRAMMAR
For example : trousers, arms, drawers, assets, scales, 9. Please go through the following singulars and plurals as
alms, thanks, cards; ashes, riches, premises, scissors, plural forms are commonly known but their singular forms
credentials, proceeds. are not commonly kown.
6. Tell which sentence is correct: Singular Form Plural form
a) The cattle was grazing in the field. Agendum Agenda
b) The cattle were grazing in the field. Alumnus Alumni
Sentence b) is correct .The reason being that some nouns Index Indices
are always used as plurals though they look like singular. Phenomenon Phenomena
Other nouns like this are public; people, folk, mankind, Criterion Criteria
poultry, sheep, police, gentry, peasantry, bulk, majority, Radius Radii
etc. Formula Formulae
For example: Memorandum Memoranda
a) The majority are with the leader. 10. Some noun words have two plurals with different
b) Police, though late, have come. meanings. So, that plural form should be selected which
c) Public wants results. will convey the right meaning.
7. Tell which sentence is correct. For example :
a) This project will lead to lots of expenditures a) I have one brother and one sister (meaning- children
sons of the same parents).
b) This project will lead to lots of expenditure.
b) Why should only select brethren be allowed to
Sentence b) is correct. The reason is that some nouns are
attend the meeting? (meaning - members of same
always used as singular. Preceding adjectives or the
society, organisation)
verb form indicates the singularity or plurality. Other
c) I took off my shoes and clothes (meaning- things
nouns are expenditure, furniture, information, machinery,
that people wear).
issue, offspring, alphabet, scenery, poetry.
d) Cotton, Nylon, Silk are different kinds of cloths
For example : (meaning- kinds or pieces of cloth).
a) All the furniture was bought last year. Other nouns having two plurals with different meanings
b) All the Information was given to him. are:
8. Meaning of some nouns in plural form is very different Singular Plural with
from the meaning of nouns in singular form. Hence, that different meaning
form should be used which will convey the right meaning. Die Dies - stamps
For example: Dice - small cubes used in games
a) I opened the letter and read its contents. Genius Geniuses-persons of great talent
b) Her mouth was fixed in a smile of pure content. Genie - spirit
c) The conflict between good and evil is ages old. Quarter Quarter - fourth part
d) We must produce goods at competitive prices. Quarter(s) - lodging
e) Delhites breathe the most polluted air in the world. Manner Manner - Method
f) She was just putting on airs when she came to visit Manners - Correct behaviour
us/me. Pain Pain - Suffering
g) We should renounce the use of force to settle our Pains - Careful efforts
dispute. Spectacle Spectacle sight
h) Families of people who died as a result of services Spectacles eye-glasses
in the forces should not be ignored. Penny Pence-indicate amount of money
i) I was very excited on my return to my home village. Pennies–number of coins
j) Early returns in the ballot indicate majority for FOLLOWING ARE RULES REGARDING GENDER OF THE
opposition. NOUN :
Other nouns having different meanings in the singular 11. Collective nouns, even when they denote living beings,
and plural form are: are considered to be of the neuter gender.
Singular with Plural with For example :
meaning meaning a) Shahrukh Khan had a herd of cows. He kept a
Advice - counsel Advises - information herdsman to look after her.
Respect - regard Respects - compliments b) Shahrukh Khan had a herd of cows. He kept a
Compass - extent Compasses - instrument or range herdsman to look after it.
Custom - habit Customs - duties levied on Sentence b) is correct. Though herd consists of cows
Ground - Earth Grounds - reasons (females), herd is not a feminine noun as it a collective
Iron - metal Irons - fetters made of iron noun.
Mean - average Means - way or method
12. Young children and the lower animals are also referred to
Respect - regard Respects - polite greetings
as of the neuter gender.
Colour - hue Colours - appearance
For example :
Physic - medicine Physics - natural science
a) The baby loves his toys. (Incorrect)
GRAMMAR C-23
b) The baby loves its toys. (correct) f) ’S is not used with inanimate or non-living things.
c) The mouse lost his tail when the cat pounced on For example: leg of the table, cover of the book.
him. (Incorrect) g) But in nouns that denote time, distance or weight, ’s
d) The mouse lost its tail when the cat pounced on it. is used. For example: a stone’s throw, in a year’s
(correct) time, the earth’s surface.
We are often uncertain regarding the gender of the h) Some other common phrases where ’s is used are to
animals. The mouse here may be a male or a female. So, his heart’s content, at his wit’s end, out of harm’s
English language prefers the easy way out: treat it as of way.
the neuter gender. i) When a noun consists of several words, the
13. When objects without life are personified they are possessive sign is attached only to the last word.
considered of For example:
(i) The masculine gender if the object is remarkable for a) The Queen’s of England reaction is important in
strength and violence. Ex. Sun, Summer, Winter, Time, the Diana episode. (Incorrect)
Death etc. b) The Queen of England’s reaction is important in
the Diana episode. (Correct)
(ii) The feminine gender if the object is remarkable for
beauty, gentleness and gracefulness. Ex: Earth, Do not be mistaken that since it is the Queen’s
Moon, Spring, Nature, Mercy etc. reaction, the ’s should come after queen. You might
think that putting it after England would make the
For example:
reaction England’s and not the Queen’s. This is
(a) The Sun came from behind the clouds and with shortsightedness. Do not see Queen and England in
her brilliance tore the veil of darkness. (Incorrect) isolation, Queen of England is one whole unit and
(b) The Sun came from behind the clouds and with the apostrophe should come at its end.
his brilliance tore the veil of darkness. (Correct) j) When two nouns are in apposition, the possessive
Convention does not see brilliance as a womanly sign is put to the latter only.
quality, but a manly one. For example :
(a) Nature offers his lap to him that seeks it. a) I am going to Stephen Hawking’s the scientist’s
(Incorrect) country. (Incorrect)
(b) Nature offers her lap to him that seeks it. b) I am going to Stephen Hawking the scientist’s
(Correct) country. (Correct)
The offering of a lap is usually the mother’s role. k) When two or more nouns show joint possession,
Hence, Nature here should be treated as a feminine the possessive sign is put to the latter only.
noun. For example:
Tell which sentence is correct. a) Amitabh and Ajitabh are Bachchanji’s sons. So
(a) The earth goes round the sun in 365 days. Can Bachchanji is Amitabh’s and Ajitabh’s father.
you calculate her speed? (Incoreect)
(b) The earth goes round the sun in 365 days. Can b) Amitabh and Ajitabh are Bachchanji’s sons. So
you calculate its speed? Bachchanji is Amitabh and Ajitabh’s father.
Sentence b is correct. The error being made here is (Correct)
that personification is being brought where it does I) When two or more nouns show separate possession,
not exist. In the above statement the earth is being the possessive sign is put with both.
treated as a body (a thing), not a person. The scientist For example.
here is not concerned with the womanly qualities of a) The audience listened to Javed and Vajpayee’s
the planet. So, neuter gender should be applied. poems. (Incorrect)
b) The audience listened to Javed’s and Vajpayee’s
FOLLOWINGARE RULES REGARDINGAPOSTROPHE : poems. (Correct)
14. Rules regarding apostrophe S (’s):
a) Singular noun: ’s is added after the word. PRONOUNS
b) Singular noun: Only an apostrophe is added when A pronoun is a word used instead of a noun
there are too many hissing sounds. For example: Now consider the following cases :
Moses’ laws, for goodness’ sake, For justice’ sake. 1. Since a pronoun is used instead of a Noun, it must be of
c) Plural nouns ending in s like boys, cows: only’ is the same number, gender and person as the noun for
added after the word which it stands. For example: Those beggars are idle.
d) Plural nouns not ending in s like men, children: ’s is They refuse to work for their living.
added after the word. 2. Please consider the following two sentences.
e) ’S is added primarily after the living things and a) After a few hearings the jury gave its verdict.
personified objects. For example: Governor’s (Pronoun ‘its is used in place of noun 'jury').
bodyguard, horse’s head, Nature’s law, Fortune’s b) The jury were divided in their opinions. (Pronoun
favourite. ‘their’ is used in place of noun ‘jury’
C-24 GRAMMAR
You must be wondering why different pronoun 'its' and two clauses joined by ‘than’ are ‘He loves you more’ and
‘their’ is used in place of the same noun ‘jury’ The reason ‘I love you’. Being a subjective case, ‘I’ should be used.
is when a pronoun stands for a collective noun it must For example:
be in the singular number and neutral gender. (Sentence a) He is taller than I (am).
a). But when collective noun conveys the idea of separate b) He loves you more than (he loves) me.
individuals comprising the whole, the pronoun standing 10. When a pronoun refers to more than one noun or
for it must be of the plural number. In sentence b, it is pronouns of different persons, it must be of the first
clear that members of the jury are not behaving as whole.
person plural in preference to the second and of the
For example :
second person plural in preference to the third.
a) The committee is reconsidering its decision.
For example :
b) The committee decided the matter without leaving
their seats. a) You and I, husband and wife, have to look after your
PRONOUNS IN SENTENCES FOUND BY CONJUNCTION : home. (Incorrect)
3. When two or more singular nouns are joined by ‘and’, the b) You and I, husband and wife, have to look after our
pronoun used for them must be plural. home. (Correct)
For example : Rama and Hari work hard. Their teachers Now, common sense tells us that if we are a couple, wife
praise them. and husband, the feeling of togetherness is expressed by
But when two Singular nouns joined by ‘and’ refer to the our home, not your home. And so does grammar
same person or thing, the pronoun should be singular. Rule: 123. I stands for first person, 2 for second person
For example : The Secretary and Treasurer is negligent and 3 for third person. The order of precedence is: 1
of his duty. before 2 and 2 before 3. In the given example, we have
Here the same person is acting as Secretary and Treasurer. 2 and 1. So I will apply; that is, first person. The number,
That’s why singular pronoun is used. of course, will be plural.
4. When two singular nouns joined by ‘and’ are preceded Let us take another example.
by ‘each’ or ‘every’, then the pronoun must be singular a) You and Hari have done their duty, (Incorrect)
For example : Every soldier and every sailor was in his
b) You and Hari have done your duty. (Correct)
place.
Applying 123 rule. You = 2 and Hari =3. So, 2. Second
5. When two or more singular nouns are joined by ‘or’,
‘either...or’, ‘neither...nor’, the pronoun is generally person plural gives ‘your’.
singular. Similarly ,when all the three persons are taken into
For example : account, it has to be I; that is, first person plural.
a) Neither Abdul nor Rehman has done his lessons. a) You, he and I have not forgotten your roots.
b) Either Rama or Hari must help his friend. (Incorrect)
6. When a plural and a singular noun are joined by ‘or’ or b) You, he and I have not forgotten our roots. (Correct)
‘nor’, the pronoun must be in the plural 11. Each, either and neither are always singular and are
For example : Either the manager or his assistants failed followed by the verb in the singular
in their duty. For example :
7. When two things which have been already mentioned are a) Neither of the accusations is true.
referred to, ‘this’ refers to the thing last mentioned and b) Each boy took his turn.
‘that to the thing first mentioned. c) Each of the ladies performs her duty well.
For example : Alcohol and Tobacco are both injurious: 12. (A) Please consider the following sentences.
this perhaps less than that. a) This is the boy. He works hard. (He subjective
RULES REGARDING PERSONAL PRONOUNS : case)
8. Tell which sentence is correct b) This is the boy. His exercise is done well. (His
a) The presents are for you and me. is possessive case)
b) The presents are for you and I. c) This is the boy. All praise him. (Him is objective
Sentence a is correct. Pronoun has to agree with the case. case)
Here it is the objective case. So, ‘me’ should be used 13. An apostrophe is never used in ‘its’, ‘yours’ and ‘theirs’.
instead of ‘I’. For example : My uncle asked my brother 14. The complement of the verb be, when it is expressed by
and me to dinner. a pronoun, should be in the nominative form.
9. Tell which sentence is correct For example.
a) It was he (not him),
a) He loves you more than I.
b) It is I (not me) that gave the prizes away.
b) He loves you more than me.
c) It might have been he (not him).
Sentence a is correct ‘Than’ is a conjunction joining 15. The case of a pronoun following than or as is determined
clauses. And the case of the pronoun to be used may be by mentally supplying the verb.
found by writing the clauses in full. So, in sentence a.)
GRAMMAR C-25
For example : 24. Who, Which, Whom, That, Whose should be placed as
a) He is taller than I (am). near to the antecedent as possible.
b) I like you better than he (likes you). For example :
c) They gave him as much as (they gave) me. a) I with my family reside in Delhi, which consists of
16. A pronoun must agree with its Antecedent in person, my wife and parents.
number and gender. This sentence is wrong as which relates to ‘my’
For example: family’. So ‘which’ should be placed as near to family’
a) All passengers must show their (not his) tickets. as possible. So, the correct sentence is
b) I am not one of those who believe everything they b) I with my family which, consists of my wife and
(not I) hear parents, reside in Delhi.
RULES REGARDING DEMONSTRATIVE PRONOUNS : 25. Who is used In the nominative cases and whom in the
17. That is used objective cases.
A. After adjectives in the superlative degree. For example :
For example. a) There is Mr. Dutt, who (not whom) they say is the
a) This is the best that we can do. best painter in the town.
b) He is the best speaker that we ever heard. b) The Student, whom (not who) you thought so highly
B. After the words all, same, any, none, nothing, only. of, has failed to win the first prize.
For example: 26. When the subject of a verb is a relative pronoun, the
a) Man is the only animal that can talk. verb should agree in number and person with the
b) He is the same man that he has been. antecedent of the relative.
C. After two antecedents, one denoting a person and For example :
the other denoting an animal or a thing. a) This is one of the most interesting novels that have
For example : The man and his pet that met with an (not has) appeared this year. (Here, antecedent of
accident yesterday died today. relative pronoun that is novels and not one)
18. What and That refer to persons as well as things. b) This is the only one of his poems that is (not are)
worth reading. (Here the antecedent of that is one
RULES REGARDING RELATIVE PRONOUNS :
and not poems. Kindly note the difference between
19. On combining each of the above pairs into one sentence
a) This is the boy who works hard (Who in place sentence a and b)
of He) OTHER USEFUL RULES :
b) This is the boy whose exercise is done well. 27. None is used in the singular or plural as the sense may
(whose in place of His) require.
c) This is the boy whom all praise. (Whom in place For example:
of Him) a) Each boy was accompanied by an adult but there
The above sentences show when to use who, whose and were none, with the orphan (Incorrect)
whom. Who is the subjective case, Whose the possessive b) Each boy was accompanied by an adult but there
case and Whom the objective case. was none with the orphan. (Correct)
20. Who is used for persons only. It may refer to a singular c) I am used to many guests everyday but there was
or plural noun. none today. (Incorrect)
For example : d) I am used to many guests everyday but there were
a) He who hesitates is lost.
none today. (Correct)
b) Blessed is he who has found his work.
28. When ‘one’ is used as pronoun, its possessive form ‘one’s’
21. Whose can be used for persons as well as things without
should follow instead of his, her etc.
life also.
For example : One must put one’s best efforts if one
For example :
wishes to succeed.
a) This is the hotel whose owner is a criminal.
29. With let objective case of the pronoun is used.
b) This is the person whose will power is extraordinary.
For example : let you and me do it.
22. Which is used for inanimate things and animals. Which
30. If a pronoun has two antecedents, it should agree with
is used for both singular as well as plural noun.
the nearer one.
For example :
For example :
a) I have found the book which I had lost last week.
a) I hold in high esteem everything and everybody who
b) The horse, which won the race yesterday, is my
reminds me of my failures.
favourite.
b) I hold in high esteem everybody and everything,
23. When ‘which’ is used for selection, it may refer to a
which reminds me of my failures.
person as well as things.
31. In referring to anybody, everybody, anyone, each etc., the
For example :
pronoun of the masculine or the feminine gender is used
a) Which of the packets is yours?
according to the context.
b) Which of the boys has not done his homework?
C-26 GRAMMAR
For example. 3. The simple Present tense is used
a) I shall be glad to help everyone of my boys in his A. To express a habitual action.
studies. For example : I get up every day at five o’clock.
b) I shall be glad to help everyone of my girls in her B. To express general truths.
studies. For example : Fortune favours the brave.
c) I shall be glad to help everyone of my students in C. In vivid narrative, as substitute for the simple past.
his studies. For example : Immediately the Sultan hurries to his
But when gender is not determined, the pronoun of capital.
the masculine gender is used as in sentence c. D. To indicate a future event that is part of a fixed
32. (A) The pronoun one should be used throughout, if programme or time table.
used at all. For example : The train leaves at 5:20 am.
For example: Note: We can also use will leave in place of leaves.
a) One must use one’s best efforts if one wishes E. It is used to introduce quotations.
to succeed. For example : Keats says, ‘A thing of beauty is a joy
b) One should be careful about what one says. forever’.
(B) Plural is commonly used with none. F. In exclamatory’ sentences beginning with here and
For example. there to express what is actually taking place in the
a) None of his poems are well known. present.
b) None of these words are now current. For example : Here comes the bus!
33. Anyone should be used when more than two persons or G. When two actions of the future are being talked
things are spoken of. about, one dependent on the other, the former action
For example : She was taller than anyone of her five is represented by present simple and the latter by
sisters. future simple.
For example : We shall go when the child comes
TENSES back home.
1. Tense is the form taken by a verb to indicate time and 4. The present continuous tense is used
continuance or completeness of action. The continuance I) For an action going on at the time of speaking.
or completen ess of action is den oted by four For example : The boys are playing cricket in the
subcategories. ground.
a) Simple Tense :It is used for habitual or routine II) For a temporary action that may not be actually
actions in the Present Tense, action which is over in happening at the time of speaking but was happening
the Past Tense & action to happen in the Future in the recent past and is still happening in recent future.
Tense. For example : I am reading Sidney Sheldon now a
b) Continuous Tense : The action is incomplete or days.
continuous or going on. III) To express changing or developing situations.
c) Perfect Tense : The action is complete, finished or For example : India is progressing day by day.
perfect with respect to a certain point of time. IV) For an action that is planned or arranged to take
d) Perfect Continuous Tense : The action is going on place in the near future.
continuously over a long period of time and is yet For example : I am going to cinema tonight.
to be finished. Note: But it is not good to use the present continuous
2. The different tenses and the verb forms used in each for slightly distant future. So, don’t say
tense are given below :
a) I am going to cinema next week.
Singular with meaning Plural with meaning Rather, use the future simple. So, it is better if
Name of Tenses Verb form used in Tenses you say
Present simple / indefinite Verb / verb + s/es
b) I will go to cinema next week.
Present continuous / Is/am/are + verb + ing
V) When the reference is to a particularly obstinate
Progressive
habit, the present continuous is used instead of
Present perfect Has / have + third form of verb
present simple. An adverb like always, continually,
Present perfect continuous Has/have + been + verb + ing
constantly is also used.
Past simple / indefinite Second form of verb only For example : It is no use scolding him; he always
Past continuous / Progressive Was/were + verb + ing
does what is forbidden. (Incorrect)
Past perfect Had + third form of verb Note: that his doing what is forbidden has become
Past perfect continuous Had been + verb + ing a die-hard habit. The habit persists in spite of advice
Future simple / indefinite Shall / will + verb or warning. So, we should use the present
Future continuous / Shall / will + be + verb + ing continuous.
Progressive For example : It is no use scolding him; he is always
Future perfect Shall/will + Have + past participle
doing what is forbidden. (Correct)
Future perfect continuous Shall/will + have been + verb + ing
GRAMMAR C-27
5. The present perfect tense is used For example:
I) To indicate the completed activities in the immediate a) The steamer sailed yesterday.
past. b) He went home some time back.
For example : He has just gone out. II) To express imaginary present situations or imaginary
II) Action completed in the immediate past or an action future events that may not happen.
of the past whose effect lingers in the present. For example :
For example : I wrote three books. (Incorrect) a) If I had longer holidays, I would be very happy.
b) If I got rich, I would travel all over the world.
The given sentence appears to be incomplete. The
III) When this tense is used without an adverb of time,
reader of the sentence immediately queries. ‘When
then time may be either implied or indicated by the
did you write three books?” It would be a different
context.
case if you said For example : I didn’t sleep well. (i.e., last night)
For example : I wrote books. IV) For past habits ‘used to’ is added to the verb.
Then the reader would infer that you wrote books in For example : She used to carry an umbrella.
the past as a profession or hobby. But when you are 8. The past continuous tense is used
being so specific as to say “three books”, we I) To denote an action going on at some time in the
immediately feel the need of a time frame. Since no past. The time of the action may or may not be
time frame is mentioned, we assume it to be ‘by indicated.
now’. So, we have something to the effect. For example :
For example : I have written three books by now. a) It was getting darker.
This ‘by now’ is implied and need not be written. So, b) We were listening to the radio all evening.
For example : I have written three books. (Correct) II) When a new action happened in the middle of a
III) The present perfect is never used with adverbs of longer action. In this case Past simple and Past
the past time. In such cases the past simple should continuous are used together. Past simple is used
be used. for the new action.
For example : India has won the match last week For example : The Light went out while I was reading.
(Incorrect) III) For persistent habits in the past.
For example : She was always chewing gum.
“Last week” is not immediate past. You may therefore
9. The past perfect tense is used when two actions happened
be tempted to use the present perfect. But remember
in the past. In this case it is necessary to show which
that the immediate past here does not go unindicated. action happened earlier than the other. Here past perfect
Last week is being used as an adverb of past time. is used for the action, which happened earlier.
So, For example :
For example : India won the match last week. a) When I reached the station the train had started.
(Correct) b) I had done my exercise when Hari came to see me.
IV) To express past actions whose time is not given and 10. If past perfect continuous tense if when an action that
not definite - actions with their effect continuing in began before a certain point of time in the past & was
the present. continuing at the given point of time in the sentence. A
For example : time expression like since last year, for the last few days
a) I have never known him to be angry. is generally put after perfect continuous tense.
b) Have you read ‘Gulliver’s Travels’? For example : At that time he had been writing a novel
V) To describe the past events when we think more of for two months.
their effect in the present than of the action itself. Here, At that time is the given point of time and for two
For example : I have cut my finger. months is the point of time in the past.
VI) For long actions and situations which started in the 11. The simple future is used for an action that has still to
near past and went on until very recently. take place.
For example : I have read three chapters since this For example :
morning. a) I shall see him tomorrow.
6. The present perfect continuous tense is used for an b) Tomorrow will be Sunday.
action, which began at some time in the past and is still 12. The future continuous tense
continuing. With the present perfect continuous tense an I) Represents an action as going on at sometime in the
adverb or phrase that expresses time is used. future.
For example : For example : I shall be reading the paper then.
a) I have been reading this book since morning. II) Represents the future events that are planned.
b) They have been building the bridge for several For example : He will be meeting us next week.
months. 13. The future perfect tense is used to indicate the completion
7. The simple past tense is used of an event by a certain future time.
I) To indicate an action completed in the past. Generally, For example : I shall have written my exercise by that
adverbs or adverb phrases of past time are used in time.
the past simple tense.
C-28 GRAMMAR
14. The future perfect continuous tense indicates an action 5. A and an are used with words ‘few’ and ‘little’ if they
represented as being in progress over a period of time refer to a small number or a small amount. Words ‘few’
that will end in the future. Generally time period is and ‘little’ without the articles means almost none.
mentioned along with it. For example:
For example : By next July we shall have been living here a) We have little time to spare. (means almost no time)
for four years. b) We have a little time to spare. (means some time)
15. Other rules to be followed : Events occurring at the same c) Few persons were present at the meeting. (means
time must be given in the same tense. almost no one was present)
For example : When he fainted his brother was with him. d) A few persons were present at the meeting. (means
16. Will or Shall cannot be used twice in the same sentence some were present)
even if both the actions refer to the future tense. 6. A is used in the following senses :
For example : A) In its original numerical sense of one.
a) I shall come if he will call me. (Wrong)
For example:
b) I shall come if he calls me. (Right)
a) Not a word was said.
17. With the phrases as if and as though the past tense and
plural form of the verb should be used. b) A word to the wise is sufficient.
B) In the vague sense of a certain time.
For example :
C) In the sense of any, to single out an individual as
a) He behaves as if he is a king. (Wrong)
the representative of a class.
b) He behaves as if he were a king. (Right)
For example : A pupil should obey his teacher.
18. With the word ‘wish’ four verbs are used namely were, D) To make a common noun of a proper noun.
had, could, would. ‘Were’ is used when the wish seems
For example : A Daniel came to judgement. (A Daniel
to be unrealisable.
= A very wise man)
For example : I wish I were a king. 7. The points out a particular person or thing or someone
‘Had’ is used when our wish is a lament over the past or something already referred to.
happening. For example : I wish I had accepted that job. For example :
‘Would’ is used when we refer to the future. For example
a) I saw the doctor. (means I saw some particular doctor)
: I wish I would get a ticket.
b) The book you want is out of print.
‘Could’ is used when we wish that something that has
8. The is used with names of gulfs, rivers, seas, oceans,
happened already should have happened otherwise.
groups of islands and mountain ranges.
For example : He did not go because he was busy For example :
yesterday. I wish he could go with you.
The Persian Gulf, The Red Sea, The Indian Ocean, The
19. ‘For’ is used for a period of time. For example : British Isles, The Alps.
He has been working for two hours.
9. The is used before the name of certain books.
‘Since’ is used with a point of time. For example : He For example : The Vedas, The Puranas, The Ramayana.
has been working since morning. But we never say ‘The Valmiki’s Ramayana’. The is not
20. In case of conditional sentences ‘had’ and ‘would have’ used when the name of a book is mentioned along with
are used. the author’s name. So, ‘Valmiki’s Ramayana’ is correct.
For example : If I had met him I would have invited him. 10. The is used before the names of things unique of their
ARTICLES kind.
For example : the sun, the sky, the ocean, the sea.
1. A or an does not refer to a particular person or thing. It 11. The is used before a plural common noun if it refers to
leaves indefinite the person or thing spoken of. a particular group among the class and not the whole
For example : I saw a doctor. (means I saw any doctor) class.
2. An is used before a word beginning with vowel sound For example : Drive away the cows from the field.
(please note a word beginning with vowel sound and not 12. The is used before a proper noun only when it is qualified
necessarily a vowel itself).
by an adjective.
For example : an ass, an enemy, an inkstand, an orange,
For example : The great Rani of Jhansi, the immortal
an umbrella, an hour.
Kalidas.
3. An is placed before an abbreviation if the first letter of an
13. The is used before superlatives.
abbreviation is F, H. L, M, N, R, S or X.
For example :
For example :
a) An MBA was required for the post. a) Sachin is the best batsman in the world today.
b) An SAO is an officer of high rank b) The best person should win.
4. A is used before a word beginning with a consonant 14. The noun if emphasis is laid on the use of such a noun.
sound. Here, noun can be proper or abstract noun
For example : a boy, a woman a horse, a one-rupee note, a) the time for doing it.
a university, a European (both university and European b) occasion to help the distressed.
begin with a consonant sound of ‘yu’ )
GRAMMAR C-29
15. The is used with ordinals. 23. No article is used before abstract nouns.
For example : For example :
a) He was the first student to finish his homework. a) Wisdom is the gift of heaven.
b) The second chapter of the book is very interesting. b) Honesty is the best policy.
16. The is used before an adjective when the noun is But consider the following examples where an article
understood. is used before an abstract noun.
For example : a) The wisdom of Solomon is famous.
a) The poor are always with us. (Here poor mean poor b) I cannot forget the kindness with which he treated
people, which is understood.)
me.
b) The weak and the strong. (Here weak means weak
Here the article is used before the abstract noun as
people and strong means strong people.)
the abstract noun has been qualified by an adjective
17. No article is used before a common noun when it refers
to all the members of the class. or adjectival clause.
For example : 24. No article is used before languages, subject of arts and
a) Man is mortal. science.
b) Fish has high protein content. For example :
c) What kind of flower is it? a) We are studying English.
18. The is used before a common noun to give it the meaning b) Geometry is the toughest subject I have ever studied.
of an abstract noun. 25. No article is used before words such as school, college,
For example : The devil in him begins its misdeeds now church, bed, table, hospital, market, prison.
and then. For example :
19. No article is used before the names of materials such as a) I went to school till last year.
gold, stone, wine, iron, wheat, wood, cloth. b) I have never been to hospital.
For example : But an article is used before these words when
a) Gold is a precious metal. reference is made to a definite place.
b) Wheat grows in Uttar Pardesh, Haryana and Madhya 26. No article is used before the name of relations like father,
Pardesh.
mother, aunt, uncle.
c) Iron is a useful metal.
For example : Mother would like to see you.
Note: But it is correct to say
But if someone else’s mother is being talked about then
For example : An iron is a useful gadget.
the should be used.
Because here we are not taking about material iron, but
the object which is used to make clothes smooth. For example : The mother would like to see you.
20. No article is used before proper nouns. 27. Article should not be used before positions that are held
For example : at one time by one person only.
For example :
a) Delhi is the capital of India.
a) S D Sharma was elected the president of the country.
b) Newton was a great philosopher.
(Incorrect)
But consider the following examples where an article b) S D Sharma was elected president of the country.
is used before a proper noun. (Correct)
a) This man is a second Newton. 28. Please consider this sentence
b) Bombay is the Manchester of India. a) I have a black and white cat.
Here Newton and Manchester are not used as proper Here I mean that I have one cat that is partly black and
nouns but as common nouns. The first sentence means partly white.
that this man is as great as Newton and the second Now, consider this sentence
sentence means that Bombay is a great manufacturing For example : I have a black and a white cat.
city like Manchester. Here I mean that I have two cats one is black and the
21. No articles are used before a common noun used in its other white. Hence the rule is that when two or more
widest sense. adjectives qualify the same noun, the article is used before
For example : the first adjective only. But when they qualify different
a) The science has developed much in the past hundred nouns, the article is used before each adjective separately.
years. (Incorrect) Consider one more example.
b) Science has developed much in the past hundred a) The President and Chairman is absent.
years. (Correct). b) The President and the Chairman are present.
22. No article is used before the noun following ‘Kind of’: Sentence a means that only one person is acting as
For example : president as well as chairman. Sentence b means that two
a) What kind of a hobby is this? (Incorrect) different persons are acting as the President and the
b) What kind of hobby is this? (Correct) Chairman and both the persons are present.
C-30 GRAMMAR

ADJECTIVES But some is an exception to the above rule. Some is used


in interrogative sentences, which are commands or
Adjectives are the words that describe the qualities of a noun requests.
or pronoun in a given sentence. For example : Will you please lend me some money?
CONSIDER THE FOLLOWING : 7. Few is used for countable objects and little is used for
1. Tell which sentence is correct non-countable objects.
a) Flowers are plucked freshly. 8. Little means not much. So use of the word little has a
b) Flowers are plucked fresh. negative meaning.
Sentence b is correct as, adjective is correctly used with For example :
a verb when some quality of the subject rather than verb a) There is little hope of his recovery.
is to be expressed. Here, fresh describes the word Flowers b) He has little appreciation of hard work.
(a noun) and not plucked (a verb). A little means some though not much. So, use of a
little has a positive meaning.
RULES REGARDING DEMONSTRATIVE ADJECTIVE :
For example :
2. This and that are used with the singular nouns and these
a) There is a little hope of his recovery.
and those are used with plural nouns
b) He has a little appreciation of hard work.
For example :
The little means not much but all there is.
a) This mango is sour.
b) These mangoes are sour. For example :
c) That boy is industrious a) The little information he had was quite reliable.
d) Those boys are industrious. b) The little knowledge of management he possessed
3. This and these indicate something near to the speaker was not sufficient to stand him in good stead.
while that and those indicate something distant to the 9. Few mean not many. So use of the word few has a
speaker. negative meaning.
For example : For example : Few men are free from faults.
a) This girl sings. A few means some. So use of ‘a few’ has a positive
b) These girls sing. meaning.
c) That girl sings. For example : A few men are free from faults.
d) Those girls sing. The few means not many, but all there are.
For example : The few remarks that he made were very
RULES REGARDING DISTRIBUTIVEADJECTIVES: good.
4. Each is used when reference is made to the individuals 10. Only uncountable nouns follow much, little, some,
forming any group. Each is also used when the number enough, sufficient and whole.
of the group is limited and definite. For example :
For example : a) I ate some rice.
a) I was in Shimla for five days and it rained each day. b) There are not enough spoons.
Every is used when reference is made to total group or
when the number is indefinite. RULES REGARDING INTERROGATIVE ADJECTIVES :
For example : 11. What is used in the general sense and which is used in
a selective sense.
a) Every seat was taken.
For example :
b) I go for a movie every week.
a) Which of you haven’t brought your book?
c) Leap year falls in every fourth year.
b) What manner of man is he?
5. Each, either, neither and every are always followed by the
singular noun. RULES REGARDING DEGREES OF COMPARISON OF
For example : ADJECTIVES :
a) Each boy must take his turn. 12. The comparative form ending in ‘er’ is used when we are
b) Every word of it is false. comparing one quality in two persons.
c) Neither accusation is true. For example : Anjali is wiser than Rahul.
But if we wish to compare two qualities in the same
RULES REGARDINGADJECTIVES OFQUANTITY:
person then the comparative form ending in ‘er’ is not
6. Some is used in affirmative sentences to express quantity
used.
or degree.
For example : Anjali is wise than brave.
For example : I shall buy some bananas.
13. When two objects are compared with each other, the
Any is used in the negative or interrogative sentences to
latter term of comparison must exclude the former.
express quantity or degree.
For example :
For example :
a) Delhi is bigger than any other city in India.
a) I shall not buy any bananas.
If we say
b) Have you bought any bananas?
b) Delhi is bigger than any city in India.
GRAMMAR C-31
Then we are saying Delhi is bigger than Delhi, as any city For example :
in India includes Delhi also. And this is obviously wrong. a) Ramesh is stronger of the two boys.
14. In a comparison by means of a superlative the latter term 21. When 'than' or 'as' is followed by the third person pronoun,
should include the former. the verb is to be repeated.
For example : For example : Ram is not as clever as his brother is.
a) Delhi is the biggest of all cities in India. 22. When 'than' or 'as' is followed by first or second person
b) Of all men he is the strongest. pronoun, the verb can be omitted.
Kindly note the difference in this and the previous rule. For example : He is more intelligent than you.
15. Later and latest refer to time. 23. In comparing two things or classes of things the
For example : comparative should be used.
a) He came later than I expected. For example :
b) This is the latest news. a) Of two evils choose the lesser (not least).
Latter and last refer to position. b) Which is the better (not best) of the two?
For example : 24. A very common form of error is exemplified in the following
a) The last player could not bat as he was injured. sentence.
b) The latter chapters are very interesting.
a) The population of London is greater than any town
Latter is used when there are two only, last when there
in India.
are more than two.
b) The population of London is greater than that of any
For example :
town in India.
a) Of Manohar, Syam and Joshi, the latter is a driver.
Sentence b is correct as the comparison is between the
(Incorrect)
population of London and the population of any town in
b) Of Manohor, Syam and Joshi, the last is a driver.
(Correct) India.
16. Elder and eldest are used only of persons (usually 25. Double comparatives and superlatives should be avoided.
members of the same family). For example :
For example : a) Seldom had the little town seen a more costlier funeral.
a) My elder sister is doing MBA from IIM Ahemdabad (Wrong)
b) My eldest brother is getting married today. b) Seldom had the little town seen a costlier funeral.
Older and oldest are used of both persons and things. (Right)
For example : c) Seldom had the little town seen a more costly funeral.
a) This is the oldest building in the city. (Right)
b) Anthony is the oldest boy in the class. 26. Preferable has the force of comparative and is followed
17. Further means more distant or advanced whereas farther by to. Phrase ‘more preferable’ should not be used.
is a variation of further and means at a distance – both For example :
the words can be used to indicate physical distance. a) Coffee is more preferable to tea. (Wrong)
For example : b) Coffee is preferable to tea. (Right)
a) No one discussed the topic further. 27. Less refers to quantity whereas fewer refers to number.
b) Calcutta is farther from the equator than Colombo. For example :
18. The comparative degree is generally followed by ‘than’, a) No fewer than fifty miners were killed in the explosion.
but comparative adjectives ending in 'is' or 'are' are b) We do not sell less than ten kg of tea.
followed by the preposition ‘to’. 28. Certain adjectives do not really admit of comparison
For example : because their meaning is already superlative. Such words
a) Akshay is inferior to Aamir in intelligence. are unique, ideal, perfect, complete, universal, entire,
b) Aamir is superior to Akshay in intelligence. extreme, chief, full, square, round. Therefore phrases like
c) He is junior to me. most unique, more round, fullest, chiefest etc. are wrong.
d) Who was captain prior to Azhar ? 29. If there is a gradual increase it is generally expressed with
19. Adjectives such as square, round, perfect, eternal, two comparatives and not with positives.
universal, unique do not admit of different degrees. So For example :
they cannot be compared. Thus strictly speaking we cannot a) It grew hot and hot. (Incorrect)
say that a thing is more square more round or more b) It grew hotter and hotter. (Correct)
perfect. But sometimes we do make exceptions to this
rule. OTHER COMMON RULES :
30. ‘Verbal’ means ‘of or pertaining to words’ whereas ‘oral’
For example : This is the most perfect specimen I have
seen. means ‘delivered by word of mouth or not written’. Hence
20. When the comparative form is used to express selection the opposite of written is oral, not verbal.
from two of the same kind or class, it is followed by ‘of’ For example :
and preceded by ‘the’. a) His written statement differs in several important
respects from his oral (not verbal) statement
C-32 GRAMMAR
b) The boy was sent with a verbal message to the 6. Following examples exemplify the common mistakes
doctor. committed:
31. ‘Common’ means shared by all concerned. If a fact is a a) His diet was abstemious, his prayers long and fervent.
common Knowledge, it means the knowledge of the fact (Wrong as subjects are not in the same number.)
is shared by all. Everyone knows about it. ‘Mutual’ means b) His diet was abstemious, his prayers were long and
in relation to each other. If you and I are mutual admirers,
fervent. (Right)
it means I admire you and you admire me. We might also
have a common admirer who admires both of us. c) He never has and never will, take such strong
a) We started smoking on the advice of a mutual friend. measures. (Wrong)
(Incorrect) d) He never has taken, and never will take, such strong
b) We started smoking on the advice of a common measures. (Right)
friend (Correct) e) Ten new members have been enrolled and seven
It is apparent that there are two or more than two of us. resigned (Wrong)
Apart from us, there is a person (friend). Since he is a f) Ten new members have been enrolled and seven
friend to all of us, this friend is being shared by all of us. have resigned. (Right)
So, he is a common friend. Now, look at this sentence. g) Being a very hot day, I remained in my tent. (Wrong
For example : We started smoking on mutual advice.
as participle being is referring to none)
It means I advised, you to smoke and you advised me to
smoke. h) It being a very hot day, I remained in my tent. (Right)
i) Sitting on the gate, a scorpion stung him. (Wrong as
OTHER COMMON ERRORS : participle sitting is not referring to any word)
32. Other common errors. j) While he was sitting on the gate, a scorpion stung
a) These Kind of questions is often asked in the him (Right)
examinations. (Incorrect)
7. The verb lay (lay, laid, laid) is transitive and is always
b) This kind of question is often asked in the followed by an object. The verb lie (lie, lay, lain) is
examinations. (Correct) intransitive and cannot have an object.
c) He is as good if not better than his brother. (Incorrect) For example :
d) He is as good as if not better than his brother.
a) Lay the child to sleep.
(Correct)
b) Let me lie here.
e) The future do not hold much for you. (Incorrect)
c) I laid the book on the table.
f) The future does not hold much for you. (Correct)
AGREEMENT OF THE SUBJECT WITH THE VERB:
VERBS 1. A verb must agree with its subject in number and person.
Often due to “Error of Proximity” the verb is made to
1. Two or more singular subjects connected by ‘and’ usually
agree in number with a noun near it instead of with its
take a verb in the plural.
proper subject.
For example : Hari and Rama are there.
For example :
2. If two singular nouns refer to the same person or thing,
the verb must be singular. a) The quality of the mangoes were not good. (Wrong
For example : My friend and benefactor has come. since subject is quality, a singular and not mangoes.)
3. If two subjects together express one idea, the verb may b) The quality of the mangoes was not good (Right).
be in the singular. c) His knowledge of Indian vernaculars are far beyond
For example : The horse and carriage is at the door. the common. (Wrong)
4. Two or more singular subjects connected by ‘or’, ‘nor’, d) His knowledge of Indian vernaculars is far beyond
either... or, neither...nor take a verb in the singular. the common. (Right)
For example : Neither he nor I was there. 2. Verb should be singular even when some words are joined
But when subjects joined by ‘or’, ‘nor’ are of different to a singular subject by ‘with’, ‘as well as’ etc,
numbers, the verb must be plural, and the plural subject For example :
must be placed next to the verb. a) The chairman, with the directors, is to be present.
For example : Rama and his brothers have done this. b) Silver, as well as cotton, has fallen in prices.
When the subjects joined by ‘or’, ‘nor’ are of different 3. Two or more singular subjects connected by ‘or’, ‘nor’
persons, the verb agrees in person with the nearest one. require singular verb.
For example : For example :
a) Either he or I am mistaken. a) No nook or corner was left unexplored.
b) Neither you nor he is to blame. b) Our happiness or our sorrow is largely due to our
5. When words are joined to a singular subject by ‘with’, own actions.
‘together with’, ‘in addition to’, ‘as well as’, then also 4. If two singular nouns express one idea, the verb is in the
number of the verb remains singular. singular.
For example : The Chief with all his men, was massacred.
GRAMMAR C-33
For example : b) Six miles is a long distance.
a) Bread and Butter are essential for one’s life. c) Fifty thousand rupees is a large sum.
(Incorrect)
b) Bread and Butter is essential for one’s life. (Correct) ADVERBS
5. Either, neither, each, everyone, many a must be followed A word that modifies the meaning of a verb is called an Adverb.
by a singular verb. SOME IMPORTANT RULES :
For example : 1. Adverbs of manner such is well, fast, quickly, carefully,
a) Neither of the two men was very strong. calmly etc. are placed after the verb if there is no object
b) Every one of the prisons is full. and after the object if there is one.
c) Many a man has done so. For example :
a) It is raining heavily.
d) He asked whether either of the applicants was
b) She speaks English well.
suitable.
2. Adverbs of time such as always, often, sometimes, never,
6. When the subjects joined by ‘or’, ‘nor’ are of different generally, ever, merely, seldom etc. are placed before the
numbers, the verb must be plural, and the plural must be verb they qualify.
placed next to the verb. For example :
For example : a) I seldom meet him. (Right)
a) Neither Rekha nor her fiends was present at the b) I meet him seldom. (Wrong)
party. (Incorrect) Adverbs of degree refer to words which show “how much”, “in
b) Neither Rekha nor her friends were present at the what degree” or “to what extent” does the action takes place.
party. (Correct) CONSIDER THE FOLLOWING :
7. When a plural noun denotes some specific quantity or 3. Meaning of too is more than enough. Too denotes some
amount considered as a whole, the verb is generally kind of excess.
singular. For example :
For example : a) He is too weak to walk.
a) Five hours are too short a time to judge one’s b) It is never too late.
character. (Incorrect) Hence, use of very in place of too is wrong.
b) Five hours is too short a time to judge one’s character. For example : Instead of saying that
(Correct) a) Cow’s milk is too nutritious
This is so because five hours is considered as one chunk. We should say that
b) Cow’s milk is very nutritious.
8. Two nouns qualified by each or every, even though
4. Enough is placed after the word it qualifies.
connected by ‘and’ require a singular verb.
For example : Everyone should be strong enough to
For example : Every boy and every girl was given a packet support one’s family.
of sweets. It will be wrong if we write ‘Everyone should be enough
9. ‘None’ though singular commonly takes a plural verb. strong to support one’s family’.
For example : None are so deaf as those who will not 5. Much is used with past participles.
hear. For example :
10. Tell which sentence is correct. a) He was much disgusted with his life.
a) Put in to bat first, a huge total was expected from b) The news was much surprising.
India. Very is used with present participles.
b) Put in to bat first, India was expected to pile up a For example :
huge total. a) He is very disgusted with his life.
Now: who has been put in to bat first? A huge total of b) The news is very surpising.
India? Common sense tells us it must be India. But the 6. When very and much are used to qualify superlative form
sentence a, as it stands, appears otherwise. So, sentence of adjectives/adverbs, they should be put before the word
‘very’ and after the word ‘much’.
b is correct.
For example :
a) Being a rainy day, I decided to take my umbrella.
a) Rim is the very best boy in his class.
b) It being a rainy day, I decided to take my umbreilla. b) Rim is much the best boy in his class.
The sentence a, as it stands, gives us the impression that Adverbs of Affirmation or Negation refer to words that
being a rainy day qualifies I. This is simply not true. I am assert the action emphatically.
not a rainy day. So sentence b is correct. Consider these examples :
11. When a plural noun denotes some specific quantity or (a) He certainly was a winner among them
amount considered as a whole, the verb is generally (b) Luckily he survived the crash
singular. CONSIDER THE FOLLOWING :
For example : 7. No sooner should always be followed by than.
a) One hundred paise is equal to one rupee. For example :
C-34 GRAMMAR
a) No sooner I saw him I trembled with fear. (Wrong) 5. With often denotes the instrument and by the agent.
b) No sooner did I see him than I trembled with fear. For example :
(Right) a) He killed two birds with one shot.
8. Not should not be used with the words which have negative b) He was stabbed by a lunatic with a dagger.
meaning if we want the sentence to be negative. 6. Since is used before a noun or phrase denoting some
For example : point of time and is preceded by a verb in the perfect
a) I received no letter neither from him nor from her. tense.
(Wrong) For example :
b) I received letter neither from him nor from her. (Right) a) I have eaten nothing since yesterday.
9. Of course is used to denote a natural consequence. It b) He has been ill since Monday last.
should not be used in place of certainly, undoubtedly. From is also used before a noun or phrase denoting some
For example : point of time but is used with non-perfect tense.
a) Of course he is the best player. (Wrong) For example :
b) He is certainly the best player. (Right) a) I commenced work from 1st January.
FOLLOWING ARE COMMON RULES OF ADVERBS IN b) He will join school from tomorrow.
GENERAL: For is used with a period of time.
10. Only is used before the word it qualifies. For example :
For example : a) He has been ill for five days.
a) Only I spoke to him. b) He lived in Bombay for five years.
b) I only spoke to him. 7. Use of in before a period of time means at the end of
c) I spoke to him only. period, but use of within before a period of time means
11. Else is followed by but and not by than. before the end of the period.
For example : It is nothing else but hypocrisy. For example :
12. ‘As’ is often used in a sentence though there is no need a) I shall return in an hour. (means I shall return at the
for it. For example : end of an hour).
b) I shall return within an hour. (means I shall return
a) He is elected as the President. (Wrong)
before the end of an hour).
b) He is elected President. (Right)
8. Scarcely should be followed by when and not by but.
13. ‘Perhaps’ means possibly whereas ‘probably’ means most
For example : Scarcely had he gone, when (not than) a
likely. For example :
policeman knocked at the door.
a) Where is Govinda? Perhaps he is not here. (Wrong)
9. The phrase ‘seldom or ever’ is wrong ‘Seldom or never’
b) Where is Govinda? Probably he is not here. (Right)
is right.
PREPOSITIONS For example : Such goods are made for export, and are
seldom or never used in this country.
1. In is used with the names of countries and large towns; 10. Examine the following sentence
at is used when speaking of small towns and villages. a) This is as good, if not better than that. (Wrong)
For example : b) This is as good as, if not better than, that. (Right)
a) I live in Delhi. c) This is as good as that, if not better. (Right)
b) I live at Rohini in Delhi. 11. Beside means at the side of while besides means in addition
2. In and at are used in speaking of things at rest; to and to. For example :
into are used in speaking of things in motion. a) Beside the ungathered rice he lay.
For example : b) Besides being fined, he was sentenced to a term of
a) He is in bed. imprisonment.
b) He is at the top of the class. 12. Above and Below merely denote position While over and
c) He ran to school under also carry a sense of covering or movement.
a) The bird flew above the lake. (Wrong)
d) He jumped into the river.
b) The bird flew over the lake. (Correct)
e) The snake crawled into its hole.
Here over is used to denote upward position and
3. On is often used in speaking of things at rest; and upon movement also.
for the things in motion. For example: 13. During is used when reference is made to the time within
a) He sat on a chair. which something happens. For is used when we are talking
b) The cat sprang upon the table. about how long something lasts.
4. Till is used for time and to is used for place. a) There are few incidents of irregularity for the
For example : emergency years. (Wrong)
a) He slept till eight o’clock. b) There are few incidents of irregularity during the
b) He walked to the end of the street. emergency years. (Correct)
GRAMMAR C-35
14. Compare is followed by to when it shows that two things B) To express a purpose and is equivalent to in order
are alike. It is followed by with when we look at the ways that.
in which two things are like and unlike each other. For For example : He kept quiet that the dispute might
example : cease.
a) Sanath Jayasuria’s batting may be compared to the C) To express a consequence, result or effect.
sales of a useful book, they score right from the For example : He bled so profusely that he died.
beginning. (Wrong) 5. Lest is used to express a negative purpose and is
b) Sanath Jayasuria’s batting may be compared with equivalent to ‘in order that… not’, ‘for fear that’.
the sales of a useful book; they score right from the For example :
beginning. (Right) a) He lied lest he should be killed.
c) If we compare Delhi University with the regional b) I was alarmed lest we should be wrecked.
ones, we find the former to be much more efficient. 6. While is used to mean
(Wrong) A) During that time, as long as.
d) If we compare Delhi University to the regional ones, For example : while there is life there is hope.
we find the former to be much more efficient. (Right) B) At the same time that.
For example : While he found fault, he also praised.
CONJUNCTIONS 7. Only means except that, but, were it not that.
1. Since as conjunction means For example :
A) From and after the time when. a) A ‘very pretty woman, only she squints a little.
For example : b) The day is pleasant, only rather cold.
a) Many things have happened since I left the 8. The conjunctions after, before, as soon as, until are not
school. followed by clause in the future tense. Present simple or
b) I have never seen him since that unfortunate present perfect tense is used to express a future event.
event happened. For example :
B) Seeing that, a) I will phone you after I arrive here.
For example : b) I will phone you after I have arrived here.
a) Since you wish it, it shall be done. 9. As if used in the sense of as it would be is generally
b) Since that is the case, I shall excuse you. followed by a subject + were + complement.
2. Or is used For example :
A) To introduce an alternative. a) He loves you as if you were his own child.
For example : b) Sometimes she weeps and sometimes she laughs as
a) You must work or starve. if she were mad.
b) You may take this book or that one. 10. The clause that begins with as if should be put into the
c) He may study law or medicine or engineering or simple past tense, if the preceding clause expresses a
he may enter into trade. past action. But if it expresses a past action it should be
B) To introduce an alternative name or synonym. followed by the past perfect tense.
For example : The violin or fiddle has become the For example :
leading instrument of the modern orchesta. a) He behaves as if he were a lord.
C) To mean otherwise. b) He behaved as if he had been a lord
For example : We must hasten or night will overtake
11. While as long as is used to express time in sense of how
us.
long, Until is used to express time in sense of before.
3. If is used to mean
For example :
A) On the condition or supposition that.
a) Until you work hard you will improve. (Wrong)
For example :
b) As long as you work hard you will improve. (right)
a) If he is here, I shall see him.
c) He learnt little as long as he was 15 years old.
b) If that is so, I am content.
(Wrong)
B) Admitting that.
d) He learnt little until he was 15 years old. (Right)
For example : If I am blunt, I am at least honest.
C) Whether. 12. No sooner should be followed by verb + subject and than
For example : I asked him if he would help me. should begin another clause.
D) Whenever. For example :
For example : If I feel any doubt I enquire. a) No sooner had I reached the station than the train
4. That is used left.
A) To express a reason or cause. b) No sooner did the bell ring than all the students
For example : rushed in.
a) Not that I loved Caesar less but that I loved 13. When as well as is used, finite verb should agree in
Rome more number and person with the first subject.
b) He was annoyed that he was contradicted. For example : He as well as us is innocent.
14. As well as should never be used in place of and if the first
C-36 GRAMMAR
subject is preceded by the word ‘both’. For example :
For example : a) I am indeed happy with my school but it produces
a) Both Rani as well as Kajol came. (Wrong) famous men. (Wrong)
b) Both Rani and Kajol came. (Right) b) I am indeed happy with my school but it does not
15. Because is generally used when the reason is the most produce famous men. (Right)
important part of a sentence. c) I am indeed happy with my school that it produces
For example : Some people like him because he is honest famous men. (Right)
and hard working. 23. In a “not only ... but also...” sentence, the verb should
Since is used when the reason is already known or is less agree with the noun or pronoun mentioned second, that
important than the chief statement. is; the one after ‘but also’, because this is the part being
For example : Since you refuse to cooperate, I shall have emphasised.
to take legal steps. For example :
For is used when reason is given is an afterthought. a) Not only the students but also the teacher were
For example : The servant must have opened the box, for responsible for what happened in the class. (Wrong)
no one else had the key. For never comes at the beginning b) Not only the students but also the teacher was
of the sentence and for is always preceded by a comma. responsible for what happened in the class. (Right)
16. Scarcely should be followed by when and not by than. 24. Such ... as is used to denote a category whereas such
a) Scarcely had he arrived than he had to leave again. ...that emphasises the degree of something by mentioning
(Wrong) its consequence.
For example :
b) Scarcely had he arrived when he had to leave again.
a) Each member of the alliance agrees to take such action
(Right)
that it deems necessary. (Wrong)
17. Conjunctions such as either..or, neither.. nor, not only..but
b) Each member of the alliance agrees to take such
also, both..and, whether, or etc. always join two words or
action as it deems necessary. (Right)
phrases belonging to the same parts of speech.
Here “it seems necessary” is not a consequence of “such
For example :
action”. The sentence wants to imply that the action
a) Either he will ask me or you. (Wrong)
belongs to the category “as it deems necessary” In other
b) He will ask either me or you. (Right)
words, what kind of action? Such action as it deems
c) Neither he reads nor write English (Wrong)
necessary.
d) He neither reads nor writes English. (Right) a) She looked at him in such distress as he had to look
e) Either you shall have to go home or stay here. away. (Wrong)
(Wrong) b) She looked at him in such distress that he had to
f) You shall have either to go home or stay here. (Right) look away. (Right)
18. Conjunctions like neither...nor, either..or, should be Here, “he had to look away” is a consequence of “she
followed by the same part of speech. looked at him in such distress.” In other words, the degree
For example : of the distress of looking at him was such that (not as)
a) He neither agreed to my proposal nor to his. (Wrong) he had to look away.
b) He agreed neither to my proposal nor to his. (Right)
19. Conjunction is not used before an interrogative adverb or PHRASAL VERBS
interrogative pronoun in the indirect narration. Phrasal Verbs are a particular kind of expression, wherein the
For example : verb is made of two or more components. Mostly the combining
a) He asked me that where I stayed. (Wrong) components are verbs and prepositions. When divided these
b) He asked me where I stayed. (right) components will have a meaning of their own but would not
20. Although goes with yet or a comma in the other clause. suggest anything about the meaning of the phrasal verb. Consider
For example : the following sentences.
a) Although Manohar is hardworking but he does not (a) This sword has been handed down from father to son in the
get a job. (Wrong) family for many generations.
b) Although Manohar is hard working, yet he does not (b) I have been looking forward to meeting you since long
get a job. (Right) now.
21. Nothing else should be followed by 'but' not by 'than', (c) The patient came out of the delirium only when given
For example : tranquilizers.
a) Mr. Bureaucrat! This is nothing else than red-tapism. (d) We had almost decided to give up on the search when we
(Wrong) made the discovery.
b) Mr. Bureaucrat! This is nothing else but red-tapism. Phrasal verbs are idiomatic expressions and have a particular
(Right) meaning different from that of the combining verbs and
22. The correlative conjunctions indeed... but are used to prepositions. Following are some phrasal verbs with their
emphasise the contrast between the first and the second meanings.
parts of the statement. (a) sit in – to attend or take part as a visitor
GRAMMAR C-37
(b) sit out – to stay till the end of How to form question tags?
(c) come round – to accept circumstances and adjust yourself Three things are to be kept in mind while making a question tag :
to them. (a) The right auxiliary Verb to be used in the question.
(d) get on – to manage one’s life (b) The right pronouns to be used in the tag.
(e) turn out – to have a particular result Both (a) and (b) should be in agreement with the verb and
(f) turn up – to arrive unexpectedly noun in the main statement.
(g) show off – to brag or boast (c) Whether the verb in the question tag should be positive or
negative
(h) sort out – to successfully deal with a problem
Rules to form Question Tags
(i) hand in – to give something to someone in authority
I. If the main statement is positive, the auxiliary verb will be
(j) sit down – to take a seat
negative and vice versa e.g.
(k) sit up – to rise from a supine position • He saw that, didn’t he ?
(l) give in – to yield to some pressure • But he isn’t going to England, is he ?
(m) come forth – to find something II. If there is a single subject/noun/pronoun in the main
(o) switch on – to start something sentence, the corresponding pronoun/same pronoun will
(p) turn down – to refuse or reject an offer be used in the question tag. e.g.,
(q) turn in – to expose • You are coming with us, aren’t you ?
(r) look into – probe, or investigate a matter • Reena is leaving tonight, isn’t she?
(s) look after – take care of III. If there is more than one noun/pronoun in the main sentence
(t) take off – to remove something then the corresponding pronoun to the active subject will
(u) put out – to extinguish be used in the Question tag. e.g.
• After all this time you’d think he’d have forgotten, wouldn’t
(v) try on – to wear some clothes for first time
you ?
(w) turn down – lower the volume
• You would’nt refuse me, would you ?
(x) turn on – to start a machine IV If the verb in the main sentence is an active verb without
(y) put in – to invest something (matter or abstract) any auxillary verb, then the verb used in the Question tag
(z) look out – be careful of some danger. will be the form of verb ‘do’ that corresponds with the tense
Following are some sentences using Phrasal Verbs in the main sentence.
• Don’t throw away your opportunity to enter this University. • He knows it’s true, doesn’t he ?
• Many people believe in astrology and tarot cards now a • You wanted to come with me, didn’t you ?
days. • I told you so, didn’t I ?
• Quickly get in the car, we’re getting late. • She never informed us, did she ?
• You can put forward your point in today’s meeting. V If the main sentence has an auxillary then it is used in the
question tag, but with opposite affirmation, i.e., a positive
• To sit through his speech was very difficult.
auxiliary in the main sentence transforms to a negative
• I don’t understand why you put up with his insolent
auxiliary in the question tag and vice versa e.g.
behaviour. • He will be coming, won’t he ?
• I could see through his intentions the first time I met, but • You were there at the party, weren’t you ?
kept quiet to give him a chance to reform. • You would appear for this exam, wouldn’t you?
• Please, fill in all the necessary information in this form. • He didn’t call us, did he ?
• I am sure you will not let me down, I’ve full faith in your • She doesn’t live here anymore, does she ?
capacities.
• Why are you taking it out on me? I’m not the one
MODALS
responsible for the mishap. The verbs like can, could, may, might, would, shall, should and
ought are called modal verbs or modals. They are used with
QUESTION TAGS ordinary verbs to express meanings such as possibility,
Consider the following examples permission, certainly, etc.
(1) You wanted that, didn’t you? (1) Can usually expresses ability or capacity
(2) He is coming tonight, isn’t he? I can swim across the river
(3) You wouldn’t report me, would you? Can you lift this table?
Now, look at the last part of all the above sentences (2) Can is also use to empress permission
preceded by the comma. These are very small questions You can go now.
added to the sentence and are called question Tags.
(3) May is a more formal modal used to express permission
Remember only the question tag is a question and not the
entire sentence. So, one can say that a Question Tag is an You may come in.
added brief question to a statement. Usually a question tag May I leave the room now?
consists of two words- an auxillary verb in the positive or (4) May is also used to suggest possibility in an affirmative
negative form and a pronoun. sentence.
C-38 GRAMMAR
He may be at home You shall be punished for this.
It may rain tomorrow We shall go for a picnic this Sunday.
(5) Can is used to suggest possibility in negative/interrogative (12) Will You ? indicates an invitation or request.
sentence. Will you dine with us tonight ?
Can this be true ? Will you lend me your car for a week ?
It cannot be so. (13) Should and would are used as past forms of shall and will.
(6) May when used in a negative sentence suggests an I expected that I would get a first class.
improbability whereas can suggests impossibility. She would sit for hours listening to the radio.
He may not come today. (14) Should is used to express duty or obligation.
She cannot sing. We should obey the laws.
(7) Could and might are used as past tense forms of can and You should keep your promise.
‘may’. (15) Should is used to express a supposition
I could swim across the river when I was young. If it should rain, they will not come.
I thought he might be at home. (16) Should can also be used to express probability.
(8) Might suggests less possibility or probability than may. He should be in the library.
I might go to Bangalore next week suggests the probability (17) Must is used to express necessity.
of going is less than a sentence with ‘may’ will suggest. You must improve your spelling.
(9) Could is used as a polite form of seeking permission or (18) Must is also need to express obligation, and is a stronger
making a request. word than should.
Could you pass me the plate ? We must follow the law.
Could I please talk to Mr. Grover? (19) Must is also used to express logical certainty.
(10) Shall is used with first person and will in all the persons to Living alone in such a big city must be difficult.
denote future action. (20) Ought is used to express moral obligation and is stronger
I shall need the money tomorrow. than both should and must.
When will you come next? We ought to love our parents.
(11) Shall is used with the second and third person to express (21) Ought is also used to express probability sometimes when
command, promise or threat. the probability is very strong.
You shall never come near my child. The book ought to be very useful.
GRAMMAR C-39

EXERCISE
Directions (Qs. 1-61): Read each sentence to find out whether 18. One of the security men (a)/rushed forward and asked
there is any error in it. The error, if any, will be in one part of (b)/ me whether (c)/ had anything objectionable. (d)/ No
the sentence. The number of that part is the answer. If there error (e).
is no error, the answer is (5). (Ignore errors of punctuation, if 19. We could not (a)/ believe that one (b)/ of us was (c)/
any). responsible with the act. (d)/ No error (e).
1. a) The driver of that car /b) is sounding horn for /c) the 20. We are now (a)/ reliably learnt that (b)/ he was involved
last ten minutes /d) but nobody tells him to stop. /e) No (c)/ in the bank robbery. (d) No error (e).
error 21. I do not know (a)/ what most people feel (b)/ depressed
2. a) If you go on letting /b) your dog chase cars /c) he will
and dejected (c)/ even with the slightest provocation. (d)/
end by being /d) run down one day. /e) No error
No error (e).
3. a) He heard the guard /b) blowing the whistle and knew
22. She had such pretty (a)/ that she thinks (b)/ she can
/c) it is time for him /d) to enter the train. /e) No error
4. a) He telephoned from a public call-box /b) so that the call afford to be (c)/ careless about her clothes. (d)/ No error
/c) would not be traced /d) to his own address. /e) No (e).
error 23. After carefully examining (a)/ all the medicine bottles (b)/
5. a) It has been better /b) to put your money in a bank / he submitted a detailed report (c) to the higher authorities.
c) than to keep it under /d) your bed in a suitcase. /e) No (d)/ No error (e).
error 24. All of you have the liberty (a)/ to come home (b)/ as per
6. a) If you would have read /b) the instructions carefully the convenient (c)/ and discuss the problems. (d)/ No
/c) you would not have /d) answered the questions error (e).
wrongly. /e) No error 25. He was persuaded (a)/ by his friends (b)/ to end his fast
7. a) I can see through /b) her sudden friendliness; /c) she (c)/ because of his condition deteriorated. (d) No error (e).
wants me to look over /d) her dog while she is away. / 26. I know who (a)/ this job should be (b)/ entrusted to (c)/
e) No error for smooth handling. (d)/ No error (e).
8. You may not know it (a)/ but this engine is (b)/ claimed 27. They have the nasty habit of (a)/ looking down upon
to have twice (c)/ as powerful as the previous one. (d)/ people (b)/ and criticised them (c)/ for no reason. (d)/ No
No error (e)
error (e).
9. Nothing ever becomes real (a)/ till it is experienced. (b)/
28. Nowadays, the cost of living (a)/ is so high that (b)/
Even a proverb is no proverb to you (c)/ till your life has
people find it difficult (c)/ to make both ends meeting. (d)/
illustrated with it. (d)/ No error (e).
10. I remember my childhood days (a)/ when I was used to No error (e).
go (b) to the farm with my father (c) and help him in his 29. Karnavati is (a)/ one of the leading (b)/ business centres
work. (d)/ No error (e). (c)/ in our State. (d)/ No error (e).
11. I missed the last train (a)/ which I usually catch (b)/ and 30. As I reached the hospital a)/ I had found, a great rush of
have to stay at the station (c)/ on my way back home visitors b)/ whose relatives had been admitted there c)/
yesterday. (d)/ No error (e). for one or the other ailment. d)/ No error e)
12. Sureshbabu, who is living (a)/ in this town since 1955, 31. One should study the history a)/ of his country because
(b)/ is a well-known scholar of history (c)/ and a it alone can satisfy b)/ one’s natural curiosity to know c)/
distinguished musician. (d)/ No error (e). what happened in the past. d)/ No error e)
13. If you had read (a)/ the relevant literature carefully (b)/ 32 It is interesting to note a)/ that the greatest lines in poetry
You would have answered (c)/ most of the questions are simple b)/ and yet there is with them some quality c)/
correctly. (d) No error (e). which makes them outstandingly great. d)/ No error e)
14. The house where the dead man was found (a)/ is being 33. In order to make human life happy, a)/ man should live
guarded by police (b)/ to prevent it from being entered b)/as far as possible c)/ in perfect harmony with nature.
(c)/ and the evidence interfered with (d)/No error (e). d)/ No error e)
15. We were happy that (a)/ the audience responded well (b)/
34. You have hear d a)/ of Socrates, I suppose. b)/
and gave all the speakers (c)/ a patiently listening. (d)/ No
Undoubtedly he was one c)/ of the greatest man of the
error (e).
16. He received timely support (a)/ from his elder brother (b)/ world. d)/ No error e)
who is working abroad (c)/ for the last six years. (d)/ No 35. My daughter never (a)/would write to me (b)/so I never
error (e). know (c)/what she is doing. (d)/No error (e).
17. The notorious gang opened (a)/ the door quietly and (b)/ 36. Whenever we have a puncture (a)/she just sits in the car
escaped in the dark with (c)/ whatever they would collect. (b)/and reads a book (c)/while I changed the wheel. (d)/
(d)/ No error (e). No error (e).
C-40 GRAMMAR
37. He walked to the market (a)/with both his servants (b)/on 57. This laboratory of physics is (a)/ not only equipped with
either side of his (c)/to help him buy things. (d)/No error (b)/ all state-of-the-art instruments (c)/ but also with
(e). outstanding physicists. (d)/ No error (e)
38. Ganesh, who has been (a) driving all day (b)/was extremely 58. No method of making (a)/ other people agree to (b)/ your
tired (c)/and wanted to stop. (d)/No error (e). view-point is (c)/ as effective as this method. (d)/ No error
39. Everyone was reading quietly (a)/when suddenly the door (e)
(b)/burst open and a (c)/complete stranger rushed in. 59. I was pretty sure that (a)/ he would support my views (b)/
(d)/No error (e). for changing the age-old (c)/ and static structure of our
40. My secretary is so (a)/careful of her work that (b)/none organisation. (d)/ No error (e)
has so far found (c)/any error in her work. (d)/ No error 60. I did not like his (a)/ comments on my paper (b)/ but I had
(e) no alternative (c)/ as I had agreed to keep quiet. (d)/No
41. Our conclusion is that (a)/ between Vinayak and (b)/ error (e)
Lobo, 61. The report is candid in admitting (a)/ that the investment
Vinayak is (c)/ the most honest. (d)/ No error (e) by the government (b)/ in health and family planning (c)/
42. The new project group (a)/ would first look into the have been eroded considerably. (d)/ No error (e)
tender conditions (b)/ of both basic and value-added (c)/ Directions (Qs.62 -68): Read each sentence(s) to find out
services before submit its bid. (d)/ No error (e) whether there is any grammatical error in it. The error, if any,
43. I would have committed (a)/the same mistake of signing will be in one part of the sentence. The letter of that part is the
(b)/ the sale deed if my agent (c)/ would not have answer. If there is no error, the answer is (e), i.e., No error’.
(Ignore the errors of punctuation, if any).
forewarned me. (d)/ No error (e)
62. (a) The judge asked the man
44. The team leaders encourages (a)/ the participants who
(b) if the bag he had lost
have (b)/ difficulty in performing (c)/ the assigned task.
(c) contain five thousands rupees.
(d)/ No error (e)
(d) The man replied that it did
45. We are happy that (a)/ our prime minister (b)/ with the
(e) No error
members (c)/ of his cabinet are to be present at the
63. (a) I trust you will
function. (d)/No error (e)
(b) show forbearance to me
46. Neither the size nor the colour (a)/ of clothes which (b)/
(c) a few minutes more
I purchased for him (c)/ yesterday were right. (d)/ No
(d) so that I can finish this work.
error (e)
(e) No error
47. I heard to my surprise (a)/ that the present (b)/ I send him
64. (a) The ground outside the village
was not (c)/ to his taste. (d)/ No error (e)
(b) abounding with frogs and snakes
48. Let us refer (a)/ this matter to the principal. (b)/ We shall
(c) the enemies of mankind
abide (c)/ with his decision. (d)/ No error (e)
(d) is soft and marshy.
49. If I would have come (a)/ a little earlier, I would have (b)/
(e) No error
got a glimpse (c)/ of my beloved leader. (d)I No error (e)
65. (a) We are all short-sighted
50. Whey you buy something (a)/ on the instalment system
(b) and very often see but one side of the matter
(b)/ you are not required to pay (c)/ the whole price at
(c) our views are not extended
once. (d)/No error (e)
(d) to alI that has a connection with it.
51. I am waiting for you (a)/ for the last two hours (b)/ but
(e) No error
you
66. (a) Just laws are no restraint on
did not bother (c)/ to turn up in time (d)/ No error (e)
(b ) the freedom of the good,
52. He is certainly a man (a) / whom I know very well (b) /
(c) for the good man desires nothing
is trustworthy beyond doubt (c) / and meticulous in his
(d) which a just law interfere with.
habits. (d) / No error (e) /
(e) No error
53. No sooner did (a) / we reach the station (b) / than the train
67 (a) Had he done
had (c) / started moving out of the station. (d) / No error (e)
(b) his home work well
/.
(c) he would not have
54. I am sure about it, (a)/ nobody has lived (b) / in that
(d) suffered this embarrassment.
house (c) / for a hundred years. (d) / No error (e) /
(e) No error
55. There were no less (a) / than forty boys (b) / in the class
68 (a) He was angry with me
(c) / when this happened. (d) / No error (e) /
(b) because he thought my
56. I am glad to hear (a) / that you narrowly escaped (b) /
(c) remark was
being run over by (c) / a speeding car yesterday. (d) / No
(d) aimed before him.
error (e) /
(e) No error
GRAMMAR C-41
Directions (Qs. 69-120): Which of the phrases (a), (b), (c) or 76. All that I have described have been taken place in the last
(d) given below should replace the phrase given in bold in the four decades.
following sentence to make the sentence meaningful and (a) have taken
grammatically correct? If the sentence is correct as it is and no (b) has been taken
correction is required mark (e) as the answer. (c) has taken
69. He admitted admiringly that he had never come across a (d) was taken
painting which did not please him more. (e) No correction required
(a) pleased him more 77. The fees charged by the architect for the plans of the new
(b) would have pleased him building were unreasonable high.
(c) had not pleased him more (a) were unreasonably high
(d) had been pleased him any more (b) were unreasonably higher
(e) No correction required (c) had been unreasonably higher
70. It has always been better to use preventive measures than (d) had been unreasonable high
to cure illness (e) No correction required
(a) had always been 78. There are many new emerging fields in information
(b) is always technology and electronics.
(c) was always (a) newly.emerging
(d) would have always been (b) new emergent
(e) No correction required (c) new emergency
71. He had deliberately kept the matter pending so that people (d) newly emergent
should be bribed him. (e) No correction required
(a) could be bribed 79. People in underdeveloped countries are distressing
(b) should bribe because of the antagonistic attitude of developed countries.
(c) could be bribing (a) have been distressing with
(d) should have bribed (b) are distressed because
(e) No correction required (c) are distressed at
72. Because of a shortage the government had appealed to (d) were distressing by
the people to be extravagant with water. (e) No correction required
(a) for being extravagant 80. He had been behaved impolitely and suffered owing to
(b) to be saving that.
(c) to be economical (a) was behaved
(d) to be economic (b) had behaved
(e) No correction required (c) have been behaved
73. He was found absorbing in his studies when I reached (d) would have behaved
there. (e) No correction required
(a) was to find absorbed 81. It has become a commonly practise to talk about women’s
liberation.
(b) was found absorbed
(a) commonly practised talk about
(c) had been found absorbing
(b) common practice to talk about
(d) had to be found absorbing
(c) common practice of talking with
(e) No correction required
(d) commonly practising to talk about
74. The guide warned us that we had better be prepared for
(e) No correction required
a long, hard day.
82. He is so brisk himself that he cannot tolerate any efficiency.
(a) had been better prepared
(a) tolerate hardly any inefficiency
(b) should better be prepared (b) hardly tolerates lethargy
(c) should be prepared with (c) tolerate any haste
(d) had been better preparing (d) tolerate any delay
(e) No correction required (e) No correction required
75. Income tax rates are usualy associated to one’s annual 83. No person with a reasonably self-esteem would ever like
income. to succumb to any pressure.
(a) related to one’s (a) reasonable self-esteem
(b) dependent to one’s (b) reasonable self-esteemed
(c) depended on one’s (c) reasonably self-esteemed
(d) associated with one’s (d) reasonably a self-esteem
(e) No correction required (e) No correction required
C-42 GRAMMAR
84. It is the temple where religious rites are celebrated as (d) started moving
they were for centuries. (e) No correction required
(a) as they have been for 93. He has even venturing into areas which he had shunned.
(b) so were they for (a) had even venturing into
(c) as they are for (b) even is being venture into
(d) as they were before (c) has even been venturing into
(e) No correction required (d) has even been ventured in
85. By the time he had won his commission, the senior (e) No correction required
officer had to start seeking employment elsewhere. 94. When the boy regain consciousness he wanted to eat
(a) had started seeking something.
(b) were started seeking (a) If the boy regain
(c) had been started to seek (b) When the boy regained
(d) were to have started seeking (c) Despite the boy regain
(e) No correction required (d) On the boy regaining
86. The congestion on the streets must be seen to believe. (e) No correction required
(a) have been to believe 95. The social worker wanted to bring about little changes in
(b) have been seen for believing the lives of the people of that village.
(c) have seen for belief (a) to bring back
(d) be seen to be believed (b) to bring up
(e) No correction required (c) to bring forth
87. He had begun to develop the qualities that he was going (d) bringing about
to need in later years. (e) No correction required
(a) was going to be needed 96. Raghunath proposes to lay claim for the insurance
(b) had gone to need company as soon as he recovers from the accident.
(c) was later to need (a) lay claim to
(d) had been gone to need (b) lay claim on
(e) No correction required (c) laying claim towards
88. All round is emptiness and silence, the silence, it seems, (d) lay claim against
of a land that man has not yet set foot upon. (e) No correction required
(a) around is emptiness and silence 97. The new concession announced by the Government will
(b) round is empty and silent have only a marginalised effect on the lives of the people.
(c) round are emptiness and silence (a) marginal effect off
(d) around are empty and silence (b) margin of effect on
(e) No correction required (c) marginal effect on
89. He was quite sure that none of them were aware of the (d) marginalising effect in
truth. (e) No correction required
(a) were aware from 98. The Charitable Hospital works under the auspices from
(b) was aware of the Welfare Trust of an Industry.
(c) were beware of (a) under the auspices by
(d) had aware of (b) by the auspices from
(e) No correction required (c) through the auspices from
90. I was too overwhelmed to make any decision. (d) under the auspices of
(a) too much overwhelm to (e) No correction required
(b) so overwhelmed to 99. Government should not stop to spending money on arms
(c) extremely overwhelmed about and ammunition in the wake of the present strained
(d) quite overwhelming to relations.
(e) No correction required (a) should not stop spending
91. Shocked of finding an unknown person, the army officer (b) shall not be stopped to spend
briskly caught hold of him. (c) will not stop to spend
(a) Shockingly found (d) should not be stopping to spend
(b) Shocked at finding
(e) No Correction Required
(c) Shocked by finding
100. The one-act play was so humorous that it was hardly
(d) Finding as a shock
impossible to keep a straight face.
(e) No correction required
(a) is hardly impossible
92. No sooner did he reach the station than the train had
(b) was almost impossible
started moving.
(c) is hardly possible
(a) had started movement
(d) was barely impossible
(b) had been moving
(e) No Correction Required
(c) had been started movement
GRAMMAR C-43
101. One of the politicians have open admittance that he had (c) be considered for
resorted to corrupt practices. (d) may consider for
(a) have opened admittance (e) No correction required
(b) has opened admittance 110. A majority of the students believe that the examinations
(c) has openly admitted are unnecessary.
(d) have been open admittances (a) have been not necessary
(e) No Correction required (b) have unnecessary
102. The unkind comments passed by her superiors made her (c) are being unnecessary
resign. (d) were being unnecessary
(a) unkindly comments passing by (e) No correction required
(b) unkind comments passing on 111. No sooner the advertisement appeared in the newspapers
(c) unkind comments posed by than there was a rush on the booking window.
(d) unkindly comments passed on (a) No sooner had the advertisement appear
(e) No correction Required (b) The advertisement appear no sooner
103. The ban on public meetings have been lifted temporarily (c) The advertisement no sooner having appeared
in view of the auspicious occasion. (d) No sooner did the advertisement appear
(a) have been temporarily lifted (e) No correction required
(b) have been lifting temporarily 112. May I know whom I am talking - to?
(c) had been lifting temporary (a) who I am talking
(d) has been lifted temporarily (b) to whom I am talking
(e) No Correction Required (c) whom I talk
104. Finishing his breakfast, he started working on the problem (d) who I have talked
that had been awaiting disposal for a long time. (e) No correction required
(a) His breakfast finished 113. I am working on this job since last Monday.
(b) His breakfast having finished (a) was working
(c) Having finished his breakfast (b) have been working
(d) Finished his breakfast (c) being worked
(e) No correction required (d) were to have worked
105. One of the function of a teacher is to spot cases of (e) No correction required
maladjustment. 114. The modifications made by them in the draft were so
(a) One of the functions of drastic that the entire emphasis had been shifted.
(b) Most of the functions of (a) shall have been shifted
(c) Some of the functions (c) had shifted
(d) One of the functions by (b) was being shifted
(e) No correction required (d) had been shifting
106. In our friends’ circle it is customary for each of the (e) No correction required
members to buy their own tickets. 115. It is reliable to learn that there is not substantial evidence
(a) buying their own tickets to prove his innocence.
(b) are buying their own tickets (a) is reliably learnt
(c) buying his own tickets (b) reliably to learn
(d) to buy his own ticket (c) was reliable to learn
(e) No correction required (d) has been reliable learning
107. Where the distance is not too much I prefer walking on (e) No correction required
foot than waiting for a bus. 116. He has now succeeded in overwhelming the grief.
(a) than wait for the (a) successful in overwhelming
(b) than no waiting for (b) successful to overwhelm
(c) to waiting for a (c) succeeded to overwhelm
(d) rather than waiting for a (d) succeeded in overcoming
(e) No correction required
(e) No correction required
108. Being a pleasant morning, he went out for a walk along
117. Despite their best efforts, they could not convince the
the seashore.
members by changing their decision.
(a) With a pleasant morning
(a) and changed their (b) to change their
(b) It being a pleasant morning
(c) with changing their (d) in changing his
(c) Being a pleasing morning
(e) No correction required
(d) As a pleasant morning
(e) No correction required 118. The novel ideas suggested by the employee were
109. We are happy to recommend that his son to be considered appreciated by the management.
for the post. (a) have appreciated by
(a) considers for (b) have been appreciated for
(b) be considered with (c) were appreciative of
C-44 GRAMMAR
(d) had appreciated by 120. The sight of the accident was so frightened that the
(e) No correction required bystanders could not utter a single word.
119. The two brothers were so much similar in appearance (a) so very frightening because
that nobody believed that they were twins. (b) so frightening that
(a) very much similar in (b) so much similar at (c) extremely frightening as
(c) so different in (d) so different from (d) extremely frightened
(e) No correction required (e) No correction required

ANSWER KEY
1 (b) 16 (e) 31 (b) 46 (d) 61 (d) 76 (c) 91 (b) 106 (d)
2 (c) 17 (c) 32 (c) 47 (c) 62 (c) 77 (a) 92 (d) 107 (d)
3 (c) 18 (d) 33 (a) 48 (d) 63 (c) 78 (a) 93 (c) 108 (b)
4 (c) 19 (d) 34 (d) 49 (a) 64 (e) 79 (c) 94 (b) 109 (c)
5 (a) 20 (a) 35 (b) 50 (d) 65 (d) 80 (b) 95 (e) 110 (e)
6 (a) 21 (b) 36 (d) 51 (a) 66 (d) 81 (b) 96 (d) 111 (d)
7 (c) 22 (a) 37 (c) 52 (c) 67 (e) 82 (d) 97 (c) 112 (b)
8 (c) 23 (e) 38 (a) 53 (c) 68 (d) 83 (a) 98 (d) 113 (b)
9 (d) 24 (c) 39 (e) 54 (b) 69 (a) 84 (a) 99 (a) 114 (c)
10 (b) 25 (d) 40 (b) 55 (a) 70 (b) 85 (a) 100 (b) 115 (a)
11 (c) 26 (a) 41 (d) 56 (e) 71 (b) 86 (d) 101 (c) 116 (d)
12 (d) 27 (c) 42 (d) 57 (b) 72 (c) 87 (e) 102 (e) 117 (b)
13 (e) 28 (d) 43 (d) 58 (a) 73 (b) 88 (a) 103 (d) 118 (e)
14 (d) 29 (d) 44 (a) 59 (c) 74 (e) 89 (b) 104 (c) 119 (c)
15 (d) 30 (b) 45 (d) 60 (c) 75 (a) 90 (e) 105 (a) 120 (b)

Answers &
Explanations
1. (b) It should be ‘has been sounding horn’. 48. (d) It should be by his decision.
2. (c) Replace ‘by’ with ‘up’. 49. (a) The sentence should begin as, ‘if 1 had come ...’
3. (c) It should be ‘was’ in place of ‘is’. 50. (d) Replace ‘at once’ with ‘at the begining’
4. (c) Change ‘would not be traced’ to ‘could not be traced’. 51. (a) The sentence should start as, I had been waiting for
5. (a) Change the first part as ------- It would be better ... you.......’
6. (a) The sentence should start as ------- If you had read 52. (c) It should be " and who is ..." In the given form, the
... subject ‘of is’ is missing.
7. (c) The right phrase will be ‘to look after’ in place of ‘to 53. (c) Delete had. In a "No sooner...than..."structure, than
look over’. is followed by a subject followed by past simple
35. (b) Replace would write by writes. tense.
36. (d) Replace changed by change 54. (b) For a hundred years indicates that the verb should
37. (c) Replace his by him. be in the perfect continuous tense. Hence replace
38. (a) Replace has by had. 'has lived' by 'has been living'.
40. (b) Replace ‘of’ with `in’. 55. (a) Boys is countable. Hence replace 'less' by 'fewer'.
41. (d) Here there is a comparison between two persons, so 57. (b) It should be “equipped not only with” instead of
it should be‘more honest’ in place of ‘most honest’. “not only equipped with”.
42. (d) 'Submit' should be ‘submitting’. 58. (a) Here, as we are comparing two methods for a single
43. (d) Replace ‘would not have’ with ‘had not’. purpose, the sentence should start as – ‘No other
44. (a) ‘team leaders encourages’ should be replaced by method’.
‘team leaders encourage’ or ‘team leader encourages’. 59. (c) Views should always be followed by ‘on’ instead of
45. (d) Replace ‘are’ with ‘is’ because the subject (prime ‘for’.
minister) is singular here. 60. (c) Delete ‘I’.
46. (d) Replace ‘were’ with ‘was’. 61. (d) It should be ‘has been’ instead of ‘have been’.
47. (c) It should be ‘the present I sent for him’.
READING COMPREHENSION C-45

Reading
Comprehension
3 Chapter

Reading Comprehension that helps provide a framework for adding details. Also, look for
Comprehension is the process of making meaning from a transitional words, phrases or paragraphs that change the topic.
written text. Typically, a candidate might have difficulty Identify the type of reasoning
understanding due to limited vocabulary and/or a lack of familiarity Does the author use cause and effect reasoning, hypothesis,
with the subject matter. Both these constraints may be remedied model building, induction or deduction, systems thinking?
by reading more widely and making friends with a good dictionary.
Anticipate and predict
How to Improve Reading Comprehension
Really smart readers try to anticipate the author and predict
Reading is all about information. It's not about the number
future ideas and questions. If you're right, this reinforces your
of words you read, but the amount of value you extract from
them. The key to improved reading comprehension isn't moving understanding. If you're wrong, you make adjustments quicker.
your eyes across a page more quickly. It's about creating a mental Look for the method of organization
framework that helps you process words and ideas. Is the material organized chronologically, serially, logically,
With a bit of practice, anyone can read faster and more functionally, spatially or hierarchicaley.
productively. The steps outlined below will help you to extract Create motivation and interest
the maximum amount of information in the least amount of time. Preview material, ask questions, discuss ideas with
Before reading the text, ask yourself what you already know classmates. The stronger your interest, the greater your
about its topic. Try to recall as much information as you can. comprehension.
Think of related ideas you've learned in the past. Make brief
Pay attention to supporting cues
notes about your thoughts or discuss what you remember with
others. Reading comprehension requires motivation, mental Study pictures, graphs and headings. Read the first and last
frameworks for holding ideas, concentration and good study paragraph in a chapter, or the first sentence in each section.
techniques. Here are some suggestions. Highlight, summarize and review
Improve Your Reading Comprehension by Researching the Topic Just reading a book once is not enough. To develop a deeper
Background information may appear on book covers and understanding, you have to highlight, summarize and review
inner flaps of book jackets. Many books include an introductory important ideas.
section and a mini-biography about the author. Book publisher's Build a good vocabulary
websites may also include background information. Think about For most educated people, this is a lifetime project. The best
the information you read. Ask: way to improve your vocabulary is to use a dictionary regularly.
• What kind of text is this? You might carry around a pocket dictionary and use it to look up
• What new information did I learn, and what do I expect to new words. Or, you can keep a list of words to look up at the end
learn? of the day. Concentrate on roots, prefixes and endings.
• Is this text informative or entertaining, fact or fiction? As you read, make a list of unfamiliar vocabulary words.
• What interests me about this book? Look up the meanings of the words in the dictionary, and jot
Develop a broad background. definitions down by hand. Writing definitions by hand will help
Broaden your background knowledge by reading you remember the definition much more than by typing or by
newspapers, magazines and books. Become interested in world reading alone.
events. Monitor effectiveness
Improve Your Understanding Good readers monitor their attention, concentration and
As you read, what questions come to mind? Read on to find effectiveness. They quickly recognize if they've missed an idea
the answers. You can think about the questions and answers or and backup to reread it.
jot them down on paper. Research indicates that writing notes by Test Yourself to Determine How Well You've Learned the
hand can increase comprehension and recall among students material
who are not learning disabled in writing. For those who have LDs After your reading session, quiz yourself on the main points.
in writing mechanics, pair the writing with discussion to improve What was the main idea? Who are the characters in the story?
understanding and recall. What information did you learn? Jot down your thoughts in your
Know the structure of paragraphs own words to help you remember them and give you deeper
Good writers construct paragraphs that have a beginning, insight into the topic. If expressive writing is difficult for you, jot
middle and end. Often, the first sentence will give an overview shorter notes and discuss the reading with a friend or parent.
C-46 READING COMPREHENSION

EXERCISE
Directions (Qs. 1-204): Read the following passages carefully 2. Which of the following is the reason for the sorry state
and answer the questions that follow: of affairs of the Indian Universities as mentioned in the
Passage 1 passage?
The University Grants Commission’s directive to college and (a ) The poor quality of teachers
university lecturers to spend a minimum of 10 hours a week in (b) Involvement of teachers in extra-curricular activities
direct teaching is the product of budgetary cutbacks rather (c) Politics within and outside the departments
than pedagogic wisdom. It may seem odd, at first blush, that (d) Heavy burden of teaching hours on the teachers
teachers should protest about teaching a mere 22 hours. (e) Not getting enough financial assistance
However, if one considers the amount of time academics require 3. Which of the following statements is/are TRUE in the
to prepare lectures of good quality as well as the time they context of the passage?
need to spend doing research, it is clear that most conscientious (A) Most colleges do not carry out research worth the
teachers work more than 40 hours a week. In university systems name.
around the world lecturers rarely spend more than 12 to 15 (B) UGC wants lecturers to spend minimum 22 hours a
hours in direct teaching activities a week. The average college week in direct teaching.
lecturer in India does not have any office space. If computers (C) Indian higher education system is in unsound state.
are available, internet connectivity is unlikely. Libraries are (a) Only A and C (b) All A, B and C
poorly stocked. Now, the UGC says universities must implement (c) Only C (d) Only B
a complete freeze on all permanent recruitment, abolish all (e) Only B and C
posts which have been vacant for more than a year, and cut 4. Besides direct teaching ,university teachers spend
staff strength by 10 per cent. And it is in order to ensure that considerable time in/on
these cutbacks do not affect the quantum of teaching that (a) administrative activities such as admissions
existing lecturers are being asked to work longer. Obviously, (b) supervising examinations and corrections of answer
the quality of teaching and academic work in general will decline. papers
While it is true that some college teachers do not take their (c) carrying out research in the area of their intrest
classes regularly, the UGC and the institutions concerned must (d) maintaining research equipment and libraries
find a proper way to hold them accountable. An absentee (e) developing liaison with the user organizations.
teacher will continue to play truant even if the number of hours 5. Which of the following statements is NOT TRUE in the
he is required to teach goes up. context of the passage ?
All of us are well aware of the unsound state that the Indian (a) UGC wants teachers to spend minimum 40 hours in
higher education system is in today. Thanks to years of a week in teaching
sustained financial neglect, most Indian universities and colleges (b) Some college teachers do not engage their classes
do not carry out only research worth the name. Even as the regularly.
number of students entering colleges has increased dramatically, (c) The average college teacher in India does not have
public investment in higher education has actually declined in any office space.
relative terms. Between 1985 and 1997, when public expenditure (d) UGC wants universities to abolish all posts which
on higher education as a percentage of outlays on all levels of have been vacant for more than a year.
education grew by more than 60 per cent in Malaysia and 20 (e) All are true
per cent in Thailand, India showed a decline of more than 10 6. Between 1985 and 1997, the number of teachers in higher
per cent. Throughout the world, the number of teachers in education per million population, in India
higher education per million population grew by more than 10
(a) increased by 60% (b) increased by 20%
per cent in the same period; in India it fell by one per cent.
(c) decreased by 22% (d) decreased by 10%
Instead of transferring the burden of government apathy on to
(e) decreased by 1%
the backs of teachers, the UGC should insist that the needs of
7. Which of the following statements is NOT TRUE in the
the country’s university system be adequately catered to.
1. Why does the UGC want to increase the direct teaching context of the passage?
hours of university teachers? (a) Indian universities are financially neglected
(a) UGC feels that the duration of contact between the (b) All over the world, the university lecturers hardly
teacher and the taught should be more. spend more than 12 to15 hours a week in direct
(b) UGC wants teachers to spend more time in their teaching
departments. (c) Indian universities are being asked to reduce staff
(c) UGC wants teachers to devote some time to improve strength by 10%.
university administration. (d) Public investment in higher education has increased
(d) UGC does not have money to appoint additional in India.
teachers. (e) Malaysia spends more money on education than
(e) None of these Thailand.
READING COMPREHENSION C-47
8. Choose the word which is SIMILAR in meaning to the 14. What was the result of the application of fire to metals?
word ‘freeze’ as used in the passage. (a) The metals became soft and tempered.
(a) cold (b) halt (b) They became weak and more supple.
(c) decay (d) control (c) They disintegrated into separate elements.
(e) power (d) They could be preserved for a longer period.
9. What is the UGC directive to universities? (e) None of these
(a) Improve the quality of teaching 15. What is the meaning of “saving wit to imitate Nature”?
(b) Spend time on research activities (a) blindly following the dictates of nature
(c) Do not appoint any permanent teacher (b) abject surrender to the forces of nature
(d) Provide computer and internet facilities (c) rational and objective analysis of nature’s laws
(e) Do not spend money on counselling services to the (d) intelligent use of God’s gift
students. (e) wisdom that carried out of difficult situation
10. Choose the word which is SIMILAR in meaning to the 16. What was the duty of the virgins in the Temple of Vesta?
word 'sustained' as used in the passage. (a) to light the fire everyday exactly at a designated time
(a) continuous (b) frequent (b) to light several other fires from the fire of the Temple
(c) careless (d) deliberate of Vesta
(e) sporadic (c) to see to it that the sacred fire in the temple did not
Passage 2 go out
If man began with speech and civilisation with agriculture, (d) to carry the fire from place to place wherever the
industry began with fire. Man did not invent it; probably nature Roman’s went
produced a marvel for him by the friction of leaves or twigs,
(e) None of these
a stroke of lightning or a chance union of chemicals; man
17. Choose the word which is SIMILAR in meaning to the
merely had a saving wit to imitate Nature and to improve upon
her. He put fire to a thousand uses. First, perhaps he made it word ‘supple’ as used in the passage.
serve as a torch to conquer his fearsome enemy, darkness; (a) flexible (b) hard
then he used it for warmth and moved about freely from his (c) powerful (d) weak
native tropics to less enervating zones, slowly making the (e) useful
planet human. Then, he applied, it to metals, softening them, 18. Choose the word which is MOST OPPOSITE in meaning
tempering them and combining them into forms stronger and to the word ‘tempering’ as used in the passage.
more supple than those in which they had come to his hand. (a) bending (b) hardening
It was fire that created the old and honorable art of cooking, (c) elongating (d) shortening
extending the diet of man to a thousand foods that could not (e) softening
be eaten before. So beneficent and strange was it that fire 19. Which of the following statements is TRUE in the context
always remained a miracle to primitive man, fit to be worshipped of the passage?
as God. He offered it countless ceremonies of devotion and
(a) Man invented fire.
made it a centre or focus of his life. He carried it carefully with
(b) Civilization began with fire.
him as he moved from place to place in his wanderings and
would not willingly let it die. The Romans even punished with (c) Man applied fire first to metals.
death the careless virgins of the Temple of Vesta who allowed (d) Man’s native place was very cold.
the sacred fire to be extinguished. (e) Nature produced fire.
11. Why is fire called a wonder? 20. Which of the following statements is NOT TRUE in the
(a) Civilisation has given man this fascinating things. context of the passage?
(b) Industry could utilise it very effectively. (a) Industry began with fire.
(c) Man did not invent it nor could understand it. (b) Fire taught man the art of cooking.
(d) It could be put to several uses. (c) Fire always remained a miracle to primitive man.
(e) It has played the role of shaping the destiny of (d) Man invented fire.
human beings. (e) Man worshipped fire as God.
12. Which other single word or pair of words in the passage Passage 3
conveys/convey the idea that fire is a wonder? We are tempted to assume that technological progress is the
(a) imitate (b) marvel real progress and that material success is the criterion of
(c) civilisation (d) twig civilisation. If the eastern people become fascinated by machines
and techniques and use them as Western nations do, to build
(e) saving wit
industrial organisations and large military establishments, they
13. Mention the thing which the primitive man feared the
will get involved in power politics and drift into the danger of
most.
death. Scientific and technological civilisation brings
(a) the fury of nature (b) wild animals opportunities and great rewards but also great risks and
(c) dangerous chemicals (d) darkness temptations. If machines get into the saddle all our progress
(e) insecurity will have been in vain. The problem facing us is a universal
C-48 READING COMPREHENSION
one. Both east and west are threatened with the same danger technology. Most mechanical two-wheelers roll off the assembly
and face the same destiny. Science and technology are neither line without a proper pollution control system. Little effort is
good nor bad.They are not to be tabooed but tamed and made for R&D on simple technologies, which could make a
assigned their proper place.They become dangers only if they vital difference to people’s lives and the environment.
become idols. However, while there is no denying that South Asia must clean
21. What is man tempted to assume? up its act, skeptics might question the timing of the haze report.
(a) Western nations are superior to eastern nations. The Johannesburg meet on Rio+10 is just two weeks away and
(b) Science and technology pose no danger to humanity. the stage is set for the usual battle between the developing
(c) Technological progress is the real progress. world and the West, particularly the US President. Mr Bush
(d) As such there is nothing wrong with machines. has adamantly refused to sign any protocol which would mean
(e) None of these a change in American consumption level. UN environment
22. When do science and technology become dangerous? report is likely to find a place in the US arsenal as it points an
(a) When they become idols accusing finger towards countries like India and China. Yet the
US can hardly deny its own dubious role in the matter of
(b) When they are used with temptation
erasing trading quotas. Richer countries can simply buy up
(c) When their advantages are not used judiciously
excess credits from poorer countries and continue to pollute.
(d) When it is assumed that material success is the Rather than try to get the better of developing countries, who
criterion of civilisation undoubtedly have taken up environmental shortcuts in their
(e) None of these bid to catch up with the West, the US should take a look at
23. What will happen if eastern people use machines? the environmental profligacy. which is going on within. From
(a) It will bring great opportunities and rewards to them. opening up virgin territories for oil exploration to relaxing the
(b) It will show the victory of mind over matter. standards for drinking water, Mr Bush’s policies are not exactly
(c) They will realise that it is not an end in itself. beneficial — not even to American interests. We realise that
(d) They will get involved in power politics and drift we are all in this together and that pollution anywhere should
into the danger of death. be a global concern. Otherwise there will only be more tunnels
(e) None of these at the end of the tunnel.
24. Which of the following statements is/are true in the context 26. Both official and corporate India are allergic to
of the passage? (a) failure of monsoon
(A) Science and technology are neither good nor bad. (b) poverty and inequality
(c) slowdown in industrial production
(B) Through machines man can achieve all progress.
(d) mention of clean technology
(C) Science and technology bring great risks and
(e) crop failure
temptations.
27. Which, according to the passage, is a life-and-death
(a) A and B (b) A and C question for many Indians?
(c) B and C (d) A, B and C (a) Increase in respiratory diseases
(e) None of these (b) Use of clean technology
25. Choose the word which is SIMILAR in meaning to the (c) Thick blanket of pollution over the region
word ‘tabooed’ as used in the passage. (d) Failure in crops
(a) Confined (b) Accepted (e) Dwindling agricultural yield
(c) Damaged (d) Criticised 28. If the rate of premature deaths increases it will
(e) Forbidden (a) exert an added burden on our crumbling economy.
Passage 4 (b) have adverse social and economic consequences.
Radically changing monsoon patterns, reduction in the winter (c) make a positive effect on our efforts to control
rice harvest and a quantum increase in respiratory diseases-all population.
part of the environmental doomsday scenario which is (d) have less job aspirants in the society.
(e) have a healthy effect on our economy.
reportedly playing out in South Asia. According to a United
29 Choose the word which is similar in meaning to the
Nations Environment Programme report, a deadly three-km-
word ‘profligacy’ a used in the passage.
deep blanket of pollution comprising a fearsome cocktail of
(a) wastefulness (b) conservation
ash, acids, aerosols and other particles has enveloped this (c) upliftment (d) criticalness
region. For India, already struggling to cope with a drought, (e) denouncement
the implications of this are devastating and further crop failure 30. According to the passage, India cannot tolerate any
will amount to a life-and-death question for many Indians. The further
increase in premature deaths will have adverse social and (a) crop failure
economic consequences and a rise in morbidities will place an (b) deterioration of health care system
unbearable burden on our crumbling health system. And there (c) increase in respiratory diseases
is no one to blame but ourselves. Both official and corporate (d) proliferation of nuclear devices
India have always been allergic to any mention of clean (e) social and economic consequences
READING COMPREHENSION C-49
31. According to the passage, the two-wheeler industry is 38. According to the passage, Johannesburg meet is going to
not adequately concerned about witness
(a) passenger safety on the roads (a) calm and dispassionate thinking on the issue of
(b) life cover insurance of the vehicle owners pollution control.
(c) pollution control system in the vehicles (b) a blame game between developed and developing
(d) rising cost of the two-wheeler countries.
(e) rising cost of petrol in the country (c) refusal of UN to work as the arbitrator.
32. What could be the reason behind the timing of the haze (d) the US agreeing to look at the issue of lowering its
report just before the Johannesburg meet as indicated in consumption.
the passage? (e) countries agreeing for higher monetary allocation to
(a) The United Nations is working hand in glove with
R & D.
the US.
39. Choose the word which is most opposite in meaning of the
(b) Organisers of the forthcoming meet want to teach a
word ‘dubious’ as used in the passage?
lesson to the US.
(a) Unquestionable (b) Dissimilar
(c) Drawing attention of the world towards devastating
effects of environmental degradation (c) Illegal (d) Anti-social
(d) The US wants to use it as a handle against the (e) Innovative
developing countries in the forthcoming meet 40. Choose the word which is the most opposite in meaning
(e) The meet is a part of political agenda of the UN. to the word ‘morbidity’ as used in the passage?
33. Choose the word which is similar in meaning to the word (a) Powerfulness (b) Healthiness
‘allergic’ as used in the passage. (c) Softness (d) Acuteness
(a) Liking (b) Passionate (e) Purposeful
(c) Possessive (d) Crumbling Passage 5
(e) Repugnant Child psychology is certainly not a strong point with most
34. Which of the following is the indication of environmental Indian schools. Why else would it inflict a double trauma on a
degradation in South Asia? student faring badly in the pre-boards by banning her from
(a) Social and economic inequality taking the exams? Often with fatal results as evidenced by
(b) Crumbling health care system reports of student suicides in the run-up to the board. Now, the
(c) Inadequate pollution control system Central Board of Secondary Education (CBSE) has stepped in
(d) Overemphasis on technology and put the brakes on. This is good news for parents and
(e) Radically changing monsoon pattern students, many of whom have had to live with the threat of the
35. What must we realise, according to the passage? performance-linked department. While the schools’ logic is that
(a) No country should show superiority over other
in order to attract talented students they need to maintain their
countries.
performance records at high levels, the assumption that a student
(b) The UN is putting in hard efforts in the direction of
faring poorly in the pre-boards will replicate this at the boards
pollution control.
(c) All countries must join hands in fighting pollution. is faulty. Chances are that the student will be spurred to work
(d) Nobody should travel through a tunnel to avoid doubly hard. On the other hand, the threat of the department
health hazards. will almost certainly impact her performance adversely. Of course,
(e) We all must strive hard to increase agricultural linking pre-boards to the boards is only one of the problems
production. with our school system.
36. Which of the following finds place in the United Nations 41. Choose the word which is MOST OPPOSITE in meaning
Environment Programme Report? of the word ‘spur’ as used in the passage.
(a) Changing monsoon patterns (a) depress (b) enlarge
(b) Substantial increase in respiratory diseases (c) explicate (d) sustain
(c) A serious cover of pollution over the region (e) activate
(d) Reduction in winter rice harvest 42. Which is the good news for parents, according to the
(e) None of these passage?
37. Which of the following statements is not true in the (a) Schools will take the responsibility of preparing
context of the passage? students for the board.
(a) UN environment report blames countries like India (b) Schools will provide study facilities to the poor
and China. students.
(b) Developing countries have taken environment short- (c) Schools will enforce discipline to ensure higher
cuts in their bid to catch up with the west. attendances of students.
(c) US is also to be blamed for environmental degradation (d) No student can be barred from the boards without
and pollution. prior clearance from the CBSE.
(d) Indians cannot afford to have any further crop failure. (e) Teachers will be able to handle students well if they
(e) US has tightened safety standards for drinking water. know child psychology.
C-50 READING COMPREHENSION
43. What is the ruling of the CBSE? first and Indians last. It is time we remind ourselves what the
(a) Students must pass the pre-board exam before great visionary and builder of modern India Jawaharlal Nehru
appearing for the board exam. said, “Who dies if India lives, who lives if India dies?” We
(b) Schools should follow the practice of performance must realise, and this is unfortunately what many in public life
linked department. tend to overlook, sometimes out of ignorance of the forces of
(c) Schools should maintain the performance record of history and sometimes deliberately with a view to promoting
students at high level. their self-interest, that national interest must inevitably and
(d) Schools must motivate students to work hard. forever prevail over any other considerations proceeding from
(e) Before barring any student for the board schools regional, linguistic or communal attachments. The history of
must take prior permission of the CBSE. India over the past centuries bears witness to the fact that
44. What is the faulty assumption of schools, according to India was at no time a single political unit. Even during the
the passage? reign of the Maurya dynasty, though a large part of the country
(a) Students who do not do well at pre-boards will be was under the sovereignty of the Mauryan kings, there were
motivated to work hard. considerable portions of the territory which were under the rule
(b) Pre-boards are generally easy and therefore students of independent kingdoms. So also during the Mughal rule
take them lightly. which extended over large parts of the territory of India, there
(c) Students who fare poorly at the pre-board will fail at were independent rulers who enjoyed political sovereignty over
the boards. the territories of their respective kingdoms. It is an interesting
(d) Learning by rote is a better method of learning fact of history that India was forged into a nation, neither on
(e) Students perform well in languages than in science account of a common language nor on account of the continued
subjects. existence of a single political regime over its territories but on
45. Which of the following, according to the passage, is the account of a common culture evolved over the centuries. It is
problem with our school system? cultural unity—something more fundamental and enduring than
(a) Providing study facilities to the students any other bond which may unite the people of a country
(b) Linking pre-board performance of students to the together which has welded this country into a nation. But until
boards the advent of the British rule, it was not constituted into a
(c) Teachers’ lack of knowledge of child psychology single political unit. There were, throughout the period of
(d) Attracting talented students history for which we have fairly authenticated accounts, various
(e) Low percentage of students passing the board exam kingdoms and principalities which were occasionally engaged
46. According to the passage, parents had to live with the in conflict with one another. During the British rule, India
threat of became a compact political unit having one single political
(a) falling grades of their wards. regime over its entire territories and this led to the evolution
(b) not getting their wards admitted in quality schools. of the concept of a nation. This concept of one nation took firm
(c) schools not treating their wards with the attitude of roots in the minds and hearts of the people during the struggle
counsellor. for independence under the leadership of Mahatma Gandhi. He
(d) linking performance of their wards in pre-boards to has rightly been called the Father of the Nation because it was
the boards. he who awakened in the people of this country a sense of
(e) schools creating traumatic situations for their wards. national consciousness and instilled in them a high sense of
47. Schools wanted to enforce performance-linked department patriotism without which it is not possible to build a country into
in order to nationhood. By the time the Constitution of India came to be
(a) get regular grant-in -aid from th e education enacted, insurgent India, breaking a new path of non-violent
department. revolution and fighting to free itself from the shackles of foreign
(b) improve their public image as a social institution. domination, had emerged into nationhood and “the people of
(c) attract better quality students. India” were inspired by a new enthusiasm, a high and noble
(d) make students aware that they would aspire for their spirit of sacrifice and above all, a strong sense of nationalism
all-round development. and in the Constitution which they framed. They set about the
(e) provide better study-material to the students. task of a strong nation based on certain cherished values for
48. Choose the word which is SIMILAR in meaning to the which they had fought.
word ‘replicate’ as used in the passage. 49. The author has quoted Jawaharlal Nehru to emphasise
(a) enhance (b) repeat the point that
(c) perform (d) achieve (a) national interest must enjoy supreme importance
(e) plunder (b) India is going to survive even if the world is under
Passage 6 the spell of destruction
We find that today the unity and integrity of the nation is (c) the world will be destroyed if India is on the threshold
threatened by the divisive forces of regionalism, linguism and of destruction
communal loyalties which are gaining ascendancy in national (d) the survival of the world depends only upon the well
life and seeking to tear apart and destroy national integrity. We being of India
tend to forget that India is one nation and we are all Indians (e) None of these
READING COMPREHENSION C-51
50. What, according to the author, is the impact of the divisive 57. Why do people tend to overlook the paramount importance
forces on our nation? of national interest?
(a) They promote a sense of regional pride. A) Because they are unaware of the imperative need of
(b) They help people to form linguistic groups. the day
(c) They separate groups of people and create enmity B) Because they give undue importance to their selfish
among them. motives
(d) They encourage among people the sense of loyalty C) Because historical events force them to do so
to their community. (a) Only A (b) Only B
(e) They remind us of our national pride. (c) Only C (d) A and B only
51. “Communal loyalties” have been considered by the author (e) B and C only
as
58. The “people of India”, as highlighted by the author in the
(a) a good quality to be cherished
last sentence of the passage, refer to
(b) of no consequence to the nation
(c) a very important aspect for nation-building (a) the people of one unified nation
(d) a threat to the solidarity of the nation (b) the subjects of several independent rulers
(e) None of these (c) the patriots who sacrificed themselves in the freedom
52. Which of the following was instrumental in holding the struggle
different people of India together? (d) the people who were instrumental in writing the
(a) A common national language Constitution
(b) A common cultural heritage (e) None of these
(c) The endurance level of the people 59. India’s insurgence was for
(d) Fundamentalist bent of mind of the people (a) breaking the path of non-violence
(e) None of these (b) having one common national language
53. The passage appears to have been written with the purpose (c) insisting on a unique cultural identity
of
(d) several independent sovereign rulers
(a) giving a piece of advice to politicians of free India
(e) None of these
(b) assessing the patriotic values and sacrifices made
by people for India’s freedom 60. Transformation of our country into nationhood was
(c) justifying the teaching of Mahatma Gandhi and its possible because of
impact on the people (A) People’s spontaneously referring to Mahatma Gandhi
(d) giving a historical account of how India evolved as as the Father of the Nation
a nation (B) People’s sense of national consciousness
(e) None of these (C) Generation of a high sense of dedication to the nation
54. History shows that India, which was not a political unit among the people
earlier, became so (a) A and B only (b) A and C only
(a) during the reign of Maurya dynasty (c) B and C only (d) All the three
(b) during the Mughal rule
(e) None of these
(c) after one-national-language policy was adopted
Directions (Qs. 61-64): Choose the word/group of words which
(d) during the regime of independent rulers
(e) during the British rule is most nearly the SAME in meaning as the word given in bold
55. Which of the following statements is/are definitely true as used in the passage.
in the context of the passage? 61. awakened
(A) The people of India had fought for certain values. (a) moved (b) segregated
(B) The fight of the Indian people was for one common (c) extracted (d) kindled
culture. (e) supported
(C) The Indian people lacked sense of nationalism until 62. cherished
they gained freedom. (a) maintained carefully
(a) Only A (b) Only B (b) available abundantly
(c) Only C (d) A and B only (c) managed tactfully
(e) A and C only
(d) accepted happily
56. Which of the following, according to the passage, was
commonly applicable to both: the Maurya dynasty rule (e) protected lovingly
and Mughal rule? 63. authenticated
(A) A vast territory under governance (a) established (b) documented
(B) Various independent sovereign rulers under one (c) hea rsay (d) audited
major ruler (e) maintained
(C) Lack of political unity under the common governance 64. proceeding
(a) A and B only (b) B and C only (a) escaping (b) ranging
(c) A and C Only (d) All the three (c) emanating (d) deviating
(e) None of these (e) freeing
C-52 READING COMPREHENSION
Directions (Qs. 65-68): Choose the word which is most It measured a woman’s position in the home—whether she
OPPOSITE in meaning of the word given in bold as used in the works for cash, her age at marriage, and the difference in age
passage. and education between spouses. The study concludes that the
65. considerable lowly position of women in the family is the single most important
(a) inconsiderate (b) uncountable reason for the gap in children’s nutrition between South Asia
(c) unfathomable (d) irresolute and sub-Saharan Africa, followed by sanitation (lack of latrines)
(e) negligible and urbanisation (slum living).
66. welded
I wonder why the position of women in India is worse than that
(a) disjointed (b) installed
of women in other societies. The report seemed to suggest that
(c) disembarked (d) dislocated
(e) thwarted South Asian women were not so far behind African women as
67. attachments their inferior status too limited their ability to nurture children.
(a) predicaments (b) hatred I also wonder whether children’s well being is only a woman’s
(c) harmony (d) mistrust issue or a family concern where men play a crucial role. I
(e) loyalty suspect there are no easy answers. Women everywhere suffer
68. deliberately from lower status, but in India it appears to have devastating
(a) reluctantly (b) unintentionally consequences. The policy implications are clear: if we want to
(c) unauthorisedly (d) wrongly reduce child malnutrition, we must combine our child
(d) notoriously programmes with efforts to improve the situation of women. To
Passage 7 succeed, we need healthy children who’ll become tomorrow’s
The stubborn persistence of child malnutrition in India is one innovative adults. If we ignore gender inequality, we will continue
of the tragedies of our time. Many of us have long agonised to produce stunted children, wasted lives, and untold misery.
over this preventable problem, and we continue to ask: why do
69. A hypothesis related to low birth weight has now been
half of our children not get enough or the right food or adequate
confirmed. According to this, the major reason for this
care? Even in sub-Saharan Africa, only 30 per cent of the
children are malnourished, versus 50 per cent in South Asia. state is
And this gap exists despite our much higher levels of per (a) Vegetarianism
capita income, education and even safer water access. One- (b) Illiteracy
third of the babies in India are born with low birth weight (c) Illiteracy of women
compared to one-sixth in sub-Saharan Africa. This is (d) Status of women
heartbreaking given the dramatic improvements in our (e) Slum living
agriculture, advances in literacy, and great strides in economic 70. Which type of scheme indicates that there was no lack
growth. For more than 20 years India has even sustained the of efforts in India for the last two decades to improve the
greatest effort in history to improve nutritional standards, situation?
according to UNICEF, through its Integrated Child Development
(a) Literacy
Services (ICDS) Programme. So it is not for lack of effort. Nor
(b) Rural Development
is it due to poverty, which has been steadily declining by one
per cent a year for two decades. What accounts for this puzzle? (c) Child Development
In 1996, India’s famous physician nutritionist wrote a ground- (d) Family Planning
breaking article on this called 'The Asian Enigma’. After (e) Poverty Alleviation
considering different factors, including access to food and 71. According to the author,
income and our vegetarianism, he concluded that the lower (a) child malnutrition can be reduced with the help of
status of women might be the reason. The link between women’s child nutrition schemes.
status and child nutrition seems plausible. In many Indian (b) increased family income would result in better pre
homes, men eat first; women have to make do with leftovers. and post-natal care.
This is perhaps why 83 per cent of women in India suffer from (c) men should play more involved role in children’s
iron deficiency-anaemia versus 40 per cent in sub-Saharan Africa.
well being.
A malnourished mother will give birth to a baby with low birth
(d) India has not put sustained efforts to improve
weight. Moreover, domestic work often forces a mother to
delegate the chore of feeding solid food to her baby to older nutritional standards.
siblings. If women had more control over family income and (e) 30% of our children still do not get enough food.
decisions, they would devote them to better pre and post-natal 72. Which of the following is the major reason for a large
care and to their children. number of women in India suffering from iron deficiency?
So far this was the theory. But now a study by the International (a) Women are not getting sufficient food
Food Policy Research Institute and Emory University seems to (b) More women eating only vegetarian food
confirm this hypothesis. It brought together data from 36 (c) Women not eating balanced food
developing countries, spanning over one hundred thousand (d) Lack of proper medical check-up
children under the age of three and an equal number of women. (e) None of these
READING COMPREHENSION C-53
73. According to the passage, the problem of child Passage 8
malnutrition Job performance is affected by a number of factors. Motivation
(a) cannot be prevented alone does not lead to increase in performance. Ability and
(b) is not so severe now technology moderates the relationship between motivation and
(c) is the same in all the developing nations performance. The higher the levels of ability and motivation,
(d) is linked with poverty the higher the level of performance will be. However, increasing
(e) None of these motivation beyond an optimal level tends to produce a
74. Choose the word that is SIMILAR in meaning to the dysfunctional result because it is accompanied by an increasing
word ‘plausible’ as used in the passage. level of anxiety. A high level of anxiety often disrupts
(a) deceptive (b) certain performances.
(c) feeble (d) likely The relationship between satisfaction and performance is not
(e) uncertain clear. Satisfaction may or may not lead to high performance
75. Choose the word that is SIMILAR in meaning to the depending on the perceived availability of valued outcomes
word ‘stubborn’ as used in the passage. and the perceived expectancy that a person’s effort and
(a) incurable (b) determined performance will lead to receiving the valued rewards. If the
(c) unduly (d) regular person expects that his performance will lead to increased
(e) different rewards which he values, the level of his motivational effort will
76. Choose the word that is SIMILAR in meaning to the increase; if he anticipates less, his motivational effort will be
word ‘spanning’ as used in the passage.
lower.
(a) covering (b) pointing
The relationship between job dissatisfaction and poor
(c) bringing (d) improving
performance seems to be clearer than that between satisfaction
(e) duration
77. In which of the following areas is South Asia’s and performance. Dissatisfaction leads to poor performance by
performance better than that of sub-Saharan Africa? means of apathy, absenteeism, turnover, sabotage, and strike.
(a) Safer drinking water In addition, high performers are more vulnerable to job
(b) Lower infant mortality rate dissatisfaction because they tend to expect more from their
(c) Higher status of women jobs than low performers.
(d) Higher birth weight of children Job satisfaction is more closely related to the decision to join
(e) None of these and remain in an organisation than to the motivation to produce.
78. According to the author, the crux is The motivation to produce largely depends on the availability
(a) women have lower status everywhere as compared of valued outcomes (valence), the perceived instrumentality of
to men. performance for receiving incentive rewards, and the perceived
(b) improvement of sanitation and slum conditions. expectancy that effort leads to performance. The task of
(c) that in India, the per capita income and education satisfying employees is much easier than the task of motivating
level of women is very low. them because the former can be achieved by rewarding them
(d) low status of women has a horrifying result on child while the latter requires such additional constraints as
malnutrition.
establishing performance-reward contingencies and designing
(e) None of these
motivating work systems.
79. Which of the following was one of the measures of
82. Choose the word that is SIMILAR in meaning to the
women’s position in the home?
(a) Number of children word ‘moderate’ as used in the passage.
(b) Difference in husband’s and wife’s income. (a) produce (b) increase
(c) Weights of child at birth (c) affect (d) reduce
(d) Age of marriage (e) explain
(e) None of these 83. The individual’s decision to remain in the organisation
80. What according to the passage is heartbreaking? depends on
(a) India’s performance lower than Africa (a) relationship between satisfaction and performance
(b) Failure of ICDS programme (b) the level of anxiety induced by the job
(c) Sub-Saharan Africa’s every one-sixth child being (c) his level of motivation
born with low weight (d) the level of job satisfaction
(d) The puzzle of poverty declining only by one per cent (e) None of these
a year 84. Which of the following tasks is easier according to the
(e) Higher status of women in Asia passage?
81. Choose the word that is SIMILAR in meaning to the (a) Satisfying employees
word ‘dramatic’ as used in the passage. (b) Motivating the employees
(a) Literary (b) Striking
(c) Increasing the ability level of employees
(c) Insignificant (d) Exaggerated
(d) Reducing the anxiety level of employees
(e) Doctored
(e) None of these
C-54 READING COMPREHENSION
85. Which of the following statement/s is/are true in the (c) Cultural development
context of the passage? (d) Archaeological reserves
(A) Ability leads to performance. (e) Diverse wild life
(B) Job satisfaction certainly leads to higher performance. 91. Which of the following combinations of periods of Indian
(C) High anxiety adversely affects performance. culture is incorrect as mentioned in the passage?
(a) (A) and (B) only (b) (B) and (C) only (a) The Puranas, the Mahabharata, the medieval saints
(c) (A) and (C) only (d) (A) only (b) The Smritis, the modern reformers, the Vedas
(e) (B) only (c) The Upanishads, the philosophical system, the
86. Which of the following combination of factors affects job Smritis
performance? (d) The Puranas, the Ramayana, the Mahabharata
(a) Job satisfaction and Motivation (e) The modern reformers, the Vedas, the medieval saints
(b) Motivation and Ability 92. What changes has the spirit of Indian culture undergone
(c) Job Satisfaction and Ability during the long period of history right from the Vedic age
(d) Job Satisfaction, Motivation and Ability
down to the present time?
(e) None of these
(a) The prevalence of moral values was eclipsed at
87. High level of anxiety
certain periods of time.
(a) produces higher motivation
(b) increases the level of ability (b) The spirit of Indian culture has remained unchanged
(c) strengthens the relationship between motivation and from the ancient times down to the present.
performance (c) Materialism was the hallmark of Indian culture during
(d) decreases job satisfaction certain periods of time
(e) None of these (d) There is no such thing as any spirit of Indian culture
88. The task of motivating employees is difficult due to (e) During certain periods authoritarian values dominated
(a) apathy and lack of enthusiasm of employees over democratic values.
(b) difficulty in establishing relationship between 93. Choose the word which is SIMILAR in meaning as the
satisfaction and performance word perpetuating as used in the passage.
(c) difficulty in monitoring ability level of employees (a) Continuing (b) Appreciating
(d) unavailability of attractive rewards in organisations (c) Enjoying (d) Languishing
(e) difficulty in designing a motivating work system (e) Confirming
89. Choose the word that is MOST OPPOSITE in meaning 94. Which of the following statements is NOT TRUE in the
of the word ‘apathy’ as used in the passage. context of the passage?
(a) Satisfaction (b) Health (a) Mighty and cruel hand of time spares none.
(c) Enthusiasm (d) Discipline (b) Assyria was a great empire and nation.
(e) Reward (c) The culture of the Vedic ages and the ages of the
Passage 9
Upanishads is different in form.
Now, the question arises: what is the secret of the longevity (d) Hindu religion is growing and changing.
and imperishability of Indian culture? Why is it that such great (e) Indian culture is transient and ephemeral.
empires and nations as Babylon, Assyria, Greece, Rome and 95. “... could not last more than the footprints of a camel on
Persia could not last more than the footprints of a camel on the shifting sands of the deserts”. What does this
the shifting sands of the deserts, while India, which faces the expression mean as used in the passage?
same ups and downs, the same mighty and cruel hand of time, (a) It lost itself in desert.
is still alive and with the same halo of glory and splendour? (b) It was transient.
The answer is given by Prof JB Pratt of America. According to (c) It lacked solidity.
him, Hindu religion is “self-perpetuating and self-renewing.”
(d) It was limited only to desert area.
Unlike other religions, “not death, but development” has been
(e) It lacked cohesion.
the fate of Hinduism. Not only Hindu religion but the whole
96. What is the characteristic quality of the basic principles
culture of the Hindus has been growing, changing, and
developing in accordance with the needs of the times and of Indian culture?
circumstances without losing its essentially imperishable spirit. (a) They are static.
The culture of the Vedic ages, of the ages of the Upanishads, (b) They derive their strength from the genius of people.
the various philosophical systems, the Mahabharata, the Smritis, (c) They believe in the purity of Indian culture.
the Puranas, various scholarly commentators, the medieval (d) They can be adapted to almost any environment.
saints, and the age of modern reformers is the same in spirit (e) They project the glimpses of ancient civilization.
and yet very different in form. Its basic principles are so broad- 97. What, according to the author, has always characterized
based that they can be adapted to almost any environment of the Hindu religion?
development. (a) Spirituality and reactivity
90. In what respect is India implied to be superior to all other (b) Proactivity and individual dignity
nations and empires? (c) Dynamism and growth
(a) Democratic traditions (d) Morality and stagnation
(b) Territorial expansion (e) Collective wisdom and democracy
READING COMPREHENSION C-55
98. Choose the word which is MOST OPPOSITE in meaning (c) People will desire simple lifestyle.
of the word ‘last’ as used in the passage. (d) The advertisements will play down the comfort aspect
(a) Grow (b) Diminish of goods.
(c) Force (d) Respond (e) None of these
(e) End 103. What was the characteristic of affluent men of an earlier
99. What, according to JB Pratt, is the secret of the longevity age?
and imperishability of Indian culture? (a) He used to put higher premium on comfort.
(a) It has its origin in the remote past. (b) He was relying much on advertisements.
(b) It stems from the minds and hearts of its sages. (c) He believed more in simple and cheaper things.
(c) It is founded on religion. (d) He was more qualitative in his emphasis rather than
(d) It is founded on universal moral values. being quantitative.
(e) It is self-perpetuating and self-renewing. (e) His emphasis was on beauty.
Passage 10 104. What change according to the author has taken place in
Comfort is now one of the causes of its own spread. It has now the attitude towards comfort?
become a physical habit, a fashion, an ideal to be pursued for (a) It is taken for granted in the modern way of living.
its own sake. The more comfort is brought into the world, the (b) It has become now an ideal to be pursued for its own
more it is likely to be valued. To those who have known sake.
comfort, discomfort is a real torture. The fashion which now (c) It is now believed that discomfort handicaps thought.
decrees the worship of comfort is quite as imperious as any (d) It is thought that comfort helps body and mind to
other fashion. Moreover, enormous material interests are bound function effectively.
up with the supply of the means of comfort. The manufacturers (e) None of these
of furniture, of heating apparatus, of plumbing fixtures cannot 105. Choose the word which is SIMILAR in meaning to the
afford to let the love of comfort die. In modern advertisements word ‘decree’ as used in the passage.
they have found a means for compelling it to live and grow. A (a) order (b) spread
man of means today, who builds a house, is in general (c) project (d) attract
concerned primarily with the comfort of his future residence. (e) exhibit
He will spend a great deal of money on bathrooms, heating 106. Why does the author value comfort?
apparatus, padded furnishings, and having spent he will regard (a) It helps to project one’s image.
his house as perfect. His counterpart in an earlier age would (b) It helps to protect your values.
have been primarily concerned with the impressiveness and (c) It facilitates mental life.
magnificence of his dwelling with beauty, in a word, rather than (d) It encourages a blend of materialistic and spiritual
comfort. The money our contemporary would spend on baths thinking.
and central heating would have been spent on marble staircases, (e) None of these
frescoes, pictures and statues. I am inclined to think that our 107. Why would manufacturers of various devices not permit
present passion for comfort is a little exaggerated. Though I comfort to die?
personally enjoy comfort, I have lived most happily in houses (a) They want to manufacture more and more comfort
devoid of everything that Anglo-Saxons deem indispensable. goods.
Orientals and even South Europeans who know not comfort (b) Manufacturers are mainly interested in creating new
and live very much as our ancestors did centuries ago seem to things.
go on very well without our elaborate apparatus and padded (c) Manufacturers’ emphasis is on producing beautiful
luxuries. However, comfort for me has a justification; it facilitates things.
mental life. Discomfort handicaps thought; it is difficult to use (d) Their prosperity is closely linked with the people’s
the mind when the body is cold and aching. desire for comfort.
100. Choose the word that is SIMILAR in meaning to the (e) None of these
phrase devoid of as used in the passage. 108. Choose the word which is MOST OPPOSITE in meaning
(a) available (b) lacking of the word ‘indispensable’ as used in the passage.
(c) empty (d) false (a) unattractive (b) avoidable
(e) deficient (c) favourable (d) unelegant
101. How do people manage to keep the love of comfort alive? (e) comfortable
(a) By pumping in more comfort goods in the market 109. Which of the following statements is NOT TRUE in the
(b) By sacrificing high profit on comfort goods context of the passage?
(c) By targeting youths in the sales campaign (a) Discomfort is not liked by those who live in comfort.
(d) By appealing to the emotionality of people (b) The affluent man of an earlier age was interested
(e) None of these more in beauty than in comfort.
102. What is the author’s prediction about comfort? (c) Discomfort handicaps thought.
(a) The value of comfort will increase. (d) Orientals and South Europeans love comfort
(b) People will value more spirituality thus reducing the immensely.
value of comfort. (e) The author of the passage enjoys comfort.
C-56 READING COMPREHENSION
Passage 11 cables to create a Tamil Network which will offer ration cards,
We tend to be harsh on our bureaucracy, but nowhere do school, college and hospital admission forms, land records, and
citizens enjoy dealing with their government. They do it because pension records. If successful, WorldTel will expand the network
they have to. But that doesn’t mean that the experience has to to Gujarat, Karnataka and West Bengal. In Kerala, all the villages
be dismal. Now there is a new wind blowing through are getting linked online to the district headquarters, allowing
government departments around the world, which could take citizens to compare the development priorities of their villages
some of this pain away. In the next five years it may well with other villages in the State.
transform not only the way public services are delivered but Many are still skeptical of the real impact because so few
also the fundamental relationship between governments and Indians have computers. The answer lies in interactive cable
citizens. Not surprisingly, it is the Internet that is behind it. TV and in Internet kiosks. Although India has only five million
After e-commerce and e-business, the next revolution may be computers and thirty-eight million telephones, it has thirty-four
e-governance. million homes with cable TV and these are growing eight per
Examples abound. The municipality of Phoenix, Arizona, allows cent a year. By 2005 most cable homes will have access to the
its citizens to renew their car registrations, pay traffic fines, Internet from many of the 700000 local STD/PCO booths. Internet
replace lost identity cards, etc, online without having to stand usage may be low today, but it is bound to grow rapidly in the
in endless queues in a grubby munipal office. The municipality future, and e-governance in India may not be a dream.
is happy because it saves $5 a transaction. It costs only $1.60 110. According to the passage, which country has the most
ambitious plan for e-governance?
to process an online transaction versus $ 6.60 to do it across
(a) USA (b) Chile
the counter. In Chile, people routinely submit their income tax
(c) Singapore (d) India
returns over the Internet, which has increased transparency,
(e) UK
and drastically reduced the time taken and the number of errors 111. GovWorks is working in which of the following countries?
and litigation with the tax department. Both tax payers and the (a) India (b) UK
revenue department are happier. (c) Chile (d) Singapore
The furthest ahead, not surprisingly, is the small, rich and (e) None of these
entrepreneurial civil service of Singapore, which allows citizens 112. Choose the word that is opposite in meaning of the word
to do more functions online than any other. As in many private “dismal” as used in the passage.
companies, the purchasing and buying of Singapore’s (a) grim (b) approve
government departments is now on the Web, and cost benefits (c) pleasing (d) better
come through more competitive bidding, easy access to global (e) enrich
suppliers and time saved by online processing of orders. They 113. How can India overcome low penetration of computers
can post their catalogues on their site, bid for contracts, submit fore-governance?
invoices and check their payment status over the Net. (a) By manufacturing more computers
The most useful idea for Indian municipalities is GovWorks, a (b) Through cable TV and Internet Kiosks
private sector-run site that collects local taxes, fines and utility (c) By opening more STD/PCO booths
bills for 3600 municipalities across the United States. It is a (d) By making the Internet free
citizen’s site, which also provides information on government (e) By putting more services on the Internet
jobs, tenders, etc. The most ambitious is the British government, 114. Which of the following has not been one of the effects
which has targeted to convert 100 per cent of its transactions of submitting income tax returns over the Internet in
with its citizens to the Internet by 2005. Chile?
Cynics in India will say, ‘Oh e-governance will never work in (a) Reduction of legal cases
India. We are so poor and we don’t have computers.’ But they (b) Reduction in errors
(c) Increase in transparency
are wrong! There are many experiments afoot in India as well.
(d) Increase in number of returns
Citizens in Andhra Pradesh can download government forms
(e) Reduction in time taken
and submit applications on the Net without having to bribe 115. Choose the word that is the same in meaning as the word
clerks. In many districts, land records are online and this has “abound” as used in the passage.
created transparency. Similarly, in Dhar district of Madhya (a) around (b) proliferate
Pradesh, villagers have begun to file applications for land (c) flourish (d) plentiful
transfers and follow their progress on the Net. In seventy (e) few
villages in the Kolhapur and Sangli districts in Maharashtra, 116. Choose the word that is the same in meaning as the word
Internet booths have come up where farmers can daily check “post” as used in the passage.
the market rates of agricultural commodities in Marathi, along (a) deliver (b) send
with data on agricultural schemes, information on crop (c) put up (d) drop out
technology, when to spray and plant their crops and bus and (e) later
railway time tables. They also find vocational guidance on 117. According to the passage, which country is at present
jobs, applications for ration cards, kerosense/gas burners and the most advanced in e-governance?
land record extracts with details of land ownership. (a) Singapore (b) Chile
Sam Pitroda’s WorldTel, Reliance Industries and the Tamil (c) India (d) USA
Nadu government are jointly laying 3000 km of optic fibre (e) UK
READING COMPREHENSION C-57
118. In which direction is the new wind blowing? employment situation with a comprehensive coverage of new
(a) More and more interaction of citizens with economic establishments. For instance, the various economic
government through Internet censuses are an important source of information on the changing
(b) Outsourcing the work of infrastructure creation for employment profile of, say, the nation’s capital. Far from being
Internet a bureaucrat-dominated city, Delhi over the years has become
(c) Increasing the penetration of computers in rural areas more of an industrial metropolis. According to the fourth
(d) Integrating e-commerce, e-business and e-governance economic census, manufacturing accounted for 40 per cent of
(e) Introducing e-governance programmes in schools and jobs in the capital. The employment exchanges in the capital
colleges thus have their work cut out notably, to shift the focus away
119. According to the passage, what is the annual growth rate from government and public sector jobs more towards
of computers in India? placements in the private sector, especially in manufacturing
(a) 8% (b) 5%
and services, including the burgeoning retail trade sector. By
(c) 0.5% (d) Not mentioned
doing so, they will better reflect the imperatives of economic
(e) None of these
120. Compared to across-the-counter, the cost of online reform and remain relevant in today’s times.
transaction is 122. Choose the word that is opposite in meaning of the word
(a) little less “spurning” as used in the passage.
(b) substantially less (a) thronging (b) evaluating
(c) more or less the same (c) criticising (d) following
(d) little more (e) rejecting
(e) ` 4/- less per transaction 123. Which of the following revamped role can be entrusted
121. According to the author, e-governance in India to employment exchanges?
(a) is a dream and may not succeed (a) Conducting economic surveys
(b) will not succeed unless more computers are owned (b) To conduct vocational training programme for the
by citizens unemployed
(c) has witnessed successful attempts and plans (c) To modernise registration process through the
(d) will not work because the model is suited for Internet
developed countries (d) To reduce the number of exchanges
(e) though will bring transparency, will increase (e) None of these
corruption
124. What can be inferred about the employment exchanges
Passage 12 outside Delhi?
Employment exchanges — one of the surviving bastions of (a) The registration in them would be much less.
babudom — face the prospect of becoming irrelevant in an era (b) Their condition will be worse.
of reform. Even in the heart of the nation’s capital, the premises (c) Their condition will be better.
are often dilapidated structures with dirty passages and manned (d) They focus more on manufacturing sector.
by surly staff. Not surprisingly, job-seekers hardly throng these (e) None of these
exchanges. Paradoxically, when jobs are getting scarce due to 125. Choose the word that is same in meaning as “imperatives”
pressure of liberalisation, job-seekers are spurning an institution as used in the passage.
intended to help them secure placements. The reasons are (a) importance (b) implication
simple enough. Employment exchanges still concentrate on (c) urgency (d) indication
government and public sector placements, which are fast losing (e) authority
ground in the labour market. For most government jobs, the 126. Choose the word that is same in meaning as the word
eligibility criterion is still registration with the employment “secure” as used in the passage.
exchanges. But what is the use of going through the formalities (a) fasten (b) safe
of registration when government jobs themselves are dwindling? (c) obtain (d) re-assure
The placement effected by all the 939-odd exchanges in the (e) lock
country in 2001 was of the order of 1.69 lakh against annual 127. Choose the word that is opposite in meaning of the word
registration levels of 60 lakh. As there are too few jobs when “burgeoning” as used in the passage.
compared to the number of job-seekers, the accumulated backlog (a) flourishing (b) loss-making
of registrations is close to 4.16 crore. The latter of course (c) expanding (d) distressing
doesn’t indicate unemployment levels as those registered with (e) declining
the employment exchanges are not necessarily unemployed. 128. In order to remain relevant, which of the following should
How can the employment exchanges be revamped? The thinking be the focus of employment exchanges?
in the Union labour ministry is to transform them into (a) To make efforts to increase their registration
employment promotion and guidance centres. The plan includes (b) To shift attention to jobs in private sector
modernisation, changing the mindset of the staff and making (c) To shift focus to jobs in manufacturing in public
them into an effective instrument for monitoring and coordinating sector organisations
various employment generation schemes. This objective calls (d) To reform exchanges by recruiting trained staff
for developing a better database on the fast changing (e) To obtain more grants from government
C-58 READING COMPREHENSION
129. Which of the following is not true in the context of the 131. The author thinks that openness in budget is essential as
passage? it leads to
(a) Those who register with the employment exchange (a) prevention of tax implications
inform them if they are able to get the job on their (b) people’s reluctance to accept their moral duties
own. (c) exaggerated revelation of the strengths and
(b) The annual placement arranged by employment weaknesses of economy
exchanges is less than 3% of the registration. (d) making our country on par with Finland
(c) For government jobs, registration with employment (e) None of these
exchanges is required. 132. The author seems to be in favour of
(d) In Delhi, over the years more industries have started. (a) maintaining secrecy of budget
(e) All the above are true (b) judicious blend of secrecy and openness
Passage 13 (c) transparency in budget proposals
We have inherited the tradition of secrecy about the budget (d) replacement of public constitution by secrecy
from Britain where also the system has been strongly attacked (e) None of these
133. The secrecy of the budget is maintained by all of the
by eminent economists and political scientists including Peter
following countries except
Jay. Sir Richard Clarke, who was the originating genius of
A Finland
nearly every important development in the British budgeting
B India
techniques during the last two decades, has spoken out about
C United States
the abuse of budget secrecy: “The problems of long-term tax (a) Only A (b) Only B
policy should surely be debated openly with the facts on the (c) Only C (d) A and C
table. In my opinion, all governments should have just the (e) B and C
same duty to publish their expenditure policy. Indeed, this 134. Which of the following statements is definitely TRUE in
obligation to publish taxation policy is really essential for the the context of the passage?
control of public expenditure in order to get realistic taxation (a) The British Government has been religiously main
implications.” Realising that democracy flourishes best on the taining budget secrecy.
principles of open government, more and more (b) Budget secrecy is likely to lead to corrupt practices.
democracies are having an open public debate on budget (c) Consulting unjustifiable taxes with public helps make
proposals before introducing the appropriate Bill in the them accept those taxes.
legislature. In the United States the budget is conveyed in a (d) There should be no control on public expenditure in
message by the President to the Congress, which comes well democratic condition.
in advance of the — date when the Bill is introduced in the (e) None of these
Congress. In Finland the Parliament and the people are already 135. Sir Richard Clarke seems to deserve the credit for
discussing in June the tentative budget proposals which are to (a) transformation in the British budgetary techniques.
be introduced in the Finnish Parliament in September. Every (b) maintenance of secrecy of the British budget.
budget contains a cartload of figures in black and white - but (c) detection of abuse of transparency in budget.
(d) bringing down the tax load on British people.
the dark figures represent the myriad lights and shades of
(e) None of these
India’s life, the contrasting tones of poverty and wealth, and
136. From the contents of the passage, it can be inferred that
of bread so dear and flesh and blood so cheap, the deep tints
the author is
of adventure and enterprise and man’s ageless struggle for a
(a) authoritarian in his approach.
brighter morning. The Union budget should not be an annual (b) a democratic person.
scourge but a part of presentation of annual accounts of a (c) unaware of India’s recent economic developments.
partnership between the Government and the people. That (d) a conservative person.
partnership would work much better when the nonsensical (e) None of these
secrecy is replaced by openness and public consultations, 137. Which of the following statement(s) is/are definitely False
resulting in fair laws and the people’s acceptance of their moral in the context of the passage?
duty to pay. A Transparency helps unscrupulous elements to resort
130. How do the British economists and political scientists to corrupt practices.
react to budget secrecy? They are B Open approach of Government is a sign of healthy
(a) in favour of having a mix of secrecy and openness. democracy.
(b) indifferent to the budgeting techniques and taxation C People’s acceptance of their moral duties can best
policies. be achieved th rough openness an d public
(c) very critical about maintenance of budget secrecy. consultations.
(d) advocates of not disclosing in advance the budget (a) Only A (b) Only B
contents. (c) Only C (d) A and B
(e) None of these (e) B and C
READING COMPREHENSION C-59
138. For making the budget realistic, the Government should medicine. Although this difficulty is real, there is nevertheless
(a) refrain from making public the proposed provisions much that he can do if he has rightly diagnosed his trouble.
before finalisation. If for example, his trouble is due to a sense of sin, conscious
(b) discuss it secretly within themselves. or unconscious, he can first persuade his conscious mind that
(c) encourage the public to send in their suggestions. he has no reason to feel sinful, and then proceed, to plant this
(d) consult the public, defend their own plans and accept rational conviction in his unconscious mind, concerning himself
public suggestions. meanwhile with some more or less neutral activity. If he succeeds
(e) None of these in dispelling the sense of sin, it is possible that genuine objective
Directions (Qs. 139-144) : Choose the word which is most interests will arise spontaneously. If his trouble is self-pity, he
nearly the SAME in meaning to the word printed in bold as can deal with it in the same manner after first persuading
used in the passage. himself that there is nothing extraordinarily unfortunate in his
139. SCOURGE circumstances.
(a) ritual (b) presentation If fear is his trouble, let him practise exercises designed to give
(c) whip (d) compromise courage. Courage has been recognized from time immemorial as
(e) remedy an important virtue, and a great part of the training of boys and
140. MYRIAD young men has been devoted to producing a type of character
(a) adequate (b) functional capable of fearlessness in battle. But moral courage and
intellectual courage have been much less studied. They also,
(c) incompatible (d) abundant
however, have their technique. Admit to yourself every day at
(e) excellent
least one painful truth, you will find it quite useful. Teach
141. DUTY
yourself to feel that life would still be worth living even if you
(a) obligation (b) imposition
were not, as of course you are, immeasurably superior to all
(c) tax-liability (d) function
your friends in virtue and in intelligence. Exercises of this sort
(e) job
prolonged through several years will at last enable you to
Directions (Qs. 142-144) : Choose the word/phrase which is
admit facts without flinching and will, in so doing, free you
most OPPOSITE in meaning to the word printed in bold as
from the empire of fear over a very large field.
used in the passage.
145. According to the passage, calculated affection
142. FLOURISHES
(a) appears to be false and fabricated
(a) disappears (b) degenerates
(b) makes other person to love you
(c) vanishes (d) blooms
(c) turns into permanent affection over a period of time
(e) opens
(d) leads to self-pity
143. DEBATED
(a) questioned severely (e) gives a feeling of courage
(b) opposed strongly 146. Who according to the passage is the happy man?
(c) accepted unconditionally (a) Who is encased in self
(d) discussed frankly (b) Who has free affection and wide interests
(e) implemented forcibly (c) Who is free from worldly passions
144. IMPORTANT (d) Who has externally centred passions
(a) major (b) uncountable (e) None of these
(c) significant (d) unscheduled 147. Which of the following statements is NOT TRUE in the
(e) trivial context of the passage ?
Passage 14 (a) The happy man has wide interests.
The happy man is the man who lives objectively, who has free (b) Courage has been recognised as an important virtue.
affections and wide interests, who secures his happiness (c) Unhappy man is encased in self.
through these interests and affections and through the fact (d) A man who suffers from the sense of sin must tell
that they in turn make him an object of interest and affection himself that he has no reason to be sinful.
to many others. To be the recipient of affection is a potent (e) Issue of intellectual courage has been extensively
cause of happiness, but the man who demands affection is not studied.
the man upon whom it is bestowed. The man who receives 148. Which of the following virtues, according to the passage,
affection is, speaking broadly, the man who gives it. But it is has been recognised for long as an important virtue’?
useless to attempt to give it as a calculation, in the way in (a) Patriotism (b) Sacrifice
which one might lend money at interest, for a calculated (c) Courage (d) Self-consciousness
affection is not genuine and is not felt to be so by the recipient. (e) None of these
What then can a man do who is unhappy because he is 149. Which of the following words is SIMILAR in meaning of
encased in self ? So long as he continues to think about the the word ‘bestowed’ as used in the passage ?
causes of his unhappiness, he continues to be self-centered (a) Conferred (b) Accommodated
and therefore does not get outside it. It must be by genuine (c) Trusted (d) Withdrawn
interest, not by simulated interests adopted merely as a (e) Directed
C-60 READING COMPREHENSION
150. Which of the following, according to the passage, has 159. What happens when you think about the cause of your
not been studied much ? unhappiness?
(a) Feeling of guilt and self-pity (a) You try to introspect and look critically at yourself.
(b) The state of mind of an unhappy man (b) You realize that life can be lived in different ways.
(c) How to get absorbed in other interests (c) You try to practice exercise designed to give
(d) Moral and intellectual courage coverage.’
(e) None of these (d) You remain a self-centered person.
151. What should a man do who is suffering from the feeling (e) None of these
of self-pity ? Passage 15
(a) He sould control his passions and emotions. Management is a set of processes that can keep a complicated
(b) He should persuade himself that everything is alright system of people and technology running smoothly. The most
in his circumstances. important aspects of management include planning, budgeting,
(c) He should seek affection from others. organising, staffing, controlling, and problem solving.
(d) He should develop a feeling of fearlessness. Leadership is a set of processes that creates organizations in
(e) He should consult an expert to diagnose his trouble. the first place or adapts them to significantly changing
152. What happens to a man who demands affection ? circumstances. Leadership defines what the future should look
(a) His feelings are reciprocated by others. like, aligns people with that vision, and inspires them to make
(b) He tends to take a calculated risk. it happen despite the obstacles. This distinction is absolutely
(c) He becomes a victim of a vicious circle. crucial for our purposes here: Successful transformation is 70
(d) He takes affection for granted from others. to 90 per cent leadership and only 10 to 30 per cent management.
(e) None of these Yet for historical reasons, many organizations today don’t have
153. If a man is suffering from a sense of sin, much leadership. And almost everyone thinks about the
(a) he should invite opinion of others problems here as one of managing change.
(b) he should admit his sin at once For most of this century, as we created thousands and
(c) he should consciously realize that he has no reason thousands of large organizations for the first time in human
to feel sinful history, we didn’t have enough good managers to keep all
(d) he should develop a fearless character those bureaucracies functioning. So many companies and
(e) he should develop an internal focus of control universities developed management programmes, and hundreds
154. Which of the following statements is TRUE in the context and thousands of people were encouraged to learn management
of the passage? on the job. And they did. But, people were taught little about
(a) All passions stem from unhappiness. leadership. To some degree, management was emphasized
(b) The happy man lives subjectively. because it’s easier to teach than leadership. But even more so,
(c) Any virtue has a dark side also. management was the main item on the twentieth-century agenda
(d) One feels happy if one receives affection. because that’s what was needed. For every entrepreneur or
(e) Any affection is always genuine. business builder who was a leader, we needed hundreds of
155. Which of the following statements is SIMILAR in meaning managers to run their ever growing enterprises.
to the word ‘flinching’ as used in the passage ? Unfortunately for us today, this emphasis on management has
(a) Wincing (b) Convincing often been institutionalized in corporate cultures that discourage
(c) Explaining (d) Providing employees from learning how to lead. Ironically, past success
(e) Debating is usually the key ingredient in producing this outcome. The
156. How can one get out of the vicious circle mentioned in syndrome, as I have observed it on many occasions, goes like
the passage ?
this: success creates some degree of market dominance, which
(a) By practising skills of concentration
in turn produces much growth. After a while keeping the ever
(b) By inculcating the habit of self-absorption
larger organization under control becomes the primary challenge.
(c) Being true to others and one’s internal circumstances
So attention turns inward, and managerial competencies are
(d) Admitting to oneself that others could be right
nurtured. With a strong emphasis on management but not on
(e) None of these
leadership, bureaucracy and an inward focus take over. But
157. Which of the following words is OPPOSITE in meaning
with continued success, the result mostly of market dominance,
of the word ‘dispelling’ as used in the passage ?
(a) Giving (b) Accumulating the problem often goes unaddressed and an unhealthy
(c) Projecting (d) Scattering arrogance begins to evolve. All of these characteristics then
(e) Receiving make any transformation effort much more difficult.
158. What according to the passage is the real cause of Arrogant managers can over-evaluate their current performance
happiness ? and competitive position, listen poorly, and learn slowly.
(a) Material rewards and incentives received Inwardly focused employees can have difficulty seeing the
(b) Critical analysis of the happy state of mind very forces that present threats and opportunities. Bureaucratic
(c) Affection received from others cultures can smother those who want to respond to shifting
(d) Calculated risk taken conditions. And the lack of leadership leaves no force inside
(e) None of these these organisations to break out of the morals.
READING COMPREHENSION C-61
160. Why, according to the author, is a distinction between (c) strategies for producing change was the main focus
management and leadership crucial? of organisations
(a) Leaders are reactive whereas managers are proactive. (d) organisations wanted to create powerful guiding
(b) Organisations are facing problems of not getting coalition
good managers. (e) management was the main item of agenda in
(c) Organisations are pursuing the strategy of status organisations
quo. 167. What is the historical reason for many organisations not
(d) In today’s context, organisations need leaders much having leadership?
more than managers in transforming them. (a) A view that leaders are born, they are not made
(e) None of these (b) Leaders lack managerial skills and organisations need
161. Why did companies and universities develop programmes managers
to prepare managers in such a large number? (c) Leaders are weak in carrying out traditional functions
(a) Companies and universities wanted to generate funds
of management
through these programmes.
(d) Leaders allow too much complacency in organisations
(b) A large number of organisations were created and
(e) None of these
they needed managers in good number.
(c) Organisations did not wants spend their scarce 168. In the passage, management is equated with
resources in training managers. (a) organisation
(d) Organisations wanted to create communication (b) leadership
network through trained managers. (c) organisational vision
(e) None of these (d) bureaucracy
162. Which of the following statements is NOT TRUE in the (e) managerial training
context of the passage? 169. Why does the attention of large organisations turn inward?
(a) Bureaucratic culture can smother those who want to (a) Their managers become arrogant
respond to changing conditions. (b) They have to keep themselves under control
(b) Leadership produces change and has the potential (c) Their success creates market dominance
to establish direction. (d) They want to project their predictability
(c) Pressure on managers comes mostly from within. (e) None of these
(d) Leadership centres on carrying out important 170. Which of the following is SIMILAR in meaning of the
functions such as planning and problem-solving. word NURTURED as used in the passage?
(e) Managers believe that they are the best and that (a) Created (b) Developed
their idiosyncratic traditions are superior. (c) Thwarted (d) Surfaced
163. Which of the following is not the characteristic of (e) Halted
bureaucratic culture? 171. What, according to the author, is leadership?
(a) Managers listen poorly and learn slowly. (a) Process which keeps the system of people and
(b) Managerial competencies are nurtured. technology running smoothly
(c) Employees clearly see the forces that present threats (b) Planning the future and budgeting resources of the
and opportunities. organisation
(d) Prevalence of unhealthy arrogance. (c) Inspiring people to realise the vision
(e) Managers tend to stifle initiative and innovation. (d) Carrying out the crucial functions of management
164. Which of the following is SIMILAR in meaning to the (e) None of these
word SMOTHER as used in the passage? 172. Which of the following characteristics helps organisations
(a) Suppress (b) Encourage
in their transformation efforts?
(c) Instigate (d) Criticise
(a) Emphasis on leadership but not on management
(e) Attack
(b) A strong and dogmatic culture
165. How has the author defined management?
(a) It is the process of adapting organisations to (c) Bureaucratic and inward-looking approach
changing circumstances. (d) Failing to acknowledge the value of customers and
(b) It is the system of aligning people with the direction shareholders
it has taken. (e) None of these
(c) It refers to creating a vision to help direct the change 173. Why were people taught little about leadership in
effort. management programmes?
(d) Creating better performance through customer (a) Teachers were busy in understanding the
orientation. phenomenon of leadership
(e) None of these (b) Enough study material was not available to facilitate
166. Management education was emphasized in th e teaching of leadership
management programmes because (c) Focus of these programmes was on developing
(a) establishing direction was the main focus of managers
organisations (d) Leadership was considered only a political
(b) motivating employees was thought to be done by phenomenon
managers (e) None of these
C-62 READING COMPREHENSION
174. Which of the following statements is/are definitely true 175. In the context of the passage, the culprit’s act of emptying
in the context of the passage? a test tube containing some fluid can be classified as
A. Bureaucracy fosters strong and arrogant culture. (a) a terrorist attack
B. Leadership competencies are nurtured in large-size (b) an epidemic of a dreaded disease
organisations. (c) a natural calamity
C. Successful transformation in organisations is 70 to (d) panic created by an imaginary event
90 per cent leadership. (e) None of these
(a) Only A and B (b) Only A and C 176. In what way would the new terrorism be different from
(c) Only B and C (d) Only B that of the earlier years?
(e) Only C
A More dangerous and less baffling
Passage 16
B More hazardous for victims
In the second week of August 1998, just a few days after the
incidents of bombing the US embassies in Nairobi and Dar-es- C Less complicated for terrorists
Salaam, a high-powered, brain-storming session was held near (a) A and C only
Washington D.C., to discuss various aspects of terrorism. The (b) B and C only
meeting was attended by ten of America’s leading experts in (c) A and B only
various fields such as germ and chemical warfare, public health, (d) All the three
disease control and also by the doctors and the law-enforcing (e) None of these
officers. Being asked to describe the horror of possible bio- 177. What was the immediate provocation for the meeting held
attack, one of the experts narrated the following gloomy scenario. in August 1998?
A culprit in a crowded business centre or in a busy shopping (a) The insistence of America’s leaders
mall of a town empties a test tube containing some fluid, which
(b) The horrors of possible bio-attacks
in turn creates an unseen cloud of germ of a dreaded disease
like anthrax capable of inflicting a horrible death within 5 days (c) A culprit’s heinous act of spreading germs
on any one who inhales it. At first 500, or so victims feel that (d) People’s lack of trust in the government
they have mild influenza which may recede after a day or two. (e) None of these
Then the symptoms return again and their lungs start filling 178. What could be the probable consequences of bio-attacks,
with fluid. They rush to local hospitals for treatment, but the as mentioned in the passage?
panic-stricken people may find that the medicare services run A Several deaths
quickly out of drugs due to excessive demand. But no one B Political turmoil
would be able to realise that a terrorist attack has occurred. C Social unrest
One cannot deny the possibility that the germ involved would (a) A only (b) B only
be of contagious variety capable of causing an epidemic. The
(c) C only (d) A and B only
meeting concluded that such attacks, apart from causing
immediate human tragedy, would have dire long-term effects on (e) All the three
the political and social fabric of a country by way of ending 179. The author’s purpose of writing the above passage seems
people’s trust on the competence of the government. to explain
The experts also said that the bombs used in Kenya and (a) the methods of containing terrorism
Tanzania were of the old-fashioned variety and involved (b) the socio-political turmoil in African countries
quantities of high explosives, but new terrorism will prove to (c) the deadly strategies adopted by modern terrorists
be more deadly and probably more elusive than hijacking an (d) reasons for killing innocent people
aeroplane or a gelignite of previous decades. According to (e) the salient features of terrorism of yester years
Bruce Hoffman, an American specialist on political violence,
180. According to the author of the passage, the root cause
old terrorism generally had a specific manifesto - to overthrow
a colonial power or the capitalist system and so on. These of terrorism is
terrorists were not shy about planting a bomb or hijacking an A Religious fanatism
aircraft and they set some limit to their brutality. Killing so B Socio-political changes in countries
many innocent people might turn their natural supporters off. C The enormous population growth
Political terrorists want a lot of people watching but not a lot (a) A only (b) B only
of people dead. “Old terrorism sought to change the world (c) C only (d) A and B only
while the new sort is often practised by those who believe that (e) All the three
the world has gone beyond redemption”, he added.
181. The phrase “such attacks”, as mentioned in the last
Hoffman says, “New terrorism has no long-term agenda but is
sentence of the second paragraph, refers to
ruthless in its short-term intentions. It is often just a cacophonous
cry of protest or an outburst of religious intolerance or a (a) the onslaught of an epidemic as a natural calamity
protest against the West in general and the US in particular. (b) bio-attack on political people in the government
Its perpetrators may be religious fanatics or diehard opponents (c) attack aimed at damaging the reputation of the
of a government and see no reason to show restraint. They are government
simply intent on inflicting the maximum amount of pain on the (d) bio-attack manoeuvred by unscrupulous elements
victim.” (e) None of these
READING COMPREHENSION C-63
182. The sole objective of the old terrorism, according to home and then forget about it, while the present medical and
Hoffman, was to healthcare set-up in poor countries further entrenches itself?
(a) plant bombs to kill innocent people This does not have to be the fate of the radical resolutions that
(b) remove colonial power or capitalist system will undoubtedly be passed at Milan. Unlike creating a new
(c) make people realise the incompetence of the world economic or information order, establishing a new health
government set-up is essentially a matter for individual countries to
(d) give a setback to socio-political order accomplish. No conflict of international interests is involved.
(e) None of these But this advantage is, at least until it begins to take concrete
183. Which of the following statements is true about new shape, only theoretical. The million-dollar question is whether
terrorism? individual third-world governments are able and willing to
(a) Its immediate objectives are quite tragic. muster the will, the resources, the administrative and other
(b) It has farsighted goals to achieve. infrastructure to carry out what it is entirely within their power
(c) It can differentiate between the innocent people and to attain and implement.
the guilty. The dimensions of the problem are known and the solutions
(d) It is free from any political ideology. broadly agreed on. The present medical and healthcare system
(e) It advocates people in changing the socio-political is urban-based, closely geared to drugs, hospitals and
order expensively trained apathetic doctors. The bulk of the population
Directions (Qs. 184-186): Choose the word which is most in poor countries, who live in rural areas, are left untouched by
OPPOSITE in meaning of the word printed in bold as used in all this and must rely on traditional healers. The answer is to
the passage turn out medical/health personnel sufficiently, but not
184. gloomy expensively, trained to handle routine complaints and to get
(a) discouraging (b) disgusting villagers to pay adequate attention to cleanliness, hygienic
(c) bright (d) tragic sanitation, garbage disposal and other elementary but crucial
(e) versatile matters. More complicated ailments can be referred to properly
185. cacophonous equipped centres in district towns, cities and metropolises.
(a) loud (b) melodious Traditional healers, whom villagers trust, can be among these
(c) sonorous (d) harsh intermediate personnel. Some third-world countries, including
(e) distant India, have launched or are preparing elaborate schemes of this
186. intolerance nature. But the experience is not quite happy. There is
(a) forbearance (b) pesmissibeness resistance from the medical establishment which sees them as
(c) adaptability (c) acceptance little more than licensed quackery but is not prepared either to
(e) faithfulness offer condensed medical courses such as the former licentiate
Directions (Qs. 187-189): Choose the word which is most course available in this country and now unwisely scrapped.
nearly the SAME in meaning of the word printed in bold as There is the question of how much importance to give to
used in the passage. indigenous system of medicine. And there is the difficult matter
187. perpetrators of striking the right balance between preventive healthcare and
(a) opponents (b) followers curative medical attention. These are complex issues and the
(c) sympathisers (d) leaders Milan conference would perhaps be more fruitful if it were to
(e) manoeuvrers discuss such specific subjects.
188. elusive 190. The author is doubtful whether .......
(a) harmful (b) fatal (a) an individual country can set up a new health order.
(c) destructive (d) baffling (b) the Milan conference would pass radical resolutions.
(e) obstructing (c) under-developed countries have the capacity to
189. inflicting organize their resources.
(a) elevating (b) imposing (d) traditional healers could be trained as intermediate
(c) alleviating (d) reflecting health personnel.
(e) soothing (e) the problem has been understood at all.
Passage 17 191. The author has reservations about the utility of the Milan
After “Liberalization”, “Globalization” and the consequent Conference because .......
change in the new international economic order as well as new (a) it is expected only to discuss but not decide upon
information technology order, a new catchphrase is being anything.
coined: ‘A New Health Order’. Talking about setting it up is the (b) earlier conferences had failed to reach any decisions.
theme of the WHO-sponsored international conference on (c) the medical profession is opposed to a new health
primary health and medical care, currently being held at Milan order.
in Italy. While much has been said and written on establishing (d) while “new orders” are talked and written about, not
the “new order”, little has actually been done. Will the much is actually done.
conference at Milan too swear by the “new health order”, go (e) None of these
C-64 READING COMPREHENSION
192. The contents of the passage indicate that the author is 198. For a new health order, the author recommends all of the
opposed to ....... following EXCEPT
(a) traditional healers. (a) motivating villagers to pay attention to cleanliness
(b) licentiate practitioners. (b) setting up well equipped centres in district towns
(c) allopathic system of medicines. (c) discontinuing the present expensive medical courses
(d) hospitals. (d) training traditional healers to function as medical
(e) None of these health personnel
193. It can be inferred from the contents of the passage that (e) striking a balance between preventive healthcare and
the author’s approach is ....... curative medical attention
(a) sarcastic (b) constructive Directions (Qs. 199-201): Choose the word which is most
(c) indifferent (d) fault-finding nearly the SAME in meaning as the word printed in bold as
(e) hostile
used in the passage.
194. The author thinks that the solution to the problem of
199. LAUNCHED
medical/health care lies in .......
(a) participated (b) accomplished
(a) opening hospitals is rural areas.
(b) conducting inexpensive medical courses. (c) elevated (d) planned
(c) improving the economic condition of the masses. (e) started
(d) expediting the setting up of a new health order. 200. MUSTER
(e) making cheap drugs available. (a) enlist (b) summon
195. To make the conference really useful, the author (c) manifest (d) extend
suggests ....... (e) enrich
(a) resolving the international conflicts involved. 201. ENTRENCH
(b) that it should address itself to specific issues. (a) being deteriorating
(c) it should give importance to indigenous system of (b) surround completely
medicine. (c) establish firmly
(d) that it should not pass radical resolutions. (d) enclose carefully
(e) None of these (e) finish radically
196. What does the author suggest for the cure of the cases Directions (Qs. 202-204): Choose the word which is most
involving complications? nearly OPPOSITE in meaning of the word printed in bold as
(a) Treating such cases at well-equipped hospitals in
used in the passage.
district places
202. CONDENSED
(b) Training such victims in preliminary hygiene
(a) concentrated (b) envigoured
(c) Training semi-skilled doctors to treat such cases
(d) Issuing licenses to semi-skilled doctors to treat such (c) expanded (d) lengthened
cases (e) inexplicable
(e) None of these 203. CRUCIAL
197. The medical establishment seems to be reluctant to trust (a) trivial (b) critical
the ....... (c) significant (d) marvellous
(a) allopathic medical practitioners. (e) conspicuous
(b) traditional healers. 204. RESISTANCE
(c) urban-based medical practitioners. (a) opposition (b) agreement
(d) expensively trained allopathic doctors. (c) repulsion (d) acceptance
(e) None of these (e) compliance
READING COMPREHENSION C-65

ANSWER KEY
1 (d) 21 (c) 41 (a) 61 (d) 81 (b) 101 (a) 121 (c) 141 (a) 161 (b) 181 (d) 201 (c)
2 (e) 22 (a) 42 (d) 62 (e) 82 (d) 102 (e) 122 (a) 142 (b) 162 (d) 182 (d) 202 (d)
3 (b) 23 (d) 43 (e) 63 (a) 83 (d) 103 (e) 123 (b) 143 (c) 163 (c) 183 (a) 203 (a)
4 (c) 24 (b) 44 (c) 64 (c) 84 (a) 104 (b) 124 (e) 144 (e) 164 (a) 184 (c) 204 (d)
5 (a) 25 (e) 45 (b) 65 (e) 85 (c) 105 (a) 125 (c) 145 (a) 165 (c) 185 (b)
6 (e) 26 (d) 46 (d) 66 (a) 86 (b) 106 (c) 126 (c) 146 (b) 166 (e) 186 (a)
7 (d) 27 (d) 47 (c) 67 (b) 87 (e) 107 (d) 127 (e) 147 (e) 167 (e) 187 (e)
8 (b) 28 (b) 48 (b) 68 (b) 88 (e) 108 (b) 128 (b) 148 (c) 168 (d) 188 (d)
9 (c) 29 (a) 49 (a) 69 (d) 89 (c) 109 (d) 129 (a) 149 (a) 169 (b) 189 (b)
10 (a) 30 (a) 50 (c) 70 (c) 90 (c) 110 (e) 130 (c) 150 (d) 170 (b) 190 (c)
11 (e) 31 (c) 51 (d) 71 (c) 91 (d) 111 (e) 131 (e) 151 (b) 171 (c) 191 (d)
12 (b) 32 (d) 52 (b) 72 (a) 92 (b) 112 (c) 132 (b) 152 (c) 172 (e) 192 (e)
13 (d) 33 (e) 53 (a) 73 (e) 93 (a) 113 (b) 133 (d) 153 (c) 173 (c) 193 (b)
14 (a) 34 (e) 54 (e) 74 (d) 94 (e) 114 (d) 134 (e) 154 (d) 174 (b) 194 (b)
15 (d) 35 (c) 55 (a) 75 (b) 95 (b) 115 (b) 135 (a) 155 (a) 175 (a) 195 (b)
16 (c) 36 (c) 56 (d) 76 (a) 96 (d) 116 (c) 136 (b) 156 (c) 176 (b) 196 (a)
17 (d) 37 (e) 57 (b) 77 (a) 97 (c) 117 (a) 137 (a) 157 (c) 177 (e) 197 (b)
18 (e) 38 (b) 58 (a) 78 (d) 98 (e) 118 (a) 138 (d) 158 (c) 178 (e) 198 (c)
19 (e) 39 (a) 59 (e) 79 (d) 99 (e) 119 (d) 139 (c) 159 (d) 179 (c) 199 (e)
20 (d) 40 (b) 60 (c) 80 (a) 100 (b) 120 (b) 140 (d) 160 (d) 180 (a) 200 (a)

Answers &
Explanations
13. (d) Darkness has been mentioned as the “fearsome 130. (c) Eminent British economists and political scientists
enemy” of mankind. have strongly attacked the tradition of budget
14. (a) Metals were softened and tempered by application secrecy.
of fire to them. 131. (e) It leads to the control of public expenditure in order to
15. (d) set realistic taxation implications.
16. (c) The virgins were careless in their duty as they 132. (b) He has presented the example of both, the open
budget system and the secret budget system,
allowed the sacred fire to be extinguished.
practised by various countries and has looked into
21. (c) Read the first line of the passage.
all their aspects.
22. (a) Read the last sentence of the passage. 135. (a) Sir Richard Clarke was the originating genius of nearly
50. (c) They are responsible for national disintegration. every important development in the British budgeting
51. (d) They are harmful to national integrity. techniques during the last two decades.
52. (b) India was forged into a nation on account of a 137. (a) The statement goes against the idea of the passage.
common culture evolved over the centuries. 138. (d) An open public debate on budget proposals should
53. (a) The author wants India to remain as an ideal nation be held before introducing the appropriate bill.
and the passage has certainly a message behind it. 162. (d) As given in the passage, planning and problem-
55. (a) Read the last sentence of the passage. solving are the most important aspects of the
59. (e) India’s insurgence stood for gaining freedom by management, and not of the leadership.
adopting the path of non-violent struggle. 163. (c) “Inwardly focussed employees can have difficulty
74. (d) A plausible explanation, argument, or statement is seeing the very forces that present threat and
one that seems likely to be true or valid. opportunities.” This sentence of the last paragraph
111. (e) US makes option (c) wrong.
C-66 READING COMPREHENSION
164. (a) Bureaucratic culture is against any transformation; 177. (e) The immediate provocation for the meeting held in
so it suppresses those who want to bring any change August 1998 has not been given among the options.
in organisations. It was the incidents of bombing the US embassies in
165. (c) Planning can be defined as ‘creating a vision’, which Nairobi and Dar-es-Salaam.
is an important aspect of management. 178. (e) Bio-attack will result in several deaths which will
167. (e) For most of this century, as a large number of lead to political turmoil, creating social unrest.
organisations were created for the first time in human 180. (a) ‘Religious intolerance’, as cited in the last paragraph,
history, emphasis was given on management and stands behind terrorism.
leadership was overlooked. 190. (c) Go through the last line of the first paragraph.
191. (d) While much had been said and written on establishing
168. (d) Managers are also bureaucrats.
“ new order”, little has actually been done.
175. (a) Ascertain the hidden meaning of the sentence : “but
195. (b) These are complex issues and the Milan conference
no one would be able to realise that a terrorist
would perhaps be more fruitful if it were to discuss
attack has occurred”. So, undoubtedly the culprit’s such specific subjects.
act can be classified as a terrorist attack. 196. (a) More complicated ailments can be referred to properly
176. (b) “New terrorism has no long-term agenda but its equipped centres in district towns, cities etc.
ruthless in its short-term intentions”. This statement 197. (b) There is resistance from the medical establishment
from the passage supports (B). While, in the light of which sees them as little more than licensed quackery.
passage, (C) also seems suitable. Here reference is made to traditional healers.
Parajumbles
4 Chapter
SENTENCE OR WORD REARRANGEMENT Now we can see in option (a), C follows sentence A but the gap
In this type of question, basically, you are given a paragraph or spoken of in sentence C has no correlation with political
sentence - but the sentences (in case of paragraph) or words (in geography of the subcontinent spoken of in sentence A , so we
case of sentence) are not in the right order. It's up to you to untie can rule out Option (a).
this knot and rearrange the sentences or words so that they Therefore answer has to be option (d), as we can also see it
logically make sense. elaborates on the change mentioned in sentence A.
Sentences or words rearrangement questions are included in
BANK exams as they EXAMPLE 2.
v Help students relate events in a logical manner A. Thus begins the search for relief: painkillers, ice, yoga,
v Sequence sentences based on English usage skills herbs, even surgery
B. Most computer users develop disorders because they
HOW TO TACKLE THESE TYPES OF QUESTIONS?
ignore warnings like tingling fingers, a numb hand or a
To tackle these type of questions, you have to know three
sore shoulder
things.
C. They keep pointing and dragging until tendons chafe
v Theme of the paragraph that might be created on un-jumbling
and scar tissue forms, along with bad habits that are
the sentences.
almost impossible to chage
v Initiating sentence, which starts the paragraph
D. But cures are elusive , because repetetive stree injuries
v Links have to be found between two sentences. Once a link
present a bag of ills that often defy easy disgnosis.
of this type is created, it becomes easy to eliminate irrelevant
choices. (a) BDAC (b) BADC
(c) BCAD (d) ABCD
HOW TO SAVE TIME WHILE SOLVING THESE TYPES? Explanation : Here we can make out that sentence B is will be the
It is very important to read selectively and search for transition starting sentence as it introduces the subject matter which is
words or other keywords. 'computer users and related problems'.
The best way is to establish a link between any two (or more) Option (d) automatically gets eliminated as it starts with sentence
statements. Once a link is found, you get to know which A.
statements will come together. Then, look in the options. Select
Option (a) can be rule out as there is no corelation between
the option with those statements together.
sentence B and sentence D. Sentence B talks of warnings whereas
sentence D talks of cures for illness and hence no correlation
EXAMPLE 1. exists.
A. 1971 war changed the political geography of the
subcontinent EXAMPLE 3.
B. Despite the significance of the event there has been
A. If you are used to having your stimulation come in from
no serious book about the conflict
outside, your mind never develops its own habits of
C. Surrender at Dacca aims to fill this gap thinking and reflecting
D. It also profoundly altered the geo-strategic situation
B. Marx thought that religion was the opiate, because it
in South-East Asia
soothed people's pain and suffering and prevented
(a) ACBD (b) CADB them from rising in rebellion
(c) BADC (d) ADBC C. Karl Marx was alive today, he would say that television
Explanation : We can see that Sentence A is most likely the starting is the opiate of the people.
sentence.Now that we know that A is the starting sentence we
D. Television and similar entertainments are even more of
can eliminate choice (b) and (c) as they start with C and B
an opiate because of their addictive tendencies.
respectively.
(a) BACD (b) ADBC
This narrows down our possibilities to option (a) and
option (d). (c) BDCA (d) CBDA
C-68 PARAJUMBLES
Explanation: Sentence B has Marx (short Form) and sentence C C. A simple model was proposed in 1514 by a Polish priest,
has Karl Marx (Full form). So C will come before B. Now in Nicholas Copernicus.
given options we can clearly see (a) and (b) and (c) , B is D. Nearly a century passed before this idea was taken
placed before C and hence we reject option (a), (b) and (c) seriously.
which leaves us with only option (d) which is the correct (a) CDBA (b) CBDA
option. (c) BCAD (d) CADB
EXAMPLE 4. Explanation: Answer is option (b) as we can see that in sentence
D it says ' nearly a century has passed ' so we have to keep the
A. Then two astronomers-the German, Johannes Kepler,
timeline in consideration here also while sequencing the sentences
and the Italian, Galileo Galilei-started publicly to support
and only in option 2 the timeline fits correctly.
the Copernican theory, despite the fact that the orbits it
predicted did not quite match the ones observed.
B. His idea was that the sun was stationary at the centre
and that the earth and the planets move in circular orbits
around the sun.

EXERCISE
Directions (Qs. 1-5): Rearrange the following six sentences A, 5. Which of the following will be the LAST sentence?
B, C, D, E and F in the proper sequence to form a meaningful (a) C (b) D
paragraph; then answer the questions given below them. (c) E (d) F
(A) We were interested by contrast in understanding (e) B
what lessons actual teams and non-teams had for Directions (Qs. 6-10): Rearrange the following six sentences
others to choose to struggle with change and A, B, C, D, E and F in the proper sequence to form a meaningful
performance. paragraph; then answer the questions given below them.
(B) Still, we suspected that most of these focussed on (A) Some people believe that at present its importance is
persuading readers that “teams are important”. decreasing because of rapid economic and social
(C) After all we thought teams are a well-known subject changes.
and there must be a thousand books on the subject (B) The extent of its importance may be slightly less in
already. cities as compared to rural communities.
(D) By going down this path we hoped to discover (C) Some even go to the extreme and say that it will
something to say that was different from most books soon become obsolete because of these changes.
on the subject. (D) The family is an important socialisation agency both
(E) We approached the idea of a book on teams in rural and city communities.
cautiously. (E) The difference in the degree of importance does not
(F) Alternatively they focussed on providing you advice matter much and therefore has no significance.
on building teams as an objective in itself. (F) There are others who believe that the family has
1. Which of the following will be the SECOND sentence? survived such storms in the past and it will do so in
(a) A (b) B the future also.
(c) F (d) C 6. Which of the following should be the FIRST sentence
(e) D after rearrangement’?
2. Which of the following will be the FIRST sentence? (a) F (b) E
(a) E (b) A (c) D (d) C
(c) B (d) C (e) B
(e) D 7. Which of the following should be the SIXTH (LAST)
3. Which of the following will be the THIRD sentence? sentence after rearrangement’?
(a) E (b) C (a) F (b) E
(c) B (d) F (c) D (d) C
(e) D (e) B
4. Which of the following will be the FIFTH sentence? 8. Which of the following should be the FIFTH sentence
(a) C (b) D after rearrangement?
(c) B (d) F (a) F (b) E
(c) D (d) C
(e) A
(e) B
PARAJUMBLES C-69
9. Which of the following should be the THIRD sentence (G) The problem, therefore, reduces itself to one of
after rearrangement? agricultural development.
(a) F (b) E 16. Which of the following is the FOURTH sentence in the
(c) D (d) C paragraph?
(e) B (a) A (b) D
10. Which of the following should be the SECOND sentence (c) E (d) G
after rearrangement’? (e) F
(a) F (b) E 17. Which of the following is FIFTH sentence in the
(c) D (d) C paragraph?
(e) B (a) G (b) A
Directions (Qs. 11-15): Rearrange the following five sentences (c) D (d) C
A, B, C, D and E in the proper sequence to form a meaning (e) E
paragraph; then answer the questions given below them. 18. Which of the following is the THIRD sentence in the
(A) The reasons for formal education getting nullified
paragraph?
are that we teachers have limited vision, our
(a) F (b) A
judgements about students are hasty and we are
(c) G (d) D
more knowledge-centred then student-centred.
(e) E
(B) Life educates as nothing else does.
(C) Churchill rose to dizzy heights despite his teachers’ 19. Which of the following is the LAST sentence in the
prophesies to the contrary. And there are many more paragraph?
such examples. (a) F (b) C
(D) Life’s teachings sometimes supplement the education (c) D (d) B
received in the classroom and at other times nullify (e) G
it. 20. Which of the following is the SECOND sentence in the
(E) Education received in the classroom is insignificant paragraph?
as compared to what life teaches us. (a) B (b) E
11. Which of the following will be the SECOND sentence? (c) C (d) G
(a) A (b) B (e) F
(c) C (d) D Directions (Qs. 21-25): Rearrange the following seven sentences
(e) E (A), (B), (C), (D), (E), (F) and (G) in the proper sequence to
12. Which of the following will be the THIRD sentence? form a meaningful paragraph; then answer the questions given
(a) A (b) B below them.
(c) C (d) D (A) Japanese toys, for instance, are in great demand
(e) E despite the heavy import duty.
13. Which of the following will be the FIRST sentence? (B) The toys that they produce are, almost without
(a) A (b) B exception, of inferior quality.
(c) C (d) D (C) Their manufacturers here need to be reminded of
(e) E this.
14. Which of the following will be the FOURTH sentence? (D) The two toy libraries in Mumbai also rely largely on
(a) A (b) B foreign-made toys.
(c) C (d) D (E) But making them is no child’s play.
(e) E (F) Toys are meant for children.
15. Which of the following will be the LAST sentence? (G) Not surprisingly, many parents prefer to buy the
(a) A (b) B imported variety even though these are usually much
(c) C (d) D more expensive.
(e) E 21. Which of the following will be the THIRD sentence?
Directions (Qs. 16-20): Rearrange the following sentences to (a) C (b) B
form a meaningful paragraph and answer the questions given (c) D (d) E
below: (e) G
(A) We must explore new methods of boosting 22. Which of the following will be the LAST sentence?
agricultural development and grow more food. (a) C (b) D
(B) The scientists should be encouraged to contribute. (c) B (d) E
(C) Food can also be had by import. (e) G
(D) The most important factor in any planning for India’s 23. Which of the following will be the FIRST sentence?
development and economic uplift is that of turning (a) E (b) G
a hungry, discontented people into a happy well-fed (c) A (d) F
one. (e) B
(E) Whatever be the way and means, India must feed its 24. Which of the following will be the FOURTH sentence?
hungry millions. (a) C (b) E
(F) They should be given due scope for carrying on (c) B (d) G
experiments and researches. (e) D
C-70 PARAJUMBLES
25. Which of the following will be the SIXTH sentence? (G) There is no doubt that the methods of science and
(a) F (b) E religion are different.
(c) C (d) D 31. Which of the following will be the SECOND sentence?
(e) A (a) F (b) E
Directions (Qs. 26-30): Rearrange the following six sentences (c) D (d) B
(A), (B), (C), (D), (E) and (F) in the proper sequence to form a (e) G
meaningful paragraph; then answer the questions given below 32. Which of the following will be the FOURTH sentence?
them. (a) B (b) A
(A) They collected plants, counted birds and (c) D (d) F
photographed the terrain and the fauna and made (e) C
their recommendations. 33. Which of the following will be the LAST sentence?
(B) In spring of 1963, an alarmed King Hussain invited a (a) C (b) A
group of British scholars, scientists and naturalists. (c) D (d) B
(C) He also wanted them to cover the deserts to the east (e) E
of the mountains. 34. Which of the following will be the FIRST sentence?
(D) He wanted them to conduct an extensive survey of (a) C (b) B
the mountains on the eastern side of the Dead Sea. (c) D (d) F
(E) The problem of conservation of forests and forest (e) A
birds and nature in general was thus set rolling. 35. Which of the following will be the SIXTH sentence?
(F) Accordingly, an expedition of internationally known (a) C (b) F
experts in conservation, botany, ornithology, etc. (c) A (d) G
went to Jordan. (e) D
26. Which of the following should be the FIFTH in the Directions (Qs. 36-40) : Rearrange the following six sentences
paragraph? (A), (B), (C), (D), (E) and (F) in the proper sequence so as to
(a) A (b) B (c) C (d) make a meaningful paragraph, then answer the questions given
D (e) E below them.
27. Which of the following should be the FIRST in the A. We feel these things are glorious because of the
paragraph? splendid triumphs.
(a) A (b) B (c) C (d) D (e) B. Because of these sacrifices we realise the victories
E of peace are even more glorious than victories of
28. Which of the following should be the LAST in the war.
paragraph? C. The word victory is associated in our minds with
(a) A (b) B (c) C (d) D (e) war.
E D. We are impressed by their sacrifices.
29. Which of the following should be the SECOND in the E. It calls up visions of battles, bloodshed and
paragraph? conquests by force.
(a) F (b) E (c) D (d) F. But when we think of the philosophy of great men,
C (e) B scholars, social reformers, scientists and
30. Which of the following should be the FOURTH in the philanthropists we start thinking in a different way.
paragraph? 36. Which sentence should be the FOURTH in the paragraph?
(a) F (b) E (c) D (d) (a) B (b) C
C (e) B (c) D (d) E
Directions (Qs. 31-35): Rearrange the following seven (e) F
sentences (A), (B), (C), (D), (E), (F) and (G) in the proper 37. Which sentence should be the THIRD in the paragraph?
sequence to form a meaningful paragraph; then answer the (a) A (b) B
questions given below them. (c) C (d) D
(A) It takes its recourse to progressive march towards (e) E
perfection. 38. Which sentence should be the FIRST in the paragraph?
(B) But, one may conclude, while science is inclined (a) A (b) B
towards reason, spiritualism is the essence of religion. (c) C (d) D
(C) In religion deviation from the set course is permissible, (e) E
though some more rationalistic religious leaders also 39. Which sentence should be the LAST (SIXTH) in the
allow questioning and their satisfactory answers. paragraph?
(D) Many people believe that science and religion are (a) A (b) B
contrary to each other.
(c) C (d) D
(E) The tools of religion, on the other hand, are faith,
(e) E
intuition, and the spoken word of the enlightened.
40. Which sentence should be the SECOND in the paragraph?
(F) The method of science is observation, experiment
(a) B (b) C
and experience.
PARAJUMBLES C-71
(c) D (d) E 31st January, 1992 in pursuance of the National
(e) F Commission for Women Act, 1990.
Directions (Qs. 41-45) : Rearrange the following seven sentences (E) Towards this end of speedy justice to women, the
A, B, C, D, E, F and G in the proper sequence so as to make Commission is organising Parivarik Mahila Lok
a meaningful paragraph; then answer the questions given below Adalats, offering counselling in family disputes and
them. conducting training programmes for creating legal
A The individual owes allegiance and obedience to the awareness among women.
state. (F) They also suggest remedial action to safeguard the
B It is its organ for the present action, the custodian interest of women to the appropriate authorities.
of its tradition. (G) To carry out these functions the Commission has a
C For, the nation is not only a fellowship of contempo- chairman, five members and a Member-Secretary, all
raries. nominated by the Central Government.
D The state exists for the citizens, not the citizens for 46. Which of the following will be the FOURTH sentence?
the state. (a) A (b) C
E But it is a partnership of present with past and (c) D (d) B
future. (e) E
F It is also the trustee for its future. 47. Which of the following will be the FIRST sentence?
G This is mainly because it is the representative and (a) C (b) D
effective organ of the largest and most inclusive (c) E (d) F
comunity to which he belongs. (e) A
41. Which sentence should be the FOURTH in the para 48. Which of the following will be the LAST sentence?
graph? (a) G (b) F
(a) A (b) B (c) D (d) C
(c) C (d) D (e) E
(e) E 49. Which of the following will be the THIRD sentence?
42. Which sentence should be the SIXTH in the paragraph?
(a) G (b) B
(a) A (b) B
(c) F (d) D
(c) C (d) D
(e) C
(e) E
50. Which of the following will be the FIFTH sentence?
43. Which sentence should be the FIRST in the paragraph?
(a) C (b) D
(a) A (b) B
(c) F (d) E
(c) C (d) D
(e) B
(e) E
Directions (Qs. 51-55): Rearrange the following five sentences
44. Which sentence should be the SEVENTH (LAST) in the
into a meaningful paragraph and answer the questions given
paragraph?
below:
(a) A (b) B
(c) F (d) D (A) However, with innovation coming into play unit-
(e) E linked/ market-linked products have also found a
45. Which sentence should be the SECOND in the paragraph? place in insurance business after privatisation.
(a) A (b) B (B) It is also worth mentioning here that world over
(c) G (d) D unitlinked products constitute quite a substantial
(e) E chunk of the total portfolio of insurance companies.
Directions (Qs. 46-50) : Rearrange the following seven (C) There was a time when only traditional insurance
sentences (A), (B), (C), (D), (E), (F) and (G) in the proper products used to dominate the arena.
sequence to form a meaningful paragraph then answer the (D) The emergence of these products of various
questions given below them. insurance companies combines the characteristics of
(A) It is obvious from the above that the Commission both endowment insurance policies and mutual funds.
has accorded highest priority to securing speedy (E) The insurance industry in India is evolving and
justice to women. assuming different proportion since it was privatised.
(B) These members continue to pursue their mandated 51. Which of the following will be the Fourth sentence in the
activities, namely review of legislation, intervention paragraph?
in specific individual complaints of atrocities and (a) A (b) B
denial or rights. (c) C (d) D
(C) The functions assigned to the Commission, as per (e) E
the Act, are wide and varied covering almost all the 52. Which of the following will be the Second sentence in the
facets of issues relating to safeguarding women’s paragraph?
rights and promotion. (a) A (b) B
(D) The National Commission for Women was set up on (c) C (d) D
(e) E
C-72 PARAJUMBLES
53. Which of the following will be the Last sentence in the Directions (Qs. 61-65): Rearrange the following six sentences
paragraph? (A), (B), (C), (D), (E) and (F) in the proper sequence to form a
(a) A (b) B meaningful paragraph. Then answer the questions given below
(c) C (d) D them.
(e) E A. While doing so, we may also correct any distortions
54. Which of the following will be the First sentence in the that we may discern.
paragraph? B. With all our experience and insight, we should be
(a) A (b) B able to visualize them well in advance.
(c) C (d) D C. The celebration of the 50th anniversary of the
(e) E country’s independence is a historic moment.
55. Which of the following will be the Third sentence in the D. Also, it is a time to consolidate on the gains that we
paragraph? have made.
(a) A (b) B E. But, most of all, it is a time to gear up for the
(c) C (d) D opportunities and challenges that lie ahead.
(e) E F. It is a time to introspect and evaluate what we have
Directions (Qs. 56-60): Rearrange the following seven achieved in the last five decades.
sentences A, B, C, D, E, F and G in the proper sequence to 61. Which of the following should be the FOURTH statement
form a meaningful paragraph, then answer the questions given after re-arrangement?
below them. (a) E (b) D
A. The history of that system is, however, a warning (c) C (d) B
than a stimulus to reorganise a similar scheme. (e) A
B. However, we can’t resign ourselves merely because 62. Which of the following should be the SIXTH (LAST)
there is no data. statement after re-arrangement ?
C. One of the commonest risks which agricultural life is (a) A (b) B
exposed to in this country is famine or failure of (c) C (d) D
crops. (e) E
D. There is, however, no reliable data on which such a
63. Which of the following should be the SECOND statement
scheme of insurance can be based.
after re-arrangement ?
E. A kind of Famine Insurance System was attempted
(a) F (b) E
by the British Government of India in the last century.
(c) D (d) C
F. Still, the need for such a scheme to cover the losses
(e) B
due to famine, cattle plague, crop pests, etc. can’t be
64. Which of the following should be the THIRD statement
undermined.
after re-arrangement ?
G. It is obviously because of failure of rain and the
(a) B (b) C
consequence is starvation.
(c) D (d) E
56. Which of the following should be the FOURTH sentence
after rearrangement’? (e) F
(a) A (b) B 65. Which of the following should be the FIRST statement
(c) C (d) D after re-arrangement ?
(e) E (a) F (b) E
57. Which of the following should be the SIXTH sentence in (c) D (d) C
the paragraph? (e) B
(a) E (b) D Directions (Qs. 66-70): Rearrange the following seven
(c) B (d) C sentences (A), (B), (C), (D), (E), (F) and (G) in the proper
(e) A sequence to form a meaningful paragraph then answer the
58. Which of the following should be there in the THIRD questions given below them.
position in the paragraph? A People thoroughly dedicated to social service but
(a) B (b) C not fulfilling the eligibility requirements would not be
(c) D (d) E able to contest elections.
(e) F B Those who fulfil the stipulated criteria of age and
59. Which of the following should be the FIRST sentence formal education may not be necessarily devoted to
after rearrangement? social service.
(a) A (b) B C This system has both advantages and disadvantages.
(c) D (d) C D Therefore, imposing such eligibility requirements is
(e) E likely to be counter-productive.
60. Which of the following should be the SECOND sentence E In certain democratic countries, elections can be
after rearrangement? contested by anybody.
(a) B (b) G F People would be deprived of the probable benefit
(c) D (d) E accrued from services of such people.
(e) F
PARAJUMBLES C-73
G There are no eligibility requirements of formal 74. Which of the following will be the FIRST sentence?
education and upper age limit stipulated in their (a) D (b) E
Constitution. (c) F (d) B
66. Which sentence should be the FOURTH in the paragraph? (e) C
(a) A (b) B Directions (Qs. 75-80): Rearrange the following eight sentences
(c) C (d) D A, B, C, D, E, F, G and H in the proper sequence to form a
(e) E meaningful paragraph; then answer the questions given below
them.
67. Which sentence should be the LAST in the paragraph?
A Therefore, the duty of the advocate is to do his best
(a) A (b) B
for his client.
(c) C (d) D B That rests with the judge, and it is ultimately for the
(e) E judge to decide which side is right, and how justice
68. Which sentence should be the FIRST in the paragraph? should prevail.
(a) G (b) F C When he was asked what he thought of an advocate
(c) E (d) D supporting a cause which he knew to be bad,
(e) C Johnson’s answer was that the advocate did not
69. Which sentence should be the SECOND in the paragraph? know it to be good or bad till the judge determined
(a) G (b) F it for him and for others.
(c) E (d) D D But, he must do so fairly, and without concealing
(e) C from it anything that it is his duty to divulge.
70. Which sentence should be the THIRD in the paragraph? E There is a belief that an advocate’s function consists,
(a) A (b) B for the most part, of showing white as black and
black as white.
(c) C (d) D
F He is, after all, the client’s mouthpiece, and he must
(e) E
put before the court all aspects of the case which are
Directions (Qs. 71-74) : Rearrange the following six sentences favourable to his client.
(A), (B), (C), (D), (E) and (F) in the proper sequence to form a G The only answer that one can give to this popular
meaningful paragraph; then answer the questions given below misconception is the famous answer that Johnson
them. gave to Boswell.
A But all three have one focus-individual performance H But he is not concerned with the final result.
improvement. 75. Which of the following will be the FIRST sentence in the
B The importance of each component will vary from passage?
organization to organization according to the (a) C (b) E
complexity of the operations. (c) F (d) G
C They are individual development, career development (e) B
and organizational development. 76. Which of the following will be the FIFTH sentence in the
D Since individual performance improvement is the passage?
heart of the HRD programme, HRD can be described (a) F (b) G
(c) D (d) C
as the area of congruence among the three
(e) H
components.
77. Which of the following will be the SECOND sentence in
E There are three fundamental component areas of the passage?
human resource development. (a) C (b) A
F It will also vary according to the criticality of human (c) F (d) G
resources to organizational efficiency and (e) E
organization’s commitment to improve human 78. Which of the following will be the LAST sentence in the
resources. passage?
71. Which of the following will be the SIXTH sentence? (a) A (b) D
(a) C (b) F (c) F (d) B
(c) B (d) D (e) H
(e) A 79. Which of the following will be the FOURTH sentence in
72. Which of the following will be the FOURTH sentence? the passage?
(a) F (b) C (a) F (b) C
(c) D (d) B (c) E (d) D
(e) A
(e) A
80. Which of the following will be the SEVENTH sentence in
73. Which of the following will be the SECOND sentence?
the passage?
(a) D (b) E (a) D (b) B
(c) B (d) F (c) H (d) A
(e) C (e) C
C-74 PARAJUMBLES
Directions (Qs. 81-85): In each of the questions below four D. I have travelled a great deal in this country and I
sentences are given which are denoted by A, B, C, D. By using have grown in years.
all the four sentences you have to frame a meaningful para. The (a) DCBA (b) DBCA
correct order of the sentences is your answer. Choose from the (c) ADBC (d) CDBA
five alternatives the one having the correct order of the (e) CBAD.
sentences. Directions (Qs. 86-90): In each of the following questions five
81. A. It was with this invincible spirit that Netaji opposed phrases denoted by (A), (B), (C), (D) and (E) are given. By
Wavell’s offer. using all the five phrases, each only once, you have to frame
B. “Japan’s surrender is not India’s surrender,” he said. a meaningful and grammatically correct sentence. The correct
C. The revolutionary spirit of Netaji was never dampened
order of the phrases is your answer. Choose from the five
even after the surrender of Japan.
alternatives the one having the correct order of the phrases
D. He knew that a war of liberation demanded great
and mark it as your answer.
spirit, great sacrifice, courage and patience.
(a) BCDA (b) BCAD 86. (A) The case goes to highlight
(c) CBAD (d) DCBA (B) Ruin the honour and prestige
(e) CBDA (C) How rogue bureaucrats can
82. A. But all work is not education. (D) Of a citizen
B. In India, a majority of our people do hard work, (E) On the slightest of pretexts
strenuous physical work, but all are not educated. (a) ACBDE (b) CDABE
C. It aims at concrete and objective realization of the (c) BEDCA (d) DCAEB
ideas and is of great educative value. (e) None of these
D. “Work” is that activity of man which has a definite 87. (A) On the pull-out deadline,
objective. (B) Any intruder spotted on its territory beyond Friday
(a) DCAB (b) BCDA (C) Refusing to entertain the enemy request
(c) BACD (d) DBCA (D) India reiterated that
(e) CBAD (E) Would be forcibly evicted
83. A. Hari Prasad Nanda is one such person who worked (a) ACDBE (b) BADCE
his way to the top from the scratch. (c) CADBE (d) ADCBE
B. A few of them had a spark of proved adventure and (e) None of these
their initiative, dedication and sincerity brought them 88. (A) Nasty way and
spectacular success.
(B) Quite miserable
C. The partition of India into India and Pakistan made
(C) It is difficult to understand
a number of migrants to India penniless.
(D) Why people behave in such a
D. He rose to become a first-generation entrepreneur
with the second largest complex to his credit. (E) Make the life of everyone
(a) ADBC (b) ADCB (a) AEBDC (b) DCAEB
(c) CBAD (d) CBDA (c) EBADC (d) DACEB
(e) BCDA (e) None of these
84. A. They think that India will disintegrate like the Soviet 89. (A) Everyone had left the office premises
Union or Yugoslavia. (B) The smoke formed a dense screen
B. What will be the exact shape of India in 2000 A.D. (C) Anything about the missing file as
can only be a matter of surmise. (D) As soon as the fire broke out and
C. On the contrary, the blind patriots for see a very (E) Nobody could say
bright future for India. (a) CEDBA (b) AEBCD
D. The prophets of doom say that the future of India (c) BECDA (d) DCAEB
is doomed. (e) None of these
(a) BCDA (b) BDAC 90. (A) He found the ring
(c) DABC (d) DBAC (B) His statement that
(e) BCAD (C) Everyone knows he is a liar
85. A. I wish I had more time, so that I could visit the odd (D) Cannot be trusted because
nooks and corners of India. (E) Inside the wallet
B. And yet I have not seen many parts of the country
(a) CDAEB (b) BACDE
we love so much and seek to serve.
(c) DCBAE (d) BAEDC
C. Our own country is a little world by itself with an
infinite variety and places for us to discover. (e) None of these
PARAJUMBLES C-75

ANSWER KEY
1 (d) 11 (e) 21 (a) 31 (e) 41 (c) 51 (d) 61 (a) 71 (b) 81 (e)
2 (a) 12 (d) 22 (b) 32 (b) 42 (b) 52 (c) 62 (a) 72 (c) 82 (a)
3 (c) 13 (b) 23 (d) 33 (d) 43 (a) 53 (b) 63 (a) 73 (e) 83 (c)
4 (e) 14 (a) 24 (c) 34 (c) 44 (c) 54 (e) 64 (c) 74 (b) 84 (b)
5 (b) 15 (c) 25 (e) 35 (a) 45 (c) 55 (a) 65 (d) 75 (b) 85 (d)
6 (c) 16 (a) 26 (a) 36 (e) 46 (d) 56 (a) 66 (a) 76 (a) 86 (a)
7 (a) 17 (d) 27 (b) 37 (a) 47 (b) 57 (b) 67 (d) 77 (d) 87 (c)
8 (d) 18 (c) 28 (e) 38 (c) 48 (e) 58 (d) 68 (c) 78 (d) 88 (e)
9 (b) 19 (a) 29 (c) 39 (b) 49 (a) 59 (d) 69 (a) 79 (e) 89 (e)
10 (e) 20 (b) 30 (a) 40 (d) 50 (c) 60 (b) 70 (c) 80 (c) 90 (d)

Answers &
Explanations
(61-65):
C comes first because it is the only independent sentence. We then place the sentences having “it is a time", the phrase that
refers to C. Among three such sentences – D, E and F – F comes first because of its plain structure. Also in D makes it the
second sentence. While most of all in E makes it the last among these three. Thus, our sentences are CFDE. Now, look at the
words “visualise them” in B. These words are a clear reference to the words in E: “opportunities ... challenges ... ahead.” Thus,
E is followed by B. The remaining sentence, i.e. A, comes at the end. Thus, we have
C F D E B A
88. (e) CDAEB
89. (e) ECADB
C-76 CLOZE TEST

Cloze Test
5 Chapter

CLOZE TEST opinions. However, the placement of the blank at the end of the
sentence restricts the possible words that may complete the
Cloze tests are common on all bank exams. They usually require
sentence; following an adverb and finishing the sentence, the
you to choose the correct choice out of four possibilities.
word is most likely an adjective. Romantic, chivalrous or gallant
A cloze test (also cloze deletion test) is an exercise, test, or may, for example, occupy the blank, as well as foolish or cheesy.
assessment consisting of a portion of text with certain words
removed (cloze text), where the participant is asked to replace the HOW TO TACKLE A CLOZE TEST
missing words. Cloze tests require the ability to understand
context and vocabulary in order to identify the correct words or • Read the text through trying to understanding the general
type of words that belong in the deleted passages of a text. meaning.

1 : A language teacher may give the following passage


• Look at each missing word gap and try to imagine what the
to students: correct word should be.

Today, I went to the ________ and bought some milk and eggs. • Decide which part of speech (adjective, noun, gerund, etc.)
I knew it was going to rain, but I forgot to take my ________, and needs to be used to fill each gap.
ended up getting wet on the way ________.
• Read the text again, trying to fill a gap as you come to it by
Explanation : Students would then be required to fill in the blanks imagining what the correct answer should be.
with words that would best complete the passage. Context in
language and content terms is essential in most, if not all, cloze
• Read the text another time, this time choose the correct answer
tests. The first blank is preceded by "the"; therefore, a noun, an from the five answers given.
adjective or an adverb must follow. However, a conjunction follows • If you are unsure of any given answer, try reading the sentence
the blank; the sentence would not be grammatically correct if with each of the possibilities.
anything other than a noun were in the blank. The words "milk
and eggs" are important for deciding which noun to put in the • Try to eliminate the obvious false choices.
blank; "market" is a possible answer; depending on the student,
• Always think about the overall meaning of the text (i.e.,
however, the first blank could either be store, supermarket, shop
whether the text is negative, positive, etc.) to make sure that
or market while umbrella or raincoat fit the second.
your answer choice fits the context.
2 : I saw a man lay his jacket on a puddle for a woman
• Trust your intuition. If you feel a word is right instinctively,
crossing the street. I thought that was very ______.
it probably is correct.
Explanation : Given the above passage, students' answers may
then vary depending on their vocabulary skills and their personal
CLOZE TEST C-77

EXERCISE
Directions (Qs. 1-13): In the following passage there are blanks, 12. (a) squandered (b) preserved
each of which has been numbered. These numbers are printed (c) doubled (d) engulfed
below the passage and against each, five words are suggested, (e) coerced
one of which fits the blank appropriately. Find out the appropriate 13. (a) equilibrium (b) existence
word in each case. (c) failure (d) proportion
Man in his 1 of nature and universe has made the world 2, (e) bankruptcy
polluted. The air we breathe is polluted, the water we drink
Directions (Qs. 14-23): In the following passage, there are blanks,
is 3. There is 4 felling of trees, clearing of jungles, 5 natural
each of which has been numbered. These numbers are printed
barriers like the mountains and drying up the oceans by
below the passage and against each, five words are suggested,
way of 6. This 7 of nature by man is a grave mistake for
one of which fits the blank appropriately. Find out the appropriate
which mankind has to pay the price. Rapid industrialisation
word in each case.
means 8 the industrial effluents into the rivers and seas.
The river water has turned murky. Marine life has been 9. Studies 14 the impact of computer models to support policy-
The toxic chemicals have made the air that we breathe making processes in organisations have 15 that client
polluted. Pesticides and insecticides sprayed on plants and involvement in the model-building process is often a 16 for
the chemicals and fertilizers used for 10 plant yield have effective model-building. One important reason is that the
poisoned our food. Hence what we eat today has high toxic process of model-building is frequently more important than
11. Nature’s plentifulness is a heritage not to be 12 with the resulting model. Model-building itself is largely a 17
impunity. It must be conserved for future generations or its process about the problem. Most 18 about the
13 will extinguish all. characteristics of an ill-structured problem are gained during
1. (a) pursuit (b) view the 19 process of designing a computer model, rather than
(c) conquest (d) victim after the model is finished. Another important reason is that
(e) want most information in an organisation 20 in the mental models
2. (a) foul (b) diluted of organisation members. To support policy-making in an
(c) poor (d) precarious organisation, it is this knowledge which needs to be 21 and
(e) critical represented in the model. An important topic in client-
3. (a) disturbed (b) pure oriented or 22 model building thus becomes the 23 of
(c) counterproductive (d) suffocated relevant knowledge contained in the mental models of
(e) contaminated participants.
4. (a) dubious (b) wanton 14. (a) evaluating (b) focussing
(c) careful (d) planned (c) projecting (d) advocating
(e) useless (e) directing
5. (a) attacking (b) projecting 15. (a) devised (b) exhibited
(c) cutting (d) blasting (c) convinced (d) attributed
(e) sizing (e) indicated
6. (a) reclamation (b) inhabitation 16. (a) support (b) valuation
(c) stabilisation (d) destruction (c) prerequisite (d) material
(e) damage (e) blueprint
7. (a) provocation (b) adventure 17. (a) valuable (b) durable
(c) vandalism (d) abundance (c) tedious (d) learning
(e) evasion (e) critical
8. (a) relocating (b) divulging 18. (a) thinking (b) insights
(c) menacing (d) culminating (c) planning (d) appreciation
(e) diverting (e) opinion
9. (a) evaporated (b) endangered 19. (a) elongated (b) concentrated
(c) devalued (d) eliminated (c) iterative (d) evolving
(e) forfeiting (e) consummate
10. (a) managing (b) developing 20. (a) resides (b) follows
(c) maintaining (d) doubling (c) settles (d) lies
(e) minimising (e) committed
11. (a) damage (b) variable 21. (a) extended (b) bisected
(c) content (d) yield (c) subjected (d) captured
(e) refuge (e) attributed
C-78 CLOZE TEST
22. (a) revolving (b) interactive 34. (a) encouraging (b) flourishing
(c) dogmatic (d) accentuated (c) prosperous (d) agonizing
(e) formative (e) cheerful
23. (a) demarcation (b) formation 35. (a) play (b) bring
(c) proliferation (d) association (c) come (d) drop
(e) elicitation (e) sing
Directions (Qs. 24-38): In the following passage, there are blanks, 36. (a) light (b) day
each of which has been numbered. These numbers are printed (c) authority (d) person
below the passage and against each, five words are suggested, (d) sun
one of which fits the blank appropriately. Find out the appropriate 37. (a) scorch (b) shine
word in each case. (c) bright (c) burn
Once Gurudev Tagore asked Gandhiji: “Gandhiji, are you (e) illumine
24 unromantic’? When in the early 25 the morning sun 38. (a) brightness (b) shade
rises does it not 26 your heart with joy to see its reddish (c) dullness (d) strength
glow’? When the birds 27, does not your heart thrill with (e) stairs
its 28 music’? When the rose opens its petals and blooms Directions (Qs. 39 -48): In the following passage there are blanks;
in the garden, does its sight not bring 29 to your heart?” each of which has been numbered. These numbers are printed
The Mahatma replied, “Gurudev, I am not so dumb or 30 as below the passage and against each five words are suggested,
not to be moved by the beauty of the rose or the morning one of which fits the blank appropriately in the context of the
rays of the sun or the music of the birds. But what can I do? passage. Find out the appropriate word in each case.
My one 31, my one anxiety, my one ambition is: When shall The social 39 of the Web lifestyle and work style are
I see the red tint of the rose on the cheeks of 32 33 millions enormous. A lot of people 40 that computers and the
of my people’? When shall I hear the sweet and melodious Internet will depersonalize experience, creating a world that
song of the birds in place of their 34 sighs - when will such is less warm. But these are unfounded as we know that
music 35 out of their soul? And when will that 36 come. some people were 41 afraid that the telephone would reduce
when the light of the morning sun will 37 the heart of the face-to-face contact and will 42 society to fall apart. But the
common man in India? When will I see its lustre and 38 on 43 actually came true. Just as the phone and
his face?” e-mail have increased contact between people living in
24. (a) not (b) genuinely different communities and between people on the go, the
(c) seldom (d) s o PC and the Internet give us 44 way to communicate. They
(d) fairly do not take any away. In reality, the ability to use the Internet
25. (a) season (b) dawn to redefine 45 in our communities is strengthening personal
(c) monsoon (d) climate and cultural 46. The Web lifestyle is about broadening 47,
(e) days not narrowing them. Community building is going to be
one of the biggest growth areas on the Web. It dramatically
26. (a) involve (b) impeach
increases the number of communities you can bond to
(c) move (d) fill
because of its ability to 48 groups of like-minded people
(e) penetrate
independent of geography or time zones.
27. (a) fly (b) nestle
39. (a) groups (b) needs
(c) flock(d) cry
(c) factor (d) teaching
(e) sing
(e) implications
28. (a) alarming (b) fearful
40. (a) accept (b) dare
(c) divine (d) irritating
(c) fear (d) propose
(e) loud
(e) reject
29. (a) aroma (b) cheer
41. (a) strongly (b) initially
(c) fragrance (d) agony (c) always (d) never
(e) fear (e) possibly
30. (a) insensitive (b) lethargic 42. (a) let (b) decay
(c) ambitious (d) idle (c) develop (d) cause
(e) romantic (e) destroy
31. (a) slogan (b) request 43. (a) opposite (b) found
(c) interpretation (d) desire (c) finding (d) different
(e) demand (e) negative
32. (a) old (b) rich 44. (a) cheaper (b) economical
(c) happy (d) noble (c) another (d) second
(e) hungry (e) many
33. (a) naked (b) fashioned 45. (a) groups (b) ethics
(c) poor (d) fellow (c) culture (d) bonds
(e) playful (e) boundaries
CLOZE TEST C-79
46. (a) distances (b) connections The weaker sections of the rural population are mostly from
(c) differences (d) implications the socially and economically backward and 59 sections of
(e) suggestion the village community. Because of their 60 and financial
47. (a) horizons (b) values difficulty, they are not readily 61 to change their work habits
(c) nations (d) means and adopt modern technology. 62 sure about the traditional
(e) status methods, they are 63 to take to 64 equipment and techniques
48. (a) reduce (b) focus which require some time to get accustomed for 65 work.
(c) prepare (d) connect After holding a number of group meetings with rural people
(e) develop 66 to different vocations and spread over the entire country,
Directions (Qs. 49-58): In the following passage there are blanks, we can safely say that persons in the villages are not 67 for
each of which has been numbered. These numbers are printed training to improve upon their traditional and hereditary 68
below the passage and against each, five words are suggested, of working.
one of which fits the blank appropriately. Find out the appropriate 59. (a) depressed (b) different
word in each case. (c) rich (d) privileged
The study of accountancy is 49 in demand in the view of (e) forward
50 of greater complexity in our business organisation. 60. (a) ability (b) dependence
Formerly a 51 of day-to-day income and expenditure was (c) illiteracy (d) number
more than 52. A business organisation today has to 53 a (e) majority
clear account of the 54 it uses, the amounts that are owing 61. (a) discarding (b) feeling
to it, the amount that it owes to others, the profit or loss it (c) bending (d) undertaking
has made and the 55 it employs. Without a scientific 56 of (e) willing
accounting no businessman can be fully 57 of his real 58 62. (a) Making (b) Having
position and run his organisation. (c) Quite (d) Being
49. (a) progressing (b) getting (e) Not
(c) powering (d) moving 63. (a) forced (b) reluctant
(e) growing (c) bound (d) prepared
(e) curious
50. (a) demand (b) growth
64. (a) farming (b) traditional
(c) status (d) position
(c) improved (d) powerful
(e) slackness
(e) old
51. (a) mixture (b) map
65. (a) routine (b) monotonous
(c) measure (d) record
(c) excessive (d) wasteful
(e) transaction
(e) effective
52. (a) sufficient (b) anticipated
66. (a) accruing (b) helping
(c) expected (d) required
(c) enabling (d) belonging
(e) necessary (e) referring
53. (a) gather (b) observe 67. (a) eager (b) capable
(c) maintain (d) organize (c) indifferent (d) antagonistic
(e) assimilate (e) unwilling
54. (a) manpower (b) infrastructure 68. (a) theories (b) techniques
(c) money (d) resources (c) desires (d) hours
(e) capabilities (e) policies
55. (a) capital (b) strength Directions (Qs. 69-78): In the following passage, there are blanks,
(c) authority (d) strategies each of which has been numbered. These numbers are printed
(e) principles below the passage and against each, five words are suggested,
56. (a) way (b) plan one of which fits the blank appropriately. Find out the appropriate
(c) system (d) goal word in each case.
(e) purpose In the past, it was thought learning knowledge took place
57. (a) ignorant (b) alert in school and for some also in further education. Then, it
(c) prepared (d) vigilant was a matter of 69 practical skills at work at the beginning
(e) aware of a career, and with a bit of luck, that 70 it. Now, things
58. (a) administrative (b) financial have changed. Global competition is 71 the shelf life of
(c) capacity (d) business products and the knowledge and skills that 72 behind them.
(e) hierarchical The pace of change can be 73. Knowledge that was at the
Directions (Qs. 59-68): In the following passage, there are blanks, leading edge one minute can become 74 the next. Therefore,
each of which has been numbered. These numbers are printed it is 75 rather than knowledge that is the key. Successful
below the passage and against each, five words are suggested, organizations have to learn, adapt and change continuously
one of which fits the blank appropriately. Find out the appropriate as do the 76 within them. This is 77 in the rapid growth of
word in each case. knowledge workers. It is 78 all levels of organizations.
C-80 CLOZE TEST
69. (a) fostering (b) projecting 83. (a) absorbing (b) relieving
(c) acquiring (d) manipulating (c) avoiding (d) resolving
(e) culminating (e) recognising
70. (a) for (b) was 84. (a) inserted (b) developed
(c) from (d) with (c) conceived (d) controlled
(e) may (e) applied
71. (a) replacing (b) retailing 85. (a) recuperate (b) alleviate
(c) rotating (d) re- regulating (c) conceal (d) indicate
(e) reducing (e) slow
72. (a) lie (b) profess 86. (a) to (b) also
(c) exhibit (d) manifest (c) although (d) as
(e) express (e) hardly
73. (a) analytical (b) absorbing 87. (a) ingredient (b) aspect
(c) interesting (d) frightening (c) offshoot (d) outcome
(e) discovery
(e) valuable
88. (a) that (b) these
74. (a) critical (b) obsolete
(c) their (d) our
(c) modern (d) devastating
(e) remote
(e) lamentable
Directions (Qs. 89-98): In the following passage there are blanks,
75. (a) durability (b) reactivity each of which has been numbered. These numbers are printed
(c) activity (d) Proactivity below the passage and against each, five words are suggested,
(e) capacity one of which fits the blank appropriately. Find out the appropriate
76. (a) systems (b) managements word in each case.
(c) processes (d) individuals Man has always considered himself to be the ruler of his
(e) units planet. This 89 and the attendant superiority feeling has
77. (a) echoed (b) supported made him look down 90 other creatures who co-exist with
(c) adjusted (d) provided human on this earth. The so-called civilized human race has
(e) developed 91 and ill-treated small and large animal species and birds in
78. (a) directing (b) providing an attempt to prove his 92. It is common knowledge that 93
(c) affecting (d) questioning number of animals have been 94 for centuries under the 95
(e) projecting of conducting scientific experiments or for sports. Till
Directions (Qs. 79-88): In the following passage there are blanks, recently, in the 96 of scientific experiments, monkeys and
each of which has been numbered. These numbers are printed frogs have been 97 to dissection and 98 in the laboratory.
below the passage and against each, five words are suggested, 89. (a) pleasure (b) fact
one of which fits the blanks appropriately. Find out the appropriate (c) achievement (d) force
words. (e) arrogance
The tea plant, a native of Southern China, was known 79 90. (a) in (b) upon
very early times in Chinese botany and medicine. It is 80 in (c) with (d) for
the classics 81 the various names of Tou, Tseh, Chung, (e) into
Kha and Ming and was 82 praised for possessing the 91. (a) criticised (b) devalued
virtues of 83 fatigue, delighting the soul, strengthening the (c) protected (d) abused
will and repairing the eyesight. It was not only administered (e) enlarged
as an internal dose, but often 84 externally in the form of 92. (a) supremacy (b) wisdom
paste to 85 rheumatic pains. The Taoists claimed it 86 an (c) cleverness (d) instinct
(e) possession
important 87 of the elixir of immortality. The Buddhists used
93. (a) tall (b) plenty
it extensively to prevent drowsiness during 88 long hours
(c) countless (d) diverse
of meditation.
(e) numerous
79. (a) to (b) after
94. (a) tortured (b) exposed
(c) from (d) beyond (c) treated (d) vanished
(e) behind (e) extinct
80. (a) taken (b) resorted 95. (a) projection (b) criticism
(c) awarded (d) alluded (c) pretext (d) game
(e) introduced (e) study
81. (a) under (b) between 96. (a) matter (b) set
(c) among (d) besides (c) scheme (d) virtue
(e) like (e) name
82. (a) rarely (b) loosely 97. (a) confined (b) subjected
(c) under (d) severely (c) condemned (d) allied
(e) highly (e) performed
CLOZE TEST C-81
98. (a) cruelty (b) deformation 106. (a) economic (b) appropriate
(c) study (d) vivisection (c) ample (d) higher
(e) proliferation (e) social
Directions (Qs. 99-108) : In the following passage, there are 107. (a) unequal (b) prevailing
blanks, each of which has been numbered. These numbers are (c) appropriate (d) commendable
printed below the passage and against each, five words are (e) deplorable
suggested, one of which fits the blank appropriately. Find out the 108. (a) reflected (b) exempted
appropriate word in each case. (c) barred (d) considered
In 99 of constitutional guarantees relating to equality of (e) neglected
opportunity and various other guarantees of equality before Directions (Qs. 109-118): In the following passage, there are
the law, the social and economic 100 of women, especially blanks, each of which has been numbered. These numbers are
of poor women in India, is well-known. We are referring printed below the passage and against each, five words are
mainly to the poor rural women who have little or no assets suggested, one of which fits the blank appropriately. Find out the
and who 101 the bulk of the female population in rural appropriate word in each case.
areas. It is not as if only poor rural women get less wages or The urgent need of the hour is to 109 up the moral 110 of
suffer from social 102 because they belong to a particular our society in general and of our student community in
community. Even at higher levels of the socio-economic particular, if we want to save ourselves and our society
hierarchy among the well-to-do groups, women are not 103 from the present 111 of mass indiscipline and 112 of basic
to men. Among the economically 104 sections of society, human values, which has become a 113 phenomenon. We
women’s proper place is 105 to be the home. In rural areas, must, therefore, 114 and practise the most 115 basic human
women of 106 status families normally do not go out to values like cooperation, tolerance, patriotism, generosity,
work. In the 107 value system, there is a gradation of truth, justice and excellence — the ideals which are universal
economic activities, which is 108 in the socio-economic in nature and which are 116 in themselves and which are
status of the family. worthy of 117 for their own sake. These ideals are both
Thus, if the women of the family do manual labour in the personally as well as socially 118.
fields, it denotes low status. Women earning a living, or 109. (a) give (b) stand
supplementing their family income through economic (c) jack (d) climb
activities like stitching, garment-making, or some handicraft (e) tone
work, are also considered low because it clearly shows that 110. (a) fibre (b) enactment
their family is poor and they are forced to make ends meet. (c) reconstruction (d) situation
It is considered right and proper for a woman to cook, sew (e) appreciation
and take up activities like pickle-making for her own family. 111. (a) polarisation (b) degradation
But, if she were to earn a wage through these same activities, (c) chaos (d) provocation
it denotes poverty and also, often, low socio-economic (e) sentiments
status. 112. (a) calamity (b) focus
99. (a) support (b) spite (c) realisation (d) erosion
(c) contrast (d) wake (e) criticism
(e) view 113. (a) durable (b) universal
100. (a) condition (b) prosperity (c) perpetual (d) segmental
(c) progress (d) deprivation (e) prolific
(e) value 114. (a) incorporate (b) induce
101. (a) constitute (b) deploy (c) implicate (d) inculcate
(c) measure (d) define (e) involve
(e) exploit 115. (a) absorbing (b) cherished
102. (a) status (b) service (c) introspective (d) famous
(c) indifference (d) ignorance (e) productive
(e) discrimination 116. (a) distinctive (b) appreciated
103. (a) dedicated (b) accountable (c) formative (d) helping
(c) equal (d) responsible (e) end
(e) antagonistic 117. (a) evolving (b) spreading
104. (a) marginal (b) significant (c) esteem (d) wisdom
(c) well-off (d) affordable (e) popularity
(e) dependable 118. (a) desirable (b) manageable
105. (a) entitled (b) decided (c) redundant (d) valnerable
(c) indicated (d) debated (e) possible
(e) considered
C-82 CLOZE TEST

ANSWER KEY
1 (c) 13 (e) 25 (b) 37 (e) 49 (e) 61 (e) 73 (d) 85 (b) 97 (b) 109 (e)
2 (a) 14 (a) 26 (c) 38 (a) 50 (a) 62 (d) 74 (b) 86 (d) 98 (d) 110 (a)
3 (e) 15 (e) 27 (e) 39 (e) 51 (d) 63 (b) 75 (d) 87 (a) 99 (b) 111 (c)
4 (b) 16 (c) 28 (c) 40 (c) 52 (a) 64 (c) 76 (d) 88 (c) 100 (a) 112 (d)
5 (c) 17 (d) 29 (b) 41 (b) 53 (c) 65 (e) 77 (a) 89 (e) 101 (a) 113 (b)
6 (a) 18 (b) 30 (a) 42 (d) 54 (c) 66 (d) 78 (c) 90 (b) 102 (e) 114 (d)
7 (c) 19 (c) 31 (d) 43 (a) 55 (a) 67 (a) 79 (c) 91 (d) 103 (c) 115 (b)
8 (e) 20 (d) 32 (e) 44 (c) 56 (c) 68 (b) 80 (d) 92 (a) 104 (c) 116 (a)
9 (b) 21 (d) 33 (a) 45 (d) 57 (e) 69 (c) 81 (a) 93 (c) 105 (e) 117 (c)
10 (b) 22 (b) 34 (d) 46 (b) 58 (b) 70 (b) 82 (e) 94 (a) 106 (d) 118 (a)
11 (c) 23 (e) 35 (c) 47 (a) 59 (a) 71 (e) 83 (b) 95 (c) 107 (b)
12 (a) 24 (d) 36 (b) 48 (e) 60 (c) 72 (a) 84 (e) 96 (e) 108 (a)
Section D : TEST OF COMPUTER KNOWLEDGE

Computer

1 Chapter
Knowledge

ABBREVIATIONS Blog : Web Log


BMP : Basic Multilingual Plane
AAC : Advanced Audio Coding (audio compression format BOOTP : Bootstrap Protocol
defined by the MPEG-2 standad) BPEL : Business Process Execution Language
ABI : Application Binary Interface bps : bits per second
ABR : Area Border Router CAD : Computer- Aided Design
ABR : Available Bit Rate CAE : Computer- Aided Engineering
AD : Active Directory CAI : Computer- Aided Instruction
ADC : Analog - To - Digital Converter CAT : Computer- Aided Translation
ADC : Apple Display Connector (DVI variant) CAQ : Computer- Aided Quality Assurance
AH : Active Hub cc : C Compiler
AHA : Accelerated Hub Architecture CD : Compact Disc
Ajax : Asynchronous Java Script and XML CD-R : CD-Recordable
AL : Active Link CD-ROM : CD Read-only Memory
ALGOL : Algorithmic Language CD-RW : CD- Rewritable
ALU : Algorithmic and Logical Unit CG : Computer Graphics
CGA : Colour Graphics Array
AM : Active Monitor
CGI : Common Gateway Interface
AMD : Advanced Micro Devices
CGI : Common -Generated Imagenry
AMR : Audio Modem Riser
CIFS : Common Internet Filesystem
AoE : ATA over Ethernet
CLI : Command Line Interface
APCI : Application- Layer Protocol Control Information CLR : Common Language Runtime
API : Application Programming Interface CNC : Computer Numerical Control
APIPA : Automatic Private IP Addressing COBOL : Common Business-Oriented Language
ASCH : American Standard Code for Information Inter CPU : Central Processing Unit
change CRT : Cathode Ray Tube
ASG : Abstract Semantic Graph CSI : Common System Interface
ASP : Application Service Provider CT : Computerised Tomography
AST : Abstract Syntax Tree CTCP : Client-To-Client Protocol
ATA : Advanced Technology Attachment CTL : Computational Tree Logic
ATM : Asynchronous Transfer Mode CTS : Clear To Send
AVC : Advanced Video Interleaved CUA : Common User Access
DAC : Digital-To-Analog Converter
AWT : Abstract Windowing Toolkit
DAP : Directiory Access Protocol
BASIC : Beginner's All-Purpose Symbolic Instruction Code
DB : Databse
BCD : Binary Coded Decimal
DBA : Database Administrator
BEEP : Blocks Extensible Exchange Protocol DBMS : Database Management System
BER : Bit Error Rate DCC : Direct Client-to-Client
BFD : Binary File Descriptor DDR : Double Data Rate
BGP : Border Gateway Protocol DES : Data Encryption Standard
bin : binary DFD : Data Flow Diagram
BINAC : Binary Automatic Computer DFS : Distributed File System
BIOS : Basic Input Output System DHTML : Dynamic HTML
bit : binary digit DIVX : Digital Video Express
Blob : Binary large object DLL : Dynamic Link Library
D-2 COMPUTER KNOWLEDGE
DLP : Digital Light Processing HHD : Hybrid Hard Drive
DMA : Direct Memory Access HID : Human Interface Guidelines
DOS : Disk Operating System HIG : Human Interface Guidelines
DPI : Dots Per Inch HPFS : High Performance File System
DPMI : DOS Protected Mode Interface HSM : Hierarchical Storage Management
DSL : Digital subscriber Line HTM : Hierarchical Temporal Memory
DSL : Domian- Specific Language HTML : Hypertext Markup Language
DSN : Database Source Name (ODBC) HTTP : Hypertext Transfer Protocol
DTE : Data Terminal Equipment HTX : Hyper Transport eXpansion
DTR : Data Terminal Ready HURD : Hird of Unix- Replacing Daemons
DVD : Digital Versatile Disc HVD : Holographic Versatile Disc
DVD : Digital Video Disc IBM : International Business Machines
DVD-R : DVD- Rewritable ICMP : Internet Control Message Protocol
DVI : Digital Visual Interface ICP : Internet Cache Protocol
DVR : Digital Video Recorder IDL : Interface Definition Language
EAP : Extensible Authentication Protocol IE : Internet Explorer
EBCDIC : Extended Binary Coded Decimal Interchange Code IGMP : Internet Group Management Protocol
EDO : Extended Data Out IGRP : Interior Gateway Routing Protocol
EEPROM : Electronically-Erasable Programmable Read-Only IHV : Independent Hardware Vendor
Memory IIOP : Internet Inter-Orb Protocol
EFF : Electronic Frontier Foundation IIS : Internet Information Services
EFI : Extensible Firmware Interface IM : Instant Messaging
EGA : Enhanced Graphics Array IMAP : Internet Message Access Protocol
EGP : Exterior Gateway Protocal I/O : Input/Output
eID : electronic ID card IP : Internet Protocol
EIGRP : Enhanced Interior Gateway Routing Protocol IPC : Inter-Process Communication
ELF : Executable and Linkable Format IPP : Internet Printing Protocol
ELM : Electronic Mail Ipsec : Internet Protocol security
EOM : End of Message IPTV : Internet Protocol Television
EPROM : Erasable Programmable Read-Only Memory IPX : Internetwork Packet Exchange
EUC : Extended Unix Code IRC : Internet Relay Chat
EXE : EXEcutable IRP : I/ORequest Packet
FAP : FORTRAN Assembly Program IRQ : Interrupt Request
FAT : File Allocation Table ISC : Internet Storm Center
FAQ : Frequently Asked Questions ISO : International Organisation for Standardisation
FDC : Floppy Disk Controller ISOC : Internet Society
FDD : Floppy Disk Drive ISP : Internet Service Provider
FIFO : First In Fist Out ISR : Interrupt Service Routine
FHS : Filesystem Hierarchy Standard ISV : Independent Software Vendor
FCS : Frame Check Sequence IT : Information Technology
FPU : Floating Point Unit ITU : International Telecommunication Union
FS : File System J2CE : Java 2 Cryptograhic Edition
FSB : Front Side Bus JDS : Java Desktop System
FTP : File Transfer Protocol JMX : Java Management Extensions
FXP : File eXchange Protocol JMS : Java Message Service
Gb : Gigabit JNDI : Java Naming and Directory Interface
GB : Gigabyte JNI : Java Native Interface
GCR : Group Code Recording JPEG : Joint Photographic Experts Group
GDI : Graphics Device Interface JS : Java Script
GIF : Graphics Interchange Format JSON : Java Script Object Notation
GIGO : Garbage In, Garbage Out JSP : Java Server Pages
GPU : Graphics Processing Unit JUG : Java Users Group
GUI : Graphical User Interface Kb : Kilobit
HAL : Hardware Abstraction Layer KB : Kilobyte
HCI : Human Computer Interaction kHz : Kilohertz
HDD : Hard Disk Drive LAN : Local Area Network
HD DVD : High Definition DVD LIFO : Last In first Out
HDL : Hardware Description Language LSB : Least Significant Bit
COMPUTER KNOWLEDGE D-3
MAN : Metropolitan Area Network RARP : Reverse Address Resolution Protocol
MANET : Mobile Ad- Hoc Network RDBMS : Relational Database Management System
Mb : Megabit RDF : Resource Description Framework
MB : Megabyte REFAL : REcursive Functions Algorithmic Language
MBCS : Multi Byte Character Set RIP : Routing Information Protocol
MBR : Master Boot Record ROM : Read Only Memory
MDI : Multiple Document Interface ROMB : Read-Out Motherborad
MICR : Magnetic Ink Character Recognition RTOS : Real Time Operating System
MIMO : Multiple-Input Multiple-Output SaaS : Software as a Service
MIPS : Million Instructions Per Second SAN : Storage Area Network
MIME : Multipurpose Internet Mail Extensions SATA : Serial ATA
MMX : Multi-Media Extensions SAX : Simple API for XML
MNG : Multiple-Image Network Graphics SBP-2 : Serial Bus Protocol 2
MPEG : Motion Pictures (Coding) Experts Group SBU : Standard Build Unit
MPL : Mozilla Public License SCSI : Small Computer System Interface
MSB : Most Significant Bit SDL : Simple Direct Media Layer
MS-DOS : Microsoft DOS SDN : Service Delivery Network
MVS : Multiple Virtual Storage SDR : Software-Defined Radio
NEXT : Near- End Cross Talk SDRAM : Synchronous Dynamic Random Access Memory
NFS : Network File System SMBIOS : System Management BIOS
NIO : New I/O SMTP : Simple Mail Transfer Protocol
NMI : Non- Maskable Interrupt SP : Service Pack
NNTP : Network News Transfer Protocol SPI : Serial Peripheral Interface
NOP : No Operation SQL : Structured Query Language
NOS : Netwrok Operating System SUS : Single UNIX Specification
NT (Windows) New Technology SVD : Structured VLSI Design
NTP : Network Time Protocol TCP : Transmission Control Protocol
OOP : Object- Oriented Programming TCP/IP : Transmission Control Protocol/Internet Protocol
OPML : Outline Processor Markup Language TTA : True Tap-Audio
OS : Operating System TTF : True Type Font
OSS : Open- Source Software TTS : Text-to-Speech
P2P : Peer-To- Peer TTY : Teletype
PAN : Personal Area Network UAC : User Account Control
PAP : Password Authentication Protocol UART : Universal Asynchronous Receiver Transmitter
PATA : Parallel ATA UEFI : Unified Extensible Firmware Interface
PC : Personal Computer UI : User Interface
PCI : Peripheral Component Interconnect UL : Upload
PCLe : PCI Express UPS : Uninterruptible Power Supply
PCL : Printer Command Language URI : Uniform Resource Identifier
PGA : Pin Grid Array URN : Uniform Resource Name
PIC : Peripheral Interface Controller USB : Universal Serial Bus
PIC : Programmable Interrupt Controller Var : variable
PINE : Program for Internet News & Email VB : Visual Basic
PIO : Programmed Input/ Output VBA : Visual Basic for Applications
PnP : Plug-and- Play VBS : Visual Basic script
PoE : Power over Ethernet VFAT : Virtual FAT
PPC : Power PC VFS : Virtual File System
PPI : Pixels Per Inch VGA : Video Graphics Array
PPP : Point-to-Point Protocol VGCT : Video Graphics Character Table
PPPoA : PPP over Ethernet VLAN : Virtual Local Area Network
PPTP : Point- to-Point Tunneling Protocol VM : Virtual Memory
PSU : Power Supply Unit VOD : Video On Demand
QDR : Quad Data Rate VoIP : Voice over IP
QFP : Quoted For Permanence VPN : Virtual Private Network
QoS : Quality of Service VT : Video Terminal
RADIUS : Remote Authentication Dial In User Service WAFS : Wide Area File Services
RAID : Redundant Array of Independent Disks WAIS : Wide Area Information Server
RAM : Random Access Memory WAN : Wide Area Network
D-4 COMPUTER KNOWLEDGE
WAP : Wireless Application Protocol Artificial Intelligence: Artificial intelligence (AI) is the
Wi-Fi : Wireless Fidelity intelligence of machines and the branch of computer science that
WiMAX : Worldwide Interoperability for Microwave Access aims to create it. AI textbooks define the field as "the study and
WInFS : Windows Future Storage design of intelligent agents" where an intelligent agent is a system
WINS : Windows Internet Naming Service that perceives its environment and takes actions that maximize its
WLAN : Wireless Local Area Network chances of success.
WMA : Windows Media Audio ASCII (American Standard Code for Information
WMV : Windows Media Video Interchange): ASCII a code for information exchange between
WOL : Wake-on-LAN computers made by different companies; a string of 7 binary digits
WOM : Wake-on-Modem represents each character; used in most microcomputers
WPA : Wi-Fi Protected Access Assembly Language: A programming language that is once
WSDL : Web Services Description Language removed from a computer's machine language. Machine languages
WUSB : Wireless Wide Area Network consist entirely of numbers and are almost impossible for humans
WWID : World Wide Identifier to read and write. Assembly languages have the same structure
WWW : World Wide Web and set of commands as machine languages, but they enable a
XAML : eXtensible Application Markup Language programmer to use names instead of numbers.
XHTML : eXtensible Hypertext Markup Language Auxilliary Memory: A high-speed memory bank used in
XML : eXtensible Markup Language mainframes and supercomputers. It is not directly addressable by
XMMS : X Multimedia System the CPU; rather, it functions like a disk. Data are transferred from
XNS : Xerox Network Services auxiliary memory to main memory over a high-bandwidth channel.
XSL : eXtensible Styleheet Language Backup: A backup or the process of backing up is making
XSL-FO : eXtensible Stylesheet Language Formatting Objects copies of data which may be used to restore the original after a
XSLT : eXtensible Stylesheet Language Transformatons
data loss event.
XUL : XML User Interface Language
Y2K : Year Two Thousand Band Width: In computer networking and computer science,
ZIFS : Zero Insertion Force Socket bandwidth, network bandwidth, data bandwidth or digital
ZISC : Zero Instruction Set Computer bandwidth is a bit rate measure of available or consumed data
ZMA : Zone Multicast Address communication resources expressed in bits/second or multiples
of it (kilobits/s, megabits/s etc.).
IMPORTANT TERMS BIOS: Basic Input Output System. This is the basic set of
Access Time: Access time is the time from the start of one instructions that tell the computer how to act. Most computers
storage device to the time when the next access can be started. have these instructions built into a chip that plugs into the
Accessory: An Accessory is a device attached to a host motherboard,
computer, but not part of it, and is more or less dependent on the Bar Code: A bar code (often seen as a single word, barcode)
host. It expands the host's capabilities, but does not form part of is the small image of lines (bars) and spaces that is affixed to retail
the core computer architecture. store items, identification cards, and postal mail to identify a
Examples are computer printers, image scanners, tape drives,
particular product number, person, or location.
microphones, loudspeakers, webcams, and digital cameras.
Binary: Computers are based on the binary numbering
Active Cell: The cell that continues the value being used or
modified in a spreadsheet program, and that is highlighted by the system, which consists of just two unique numbers, 0 and 1.
cell pointer. Also known as current cell. Biometric Device:Biometrics (or biometric authentication)
Active Window : The window in Microsoft Windows with consists of methods for uniquely recognizing humans based upon
which the user may interact. one or more intrinsic physical or behavioral traits.
Accumulator: The computer register in which the result of Bitmap: In computer graphics, a bitmap or pixmap is a type
an arithmetic or logic operation is formed (related to arithmetic of memory organization or image file format used to store digital
and logic unit). images.
Algorithm: A standard method for computing something; Blue tooth: Bluetooth is a proprietary open wireless
essentially, a mathematical recipe. technology standard for exchanging data over short distances
Analog: A continuous waveform signal that can be used to (using short wavelength radio transmissions in the ISM band
represent such things as sound, temperature, and velocity. from 2400-2480 MHz) from fixed and mobile devices, creating
Analog Computer: A computer in which numerical data are personal area networks (PANs) with high levels of security.
represented by measurable physical variables, such as electrical. Booting: To boot (as a verb; also "to boot up") a computer
Antivirus: Computer antivirus refers to a software program is to load an operating system into the computer's main memory
that can protect your computer from unwanted viruses and remove or random access memory (RAM).
any, that penetrate your computer's defenses. Browse: In database systems, browse means to view data.
Arithmatic Logic unit (ALU): An arithmetic-logic unit (ALU) Many database systems support a special browse mode , in which
is the part of a computer processor (CPU) that carries out arithmetic you can flip through fields and records quickly. Usually, you
and logic operations on the operands in computer instruction cannot modify data while you are in browse mode.
words.
COMPUTER KNOWLEDGE D-5
Bug: A software bug is the common term used to describe Disk: A magnetically encoded storage medium in the form
an error, flaw, mistake, failure, or fault in a computer program or of a plate (also called a platter).
system that produces an incorrect or unexpected result, or causes Disk Operating System (DOS): A disk operating system
it to behave in unintended ways. manages disks and other system resources. Sort of a subset of
Byte: Byte is a unit of digital information in computing and OSes, sort of an archaic term for the same. MS-DOS is the most
telecommunications that most commonly consists of eight bits. popular program currently calling itself a DOS. CP/M was the
CD ROM (Compact Disk- Read Only Memory): a type of most popular prior to MS-DOS.
optical disk capable of storing large amounts of data -- up to 1GB, Domain Names: A name given to a host computer on the
although the most common size is 650MB (megabytes). Internet, E-mail names are good examples of domain names (for
CD-R/W (Compact Disk-Recordable):a type of CD disk that example bijendra@kbscontent.com).
enables you to write onto it in multiple sessions. One of the Downloading:Retrieving a file or group of files from the
problems with CD-R disks is that you can only write to them once. Internet so that they can be stored on a local hard drive. By
Central Processing Unit (CPU):The CPU is the computer's accessing this page, you have, in fact, downloaded all the
information here so that it can be viewed and interpreted by your
control center. Think of it as the brain that does all the thinking
(computation), thus it is called the Central Processing Unit. The web browser.
actual CPU is about 1.5 inches square, yet it is the most critical Electronic Mail:When a message is sent, the message is
part of the computer. Having a fast CPU (measured in MegaHertz) sent first to the SMTP server, which acts as an "outbox" for users.
greatly aids in the overall speed of your computer. The message is then relayed to the appropriate mail server, which
CMOS: Acronym " Complimentary Metal Oxide can be found listed after the @ symbol in the recipient's address.
Semiconductor" A CMOS computer ciruit consumes very little The message then waits on that server until the recipient accesses
power and is used in computers to keep track of the system setup the message and then deletes it.
information, data, time, type of disk and hard drives, etc, that a Ethernet:A transport method (protocol) used to connect
computer has in stalled. computers to a LAN (Local Area Network) and exchange data.
Compressed File: Computer files that have been reduced in File: (1) A collection of related records. (2) A named area on
size by a compression program. Such programs are available for a disk-storage device that contains a program or digitized
all computer systems. information (text, image, sound, and so on). (3) A component of
Central Processing Unit (CPU):The Central Processing Unit an overall program or application.
(CPU) is an electronic component that interprets and carries out Font: In a simplistic sense, a font can be thought of as the
the instructions of any application that run on a computer. It is a physical description of a character set. While the character set
place where all the computing is done. will define what sets of bits map to what letters, numbers, and
Data: Representations of facts. The raw material of other symbols, the font will define what each letter, number, and
information.(Plural of datum.) other symbol looks like.
Database: The integrated data resource for a computer- Format:(1) The logical or physical arrangement of the tracks
based information system. and sectors on a floppy diskette or a hard disk. To be usable, a
DDR: This is a new type of RAM called Double Data Rate disk must be formatted so that the tracks and sectors are laid out
RAM. It is used in some of the newer video cards such as the in a manner compatible with the operating system in use. (2) To
Nvidia GeForce cards. prepare a disk or diskette, dividing it into sectors so that it is
Desk Top:The screen in Windows upon which icons, ready to receive data.
windows, a background, and so on are displayed. Gigahertz: One gigahertz is equivalent to 1000 megahertz,
Desk Top Publishing (DTP): Software that allows users to or 1,000,000,000 hertz.
produce near-typeset-quality copy for newsletters, Hacker:An individual with vast experience with security
advertisements, and many other printing needs, all from the protocols who attempts to illegally access secure servers in an
attempt to download private information, damage systems, or act
confines of a microcomputer.
Dial up :A dial-up Internet account allows you to use a in some other way to "free information".
computer with a modem and appropriate software to connect to Hard Copy: A readable printed copy of computer output.
the Internet through an Internet Service Provider (ISP). The Hard Disk:Hard disk (internal) is a permanent file and data
software “dials” the ISP’s access numbers and you can then send storage device housed in a computer case.
e-mail, browse the World Wide Web or engage in other Internet Hardware:Collective term for any computer-related object
activities. that can be kicked or battered.
Digital: Terms used to describe any information that has Hexadecimal Number System:A numeric notation system
been translated into a corresponding series of is and 0s; any with a base of 16 decimal frequently used to specify addresses in
information text, sound, image color, may be digitized. computer memory. In hexadecimal notation, the decimal numbers
Digital Computer:A reference to any system based on 0 through 15 are represented by the decimal digits 0 through 9
discrete data, such as the binary nature of computers. and the alphabetic “digits” A through F (A = decimal 10, B =
Digital Video/ Versatile Disk (DVD):The successor decimal 11, etc.). Can be formed as two 4-bit binary numbers from
technology to the CD-ROM that can store up to 10 gigabytes. an 8-bit binary number split into two parts.
D-6 COMPUTER KNOWLEDGE
Home Page:The Web page which is the starting point for tape cartridges, or disk modules enabling ready access to vast
accessing information at a site or in a particular area. amounts of online data.
Host: A computer, attached to a network which provides Key-Board: is one of computer components which used to
services to another computer beyond simply storing and input data to a computer. It is called an input device.
forwarding information. Laptop: Laptop is small and lightweight computer in which
Hyper Text Markup Language: This is the code by which all the main parts fitted into single unit. It is designed to carry it
web pages are created so they can be graphically organized in around. Particularly, it is ideal for travelers, journalists,
various ways. The web browser downloads the text of the HTML commentators and professionals who want to work both at the
file, and then decodes the text into what you can see here. Many office and home.
books and online manuals are available to anyone wishing to LCD: Acronym " Liquid Crystal Display" is the technology
learn this code. used for displays in notebook and other smaller computers.
HTML: Acronym "Hyper Text Mark-up Llanguage" which Linux: An open source spinoff of the UNIX operating
is used to format information so that it can be structured and system that runs on a number of hardware platforms and is
made accessible to the World Wide made available for free over the Internet.
HTTP: Acronym "Hypertext Transfer protocol" The protocol Local Area Network: Many multiple-computer homes have
that forms the basis of World Wide Web technology. HTTP is the found ways to link their computers through a central device called
set of rules governing the software that transports hyperlinked a "hub". This way, each computer can share information directly,
files along the Internet. without the need to transfer data via a portable storage device,
Information Technology (IT): including ICT (Information like a floppy disk. A properly set up LAN can also permit the
and Communication Technology) is the application of appropriate connected computers to access the Internet through a single
(enabling) technologes to information processing. Internet account.
Input/output (I/O): A generic reference to input and/or output Log on & Log off: Each server that is accessed must have
to a computer. some way to ensure security of their sensitive information. Thus,
IP: Acronym "Internet Protocol" The standard protocol used servers restrict access by forcing users to "log on" with either
by systems communicating across the Internet. personal access codes or anonymously. Anonymous access
IP Address: A digital code that precisely locates a computer usually requires the individual's e-mail address, and the user's IP
connected to the Internet. address is also logged. Once the desired information has been
MAC: Short for " Macintosh"; the other type of personal obtained, the user can "log off", disconnecting access to the
computer, manufactured by Apple Computer server.
Inkjet Printer:A non-impact printer in which the print head Machine Language:Machine language consists of the raw
contains independently controlled injection chambers that squirt numbers that can be directly understood by a particular processor.
ink droplets on the paper to form letters and images. Each processor's machine language will be different from other
Integrated Services Digital Network (ISDN):A digital processors' machine language. Although called "machine
telecommunications standard for data delivery over twisted-pair language", it is not usually what people think of when talking
lines with transmission speeds up to 128 Kbps (two 64 Kbps line about computer languages. Machine language dressed up with
pairs). mnemonics to make it a bit more human-readable is called assembly
InterFace: (1) A specific hardware or software connection. language.
(2) Making two devices capable of communication. Used most Main Frame Computer:A large computer that can service
often to refer the design of hardware and software that allows many users simultaneously in support of enterprise-wide
connection of network components and transfer of information. applications.
Internet: Internet is the largest wide area network in the Memory: One of the essential components of a computer’s
world which links millions of computers. Through internet central processing unit. Memory is the area where information
information can be shared, business can be conducted and and programs are actively processed.
research can be done. Micro Computer:A small computer.
IP Address (Internet Protocol Address) :A unique numerical Microprocessor:A computer on a single chip. The central
Internet address identifying any piece of equipment hooked up processing component of a microcomputer.
to the Internet (see DNS, dotted quad notation, and IP). Modem: Modem is a telecommunication device that converts
Intranet:An Internet-like network whose scope is restricted digital signals to analog and vice versa. It is used in dial-up internet
to the networks within a particular organization. connection to connect telephone line to a computer.
Java: Java is a programming language and has a “sandboxed” Monitor: The high-resolution TV-like tube that displays your
code interpreter which permits programs to be downloaded to computer's output. Today's monitors have much better quality
PC’s from the Web, but isolates these applications from access to displays than any TV is capable of producing.
other applications running on the PC. Mother Board:is the core of a computer system. It is the
JPEG (Joint Photographic Experts Group): A bit-mapped circuit board where all other parts connect. It communicates and
file format that compresses image size. controls the overall system. No motherboard means no computer
Jukebox: A storage device for multiple sets of CD-ROMs, system.
COMPUTER KNOWLEDGE D-7
MP3 : this stands for "MPEG I Audio Lalyer- 3" and is a ROM: Acronym "Read Only Memory". in which information
digital. compressed music file (these files always end with a mp3). is saved once and can never be altered For example. CD-ROM
MP3 files are often downloaded or exchanged between people drives read information saved on compact disks (CD's). A CD-
online. ROM drive can read that information, but cannot make changes
MPEG: Acronym " Motion Picture Experts Group" A video to it. for that you need a CD- RW drive. Some ROM is built into
file compression system used on the web. your computer to help it get started when you turn it on.
Mouse: A small, handheld device attached to a computer; Scanner: A scanner is a piece of hardware that will examine
when moved across any flat surface (such on the computer screen a picture and produce a computer file that represents what it sees.
called a cursor) includes one or more buttons that allow the user A digital camera is a related device. Each has its own limitations.
to select graphics or text onscreen. Search Engine: A tool used which matches key words you
Multimedia application: Computer applications that enter with titles and description on the Internet. It then displays
involve the integration of text, sound, graphics, motion video, the matches allowing you to easily locate a subject. Similar to a
and animation. card catalog, but not as efficient. Common search engines are
Multitasking:The concurrent execution of more than one Webcrawler, Yahoo, Alta Vista, Infoseek, and Lycos.
program at a time. Server: A computer or its software that "serves" other
Offline: Pertaining to data that are not accessible by, or computers by administering network files and network operations.
hardware devices that are not connected to a networked Three types of Internet servers are Web servers, e-mail servers,
computer system. and Gopher servers.
Online: Pertaining to data and/or hardware devices Surfing: The random, atmless exploration of web pages
accessible to and under the control of a networked computer acluved through following links that look interesting within a
system. document
Operating Systems or Platform: these terms refer to the Software: Software is set of instructions developed by
software that your computer uses to operate (otherwise known programming language which tells a computer what to do.
as your OS) and not to a manufacturer or company. Windows System software : controls the overall operation of a
2000, Windows XP, and OSX (Mac) are common platforms. computer. Some of the activities include managing system memory,
Online: Connected. You are online if you are working on controlling system resources, executing computer hardware
your computer while it is connected to another computer. Your functions and interfacing a user with computer hardware and
printer is online if it is connected to your computer and ready to applications.
accept data. Unix: UNIX is a family of OSes, each being made by a different
Password:Password is a series of characters used to protect company or organization but all offering a very similar look and
resources in a computer from unauthorized access. It is one of the feel.
Upload: The process of transferring information from one
ways to secure computer information from unauthorized users.
computer to another, generally from a client to a server. For example,
Peripherals:A physical device (such as a printer, scanner,
you upload a file from your computer to another.
or disk subsystem) that is externally attached to a workstation or
to the network. USB: Acronym "Universal Serial Bus" (the plug is very flant
Plugin: A helper application that works within a browser. It and has no pins or pronga). This is a style of port connection that
adds more functionality to a browser commonly associated with is used by many peripheral devices such as Palm Pilots, phones,
scanners, printers etc. This type of connection is much faster
the Netscape Navigator browser software.
than more traditional kind of connections such as serial and
Personal Computer:A small computer designed to use by parallel ports. Often used by older printers, these port have plugs
an individual, A microcomputer. with little screws attached).
Procesor: The logical component of a computer system that URL: Acronym "Uniform Resource Locator" The specific
interprets and executes program instructions. path to a World Wide Web file, including filename and extension.
Program:(1) Computer instructions structured and ordered UPS: "Uninterruptible Power Supply" An uninterruptible
in a manner that, when executed, causes a computer to perform a power supply (UPS) is a device that allows your computer to keep
particular function. (2) The act of producing computer software running for at least a short time when the primary power source is
to perform some application. lost.
Programming: The act of writing a computer program. Virus: A virus is a program that will seek to duplicate itself
Programming language: A language programmers use to in memory and on disks, but in a subtle way that will not
communicate instructions to a computer. immediately be noticed. A computer on the same network as an
RAM: Acronym "Random Access Memory" Random Access infected computer or that uses an infected disk (even a floppy) or
memory. the computer's "short term" memory used whenever an that downloads and runs an infected program can itself become
action is performed by a program. It is also called the "active infected.
memory". RAM is what the computer used to run all applications. WAN: Acronym "Wide Area Network" A larger computer
RAM is usually specified in Megabites or MB. (The other kind of network that is geographically dispersed, such as one that
memory dealers refer to is "storage" memory or hard drive size. it stretched across a university campus.
usually is specified in Giga bytes or GB.) Web Page: A single screen (document) on a Web site.
D-8 COMPUTER KNOWLEDGE
Webcasting: "Webcasting" is a term that describes the ability • Website are files in servers, which are powerful computers.
to use the Web to deliver or delayed versions of sound or video • Website contain pages to be known as Web Pages.
broadcasts. • The collection of all websites is known as World Wide Web
Website: The location of published hypertext content (WWW).
Physically, a Website can occupy an entire Web server or a part • Ted Nelson, in 1960s, first coined the term 'Hyper Text'
of a server; or it can be spread out among different servers as • Html is a markup language.
long as its sections are all linked, directly, to the same home page. • HTML is used to create Web Pages. It uses commands to be
WLAN: Acronym "Wireless Local Area Network" in a known as Tags.
wireless local area network (WLAN), an access point, is a station • To compose HTML documents, text editors are used. HTML
that transmits and receives data sometimes refered to as a documents are viewed in Web browser.
transceiver). • The tools are grouped by type in the Photoshop toolbox.
World Wide Web or WWW:This is the part of the Internet • Some of the tool icons have a tiny black triangle in the lower-
that you are accessing right now. The World Wide Web is so right corner of their icons. This means that there are more
named because each page in the WWW has links to other pages, tools of the same general kind available on a pop up-menu.
which have links to other pages, and so on, creating what could What is a computer
visually be seen as a web-like network of links.
Points to remember 20th century saw rapid developments in science and
technology influencing every aspect of human life. This period
A computer is a data processing machine having two main parts: witnessed one of the greatest achievements for human who
Hardware and Software. created 'the Computer'. A computer is an electronic and
• Hardware comprises of the physical units of a computer programmable device that receives input, stores, manipulates and
system while software is a set of programs. processes data to provide output in a useful format. It is used to
• Hardware and software together make a computer system solve problems relating to almost all fields such as administration,
functional. defence, education, home, medicine, science and technology,
• Data are raw facts and figures. research, designing, accounts, publishing etc. No doubts, in recent
• An operating system is an interface between the user and years, Computer and information Technology (I T), has became
the computer hardware and it manages computer resources. an integral part of human life. As computer is an information-
• An operating system performs different functions and is processing and information-accessing tool ,it accepts some
responsible for process management, file management, etc. information / data from the outside world, processes those data
• There are many kind of operating system. Some popular and produce a new information/data. Hence, information
names are:- DOS, UNIX, Windows, Linux, Mac OS, etc. processing is the essence of computing.
• The Windows Explorer program is more efficient for viewing Meaning of the word 'Computer': The word computer has
folders in Windows. its origin from an English word 'Computer', which means 'to
• Windows Explorer is divided vertically into two parts of two calculate'.
panes. The left side pane displays disk drives and folders in Program: Computer is an electronic machine which
a hierarchical order.while the right pane displays the content processes the input information/data as per the given set of
of the folder / drive that is selected on left side pane. instructions that is called program.
• The process of linking text values in a series within a formula
Characteristics of Computer
is called 'concatenation'.
• A computer is a data processing machine.Data processing Computer has become an essential part of our day-to-day activities.
involves some activities like data capturing, data Computers are used more or less in every sphere of life. Its growing
manipulation and information management. importance, is because of its unique features.
• Collection of interrelated data is called a database. v Accuracy : Computers are very accurate. They do make
• Computers are very useful for maintaining databases. mistakes but seldom. This is because of their physical
• A relational database is a collection of data items arranged circuit. Even if they make mistakes, It might be because of
as a set of formally described tables from which data can be
the faulty programs, some mistake made while feeding in
accessed or reassembled in many different ways without
the data or poorly designed system. The highly efficient
having reorganize the database tables.
error detecting techniques of the computer prevent its from
• MS Access is a powerful program to create and manage our
showing false results.
databases.
• Collection of data about a specific topic is called a table. v Speed : The computer was initially invented as a very high
• A form is a graphical representation of a table. speed calculator. This helped in completing many scientific
• A report is a Presentation of data in a printed form. projects that were previously impossible. The landing on
• We can create mailing labels for your database using MS the moon would not have been possible if computer had
not been there, neither would we take an umbrella if we
Access.
saw clear sky and weather forecast told us that it would
• Internet is the network of computer networks with million of
rain in the afternoon. We would have taken a lot of time in
computers attached to it.
making the arrangements for flying abroad if computers
COMPUTER KNOWLEDGE D-9
were not there to book our seats so easily and fast. This v 1642 A.D– ADDING MACHINE– BLAISE
ability to get the answers fast enough so that one has time PASCAL– PRANCE
to take an action on them (to make alternative arrangements The well known French Scientist and Mathematician, Blaise
in case of reservations) makes real- time- computing pascal invented the first machine which could add, carry
possible. Electrical pulses, so its speed is virtually digits automatically. he was only nineteen years old at that
instantaneous. When talking about speed of the computer, time. His machine was so revolutionary that the principle
we don't talk in seconds or microseconds but in behind it is still used in most of the machanical counters
nanoseconds (10–9 seconds) or even picoseconds (10–12 being used today.
seconds). v 1692 A.D.– MULTIPLYING MACHINE- COTTFRIED
v Versatility : This means that the computers are capable of LEIBNITZ- GERMANY
performing any type of task, provided the activity could Gottfried improved upon Pascal's machine and introduced a
be put into logial steps. It can be used from cooking mechanism to carry out automatic multiplication of numbers.
(microwave oven) to spending a night on the moon
Leibnita is best known for his work with Sir Isaac Newton in
(through satellities). In today's world it is difficult to imagine
developing a branch of Mathematics, known as Calculus.
even a single field which is untouched by computer
invation. The calculator in vented by him could add, subtract, multiply
v Storage : A human mind acquires some knowiedge and and divide accurately. It could even perform square root
affter it has used, it might keep it in its subconscious mind function, although not always accurately.
or might even forget it after some time. But computers can v 1813 A.D. – DIFFERENCE ENGINE– CHARLES
store massive amounts of information. This information BABBAGE– ENGLAND
can be used and reused time and again for years (unless Since early 19th century, Charles Babbage, an Englishman,
something goes wrong with the hardware). Today's
had been working on the development of a machine, which
computers have the disks. which have the capacity of
storing billions of characters. This is big enough to store could perform complex calculations In 1813 A.D. he invented
the complete Britanica Theasaurus, dozens of computer the 'Difference Engine' which could perform complex
programs or the applications, thousands of songs, huge calculations and print them out as well. This machine was a
databases, all the projects we have ever done in our life steam powered machine.
and much more. v EARLY 1800'S JACQUARD LOOM- JOSEPH MARIE
v Memory : Sometimes if we try to recall what we studied JACQUARD
last year, we are not able to recollect. In case of computer, In the early nineteenth century, a French weaver Joseph
it's not like that. If we store any information in the computer's
marie Jacquard developed a programmable loom, which used
memory, it remains there till we do not delete it. Moreover,
the memory of computer is unlimited, we can store as long large cards and holes punched in them to control the pattern
as we want. automatically. The output was a thick rich cloth with
v Automation : A computer is much more than just a calculator repetitive floral or geometric patterns.
in which we need to give the instructions at every step. It Jacquard patterns are still produced to this day. Others
is an intelligent device and if programmed for an activity, it adapted the punched cards and used as the primary form of
keeps doing it till it finishes, without any human input. They were used till about 20 - 25 years ago.
intervention.
v Diligence: Computer being a machine, does not show any Computer Generations
signs of fatigue, tiredness, lack of concentration, or lost
interest. The speed, accuracy and the quality would be In recent years, the computer industry has grown at a phenomenal
absolutely same in the first and the last calculation, even if pace. In a short time of 35 years or so computers have improved
millions of calculations are done by computer. It will not tremendously. In the last decade the speed of computer has
complain even once that they are bored. Thus, it is best increased 200 times, Not only that the reliability curve has also
specially for monotonous and voluminous work. Although taken a sharp increase. The cost per unit of calculating has gone
that is a threat for the people who are working on the same
down by 500 times. The storage capacity is increasing so fast that
kind of jobs.
v Reliability : Above all qualities of the computer make now it seems that nothing is impossible to store. Large data can
them reliable and also make us too dependent on them. be stored in very small devices.
They can be run for years and years without any loss of The term "generations" was initially introduced to distinguish
data or any other problem. between different hardware technologies. Gradually it shifted to
both hardware and software as the total systems consists of both
Development of Computer of them. The computers can be divided in five past generations,
v 1600 A.D.– NAPIER BONES: Another counting device is i.e., depending upon the technologies used. The five generations
Napier Bones, "John Napier. a Scottish Mathematician, of computer are:
invented it. The "bones" were strips of ivory with numbers Ist GENERATION (1942-1955)
written in them. When the bones were arranged properly. v Until 1951, electronics computers were the exclusive
the user could read the numbers in adjacent columns to get possession of scientists and the military. Till then nobody
the answer of a multiplication operation. tried to use them for business purpose. The idea of marketing
D-10 COMPUTER KNOWLEDGE
them was conceived by Mauchy and Eckert, creators of v Computers of this generation has the following
ENIAC's. As US census bureau was already using IBCP characteristics:
cards, they were the pioneers in buying this computer for 1. Smaller in size as compared to second generation
the first time in 1951. The company created by M and ETS computers.
became UNIVAC division of Sperry and Corporation. 2. High capacity internal storage.
v The bringing of first UNIVAC (Universal automatic 3. Remote communication facilities.
computers) general purpose electric digital computer, marks 4. Multiprogramming facilities
the beginning of the first generation of electronic computers. 5. Wide range of optional peripherals.
These computers used valves and all the components were 4th GENERATION (1975-1989)
joined by copper wires. Due to large size of the components v The 1970's marked the beginning of a new generation of
and due to the facts that the components had to be spaced computers. the development of microprocessor chip which
apart as the valves dissipated a lot of heat, the computers contains an entire central processing unit (CPU) on a single
were very bulky and required huge electric power, silicon chip led to the mushroom growth of expensive
airconditioners, maintenance and space for their installation. computers. They were not computers by themselves but
v Computer belonging to this generation had the following they can perform all the functions of arthimatic logic unit
characteristics: and control units of the CPU, memory and input ouput
1. Comparatively large in size as compared to present day devices, they become microcomputers. The semiconductor
computers. memores were also very small and very cheap. There were
2. Generated lot of heat, they were not consistent and reliable several types of memory chips. Three of the most commonly
as the valves tended to fail frequently. used are (a) Random Access Memory (RAM) in which data
3. Low capacity internal storage. can be read or written corresponding to the main memory of
4. Individual, non- related models. the conventinal computer. (b) Read Only Memory (ROM)
5. Processors operated in the milliseconds speed range. and (c) Programmable Read Only Memory (PROM).
6. Internal storage consisted of magnetic drum and relay 5th GENERATION (1989-Present)
lines. v Till fourth generation of computers, the major stress was on
2nd GENERATION (1955-1964) improving the hardware from valves to transistors and then
v FGC were very unreliable, mainly because of vacuum tubes to integrated circuits, which resulted in miniaturization and
which kept on burning out. Users had to be prepared all the fast speed of computers. However, the lack of thinking power
time with dozen of extra tubes to replace them. The computers has forced the scientists to work further for Fifth generation
of this generation were charaterized by the use of solid state computers. The concept of "Artificial Intelligence" is being
devices (transistors) instead of vacuum tubes. used in these computers and Japanese call them "Knowledge
Transistorizsed circuits were smaller, generated little heat, Processors".
were less expensive and consumed less power than vacuum v The fifith generation has three functional requirements.
tube circuits and were much greater in porcessing capacity. 1. Easy to use computers with high intelligence and
v Computers of this generation had the following natural human input and output mechanism.
charateristics. 2. Reliable and efficient software development by new
1. Smaller in size compared to the first generation languages, new computer architectures and systems
computers. software which overcome previous problems
2. Generated a lower level of heat, as components were 3. Improved overall functions and performance aimed
much smaller. at making computers smaller, lighter, faster of greater
3. Greater degree of reliability because of solid state capacity. more flexible and more reliable.
technology. These are the objectives which set the main themes for the
4. Higher capcity of internal storage. future of computing. whatever techniques are used to
5. Use of core storage instead of magnetic drum and relay achieve them.
lines. Classification of Computers
v Initially computers were clasified on the basis of their size,
3rd GENERATION (1964-1975) speed and cost but now there are many more attributes
v A revolution in the computer development took place with attached to them.
the development of integreated circuits (IC) on a single v Each and every computer must fall in one of the four
silicon chip. In 1958, Jack St. Clair Kelby and Robert Noyce categories described below:
invented the first IC. IC incorporated number of transistors 1. Supercomputer
and electronic circuits on a single wafer or chip of silicon. IC 2. Mainframe computer
was called chip because of the way they were made. 3. Mini computer
One more technology development which took place was 4. Micro computer
the launching of first telecommunication satellite. The SUPER COMPUTERS : Consider the application where a
communication stations on the earth were now in a position space shuttle or a satellite launcher has to be controlled. The
to send and receive data by means of satellite volume of data involved in these applications is enormous. But,
communications between the computer systems around the the analysis of this data is to be done in fractions of a second. To
world.
COMPUTER KNOWLEDGE D-11
achieve their performance, some special computers have been 4. Joystick
built. They are known as Super Computers. In India these types 5. Light pen
of computers have been installed at weather forecasting 6. Scanner
departments and other leading research institutions. 7. Secondary storage devices such as floppy disks, magnetic
MAINFRAME COMPUTER: The mainframes are bigger in tapes etc.
size, have vast amount of memory and can handle larger than The data in any form is first digitized i.e., converted into
minicomputers.Apart from that their input/ output operations are binary form, by the input device before being fed to the
very fast. For these reasons. the mainframes are generally used in Central Processing unit (CPU).
environments where lot of data has to be processed like banks THE OUTPUT UNIT : Like the Input Unit, the Output Unit
and research Institutions. also provides an interface between the user and the machine. A
MINI COMPUTER: The Mini computers are slightly bigger common example is the visual display unit (monitor) of a personal
in size, memory and speed when compared to Micro computers. computer. The output unit receives the data from the CPU in the
The major differences is that the Minicomputer are multi user form of binary bits. This is then converted into a desired form
systems. A multi user system means, more than one user can work (graphical, audio, visual etc.) understandable by the user. Some
in the computer systems at the same time. These computers are common output devices. are:
generally used for scientific purposes and for small business (i) Visual Display Unit (Monitor)
establishments. (ii) Printers
MICROCOMPUTERS : The Microcomputers are mainly (iii) Speakers
used for personal use like word processing. Apart from that, these The input and output unit collectively are referred to as
computer systems are generally used in those applications where 'peripherals'
there are lot of interaction between the computer and the user is THE CENTRAL PROCESSING UNIT : The Central
required. Normally, only one user will be able to work on this processing unit is the brain of the computer system. The input
system at a given time, which is called single user system. (The 16 and output devices may vary for different application, but there is
wire bus) microcomputer, is often called the super- micro, and the only one CPU for a particular computer. The specifications of a
32 wire bus microcomputer is often called as megamicro. But these computer are basically characterized by its Central Processing
computers have become common and these are generally known Unit.
as Micro- Computers). The central processing unit can be further divided into:
1. The Arithmetic Logic Unit (ALU)
Computer : The Definition 2. The Control Unit
"Computer is an electronic device for performing arithmetic 3. Main Memory
and logical operations", or "Computer is a device or flexible The CPU performs many tests, some of which are listed below:
machine to process data and convert it into information 1. The CPU can perform arithmetic calculations such as
The Computer Hardware (actual machine) is defined in such addition, subtraction etc.
a way that it does whatever the Software (computer programs) 2. The CPU can perform logical decisions.
tells it to do. 3. The CPU with the help of other devices can per form data
There are four basic operations which a computer performs transmission.
irrespective of the program which is running on it. They are 4. The CPU can perform manipulating tasks such as word
classified as: processing.
1. INPUT: This is for the purpose of inserting or feeding data 5. After performing the required task the CPU may place results
into the computer by means of an input device like keyboard.
in memory of send results to the output device according to
2. PROCESSING : Some kind of processing is done in the
the instruction given to it.
computer to take out or transform the data in some way.
3. OUTPUT: The computer produces output on a device, such 6. The CPU with the help of its control unit generates timing
as printer, scanner or a monitor, that shows the result of signals (also known as enable signals) which provide
processing operations. synchronization between the different devices and the CPU.
4. STORAGE: The computer stores the result of processing As mentioned earlier, the central processing unit
operations for future use in some storage device like hard consists of:
disk, compact disk drive (CD Rom) or a floppy disk. 1. The Arithmetic Logic Unit (ALU)
THE INPUT UNIT : The Input Unit provides an interface 2. The Control Unit
between the users and the machine, for inputing data and 3. The Main Memory Unit
instruction etc. One of the most common examples is the keyboard. THE ARITHMETIC LOGIC UNIT (ALU) : As the name may
Data can be input in many more forms- audio. visual, graphical indicate the arithmetic logic unit performs all arithmetic and logic
etc. calculations on the data it receives.
Some common input devices are listed below: ARITHMETIC CALCULATIONS: The arithmetic
1. Keyboard calculations may be addition, subtraction, multiplication, division,
2. Mouse exponentiation etc.
3. Voice data entry THE MAIN MEMORY UNIT : The main memory also known
D-12 COMPUTER KNOWLEDGE
as the primary memory is a part of the central processing unit and v USB: Most cable today are USB. USB cables can have a
is a combination of both RAM (random access memory) and couple of different connectors. The one given below is the
ROM (read only memory). most common type. You will find USB ports in the back or
RAM : The random access memory is read write memory i.e. side of your computer. also sometimes on the monitor and
information can be read as well as written into this type of memory. keyboard.
It is voltile in nature. i.e., the information it contains is lost as v Firewire (IEEE) Firewire is a high speed cable used primarily
soon as the system is shut down unless 'saved' for further usage to transfer video from a digital video camera to a computer
by users. It is basically used to store programs and data during and vice versa.
the computer's operation. v CPU: The CPU, or Central Processing Unit is the actual
ROM :The read only memory as the name may suggest computer or brains of the computer. This is where the
contains information that can only be read, i.e., you can't write on "computing" takes place.
this type of memory. It is non-volatile or permanent in nature. It is v Hard Drive: All computers have a hard drive. The hard drive
basically used to store permanent programs such as program for (HD) is where everything is saved. Every program, every file
the functioning of the monitor. or document is saved on the hard drive.
1. Access Mode: which means how easily they are accessible.
v RAM (memory): When we talk about the memory of a
2. Access time: the average time required to reach a storage
computer, we are talking about RAM, or Ramdon Access
location and obtain its content is called access time.
Memory. RAM is your temporary working space, This is
3. Transfer Rate: the transfer rate is the number of characters
where you do your work. but it is temporary. If you don't
or words that a device can transfer per second after it has
save your work back to your hard drive, you will lose your
been positioned at the beginning of the record.
work, (this is why when your computer freezes and have to
4. Capacity and cost: the capacity and cost may depend upon
restart, you lose your work)
the requirement and the budget.
The main memory has a very low access time and a very v Bytes: We us Bytes as unit of measurement used to indicate
high transfer rate. It is limited in capacity and costlier than how much memory information will take up. Bytes is the root
secondary storge devices. word, prefixes are added to it to change it's value:
COMPUTER HARDWARE : The term computer hardware (K) kilo = 1000
refers to the various electronic components that are required for (M) mega = million
you to use a computer along with the hardware components inside
(G) giga = billion
the computer case. Computer equipment is made of several
common components. These include: 1 KB = 1,000 bytes (approx.)
• The main computer box. 1MB = 1,000, 000 (1 million) bytes = 1000 kb (approx.)
• A monitor- Looks like a television screen. 1 GB = 1, 000, 000, 000 (1 billion) bytes = 1, 000 MB (approx.)
• A keyboard. Every letter or number you type takes up 1 byte of space.
• A mouse.
v CD ROM: stands for Compact Disk-Read Only Memory,
• Speakers
which means you can only read what is on the CD, you can't
• An optional printer
THE MAIN COMPUTER BOX : The main computer box is change or delete it. When something is burned on a disk. it's
made of several computer hardware components and more or less permanent.
subcomponents which include: v Floppy Disks : Floppy disk have pretty much become
The case: The outside component which provides protection obsolete or out dated. Most people do not use them anymore
for the parts inside and provids a fan and power supply which are because they do not hold very much information (I megabyte)
used to both cool the working parts inside and provide power to and are not very reliable. There are much better options for
them. such as these other type of media listed here.
Input devices: Input devices are pieces of equipment that
Software is a general term which is used to describe the
are used to get information INTO a computer. Such as.
instructions that are given to computer. These instructions can
• keyboard
• mouse be either a single programme or a group of programmes.
• microphone
Types of Software
• joystick
• scanner Software is generally classified into theree specific categories in
• digital camera (still or video) the computer world:
• thumb drive 1. System software
Output devices: Output devices are pieces of equipment 2. Application software
that are used to get information OUT of a computer. Such as: 3. Utility software
• monitor
• printer 1. System software: This consists of all the programmes,
• speakers languages and documentation supplied by the manufacturer
• projector of the computer. This type of software is required to use the
computer efficiently and conveniently. These programmes
COMPUTER KNOWLEDGE D-13
allow the application developer to write and develop their 3. Input output – Read and write
own programmes. 4. Direct use – Halt, start and end
2. Application software: These programmes are developed by No arithmatic or comparison operations are done in the
the user in order to perform some specific function for the primary memory of the computer. Instead it is done in the ALU's
organisation. For example, a payroll system to compute the special register called accumulator. Thus if we need to add two
salaries of the employees of an organisation is termed as an numbers, we require one instruction which will order the control
application software. unit to place a nuber in the accumulator and another instruction
3. Utility software: Utility software may be considered as an to identify the operation of addition.
application software or a system software which is very Symbolic/ Assembly Languages: In order to reduce the
often used in the development of a programme. burden, symbolic languages, commonly known as assembly
PROGRAMMING LANGUAGES : A programming language languages was developed in 1950's for the second generation
consists of words, symbols and usage rules pertaining to the computers.
grammar that permits people to communicate with the computer. This language permits the use of symbols or mnemonics
permits people to communicate with the computer. Understanding which are two or three letter abbreviations for the function to be
of computer software is imperfect with out a basic knowlede of performed by the instruction. These are then translated by using
programming languaes, Programming languages allow the symbolic equivalence table. to control registers etc. However, the
programmers and end users to develop the programmes that are disadvantage of using binary has been removed.
exceuted by users to develop the programmes that are executed Functions of Assembler
by the computer. Many programming languages exist in the world (i) The Assembler translates the function code into its machine
today. Each one of the languages have their own unique code equivalent.
vocabulary, grammar and usage. Some of these languages have (ii) It assigns absolute addresses to any symbolic address or
been created to serve a special purpose while others are more label names.
flexible and general purpose and are suitable for many types of (iii) It places each instruction in central memory.
applications. However ingeneral, programming languages must (iv) It identifies indirect addresses from direct addresses and
cater to the following tasks: sets the appropriate bit in the address portion of the
– input/ output instruction.
– text manipulations/ calculations (v) It checks the syntax of each instruction and generates error
– logic/ comparison messages.
– storage/ retrieval (vi) It provides, optionally, a cross reference table between all
symbolic names and their absolute addresses.
CLASSIFICATION OF PROGRAMMING LANGUAGE
(vii) It informs the control unit to exceute the program after all
Machine Languages: Machine language is the lowest form errors have been corrected.
of computer language. Programmes were only written in binary Advantages of Assembly languages
based machine level language in the first generation computers.
(i) They save time and reduce detail as compared to machie
The computer understands this language only at its lowest level.
language.
An instruction perpared in machine language has two parts: (ii) Lesser number of errors are made and errors are easier to
1. Op-code: This is the first part and is the command or operation detect.
and it tells the computer what function to perform. (iii) Assembly programs are easier to modify than machine
2. Operand: The second part of the instruction is the operand language programs
and it tells the computer where to find or store the data or Disadvantages of Assembly Language
instructions that are to be manipulated. The number of (i) Writing a code is time consuming.
operands in an instrution varies from computer to computer. (ii) Assembly languages are machine dependent.
In a single operand machine, the binary equivalent of
"ADD0481" could cause the value in a storage location High Level Languages
0481 to be added to a value stored in the arithmatic & logic The disadvantages of using assembly language brought
unit. The single operand format is popular in the smallest about the development of higher level languages. Unlike the
microcomputers whereas the two operand structure is found assembly programs, high level language programs may be used
in most other machines. with little modification. High level languages are easier to learn
The set of instructions in a machine level language can be divided than symbolic languages. They require less time to write, are easier
into four categories: to maintain, provide better documentation and 4 or 5 low- level in
1. Arithmatic – add, subtract, multiply and divide structions are reduced to a single high level statement. Some of
2. Controlled– load, store, jump instructions the popular high level languages are given in the table below.
D-14 COMPUTER KNOWLEDGE
later was much better with many errors resolved.
Language Meaning
v Windows NT: A version of Windows made specifically for
Scientific &
FORTRAN Formula Translator Engineering businesses offering better control over work station
Common Business capabilities to help network administrators.
COBOL Oriented Language Commercial v Windows 95: The first version of Windows after the older
ALGOL Algorithmic Language Scientific Windows 3.x, versions offering a better in terface and better
library functions for programs.
RPG Report Generator Commerical
There are other worthwhile types of operating systems not
A Programming
APL The Sharing System made by Microsoft. The greatest problem with these
Language
operating systems lies in the fact that not as many
PL/1 Programming Language The Sharing System application programs are written for them. However if you
Beginners All Symbolic can get the type of application programs you are looking
BASIC Teaching
Instruction Code for, one of the systems listed below may be a good choice.
Named after the French v Unix: A system that has been around for many years and it
PASCAL Teaching
Philosopher is very stable. It is primarily used as a server rather than a
workstation and should not be used by anyone who does
The operating system not understand the system. It can be difficult to learn, Unix
must normally run on a computer made by the same company
The operating system is the core software component of
that produces the software.
the computer. It performs many functions and is in very basic
v Linux: Linux is similar to Unix in operation but it is free. It
terms an interface between your computer and the outside world.
also should not be used by anyone who does not understand
In the section about hard ware, a computer is described as
the system and can be difficult to learn.
consisting of several components including your monitor,
v Apple Macintosh- Most recent versions are based on Unix
keyboard, mouse and other parts. The operating system provides but it has a good graphical interface so it is both stable
an interface to these parts using what is referred to as "drivers". (does not crash often or have as many to learn). One
This is why sometimes when you install a new printer or other drawback to this system is that it can only be run on Apple
piece of hardware, you system will ask you to install more software produced hardware.
called a driver. v Windows XP: An operating system, sometimes called an
Operating System Types "OS", is the main program the computer used to function
properly. Operating systems act as a link between you, the
There are many types of operating systems. The most common is
user, and the programs you use on a computer. Different
the Microsoft's operating systems. They include from most recent
types of computers use different types of operating systems.
to the oldest:
The majority of computers used either run Microsoft
v Windows XP Professional Edition: A version used by many
Windows or MacOS. While files can be shared between
businesses at workstations. It has the ability to become a
these two types of systems. they are generally incompatible.
member of a corporate domain.
v Desktop: The desktop is the area you see when the computer
v Windows XP Home Edition: A lower cost version of Windows
is not running applications. It consists of the icons on top
XP which is for home use only and should not be used in a
of it. as well as the Start bution and other features. The
business.
desktop can be used to temporary store information or to
v Windows 2000: A better version of the Windows NT move around documents and windows.
operating system which works well both at home and as a
v Icon: Icons are little pictures that represent different programs
workstation in a business. It includes technologies which
or saved items. Double- clicking on the Icon accesses the
allow hardware to be automatically detected and other
information represent.
enhancements over Windows NT.
v Window: Each application opened will appear in its own
v Windows 2000: A better version of the Windows NT
window. or its own little section of the screen. Windows can
operating system which works well both at home and as a
be moved and resized so that you can operate many different
workstation at a business. It includes technologies which
applications at the same time. To learn how to manipulate
allow hardware to be automatically detected and other
windows, refer to the Manipulating Windows portion of
enhancements over Windows NT.
Level.
v Windows ME: An upgraded version from windows 98 but it
v Dialogue Box: When you ask the computer to act on certain
has been historically plagued with programming errors
commands, as to save your work, the computer will need
which may be frustrating for home users.
more information from you. and this will appear in a dialogue
v Windows 98: This was produced in two main versions. The box. These boxes contain options and commands for
first Windows 98 version was plagued with programming computers to execute.
errors but the Windows 98 Second Edition which came out
COMPUTER KNOWLEDGE D-15
v Start Menu: In the lower left- hand corner of the Windows Netscape Navigator and Microsoft Internet Explorer are two
screen is the Start button. When you click on the button, a of the most commonly used web browsers.
menu will appear, which we will call the Start menu. This v Web Site: contains all the information offered by a particular
menu gives you access to all the different parts and functions organization, individual, or company and will sometimes
of the computer. include links to other sites as well. Each web site generally
v Task Bar: At the very bottom of the screen is a horizontal starts with a home page and then links to other pages within
bar called the task bar. This bar contains (From left to right) the site containing various types of information and/ or
the Start button, shortcuts to various programs, minimized services/ products. A website can be made up of a single
programs, and another section of shortcuts that includes webpage document or hundreds/ thousands, limited only
sound, volume, printers and the time. by the size restrictions of the webserver it is housed in.
v The Internet: the world Wide Web, or the Web are all names v Menu Bar: Contains menu items that open up dropdown
used to describe the vast network of information in lists for related options. Among the items are options for
cyberspace, available to anyone who has access to a printing, customizing IE, copying and pasting text, managing
computer, a browser (software), and a connection to an Favourites, and accessing Help.
Internet service provider through a modem (or other v Navigation Toolbar: Contains icons for a variety of features
connection such as DSL, ISDN, LAN etc). Many people including navigating among Web pages. searching the Web
use the terms Internet and World Wide Web (a. k. a. the using a selection of search tools, accessing and managing
Web) Interchangeably, but in fact the two terms are not Favourites, viewing a History of visited pages, printing,
synonymous. The Internet and the Web are two separate and accessing email and newsgroups.
but related things. v Address Bar: This is not really a toolbar, but this is where
The Internet is a massive network of networks, i.e. a you type in the URL. (Web address) of the page. When you
networking infrastructure. It connects millions of computers press the Enter key. it will take you to this address.
together globally, forming a network in which any computer v Home: The home icon takes you back to the page that was
can communicate with any other computer as long as they on the screen when you first started IE, You can customize
both are connected to the Internet. your seletion.
v BLOG: A Blog, short for weblog, is usually a personal, time v Search: The search button opens up a function that uses
stamped, online journal that apperars on a website. It can be one or more Web search tools. You can choose the search
periodically updated by the owner. sometimes called a tool(s) you want as default.
blogger. Many sites of fer free software to create blogs on v History: The history function allows you to view and select
personal websites. Web pages you have recently visited. You can sort your
v Downloading: is when you take a file from some one else's items by clicking on the black triangle to the right of the
computer and put it on your own. A file can be anything word View. You can sort by size, date the number of times
from a pretty picture to the enitre text of the Declaration of visited, and the order you have visited today.
Independence. v Mail: you can read email from this window. Choose the
v E- mail: is the more common abbreviation for Electronic email software you wish to use by going back to the Menu
mail. It allows computer users locally and worldwide to Bar and choosing Tools/ Internet Options/ Programs.
exchange messages. Each user of e-mail has a mailbox
address to which messages are sent Toolbar Options for the Menu Bar
v Internet Service Provider (ISP): It connects you to the The menu bar at the top of the screen includes some useful
Internet. options. Here are a few highlights.
v Search Engines/ Search Directories: a search engine is a v File/ New/ Window: You can open up a second copy of IE
searchable database of Internet files collected by a computer by using this feature. This allows you to visit more than one
program (this program is sometimes called a wanderer, Web page at a time.
crawler, robot, worm, spider, etc) An index is created for the v File/ Edit with ......: You can edit the current Web page using
collected files. e,g., title, full text, size, URL, etc. There is the editor of your choice. Select the editor by going back to
often no selection criteria for these collection of files, except the Menu bar and choosing Tools/ Internet Options/
a ranking of "best fit" results. Programs. Your choices will be determined by software
v URL: stands for Uniform Resource Locator. The URL installed on your computer.
specifies the internet address of a file stored on a host v Edit/ Find (on This Page): IE allows you to do a text search
computer connected to the Internet. Every file on the of the document on your screen. Choose this option and
Internet, no matter what its accessing protocol, has a unique type in the word or phrase you wish to search.
URL, Each web site must have its own specific address,
v Tools/ Show Related Links: IE will display pages that are
similar to the way, each home must have a unique address in
related in context to the current page. This is a Web content
order to receive mail delivery service.
and traffic analysis company. The Tools menu offers you
v Web Browser: a web browser is a software, installed on
many ways to customize IE.
your computer, that allows you to navigate. Internet
D-16 COMPUTER KNOWLEDGE

EXERCISE
1. What is called as the main folder on a storage device? 14. The smallest unit of information, a computer can understand
(a) Platform (b) Interface and process is known as a ............
(c) Root Directory (d) Home Page (a) digit (b) kilobyte
(e) None of the above (c) bit (d) byte
2. RAM is .............. and ............. . (e) None of the above
(a) volatile, temporary (b) nonvolatile, permanent 15. For creating a document, you use ........ command at File Menu.
(c) nonvolatile, temporary (d) volatile, permanent (a) Open (b) Close
(e) None of the above (c) New (d) Save
3. Which is not an item of hardware?
(e) None of the above
(a) An MP3 file (b) A keyboard
(c) A monitor (d) A mouse 16. Applications are often referred to as ............
(e) None of the above (a) Data files (b) executable files
4. The box that contains the central electronic components of (c) system software (d) the operating system
the computer is the ............ (e) None of the above
(a) motherboard (b) system unit 17. PC stands for
(c) peripheral (d) input device (a) Personal Comprehension
(e) None of the above (b) Personal Computing (c) Personal Computer
5. Which type of device is computer monitor? (d) Personal Calculations (e) None of the above
(a) Input (b) Output 18. A directory within a directory is called ............
(c) Processing (d) Software (a) Mini Directory (b) Junior Directory
(e) None of the above (c) Part Directory (d) Sub Directory
6. How many options does a binary choice offer? (e) None of the above
(a) One (b) Two 19. A(n) ................. is created by an application.
(c) Three
(a) executable file (b) software program
(d) It depends on the amount of memory in the computer
(e) None of the above (c) document (d) operating system
7. Which menu is selected to cut, copy, and paste? (e) None of the above
(a) File (b) Edit 20. Compatibility in regard to computers refers to ............
(c) Tools (d) Table (a) the software doing the right job for the user
(e) None of the above (b) it being versatile enough to handle the job
8. Storage device, inside the computer is (c) the software being able to run on the computer
(a) CDROM (b) Zip Disk (d) software running with other previously installed
(c) Super Disk (d) Hard Disk software
(e) None of the above (e) None of the above
9. The ................. indicates how much data a particular storage 21. What is a file?
medium can hold. (a) A file is a section of main storage used to store data
(a) access (b) capacity (b) A file is a collection of information that has been given
(c) memory (d) storage
a name and is stored in secondary memory
(e) None of the above
10. If you are going to a site you use often, instead of having to (c) A file is the part of a program that is used to describe
type in the address every time, you should ............ what the program should do
(a) save it as a file (b) make a copy of it (d) A file is another name for floppy disk
(c) bookmark it (d) delete it (e) None of the above
(e) None of the above 22. The ........... key and the ........... key can be used in combination
11. Which of these keys is not on the number keypad? with other keys to perform shortcuts and special tasks.
(a) Ctrl (b) Delete (a) Control, Alt (b) Function, toggle
(c) Enter (d) Num Lock (c) Delete, Insert (d) Caps Lock, Num Lock
(e) None of the above (e) None of the above
12. A program that converts a high-level language source file 23. How is it possible that both programs and data can be stored
into a machine-language file is called a ............ on the same floppy disk?
(a) translator (b) assembler (a) A floppy disk has two sides, one for data and one for
(c) compiler (d) linker programs
(e) None of the above
(b) Programs and data are both software, and both can be
13. A CD - ROM disk
(a) cannot be erased and rewritten stored on any memory device
(b) has more storage capacity than a CD-R (c) A floppy disk has to be formatted for one or for the
(c) holds less data than a floppy disk other
(d) can be written to only once (d) Floppy disks can only store data, not programs
(e) None of the above (e) None of the above
COMPUTER KNOWLEDGE D-17
24. The primary output device for computers is a ................. (c) a file compression utility is used
(a) video monitor (b) printer (d) the disk is scanned
(c) keyboard (d) mouse (e) None of the above
(e) None of the above 35. Output which is made up of pictures, sounds, and video is
25. The name of the location of a particular piece of data is its called .................
................. (a) COM (b) hard copy
(a) address (b) memory name (c) graphics (d) multimedia
(c) storage site (d) data location (e) None of the above
(e) None of the above 36. Several computers linked to a server to share programs and
26. Two different files can have the same name if ................. storage space .................
(a) they are in different folders (a) Network (b) grouping
(b) they are on different drives (c) library (d) integrated system
(c) they are on the same drive (e) None of the above
(d) they are in same folder 37. A prescribed set of well-defined instructions for solving
(e) both (a) and (b) mathematical problems is called .................
27. A device that is connected to the motherboard is ................. (a) a compiler (b) a code
(a) called an external device(b) called an adjunct device (c) a description (d) an algorithm
(c) called a peripheral device (e) None of the above
(d) must connect using ribbon cable 38. The process of preparing a floppy diskette for use is called
(e) None of the above .................
28. The first computers were programmed using ................. (a) assembling (b) translating
(a) assembly language (b) machine language (c) parsing (d) formatting
(c) spaghetti code (d) source code (e) None of the above
(e) None of the above 39. LAN stands for .................
(a) Local Access Network (b) Local Area Network
29. Documentation of computer programs is important so that
(c) Logical access network(d) Logical Area Network
.................
(e) None of the above
(a) users can learn how to use the program
40. A Field is a related group of .................
(b) other programmers can know how to maintain the
(a) Records (b) Files
program
(c) Characters (d) Cables
(c) the programmer can see why the code is written that (e) None of the above
way while hunting for source of error
41. Meaningful filename helps in easy file .................
(d) All of the above (a) Storing (b) Accessing
(e) None of the above (c) Identification (d) Printing
30. Provide the means to move the pointer on the screen and (e) None of the above
give information to the computer by clicking its buttons 42. To restart the computer ................. key is used.
................. (a) Del + Ctrl (b) Backspace + Ctrl
(a) scanner (b) mouse (c) Ctrl + Alt + Del (d) Reset
(c) keyboard (d) program (e) None of the above
(e) None of the above 43. Housing all hardware, software, storage, and processing in
31. When you cut or copy information it gets place in the one site location is called .................
................. (a) time-sharing (b) a distributed system
(a) Clipart (b) Clipboard (c) centralized processing (d) A host computer
(c) Internet (d) Motherboard (e) None of the above
(e) None of the above 44. A computer works on a ................. number system.
32. Secondary storage ................. (a) binary (b) octal
(a) does not require constant power (c) decimal (d) hexadecimal
(b) does not use magnetic media (e) None of the above
(c) consists of four main types of devices 45. A record is related to a file, as a statement is related to a
(d) does not store information for later retrieval .........
(e) None of the above (a) procedure (b) file
33. A device that provides emergency power to your computer, (c) program (d) data
conditions the voltage, and protects against powers surges (e) None of the above
is called a ......... 46. Soft copy refers to .................
(a) PSU = Power Supply Unit (a) printed output (b) music sounds
(b) USP = Universal Surge Protector (c) screen output (d) digitizing
(c) UPPS = Universal Power Protection and Supply (e) None of the above
(d) UPS = Uninterruptible Power Supply 47. A program that enables you to perform calculations
(e) None of the above involving rows and columns of numbers is called a .................
34. Deleted data remains on a disk until ................. (a) spreadsheet program (b) word processor
(a) the data is overwritten (c) graphics package (d) window
(b) the recycle bin is emptied (e) None of the above
D-18 COMPUTER KNOWLEDGE
48. WWW stands for ................. 60. The first page of a Web site is called the .......... .
(a) World Work Web (b) Wide Work Web (a) Homepage (b) Index
(c) Wide World Web (d) World Wide Web (c) Java Script (d) Book mark
(e) None of the above (e) None of those
49. The physical components of a computer system is ................. 61. A word in a web page that, when clicked, opens another
(a) Software (b) Hardware document ............. .
(c) ALU (d) Control Unit (a) anchor (b) URL
(e) None of the above (c) hyperlink (d) reference
50. Which is a graphical representation of an application? (e) None of these
(a) Windows 95 (b) Windows Explorer 62. The ............ manual tells you how to use a software program.
(c) Icon (d) Taskbar (a) documentation (b) programming
(e) None of the above (c) technical (d) user
51. OCR stands for ............. (e) None of these
(a) Optical Character Recognition 63. What disk is used to cold boot a PC?
(b) Optical CPU Recognition (a) Setup disk (b) System disk
(c) Optimal Character Rendering (c) Diagnostic disk (d) Program disk
(d) Other Character Restoration (e) None of these
(e) None of these 64. The ............. tells the computer how to use its components.
52. If a new device is attached to a computer, such as a printer (a) utility (b) network
or scanner, its............. must be installed before the device (c) operating system (d) application program
can be used. (e) None of these
(a) buffer (b) driver 65. A ............. contains buttons and menus that provide quick
(c) pager (d) server access to commonly used commands.
(e) None of these (a) menu bar (b) toolbar
(c) window (d) action bar
53. The software that allows users to surf the Internet is called
(e) None of these
a/ an ............. .
66. Numbers in table columns are usually ..............
(a) Search engine
(a) right-aligned (b) left-aligned
(b) Internet Service Provider (ISP)
(c) justified (d) centered
(c) Multimedia application
(e) None of these
(d) Browser 67. The ............. of software contains lists of commands and
(e) None of these
options.
54. A tuple is a ............. . (a) menu bar (b) title bar
(a) column of a table (b) two dimensional table (c) formula bar (d) tool bar
(c) row of a table (d) key of a table (e) None of these
(e) None of these 68. To access a mainframe or supercomputer, users often use a
55. The method of file organization in which data records in a ............. .
file are arranged in a specified order according to a key field (a) terminal (b) node
is known as the ............. (c) desktop (d) handheld
(a) Direct access method (b) Queuing method (e) None of these
(c) Predetermined method (d) Sequential access method 69. By default, your documents print in ............. mode.
(e) None of these (a) Landscape (b) Portrait
56. In Excel ............. contains one or more worksheets. (c) Page Setup (d) Print View
(a) Template (b) Workbook (e) None of these
(c) Active cell (d) Label 70. What characteristic of read-only memory (ROM) makes it
(e) None of these useful?
57. Which of the following is a popular programming language (a) ROM information can be easily updated
for developing multimedia web pages, websites, and web- (b) ROM provides very large amounts of inexpensive data
based applications? storage
(a) COBOL (b) Java (c) Data in ROM is nonvolatile, that is, it remains there
(c) BASIC (d) Assembler even without electrical power
(e) None of these (d) ROM chips are easily swapped between different
58. Compiling creates a (n) ........... brands of computers
(a) program specification (b) algorithm (e) None of these
(c) executable program (d) subroutine 71. What are bas, doc, and htm examples of?
(e) None of these (a) extensions (b) domains
59. A CD-RW disk ............. . (c) protocols (d) databases
(a) has a faster access than an internal disk (e) None of these
(b) is a form of optical disk, so it can only be written once 72. ctrl, shift and alt are called ............. keys.
(c) holds less data than a floppy disk (a) adjustment (b) function
(d) can be erased and rewritten (c) modifier (d) alphanumeric
(e) None of these (e) None of these
COMPUTER KNOWLEDGE D-19
73. Which type of file is created by word processing programs? (a) Keyboard (b) Mouse
(a) database file (b) storage file (c) joystick (d) Track ball
(c) worksheet file (d) document file (e) None of these
(e) graphical file 86. The simultaneous execution of two or more instructions is
74. Personal computers can be connected together to form a ............. called ............. .
(a) server (b) supercomputer (a) sequential access
(c) network (d) enterprise (b) reduced instruction set computing
(e) None of these (c) multiprocessing
75. A modem ............. . (d) disk mirroring
(a) translates analog signals from a computer into digital (e) None of these
signals that can travel along conventional telephone lines 87. Multiprogramming systems ............. .
(b) translates digital signals from a computer into analog (a) Are easier to develop than single programming systems
signals that can travel along conventional telephone lines . (b) Execute each job faster
(c) demodulates digital signals from a computer (c) Execute more jobs in the same time period
(d) modulates signals from an analog telephone line (d) Use only one large mainframe computer
(e) None of these (e) None of these
76. Which of the following menu types is also called a drop- 88. Which device can understand difference between data and
down menu? programs?
(a) fly-out (b) cascading (a) Input device (b) Output device
(c) pop-up (d) pull-down (c) Memory (d) Microprocessor
(e) None of these (e) None of these
77. Data (information) is stored in computer as ............. . 89. Which of the following devices have a limitation that we
(a) files (b) directories can only read it but cannot erase or modify it?
(c) floppies (d) matter (a) Tape Drive (b) Hard Disk
(e) None of these (c) Compact Disk (d) Floppy Disk
78. The central processing unit contains which of the following (e) None of these
as a component?
90. Which of the following is not an output device?
(a) Memory Regulation Unit
(a) Plotter (b) Printer
(b) Flow Control Unit
(c) Monitor (d) Touch Screen
(c) Arithmetic Logic Unit
(e) None of these
(d) Instruction Manipulation Unit
91. Data that is copied from an application is stored in the .............
(e) None of these
(a) driver (b) clipboard
79. Memory unit is one part of ............. .
(c) terminal (d) prompt
(a) Control unit (b) Central Processing Unit
(c) Input device (d) Output device (e) None of these
(e) None of these 92. Every component of your computer is either ............. .
80. The process of writing out computer instructions is known (a) software or CPU/RAM
as ............. . (b) input devices or output devices
(a) assembling (b) compiling (c) application software or system software
(c) executing (d) coding (d) hardware or software
(e) None of these (e) None of these
81. A Web site address is a unique name that identifies a specific 93. A collection of interrelated records is called a ............. .
............. on the Web. (a) management information system
(a) Web browser (b) PDA (b) spread sheet (c) database
(c) Web Site (d) link (d) text file (e) None of these
(e) None of these 94. Which of the following is the storage area within the
82. A character of information is represented by a(n) ............ . computer itself which holds data only temporarily as the
(a) byte (b) bit computer processes instructions ?
(c) field (d) attribute (a) The hard disk (b) main memory
(e) None of these (c) The control unit (d) read-only memory
83. An example of a telecommunications device is a ............. . (e) None of these
(a) keyboard (b) mouse 95. Codes consisting of bars or lines of varying widths or
(c) printer (d) modem lengths that are computer- readable are known as .............
(e) None of these (a) a bar code (b) an ASCII code
84. ............ is a procedure that requires users to enter an (c) a magnetic tape (d) a light pen
identification code and a matching password. (e) None of these
(a) Paging (b) Logging on 96. A ............. contains specific rules and words that express
(c) Time-sharing (d) Multitasking the logical steps of an algorithm.
(e) None of these (a) programming language (b) programming structure
85. Which device is used as the standard pointing device in a (c) syntax (d) logic chart
Graphical User Environment? (e) None of these
D-20 COMPUTER KNOWLEDGE
97. ............. is a set of keywords, Symbols and a system of rules (c) web page (d) web server
for constructing statements by which humans can communi- (e) None of these
cate the instructions to be executed by a computer. 108. ............ are a type of inexpensive digital camera that remains
(a) A computer program (b) A programming language tethered to a computer and used for videoconferencing,
(c) An assembler (d) Syntax video chatting, and live Web broadcast.
(e) None of these (a) Webcams (b) Webpics
98. The general term “peripheral equipment” is used for ............. (c) Browsercams (d) Browserpics
(a) any device that is attached to a computer system (e) None of these
(b) large-scale computer systems 109. Which one of the following is a key function of a firewall?
(c) a program collection (a) Monitoring (b) Deleting
(d) other office equipment not associated with a desktop (c) Copying (d) Moving
computer (e) None of these
(e) None of these 110. The standard protocol of the Internet is ............
99. If a memory chip is volatile, it will ............. .
(a) TCP/IP (b) Java
(a) explode if exposed to high temperatures
(c) HTML (d) Flash
(b) lose its contents if current it turned off
(e) None of these
(c) be used for data storage only
111. A program that generally has more user-friendly interface
(d) be used to both read and write data
(e) None of these than a DBMS is called a .............
100. ............. is the process of finding errors in software code. (a) front end (b) repository
(a) Compiling (b) Assembling (c) back end (d) form
(c) Interpreting (d) Debugging (e) None of these
(e) None of these 112. The acronym HTML stands for ............
101. Which of the following converts all the statements in a (a) High Transfer Machine Language
program in a single batch and the resulting collection of (b) High Transmission Markup Language
instructions is placed in a new file? (c) Hypertext Markup Language
(a) compiler (b) interpreter (d) Hypermedia Markup Language
(c) converter (d) instruction (e) None of these
(e) None of these 113. Computers connected to LAN can ............
102. Digital photos and scanned images are typically stored as (a) run faster
............ graphics with extensions such as bmp,. png,. jpg,. (b) Share-information and/or share peripheral equipment
tif, or gif. (c) e-mail
(a) vector (b) bitmap (d) go online
(c) either vector or bitmap (d) neither vector nor bitmap (e) None of these
(e) None of these 114. Input, output, and processing devices grouped together to
103. Which of the following contains information about a single represent a(n) ..................
“entity in the database – like a person, place, event, or thing? (a) mobile device
(a) query (b) form (b) information processing cycle
(c) record (d) table (c) circuit board
(e) None of these (d) computer system
104. When you install a new program on your computer, it is (e) None of these
typically added to the ............ menu. 115. Most Web sites have a main page, the .................., which
(a) all Programs (b) select Programs acts as a doorway to the rest of the Web site page.
(c) start Programs (d) desktop Programs (a) search engine (b) home page
(e) None of these (c) browser (d) URL
105. After a user has saved and deleted many files, many (e) None of these
scattered areas of stored data remained that are too small to
116. Which of the following system components is the brain of
be used efficiently, causing ............
the computer?
(a) disorder (b) turmoil
(a) Circuit board (b) CPU
(c) disarray (d) fragmentation
(c) Memory (d) Network card
(e) None of these
106. Which of the following is the communications protocol that (e) None of these
sets the standard used by every computer that accesses 117. A .................. shares hardware, software, and data among
Web-based information? authorized users.
(a) XML (b) DML (a) network (b) protocol
(c) HTTP (d) HTML (c) hyperlink (d) transmitter
(e) None of these (e) None of these
107. A ............ is a computer attached to the Internet that runs a 118. Another word for software is .............
special Web server software and can send Web pages out (a) input (b) output
to other computers over the Internet. (c) program (d) system
(a) web client (b) web system (e) None of these
COMPUTER KNOWLEDGE D-21
119. The abbreviation ISP stands for 131. A(n) .......... camera is a peripheral device used to capture
(a) International Spy Project still images in a digital format that can be easily transferred
(b) Indian Social Planning into a computer and manipulated using graphics software.
(c) Internet solution Provider (a) digital (b) analog
(d) Internet Service Provider (c) classic (d) film
(e) None fo these (e) None of these
120. This component is required to process data into informa-
132. .......... makes it possible for shoppers to make purchases
tion and consists of integrated circuits
using their computers.
(a) Hard disk (b) RAM
(c) CPU (d) ROM (a) E-world (b) E-commerce
(e) None of these (c) E-spend (d) E-business
121. A .......... typically connects personal computers within a (e) None of these
very limited geographical area usually within a single 133. Networks are monitored by security personnel and
building. supervised by – who set(s) up accounts and passwords for
(a) LAN (b) BAN authorized network users.
(c) TAN (d) NAN (a) IT managers (b) the government
(e) None of these (c) network administrators (d) password administrators
122. Computers manipulate data in many ways, and this (e) None of these
manipulation is called ........... 134. Application software is designed to accomplish ...........
(a) utilizing (b) batching (a) real-world tasks (b) computer-centric tasks
(c) upgrading (d) processing (c) gaming tasks (d) operating system tasks
(e) None of these
(e) None of these
123. An e-mail address typically consists of a user ID followed
135. A telephone number, a birth date, and a customer name are
by the .......... sign and the name of the e-mail server that
manages the user’s electronic post office box. all examples of ..........
(a) @ (b) # (a) a record (b) data
(c) & (d) (c) a file (d) a database
(e) None of these (e) None of these
124. Software applies .........., also called algorithms, to process 136. The human-readable version of a program is called ...........
data. (a) source code (b) program code
(a) arithmetic (b) procedures (c) human code (d) system code
(c) objects (d) rules (e) None of these
(e) None of these 137. A .......... computer (also referred to as a laptop), is a small,
125. A file extension is separated from the main file name with lightweight personal computer that incorporates the screen,
a(n) ........... but no spaces. the keyboard, storage, and processing components into a
(a) question mark (b) exclamation mark single portable unit.
(c) underscore (d) period
(a) notebook (b) journal
(e) None of these
(c) diary (d) briefcase
126. An adhoc query is a ...........
(a) pre-planned question (b) pre-scheduled question (e) None of these
(c) spur-of-the-moment question 138. ........ is the result produced by a computer.
(d) question that will not return any results (a) Data (b) Memory
(e) None of these (c) Output (d) Input
127. A Web .......... consists of one or more Web pages located on (e) None of these
a Web server. 139. Programs such as Internet Explorer that serve as navigable
(a) hub (b) site windows into the Web are called............
(c) story (d) template (a) Hypertext (b) Networks
(e) None of these (c) Internet (d) Web browsers
128. A computer .......... is a set of program instructions that can (e) None of these
attach itself to a file, reproduce itself, and spread to other 140. A ........ is a device that not only provides surge protection,
files. but also furnishes your computer with battery backup power
(a) worm (b) virus
during a power outage.
(c) trojan horse (d) phishing scam
(a) surge strip (b) USB
(e) None of these
129. The desktop contains small graphics called .......... (c) UPS (d) battery strip
(a) windows (b) logos (e) None of these
(c) icons (d) pictures 141. When you save to ........, your data will remain intact even
(e) None of these when the computer is turned off.
130. C, BASIC, COBOL, and Java are examples of ...... languages. (a) RAM (b) motherboard
(a) low-level (b) computer (c) secondary storage device
(c) system programming (d) high-level (d) primary storage device
(e) None of these (e) None of these
D-22 COMPUTER KNOWLEDGE
142. The motherboard is the ........ (a) dictionary (b) word finder
(a) circuit board that houses peripheral devices (c) encyclopedia (d) thesaurus
(b) same as the CPU chip (e) None of these
(c) the first chip that is accessed when the computer is 155. Which key is used in combination with another key to
turned on perform a specific task?
(d) circuit board that contains a CPU and other chips (a) Function (b) Control
(e) None of these (c) Arrow (d) Space bar
143. A computer-intensive problem runs on a ............. (e) None of these
(a) server (b) main frame 156. A disk's content that is recorded at the time of manufacture
(c) supercomputer (d) super PC and that cannot be changed or erased by the user is ............
(e) None of these (a) read-only (b) memory-only
144. A(n) ............ converts and executes one statement at a time. (c) run-only (d) write-only
(a) compiler (b) interpreter (e) None of these
(c) converter (d) instructions 157. In an information system, alphanumeric data normally takes
(e) None of these the form of ............
145. The term ............ designates equip-ment that might be added (a) Sentences and paragraphs
to a computer system to enhance its functionality. (b) Numbers and alphabetical characters
(a) digital device (b) system add-on (c) Graphic shapes and figures
(c) disk pack (d) peripheral device (d) Human voice and other sounds
(e) None of these (e) None of these
146. Approximately how many bytes make one Megabyte ? 158. A web site's main page is called its ............
(a) One Thousand (b) Ten Thousand (a) home page (b) browser page
(c) One Hundred (d) One Million (c) search place (d) bookmark
(e) None of these (e) None of these
147. What is MP3 ? 159. When installing ............,the user must copy and usually
(a) A mouse (b) A printer decompress program files from a CD -ROM or other medium
(c) A Sound format (d) A Scanner to the hard disk.
(e) None of these (a) programming software (b) system hardware
148. All the deleted files go to ............ (c) applications hardware (d) applications software
(a) Recycle Bin (b) Task Bar (e) None of these
(c) Tool Bar (d) My Computer 160. A collection of interrelated files in a computer is a ............
(e) None of these (a) file manager (b) field
149. ............ this is the act of copying or downloading a program (c) record (d) database
from a network and making multiple copies of it. (e) None of these
(a) Network piracy (b) Plagiarism 161. A ........ computer is a large and expensive computer capable
(c) Software piracy (d) Site-license piracy of simultaneously processing data for hundreds or
(e) None of these thousands of users.
150. A directory within a directory is called ............ (a) server (b) mainframe
(a) Mini Directory (b) Junior Directory (c) desktop (d) tablet
(c) Part Directory (d) Sub Directory (e) None of these
(e) None of these 162. The trend in computer systems is toward the use of
151. Which is the best definition of a software package? graphical user interfaces (GUIs). In these operating
(a) An add-on for your computer such as additional systems, a trackball is described as
memory (a) a roller ball which moves the cursor ............
(b) A set of computer programs used for a certain function (b) a pen-shaped device which allows data to be entered
such as word processing through the CRT screen
(c) A protection you can buy for a computer (c) a figure which resembles a familiar office device
(d) The box, manual and license agreement that accompany (d) an outdated input device
commercial software (e) None of these
(e) None of these 163. Various applications and documents are represented on the
152. In MICR, C stands for ............ Windows desktop by ............
(a) Code (b) Colour (a) Symbols (b) Labels
(c) Computer (d) Character (c) Graphs (d) Icons
(e) None of these (e) None of these
153. Fax machines and imaging systems are examples of .......... 164. What is usually used for displaying information at public
(a) bar-code readers (b) imaging systems places ?
(c) scanning devices (d) pen-based systems (a) Monitors
(e) None of these (b) Overhead Projections
154. When writing a document, you can use the ......... feature to (c) Monitors and Overhead Projection
find an appropriate word or an alternative word if you find (d) Touch Screen Kiosks
yourself stuck for the right word. (e) None of these
COMPUTER KNOWLEDGE D-23
165. The real business and competitive value of information (a) digitalization (b) kilobyte
technology lies in ............ (c) record (d) file
(a) The software applications that are used by many (e) None of these
companies 174. ............ is a type of high-speed memory that a processor can
(b) The capabilities of the software and value of the access more rapidly than RAM.
information a business acquires and uses (a) Cache memory (b) Magnetic-storage
(c) The infrastructure of hardware, networks, and other IT (c) Read-only memory (ROM)
facilities that are commonly used by most companies (d) Solid state storage
(d) The capabilities of the hardware and the speed at (e) None of these
which it processes information 175. To contact people using the Internet, you most often use
(e) None of these their ............ .
166. Companies use which of the following vendors to provide (a) domain names (b) e-mail addresses
access to software and services rather than purchasing the
(c) usernames (d) passwords
applications and maintaining the applications themselves ?
(e) None of these
(a) Open source vendors
(b) Alliances 176. The trend of digital electronic devices becoming smaller
(c) Application service providers and increasingly powerful has fully
(d) All of the above supported the move to an increasingly ............ workforce.
(e) None of these (a) desktop (b) intelligent
167. Which one of the following would be considered as a way (c) server (d) mobile
that a computer virus can enter a compute system ? (e) None of these
(a) Opening an application previously installed on the 177. ............ hard drives are permanently located inside the system
computer unit and are not designed to be removed, unless they need
(b) Borrowed copies of software to be repaired or replaced.
(c) Viewing a website without causing any additional (a) Static (b) Internal
transactions (c) External (d) Remove
(d) Running antivirus programs (e) None of these
(e) None of these 178. A barcode is ............ code that represents data with bars of
168. Collecting personal information and effectively posing as another varying widths or heights.
individual is known as the crime of ............ (a) read/write (b) magnetic
(a) spooling (b) identity theft
(c) optical (d) laser
(c) spoofing (d) hacking
(e) None of these (e) None of these
169. The first step in the transaction processing cycle is............ 179. The Internet allows you to............
(a) database operations (b) audit (a) send electronic mail
(c) data entry (d) use inquiry (b) view Web pages
(e) None of these (c) connect to servers all around the world
170. In the information systems concept, the output function (d) All of the above
involves ............ (e) None of these
(a) Capturing and assembling elements that enter the system 180. The name that the User gives to a document is referred to
to be processed as............
(b) transformation processes that convert input into (a) name given (b) document given
output (c) file name (d) document identity
(c) Transferring elements that have been produced by a (e) none of these
transformation process to their ultimate destination 181. Editing a document consists of reading through the
(d) Monitoring and evaluating feedback to determine document you’ve created, then ............ .
whether a system is moving toward the achievement of (a) correcting your errors (b) printing it
its goal.
(c) saving it (d) deleting it
(e) None of these
171. When a computer runs a program, the ............ processes (e) None of these
through the program’s sequence of instructions. 182. Which of the following controls the manner of interaction
(a) AMD (b) ASCII between the user and the operating system ?
(c) CPU (d) transistor (a) User interface (b) Language translator
(e) None of these (c) Platform (d) Screen saver
172. The computer needs additional components called ............ (e) None of these
to accomplish its input, output, and storage functions. 183. What type of software is most useful for the creation of
(a) mice (b) specifications brochures, posters, and newsletters?
(c) architecture (d) peripherals (a) Spreadsheet software
(e) None of these (b) Web authoring software
173. Bytes combined to represent a named collection of (c) Multimedia authoring software
instructions or data stored in the computer or digital device (d) Desktop publishing software
is a(n) — (e) None of these
D-24 COMPUTER KNOWLEDGE
184. The quickest and easiest way in Word, to locate a particular (b) make efficient use of time
word or phrase in a document is to use the _______ (c) retain confidentiality of files
command. (d) simplify file structures
(a) Replace (b) Find (e) None of these
(c) Lookup (d) Search 195. In page preview mode ............
(e) None of these (a) You can see all pages of your document
185. One or more defects or problems that prevent the software (b) You can only see the page you are currently working on
from working as intended or working at all is a(n). (c) You can only see pages that do not contain graphics
(a) bug (b) bot (d) You can only see the title page of your document
(c) programming language (d) fuzzy logic (e) None of these
(e) None of these 196. To navigate to a new Web page for which you know the
186. ............ shows the files, folders, and drives on your computer, URL, type that URL in the brower’s ............ and press
making it easy to navigate from one location to another Enter
within the file hierarchy. (a) Address bar (b) Domain bar
(a) Microsoft Internet Explorer (c) Address button (d) Name button
(b) Windows Explorer (e) None of these
(c) My Computer 197. The CPU, also called the ............ when talking about PCs,
(d) Folders Manager does the vast majority of the processing for a computer.
(e) None of these (a) macroprocessor (b) RAM
187. A(n) ............ provides commands for writing software that is (c) Memory System (d) microprocessor
translated to the detailed step-by-step instructions executed (e) None of these
by the processor to achieve an objective or solve a problem. 198. A computer’s type, processor, and operating system define
(a) programming language (b) software patch its —
(c) presentation language (d) All language (a) brand (b) size
(e) None of these (c) platform (d) speed
188. A program written in a high level language is referred to as (e) None of these
(a) source code (b) object code 199. A kiosk—
(c) machine code (d) assembly code (a) is data organized and presented in a manner that has
(e) none of these additional value beyond the value of the data itself.
189. In order to save an existing document with a different name (b) combines microscopic electronic components on a
you need to ............ single integrated circuit that processes bits according
(a) Retype the document and give ita different name to software instructions.
(b) Use the Save as command (c) is a computer station that provides the public with
(c) Copy and paste the original document to a new specific and useful information and services.
document and then save (d) describes a compuuter’s type, processor, and operating
(d) Use Windows Explorer to copy the document to a system
different location and then rename it. (e) None of these
(e) None of these 200. The part of the CPU that accesses and decodes program
190. The piece of hardware that converts your computer’s digital instructions, and coordinates the flow of data among various
signal to an analog signal that can travel over telephone system components is the—
lines is called a ............ (a) ALU (b) control unit
(a) red wire (b) blue cord (c) megahertz (d) motherboard
(c) tower (d) modem (e) None of these
(e) None of these 201. Computer programs are written in a high-level programming
191. ............ is the part of the computer that does the arithmetical language; however, the human-readable version of a program
calculations. is called—
(a) Memory (b) OS (a) cache (b) instruction set
(c) CPU (d) ALU (c) source code (d) word size
(e) None of these (e) None of these
192. A ............ can make it easier to play games. 202. What is the difference between a CD-ROM and a CD-RW ?
(a) mouse (b) Joystick (a) They are the same-just two different terms used by
(c) keyboard (d) pen different manufacturers
(e) None of these (b) A CD-ROM can be written to and a CD-RW cannot
193. File extensions are used in order to ............ (c) A CD-RW can be written too, but a CD-ROM can only
(a) name the file be read from
(b) ensure the filename is not lost (d) A CD-ROM holds more information than a CD-RW
(c) identify the file (e) None of these
(d) identify the file type 203. What is the term for how words will appear on a page ?
(e) None of these (a) text formatting (b) Character formatting
194. Passwords enable users to ............ (c) Point size (d) typeface
(a) get into the system quickly (e) None of these
COMPUTER KNOWLEDGE D-25
204. The process of a computer receiving information from a server 214. The operating system determines the manner in which all of
on the Internet is known as – the following occurs except —
(a) pulling (b) pushing (a) user creation of a document
(c) downloading (d) transferring (b) user interaction with the processor
(e) None of these (c) printer output
205. When sending an e-mail, the ............ line describes the (d) data displayed on the monitor
contents of the message (e) None of these
(a) subject (b) to 215. Office LANs that are spread geographically apart on a large
(c) contents (d) cc scale can be connected using a corporate—
(e) None of these (a) CAN (b) LAN
206. All the deleted files go to — (c) DAN (d) WAN
(a) Recycle Bin (b) Task Bar (e) TAN
(c) Toll bar (d) My Computer 216. The taskbar is located .............
(e) None of these (a) on the Start menu
207. You organize files by storing them in — (b) at the bottom of the screen
(a) archives (b) folders (c) on the Quick Launch toolbar
(c) indexes (d) lists (d) at the top of the screen
(e) None of these (e) None of these
208. A Web site address is a unique name that identifies a specific 217. Generally, you access the Recycle Bin through an icon
located ..................
............ on the Web (a) on the desktop (b) on the hard drive
(a) Web browser (b) Web site (c) on the shortcut menu
(c) PDA (d) link (d) in the Properties dialog box
(e) None of these (e) None of these
209. ............ are specially designed computer chips that reside 218. The physical arrangement of elements on a page is referred
inside other devices, such as your car or your electronic to as a document’s ..............
thermostate (a) features (b) format
(a) Servers (b) Embedded computers (c) pagination (d) grid
(c) Robotic computers (d) Mainframes (e) None of these
(e) None of these 219. Where is data saved permanently?
210. Which of the following places the common data elements in (a) Memory (b) Storage
order from smallest to largest ? (c) CPU (d) Printer
(a) Character, file, record, field, database. (e) None of these
(b) Character, record, field, file, database. 220. Which of the following is not true about computer files?
(c) Character, field, record, file, database (a) They are collections of data saved to a storage medium
(d) Bit, byte, character, record, field, file, database (b) Every file has a filename
(e) None of these (c) A file extension is established by the user to indicate
211. Which of the following statements is false concerning file the file’s contents
names ? (d) Files usually contain data
(a) Files may share the same name or the same extension (e) None of these
but not both 221. What type of computer could be found in a digital watch?
(b) Every file in the same folder must have a unique name (a) Mainframe computer (b) Super computer
(c) Embedded computer (d) Notebook computer
(c) File extension is another name for file type.
(e) None of these
(d) The file extension comes before the dot (.) followed by
222. Which is not a basic function of a computer ?
the file name
(a) Copy text (b) Accept input
(e) None of these
(c) Process data (d) Store data
212. A disk’s content that is recorded at the time of manufacture (e) None of these
and that cannot be changed or erased by the user is — 223. The .................. is the box that houses the most important
(a) memory-only (b) write-only parts of a computer system.
(c) read-only (d) run-only (a) software (b) hardware
(e) None of these (c) input device (d) system unit
213. Distributed processing involves — (e) None of these
(a) solving computer component problems from a different 224. A .................. is a collection of information saved as a unit.
computer (a) folder (b) file
(b) solving computing problems by breaking them into (c) path (d) file extension
smaller parts that are separately processed by different (e) None of these
computers. 225. Peripheral devices such as printers and monitors are
(c) allowing users to share files on a network. considered to be ..................
(d) allowing users to access network resources away from (a) hardware (b) software
the office (c) data (d) information
(e) None of these (e) None of these
D-26 COMPUTER KNOWLEDGE

1. (c) 26. (e) 51. (a) 76. (d) 101. (b) 126. (c) 151. (b) 176. (b) 201. (c)
2. (a) 27. (d) 52. (b) 77. (a) 102. (c) 127. (a) 152. (d) 177. (b) 202. (c)
3. (a) 28. (b) 53. (d) 78. (c) 103. (c) 128. (b) 153. (c) 178. (c) 203. (a)
4. (a) 29. (d) 54. (e) 79. (b) 104. (a) 129. (c) 154. (d) 179. (d) 204. (c)

5. (b) 30. (b) 55. (d) 80. (d) 105. (d) 130. (d) 155. (b) 180. (c) 205. (a)
6. (b) 31. (b) 56. (b) 81. (d) 106. (c) 131. (a) 156. (a) 181. (a) 206. (a)
7. (b) 32. (e) 57. (b) 82. (a) 107. (d) 132. (b) 157. (b) 182. (a) 207. (b)

8. (d) 33. (d) 58. (c) 83. (d) 108. (a) 133. (c) 158. (a) 183. (d) 208. (b)
9. (b) 34. (a) 59. (d) 84. (b) 109. (a) 134. (a) 159. (a) 184. (d) 209. (b)
10. (c) 35. (d) 60. (a) 85. (c) 110. (a) 135. (a) 160. (d) 185. (a) 210. (c)
11. (a 36. (a) 61. (c) 86. (c) 111. (a) 136. (a) 161. (a) 186. (b) 211. (c)

12. (c) 37. (d) 62. (d) 87. (c) 112. (c) 137. (a) 162. (a) 187. (a) 212. (c)
13. (d) 38. (d) 63. (a) 88. (d) 113. (b) 138. (c) 163. (d) 188. (b) 213. (c)
14. (c) 39. (b) 64. (a) 89. (d) 114. (b) 139. (d) 164. (b) 189. (b) 214. (b)
15. (c) 40. (a) 65. (a) 90. (d) 115. (b) 140. (c) 165. (b) 190. (d) 215. (d)

16. (b) 41. (b) 66. (a) 91. (b) 116. (b) 141. (d) 166. (d) 191. (d) 216. (b)
17. (c) 42. (c) 67. (a) 92. (d) 117. (a) 142. (d) 167. (b) 192. (b) 217. (a)
18. (d) 43. (c) 68. (a) 93. (c) 118. (c) 143. (c) 168. (c) 193. (d) 218. (b)

19. (a) 44. (a) 69. (b) 94. (d) 119. (d) 144. (b) 169. (c) 194. (c) 219. (b)
20. (c) 45. (c) 70. (c) 95. (a) 120. (c) 145. (d) 170. (c) 195. (b) 220. (c)
21. (b) 46. (3) 71. (a) 96. (b) 121. (a) 146. (d) 171. (c) 196. (a) 221. (c)
22. (a) 47. (a) 72. (c) 97. (a) 122. (d) 147. (c) 172. (d) 197. (d) 222. (a)

23. (b) 48. (d) 73. (d) 98. (a) 123. (a) 148. (a) 173. (d) 198. (d) 223. (d)
24. (a) 49. (b) 74. (c) 99. (b) 124. (b) 149. (c) 174. (a) 199. (c) 224. (b)
25. (a) 50. (c) 75. (b) 100. (d) 125. (d) 150. (d) 175. (b) 200. (b) 225. (a)
Section E : TEST OF GENERAL AWARENESS

General
Awareness
1 Chapter
HISTORY OF BANKING Phase II
For the past three decades India's banking system has several Government took major steps in this Indian Banking Sector Reform
outstanding achievements to its credit. The most striking is its after independence. In 1955, it nationalised Imperial Bank of India
extensive reach. It is no longer confined to only metropolitans or with extensive banking facilities on a large scale specially in rural
cosmopolitans in India. In fact, Indian banking system has reached and semi-urban areas. It formed State Bank of India to act as the
principal agent of RBI and to handle banking transactions of the
even to the remote corners of the country. This is one of the main
Union and State Governments all over the country.
reasons for India's growth.
Seven banks which were subsidiaries of State Bank of India were
The government's regular policy for Indian banks since 1969 has nationalised in 1960. On 19th July, 1969, a major process of
paid rich dividends with the nationalisation of 14 major private nationalisation were carried out. It was the effort of the then Prime
banks of India in 1969. Minister of India, Mrs. Indira Gandhi. 14 major commercial banks
The first bank in India, though conservative, was established in in the country were nationalised.
1786. From 1786 till today, the journey of the Indian Banking System The second phase of nationalisation of Indian Banking Sector
can be segregated into three distinct phases. They are as mentioned was carried out in 1980 with seven more banks. This step brought
below: 80% of the banking segment in India under Government
Phase I Early phase from 1786 to 1969. ownership.
Phase II Nationalisation of Indian Banks and up to 1991 prior to In summary, the following are the steps taken by the Government
Indian banking sector Reforms. of India to Regulate Banking Institutions in the Country:
Phase III New phase of Indian Banking System with the advent 1. 1949 : Enactment of Banking Regulation Act.
of Indian Financial & Banking Sector Reforms after 2. 1955 : Nationalisation of State Bank of India.
1991. 3. 1960 : Nationalisation of SBI subsidiaries.
Phase I 4. 1961 : Insurance cover extended to deposits.
5. 1969 : Nationalisation of 14 major banks.
The General Bank of India was set up in the year 1786. Next came
7. 1975 : Creation of regional rural banks.
Bank of Hindustan and Bengal Bank.
8. 1980 : Nationalisation of seven banks with deposits over
The East India Company established Bank of Bengal (1809), Bank 200 crore.
of Bombay (1840) and Bank of Madras (1843) as independent After the nationalisation of banks, the branches of the public
units and called them Presidency Banks. These three banks were sector banks in India rose approximately 800% in deposits and
amalgamated in 1920 and Imperial Bank of India was established advances took a huge jump by 11,000%.
which started as a private shareholders bank with mostly Banking in the sunshine of Government ownership gave the public
Europeans shareholders. implicit faith and immense confidence about the sustainability of
In 1865 Allahabad Bank was established and for the first time these institutions.
exclusively by Indians, Punjab National Bank Ltd. was set up in 1894 Phase III
with headquarters at Lahore. Between 1906 and 1913, Bank of India, This phase has introduced many more products and facilities in
Central Bank of India, Bank of Baroda, Canara Bank, Indian Bank, the banking sector and is a major reforms measure. In 1991, under
and Bank of Mysore were set up. Reserve Bank of India came in 1935. the chairmanship of M Narasimhan, a committee was set up in his
During the first phase the growth was very slow and banks also name which worked for the liberalisation of banking practices.
experienced periodic failures between 1913 and 1948. There were Now-a-days the country is flooded with foreign banks and their
approximately 1100 banks, mostly small. To streamline the ATM stations. Efforts are being put to give a satisfactory service
to customers. Phone banking and net banking is introduced. The
functioning and activities of commercial banks, the Government
entire system has become more convenient and swift.
of India came up with The Banking Companies Act, 1949 which
The oldest bank in existence in India is the State Bank of
was later changed to Banking Regulation Act 1949 as per amending India, which originated as the Bank of Calcutta in June 1806, which
Act of 1965 (Act No. 23 of 1965). Reserve Bank of India was almost immediately became the Bank of Bengal.
vested with extensive powers for the supervision of banking in For many years the Presidency banks acted as quasi-central
India as the Central Banking Authority. banks, as did their successors. The three banks merged in 1921 to
During those days public had lesser confidence in the banks. As form the Imperial Bank of India, which, upon India's independence,
an aftermath, deposit mobilisation was slow. Abreast of it the became the State Bank of India. Indian merchants in Calcutta
savings bank facility provided by the Postal department was established the Union Bank in 1839, but it failed in 1848 as a
comparatively safer. Moreover, funds were largely given to traders. consequence of the economic crisis of 1848-49.
E-2 GENERAL AWARENESS
The Allahabad Bank, established in 1865 and still The years of the First World War were turbulent, and it took
functioning today, is the oldest Joint Stock bank in India. its toll with banks simply collapsing despite the Indian economy
Joint Stock Bank: A company that issues stock and requires gaining indirect boost due to war-related economic activities. At
shareholders to be held liable for the company's debt. It was not least 94 banks in India failed between 1913 and 1918 as indicated
the first though. in the following table:
That honour belongs to the Bank of Upper India, which was
established in 1863, and which survived until 1913, when it failed, Year Number of Authoris ed Paid-up
with some of its assets and liabilities being transferred to the bank s that capital Capital
Alliance Bank of Simla. failed (Rs . Lak hs ) (Rs. Lak hs )
Foreign banks too started to arrive, particularly in Calcutta, 1913 12 274 35
in the 1860s. The Comptoire d'Escompte de Paris opened a branch 1914 42 710 109
in Calcutta in 1860, and another in Bombay in 1862; branches in 1915 11 56 5
Madras and Puducherry, then a French colony, followed. HSBC 1916 13 231 4
established itself in Bengal in 1869. 1917 9 76 25
Calcutta was the most active trading port in India, mainly 1918 7 209 1
due to the trade of the British Empire, and so became a banking
center. INDIAN BANKS AFTER INDEPENDENCE
The first entirely Indian joint stock bank was the Oudh The partition of India in 1947 adversely impacted the economies
Commercial Bank, established in 1881 in Faizabad. It failed in 1958. of Punjab and West Bengal, paralyzing banking activities for
The next was the Punjab National Bank, established in Lahore in months. India's independence marked the end of a regime of the
1895, which has survived to the present and is now one of the Laissez-faire for the Indian banking. The Government of India
largest banks in India. initiated measures to play an active role in the economic life of the
Around the turn of the 20th Century, the Indian economy nation, and the Industrial Policy Resolution adopted by the
was passing through a relative period of stability. Around five government in 1948 envisaged a mixed economy. This resulted
decades had elapsed since the Indian Mutiny, and the social, into greater involvement of the state in different segments of the
industrial and other infrastructure had improved. Indians had economy including banking and finance. The major steps to
established small banks, most of which served particular ethnic regulate banking included:
and religious communities. 1. The Reserve Bank of India, India's central banking authority,
The presidency banks dominated banking in India but there was nationalized on January 1, 1949 under the terms of the
were also some exchange banks and a number of Indian joint Reserve Bank of India (Transfer to Public Ownership) Act,
stock banks. All these banks operated in different segments of 1948 (RBI, 2005b).[Reference www.rbi.org.in]
the economy. 2. In 1949, the Banking Regulation Act was enacted which
The exchange banks, mostly owned by Europeans, empowered the Reserve Bank of India (RBI) "to regulate,
concentrated on financing foreign trade. Indian joint stock banks control, and inspect the banks in India."
were generally undercapitalized and lacked the experience and 3. The Banking Regulation Act also provided that no new bank
maturity to compete with the presidency and exchange banks. or branch of an existing bank could be opened without a
This segmentationled Lord Curzon to observe, "In respect license from the RBI, and no two banks could have common
of banking it seems we are behind the times. We are like some old directors.
fashioned sailing ship, divided by solid wooden bulkheads into
separate and cumbersome compartments." BANK NATIONALISATION IN INDIA
The period between 1906 and 1911 saw the establishment of banks Despite the provisions, control and regulations of Reserve
inspired by the Swadeshi movement. The Swadeshi movement Bank of India, banks in India except the State Bank of India or SBI,
inspired local businessmen and political figures to found banks continued to be owned and operated by private persons. By the
of and for the Indian community. 1960s, the Indian banking industry had become an important tool
A number of banks established then have survived to the to facilitate the development of the Indian economy.
present such as Bank of India, Corporation Bank, Indian Bank, At the same time, it had emerged as a large employer, and a
Bank of Baroda, Canara Bank and Central Bank of India. debate had ensued about the nationalization of the banking
The fervour of Swadeshi movement lead to establishment industry. Indira Gandhi, then Prime Minister of India, expressed
of many private banks in Dakshina Kannada and Udupi districts the intention of the Government of India to nationalize banks.
which were unified earlier and known by the name South Canara Thereafter, her move was swift and sudden. The Government
(South Kanara) district. of India issued an ordinance and nationalised the 14 largest
Four nationalised banks started in this district and also a commercial banks with effect from the midnight of July 19, 1969.
leading private sector bank. Hence undivided Dakshina Kannada Jayaprakash Narayan, a national leader of India, described
district is known as the "Cradle of Indian Banking". the step as a "masterstroke of political sagacity." Within two weeks
During the First World War (1914-1918) through to the end of the of the issue of the ordinance, the Parliament passed the Banking
Second World War (1939-1945), and two years thereafter until the Companies (Acquisition and Transfer of Undertaking) Bill, and it
independence of India were challenging for Indian banking. received the presidential approval on 9 August 1969.
GENERAL AWARENESS E-3
A second dose of nationalization of 7 more commercial 1. Pre-Nationalization Era : In India the business of
banks followed in 1980. The stated reason for the nationalization banking and credit was practiced even in very early times. The
was to give the government more control of credit delivery. remittance of money through Hundies, an indigenous credit
With the second dose of nationalization, the Government of instrument, was very popular.
India controlled around 91% of the banking business of India. The Hundies were issued by bankers known as Shroffs,
Later on, in the year 1993, the government merged New Bank of Sahukars, Shahus or Mahajans in different parts of the country.
India with Punjab National Bank.
The modern type of banking, however, was developed by the
It was the only merger between nationalized banks and
Agency Houses of Calcutta and Bombay after the establishment
resulted in the reduction of the number of nationalised banks
from 21 to 20. After this, until the 1990s, the nationalised banks of Rule by the East India Company in 18th and 19th centuries.
grew at a pace of around 4%, closer to the average growth rate of During the early part of the 19th Century, volume of foreign
the Indian economy. trade was relatively small. Later on as the trade expanded, the
need for banks of the European type was felt and the government
LIBERALISATION of the East India Company took interest in having its own bank.
In the early 1990s, the Narasimha Rao government embarked The government of Bengal took the initiative and the first
on a policy of liberalization, licensing a small number of private presidency bank, the Bank of Calcutta (Bank of Bengal) was
banks. These came to be known as New Generation tech-savvy established in 1860.
banks, and included Global Trust Bank (the first of such new In 1840, the Bank of Bombay and in 1843, the Bank of Madras
generation banks to be set up), which later amalgamated with was also set up.
Oriental Bank of Commerce, Axis Bank(earlier known as UTI Bank), These three banks are also known as "Presidency Bank".
ICICI Bank and HDFC Bank.
The Presidency Banks had their branches in important trading
This move, along with the rapid growth in the economy of
centers but mostly lacked in uniformity in their operational policies.
India, revitalized the banking sector in India, which has seen rapid
growth with strong contribution from all the three sectors of banks, In 1899, the Government proposed to amalgamate these three
namely government banks, private banks and foreign banks. banks into one so that it could also function as a Central Bank,
The next stage for the Indian banking has been set with the but the Presidency Banks did not favour the idea.
proposed relaxation in the norms for Foreign Direct Investment, However, the conditions obtaining during World War I
where all Foreign Investors in banks may be given voting rights period (1914-1918) emphasized the need for a unified banking
which could exceed the present cap of 10%. At present it has institution, as a result of which the Imperial Bank was set up
gone up to 74% with some restrictions. in1921. The Imperial Bank of India acted like a Central bank and as
a banker for other banks.
Currently banking in India is generally fairly mature in terms The RBI (Reserve Bank of India) was established in 1935 as
of supply, product range and reach-even though reach in rural the Central Bank of the Country. In 1949, the Banking Regulation
India still remains a challenge for the private sector and foreign act was passed and the RBI was nationalized and acquired
banks. In terms of quality of assets and capital adequacy, Indian extensive regulatory powers over the commercial banks.
banks are considered to have clean, strong and transparent
In 1950, the Indian Banking system comprised of the RBI,
balance sheets relative to other banks in comparable economies
the Imperial Bank of India, Cooperative banks, Exchange banks
in this region.
and Indian Joint Stock banks.
The Reserve Bank of India is an autonomous body, with
2. Nationalization Stages:
minimal pressure from the government. The stated policy of the
After Independence, in 1951, the All India Rural Credit
Bank on the Indian Rupee is to manage volatility but without any
survey, committee of Direction with Shri. A. D. Gorwala as Chairman
fixed exchange rate-and this has mostly been true.
recommended amalgamation of the Imperial Bank of India and ten
In March 2006, the Reserve Bank of India allowed Warburg others banks into a newly established bank called the State Bank
Pincus to increase its stake in Kotak Mahindra Bank (a private of India (SBI).
sector bank) to 10%. The Government of India accepted the recommendations of
This is the first time an investor has been allowed to hold the committee and introduced the State Bank of India bill in the
more than 5% in a private sector bank since the RBI announced Lok Sabha on 16th April 1955 and it was passed by Parliament and
norms in 2005 that any stake exceeding 5% in the private sector got the president's assent on 8th May 1955. The Act came into
banks would need to be vetted by them. force on 1st July 1955, and the Imperial Bank of India was
In recent years critics have charged that the non- nationalized in 1955 as the State Bank of India.
government owned banks are too aggressive in their loan recovery The main objective of establishing SBI by nationalizing the
efforts in connection with housing, vehicle and personal loans. Imperial Bank of India was "to extend banking facilities on a large
There are press reports that the banks' loan recovery efforts scale more particularly in the rural and semi-urban areas and to
have driven defaulting borrowers to suicide. diverse other public purposes."
The three different phases in the history of banking in India are In 1959, the SBI (Subsidiary Bank) act was proposed and
the following eight state-associated banks were taken over by
described in detail below.
the SBI as its subsidiaries.
E-4 GENERAL AWARENESS
4. The credit facilities extended to the priority sector at
Name of the Bank Subsidiary with effect from concessional rates.
5. The high level of low yielding SLR investments adversely
1. State Bank of Hyderabad 1st October 1959
affected the profitability of the banks.
2. State Bank of Bikaner 1st January 1960 6. The rapid branch expansion has been the squeeze on
3. State Bank of Jaipur 1st January 1960 profitability of banks emanating primarily due to the increase
4. State Bank of Saurashtra 1st May 1960 in the fixed costs.
5. State Bank of Patiala 1st April 1960 7. There was downward trend in the quality of services and
efficiency of the banks.
6. State Bank of Mysore 1st March 1960
3. Post-Liberalization Era-Thrust on Quality and Profitability:
7. State Bank of Indore 1st January 1968 By the beginning of 1990, the social banking goals set for the
8. State Bank of Travancore 1st January 1960 banking industry made most of the public sector resulted in the
presumption that there was no need to look at the fundamental
With effect from 1st January 1963, the State Bank of Bikaner financial strength of this bank. Consequently they remained
and State Bank of Jaipur with head office located at Jaipur. undercapitalized. Revamping this structure of the banking industry
Thus, seven subsidiary banks State Bank of India formed was of extreme importance, as the health of the financial sector in
the SBI Group. particular and the economy was a whole would be reflected by its
The SBI Group under statutory obligations was required to open performance.
new offices in rural and semi-urban areas and modern banking The need for restructuring the banking industry was felt
was taken to these unbanked remote areas. greater with the initiation of the real sector reform process in 1992.
On 19th July 1969, then the Prime Minister, Mrs. Indira The reforms have enhanced the opportunities and
Gandhi announced the nationalization of 14 major scheduled challenges for the real sector making them operate in a borderless
Commercial Banks each having deposits worth Rs. 50 crore and global market place.
above. This was a turning point in the history of commercial However, to harness the benefits of globalization, there
banking in India.
should be an efficient financial sector to support the structural
Later the Government Nationalized six more commercial
reforms taking place in the real economy. Hence, along with the
private sector banks with deposit liability of not less than Rs. 200
reforms of the real sector, the banking sector reformation was also
crores on 15th April 1980, viz.
addressed.
1. Andhra Bank.
The route causes for the lackluster performance of banks,
2. Corporation Bank.
formed the elements of the banking sector reforms.
3. New Bank if India.
Some of the factors that led to the dismal performance of banks
4. Oriental Bank of Commerce.
5. Punjab and Sind Bank. were.
6. Vijaya Bank. 1. Regulated interest rate structure.
In 1969, the Lead Bank Scheme was introduced to extend 2. Lack of focus on profitability.
banking facilities to every corner of the country. Later in 1975, 3. Lack of transparency in the bank's balance sheet.
Regional Rural Banks were set up to supplement the activities of 4. Lack of competition.
5. Excessive regulation on organization structure and
the commercial banks and to especially meet the credit needs of
managerial resource.
the weaker sections of the rural society.
6. Excessive support from government.
Nationalization of banks paved way for retail banking and
Against this background, the financial sector reforms were initiated
as a result there has been an alt round growth in the branch
network, the deposit mobilization, credit disposals and of course to bring about a paradigm shift in the banking industry, by
employment. addressing the factors for its dismal performance.
The first year after nationalization witnessed the total In this context, the recommendations made by a high level
growth in the agricultural loans and the loans made to SSI by 87% committee on financial sector, chaired by M. Narasimham, laid
and 48% respectively. the foundation for the banking sector reforms. These reforms
The overall growth in the deposits and the advances tried to enhance the viability and efficiency of the banking sector.
indicates the improvement that has taken place in the banking The Narasimham Committee suggested that there should
habits of the people in the rural and semi-urban areas where the be functional autonomy, flexibility in operations, dilution of
branch network has spread. Such credit expansion enabled the banking strangulations, reduction in reserve requirements and
banks to achieve the goals of nationalization, it was however, adequate financial infrastructure in terms of supervision, audit
achieved at the coast of profitability of the banks. and technology.
Consequences of Nationalization The committee further advocated introduction of prudential
1. The quality of credit assets fell because of liberal credit forms, tran sparency in operations and improvement in
extension policy. productivity, only aimed at liberalizing the regulatory framework,
2. Political interference has been as additional malady. but also to keep them in time with international standards. The
3. Poor appraisal involved during the loan meals conducted emphasis shifted to efficient and prudential banking linked to
for credit disbursals. better customer care and customer services.
GENERAL AWARENESS E-5
Since the days of the Rural Credit Survey Committee (1954), Today the banking sector is seen as a vehicle for all inclusive
India has come a long way in its search for an appropriate rural economic growth, social responsibility and equiv-distribution of
banking set-up. Though there has been some improvement, the national resources.
problem remains. The entire process of customer service is dependent on following.
There has been tremendous progress in quantitative terms 1. Human resources
but quality has suffered, progress has been slow and halting and Any organization's success or failure is the result of success
significant regional disparities persist.
or failure of its employees collectively. Here the employee doesn't
Stagnation in rural banking is noticed in the north and
mean only the staff working down the ladder, but also includes
northeastern regions. The focus should be on assisting and
people right up to the top.
guiding small farmers. It is in this context that the role of rural
banking institutions has to be reconsidered. All the functions in an organization are undertaken by
The development strategy adopted and the increasing humans, whether it is selection of staff, development of product,
diversification and commercialisation of agriculture underline the making software, formulating policies, devising systems,
need for the rapid development of rural infrastructure and a larger procedures, defining processes, delivery channels, undertaking
flow of credit. market studies etc.
Progressive and not-so-small farmers have no difficulty in Humans may be assisted by the technology for arriving at
obtaining credit from the commercial banks. Credit for the poorer the decisions. In all the functions enumerated above, different
households is the real problem. departments do the work separately but the same are ultimately
Experience of RRBs that have locally-recruited employees; linked to each other to achieve the corporate goal.
the employees are unhappy in view of the lack of adequate career It is just like gears though rotating independently, move the
prospects. entire structure in the desired direction. If any gear malfunctions,
Apart from having a basic knowledge of agriculture and it brings the entire process to halt. Thus the human beings working
rural development, a rural banker is required to handle credit
in an organization are very important. Handling of humans by
extension work, scheme appraisal work in connection with farm
humans is a very complex job also.
and non-farm investments and the production of different crops,
The job requirements of HRD are to select, train, develop,
the monitoring/supervision and recovery of loans spread over
villages which are not even connected by all-weather roads and deploy, and motivate the human resources in the organization so
in an environment in which vested interests are quite powerful. as to get optimum results for the organization.
A person who says he has been in bank service for more 2. Products/services
than 25 years writes: "That rural credit has become unfashionable Banks do not provide physical goods to its customers. The
is evident from the fact that the subject is accorded only residual products which a bank offers are mostly financial products and
focus in the various congregations of our bankers. along with these products also provide other services which are
The placement policy in vogue in our banks is such that not financial in nature, like safe deposit vaults, Locker facilities
exposures in rural credit or agro-financing rarely count for etc.
promotions. In financial products there are basically two types of
Unfortunately a uniform standardized approach to lending activities, namely deposit procurement and its deployment
has led to rigidities as a result of which a farmer-borrower becomes profitably. These two activities constitute more than 80% of
a defaulter for no fault of his. Also, the agricultural sector is beset banking business in all the banks.
with considerable uncertainties - the weather and rainfall problem,
Deposits: Basic structure of deposit is to attract the customer
the pest problem and the market and price problem.
by offering interest on funds or some facility in lieu of interest.
Government interference that leaves no scope for these apex
bodies to show initiative and work out action plans for However depending upon the needs of different set of
development on their own is partly responsible for this situation. customers various types of deposit schemes are formulated. For
Another reason for such a state of affairs is that the apex example, savings bank accounts are for those who want short
bodies have expanded and prospered at the cost of primary bodies term savings with liquidity and to make regular deposits and
by taking over functions like deposit mobilisation even at the withdrawals etc.
rural level. Term deposits are for those who want to invest for longer
Today, banking sector is seen as a catalyst in economic duration having surplus funds not needed immediately.
growth of a country and, lot is expected from the banking fraternity. Some may want savings to grow gradually by contributing
The recognition of banking, as a tool for all inclusive growth smaller amounts at set intervals. The ultimate goal of depositor is
by economists, financial planners, reformist etc has made it an to keep his money safely in the bank and be able to use when
important sector in the Government's planning of economic needed. Likewise there are various combinations of deposit
growth. schemes based on liquidity, returns and safety.
The banking sector in India is therefore witnessing Advances: Banks, in a similar way deploy deposits by lending
tremendous changes because of political, social and economic
to those who need it at a cost in the shape of interest. Here again
changes that are taking place domestically and internationally.
the products differ depending upon the need of the customer. It
The concept of banking, which was earlier restricted to
accepting of deposits from public for the purpose of, has also may be overdraft facility, working capital finance, term loan, etc
undergone sea change. for business or personal needs.
E-6 GENERAL AWARENESS
Other products/ services: Apart from deposit and advances, Each complaint when made may be because of so many
banks offer various other facilities/services to their clients, like factors, not necessarily the fault of the person or branch against
remittances, investment services, fund management, financial which it is made. It may be due to system lapse, procedural
advisory services, tax collections, bill payment services etc. to deficiency, inapt technology, poor in-house work allocation, work
earn fee based incomes. flow module etc. Some times the complaints are frivolous and
The flexibility of banks to adopt changing needs and
made to harass the person concerned. Though in customer
expectation of customers and bring out products/ services to suit
oriented markets, customer is always right but care should be
customers is an important area in banking services.
A robust Research and Development department which can taken that the staff is also protected from frivolous complainants.
effectively and efficiently bring out newer products/ services Each complaint of the customer should be properly analyzed,
based on market feel and futurist visualization of customer assessed. It may be possible that route cause may be some where
preferences is an important aspect in banking services. else which should be rectified rather then the concerned staff or
3. Processes. branch made the scapegoat.
Today's customer is short of time and feels uncomfortable 6. Grievances redressal Mechanism.
when the process involved in getting the product or service is Improving upon the services is an ongoing process. The essential
lengthy and cumbersome. The customer wants very simple inputs are customer feedback, market surveys and the complaints
processes to get his work done. received by an organization. No organization can say that they
The processes for any product or service should be at the have zero customer complaints. However an organization which
minimum and at one go. Frequent back references and repeated
has robust mechanism to redress the complaints and resolve
information and excessive documentation dissatisfy the customer.
problem of the customer gets recognition as a customer friendly
The processes devised for getting the services should be
organization. Accepting the mistake and offering compensation
very customer friendly, easy to understand and complete.
4. Delivery channels goes a long way in retaining the customer. Most of the banks
Customer satisfaction is also dependent upon the delivery have come out with their compensation policies and customer
channels used by banks in providing the services. Today's grievances cells.
customer wants effortless, efficient, secure, simple and dependable 7. Market Studies.
channels of delivery, whether it is through humans or technology Market studies are effective tools to study the behavior of
driven channels. customers and their response to present standard of services. It
To quote an example, suppose a customer uses internet also helps to understand future trends and requirements as needs
banking and made a third party payment. He would like to know of the customer's keeps of changing with change of times.
what happened to his payment instructions. He should be able to Market research gives way to innovations in products and
track the payment on line till it reaches the beneficiaries account. services. Market studies may be done in-house, or assigned to
If this facility is not available, he may not be comfortable
outside expert agencies or both depending upon the vision of the
with the internet banking. Another thing mostly observed in Public
bank.
sector banks is that their websites are not updated regularly and
navigation is very tardy. The forms/ applications are scanned and The Reserve Bank of India participates in the payment
can not be filled on line. systems as a user of the system, as the service provider for various
5. Customer feedback and complaints components of the systems and is also the regulator of the systems
Feedback from customers is of immense help in formulating in many instances.
products, fine tune services and plug the loopholes. However As a user, the RBI submits instruments for clearing in the
most of the time, feedbacks are generally not available and public cheque-based clearing operations. RBI also participates as a user
sector banks are normally not enthused about taking feed back in the Electronic Clearing Service (ECS)and EFT systems for
on their services.
making its own internal payments to its employees, vendor
Rather wherever a customer gives his feedback (read
payments etc.
complaint), it is not taken in right spirit by the bank/ concerned
staff. Instead of looking into the real cause an effort is made to Similarly, RBI transactions in Repo / Reverse Repo under
provide alibis or blame the staff. It may be possible that that the LAF, Open Market Operations etc., would also be settled through
procedure itself is the cause of complaint or it is because of the respective components of payment systems.
reasons which are not under control of the branch. As a provider of payment system services, the RBI has
Today no bank is willing to accept complaints from the taken many initiatives as can be seen under the evolution of
customers and normally effort is made to somehow get the
payment systems in the country in the development and
complaint withdrawn or resolved without analyzing why the
operationalisation of the systems.
complaint has originated. It becomes very difficult for field level
staff to get the complaint redressed when the cause or reason of Under this, the clearing houses and ECS systems are
complaint is not because of them. However they are made to beg managed by the Reserve Bank of India at 16 and 15 centres
the customer to give satisfaction letter. respectively and EFT systems are completely managed by RBI at
GENERAL AWARENESS E-7

the 15 centres. The CFMS, NDS/SSS and RTGS systems have The National Payments Council is assisted by five
been fully developed, operationalised and maintained by RBI. permanent Task Forces, each of which is headed by a member of
Besides the above, RBI (through IDRBT) has also provided the the National Payments Council and comprises of a few experts
communication back bone to the financial system in the country appointed by the Chairman from different disciplines / institutions.
in the form of Indian Financial Network (INFINET). It is assisted by the respective Head of the Department concerned
By way of being the central bank, the RBI derives regulatory within the Reserve Bank. These are the:
powers in certain jurisdictions of payment systems. However, 1. Task Force on Monetary Policy and related issues;
specific oversight powers for payment systems for RBI is sought
2. Task Force on Payment and Settlement Systems Oversight;
to be obtained through appropriate legislation in the form of the
3. Task Force on Legal Issues;
Payment Systems Legislation and the setting of Board for Payment
and Settlement Systems. 4. Task Force on Technology Related Issues;
ORGANIZATIONALFRAMEWORK 5. Task Force on Systems and Procedures related issues.
Moving from a technology-based solution towards issues
of Payment and Settlement Systems, the Reserve Bank of India IMPORTANT FACTS
has adopted a holistic approach, in which Information Technology The first major step was Nationalization of the Imperial Bank
is an integral component. In order to usher in and establish a of India in 1955 via State Bank of India Act.
modern, robust payments and settlement system consistent with
State Bank of India was made to act as the principal agent of
international best practices, the Reserve Bank has adopted a three-
RBI and handle banking transactions of the Union and State
pronged strategy of Consolidation of existing Payment Systems,
Governments.
Development of Payment Systems and Integration of the Payment
and Settlement System. In a major process of nationalization, 7 subsidiaries of the
State Bank of India were nationalized by the Indira Gandhi regime.
In order to drive this Payment System reform process an
institutional framework and structure has been created within the In 1969, 14 major private commercial banks were nationalized.
Reserve Bank. The base layer of this structure consisted of the These 14 banks Nationalized in 1969 are as follows:
Payment Systems Group, which included an exclusive team of 1. Central Bank of India
inter-disciplinary professionals representing IT, Banking 2. Bank of Maharastra
Operations, Supervision, Legal, Economics, Government & Bank
3. Dena Bank
Accounts, and Foreign Exchange operations.
4. Punjab National Bank
The Group focused on the System Design of an integrated
5. Syndicate Bank
payments system, Payment Instruments, Electronic Banking
systems, Clearing and settlement arrangements, technological 6. Canara Bank
infrastructure, legal issues, Monetary Policy implications, Change 7. Indian Bank
management and responsibilities of banks. The Group was 8. Indian Overseas Bank
disbanded in December 2002.
9. Bank of Baroda
The next tier in the institutional framework is the Payment
10. Union Bank
Systems Advisory Committee which is a permanent body and
oversees the operations of the Payment Systems Group and 11. Allahabad Bank
reviews the developments in the area of Payment Systems. 12. Union Bank of India
The apex layer in the institutional structure is the National 13. UCO Bank
Payments Council. The council lays down the broad policy 14. Bank of India.
framework and guidelines for the implementation of a sound and The above was followed by a second phase of
efficient payments and settlement system for the country. nationalization in 1980, when Government of India acquired the
The NPC is chaired by the Deputy Governor in charge of ownership of 6 more banks, thus bringing the total number of
the Department of Information Technology and represented by Nationalised Banks to 20.
the Executive Director-in-Charge of the Department of Information The private banks at that time were allowed to function side
Technology, Chairman of the Indian Banks Association, Joint by side with nationalized banks and the foreign banks were allowed
Secretary, Banking Division, Ministry of Finance, Chairmen and to work under strict regulation.
Managing Directors of two Public Sector banks, one Private bank,
After the two major phases of nationalization in India, the
a Nonbanking financial company, Securities Exchange Board of
80% of the banking sector came under the public sector /
India and the National Stock Exchange.
government ownership.
E-8 GENERAL AWARENESS

Banking History : at a glance approved securities. Higher liquidity ratio forces commercial banks
Creation of Reserve bank of India: 1935 to maintain a larger proportion of their resources in liquid form
and thus reduces their capacity to grant loans and advances,
Nationalization of Reserve Bank of India : 1949 (January )
thus it is an anti-inflationary impact. A higher liquidity ratio diverts
Enactment of Banking Regulation Act : 1949 (March) the bank funds from loans and advances to investment in
Nationalization of State Bank of India : 1955 government and approved securities.
Nationalization of SBI Subsidiaries : 1959 Facts about Banking System in India
Nationalization of 14 major Banks : 1969 • The first bank in India to be given an ISO Certification
Creation of Credit Guarantee Corporation: 1971 – Canara Bank
Creation of Regional Rural Banks : 1975 • The first bank in Northern India to get ISO 9002 certification
Nationalization of 7 more banks with deposits over Rs. 200 Crore: for their selected branches
1980 – Punjab and Sind Bank
The result was outstanding. The public deposits in these banks • The first Indian bank to have been started solely with Indian
increased by 800% , as capital -Punjab National Bank
the government ownership gave the public faith and trust. • The first among the private sector banks in Kerala to become
The third phase of development of banking in India started in the a scheduled bank in 1946 under the RBI Act
early 1990s when India started its economic liberalization. – South Indian Bank
Reserve Bank of India India's oldest, largest and most successful commercial bank,
The Reserve Bank of India (RBI, ) is the central banking offering the widest possible range of domestic, international and
institution of India and controls the monetary policy of the rupee NRI products and services, through its vast network in India and
as well as US$300.21 billion (2010) of currency reserves. The overseas
institution was established on 1 April 1935 during the British Raj – State Bank of India
in accordance with the provisions of the Reserve Bank of India India's second largest private sector bank and is now the
Act, 1934 and plays an important part in the development strategy largest scheduled commercial bank in India
of the government. It is a member bank of the Asian Clearing – The Federal Bank Limited
Union
Bank which started as private shareholders banks, mostly
The central bank was founded in 1935 to respond to economic Europeans shareholders
troubles after the first world war.
– Imperial Bank of India
Policy rates and Reserve ratios
The first Indian bank to open a branch outside India in
Bank Rate: RBI lends to the commercial banks through its discount London in 1946 and the first to open a branch in continental
window to help the banks meet depositor’s demands and reserve Europe at Paris in 1974 -Bank of India, founded in 1906 in Mumbai
requirements. The interest rate the RBI charges the banks for this
The oldest Public Sector Bank in India having branches all
purpose is called bank rate. If the RBI wants to increase the liquidity
over India and serving the customers for the last 132 years.
and money supply in the market, it will decrease the bank rate and
if it wants to reduce the liquidity and money supply in the system, – Allahabad Bank
it will increase the bank rate. As of January, 2014 the bank rate was The first Indian commercial bank which was wholly owned and
9%. managed by Indians
Cash Reserve Ratio(CRR): Every commercial bank has to keep – Central Bank of India
certain minimum cash reserves with RBI. RBI can vary this rate Bank of India was founded in 1906 in Mumbai. It became the first
between 3% and 15%. RBI uses this tool to increase or decrease Indian bank to open a branch outside India in London in 1946 and
the reserve requirement depending on whether it wants to affect the first to open a branch in continental Europe at Paris in 1974.
a decrease or an increase in the money supply. An increase in List of Public Sector Banks in India
Cash Reserve Ratio (CRR) will make it mandatory on the part of
• Allahabad Bank
the banks to hold a large proportion of their deposits in the form
of deposits with the RBI. This will reduce the size of their deposits • Andhra Bank
and they will lend less.This will in turn decrease the money • Bank of Baroda
supply.The current rate is 4%. • Bank of India
Statutory Liquidity Ratio(SLR): Apart from the CRR, banks are • Bank of Maharashtra
required to maintain liquid assets in the form of gold, cash and
• Canara Bank
GENERAL AWARENESS E-9
• Central Bank of India List of Foreign Banks in India
• Corporation Bank • ABN-AMRO Bank
• Dena Bank • Abu Dhabi Commercial Bank
• Indian Bank • Bank of Ceylon
• Indian Overseas Bank • BNP Paribas Bank
• Oriental Bank of Commerce
• Citi Bank
• Punjab & Sind Bank
• China Trust Commercial Bank
• Punjab National Bank
• Deutsche Bank
• Syndicate Bank
• UCO Bank • HSBC

• Union Bank of India • JPMorgan Chase Bank


• United Bank of India • Standard Chartered Bank
• Vijaya Bank • Scotia Bank
• Taib Bank
List of Private Banks in India
• Bank of Punjab Upcoming Foreign Banks In India
• Bank of Rajasthan
By 2009 few more names is going to be added in the list of foreign
• Catholic Syrian Bank banks in India. This is as an aftermath of the sudden interest
• Centurion Bank shown by the Reserve Bank of India paving roadmap for foreign
banks have greater freedom in India. Among them is the world's
• City Union Bank
best private bank by EuroMoney magazine, Switzerland's UBS.
• Dhanalakshmi Bank
The following are the list of foreign banks going to set up business
• Development Credit Bank in India:
• Federal Bank
Royal Bank of Scotland
• HDFC Bank
Switzerland's UBS
• ICICI Bank
US-based GE Capital
• IDBI Bank
Credit Suisse Group
• IndusInd Bank
• ING Vysya Bank Industrial and Commercial Bank of China
• Jammu & Kashmir Bank Merrill Lynch is having a joint venture in Indian investment
banking space – DSP Merrill Lynch. Goldman Sachs holds stakes
• Karnataka Bank
in Kotak Mahindra arms.
• Karur Vysya Bank
GE Capital is also having a wide presence in consumer finance
• Laxmi Vilas Bank through GE Capital India.
• South Indian Bank India's GDP is seen growing at a robust pace of around 7% over
the next few years, throwing up opportunities for the banking
• United Western Bank
sector to profit from the present economic scenario.
• UTI Bank

³³³
E-10 GENERAL AWARENESS

EXERCISE
1. CONCEPTUAL BANKING
1. Which of the following terms is used in Banking Field? 9. When a loan is granted by a bank for purchase of white
(a) Interest Rate Swap (b) Input Devices goods it is called
(a) Consumption loan (b) White goods loan
(c) Sedimentary (d) Zero Hour (c) Consumer durable loan (d) All of these
(e) Privilege Motion (e) None of these
2. On which one of the following issues can SEBI penalize any 10. As per the reports in various newspapers many private
company in India? companies are trying to obtain the licences to launch a
(A) Violation of Banking Regulation Act. banking company in India. Which of the following
(B) Violation of foreign portfolio investment guidelines. organizations/agencies issue the licence for the same?
(a) Securities and Exchange Board of India (SEBI)
(C) For violation of Negotiable Instrument Act.
(b) Indian Institute of Banking and Finance (IIBF)
(a) Only (A) (b) All (A), (B) & (C) (c) Indian Banks Association
(c) Only (A) & (B) (d) Only (B) & (C) (d) Registrar of Companies
(e) Only (B) (e) None of the above
3. Expand the term ALM as used in Banking/Finance sector? 11. Which of the following methods is being adopted by Banks
(a) Asset Liability Mismatch for calculating and applying interest on Savings Bank
(b) Asset Liability Maturity accounts?
(a) On monthly products based on minimum balance
(c) Asset Liability Management
between 10th and last working day.
(d) Asset Liability Manpower (b) Daily balance is counted and interest is paid
(e) None of the above accordingly
4. What is the full form of the term LIBOR as used in financial (c) On average quarterly balance.
banking sectors? (d) On average half yearly balance.
(a) Local Indian Bank Offered Rate (e) None of the above.
(b) London-India Bureau of Regulations 12. The main function of IMF is to
(a) give financial investment loans to devoloping countries
(c) Liberal International Bank Official Ratio (b) act as a private sector lending arm of the World Bank
(d) London inter Bank Offered Rate (c) help of solve balance of payment problems of member
(e) None of the above countries
5. Expand the term FSDC which is used in financial sectors? (d) arrange international deposits from banks
(a) Financial Security and Development Council (e) None of the above
(b) Financial Stability and Development Council 13. Which of the following is not a part of the scheduled
(c) Fiscal Security and Development Council banking structure in India ?
(a) Money lenders (b) Public sector banks
(d) Fiscal Stability and Development Council (c) Private sector banks (d) Regional rural banks
(e) None of the above (e) State cooperative banks
6. For which one of the following reasons are Basel II norms to 14. The rate of interest on savings bank account is stipulated by
be followed by Commercial Banks? (a) the concerned bank
(a) Risk management (b) RBI
(b) Adoption of international accounting standards (c) Indian banks association
(d) Government of India
(c) Restriction on bonus payments to bank executives
(e) Banking codes and standards board of India
(d) Transparency in disclosures 15. Many times we read a term 'CBS' used in banking operation.
(e) None of the above What is the full form of the letter 'C' in the term 'CBS'?
7. Which one of the following will set up core banking (a) Core (b) Credit
infrastructure for rural banks? (c) Continuous (d) Complete
(a) RBI (b) NABARD (e) None of these
(c) SIDBI (d) IBA 16. The RBI reviews its credit and monetary policy at regular
intervals and also in between. What is the purpose of the
(e) None of these same ?
8. Which of the following rates/ratios is not covered under A. To ensure that inflation does not cross limits.
the RBI monetary and credit policy ? B. To ensure that banks have enough liquidity.
(a) Bank rate C. To ensure that cost of the fund does not reach a very
(b) Exchange rate of foreign currencies high level
(c) Repo rate (a) Only A (b) Only B
(d) Reverse repo rate (c) All A, B and C (d) Only B and C
(e) None of these
(e) Cash reserve ratio
GENERAL AWARENESS E-11
17. Under provisions of which one of the following Acts does 24. As we all know, the RBI has raised the Cash Reserve Ratio
the RBI issue directives to the Banks in India ? (CRR) by 25 bps, in April 2010. What action will banks have
(a) RBI Act to take to implement the same ?
(b) Banking Regulation Act (A) They will have to deposit some more money with the
(c) Essential Commodities Act RBI as reserve money.
(d) RBI and Banking Regulation Act (B) Banks will be required to give some more loan to projects
(e) None of the above of the priority sector.
18. Which one of the following tools is used by RBI for selective (C) Banks will be required to give a loan equivalent to the
credit control ? CRR to the Govt. of India for its day to day expenditure
(a) It advises banks to lend against certain commodities as and when required.
(b) It advises banks to recall the loans for advances against (a) Only A (b) Only B
certain commodities (c) Only C (d) Only A and B
(c) It advises banks to charge higher rate of interest for (e) All A, B and C
advance against certain commodities. 25. Insurance service provided by various banks is commonly
(d) It discourages certain kinds of lending by assigning known as ....
higher risk weights to loans it deems undesirable. (a) Investment Banking (b) Portfolio Management
(e) None of the above
(c) Merchant Banking (d) Bancassurance
19. For which one of the following are loan products 'teaser
(e) Micro Finance
loans' offered by banks ?
26. The rate on which banks borrow from the RBI is called .......
(a) Education loans
(a) SLR (b) CRR
(b) Commercial laons
(c) Loans against security of gold (c) Interest Rate (d) Bank Rate
(d) Retail trade loans (e) Repo Rate
(e) Home loans 27. As we know, the RBI is the apex bank of India; similarly, the
20. Under provisions of which one of the following Acts has apex Bank of USA is called.....
the Reserve Bank of India has the power to regulate, (a) Federal Reserve
supervise and control the banking sector? (b) The Central Bank of USA
(a) RBI Act (c) Bank of America
(b) Banking Regulation Act (d) Central Bank of USA
(c) Negotiable Instruments Act (e) None of the above
(d) RBI and Banking Regulation Act 28. What is the full form of 'NBFC' as used in the Financial
(e) None of the above Sector?
21. Credit rating is (a) New Banking Finance Company
(a) used to rate the borrowers while giving advances (b) National Banking & Finance Corporation
(b) used to work out performance of the employees (c) New Business Finance & Credit
(c) used to calculate the number of excellent audit rated (d) Non Business Fund Company
branches (e) None of the above
(d) not used in any bank 29. Cross-selling is not effective for which one of the following
(e) necessary before giving promotion to employees products ?
22. RTGS stands for (a) Debit cards (b) Savings accounts
(a) Real Time Gross Settlements (c) Internet banking (d) Pension loans
(b) Reduced Time Gross Settlements (e) Personal loans
(c) Relative Time Gross Settlements
30. As per the news published in a financial publication, RBI is
(d) Real Total Gross Securities
redesigning its 'ECS' to function as an Automated Clearing
(e) None of the above
House (ACH) for bulk transactions. What is full form of
23. As we all know, the Bank Rate at present is fixed at 6%.
'ECS'?
What does it mean in context to the banking operations ?
(a) Extra Closing System
(a) No bank will be able to give loan to any patron at a rate
(b) Electronic Cheque System
lower than the Bank Rate.
(c) Evening Cheque Sorter
(b) Bank should give loan to their priority sector customers/
borrowers at the rate of 6% only. They cannot charge (d) Evening Cheque System
less or more than this from their priority sector clients. (e) Electronic Clearing Service
(c) Banks are required to invest 6% of their total capital on 31. What is “wholesale banking”?
financial inclusion and inclusive banking operations. (a) It is a bank-to-bank or B2B dealing.
(d) Banks will be required to give 6% of their total deposits (b) It is a bank-to-customer dealing.
to the Govt. of India for meeting its Balance of Trade (c) It is a bank-to-trustworthy customer dealing.
requirements. (d) It is a bank-to-government dealing
(e) None of the above (e) None of these
E-12 GENERAL AWARENESS
32. Which of the following is the full form of the term SLR as 1. It will bring substantial economic benefits to India
used in the banking sector? and also to the countries where branches will be
(a) Social Lending Ratio opened.
(b) Statutory Liquidity Ratio 2. It will help in smooth inflow of FDI, long term capital
(c) Scheduled Liquidity Rate inflow and also technical know-how to India.
(d) Separate Lending Rate 3. It will give better opportunities to Indians to visit
(e) None of these foreign nations and get better facilities like education,
33. What is hot money? medical treatment, etc.
(a) Money which has tendency to migrate towards (a) Only 1 (b) Only 2
highprofit-oriented places. (c) Only 3 (d) Only 1 and 2
(b) Money which has tendency to migrate towards
(e) All 1, 2 and 3
lowprofit-oriented places
39. "Higher provisioning dragged down the profits of down
(c) Money, which has no interest
(d) Only (a) and (b) the 4th quarter of some banks" was the news in some major
(e) Only (a) and (c) newspapers. This means that the banks
34. As we all know, the major source of income of the banks is (a) had many NPAs
lending money (providing credit) and earning interest on it. (b) had no NPA
In normal circumstances, the demand of credit comes mainly (c) was in great loss
from which of the following sectors ? (d) was not able to earn any profit last year
1. Personal Loans (e) None of these
2. Priority Sector Lending and Bailout Packages 40. As we all know, RBI hiked the CRR by 0·25 basis points.
3. Project Finance What immediate impact will it have on the functioning of
(a) Only 1 (b) Only 2 banks ?
(c) Only 3 (d) All 1, 2 and 3
1. Banks will have to lock some additional fund with the
(e) None of these
RBI without any income generation to the banks on
35. One of the sources of income of banks is to charge fee for
these funds.
certain services. What are some of the services provided by
the banks for which they charge fee? 2. Banks will have to raise their capital base by bringing
1. Issuing Demand Drafts/Pay Orders IPOs or by taking money from the Govt. of India.
2. Issue of ATM/Credit/Debit Cards 3. Banks will have to give more funds to priority sectors
3. Electronic Transfer of Money and less to corporate sector.
(a) Only 1 (b) Only 2 (a) Only 1 (b) Only 2
(c) Only 3 (d) All 1, 2 and 3 (c) Only 3 (d) All 1, 2 and 3
(e) None of these (e) None of these
36. Many banks have launched/ floated their subsidiaries which 41. When the Reserve Bank says that the Rupee is over-valued,
are fully owned by them. Banks launch subsidiaries normally it means?
for which of the following businesses? (a) RBI has shortage of hard currency and it has to order
1. Home Loan Business for printing of new notes to the press concerned
2. To sell Insurance Policies
(b) Rupee is appreciating against other currencies
3. To control Online Operations or Internet Banking
whereas other currencies are weakening against US
business
Dollar
(a) Only 1 (b) Only 2
(c) Only 3 (d) All 1, 2 and 3 (c) Money is locked in banks and people do not have
(e) None of these enough to make their day to day purchases. It is a
37. "The set of directive principles issued by the Central Bank strategy which brings down inflation
of a country or the process adopted by it to control the (d) More and more people are selling their stocks. As a
supply of money, availability of money, cost of money and result stock market may crash, as there is no hard
rate of interest, etc. in order to bring stability and growth of cash for such voluminous transactions
the economy" are commonly known as- (e) None of these
(a) Monetary policy of the Central Bank of the country 42. Which of the following is not a nationalised bank?
(b) Budget of the Govt. (a) Canara Bank (b) Allahabad Bank
(c) Profit and Loss Account (c) IDBI Bank (d) PNB
(d) Business Policy of the Bank
(e) None of the above
(e) None of these
43. Which of the following is not a financial regulator?
38. As we see every day, many Indian banks are in close
competition to open their branches / offices in various (a) IRDA (b) PFRDA
countries. What benefit will the Indian economy have if (c) AMFI (d) SEBI
banks open branches in a foreign country ? (e) None of the above
GENERAL AWARENESS E-13
44. Which of the following key rates has not been altered by (c) They have two tier structures: at state and district
RBI since 2003? level
(a) Bank Rate (d) They grant loans on mortgage security
(b) Statutory Liquidity Ratio (e) They are customers of NABARD
(c) Cash Reserve Ratio 54. Which among the following statements is incorrect
(d) Repo Rate regarding the Regional Rural Banks?
(e) None of the above (a) They have been established as per the provisions of
45. Mobile banking is set to get a boost from IMPS, which Regional Rural Banks Act 1976
stands for (b) Ever y Region al Rural Bank is sponsored by
(a) Inter-Bank Mobile Payment Service commercial banks
(b) Inter-Bank Money Payment Service (c) The commercial banks need to approach to the state
(c) Inter-Bank Mobile Payment System governments to establish Regional Rural Banks
(d) Inter-Bank Money Payment System (d) The Regional Rural Banks may open branches in
(e) None of the above notified area
46. Which of the following is not correctly matched? (e) All statements are correct
55. Which among the following is the correct statement?
(a) Mutual Fund & SEBI (b) ULIP & SEBI
(a) Cooperative Banks are fully controlled by RBI
(c) Pension & PFRDA (d) Insurance & IRDA
(b) Cooperative Banks are partially controlled by RBI
(e) None of the above (c) Cooperative Banks are not controlled by RBI
47. Which among the following are the oldest financial (d) Cooperative Banks are fully controlled by Banking
institutions in India? Regulation Act 1949
(a) Private Agencies (b) Commercial Banks (e) None of the above is correct
(c) Reserve Bank of India (d) State bank of India 56. Agency Functions of the commercial Banks are __?
(e) None of the above (a) Primary Functions
48. In which year was SIDBI established? (b) Secondary Functions (c) Tertiary Functions
(a) 1975 (b) 1998 (d) Social Functions (e) None of the above
(c) 1990 (d) 1978 57. Which among the following is an essential feature of a
(e) 1981 commercial bank?
49. Which among the following is the first Development (a) providing Locker facilities
Financial Institution in the country to cater to the long-term (b) dealing with credit
finance needs of the industrial sector? (c) providing business information and data
(a) IFCI (b) IDBI (d) Underwriting (e) Sale of securities
58. Which among the following Banks in India was the first to
(c) ICICI (d) SIDBI
take a major step of reorganizing based upon market
(e) SBI
segmentation?
50. Which among the following is the main function of (a) ICICI Bank (b) State Bank of India
Agricultural Finance Corporation Limited ? (c) Punjab National Bank (d) Bank of Baroda
(a) consultancy services in the field of agriculture (e) Indian Bank
(b) social service organization 59. When a person has a savings account in the bank , the bank
(c) Agricultural commodities exchange assumes the position of ___?
(d) Agricultural cooperative society (a) Debtor (b) Creditor
(e) none of the above (c) Agency (d) Depositor
51. National Rural Credit Stabilization Fund is a Institution of (e) Agent
purpose-specific funds in which of the following? 60. Which among the following is the Bank rate?
(a) IDBI (b) SIDBI (a) the rate at which a commercial Bank lends to its
(c) NABARD (d) IFCI customers
(e) AFCL (b) the rate at which a commercial Bank lends to its best
52. To meet the need of which of the following has the concept customers
of Land Development Bank been developed? (c) the rate at which a central Bank lends to commercial
Banks
(a) Industries
(d) the rate at which a Commercial Bank lends to Central
(b) Real estate
bank
(c) Farmers (e) the rate at which a commercial Bank accepts deposits
(d) Export Oriented Units and SEZ from its customers
(e) Education Institutes 61. An investor or speculator who subscribes to a new issue
53. Which among the following statement is incorrect regarding with the intention of selling them shares soon after allotment
Land Development Banks? to realize a quick profit is called?
(a) They are cooperative institutions (a) Bull (b) Bear
(b) They meet the needs of the farmers for development (c) Stag (d) Short
purposes (e) Tall
E-14 GENERAL AWARENESS
62. The Unclaimed deposits are those deposits which haven't (a) Only 1 (b) Only 2
been operated for ______? (c) Only 3 (d) All 1, 2 & 3
(a) 5 years or more (b) 7 years or more (e) None of these
(c) 10 years or more (d) 12 years or more 72. The Reserve Bank of India keeps on changing various ratio/
(e) 15 years or more rates frequently. Why is this done?
63. In which year Statutory Liquidity Ratio was first imposed (1) To keep inflation under control.
on banks? (2) To ensure that Indian Rupee does not lose its market
(a) 1949 (b) 1951 value.
(c) 1955 (d) 1968 (3) To ensure that Banks do not earn huge profits at the
(e) 1971 cost of public money.
64. On which of the following dates does a Bank publish its (a) Only 1 (b) Only 2
balance sheet ? (c) Only 3 (d) All 1, 2 & 3
(a) March 31 (b) April 1 (e) None of these
(c) December 31 (d) January 1 73. Deficit financing increases—
(e) None of the above (a) Rate of money inflation (b) Rate of money deflation
65. Which among the following was the first Commercial Bank (c) Rate of devaluation (d) All of the above
of India? (e) None of these
(a) Hindustan Bank (b) The bank of Bengal 74. Which is not a monetary measure to control inflation ?
(c) Imperial Bank of India (d) Punjab National Bank (a) Soft loan policy
(e) Allahabad Bank (b) Hard credit policy
66. A new private bank is established under which of the (c) Tighten the regulations of money issue
following? (d) To reduce the quantity of money
(a) Banking regulation Act 1949 (e) None of these
(b) Companies Act 1956 75. Which is the monetary measure to control inflation ?
(c) RBI Act 1934 (a) Increase in taxation (b) Decrease in taxation
(d) All of the above (c) Soft credit policy (d) Hard credit policy
(e) None of the above (e) None of these
67. In which stock exchange, ‘currency futures’ trading was 76. Selling of securities in the open market by the central bank
first started ? creates—
(a) BSE (a) Inflation (b) Deflation
(b) NSE (c) Both of the above (d) None of the above
(c) MCX-SX (e) None of these
(d) All the above had the trading simultaneously 77. To control inflation the central bank should —
68. ‘ECB’ stands for (a) Sell government securities and decrease bank
(a) Extra Cornmercial Banking rate
(b) Extra Commerical Borrowing (b) Sell government securities and increase bank
(c) External Commerical Borrowing rate
(d) Extra Community for Bank (c) Purchase government securities and increase
(e) Noneof these bank rate
69. What is a “Consortium”? (d) Purchase government securities and to decrease
(a) When more than one bank allow credit facility to one bank rate
party in coordination with another under a formal (e) None of these
arrangement . 78. “Smart Money” is a term used for—
(b) In a consortium more than two persons together take (a) Internet Banking (b) Credit Card
loan from one bank. (c) Cash with Bank (d) Cash with Public
(c) In a consortium, banks provide loan only to the poor. (e) None of these
(d) Both (a) and (b) are correct 79. When was RBI nationalised ?
(e) None of these (a) 1st April, 1935 (b) 1st January, 1949
70. The Stock market index of London Stock Market is referred (c) 1st January, 1935 (d) 1st July, 1969
as— (e) None of these
80. How many banks are there in public sector at present ?
(a) Sensex (b) Footsie (FTSE)
(a) 28 (b) 27
(c) NIFTY (d) Bullish
(c) 19 (d) 20
(e) None of these
(e) None of these
71. As we all know Ministry of Finance every year prepares
81. The central banking functions in India are performed by the
Union Budget and presents it to the Parliament . Which of
I. Central Bank of India II. Reserve Bank of India
the following is / are the elements of the Union Budget ?
III. State Bank of India IV. Punjab National Bank
(1) Estimates of revenue and capital receipts.
(a) I, II and III (b) II
(2) Ways and Means to raise the revenue.
(c) I (d) II and IV
(3) Estimates of expenditure.
(e) None of these
GENERAL AWARENESS E-15
82. ICICI is the name of a (b) to reduce the freedom given to banks to rationalize
(a) chemical industry (b) bureau their existing branch network
(c) corporation (d) financial institution (c) to set up more foreign exchange banks
(e) None of these (d) to lend more easily for industrial development
83. If all the banks in an economy are nationalized and converted (e) None of these
into a monopoly bank, the total deposits 93. Deficit financing means that the government borrows money
(a) will decrease from the
(b) will increase (a) RBI (b) local bodies
(c) will neither increase nor decrease (c) big businessmen (d) IMF
(d) None of the above (e) None of these
84. The association of the rupee with pound sterling as the 94. The co-operative credit societies have a
intervention currency was broken in
(a) two-tier structure (b) three-tier structure
(a) 1990 (b) 1991
(c) four-tier structure (d) five-tier structure
(c) 1992 (d) 1993
(e) None of these
(e) None of these
95. Regional rural banks
85. On July 12, 1982, the ARDC was merged into
(a) RBI (b) NABARD I. have limited area of operation
(c) EXIM Bank (d) HDFC Bank II. have free access to liberal refinance facilities from
(e) None of these NABARD
86. If the cash reserve ratio is lowered by the RBI, its impact on III. are required to lend only to weaker sections
credit creation will be to (a) I, III (b) II, III
(a) increase it (b) decrease it (c) I, II, III (d) I, II
(c) no impact (d) constant (e) None of these
(e) None of these 96. The Board of Industrial and Financial Reconstruction (BIFR)
87. Which of the following items would not appear in a came into existence in
company's balance sheet? (a) 1984 (b) 1986
(a) Value of stocks of raw materials held (c) 1987 (d) 1989
(b) Total issued capital 97. The banks are required to maintain a certain ratio between
(c) Revenue from sales of the company's products their cash in hand and totals assets. This is called
(d) Cash held at the bank (a) Statutory Bank Ratio (SBR)
(e) None of these (b) Statutory Liquidity Ratio (SLR)
88. The currency convertibility concept in its original form (c) Central Bank Reserve (CBR)
originated in (d) Central Liquid Reserve (CLR)
(a) Wells Agreement (e) None of these
(b) Bretton Woods Agreement 98. How banks were nationalized in 1969?
(c) Taylors Agreement (a) 16 (b) 14
(d) Both a and b (c) 15 (d) 20
(e) None of these
(e) None of these
89. The central co-operative banks are in direct touch with
99. The accounting year of the Reserve Bank of India is
(a) farmers
(a) April-March (b) July-June
(b) state co-operative banks
(c) October-September (d) January-December
(c) land development banks
(d) central government (e) None of these
(e) None of these 100. In India, the first bank of limited liability managed by Indians
90. The first wholly Indian Bank was set up in and founded in 1881 was
(a) 1794 (b) 1894 (a) Hindustan Commercial Bank
(c) 1896 (d) 1902 (b) Oudh Commercial Bank
(e) None of these (c) Punjab National Bank
91. Our financial system has provided for the transfer of (d) Punjab and Sind Bank
resources from the centre to the states; the important means (e) None of these
of resource transfer are 101. If the RBI adopts an expansionist open market operations
(a) tax sharing (b) grant-in-aids policy, this means that it will
(c) loans (d) All the above (a) buy securities from non-government holders
(e) None of these (b) sell securities in the open market
92. In pursuance of the recommendations of Narsimhan (c) offer commercial banks more credit in the open market
Committee, the RBI has framed new guidelines (d) openly announce to the market that it intends to
(a) to govern entry of new private sector banks to make expand credit
the banking sector more competitive (e) None of these
E-16 GENERAL AWARENESS
102. Which of the following is NOT the name of a multinational 109. Which of the following is the apex institution which handles
bank? refinance for agriculture and rural development in India?
(a) BNP Paribas (a) RBI (b) SIDBI
(b) British Bank of Middle East (c) NABARD (d) SEBI
(c) Standard Chartered Bank (e) None of these
(d) HSBC 110. Which of the following is NOT a part of India's Money
(e) Cathay Pacific Market?
103. In order to avoid crowding of customers wanting to withdraw (a) Bill Market (b) Call Money Market
cash in the branches, banks have provided many delivery (c) Banks (d) Mutual Funds
channels. Which of the following is one of the most popular (e) Indian Gold Council
channels of getting instant hard cash? 111. Which of the following is the Regulator of the credit rating
(a) Core Banking Solution agencies in India?
(b) Pay Orders (a) RBI (b) SBI
(c) Demand Drafts (c) SEBI (d) SIDBI
(d) Automated Teller Machines (e) None of these
(e) None of these 112. Bank rate is decided by which of the following?
104. As we all know, when we deposit a cheque issued in our (a) RBI (b) Govt. of India
name in the bank, the bank always checks if the cheque has (c) SEBI (d) IBA
been crossed or not. Why is this done? (e) NABARD
(a) It ensures that the money is deposited only in the 113. FDI refers to –
account of the person in whose name the cheque has
(a) Fixed Deposit Interest
been drawn.
(b) Fixed Deposit Investment
(b) It is a process by which the person who has issued the
(c) Foreign Direct Investment
cheque comes to know whether the cheque is encashed
(d) Future Derivative Investment
or not.
(e) None of these
(c) The bank insists on it only when the party wants the
114. Teaser Rates are related to which of the following types of
payment immediately and that too in cash only.
loans?
(d) This is the instruction of RBI that all the cheques of
the amount of ` 10,000 should be accepted only if they (a) Home Loans
are crossed. (b) Personal Loans
(e) None of these (c) Auto Loans
105. Which of the following terms is NOT associated with (d) Reverse Mortgage Loans
banking operations? (e) Crop Loans
(a) Repo Rate (b) Prime Lending Rate 115. As we all know, the major source of income of the banks is
(c) Equator (d) Corporate Finance lending money (providing credit) and earning interest on it.
(e) Cash Reserve Ratio In normal circumstances, the demand of the credit comes
106. A student has got admission to a foreign university. From mainly from which of the following sectors?
where can he/she get the foreign currency ? (A) Personal Loans
(a) From the Bank of that country only (B) Priority Sector Lending and Bailout Packages
(b) From the Ministry of Foreign Affairs (C) Project Finance
(c) From office of the Consulate General of that country (a) Only (A) (b) Only (B)
(d) From an authorised foreign exchange dealer (c) Only (C) (d) All (A), (B) and (C)
(e) From any big five star hotel as many foreigners come (e) None of these
to stay there 116. Many banks have launched/floated their subsidiaries which
107. What is the full form the term 'FDIs' as used in the economic are fully owned by them. Banks launch subsidiaries normally
world? for which of the following businesses?
(a) Fixed Duration Investments (A) Home Loan Business
(b) Financial and Debt Instruments (B) To sell Insurance Policies
(c) Foreign Deployment Index (C) To control Online Operations or Internet Banking
(d) Foreign Direct Investments business
(e) Formal Direct Issues (a) Only (A) (b) Only (B)
108. Which of the following is NOT a Govt. sponsored (c) Only (C) (d) All (A), (B) and (C)
organization? (e) None of these
(a) Small Industries Development Bank of India 117. "The set of directive principles issued by the Central Bank
(b) NABARD of a country or the process adopted by it to control the
(c) National Housing Bank supply of money, availability of money, cost of money and
(d) ICICI Bank rate of interest, etc, in order to bring stability and growth of
(e) All are Govt. sponsored the economy" are commonly known as
GENERAL AWARENESS E-17
(a) Monetary policy of the Central Bank of the country (d) rate at which RBI purchases or rediscounts bills of
(b) Budget of the Govt. exchange of commercial banks
(c) Profit and Loss Account (e) None of the above
(d) Business Policy of the Bank 123. What is an Indian Depository Receipt?
(e) None of the above (a) A deposit account with a Public Sector Bank
118. Consider the following statements: (b) A depository account with any depositorys in India
1. In call money market, funds are borrowed and lent for (c) An instrument in the form of depository receipt
one day and for a period of up to 14 days without any created by an Indian depository against underlying
collateral security. equity shares of the issuing company
2. Ways and Means Advances are made the commercial (d) An instrument in the form of deposit receipt issued
banks to maintain mandatory reserves. by Indian depositories
3. Treasury Bills are promissory notes issued by State (e) None of the above
Governments for fixed period extending up to one year. 124. An instrument that derives its value from a specified
Which of the above statement/s is/are correct? underlying currency, gold, stocks etc, is known as
(a) Only (1) (b) Only (1) and (2) (a) Derivative (b) Securitisation Receipt
(c) Only (3) (d) Only (1) and (3) (c) Hedge Fund (d) Factoring
(e) All the above (e) Venture Capital Funding
119. Consider the following statement/s about ‘Open Market 125. Reverse repo means
Operations’ is/are true: (a) injecting liquidity by the Central Bank of a country
1. ‘Open Market Operation’ stands for the purchase and through purchases of Govt. securities
sale of government securities by the RBI from/ to the (b) absorption of liquidity from the market by sale of Govt.
public and banks. securities
2. The RBI buys all the unsold stock new government (c) balancing liquidity with a view to enhance economic
loans at the end of the subscription period. growth rate
3. Such purchases of the government securities by the (d) improving the position to availability of the securities
RBI constitute only an internal arrangement and are in the market
not genuine market purchases. (e) None of the above
(a) Only (1) (b) Only (1) and (2) 126. The stance of RBI monetary policy is
(a) inflation control with adequate liquidity for growth (b)
(c) Only (2) and (3) (d) Only (3)
improving credit quality of the banks
(e) All the above are true
(c) strengthening credit delivery mechanism
120. When the Reserve-Bank says that the Rupee is over-valued,
(d) supporting investment demand in the economy
it means
(e) None of the above
(a) RBI has shortage of hard currency and it has to order
127. Currency Swap is an instrument to manage
for printing of new notes to the press concerned
(a) currency risk
(b) Rupee is appreciating against other currencies
(b) interest rate risk
whereas other currencies are weakening against US
(c) currency and interest rate risk
Dollar
(d) cash flows in different currencies
(c) Money is locked in banks and people do not have
(e) All of the above
enough to make their day to day purchases. It is a
128. Euro Bond is an instrument
strategy which brings the inflation down
(a) issued in the European market
(d) More and more people are selling their stocks. As a (b) issued in Euro currency
result stock market may crash, as there is no hard (c) issued in country other than the country of the
cash for such voluminous transactions currency of the Bond
(e) None of the above (d) All of the above
121. RBI's open market operation transactions are carried out (e) None of the above
with a view to regulate 129. Money Laundering normally involves
(a) liquidity in the economy (a) placement of funds (b) layering of funds
(b) prices of essential commodities (c) integration of funds (d) All of (a), (b) and (c)
(c) inflation (e) None of (a), (b) and (c)
(d) borrowing power of the banks 130. The IMF and the World Bank were conceived as institutions to
(e) All of the above (a) strengthen international economic co-operation and
122. The bank rate means to help create a more stable and prosperous global
(a) rate of interest charged by commercial banks from economy
borrowers (b) Promote international monetary cooperation
(b) rate of interest at which commercial banks discounted (c) Promote long term economic development and
bills of their borrowers poverty reduction
(c) rate of interest allowed by commercial banks on their (d) All of (a), (b) and (c)
deposits (e) None of (a), (b) and (c)
E-18 GENERAL AWARENESS
131. Capital Market Regulator is 139. Which of the following is not a public sector unit/
(a) RBI (b) IRDA undertaking/agency?
(c) NSE (d) BSE (a) ECGC (b) SIDBI
(e) SEBI (c) SEBI (d) Axis Bank
132. What is the Call Money? (e) None of these
(a) Money borrowed or lent for a day or overnight 140. CRR stands for
(b) Money borrowed for more than one day but upto (a) Cash Rate Requirements
(b) Cash Reserve Ratio
3 days
(c) Credit Rate Requirements
(c) Money borrowed for more than one day but upto
(d) Credit Reserve Requirements
7 days (e) None of these
(d) Money borrowed for more than one day but upto 141. Green Banking means
14 days (a) financing of irrigation projects by banks
(e) None of the above (b) development of forestry by banks
133. Which of the following is the Regulator of the credit rating (c) financing of environment friendly projects by banks
agencies in India ? (d) Managing fishery by banks.
(a) RBI (b) SBI (e) None of these
(c) SIDBI (d) SEBI 142. The Securities and Exchange Board of India (SEBI) and the
(e) None of these Reserve Bank of India have given guidelines to enable
134. The logo of Bank of Baroda is known as trading in currency derivatives on stock exchanges. Which
(a) Sun of Bank of Baroda of the following statements is/are TRUE in this context?
(b) Baroda Sun A. Any bank is eligible to become a clearing member and/
(c) Bank of Baroda's Rays or a trading member of the currency derivatives segment
(d) Sunlight of Bank of Baroda of an exchange.
B. The last day for trading of the contract shall be two
(e) None of the above
working days prior to the final settlement day.
135. Many times we read about Future Trading in newspapers.
C. Currency futures are permitted now only in U.S. dollar-
What is 'Future Trading' ? Indian Rupee.
(A) It is nothing but a trade between any two stock (a) Only A (b) Only C
exchanges where in it is decided to purchase the (c) Both A & B (d) All A, B & C
stocks of each other on a fixed price throughout the (e) None of these
year 143. Fiscal deficit in the Union Budget means
(B) It is an agreement between two parties to buy or sell (a) The difference between crucial expenditure and current
an underlying asset in the future at a predetermined revenue.
price (b) The sum of budgetary deficit and net increase in
(C) It is an agreement between stock exchanges that they internal and external borrowings
will not trade the stocks of each other under any (c) Net increase in union government borrowings from
circumstances in future or for a given period of time Reserve Bank of India
(a) Only A (b) Only B (d) The sum of monetized deficit and budgetary deficit.
(c) Only C (d) All A, Band C (e) None of these
(e) None of these 144. Free international trade leads to equalization of
(a) prices in both the countries
136. The expansion for the abbreviation BIS, in the context of
(b) international price with domestic price
the banking industry is
(c) commodities of both the countries
(a) Bank for International Settlements (d) Trade between two countries
(b) Bank for Industrial Settlements (e) None of the above
(c) Bank for Industrial Sectors 145. Which one of the following is not a quantitative control
(d) Bank for International Services method of credit control?
(e) None of these (a) Cash reserve Ratio
137. As per the guidelines issued by the SEBI the Permanent (b) Statutory Liquidity Ratio (SLR)
Account Number (PAN) is a must for which of the following? (c) Bank – Rate
(a) Demat Accounts (d) Selective Credit Control
(b) All Saving Banks Accounts (e) None of these
(c) All Housing Loan Accounts 146. The RBI has helped to finance India’s foreign trade through
(d) All Current Accounts (a) IDBI (b) SBI
(e) None of these (c) EXIM Bank (d) NABARD
138. Which of the following names is not associated with the (e) None of these
insurance business in India? 147. When the price of a substitute of a commodity X falls, then
the demand for X
(a) Bajaj Allianz (b) GIC
(a) Rises (b) Falls
(c) LIC (d) GE Money
(c) Remains unchanged (d) First rises and then falls
(e) None of these (e) None of these
GENERAL AWARENESS E-19
148. The provision of credit and other financial services and 154. Which of the following organizations is known as Market
products of very small amount to the poor in rural semi Regulator in India?
urban and urban areas to enable them to raise their income (a) SEBI (b) IBA
level and living standard is known as (c) AMFI (d) NSDL
(a) Micro Credit (b) Personal Banking (e) None of these
(c) Corporate Banking (d) Non-Banking Finance 155. FSLRC stands for-
(e) None of these (a) Financial Sector Legislative Reforms Commission
149. Who among the following is known as the guardian of the (b) Financial Sector Legal Reforms Commission
‘Public Purse’ in India? (c) Financial Sectional Law Reforms Commission
(a) The President (d) None of the above
(b) The Comptroller and Auditor General (e) None of these
(c) The Finance Minister 156. With the recent increase in reports and reverse repo rates
(d) The Parliament announced by RBI there is speculation that the leading rates
(e) None of these of banks would also increase. Why?
150. The National Stock Exchange functions from (a) Cost of funds for banks would increase
(a) New Delhi (b) Kolkata (b) Banks use this opportunity raise lending rates
(c) Mumbai (d) Chennai (c) RBI would instruct the banks to raise the lending rates
(e) None of these (d) None of these
151. Fiscal Policy is related to 157. The terms Entry Load and Exit Load are most frequently
(a) Issue of currency used in context with the following?
(b) Credit creation (a) Health Insurance (b) Real Estate
(c) Public revenue and expenditure (c) Mutual Funds (d) Housing Loans
(d) All of these (e) None of these
(e) None of these 158. The Federation of Indian Chambers of Commerce and
152. The Foreign Exchange Reserves of India are kept in the Industry (FICCI) was founded in 1927 by
custody of (a) Birla and Tata (b) Tata and Thakurdas
(a) Prime Minister Rahat Kosh (c) Thakurdas and Birla (d) Tata and Godrej
(b) International Monetary Fund (e) None of these
(c) World Bank 159. Which of the following is not a Central Government tax?
(d) Reserve Bank of India (a) Income tax (b) Customs
(e) None of these (c) Land revenue (d) Corporation tax
153. Inflation implies (e) None of these
(a) Rise in budget deficit 160. Interest rates on Savings Bank Account is calculated on a
(b) Rise in general price index ..............basis.
(c) Rise in price of consumer goods (a) Quarterly (b) Monthly
(d) Rise in money supply (c) Daily (d) Half-yearly
(e) None of these (e) None of these

ANSWER KEY
1 (a) 17 (d) 33 (a) 49 (a) 65 (a) 81 (b) 97 (b) 113 (c) 129 (d) 145 (d)
2 (d) 18 (b) 34 (d) 50 (a) 66 (b) 82 (d) 98 (b) 114 (a) 130 (d) 146 (c)
3 (c) 19 (e) 35 (d) 51 (c) 67 (b) 83 (c) 99 (a) 115 (c) 131 (e) 147 (b)
4 (d) 20 (b) 36 (d) 52 (c) 68 (c) 84 (c) 100 (b) 116 (d) 132 (a) 148 (a)
5 (b) 21 (a) 37 (a) 53 (c) 69 (a) 85 (b) 101 (c) 117 (a) 133 (d) 149 (b)
6 (a) 22 (a) 38 (d) 54 (c) 70 (b) 86 (a) 102 (e) 118 (a) 134 (b) 150 (c)
7 (b) 23 (a) 39 (a) 55 (b) 71 (d) 87 (c) 103 (d) 119 (e) 135 (b) 151 (c)
8 (b) 24 (a) 40 (a) 56 (d) 72 (d) 88 (b) 104 (a) 120 (e) 136 (a) 152 (d)
9 (c) 25 (d) 41 (b) 57 (b) 73 (a) 89 (b) 105 (c) 121 (e) 137 (a) 153 (d)
10 (c) 26 (d) 42 (c) 58 (b) 74 (a) 90 (b) 106 (d) 122 (d) 138 (d) 154 (a)
11 (b) 27 (a) 43 (c) 59 (a) 75 (d) 91 (d) 107 (d) 123 (c) 139 (d) 155 (a)
12 (c) 28 (e) 44 (a) 60 (c) 76 (b) 92 (a) 108 (d) 124 (a) 140 (b) 156 (a)
13 (a) 29 (d) 45 (a) 61 (c) 77 (b) 93 (a) 109 (c) 125 (a) 141 (c) 157 (c)
14 (d) 30 (d) 46 (b) 62 (c) 78 (b) 94 (b) 110 (e) 126 (a) 142 (d) 158 (c)
15 (a) 31 (a) 47 (b) 63 (a) 79 (b) 95 (d) 111 (c) 127 (d) 143 (b) 159 (c)
16 (c) 32 (b) 48 (c) 64 (a) 80 (b) 96 (c) 112 (a) 128 (c) 144 (d) 160 (c)
E-20 GENERAL AWARENESS

EXERCISE
2. GENERAL AWARENESS
1. Which of the following books has been written by Bill Gates? 10. The Tropic of Cancer passes through which one of the
(a) Microsoft Secrets (b) The Road Ahead following?
(c) The Elephant Paradigm (d) How Nations Compete (a) Assam (b) Manipur
(e) None of these (c) Mizoram (d) Nagaland
2. Where is the headquarters of the UNO ? (e) None of these
(a) Geneva (b) The Hague 11. Which of these is not a film directed by Mira Nair?
(c) New York (d) Paris (a) Earth (b) Monsoon Wedding
(e) None of these (c) Kamasutra (d) Salaam Bombay
3. The inaugural DLF Indian Premier League Trophy (e) None of these
Championship was won by :
12. Which of the following owns the ‘Easy Day’ retail stores?
(a) Chennai Superkings (b) Kings XI Punjab
(a) Reliance Retail (b) Bharti Retail
(c) Rajasthan Royals (d) Delhi Dare Devils
(c) Wal-Mart (d) Birla Retail
(e) None of these
4. Which group has stepped into the retailing segment with (e) None of these
the launch that will operate under the brand name ‘more’? 13. ‘No Tobacco Day’ is observed on
(a) Tata group (a) 20th May (b) 22nd May
(b) Reliance group (c) 25th May (d) 31st May
(c) Aditya Birla group (e) None of these
(d) Mahindra & Mahindra group 14. September 8 is observed every year as
(e) None of these (a) World Health Day (b) World Peace Day
5. Which of the following schemes was launched to promote (c) World Literacy Day (d) Friendship Day
basic education in India and attract children in the school (e) None of these
going age to attend the classes?
15. ‘Big Bazaar’ chain of stores is run by which group?
(a) Operation Flood (b) Pulse Polio Abhiyan
(a) Jindal Group (b) Reliance Group
(c) Mid Day Meal Scheme (d) Operation Black Board
(c) Future Group (d) Aditya Birla Group
(e) None of these
(e) None of these
6. A new scheme for the girl child, ‘Dhan Laxmi’ – a conditional
cash transfer scheme – was launched by the Union 16. What does ‘Financial Inclusion’ mean?
Government in (a) Allow the merger and acquisition of banks so that only
(a) 2007 (b) 2009 few big banks exist and continue to cater to the need
(c) 2008 (d) 2010 of corporate sector.
(e) None of these (b) Providing insurance cover to each and every citizen
7. INS Airavat is India’s so that he/she can live a healthy and long life.
(a) Naval Ship (c) Expanding the network of banks of such a way that
(b) Amphibious Ship people from lower strata of society also get the benefit
(c) Submarine of services provided by banks
(d) Army Command Centre (d) To manage banking operations smoothly and merge
(e) None of these nationalised banks for further financial settlements.
8. Saina Nehwal is associated with which of the following (e) None of these
games? 17. Government of India is running a scheme, Jawaharlal Nehru
(a) Badminton (b) Chess National Urban Renewal Mission (JNNURM). Its objective
(c) Athletics (d) Cricket is to
(e) None of these (a) Improve urban Infrastructure
9. Who was the author of Das Kapital? (b) Providing employment in urban areas
(a) J. M. Keynes. (b) Karl Marx (c) Environment programme for cities
(c) James Tobin (d) Adam Smith (d) Establishing of small scale industries in urban areas
(e) None of these (e) None of these
GENERAL AWARENESS E-21
18. Which one of the following is not included in Article 19 of (3) The wages under the scheme are paid partly in cash
the constitution of India, pertaining to the Right to Freedom? and partly by giving food grains.
(a) Right to reside and settle in any part of the territory of (a) All 1,2 & 3 (b) Only 3
India (c) Only 3 (d) Only 1
(b) Right to form associations or unions (e) None of these
(c) Right of minorities to establish and administer 27. The aim of the ‘Sarva Shiksha Abhiyan’ is to universalise
educational institutions elementary eduction by the year
(d) Right to assemble peaceably and without arms (a) 2012 (b) 2009
(e) None of these (c) 2010 (d) 2011
19. India has recently signed Safeguard agreement with IAEA. (e) None of these
What does the ‘IAEA’ stand for? 28. World Population Day is observed on
(a) International Atomic Energy Agency (a) July 6 (b) July 7
(b) International Automobile Energy Agency (c) July 11 (d) July 1
(c) India Atomic Energy Agency (e) None of these
(d) India Atomic Emergency Agency 29. The World Trade Organization (WTO) was earlier known as
(e) None of these (a) UNICEF (b) GATT
20. Which is the first Asian country to host commonwealth (c) UNCTAD (d) FAO
games? (e) None of these
(a) South Korea (b) Japan 30. What does SAPTA stand for:
(c) Malaysia (d) India (a) South Asian Preferential Trade Agreement
(e) None of these (b) South Asian Post Trade Agreement
21. Which was the first Indian Company to be listed on (c) SAARC Preferential Trade Agreement
NASDAQ? (d) SAARC Post Trade Agreement
(a) Infosys (b) Satyam (e) None of these
(c) Reliance (d) TISCO 31. Merdeka Cup is associated with
(e) None of these (a) Hockey (b) Football
22. The Mahatma Gandhi National Rural Employment Guarantee (c) Basketball (d) Badminton
Act (MNREGA) provides for at least how many days of (e) None of these
wage employment to every rural household in a year?
32. Which of the following awards is given by UNESCO to
(a) 90 days (b) 180 days those who popularize use of science in life?
(c) 100 days (d) 300 days (a) Booker Prize
(e) None of these (b) Magsaysay Award
23. Which of the following is NOT included in the list of new (c) Kalinga Award
Seven Wonders of the World?
(d) Kalidas Samman
(a) Statue of Liberty (b) Taj Mahal
(e) None of these
(c) Great Wall of China (d) Roman Colosseum
33. Under the National Rural Employment Guarantee Programme,
(e) None of these the concerned rural households are to be given job cards
24. Which department presents report on Economic Survey of by
India?
(a) Gram Vikas Adhikari
(a) Ministry of Commerce
(b) Gram Panchayat
(b) Ministry of Finance
(c) Block Development Officer
(c) Planning Commission of India
(d) Kshettra Panchayat
(d) Prime Minister Office
(e) None of these
(e) None of these
34. Cristiano Ranaldo whose name was in news recently is a
25. The global community comes together to celebrate World famous
Environment Day on
(a) Politician (b) Journalism
(a) June 5 (b) June 4
(c) Sportsman (d) Author
(c) July 6 (d) October 2
(e) None of these
(e) None of these
35. The word ‘NELP’ is associated with
26. Which of the following statement (s) is/are correct about
(a) Discovery of Oil and Natural Gas
the Sampoorna Grameen Rozgar Yojana?
(b) National Highways Extension
(1) The scheme was launched in 2001 by merging some of
(c) Communications
the schemes running that time.
(d) Space Research
(2) In this scheme preference is given to Below Poverty
line families for the jobs. (e) None of these
E-22 GENERAL AWARENESS
36. Shovna Narayan is associated with which classical dance 45. According to the Indian Constitution, what is the minimum
form of India? educational qualification required for contesting the Lok
(a) Bharatnatyam (b) Kathakali Sabha elections?
(c) Kuchipudi (d) Kathak (a) Post Graduation
(e) None of these (b) Graduation
37. NCHER stands for (c) Higher Secondary
(a) National Capital for Higer Education and Research (d) No such qualification is required
(b) National Committee for Higher Education and Research (e) None of these
(c) Non-Capitalisation of Higher Education and Research 46. TRAI stands for
(d) National Commission for Higher Education and (a) Telephone Regulatory Authority of India
Research (b) Telegram Regulatory Authority of India
(e) None of these (c) Telecome Regulatory Authority of India
38. What are the temperate grasslands of Australia called? (d) Telecom Regulation Authority of India
(a) The Pampas (b) The Downs (e) None of these
(c) The Prairies (d) The Aborigine Land 47. Member of Parliament's Constituency fund has been hiked
(e) None of these up to—
39. Which of the following organisations gives the Kalinga (a) ` 5 crore (b) ` 3 crore
Prize? (c) ` 4 crore (d) ` 6 crore
(a) UNESCO (e) None of these
(b) CSIR 48. The following personalities recently passed away. With
(c) Ministry of Welfare reference to their respective field of work which of the
(d) Department of Science and Technology following is not correctly matched?
(e) None of these Person Field of Work
40. Which of the following is/are in the Tata Group of industries? (a) Arjun Singh – Politics
(a) Voltas (b) Indian Hotels (b) Navin Nischal – Film-acting
(c) Rallis India (d) All of these (c) Elizabeth Taylor – Literature
(e) None of these (d) Raminder Singh – Diplomacy / foreign service
41. Aam Admi Bima Yojana provides social security to ____ 49. Cryogenic Engines are used in which of the following
areas of technology?
(a) All landless labours living below poverty line in rural
areas (a) Atomic Energy (b) Food Technology
(b) All labours in urban areas (c) Oceanography (d) Space Research
(c) All labours in both rural and urban areas (e) None of these
(d) All labours in rural areas 50. Who is the author of the book Patrons of the Poor : Caste
Politics and Policy Making in India?
(e) None of these
(a) Narayan Lakshman (b) Greg Lindsay
42. The term ‘Fourth Estate’ refers to
(c) Khushwant Singh (d) Salman Rushdie
(a) Under-Developed State
(e) None of the above
(b) Parliament
51. First time India won the Cricket World Cup in 1983 by
(c) Judiciary
defeating—
(d) Press
(a) Australia (b) West Indies
(e) None of these
(c) Pakistan (d) Canada
43. Which group owns the Aquaguard water purifiers?
(e) None of these
(a) Tata (b) Birla
52. International Labour Day is observed on-
(c) Reliance (d) SPIC
(a) May 1 (b) May 3
(e) None of these
(c) May 9 (d) May 12
44. The three core values of the Commonwealth Games
(e) None of these
movement are:
53. Which team posted highest team total in Cricket World Cup
(a) Equality, Brotherhood and Unity
2011 ?
(b) Humanity, Equality and Destiny
(a) India (b) Pakistan
(c) Humanity, Equality and Brotherhood
(c) West Indies (d) Canada
(d) Unity, Humanity and Equality
(e) None of these
(e) None of these
GENERAL AWARENESS E-23
54. Which of the following decade registered highest growth 64. 'Dettol' is a product of which company?
rate in India? (a) Procter and Gamble (b) Hindustan Lever
(a) 1961-71 (b) 1971-81 (c) Johnson & Johnson (d) Reckitt-Beneckiser
(c) 1991-2001 (d) 2001-11 (e) None of these
(e) None of these 65 A French impressionist painter was the inspiration for a
55. The Population Stabilization Scheme launched by the Govt. range of shirts form Louis Philippe. Name the painter.
of India is named- (a) Manet (b) Mondrian
(a) Prerna (b) Sthapana (c) Monet (d) Win Gogh
(c) Unmesh (d) Aaradhana (e) None of these
(e) None of these 66. Who is/are the founder/s of Rolls Royce?
(e) None of these (a) Henry and Jack Ford
56. World Day for Water is observed on which of the following (b) Robert Zutshi
days ? (c) Charles Rolls and Henry Royce
(a) 22nd May (b) 22nd April (d) Katie Royce and Linda Rolls
(c) 22nd March (d) 20th March (e) None of these
(e) 20th April 67. Who owns the cosmetic brand, Lakme?
57. Which of the following awards is given by the Govt. of (a) Procter & Gamble (b) CavinKlare
India ? (c) Hindustan Unilever Ltd. (d) Wipro
(a) Kalinga Prize (b) Oscar Awards (e) None of these
(c) Man Booker Prize (d) Pulitzer Prize 68. Which company was the first to launch mobile phone
(e) Jawaharlal Nehru Award for International operations in India?
Understanding (a) Modi Telstra (b) Reliance India
58 What is the full form of IBM? (c) Bharati (d) Tata Indicom
(a) International Business Machines (e) None of these
(b) Indian Business Modifier 69. Hyundai belongs to which country?
(c) International Business Modifier (a) South Korea (b) China
(d) Indian Bureau of Mass communication (c) Italy (d) Japan
(e) None of these (e) None of these
59. The determent 'Tide' is manufactured by which company? 70. What is the full form of LG?
(a) Procter and Gamble (b) Hindustan Unilever (a) Lalvani-Grewals (b) Life is Good
(c) Henkel (d) Wipro (c) Lekraj Goods (d) Lufthansa Gateways
(e) None of these (e) None of these
60. The car Manza belongs to which of the following companies? 71. Which company under the brand name "Vardaan" sells low-
(a) Hyundai (b) Tata priced television sets in the rural and semi-rural markets?
(c) Honda (d) Toyota (a) Videocon (b) Panasonic
(e) None of these (c) BPL (d) Philips
61 The Allen Solly brand belongs to (e) None of these
(a) Raymonds (b) Madura Garments 72. Who is /are the founder/s of Microsoft?
(c) Arvind Mills (d) Lee Cooper (a) Bill Gates
(e) None of these (b) Paul Allen
62. The efforts of the Swiss Public Welfare Society in the late (c) Bill Gates and Paul Allen
nineteenth century led to the birth of a famous Nestle brand (d) Bill Gates and Ray Ozzie
for a quick-to-prepare and easy-to-digest packaged food.
(e) None of these
Name the brand and what it began with.
73. Which from the following wireless networks has a range of
(a) Maggi; soup
50 kilometres?
(b) Maggi; noodles (a) Macintosh (b) Wi-Max
(c) Taster's Choice; baked beans
(c) Wi-Fi (d) Blue Tooth
(d) Taster's Choice; yoghurt
(e) None of these
(e) None of these
74. What is the full form of ADAG?
63. 'Sugar Free' is a leading brand of artificial sweetener in India.
(a) Additional District Authority Group
It is owned by
(b) Area Development and Group
(a) Dabur (b) Cadila
(c) Army Development and Growth
(c) Cipla (d) Glaxo
(d) Anil Dhirubhai Ambani Group
(e) None of these
(e) None of these
E-24 GENERAL AWARENESS
75. Which of the following watch brands is not owned by 86. Who amongst the following is the author of the book ‘Half
Swatch Group, the Swiss watch company? a Life’?
(a) Tag Heuer (b) Rado (a) Mark Tully (b) Deepak Chopra
(c) Omega (d) Longines (c) Chetan Bhagat (d) Ved Mehta
(e) None of these (e) V. S. Naipaul
76. 'Eno' is produced by which company? 87. Expand the term CCEA as used in administrative circles.
(a) Paras Pharmaceuticals (b) Glaxo SmithKline (a) Cabinet Committee on External Affairs
(c) Johnson and Johnson (d) Ranbaxy Ltd (b) Cabinet Committee on Economic Affairs
(e) None of these (c) Cabinet Council on External Affairs
77. Which of the private telecom operator became the first to (d) Cabinet Council on Economic Affairs
introduce 3G services in the country? (e) None of the above
(a) Airtel (b) DOCOMO 88. With which one of the following games, is Baichung Bhutia
(c) IDEA (d) Vodafone associated?
(e) None of these (a) Hockey (b) Kabaddi
78. 'Boost' is a product of which company? (c) Golf (d) Football
(a) Cadbury (b) Vicco (e) Tennis
(c) Wipro (d) Dabur 89. With which one of the following games is ‘Tiger Woods
(e) None of these associated?
79. Name the founder of Ranbaxy Laboratories Ltd. (a) Golf (b) Swimming
(a) Rajendra Prasad Shukla (c) Tennis (d) Table Teennis
(b) Gurjar Sharan (e) Badminton
(c) Bhai Mohan Singh 90. Which one of the following is the largest producer of power
(d) Ramakrishna Gupta in India?
(e) None of these (a) Reliance Power (b) Tata Power
80. Which of the following brands does not belong to P & G ? (c) National Hydro Power (d) Adani Power
(a) Kotex (b) Pantene (e) NTPC
(c) Rejoice (d) Tide 91. For which one of the following States has the centre
(e) None of these appointed interlocutors?
81. Which among the following is the world's largest motor (a) Himachal Pradesh (b) J & K
vehicle manufacturer? (c) Nagaland (d) Manipur
(a) General Motors (b) Suzuki (e) None of these
(c) Hyundai (d) Toyota 92. Expand the term GSLV.
(e) None of these (a) Geosynchronous Satellite Launch Vehicle
82. What is the full form of SMS? (b) Global Satellite Launch Vehicle
(a) Short Messaging Service (c) Geosynchronous Station Launch Vehicle
(b) Simplified Messaging Service
(d) Global Station Launch Vehicle
(c) Swift Messaging Service
(e) None of the above
(d) Short Messaging System
93. Which of the following terms is used in Cricket?
(e) None of these
(a) Centre forward (b) Goal
83. Oracle and i2 Technologies are connected with
(c) Love (d) LBW
(a) Computer hardware (b) Networking
(e) Bull’s eye
(c) Education (d) Operation systems
94. With which one of the following activities are Golden Globe
(e) None of these
Awards associated?
84. What is an ISO series?
(a) Journalism (b) Social work
(a) Documentation of production processes
(c) Peace initiatives (d) Films
(b) Engineering process flowchart
(e) None of these
(c) Quality management and quality assurance standards
95. With which one of the following sports is Narain Karthikeyen
(d) None of these
is connected?
85. L'Oreal is a company of
(a) F-1 motor racing (b) Tennis
(a) France (b) England
(c) Badminton (d) Table Tennis
(c) Sweden (d) Japan
(e) Snooker
(e) None of these
GENERAL AWARENESS E-25
96. In which one of the following States is Sabari mala temple 104. Which of the following terms is used in Banking?
located? (a) Epicenter (b) Pascal's Law
(a) Tamil Nadu (b) A.P. (c) Currency (d) Centre of Gravity
(c) Kerala (d) Karnataka (e) Latitude
(e) None of these 105. Which of the following IT companies of India has for the
97. Which of the followmg is a book authored by Shri Pranab first time overtaken Infosys in quarterly revenues and
Mukherji? emerged as the second largest software company of India
(a) The Inheritance of Loss after Tata Consultancy Services (TCS)?
(b) The Argumentative Indian (a) HCL Technologies
(c) Beyond Survival: Emerging Dimensions of Indian (b) Wipro
Economy (c) Mahindra Satyam
(d) The Family and the Nation (d) Patni Computer System
(e) None of these (e) Cognizant Technology
98. Which of the following schemes of the Govt. of India is 106. With which one of the following sports is the name of
associated with the health sector? Parupalli Kashyap associated?
(a) NRDP (b) IRDP (a) Chess (b) Tennis
(c) ASHA (d) Bharat Nirman (c) Badminton (d) Athletics
(e) All of these (e) None of these
99. Which of the following is not the abbreviated name of a 107. ‘Goodbye Shahzadi’ is a book written by
scheme launched for watershed developments in India? (a) Shyam Bhatia (b) Ashok Mehta
(a) DPAP (b) DDP (c) Janardhan Thakur (d) Arun Gandhi
(c) VHNDs (d) IWDP (e) None of these
(e) Hariyali 108. Which of the following awards is given only to individuals
100. Which of the following is not the name of a Chemical and organisations of Asian countries for excellence in their
Fertilizer? respective fields?
(a) Urea (b) Murate of Potash (a) Booker Prize
(c) Diammonium Phosphate (d) Sodium Sulphate (b) Nobel Prize
(e) All are fertilizers (c) Templeton Prize
101. Which of the following is a sort of Poverty Eradication (d) Ramon MagsaysayAward
Programme of the Govt. of India, specially designed for rural (e) Oscar Awards
areas ? 109. As per the news in some major newspapers, 'Kawasaki' has
(a) Self Help Group established an Indian unit to manufacture its products
(b) Antodaya Anna Yojana locally. What is the product of Kawasaki ?
(c) Social Security for Agricultural and Unorganized (a) Luxury cars
Labour (b) Motor-cycles
(d) Enforcement of Minimum Wages (c) Tractors
(e) None of these (d) Power tillers and agricultural equipments
102. Which of the following programmes has been launched by (e) None of these
the "Ministry of Food and Civil Supplies" of the Govt. of 110. Which of the following is now a fundamental right of every
India? child in India?
(a) Mid-Day Meal Scheme (a) To get a unique identity card as a proof of citizenship
(b) Wheat-based Nutrition Programme (b) To get free medical aid in any hospital in India,
(c) Public Distribution System private or govt.
(d) Annapurna Scheme (c) To get enough food to survive even without any
(e) None of these parental support
103. Which of the following services is NOT provided by the (d) To get elementary education
post offices in India? (e) All of these
(a) Savings Bank Scheme 111. Which brand/company uses the ad-line 'We Know India
(b) Retailing of Mutual Funds Better'?
(c) Sale of stamps (a) LIC (b) UTI
(d) Issuance of Demand Drafts (c) SBI (d) GIC
(e) (b) and (d) both (e) None of the above
E-26 GENERAL AWARENESS
112. The North Atlantic Treaty Organisation (NATO) is 123. National Trust an NGO kicks off its nationwide campaign
headquartered at ‘Badhte Kadam’ every year. It is related to
(a) Brussels (b) Geneva (a) spreading awareness about proper sanitation.
(c) New York (d) Sweden (b) giving boost to and spreading awareness about the
(e) Camp David, Maryland USA rights of people with disability.
113. The Right of Children to Free and Compulsory Education (c) putting an end to the practice of child labour.
Bill was passed in the Lok Sabha. It falls under (d) the mission to improve literacy rate.
(a) The 86th Constitutional Amendment (e) None of these
(b) The 94th Constitutional Amendment
124. Which of the following is not correctly matched?
(c) The 90th Constitutional Amendment
Winner Golf Title
(d) The 89th Constitutional Amendment
(a) Charl Shwartzel – US Masters
(e) None of these
114. Who amongst,the following is the founder of Facebook? (b) Anirban Lahiri – Panasonic Open
(a) Steve Jobs (b) Steve James (c) Lee Westwood – Indonesia Masters
(c) Sabeer Bhatia (d) Mark Zuckerberg (d) Mithun Perera – Nepal Masters
(e) Bill Gates 125. Who is the author of the newly published novel NY?
115. Book ‘Devil May Care’ is written by (a) Kavin Maurer (b) Martin Dugard
(a) Salman Rushdie (b) Ian Fleming (c) Pete Townshend (d) Zadie Smith
(c) J K Rowling (d) Sebastian Falks (e) None of these
(e) None of these 126. Which of the following states has the highest number of
116. Book ‘Indian Army Vision 2020’ is written by Special Economic Zones?
(a) Deepak Kapoor (b) Gurmeet Kanwal Malik (a) Tamil Nadu (b) Karnataka
(c) VP Malik (d) Y. V. Reddy (c) Kerala (d) Andhra Pradesh
(e) None of these (e) None of these
117. Of the following, who has authored the Book “I Too Had a 127. During 1991-2001, India’s population had population growth
Dream”? of 21·65% which during 2001-11 became-
(a) Anita Nair (b) Verghese Kurien (a) 16·62% (b) 17·64%
(c) Okram Ibobi Singh (d) S.S. Sidhu
(c) 18·62% (d) 20·61%
(e) None of these
(e) None of these
118. ‘Out of My comfort Zone’ is the autobiography of a Cricketer
128. As per Census 2011, which of the two States have been
in which he has written the adverse effects of drinking
culture of his term. Name the Cricketer. ranked first and second most populous states?
(a) Imran Khan (b) Sanath Jaisurya (a) U.P. and Bihar (b) Bihar and U.P.
(c) Allen Boarder (d) Steve Waugh (c) Maharashtra and Bihar (d) U.P. and Maharashtra
(e) None of these (e) None of these
119. Who has written the book, ‘Gifted’? 129. Which of the State in India shows the lowest population as
(a) Anita Desai (b) Shobha De per Census 2011 ?
(c) Nikita Lalvani (d) Indira Sinha (a) Sikkim (b) Puducherry
(e) None of these (c) Tripura (d) Manipur
120. “Brarak Obama, the New Face of American Politics” has (e) None of these
been written by 130. Sex-ratio in India as per Census 2011 has gone up from 933
(a) Erez Mandela (b) Henary Keith (Census 2001) to-
(c) Bill Clinton (d) Martin Dupuls (a) 936 (b) 938
(e) None of these (c) 940 (d) 942
121. Aditya is a satellite to study (e) None of these
(a) the Sun (b) the Moon
131. In world’s population, India and China now hold, population
(c) the Mars surface (d) the Earth shares of-
(e) None of these
(a) 16·5% and 17·4% respectively
122. Who among the following is the author of the book ‘The
(b) 17·5% and 20·2% respectively
Namesake’?
(c) 17·5% and 19·4% respectively
(a) Arundhati Roy (b) Kiran Desai
(c) Amitava Ghosh (d) Jhumpa Lahiri (d) 18·2% and 19·4% respectively
(e) None of these (e) None of these
GENERAL AWARENESS E-27
132. Population density in India as per Census 2001 and Census 141. Indira Awaas Yojana (IAY) is a scheme of which among the
2011 stand at- following ministries?
(a) 325 and 378 per sq km respectively (a) Ministry of Housing & Urban Poverty Alleviation
(b) 325 and 382 per sq km respectively (b) Ministry of Rural Development
(c) 323 and 382 per sq km respectively (c) Ministry of Urban Planning
(d) 333 and 392 per sq km respectively (d) Ministry of Home Affairs
(e) None of these (e) None of these
133. During 2001-11, male and female population growth stood 142. Kitchens of India ready to eat food, is the product of which
at of the following?
(a) 17·2% and 18·1% respectively (a) Hamdard (b) ITC Limited
(b) 18·1% and 17·2% respectively 16 (c) Britannia (d) MTR
(c) 16·3% and 18·1% respectively (e) None of these
143. Which of the following financial institutions has funded
(d) 17·2% and 17·6% respectively
Rs.650 crore for upgrading transmission lines in seven towns
(e) None of these
of Bihar, a project to be completed by 2016?
134. Child sex-ratio (age group 0-6) which was 927 females /1000 (a) World Bank
males in 2001 became females/1000 males in 2011. (b) International Monetary Fund
(a) 912 (b) 913 (c) Asian Development Bank
(c) 914 (d) 915 (d) European Central Bank
(e) None of these (e) None of these
135. The States Union Territories showing highest and lowest 144. 'Discover a Passion' is the advertising punchline of which
sex ratio in Census 2011 are- product?
(a) Puducherry and Daman & Diu (a) Royal Stag (b) Chivas Regal
(b) Kerala and Daman & Diu (c) Wills Classic Milds (d) Marlboro Royale
(c) Kerala and Dadra & Nagar Haveli (e) None of these
(d) Puducherry and Dadra & Nagar Haveli 145. With which business school do you associate the ad-line,
(e) None of these 'To be amongst the top is not an act, but a habit'?
136. Literacy rate in India shows an improvement in Census 2011 (a) Harvard Business School
which rose from 64·83% (Census 2011) to- (b) Amity Business School
(a) 68·6% (b) 70·8% (c) IIPM
(c) 72.3% (d) 74·1% (d) IIMS
(e) None of these (e) None of these
137. Who amongst the following is the author of the book "Many 146. The water purifier Zero-B belongs to
Lives Many Masters" ? (a) Ion Exchange (b) Eureka Forbes
(a) Robin Cook (c) Zeolite India (d) Dabur
(b) Dr. Brian Weiss (e) None of these
(c) L. K. Advani 147. 'Talk to me.' This is an ad line of
(d) Admiral Vishnu Bhagwat (a) Listerine (b) Colgate Fresh Energy
(c) Airtel (d) Close-UP
(e) None of these
(e) None of these
138. Management thinker M.B. Athreya bases his concepts on
148. 'Paints for dream homes' is a slogan for
(a) Common sense (b) The Holy Bible
(a) Asian Paints (b) Nerolac All Scapes
(c) The Gita and the Vedas (d) Modern science
(c) Dulux Paints (d) Berger Paints
(e) None of these
(e) None of these
139. The 'big bold beautiful' line in watches comes from
149. Entertainment doesn't get bigger than this' is the claim for
(a) Titan (b) HMT (a) LG Flatron television (b) Philips home theatre
(c) Swatch (d) Timex (c) Sony television (d) Star TV
(e) None of these (e) None of these
140. Which brand would you associate with the famous line, 150. 'More shoes to chose' is the ad slogan for which of the
'Come to where the flavor is'? following range of footwear?
(a) Charminar (b) Four Square (a) Liberty (b) Metro
(c) Dunhill (d) Marlboro (c) Bata (d) Action
(e) None of these (e) None of these
E-28 GENERAL AWARENESS
151. Complete this ad with one word: 'The first Indian suiting 155. 'Nobody delivers Kerala better' is the punch line of which
ever worn came from…' newspaper daily?
(a) Raymond (b) Grasim (a) Malyala Manorama (b) Mathrabhumi
(c) Gwalior (d) Graviera (c) The Hindu (d) The Week
(e) None of these (e) None of these
152. 'The science of the picture. From the inventors of wrinkle- 156. 'Your potential, Our Passion' is the punchline of which of
free viewing'- this was the ad line for ………. colour the following?
televisions. (a) Oracle (b) Microsoft
(a) LG (b) Philips (c) IBM (d) HCL
(c) Samsung (d) Sony (e) None of these
(e) None of these 157. Making tomorrow brighter' is the promotional line of which
153. 'What you dream' is the ad line of which company? major corporate?
(a) Philips (b) Panasonic (a) ONGC (b) SAIL
(c) Electrolux (d) Sony (c) Hewlett-Packard (d) Wipro
(e) None of these (e) None of these
154. 'Tata-Sky' is a joint venture between which of the following?
(a) Tata-Star DTH (b) HBO-Tata
(c) Tata-Dishnet (d) Dishnet-Star
(e) None of these

ANSWER KEY
1 (b) 21 (a) 41 (a) 61 (b) 81 (a) 101 (b) 121 (b) 141 (b)
2 (c) 22 (c) 42 (d) 62 (a) 82 (a) 102 (c) 122 (d) 142 (a)
3 (c) 23 (a) 43 (a) 63 (b) 83 (b) 103 (d) 123 (c) 143 (c)
4 (c) 24 (b) 44 (c) 64 (d) 84 (c) 104 (c) 124 (a) 144 (a)
5 (c) 25 (a) 45 (b) 65 (c) 85 (a) 105 (e) 125 (b) 145 (b)
6 (c) 26 (a) 46 (c) 66 (c) 86 (e) 106 (a) 126 (a) 146 (b)
7 (b) 27 (c) 47 (a) 67 (c) 87 (b) 107 (a) 127 (c) 147 (c)
8 (a) 28 (b) 48 (c) 68 (a) 88 (d) 108 (d) 128 (d) 148 (a)
9 (b) 29 (b) 49 (d) 69 (a) 89 (a) 109 (b) 129 (a) 149 (c)
10 (c) 30 (a) 50 (b) 70 (b) 90 (e) 110 (d) 130 (c) 150 (c)
11 (a) 31 (b) 51 (b) 71 (d) 91 (b) 111 (a) 131 (b) 151 (b)
12 (b) 32 (c) 52 (c) 72 (c) 92 (a) 112 (a) 132 (a) 152 (a)
13 (d) 33 (b) 53 (b) 73 (b) 93 (d) 113 (a) 133 (c) 153 (c)
14 (c) 34 (c) 54 (a) 74 (d) 94 (d) 114 (d) 134 (b) 154 (b)
15 (c) 35 (a) 55 (b) 75 (a) 95 (a) 115 (d) 135 (a) 155 (d)
16 (c) 36 (d) 56 (a) 76 (b) 96 (c) 116 (b) 136 (c) 156 (a)
17 (a) 37 (d) 57 (e) 77 (b) 97 (c) 117 (b) 137 (b) 157 (a)
18 (c) 38 (b) 58 (a) 78 (a) 98 (c) 118 (d) 138 (a)
19 (a) 39 (a) 59 (a) 79 (c) 99 (c) 119 (c) 139 (c)
20 (c) 40 (b) 60 (b) 80 (a) 100 (d) 120 (c) 140 (d)
GENERAL AWARENESS E-29

EXERCISE
3. CURRENT BANKING
1. According to Reserve Bank of India's latest study, which 9. Who is the Governor of the Bank of Japan?
one of the following states ranks as the best performing (a) Matsukata Masayoshi (b) Haruhiko Kurodo
state in most of the key fiscal parameters? (c) Toyotaro Yuki (d) Masaaki Shirakawa
(a) Gujarat (b) Kerala (e) None of these
(c) Chhattisgarh (d) Odisha 10. National Bank for Agriculture and Rural Development
(e) None of these (NABARD) recently reduced the refinance rates for the
2. The Reserve Bank of India has power to print currency notes banks and other lending agencies by 0.20 percent. The
of up to Rs ____? NABARD was established on the recommendation of which
(a) 1,000 (b) 5,000 committee?
(c) 10,000 (d) 15,000 (a) Narendran Commission
(e) None of these (b) Mukherjee Commission
3. The RBI has recently decided to withdraw from circulation (c) Justice Shah Commission
of currency notes that had been issued before___? (d) B Sivaraman Committee
(a) 2004 (b) 2006 (e) None of these
(c) 2005 (d) 2008 11. Reserve Bank of India (RBI) on 15 January 2014 included
(e) None of these two countries in the sensitive list under FEMA Act, 2000.
4. India on 2 January 2014 has signed a credit agreement of The countries in question are
(a) Iran and China (b) Hong Kong and Macau
160 million dollar with World Bank for the modernization of
(c) Syria and Sudan (d) Israel and Libya
(a) Uttar Pradesh Road Sector
(e) None of these
(b) Rajasthan Road Sector
12. Reserve Bank of India on 20 January 2014 constituted an
(c) Bihar Road Sector
Expert Committee to Review Governance of Bank Boards in
(d) West Bengal Road Sector
India. The expert committee will be chaired by?
(e) None of these
(a) Urjit R. Patel (b) HR Khan
5. Which institution has been recognized by RBI recently to (c) KC Chakrabarty (d) PJ Nayak
act as local operating unit for issuing globally compatible (e) None of these
legal entity identifiers (LEI) in India? 13. Recently, an Expert Panel on Monetary Framework set up
(a) FICCI by the RBI recommended to strengthen and revise the
(b) India Inc current monetary policy framework. The Expert Panel was
(c) Clearing Corporation of India Ltd. headed by?
(d) Financial Stability Board (a) Raghuram G Rajan (b) Peter J. Montiel
(e) None of these (c) Rupa Nitsure (d) Urjit Patel
6. A panel of Reserve Bank of India has suggested to set-up (e) None of these
specialized banks to cater to the low income households 14. Recently RBI issued new guidelines on loan restructuring
and small businesses. These banks will ensure that every of Non-banking Financial Companies (NBFCs). The
citizen have a bank account by 2016. The panel was headed guidelines were based on the recommendation of which
by Committee?
(a) Mukul Mudgal (b) Nachiket Mor (a) Nachiket Mor Committee (b) Urijit Patel Committee
(c) Raghuram Rajan (d) Bilal Nazaki (c) Bimal Jalan Committee (d) B Mahapatra Committee
(e) None of these (e) None of these
7. Who has been appointed as the Managing Director and 15. In international banking system, what does the TIBOR
Chief Executive Officer of General India Life Insurance? stands for ___?
(a) Arvind Mayaram (b) Santosh Hegde (a) Tokyo Interbank Offered Rate
(c) JS Mathur (d) Munish Sharda (b) Taiwan Interbank Offered Rate
(e) None of these (c) Thailand Interbank Offered Rate
8. The Bank of Japan (BOJ) and the Reserve Bank of India (d) Tongling Interbank Offered Rate
(RBI) recently agreed to expand the amount of the Bilateral (e) None of these
Swap Agreement (BSA) to 16. Which one of the following committees was set up by RBI
(a) 15 billion dollars (b) 35 billion dollars to study the various issues relating to financial benchmarks?
(c) 50 billion dollars (d) 75 billion dollars (a) Umesh Chandra (b) Vijaya Bhaskar
(e) None of these (c) Urjit Patel (d) Subbu Rao
(e) None of these
E-30 GENERAL AWARENESS
17. Who among the following has been appointed new 25. Consider the following statements:
chairman of NABARD? 1. The RBI has announced the 'Depositor Education and
(a) Harsh Kumar Bhanwala (b) Chandra Shekar Awareness Fund Scheme, 2014'.
(c) Hemanth Banswal (d) Amith Mishra 2. The Depositor Education and Awareness Fund
(e) None of these Scheme will utilize unclaimed bank deposits for
18. Which of the following foreign banks has been granted education and awareness of depositor.
license to start its banking operation in India, recently? 3. The scheme will be funded by centre for depositor 's
(a) Doha Bank (b) Bank of Japan education.
(c) Barclays Bank (d) Kookmin Bank Which among the above is/ are correct statements?
(e) None of these (a) 1, 2 & 3 (b) 2 & 3
19. Consider the following banks: (c) Only 1, & 2 (d) one of the above
1. ABN Amro Bank 2. Barclays Bank (e) None of these
3. Kookmin Bank 26. Consider the following statements :
Which of the following correctly represents their countries (1) Umesh Chandra committee was set up by RBI to study
of origin? the various issues relating to financial benchmarks.
(a) Dutch, USA, Japan (b) Japan, USA, China (2) The committee has recommended change in
(c) Dutch, UK, South Korea (d) Dutch, USA, China determining money market benchmark.
(e) None of these Which of the statements given above is/are correct?
20. Who is the chairperson of State Bank Of India? (a) 1 only (b) 2 only
(a) Arundhati Bhattacharya (b) Naina Lal Kidwai (c) Both 1 and 2 (d) Neither 1 nor 2
(c) Chanda Kochar (d) Shikha Sharma (e) None of these
(e) None of these 27. Which among the following finance companies has received
21. The World Bank has set a new goal of reducing extreme RBI approvals to set up a minimum of 9,000 white label
poverty to: ATMs (WLAs) in the next three years in rural India?
(a) 9 percent by 2020 (correct Answer) (a) CMS Finvest Ltd.
(b) 6 percent by 2015 (b) SREI Infrastructure Finance Ltd.
(c) 3 percent by 2025 (c) Reliance Capital Ltd.
(d) 0 percent by 2015 (d) Tokyo Finance Ltd.
(e) None of these (e) None of these
22. Consider the following statements in regard to inclusive 28. Which of the following is the largest bank in the world in
development : terms of market capitalization?
(1) As per the RBI guidelines Public Sector Banks has to (a) China Construction Bank
give loans to women SHGs at the interest rate of 7% (b) Industrial & Commercial Bank of China
(2) The initiative is to get benefit of interest rate (c) Wells Fargo & Co
subvention scheme under the Damini scheme (d) Bank of America
Which of the statements given above is/are correct? (e) None of these
(a) 1 only (b) 2 only 29. Recently, RBI has proposed setting up of a trade Receivables
(c) Both 1 and 2 (d) Neither 1 nor 2 and credit exchange (TCE) for financing ____?
(e) None of these (a) Regional Rural Banks
23. Consider the following statements regarding medium term (b) Micro, Small and Medium Enterprises
note (MTN) programme : (c) Public Sector Banks
(1) Medium term note (MTN) is a debt bond which usually (d) Non Banking Financial Corporations
matures in 5 to 10 years (e) None of these
(2) Recently Union Bank of India has hit the global debt 30. Which among the following is the largest foreign bank
market to raise around USD 500 million through bond operating in India in terms of asset base?
issuance (a) Stanchart (b) HSBC
Which of the statements given above is/are correct? (c) Citibank (d) DBS
(a) 1 only (b) 2 only (e) None of these
(c) Both 1 and 2 (d) Neither 1 nor 2 31. Consider the following statements:
(e) None of these 1. The Reserve Bank of India (RBI) on 2 April 2014
24. Consider the following statements : granted in-principle approvals to IDFC Ltd and
(1) Instant Money Transfer (IMT scheme is launched by Bandhan Financial Services Pvt. Ltd to start new banks
Bank of India in India.
(2) It allows withdrawal of funds to individuals from its 2. These approvals have been granted under the
ATMs without an account in the bank guidelines on licensing of new banks in the public
Which of the statements given above is/are correct? sector.
(a) 1 only (b) 2 only 3. The two have been granted the permission to set-up
(c) Both 1 and 2 (d) Neither 1 nor 2 banks from a field of 25 aspirants.
(e) None of these
GENERAL AWARENESS E-31
Which of the statements given above is/are correct? 38. Which of the following constitutes a significant portion of
(a) 1 only (b) 1 and 2 only banks' lending portfolios?
(c) 1 and 3 only (d) 1, 2 and 3 (a) Credit finance (b) Foreign Exchange
(e) None of these (c) Consumer finance (d) Money laundering
32. The Reserve Bank of India on 1 April 2014 adopted the (e) None of these
Consumer Price Index (CPI) as the key measure of Inflation. 39. Consider the following statements:
It was adopted in the first bi-monthly monetary policy 1. The Reserve Bank of India (RBI) has said the
statement for 2014-15. It was adopted on the basis of investment limit in the inflation indexed bonds for
recommendations of which Committee report? individuals has been doubled to Rs 10 lakh.
(a) Urjit R Patel Committee (b) A Ghosh Committee 2. Interest rates on the bonds are linked to Consumer
(c) C Rao Committee (d) Bhagwati Committee Price Index (CPI).
(e) None of these Which of the statements given above is/are correct?
33. Consider the following statements: (a) 1 only (b) 1 and 2
1. Asian Development Outlook 2014 was released on 1 (c) 2 only (d) Either 1 or 2
April 2014 by the Asian Development Bank. (e) None of these
2. According to the ADB Outlook 2014, developing Asia 40. The public sector banks are banks where a majority state is
is expected to extend its steady growth from 6.1 percent
held by the Government. Which of the following banks is
in 2013 to 6.2 percent in 2014 and 6.4 percent in 2015.
the second largest public sector bank among the 236 PSBs
Which of the statements given above is/are correct?
in India in terms of profit?
(a) 1 only (b) 1 and 2
(a) Bank of India (b) Punjab National bank
(c) 2 only (d) None of the above
(c) bank of baroda (d) Central Bank of India
(e) None of these
(e) None of these
34. President Pranab Mukherjee on 30 March 2014 cleared the
41. Consider the following statements:
re-promulgation of the SEBI Ordinance Securities Laws
(Amendment) Bill 2013. The ordinance aims to give powers to 1. RBI in consultation with Union Government on 1 April
1. Securities and Exchanges Board of India (SEBI) to act 2014 capped the Ways and Means Advances (WMA)
against Ponzi scheme. limits for the first half of the new financial year 2014-15
2. The move was to arm the regulator with more stringent (April 2014-Sep 2014) at 35000 crore rupees.
powers comes in the wake to curb the illegal 2. The second half of the limit would be fixed in September
involvement of chit fund companies. 2014.
3. Parliament passed the Securities Laws (Amendment) 3. This notification of Reserve Bank of India (RBI) is aimed
Bill 2013 in the Winter session. at triggering fresh flotation of market loans when the
Consider the above statements and choose the correct government utilises 75 percent of the WMA limit.
option. Consider the above statements and choose the correct
(a) 1 only (b) 1 and 2 only option.
(c) 1 and 3 only (d) 1, 2 and 3 (a) 1 only (b) 1 and 2 only
(e) None of these (c) 1 and 3 only (d) 1, 2 and 3
35. The Reserve Bank of India (RBI) extended the timeline for (e) None of these
full implementation of the Basel III capital regulations by a 42. CSO in its advanced estimates for the year 2013-14 projected
year. Now the banks are required to comply with the Basel the GDP growth rate at constant prices to be-
III norms by 31 March 2019 instead of (a) 4.6% (b) 4.8%
(a) 31 March 2016 (b) 31 March 2018 (c) 4.9% (d) 5.1%
(c) 31 March 2017 (d) 31 March 2015 (e) None of these
(e) None of these 43. At current prices, the GDP growth rate for the year 2013-14,
36. The Purchasing Managers' Index (PMI) is a venture of ___? as projected by CSO stands at-
(a) HDFC (b) HSBC (a) 11.9% (b) 12.3%
(c) ICICI (d) EXIM Bank of India (c) 13.3% (d) 14.5%
(e) None of these (e) None of these
37. Factors responsible for the widespread popularity of 44. RBI decided to withdraw currency notes from the circulation
consumer finance in recent years: Providing access to which were released prior to the year-
purchasing power to the middle class consumer has been (a) 2003 (b) 2004
the most significant achievement of this product class. (c) 2005 (d) 2006
A complainant not satisfied with the decision of the Banking (e) None of these
Ombudsman, has the right to appeal to the_______. 45. As per the status on January 28, 2014 the Marginal Standing
(a) Governor SBP Facility (MSF) stands at-
(b) AVP of the concerned bank (a) 8.5% (b) 8.75%
(c) High Court (c) 9.0% (d) 9.25%
(d) Supreme Court
(e) None of these
(e) None of these
E-32 GENERAL AWARENESS
46. Tele-density in India at end November 2013 has been 57. In the Interim Union Budget 2014-15, out of ` one, how
estimated to be- many paise have been allotted to interest payment-
(a) 73.32 (b) 73.69 (a) 19 (b) 20
(c) 78.41 (d) 79.80 (c) 21 (d) 22
(e) None of these (e) None of these
47. India’s economic freedom score in 2014 stands at 55.7 and 58. In the Interim Union Budget 2014-15, out of ` one, how
on the basis of it, India has been placed at...rank in the list of many paise have been allotted to defence-
Economic Freedom Index 2014. (a) 8 (b) 9
(a) 118th (b) 119th (c) 10 (d) 11
(c) 120th (d) 121th
(e) None of these
(e) None of these
59. Which of the following gives the maximum share in ` one
48. Petroleum, Ministry has raised the cap of subsidized LPG
collection as per the Interim Budget 2014-15?
cylinders-
(a) Income Tax (b) Corporation tax
(a) From 6 to 9 (b) From 6 to 12
(c) From 9 to 12 (d) Made unlimited (c) Custom duty (d) Union excise duty
(e) None of these (e) None of these
49. Which of the following year has registered the maximum 60. The share of Income Tax in ` one collection as per Interim
GDP growth rate? Budget 2014-15 comes to be-
(a) 2005-06 (b) 2006-07 (a) 11 paise (b) 12 paise
(c) 2007-08 (d) 2010-11 (c) 13 paise (d) 14 paise
(e) None of these (e) None of these
50. During the first nine months of the financial year 2013-14, 61. As per the first revised estimates of national income, the
the trade deficit in India stood at- GDP growth rate of Indian Economy slipped to .........in 2012-
(a) 105 billion dollar (b) 108 billion dollar 13.
(c) 110 billion dollar (d) 112 billion dollar (a) 4.0 per cent (b) 4.5 per cent
(e) None of these (c) 3.0 per cent (d) 3.8 per cent
51. During the first nine months of the financial year 2013-14, (e) None of these
India’s exports registered the growth of- 62. As per the first revised estimates of national income for the
(a) + 2.98% (b) + 5.94% year 2012-13, the per capita income at constant prices (2004-
(c) – 4.76% (d) – 5.94% 05) is-
(e) None of these (a) ` 38048 (b) ` 38856
52. During the first nine months of the financial year 2013-14, (c) ` 39178 (d) ` 40126
India’s imports registered the growth of- (e) None of these
(a) – 4.64% (b) – 6.55% 63. Who is the new President of NASSCOM?
(c) + 4.64% (d) + 6.55%
(a) R. Chandrashekher (b) Som Mittal
(e) None of these
(c) Nandan Nelikini (d) None of the above
53. A global organization Transparency International presents
(e) None of these
Corruption Perception Index for 177 countries in the world
64. Who is the first women President of Federal Reserves?
on the basis of their corruption levels. In this list India has
been placed at- (a) Ellen Costello (b) Janet Yellen
(a) 89th rank (b) 92nd rank (c) Karen Peetz (d) Carrie Tolstedt
(c) 94th rank (d) 96th rank (e) None of these
(e) None of these 65. Consider the list-I and list-II and choose the correct code :
54. In the Interim Union Budget 2014-15, the revised estimate of List-I List-II
fiscal deficit for the year 2013-14 has been placed at- (A) C. N. Rao 1. Padma Shree
(a) 4.5% of GDP (b) 4.6% of GDP (B) Raghunath A. Mashelker 2. Padma Bhushan
(c) 4.7% of GDP (d) 4.8% of GDP (C) J. S. Verma 3. Padma Vibhushan
(e) None of these (D) Raskin Bond 4. Bharat Ratna
55. In the Interim Union Budget 2014-15, the budget estimate of (a) (A) ® 4; (B) ® 3; (C) ® 2; (D) ® 1;
fiscal deficit for the year 2014-15 stands at- (b) (A) ® 1; (B) ® 2; (C) ® 3; (D) ® 4;
(a) 4.1% of GDP (b) 4.3% of GDP (c) (A) ® 4; (B) ® 2; (C) ® 3; (D) ® 1;
(c) 4.5% of GDP (d) 4.8% of GDP (d) (A) ® 2; (B) ® 4; (C) ® 3; (D) ® 1;
(e) None of these (e) None of these
56. In the Interim Union Budget 2014-15, the budget estimate of 66. As per the interim budget 2014-15, the interest payment as
revenue deficit for the year 2014-15 stands at- per cent of total tax revenue is-
(a) 3.6% of GDP (b) 3.3% of GDP (a) 46.7% (b) 41.0%
(c) 3.2% of GDP (d) 3.0% of GDP (c) 45.5% (d) 43.3%
(e) None of these (e) None of these
GENERAL AWARENESS E-33
67. As per the interim budget 2014-15, total subsidy bill of the 78. Which is the latest round of NELP unveiled by the
central government is- government for the auction?
(a) ` 255708 crore (b) ` 257079 crore (a) NELP-IX (b) NELP-X
(c) ` 217941 crore (d) ` 250000 crore (c) NELP-XI (d) NEP-XIII
(e) None of these (e) None of these
68. As per the interim railways budget the operationg ratio in 79. In the fortune's updated and latest list of Indian Global 500
2014-15 is projected to- Companies, which stands at the top?
(a) 89.8% (b) 90.8% (a) Reliance industries (b) Indian Oil Corporation
(c) 87.8% (d) 90.2% (c) Bharat Petroleum (d) State Bank of India
(e) None of these (e) None of these
69. As per revised estimates for the year 2013-14, the operating 80. Government has notified the New Gas Price Mechanism,
ratio in Indian Railways stood at- applicable to all domestically produced natural gas for a
(a) 87.8% (b) 89.2% period of five years which has/will become effective since-
(c) 90.8% (d) 92.1% (a) January 1, 2014 (b) February 15, 2014
(e) None of these (c) April 1, 2014 (d) June 1, 2014
70. In the Interim Rail Budget 2014-15, the operational ratio for (e) None of these
the year 2014-15 has been estimated at- 81. Which of the following committee is associated with the
(a) 89.8% (b) 90.6% review of Insider Trading Regulations?
(c) 92.1% (d) 93.5% (a) Sodhi Committee
(e) None of these (b) Sinha Committee
71. In Interim Budget 2014-15, the excise duty on some capital (c) Tarapore Committee
goods and consumer durables has been cut down- (d) Chandrashekhar Committee
(a) From 14 to 12% (b) From 12 to 10% (e) None of these
(c) From 14 to 10% (d) From 10 to 8% 82. World Economic Situation and Prospects Report is
(e) None of these associated with :
72. In Interim Budget 2014-15, the excise duty on small cars, (a) IMF (b) World Bank
two wheelers and commercial vehicles has been cut down (c) United Nations (d) WTO
from- (e) None of these
(a) 12 to 10% (b) 12 to 8% 83. In India's total foreign debt on September 30, 2013, the share
(c) 10 to 8% (d) 10 to 6% of Multilateral Debt stands at
(e) None of these (a) 13.1% (b) 14.1%
73. Credit policy review of RBI on January 28, 2014 made (c) 14.6% (d) 15.1%
alteration in (e) None of these
(a) Repo Rate and CRR 84. Nachiket Mor committee has submitted its recommendations
(b) CRR and SLR which are related to:
(c) Repo Rate and Reverse Repo Rate (a) Insider Trading (b) Financial Inclusion
(d) Only CRR (c) Micro Financing (d) None of the above
(e) None of these (e) None of these
74. The Ministerial Conference of World Trade Organisation 85. Interest rate on EPF deposits of the employees for the year
(WTO) in December 2013 was held in 2013-14 has been declared to be:
(a) 8.25% (b) 8.50%
(a) Japan (b) Mexico
(c) 8.75% (d) 9.00%
(c) Canada (d) Indonesia
(e) None of these
(e) None of these
86. New Nation Youth Policy 2014 includes the youth between
75. Who among the following succeeded Som Mittal as
the age group of
President of NASSCO M on January 5, 2014?
(a) 14-30 years (b) 15-29 years
(a) Subodh Agarwal (b) R. Chandrashekhar
(c) 15-35 years (d) 15-25 years
(c) Sidharth Birla (d) Rana Kapoor
(e) None of these
(e) None of these
87. RBI has introduced Inflation Indexed Bond which have the
76. Who is the newly appointed Managing Director and CEO
locking period of :
of MCX-SX?
(a) 5 years (b) 10 years
(a) Srikant Javlekar (b) Saurabh Sarkar
(c) 15 years (d) 20 years
(c) U.K. Sinha (d) Prateep Chowdhary
(e) None of these
(e) None of these
88. Consider the following statements in regard to money market
77. During the first half on 2013-14 India's foreign trade deficit
in India:
stood at :
(1) It is a market for short-term and long-term funds with
(a) $ 110.042 billion (b) $ 126.231 billion
maturity ranging from overnight to one year.
(c) $ 146.826 billion (d) $ 156.562 billion
(2) It acts as an instrument of liquidity adjustment for the
(e) None of these
Central Bank.
E-34 GENERAL AWARENESS
Which of the statements given above is/are correct? Which of the statements given above is/are correct?
(a) 1 only (b) 2 only (a) 1 only (b) 2 only
(c) Both 1 and 2 (d) Neither 1 nor 2 (c) Both 1 and 2 (d) Neither 1 nor 2
(e) None of these (e) None of these
89. Which of the following can be the outcomes of very high 95. ‘Based III’ norms target at which of the following?
inflation in the economy? (1) Improve the banking sector’s ability to absorb shocks
(1) Reduction in economic growth arising from financial and economic stress.
(2) Increase in savings (2) Improve risk management and governance.
(3) Reduction in exports (3) Strengthen banks’ transparency.
Select the correct answer using the codes below : Choose the correct answer using the codes given below:
(a) 1 and 3 only (b) 3 and 4 only (a) 1 only (b) 1 and 2 only
(c) 2 and 3 (d) 1 and 4 only (c) 1 and 3 only (d) 1, 2 and 3
(e) None of these (e) None of these
90. Which of the following activities can lead to financial 96. Which of the following is not correctly matched?
inclusion in India? (a) Chitra Ramkrishna – National Stock Exchange
(1) Issuing of general purpose credit cards. (b) Chanda Kochar – ICICI Bank
(2) Strict know your customer’ (KYC) norms (c) Naina Lal Kidwai – HSBC
(3) Opening of Bank branches in unbanked rural areas. (d) Shikha Sharma – SBI
(4) Opening of no-frills account. (e) None of these
Select the correct answer using the codes given below : 97. Which of the following definitions are correct?
(i) Basis points: increase in interest rates in percentage
(a) 1, 2 and 3 only (b) 2, 3 and 4 only
terms.
(c) 1, 3 and 4 only (d) 2 and 3 only
(ii) Repo rate: rate at which commercial banks borrow from
(e) None of these
the RBI by selling their securities or financial assets to
91. Consider the following statements :
the RBI for a long-period of time.
(1) Fiat money is a term used for Gold coins
(iii) Reverse repo rate: rate of interest at which the central
(2) Currency Deposit Ratio is the proportion of the total
bank borrows funds from other banks for a short
deposits commercial banks keep as reserves. duration.
Which of the above statements is/are correct? (iv) Cash reserve ratio: minimum percentage of cash
(a) 1 only (b) 2 only deposits that banks must keep with itself to avoid
(c) Both 1 and 2 (d) Neither 1 nor 2 liquidity issues.
(e) None of these (a) (i) & (ii) (b) (ii), (iii) & (iv)
92. When the RBI wants to inject liquidity into economy, it may (c) (ii) & (iv) (d) (iii) & (iv)
adopt the following : (e) None of these
(1) Buy the government securities from the banks. 98. During which plan was the National Bank for Agriculture
(2) Enter into reverse repo operations and Rural Development (NABARD) established to facilitate
(3) Raise cash Reserve Ratio the rural credit and agricultural development?
(4) Reduce SLR. (a) Third Five Year Plan (b) Fifth Five Year Plan
Select the correct answer using the codes given below: (c) Sixth Five Year Plan (d) Eighth Five Year Plan
(a) 1 only (b) 1 and 4 only (e) None of these
(c) 1, 2 and 4 only (d) 1, 2, 3 and 4 99. In India, which of the following have the highest share in
(e) None of these the disbursement of credit to agriculture and allied activities?
93. Which of the following statements is/are correct in regard (a) Commercial Banks (b) Co-operative Banks
to ‘micro-finance’? (c) Regional Rural Banks (d) Microfinance Institutions
(1) Micro-credit extended by banks to individual is (e) None of these
reckoned as a part of their priority sector lending. 100. Choose the correct statements in the context of Cooperative
(2) RBI has prescribed a particular model the banks to Banks in India.
provide micro-finance. 1. Cooperative Banks operate on no profit no loss basis.
Choose the correct answer using the codes given below: 2. Cooperative Banks are allowed to operate only in the
(a) 1 only (b) 2 only agriculture sector.
(c) Both 1 and 2 (d) Neither 1 nor 2 3. NABARD is a Cooperative Bank.
(e) None of these (a) 1 only (b) 1 and 2
94. Consider the following statements in regard to ‘marginal (c) 1 and 3 (d) Either 1 or 2
standing facility (MSF)’ of RBI : (e) None of these
(1) It will help in reducing volatility in the overnight lending 101. Consider the following statements :
rates in the inter-bank market. 1. Regional Rural Banks grant direct loans and advances
(2) The borrowing under the MSF should be over and to marginal farmers and rural artisans.
above the statutory liquidity requirement. 2. NABARD is responsible for laying down policies and
to oversee the operations of the RRBs.
GENERAL AWARENESS E-35
Which of the statements given above is/are correct? 107. Under which of the following circumstances Reserve Bank
(a) 1 only (b) 2 only of India (RBI) raised the Repo rate under Liquidity
(c) Both 1 and 2 (d) Neither 1 nor 2 Adjustment Facility (LAF) from 7.75 to 8 per cent in January
(e) None of these 2014?
102. Which one of the following agencies is not included in the 1. Global uncertainty continues to surround the prospects
operation of the Kisan Credit Cards? for some emerging economies.
(a) Scheduled Commercial Banks
2. Domestic fragilities getting accentuated.
(b) Co-operative Banks
(c) Regional Rural Banks 3. Financial market contagion is a clear potential risk.
(d) NABARD 4. The CPI inflation is expected to be on the upside risk .
(e) None of these Select the answer from the codes given below:
103. The Reserve Bank of India, released its June Mid-quarter Codes:
Monetary Policy. (a) 1, 2 and 3 (b) 2, 3 and 4
Consider the following statements: (c) 2, 3 and 4 (d) All of the above
1. RBI left its key policy, repo rate unchanged at 7.25% in
line. (e) None of these
2. CRR (Cash Reserve Ratio), remained at 4%. 108. The RBI Expert Committee to revise and strengthen the
3. Repo is the rate at which banks borrow from the central monetary policy framework is headed by
bank. (a) Dr. Urjit R. Patel Committee
Select the correct answer using codes given below: (b) Suresh Mathur Panel
Codes: (c) Vijay Kelkar Committee
(a) 1 and 2 only (b) 2 and 3 only (d) Shah Nawaz Committee
(c) 1 and 3 only (d) 1, 2 and 3
(e) None of these
(e) None of these
104. Consider the following statements about Bharitiya Mahila 109. RBI appointed a Committee to Review Governance of Boards
Bank: of Banks in India chaired by P.J. Nayak would
1. The Proposed Bank will be headquartered in Mumbai. 1. review the regulatory compliance require-ments of
2. It will start 6 branches in Mumbai, Delhi, Kolkata, banks' boards in India.
Chennai, Indore and Guwahati. 2. judge what can be rationalised and where requirements
Select the correct answer using codes given below: need to be enhanced.
Codes:
3. examine the working of banks’ boards.
(a) 1 only (b) 2 only
4. analyse the representation on banks’ boards.
(c) Both 1 and 2 (d) Either 1 or 2
(e) None of these Select the answer from the codes given below:
105. Match the followings: Codes:
List-I List-II (a) 1, 2 and 3 (b) 2, 3 and 4
Designation Chairperson/President (c) 1, 3 and 4 (d) All of the above
A. ASSOCHAM 1. Krishna Kumar (e) None of these
Natarajan 110. Which of the following are the recommendations of the RBI
B. FICCI 2. K. R. Kamath
Expert Committee to revise and strengthen the monetary
C. NASSCOM 3. Rana Kapoor
policy framework?
D. Indian Bank 4. Naina Lal Kidwai
Association (IBA) 1. It recommended that a new Consumer Price Index (CPI)
Select the correct answer using codes given below: should be adopted by Reserve Bank of India (RBI) to
Codes: anchor the monetary policy.
A B C D A B C D 2. The committee has also set an inflation target at 4
(a) 4 3 2 1 (b) 3 4 1 2 percent with a band of plus/minus 2 percent around it.
(c) 2 1 3 4 (d) 1 2 4 3 3. The monetary policy decision should be vested in the
106. Core Banking Solution(CBS) provides: hands of the Monetary Policy Committee (MPC) that
1. multiple delivery channels to the customers. will be headed by the Governor.
2. better MIS and reporting to external agencies such as 4. Not to detach Open Market Operations (OMOs) from
government, RBI, etc. the fiscal operations and instead linked solely to the
3. better asset liability management and risk management liquidity management.
by banks.
Select the answer from the codes given below:
Select the answer from the codes given below:
Codes: Codes:
(a) 1 and 2 (b) 2 and 3 (a) 1, 2 and 3 (b) 2, 3 and 4
(c) 1 and 3 (d) All of the above (c) 1, 3 and 4 (d) All of the above
(e) None of these (e) None of these
E-36 GENERAL AWARENESS
111. Which of the following organizations is selected by the RBI Choose the correct code from the options given below:
for issuing globally compatible Legal Entity Identifiers (a) 1 and 2 only (b) 1 and 3 only
(LEIs)? (c) 2 and 3 only (d) 1, 2 and 3
(a) Industrial Credit and Investment Corporation of India (e) None of these
(ICICI ) 116. RBI increased the validity period of the in-principle approval
(b) Clearing Corporation of India Ltd (CCIL) of setting up of new banks from one year to
(c) Housing Development Finance Corporation (HDFC) (a) 14 months (b) 16 months
(d) Security and Exchange Board of India (SEBI) (c) 18 months (d) 20 months
(e) None of these (e) None of these
112. The primary objective of the European Central Bank, as 117. Name the bank that purchased the HSBC bank's Swiss private
mandated in Article 2 of the Statute of the Bank, is banking assets?
(a) to define and implement the monetary policy for the (a) Six SIS AG (b) Zurich Cantonal Bank
Eurozone (c) LGT Bank (d) WIR Bank
(b) to conduct foreign exchange operations (e) None of these
(c) to maintain price stability within the Eurozone. 118. RBI constituted an expert committee for examining its current
(d) to take care of the foreign reserves of the European monetary policy framework. Who is the chairman of the
System of Central Banks and operation of the financial committee?
markets. (a) Dr. Urjit Patel (b) H R Khan
(e) None of these (c) Dr Anand Sinha (d) K C Chakrabarty
113. Reserve Bank of India has decided to withdraw all currency (e) None of these
notes printed before 2005. It is being done: 119. RBI reduced the Marginal Standing Facility (MSF) Rate to
(a) to detect, weed out and estimate the quantum of fake 9% from
currency in India. (a) 9.5% (b) 9.3%
(b) to unearth the black money in the country. (c) 9.8% (d) 9.9%
(c) for no stated reason.
(e) None of these
(d) to issue plastic currency in near future.
120. Who is the first chairperson and managing director of public
(e) None of these
sector Bharatiya Mahila Bank (BMB)?
114. Consider the following facts about Bhartiya Mahila Bank : (a) Shilpa Phadnis
1. BMB was inaugurated on 19th November, 2013, the (b) Usha Anantha Subramanian
birth anniversary of Late Indira Gandhi. (c) Reeba Zachariah
2. Usha Ananthasubramanian is the founder Chairperson (d) Sudha Misra
of the BMB.
(e) None of these
3. BMB offered 4.5% interest rate on savings bank
deposits upto ` 1 lakh and 5% on deposits above ` 1 121. Which is not a source of Agriculture finance in India?
lakh. (a) Co-operative societies
The correct codes are (b) Commercial Banks
(a) 1 only (b) 1 and 2 only (c) Regional Rural Banks
(c) 2 and 3 only (d) All of these (d) Central Rural Bank
(e) None of these
(e) None of these
122. In India ‘Report on Currency and Finance’ is the annual
115. Which of the following statements regarding New publication of
Development Bank to be established by BRICS nations are (a) SEBI
correct? (b) RBI
(1) The headquarter of the bank will be in Shanghai (c) Finance Commission
(2) It will provide financial assistance to infrastructure (d) Finance Ministry
projects of BRICS nations only (e) None of these
(3) The proposal for the bank was first made in 2012
summit in New Delhi.
GENERAL AWARENESS E-37

ANSWER KEY
1 (c) 16 (b) 31 (c) 46 (b) 61 (a) 76 (c) 91 (d) 106 (d) 121 (d)
2 (c) 17 (a) 32 (a) 47 (c) 62 (b) 77 (a) 92 (b) 107 (c) 122 (b)
3 (c) 18 (a) 33 (b) 48 (c) 63 (a) 78 (b) 93 (a) 108 (a)
4 (b) 19 (c) 34 (b) 49 (b) 64 (b) 79 (b) 94 (a) 109 (d)
5 (c) 20 (a) 35 (b) 50 (c) 65 (a) 80 (c) 95 (d) 110 (a)
6 (b) 21 (a) 36 (b) 51 (b) 66 (d) 81 (a) 96 (d) 111 (b)
7 (d) 22 (a) 37 (a) 52 (b) 67 (a) 82 (c) 97 (c) 112 (c)
8 (c) 23 (c) 38 (c) 53 (c) 68 (a) 83 (a) 98 (c) 113 (a)
9 (b) 24 (c) 39 (b) 54 (b) 69 (c) 84 (b) 99 (a) 114 (a)
10 (d) 25 (c) 40 (c) 55 (a) 70 (a) 85 (c) 100 (a) 115 (b)
11 (b) 26 (b) 41 (d) 56 (b) 71 (b) 86 (b) 101 (c) 116 (c)
12 (d) 27 (b) 42 (c) 57 (c) 72 (b) 87 (b) 102 (d) 117 (c)
13 (d) 28 (b) 43 (b) 58 (c) 73 (c) 88 (b) 103 (d) 118 (a)
14 (d) 29 (b) 44 (c) 59 (b) 74 (d) 89 (d) 104 (b) 119 (a)
15 (a) 30 (c) 45 (c) 60 (d) 75 (b) 90 (c) 105 (b) 120 (b)

Answers &
Explanations
88. (b) The money market consists of financial institutions 92. (b) By Buying the government securities from the banks
and dealers in money or credit who wish to either and reducing SLR may inject money into the system.
borrow or lend. Participants borrow and lend for short However Raising cash Reserve Ratio may take away
periods of time, typically up to thirteen months. Money liquidity from the market as the banks will have to
market trades in short-term financial instruments deposit more money with the RBI and similarly entering
commonly called “paper.” the reverse repo operations may also take away
89. (d) inflation is a persistent increase in the general price liquidity from the market.
level of goods and services in an economy over a period 93. (a) Micro-credit extended by banks to individuals is
of time . When the general price level rises, each unit reckoned as a part of their priority sector lending and
of curren cy buys fewer goods and services. no particular model has been prescr ibed for
Consequently, inflation reflects a reduction in the microfinance and banks have been extended freedom
purchasing power per unit of money – a loss of real to formulate their own models.
value .High inflation means excessive supply of money 94. (a) Marginal Standing Facility (MSF) is the rate at which
and thus leads to rise in the cost of credit and interest scheduled banks could borrow funds overnight from
rates. Higher inflation leads to reduction in economic the Reserve Bank of India (RBI) against approved
growth, decrease in cost of credit, increase in spending government securities. Banks can borrow funds
rather than saving as the value of money is declining. through MSF during acute cash shortage (considerable
90. (c) Financial inclusion or inclusive financing is the delivery shortfall of liquidity). This measure has been introduced
of financial services at affordable costs to sections of by RBI to regulate short-term asset liability mismatch
disadvantaged and low-income segments of society. more effectively and the borrowing is within Statutory
For financial inclusion ‘Know your customer’ norms liquidity requirements.
should be relaxed and no frills account should be The Marginal Standing Facility (MSF) is pegged
opened for low income segments which are looking for 100 bps or 1 % above the Repo Rate.
basic banking only . Along with general purpose credit 95. (d) Basel III (or the Third Basel Accord) is a global,
cards should be issued and bank branches should be voluntary regulatory standard on bank capital
opened in unbanked rural areas. adequacy, stress testing and market liquidity risk. So
91. (d) Currency notes and coins are called fiat money. They all the given statements are correct.
don’t have intrinsic value like a gold or silver coin. The 96. (d)
currency-deposit ratio measures the relationship 97. (c) Basis points: It is the increase in interest rates in
between the cash people have on hand and what they percentage terms. For instance, if the interest rate
have in their accounts. increases by 50 basis points (bsp), then it means that
E-38 GENERAL AWARENESS

interest rate has been increase by 50%. One percentage the credit flow for elevation of agriculture and rural
point is broken down into 100 basis points. Therefore, non-farm sector and laying down policies and to
an increase from 2% to 3% is an increase of one oversee the operations of the RRBs Moreover Regional
percentage point or 100 basis points. Rural Banks grant direct loans and advances to marginal
Repo rate: Repo rate is the policy rate and is part of farmers and rural artisans. So both statements are
RBI’s Liquidity Adjustment Facility (LAF). It is the rate correct.
at which commercial banks borrow from the RBI by 102. (d) A Kisan Credit Card is a credit card to provide affordable
selling their securities or financial assets to the RBI for credit for farmers in India. It was started by the
a short-period of time. It comes with an agreement that Government of India, Reserve Bank of India (RBI), and
the sold securities will be repurchased by the National Bank for Agricultural and Rural Development
commercial banks from the RBI at a future date at (NABARD) in 1998-99 to help farmers access timely
predetermined price. The repo rate is used by the central and adequate credit. The farmers can use cards to
bank to increase liquidity in the system. Reverse repo withdraw money from most of the banks like commercial
rate: Reverse Repo Rate is also a part of LAF. It is the bank,cooperative bank and regional rural banks.
rate of interest at which the central bank borrows funds However in day-to-day operations NABARD is not
from other banks for a short duration. The banks included.
deposit their short term excess funds with the central 103. (d) According to June-Mid-quarter Monetary Policy of
bank and earn interest on it. This rate is used by the Reserve Bank of India its key policy repo rate, remained
central bank to absorb liquidity from the economy. unchanged at 7.25% in line. Repo is the rate at which
Generally it is one percentage less than the Repo rate. banks borrow from the central bank. CRR (Cash Reserve
Bank rate: The only way the bank rate is different from Ratio) remained at 4%.
the repo rate is that the bank rate is the rate at which 104. (b) Bharitiya Mahila Bank will be head quartered in New
banks borrow money from the central bank without Delhi. It will start 6 branches in Mumbai, Delhi, Kolkata,
any sale of securities. It is generally for a longer period Chennai, Indore and Guwahati.
of time. Cash reserve ratio: CRR is the minimum 105. (b) ASSOCHAM - Rana Kapoor
percentage of cash deposits that banks must keep with FICCI - Naina Lal Kidwai
the central bank. The current rate is 4%, which means NASSCOM - Krishna Kumar Natarajan
for a cash deposit of Rs. 100, the bank has to park 4 IBA - K R Kamath
rupee with the central bank. 106. (d) The CBS also provides quick and safe transfer of funds,
98. (c) National Bank for Agriculture and Rural Development low transaction cost, and anytime anywhere banking
(NABARD) is an apex development bank in India facilities. Nowadays, most banks use core banking
having headquarters in Mumbai. It was established on applications to support their operations where CORE
12 July 1982 in sixth five year plan by a special act by Banking stands for “centralized online real-time
the parliament and its main focus was to uplift rural electronic banking”. This basically means that all the
India by increasing the credit flow for elevation of bank’s branches access applications from centralized
agriculture and rural non-farm sector. datacenters. This means that the deposits made are
99. (a) In India, commercial banks have the highest share in reflected immediately on the bank’s servers and the
the disbursement of credit to agriculture and allied customer can withdraw the deposited money from any
activities.The commercial banks disburse around 60% of the bank’s branches throughout the world. These
credit followed by cooperative banks around 30% and applications now also have the capability to address
RRB and others. the needs of corporate customers, providing a
100. (a) Cooperative Banks operate on no profit no loss basis, comprehensive banking solution.
and they operate in all sectors including agriculture 107. (c) Also, domestically, some loss of momentum of growth
sector. NABARD is not a Cooperative Bank National is likely. Importantly, following the recommendation of
Ban k for Agriculture and Rural Development the Dr. Urjit Patel Committee, monetary policy reviews
(NABARD) is an apex development bank in India. will ordinarily be undertaken in a two-monthly cycle,
101. (c) National Bank for Agriculture and Rural Development consistent with the availability of key macroeconomic
(NABARD) is an apex development bank in India . It and financial data.
was established on 12 July 1982 in sixth five year plan 108. (a) Expert Committee to revise and strengthen the
and its main focus was to uplift rural India by increasing monetary policy framework submitted its report to the
GENERAL AWARENESS E-39

Governor of RBI on 21 January 2013. The committee a financial transaction.Clearing Corporation of India
was headed by Dr. Urjit R Patel, the Deputy Governor Ltd (CCIL) will issue unique identifier codes to the
of RBI and was constituted on 12 September 2013 by eligible legal entities participating in financial markets
the Governor Dr. Raghuram G Rajan. It has been done across the globe on a non-profit cost recovery basis.
to make it transparent and predictable. CCIL is recognised by the Regulatory Oversight
109. (d) The committee would examine the working of banks’ Committee of the global LEI system and the unique
boards including whether adequate time is devoted to identity codes issued by it will be accepted globally.
issues of strategy, growth, governance and risk Once the infrastructure is set up, the LEI numbers will
management and will also review central bank regulatory be mandatory for Over-the-Counter (OTC) derivative
guidelines on bank ownership, ownership transactions. The implementation of the global LEI
concentration and representation in the board. It would system is led by the Financial Stability Board (FSB).
analyse the representation on banks’ boards to see The functioning of CCIL as LEI will be under the
whether the boards have the appropriate mix of regulation and oversight of the Reserve Bank of India.
capabilities and the necessary independence to govern 112. (c) The primary objective of the European Central Bank,
the institution, and to investigate possible conflicts of as mandated in Article 2 of the Statute of the Bank, is
interest in board representation, including among to maintain price stability within the Eurozone. The
owner representatives and regulators. basic tasks, as defined in Article 3 of the Statute, are to
110. (a) The committee has suggested detachment of Open define and implement the monetary policy for the
Market Operations (OMOs) from the fiscal operations Eurozone, to conduct foreign exchange operations, to
and instead linked solely to the liquidity management. take care of the foreign reserves of the European
OMOs should not be used for managing yields on System of Central Banks and operation of the financial
government securities.These recommendations are market infrastructure under the TARGET2 payments
intended to better ground inflation expectations by system and the technical platform (currently being
making it clear that inflation is the RBI’s primary developed) for settlement of securities in Europe
objective. It also expects to be held accountable for its (TARGET2 Securities).
performance in this regard. As per the suggestions the 115. (b) New Development Bank will provide financial assis-
government also needs to ensure the fiscal deficit as a tance to developing and emerging economies mainly
ratio of GDP should be brought down to 3 percent by for infrastructure projects.
2016-17 which should be consistent with the Fiscal 120. (b) The union government appointed Usha Subramaniam
Responsibility and Budget Management (Amendment) as the first chairperson and managing director of
Rules 2013. In its suggestion it has suggested two Bhartiya Mahila Bank.
schemes namely Market Stabilisation Scheme (MSS) 121. (d) Co-operative societies, Commercial Banks and Re-
and Cash Management Bills (CMBs) may be phased gional Rural Banks are source of agriculture finance in
out and the government debt and cash management India. They provide finance under various schemes
should be taken over by the Debt Management Office run by central government and state governments to
of the government. It has also suggested that all fixed purchase seeds, implements, fertilizer, pesticides etc.
income financial products should be treated on par 122. (b) In India, ‘Report on Currency and Finance’ is the an-
with the bank deposits for the purposes of taxation nual publication of Reserve Bank of India.The report
and TDS. highlights the evaluation or devaluation of rupee as a
111. (b) Legal Entity Identifiers (LEI) are a 20-character unique currency along with it other aspects and projects the
identity code assigned to entities which are parties to financial condition of the economy.
E-40 GENERAL AWARENESS

EXERCISE
4. CURRENT AFFAIRS
1. Recently, which among the following countries has 9. Who among the following has become the first woman
overtaken India to become the largest contributor of FDI to director of "National Police Academy"?
Nepal? (a) Aruna Bahuguna (b) Shwetha Bhandari
(a) China (b) Russia (c) Chandrika Kumari (d) Niveditha Singh
(c) USA (d) Sri Lanka (e) None of these
(e) None of these 10. Park Geun-hye, who visited India recently, is a president of
2. What is India's ranking in the 2014 Environmental which country?
Performance Index released, recently? (a) South Korea (b) South Africa
(a) 140 (b) 155 (c) North Korea (d) Singapore
(c) 165 (d) 168 (e) None of these
(e) None of these 11. The "Sunil Gangopadhyay Memorial Award" is given for
3. Who among the following Indians have been awarded the excellence in ___?
2014 Marconi Prize, a Nobel equivalent for technology (a) Telugu (b) Bengali
engineers? (c) Odisi (d) Rajasthani
(a) A Joseph Paulraj (b) Naveen Natarajan (e) None of these
(c) Sumitraraj Gupta (d) Jagadish Singh 12. As per the YouGov poll survey, who among the following
(e) None of these has been named as the most admired person in world?
4. Recently, Aravind Mayaram headed panel has recommended (a) Sachin Tendulkar (b) Bill Gates
all investment above __ percent as foreign direct investment (c) Barack Obama (d) Narendra Modi
(FDI)?
(e) None of these
(a) 15 (b) 10
13. Who among the following has won the FIFA Ballon d' Or
(c) 20 (d) 25 award for 2013?
(e) None of these
(a) Lionel Messi (b) Cristiano Ronaldo
5. MINT is a newly developed acronym for group of countries
(c) Franck Ribery (d) Manuel Neuer
which are promising investment destination. Which of the
(e) None of these
following correctly represents MINT countries?
14. Recently, which of the following has become the first "5S"
(a) Myanmar, India, Nepal and Thailand
certified public sector enterprises in the steel industry?
(b) Mexico, Indonesia, Nigeria and Turkey
(a) SAIL (b) RINL
(c) Malaysia, Indonesia, Nigeria and Thailand
(c) MECON (d) NMDC
(d) Mexico, India, Nepal and Turkey
(e) None of these
(e) None of these
15. Which among the following countries has claimed that it
6. Which one of the following has been named "Leader in
overtook the United States as the world's top trading nation,
Worldwide Life Science Manufacturing and Supply Chain
recently?
ITO", recently?
(a) Japan (b) Russia
(a) Infosys (b) Wipro
(c) TCS (d) IBM (c) China (d) Australia
(e) None of these (e) None of these
7. Which among the following has been named "destination 16. As per the Index of Economic Freedom 2014, which among
flyways" by United Nation World Tourism Organisation the following is the country / territory with highest economic
(UNWTO), recently? freedom?
(a) Sunder bans Delta (b) Chilika Lagoon (a) Hong Kong (b) Switzerland
(c) Western Ghats (d) Astamudi Lake (c) Singapore (d) Australia
(e) None of these (e) None of these
8. Which of the following has been declared as the worst 17. Currently, which among the following nations holding the
password on the Internet for the year 2013? number one position in the FIFA ranking?
(a) password (b) pass123 (a) Spain (b) Portugal
(c) 123456 (d) pass 123456 (c) Brazil (d) Egypt
(e) None of these (e) None of these
GENERAL AWARENESS E-41
18. Which among the following organisations has launched 24. Recently the Cabinet Committee on Economic Affairs (CCEA)
"No Lost Generation" project for Syrian children, recently? approved the continuation of the Indian Leather
(a) United Nations (b) European Union Development Programme (ILDP) in the 12th Plan. Which
(c) ASEAN (d) SAARC one of the following sub-schemes has been included under
(e) None of these the ILDP?
19. Which one of the following mentioned below is/are feature (a) Integrated Development of Leather Sector
of Lokpal and Lokayuktas Act, 2013? (b) Support to Artisan
I. To set up the institution of Lokpal at the central level (c) Leather Technology Innovation
and Lokayukta's at the State level. (d) All of the above
II. Lokpal will consist of a Chairperson and a maximum of (e) None of these
eight members, of which 50 percent should be judicial
25. The Cabinet Committee on Economic Affairs (CCEA) on 2
members.
January 2014 gave its nod for the implementation of the
III. 50 percent of the total members of the Lokpal should
Rural Water Supply and Sanitation Project for Low Income
be from SC/ST/OBCs, minorities and women.
States. The States included are
IV. The Prime Minister has been brought under the
(a) Orissa (b) Bihar
purview of the Lokpal.
(c) Jharkhand (d) Only (b) and (c)
(a) I, II and III (b) I, II and IV
(e) None of these
(c) II, III and IV (d) All of the above
(e) None of these 26. Which Power Grid of the country has been synchronously
connected with the grids of rest of India on 31 December
20. Which country became the 18th member of the Euro Zone?
2013 to successfully accomplish the mission of 'One Nation
(a) Lithuania (b) Syria
- One Grid - One Frequency?
(c) Estonia (d) Latvia
(a) Southern Power Grid
(e) None of these
(b) North-Eastern Power Grid
21. The Cabinet Committee on Economic Affairs (CCEA) on 2
January 2014 approved a proposal for setting-up of CCTVs (c) Western Power Grid
and GPS devices on public transport vehicles for women (d) Central Power Grid
safety in how many cities? (e) None of these
(a) 32 (b) 60 27. Who has been elected as Speaker of the Delhi State
(c) 50 (d) 12 Legislative Assembly?
(e) None of these (a) MS Dhir (b) Baban Rao Pajpute
22. Prime Minister of India, Manmohan Singh on 4 January 2014 (c) Vilas Rao Deshmukh (d) Manish Shisodia
launched the total E-literacy project in (e) None of these
(a) Hyderabad (b) Thiruvanathpuram 28. Who became the first male junior badminton player from
(c) Cochin (d) Bangalore India to be ranked top in the world?
(e) None of these (a) Aditya Joshi from Madhya Pradesh
23. The Cabinet Committee on Economic Affairs approved (b) Rakesh johar from Maharashtra
amendments in the Mega Power Policy 2009 on 2 December (c) Deepak Thirkey from Jharkhand
2014. The amendments were aimed at clearing the road
(d) Ramohar Manhani from Delhi
blocks in the realization of provisional Mega Power projects.
Which one of the following is not one of the amendments to (e) None of these
the Mega Power Policy 2009? 29. Name the programme launched by United Nations and its
(a) The developer must tie up at least 65 percent of the humanitarian partners on 10 January 2014 to save millions
installed capacity through competitive bidding. of Syrian Children from losing generation amidst the civil
(b) 35 percent of the installed capacity to be allocated under war in the country.
regulated tariff as per the guidelines issued by the (a) Champions of the Children of Syria
Central government from time to time. (b) No Lost Generation initiative
(c) The thermal power projects of 1000 MW and above (c) Global Education First Initiative
capacity and hydro power projects of 500 MW and (d) Building a peaceful society through education
above are allowed duty-free equipment imports. (e) None of these
(d) Project developers must submit status certificate of 30. Which country on 9 January 2014 invited Indian Institute
the provisional mega power projects along with fixed Technology (IITs) to open its branch?
deposit receipt from any scheduled bank as a security (a) Singapore (b) Japan
for a term of 36 months.
(c) Malaysia (d) South Korea
(e) None of these
(e) None of these
E-42 GENERAL AWARENESS
31. The Supreme Court of India on 13 January 2014 directed the 39. The book Sangharsh ke Safar ka Nayak Mulayam has been
Government of India to place the report of a Commission on written by
illegal mining in Odisha and Jharkhand. Name the commission (a) Akhilesh Yadav (b) Mulayam Singh Yadav
whose report has been asked to be presented? (c) Omveer Tomar (d) Harender Tomar
(a) Vijay Kelkar Commission (e) None of these
(b) Narendran Commission 40. Which State on 13 January 2014 withdrew the decision of
(c) Mukherjee Commission the previous government allowing FDI in Multi-brand
(d) Justice Shah Commission Retail?
(e) None of these (a) Tamil Nadu (b) Delhi
32. Pravasi Bharatiya Divas (PBD) is celebrated on which day (c) West Bengal (d) Rajasthan
every year to mark the contribution of Overseas Indian (e) None of these
community in the development of India. 41. Name the country that won the final of Hero Hockey World
(a) 8 January (b) 9 January League 2014
(c) 10 January (d) 11 January (a) New Zealand (b) Argentina
(e) None of these (c) Spain (d) Netherlands
33. Who was the first Indian to become the Commander-in- (e) None of these
Chief of Independent Indian Army? 42. Supreme Court (SC) of India on 20 January 2014 directed the
(a) Satyawant Mallannah Shrinagesh National Commission for Scheduled Tribes (NCST) to verify
(b) K M Cariappa the authenticity of ST status of the Chief Minister of
(c) Pran Nath Thapar Meghalaya. Name the Chief Minister of Meghalaya?
(a) Donkupar Roy (b) Mukul Sangma
(d) Paramasiva Prabhakar Kumaramangalam
(c) D.D. Lapang (d) J. D. Rymbai
(e) None of these
(e) None of these
34. The charge of Indian Army was given to an Indian after
43. Government of India has granted the minority status to a
independence of India. Who was the last Englishman to
community on 20 January 2014. Name the community?
lead the Indian Army as the Commander-in-Chief?
(a) Jain (b) Sikh
(a) Francis Robert Roy Bucher
(c) Muslims (d) Buddhists
(b) Robert McGregor Macdonald Lockhart
(e) None of these
(c) Sunith Francis Rodrigues 44. Name the country that is likely to join the World Trade
(d) Sam Hormusji Framji Organization (WTO) within the next three months?
(e) None of these (a) Syria (b) Iran
35. The director of Central Bureau of Investigation (CBI) on 15 (c) Iraq (d) Afghanistan
January 2014 announced to set-up a specialized unit to (e) None of these
handle the cases related to sports fraud, fixing and illegal 45. Who will take charge as the New Indian Army chief?
batting. Who is the director of CBI?
(a) Dalbir Singh Suhag (b) Bikram Singh
(a) Amar Pratap Singh (b) Ashwani Kumar
(c) Shekhar Sinha (d) R.K. Dhawan
(c) Vijay Shanker Tiwari (d) Ranjit Sinha
(e) None of these
(e) None of these
36. The World Bank in its Global Economic Prospects Report 46. National Girl Child Day was observed across India by the
released on 15 January 2014 forecasted that economy of Union Government on
India by (a) 24 January (b) 25 January
(c) 27 January (d) 21 January
(a) 4 percent (b) 5 percent
(e) None of these
(c) 6 percent (d) 8 percent
47. A country has unanimously amended an article of the penal
(e) None of these
code that allowed rapists of underage girls to avoid
37. Name the bank that was included in the second schedule of
prosecution by marrying their victims. Name the country?
the Reserve Bank of India (RBI) Act, 1934 on 21 May 2014.
(a) Morocco (b) Israel
(a) Neelachal Gramya Bank (c) India (d) China
(b) Purvanchal Gramin Bank (e) None of these
(c) Chikmagalur Kodagu Grameena Bank 48. India and Pakistan signed an agreement on Non
(d) Bharatiya Mahila Bank Discriminatory Market Access (NDMA) in place of Most
(e) None of these Favoured Nation Status on 20 January 2014. In this context,
38. What is the rank of India in the recently released Index of consider the following statements. I. The issue of bank
Economic Freedom published by Wall Street Journal? license is covered under the NDMA. II. The agreement on
(a) 101 (b) 111 NDMA was signed in accordance to the provisions of WTO.
(c) 120 (d) 121 III. It was signed at the 5th SAARC Business Conclave held
(e) None of these in New Delhi. Select the correct code
GENERAL AWARENESS E-43
(a) I and II (b) II and III 58. Which of the following countries has recently assumed the
(c) I and III (d) All of the above coveted chair of the ASEAN?
(e) None of these (a) Myanmar (b) Indonesia
49. An Expert Committee to review the institutional framework (c) Cambodia (d) Philippines
of Prasar Bharti submitted its report to the Union Ministry (e) None of these
of Information and Broadcasting on 25 Janauary 2014. The 59. The United Nation has recently declared March 3rd as ___?
Committee was headed by (a) World Rivers Day (b) World Wildlife Day
(a) Nachiket Mor (b) Sam Pitroda (c) World Nutrition Day (d) World Poverty Day
(c) T Chandrasekhar (d) Justice Soumitra Sen (e) None of these
(e) None of these 60. The BCCI's lifetime achievement award for the year 2013 will
50. The Union Aviation Ministry removed the chairperson of be conferred upon___?
the Airports Authority of India on 24 January 2014. Name (a) Mohammad Azharuddin (b) Kapil Dev
the person. (c) Sunil Gavaskar (d) Sachin Tendulkar
(a) Ashok Reddy (b) Arjun Paul (e) None of these
(c) V P Agarwal (d) Alok Sinha 61. Consider the following Sahitya Akademi Award winners of
(e) None of these the year 2013:
51. NASA Mars Rover Opportunity found that Mars had life 1. Javed Akhtar 2. Ambika Dutt
friendly fresh water once. The evidence of water was found 3. Subodh Sarkar 4. Joe D'Cruz
on the crater Which of the following correctly represents the languages
(a) Endeavour (b) Gale they chosen for the award?
(c) Mount Olympus (d) Curiosity (a) Urdu, Tamil, Bengali, Sindhi
(e) None of these (b) Hindi, Tamil, Sindhi, Rajasthani
52. The Best Actor Award at the 20th Screen Awards was won (c) Urdu, Rajasthani, Bengali, Tamil
by (d) Tamil, Urdu, Bengali, Hindi
(a) Shah Rukh Khan (b) Ranveer Singh (e) None of these
(c) Farhan Akhtar (d) Irfaan Khan 62. Which one of the following is the main aim of the recently
(e) None of these launched "E-Inclusion" project?
53. The Iranian Nuclear deal also called Geneva deal agreed (a) To provide core bank facility to all post offices
among P5 + 1 nations in Geneva came into effect on (b) To achieve 100 per cent financial inclusion
(a) 22 January 2014 (b) 20 January 2014 (c) To provide IT training for Rural/SC/ST women
(c) 25 January 2014 (d) 26 January 2014 beneficiaries
(e) None of these (d) To establish ATM kiosks at grama panchayat level
54. The Union Ministry of Petroleum & Natural Gas launched (e) None of these
LPG Connection Portability Scheme across the country on 63. Who among the following has been elected as President of
22 January 2014. The Scheme covers Federation of Indian Chambers of Commerce and Industry
(a) 600 districts (b) 550 districts (FICCI)?
(c) 480 districts (d) 520 districts (a) Jyotsna Suri (b) Sidharth Birla
55. Who among the following is the new chairman of (c) Kumar Mangalam (d) Shekar Chandran
NASSCOM? (e) None of these
(a) R Chandrashekar (b) Naveen Patnaik 64. Who among the following has been voted the BBC sports
(c) Kushal Mittal (d) Aditya Birla personality of 2013?
(e) None of these (a) Andy Murray (b) Novak Djokovic
56. In which one of the following states, Tata Consultancy (c) Usain Bolt (d) Rafel Nadal
Services (TCS) has decided to set up world's largest (e) None of these
corporate learning and development centre? 65. Which of the following is/are the major objectives of
(a) Kerala (b) Punjab MGNREGA?
(c) Maharashtra (d) Karnataka 1. Wage employment
(e) None of these 2. Create durable community assets
57. Which among the following has created the India's first 3. Empower gram panchayats
most comprehensive "Financial Inclusion Index", recently? Choose the correct answer from the codes given below:
(a) RBI (b) CRISIL (a) Only 1 (b) Only 1 & 2
(c) ICRA (d) CIBIL (c) Only 1 & 3 (d) 1, 2 & 3
(e) None of these (e) None of these
E-44 GENERAL AWARENESS
66. Who among the following has been named the most 76. The 22nd Commonwealth Heads of Government Meeting
powerful celebrity of 2013 by Forbes India? which concluded in Sri Lanka recently, has adopted which
(a) Mahendra Singh Dhoni (b) Shah Rukh Khan of the following three independent declarations?
(c) Salman Khan (d) Sachin Tendulkar (a) Youth, International Trade and Inclusive Development
(e) None of these (b) Youth, International Trade and Sustainable
67. Who among the following has been named as "Time Development
Magazine Person of the Year"? (c) Youth, Industry and Environment
(a) Edward Snowden (b) Ted Cruz (d) Industry, Individual and Development
(c) Bashar al-Assad (d) Pope Francis (e) None of these
(e) None of these 77. Which of the following nation has recently qualified for
68. According to annual list of Forbes 500 companies in India, World T20 Championship for the first time?
the country's biggest company in terms of annual revenue (a) Denmark (b) Holland
is _________? (c) Nepal (d) UAE
(a) Indian Oil Corporation (e) None of these
78. Which of the following statements is true about the
(b) Reliance Industries
"Operation Oliver" of coast guard?
(c) Bharat Petroleum
(a) It's a mission to check the Bangladeshi immigrants
(d) Hindustan Petroleum
(b) It's a rescue operation conducted to safeguard the
(e) None of these
Helen hit victims
69. The Organisation of the Petroleum Exporting Countries (c) An annual mission to ensure safety of breeding Olive
(OPEC) contributes nearly what per cent of total world's oil? Ridley sea turtles
(a) 25% (b) 30% (d) A combat exercise to check Naxal operations
(c) 35% (d) 40% (e) None of these
(e) None of these 79. Name the company that bagged two contracts of 151 crore
70. Who among the following has won the Economic Times rupees from Gujarat Energy Transmission Company Ltd
Lifetime Achievement Award 2013? (GETCO). The contracts are for supply of 400 KV, 220KV
(a) Narayana Murthy (b) Azim H Premji and 66KV substations located at Charanka and Sankhari on
(c) Anil Ambani (d) Mukesh Ambani turnkey basis to evacuate power from Solar Park of the state.
(e) None of these (a) Jaiprakash Power Ventures Ltd.
71. Accordin g to th e recent global survey of leading (b) Jetking Infotrain Ltd
consultancy firm Ernst and Young (EY), which among the (c) CESC Limited
following countries has emerged as the most attractive (d) Alstom T&D India
investment destination of the world recently? (e) None of these
(a) China (b) India 80. Who on 9 December 2013 of the following has taken over as
(c) United States (d) Canada the High Commissioner of India to UK?
(e) None of these (a) Nirupama Rao (b) Shivshankar Menon
72. Who among the following has won UN Human Rights Prize (c) Chokila Iyer (d) Ranjan Mathai
2013? (e) T. V. Rajeswar
(a) Malala Yousafzai (b) Angel Markel 81. Match in the following with their designation?
(c) Nelson Mandela (d) Kofi Anann (a) Nirupama Rao (i) India's first female
(e) None of these Foreign secretary
73. Which state won the 2014 Women's National Football (b) Shivshankar Menon (ii) Emeritus Indian
Championship? Ambassador to the
United States
(a) West Bengal (b) Assam
(c) Chokila Iyer (iii) High Commissioner
(c) Manipur (d) Odisha
of India to UK
(e) None of these (d) Ranjan Mathai (iv) 4th National Security
74. Which of the following has recently named as World's Advisor
leading low cost Airline at the World Travel Awards 2013? (a) 3 1 4 2 (b) 2 4 1 3
(a) Air Berlin (b) Jet Blue Airways (c) 1 2 4 3 (d) 4 3 2 1
(c) Air Asia (d) Air Arabia (e) 3 1 2 4
(e) None of these 82. Which party won the Mizoram Assembly Election 2013?
75. Which of the following has been chosen to host the "World (a) Bharatiya Janata Party (BJP)
Expo Trade Convention" in 2020? (b) Mizoram Democratic Alliance (MDA)
(a) London (b) Dubai (c) Indian National Congress (INC)
(c) Sydney (d) Mumbai (d) Independent party
(e) None of these (e) None of these
GENERAL AWARENESS E-45
83. Cabinet Committee on Uttrakhand, headed by Prime Minister 90. Irrfan Khan on 13 December 2013 won the best actor award
of India, Manmohan Singh on 9 December 2013 sanctioned for his brilliant performance in a film at the 10th edition of
an amount for relief and rehabilitation of flood-hit Dubai International Film Festival (DIFF). Name the film for
Uttarakhand. What amount has been sanctioned for the which he won the best actor award 2013?
state? (a) D-Day
(a) 2000 crore rupees (b) 7346 crore rupees (b) Life of Pi
(c) 8000 crore rupees (d) 10000 crore rupees (c) Saheb Biwi Aur Gangster Returns
(e) None of these (d) The Lunchbox
84. Supreme Court of India on 10 December 2013 set a fresh (e) None of these
four month deadline for the centre and states to make 91. India defeated Pakistan 48-39 on 14 December 2013 in the
functional 22 CBI courts across India. In failure to comply World Kabaddi Cup in the finals played. The final was played
with its order will invite contempt action against whom? at which place and stadium?
(a) Law Minister (a) War Heroes Stadium, Sangrur
(b) Chief Secretaries (b) Guru Nanak Dev Stadium, Ludhiana
(c) Chief Minister of States (c) Guru Arjun Dev Sports Stadium, Chohla Sahib, Tarn
(d) The Central Government Taran
(e) None of these (d) Nehru Stadium, Rupnagar
85. Name the Indian cricketer, who on 13 December 2013 bagged (e) None of these
the Emerging Cricketer of the Year trophy at the ICC Annual 92. The Central Board of the Reserve Bank of India (RBI) met
awards announced in Dubai. on 12 December 2013 at Kolkata. Who chaired the meeting
(a) Cheteshwar Pujara (b) Murali Vijay of the Central Board?
(c) Mahendra Singh Dhoni (d) Virat Kohli (a) Governor of Reserve Bank of India, Raghuram G. Rajan
(e) None of these (b) Deputy Governor of Reserve Bank of India, Harun R.
86. Sir Garfield Sobers Trophy 2013 of cricket was given to Khan
whom? (c) Deputy Governor of Reserve Bank of India, K.C.
(a) Kumar Sangakkara (b) Suzie Bates Chakrabarty
(c) Kevin O'Brien (d) Michael Clarke (d) Deputy Governor of Reserve Bank of India, Anand
(e) None of these Sinha
87. The Uttar Pradesh Government on 13 December 2013 set-up (e) None of these
a high-level committee to look what cause in the relief camps 93. India finished at which position in Hero Junior Men's Hockey
set after the Muzaffarnagar riots? World Cup played at the floodlit Major Dhyan Chand
(a) To look into the issue of deaths of children in the relief National Stadium, New Delhi?
camps (a) 4th (b) 6th
(b) To provide better facilities to the people leaving in the (c) 1st (d) 10th
relief camps (e) None of these
(c) To provide better medical support to them 94. Name the player who was inducted in the Hall of the Fame in
(d) All of the above Perth on 13 December 2013 during the Ashes series?
(e) None of these (a) Everton Weekes (b) Frank Woolley
88. Lal Thanhawla of Congress party has sworn in as the Chief (c) Adam Gilchrist (d) Frank Worrell
Minister of Mizoram for the second successive term on 14 (e) None of these
December 2013. He was administered the oath of office and 95. On 4 December 2013, he was reappointed as the Organisation
secrecy by whom? for the Prohibition of Chemical Weapons (OPCW) Director-
(a) E. S. L. Narasimhan General at the 18th Session of the Conference of State Parties.
(b) Vakkom B. Purushothaman This will be his second term in the office. The person is
(c) A.K. Singh (a) Ahmet Uzumcu (b) Rogelio Pfirter
(d) Janaki Ballabh Patnaik (c) José Bustani (d) Everton
(e) None of these (e) None of the above
89. The former footballer of India, Babul Phukan on 14 December 96. As per the declaration of the Organization for the Prohibition
2013 died at a hospital in Guwahati, Assam after a prolong of Chemical Weapons (OPCW) on 6 November 2013, which
Category of munitions was declared to have been destroyed
illness has represented Assam in which of the trophies nine
by the Syrian Government?
times?
(a) Category 1 (b) Category 2
(a) Todd Memorial Trophy (b) Durand Cup
(c) Category 3 (d) Category 4
(c) Santosh Trophy (d) lndira Gold Cup
(e) None of these
(e) None of these
E-46 GENERAL AWARENESS
97. An informal Group of Ministers (GoM) on 6 December 2013 (a) Miss Thailand Punika Kulsoontornrut
recommended a bailout package to the Sugar Industry. The (b) Miss Venezuela Alyz Henrich
bailout includes 7200 crore rupees at 12 percent interest rate (c) Miss Austria Katia Wagner
to sugar mills by the banks to pay off the arrears of the (d) Miss Korea Catharina Choi
sugarcane growers. The GoM was headed by (e) None of these
(a) Prime Minister of India, Manmohan Singh 105. The 9th WTO Ministerial Meet concluded in Bali on 6
(b) Union Agriculture Minister, Sharad Pawar December 2013. In this context which one of the following
(c) The Home Minister of India, Sushil KumarShinde was not agreed upon by the member countries?
(d) By the Governor of RBI, Raghuram Rajan (a) Setting up internal mechanism to safeguard minimum
(e) None of these support prices to farmers against WTO caps
98. Name the anti-apartheid revolutionary of South Africa who (b) Agreement on Trade Facilitation
died on 5 November 2013. He was 95 years old. He died in (c) Agreement on Services
his house at Johannesburg following a prolong illness. He (d) Permits countries to store food grains in time of
was also known as Madiba in South Africa. contingency
(a) Baden-Baden (b) Abdul Minty (e) None of these
(c) Nelson Mandela (d) Dennis Brutus 106. The 8th Asia Gas Partnership Summit concluded on 4
(e) None of these December 2013 which was organized by GAIL in
99. The Union Cabinet on 5 December 2013 approved the collaboration with the FICCI. The theme of the Summit was
Telangana draft Bill. This approval has set up the bifurcation (a) Exploring green alternatives to existing fossil fuel based
of which State for carving out a new state of Telangana? consumption
(a) Andhra Pradesh (b) Karnataka
(b) Challenges and opportunities in the changing paradigm
(c) Tamil Nadu (d) West Bengal in Asia
(e) None of these
(c) Collaborating effectively to meet the challenges in
100. Hyderabad will be the capital of two states, Andhra Pradesh
Asian Gas Market
and Telangana for a period of ____________?
(d) All of these
(a) more than 10 years
(e) None of these
(b) for a period not exceeding 10 years
107. Consider the following statements -
(c) for more than 15 years
1. Central Industrial Security Force (CISF) has recently
(d) for a period not exceeding 15 years
been included as an associate member of Hockey India
(e) None of these
(HI).
101. Under which Article of the Constitution, the two States of
2. The induction of the CISF will enable it to take part in
Telangana and Andhra Pradesh will have special status for
all the National championships.
equitable opportunities for education an d public
3. Hockey India currently- has 26 permanent member
employment
units, 20 associate members, 2 members through State
(a) Article 2 of the Constitution of India
Olympic Associations, 5 Academy Members and 1
(b) Article 239 A of the Constitution of India
Hoc-key Member.
(c) Article 358 of the Constitution of India
Correct Statements are?
(d) Article 371-D of the Constitution of India
(a) 2 and 3 (b) l and 3
(e) None of these
(c) 1 and 2 (d) 1, 2 and 3
102. Prohibition of Employment as Manual Scavengers and their
Rehabilitation Act 2013 came into force on 6 December 2013 (e) None of these
across India except which state? 108. India was chosen as the host country for Under-17 FIFA
(a) Manipur (b) Assam World Cup in 2017 by the FIFA executive committee in its
(c) Jammu and Kashmir (d) Rajasthan meeting at Salvador de Bahia in Brazil on 5 December 2013.
(e) None of these India outbid which other countries in the race to host U-17
103. The Supreme Court of India on 5 December 2013 favoured FIFA World Cup 2017?
punishment of _________ for people indulging in (a) Iran, China and Singapore
production and marketing of adulterated milk. (b) Uzbekistan, Azerbaijan, Ireland and South Africa
(a) Life imprisonment (c) Ireland, Iran, Singapore and South Africa
(b) Minimum six months (d) Azerbaijan, China and South Africa
(c) Minimum period of 6 months (e) None of these
(d) Minimum period of 12 months 109. Brazuka, the official 2014 FIFA World Cup match ball was
(e) None of these unveiled on 3 December 2013. Who is the manufacturer of
104. Who was crowned with the pageant of Miss Earth 2013 at the Brazuka?
the Versailles Palace, Philippines? She was crowned with (a) Reebok (b) Nike
the pageant of Miss World by Tereza Fajksova, the Miss (c) Adidas (d) Honglin Sport Gear
Earth 2012 from Czech Republic. (e) None of these
GENERAL AWARENESS E-47
110. International Volunteer Day (IVD) was celebrated on 5 119. Who among the following has become the first woman to
December. The day gives volunteers a chance to work chair United States Central Bank (Federal Reserve)?
together on projects and campaigns promoting their (a) Amelia Grace (b) Aimee Jayne
contributions to economic and social development at local, (c) Janet Yellen (d) Margaret Chloe
national and international levels. The theme for the year (e) None of these
2013 was 120. Which of the following states has been honoured with
(a) Young-Global-Active prestigious WTTCII tourism award?
(b) Cooperate-Eliminate-Economic Woes (a) Kerala (b) Gujarat
(c) Global-Cooperate-Eliminate (c) Nagaland (d) New Delhi
(d) Participate-Involve-Local Communities (e) None of these
(e) None of these 121. At which international platform have certain developed
111. Where will the 2014 Winter Olympics be held? countries raised serious concerns about india's new National
(a) Sochi in Russia (b) Vancouver in Canada Food Security Programme?
(c) Shimla in India (d) Berlin in Germany (a) Food and Agriculture Organisation
(e) None of these (b) World Trade Organisation
112. Where will the 2018 Winter Olympics held? (c) United Nations Organisation
(a) London in Britain (d) World Health Organisation
(b) PyeongChang in South Korea (e) None of these
(c) Tokyo in Japan 122. Who won the Australian Open men's singles title on 26
(d) Canberra in Australia January 2014?
(e) None of these (a) Rafael Nadal (b) Roger Federer
113. Which country has declared to phase out all nuclear power (c) Andy Murray (d) Stanislas Wawrinka
by 2022? (e) None of these
(a) France (b) Japan 123. Who won the Australian Open women's singles title?
(c) Germany (d) The United States (a) Dominika Cibulkova (b) Li Na
(e) None of these (c) Serena Williams (d) Ana Ivanovic
114. India was ranked 94th out of 177 countries on Global (e) None of these
Corruption Perception Index 2013, which was released by 124. Who won the Mixed doubles tennis titles at Australian Open
Transparency International on 3 December 2013. The least 2014?
corrupt countries according to the Index are (a) Kristina Mladenovic and Daniel Nestor
(a) Finland and Switzerland (b) Sania Mirza and Horia Tecau
(b) New Zealand and Australia (c) Ana Ivanovic and Rafael Nadal
(c) Denmark and New Zealand (d) Li Na and Andy Murray
(d) Finland and Sweden (e) None of these
(e) None of these 125. Which country won the gold in the men's basketball in the
115. Prof C N R Rao, who was recently awarded Bharat Ratna Lusofonia Games 2014?
holds which among the following offices currently? (a) Angola (b) Brazil
(a) Chairman DRDO (c) Mozambique (d) India
(b) Head of the Scientific Advisory Council to the Prime (e) None of these
Minister 126. Central Statistics Office (CSO) of India on 31 January 2014
(c) Chairman of Atomic Energy Regulatory Board announced that economy might have expanded by 4.5
(d) A Scientist at Bhabha Atomic Research Centre percent in fiscal year 2012-13 when compared to the previous
(e) None of these estimate. What was the earlier estimate?
116. According United Nation latest report, which country has (a) 6 percent (b) 5 percent
by far the largest population of illiterate adults in the world? (c) 4 percent (d) 3 percent
(a) China (b) India (e) None of these
(c) Pakistan (d) Indonesia 127. Kathy Cross on 30 January 2014 became the first woman to
117. In which of the following cities India's first monorail service be appointed in an International Cricket Council (ICC)
has been launched, recently? umpires panel. She belongs to which country?
(a) Mumbai (b) Bangalore (a) Australia (b) England
(c) Kolkata (d) Chennai (c) Zimbabwe (d) South Africa
(e) None of these (e) None of these
118. The Prime Minister of which country will be the chief guest 128. Union Government of India on 30 January 2014 signed a
at this year's India's Republic Day parade? Double Taxation Avoidance Agreement (DTAA) for
(a) China (b) Japan avoidance of double taxation and prevention of fiscal
(c) Singapore (d) Australia evasion with respect to taxes on income with a nation. Name
(e) None of these the nation?
E-48 GENERAL AWARENESS
(a) Finland (Your Answer) (b) France 138. Which Album has won the Best Album of the Year at the
(c) Fiji (d) Georgia 56th Grammy Awards?
(e) None of these (a) Get Lucky
129. Which song was withdrawn from the Best Song nomination (b) Random Access Memories
category by the Oscar Academy on 30 January 2014? (c) Royals
(a) Alone Yet Not Alone (b) Variety Noted (d) Throw a Penny in the Wishing Well
(c) The Godfather (d) I am not a Man (e) None of these
(e) None of these 139. Who has been awarded the 2014 Ramanujan Mathematics
130. The Government of India on 30 January 2014 raised the cap Award on 27 January 2014?
on number of LPG cylinders that an individual can avail in a (a) Ramdorai Sujatha (b) Marcelo Viana
year. The number raised is (c) J J Rawal (d) Anand Kumar
(e) None of these
(a) 9 (b) 12
140. According to the recently released data by Reuters, India
(c) 15 (d) 10
overtook Japan to become the third largest importer of
(e) None of these
(a) Crude Oil (b) Military hardware
131. Union Cabinet approved the second phase of the Bangalore
(c) Edible Oils (d) All of the above
Metro Rail Project to improve public transport in the city on (e) None of these
30 January 2014. The name of the Bangalore Metro is 141. New Zealand on 31 January 2014 won the five match ODI
(a) Amma Metro (b) Namma Metro Series against India by 4-0. What was the venue for the fifth
(c) Bengaluru Metro (d) Dakshin Metro ODI match?
(e) None of these (a) Queenstown Events Centre at Queenstown
132. According to the recently released data by Reuters, India (b) Westpac Stadium at Wellington
overtook Japan to become the third largest importer of (c) Auckland Domain at Grafton
(a) Crude Oil (b) Military hardware (d) Pukekura Park at New Plymouth
(c) Edible Oils (d) All of the above (e) None of these
(e) None of these 142. The Reserve Bank of India, according to figures released on
133. Which country of African continent has decide to use Indian May 26, 2014 current account deficit (CAD) in the fourth
rupee as a legal tender? quarter of fiscal 2013-14 declined sharply to 1.7 percent of
(a) Namibia (b) Zimbabwe GDP came. How many percent in fiscal 2012-13 ending?
(c) Zambia (d) Sudan (a) 4.8 (b) 4.7
(e) None of these (c) 4.9 (d) 3.9
134. According to the Lokpal & Lokayukta Act 2013, who among (e) None of these
the following is not a part of the selection committee for 143. Recently, WHO-India has launched IVR 2020 programme to
choosing the Lokpal chairperson? protect and increase the number of ____?
(a) Prime Minister (a) Tigers (b) One-horn Rhinos
(b) Leader of Opposition in Lok Sabha (c) Indian Bustard (d) Asiatic Lions
(e) None of these
(c) Speaker of Lok Sabha
144. Recently, it was reported in newspaper that CERT-In has
(d) Deputy Chairman of Rajya Sabha
detected a new smartphone virus which can compromise
(e) None of these
the data stored on the user's phone. The virus is named as
135. Who among the following has become the fastest batsman ___?
to score 11,000 test runs, recently? (a) Dendroid (b) Killer
(a) M S Dhoni (b) Kevin Peterson (c) Ebola (d) SmartX
(c) Kumar Sangakkara (d) Ashim Hamla (e) None of these
(e) None of these 145. Apart from India, which of the following South Asian
136. Recently, India has decided to provide Visa-on-arrival facility countries have been declared Polio free by WHO, recently?
to tourists of all countries except eight nations. Which 1. Bangladesh 2. Pakistan
among the following is NOT among those eight nations? 3. Nepal 4. Bhutan
(a) Pakistan (b) Bangladesh Chose the correct answer from the codes given below:
(c) Sri Lanka (d) Afghanistan (a) Only 1 & 2 (b) Only 2 & 3
(e) None of these (c) Only 1, 3 & 4 (d) 1, 2, 3 & 4
137. As per the estimates of Planning Commission, which state (e) None of these
had the highest number of poor people living below the 146. Which among the following countries clinched the South
poverty line in 2011-12? Asian Handball Championship 2014?
(a) Madhya Pradesh (b) Bihar (a) Pakistan (b) Bangladesh
(c) Uttar Pradesh (d) Maharashtra (c) India (d) Afghanistan
(e) None of these (e) None of these
GENERAL AWARENESS E-49
147. Which among the following has become the first state in 156. Which among the following national parks has been
the country to declare all election booths in the coming Lok nominated to UNESCO's world heritage site status for 2014?
Sabha polls as no smoking zones? (a) Great Himalayan National Park
(a) Kerala (b) Haryana (b) Jim Corbett National Park
(c) Gujarat (d) Jammu & Kashmir (c) Anshi National Park
(e) None of these (d) Bannerughatta National Park
148. Which among the following countries is the world's largest (e) None of these
producer of "Rare Earths"? 157. According to UNESCO's World Water Development Report,
(a) USA (b) Russia which among the following sectors accounts for maximum
(c) India (d) China water use?
(e) None of these (a) Industries (b) Agriculture
149. Recently launched Central Public Sector Enterprises (CPSE) (c) Domestic use (d) Construction
ETF is an open-ended scheme that consists of shares of ten (e) None of these
major public sector units. The (CPSE) ETF scheme is managed 158. Which among the following companies emerged as the only
by ___? Indian entities in the 2014 list of World's most ethical
(a) Goldman Sachs India companies?
(b) Bombay Stock Exchange (a) Wipro and Infosys (b) Infosys and Tata Power
(c) RBI (c) Wipro and Tata Power (d) TCS and Infosys
(d) National Stock Exchange (e) None of these
(e) None of these 159. As per the Indian Aviation report 2014, which one of the
150. Who among the following is the Chairman of Tax following is the main reason for rendering Indian aviation
Administration Reform Commission (TARC)? sector uncompetitive?
(a) Parthasarathy Shome (b) C Rangarajan (a) Over Taxation
(b) Lack of Skill
(c) V Subbarao (d) S. Mahalingam
(c) Infrastructure
(e) None of these
(d) Privatisation Over Taxation
151. Who among the following is the winner of the Life-Time
Achievement Award under 3rd National Photography (e) None of these
Awards? 160. Which among the following countries is the largest supplier
(a) J. Suresh (b) Rajesh Bedi of arms to India in 2009-13?
(a) Russia (b) USA
(c) Atul Yadav (d) Dinesh Shukla
(c) Israel (d) Japan
(e) None of these
(e) None of these
152. Govinda Mishra, who has been chosen for the Saraswati
Samman for the 2013, is a famous author in which language? 161. Recently, government has approved to set up country's
second nuclear fuel complex to reprocess atomic fuel in ___?
(a) Kannada (b) Telugu
(a) Rajasthan (b) Maharashtra
(c) Hindi (d) Marathi
(c) West Bengal (d) Jharkhand
(e) None of these
(e) None of these
153. Which among the following has become the first state in
162. "Rani-ki-Vav" which has been nominated for World Heritage
the country to declare all election booths in the coming Lok
Status is a 11th century step well located in____?
Sabha polls as no smoking zones?
(a) Karnataka (b) Gujarat
(a) Kerala (b) Haryana
(c) Rajasthan (d) Haryana
(c) Gujarat (d) Jammu & Kashmir
(e) None of these
(e) None of these
163. Name the Czech priest, philosopher who has won the 2014
154. "Shigeru Ban" who has won the 2014 Pritzker Architecture Templeton Prize?
Prize belongs to which country?
(a) Chris Herlinger (b) Tomas Halik
(a) South Korea (b) Japan
(c) Graham Greene (d) Billy Graham
(c) China (d) Singapore
(e) None of these
(e) None of these
164. Which among the following states has been ranked top in
155. The name of world's first gun for woman launched recently the recently released "Economic Freedom of the States of
is ___? India" report 2013?
(a) Nirbheek (b) Shravya (a) Tamil Nadu (b) Rajasthan
(c) Jhansi (d) Kalpana (c) Gujarat (d) Madhya Pradesh
(e) None of these (e) None of these
E-50 GENERAL AWARENESS
165. Which one of the following is not a "Navaratna" company? 174. ‘Magnus Carlsen’ who has been recently become the
(a) Bharat Electronics Limited youngest worlds chess championship belongs to which
(b) Indian Oil Corporation country?
(c) Airports Authority of India (a) Russia (b) Sweden
(d) Rural Electrification and Corporation Limited (c) France (d) Norway
(e) None of these (e) None of these
166. The Indian American author of "The Lowland " who has 175. With which one of the following countries India has signed
been listed among 13 novels long listed for the Man Booker deal for Super Hercules aircraft, recently?
Prize 2013. (a) Japan (b) USA
(a) Jhumpa Lahiri (b) Laura Jones (c) Russia (d) Italy
(e) None of these
(c) Chetan Bhagat (d) Chetna Singh
176. As per the latest Annual Health Survey, which state has
(e) None of these
highest percentage of girls marrying before completing 18?
167. Which of the following has become the first airport in the (a) Rajasthan (b) Odisha
World to receive the prestigious “Sword of Honour” for (c) Bihar (d) Himachal Pradesh
safety management? (e) None of these
(a) Kempegowda International Airport 177. Pran Chopra, who passed away recently, was a ________.
(b) Kochi International Airport (a) Journalist (b) Photographer
(c) Indira Gandhi International Airport (c) Author (d) Sports man
(d) Rajiv Gandhi International Airport (e) None of these
(e) None of these 178. Which one of the following is the main aim of the recently
168. Who among the following will be awarded the “Indira launched ‘E-Inclusion’ project?
Gandhi Prize for Peace, Disarmament and Development for (a) To provide core bank facility to all post offices
2013"? (b) To achieve 100 per cent financial inclusion
(a) Julia Gillard (b) Hilary Clinton (c) To provide IT training for Rural/SC/ST women
(c) Angela Merkel (d) Sheikh Hasina beneficiaries
(d) To establish ATM kiosks at grama panchayat level
(e) None of these
(e) None of these
169 Who among the following will be conferred with the
179. Who among the following has been named the most
“Centenary Award for India Film Personality of the Year”?
powerful celebrity of 2013 by Forbes India?
(a) Waheeda Rehman (b) Adur Gopalkrishnan (a) Mahendra Singh Dhoni
(c) Satyajith Ray (d) Amitabh Bachhan (b) Shah Rukh Khan
(e) None of these (c) Salman Khan
170. Srishsti Rana, who has recently won Miss Asia Pacific World (d) Sachin Tendulkar
2013 crown, hails from which among the following states? (e) None of these
(a) Kerala (b) Haryana 180. Who among the following has been named as ‘Time
(c) Tripura (d) Assam Magazine Person of the Year’?
(e) None of these (a) Edward Snowden (b) Ted Cruz
171. By defeating which of the following countries India clinched
(c) Bashar al-Assad (d) Pope Francis
the women's kabaddi World Cup Championship-2013?
(a) Pakistan (b) New Zealand (e) None of these
(c) Sri Lanka (d) Nepal 181. Google has recently opened its first ever data centres in
(e) None of these Asia to boost its growth further in the region. The two new
172. Who among the following has recently become the first centres are based in __________.
Indian to win the Indonesia Open? (a) India and China
(a) Shiv Kapur (b) Anirban Lahiri (b) Taiwan and China
(c) Jyoti Randhawa (d) Gaganjeet Bhullar (c) Taiwan and Singapore
(e) None of these
(d) Singapore and India
173. Who among the following has been recently elected as new
chief boxing coach of India? (e) None of these
(a) Gurbax Singh Sandhu 182. India's first heritage transport museum has recently opened
(b) Narottam Singh Rawat at __________.
(c) Bhaskar Rao (a) Bangalore (b) Gurgaon
(d) Gurubachan Singh (c) Mumbai (d) Bhopal
(e) None of these (e) None of these
GENERAL AWARENESS E-51

183. Who among the following has won UN Human Rights Prize Which one of the following acts was making news in this
2013? context?
(a) Representation of the People Act 1950
(a) Malala Yousafzai (b) Angel Markel
(b) Representation of the People Act 1951
(c) Nelson Mandela (d) Kofi Anann (c) Parliament (Prevention Of Disqualification) Act, 1959
(e) None of these (d) Constitution of India Act 1949
184. What is India's ranking at the first ever ‘Environment and (e) None of these
192. Sumitra Mahajan, the newly elected speaker of Lok Sabha
Gender Index (EGI)’ launched at a side event of the
represents which among the following constituencies?
Conference of Parties (COP19), recently? (a) Bhopal (b) Indore
(a) 46 (b) 54 (c) Nagpur (d) Jabalpur
(c) 35 (d) 41 (e) None of these
(e) None of these 193. The movie “Kai Po Che” is based on which novel of Chetan
Bhagat?
185. Which of the following countries have launched Missions
(a) Three Mistakes of My Life
to Mars apart from India?
(b) Two States
1. Russia 2. China
(c) Revolution 20-20
3. US 4. Japan
(d) A Night at the call centre
5. Italy
(e) None of these
Choose the correct answer from the codes given below:
194. Recently Apple Computers of USA has launched a touchpad
(a) Only 1, 2, 3 & 4 (b) Only 2, 3, 4 & 5 which allows a user to operate desktop computer with finger
(c) Only 1, 3, 4 & 5 (d) 1, 2, 3, 4 & 5 gestures, eliminating the need for a mouse. What is the
(e) None of these name of the device?
186. As per the latest data, which of the following cities has (a) Mirror-image pad (b) Virtual pad
more internet user than any other city in the country? (c) Fingertip track pad (d) Magic track pad
(a) Mumbai (b) New Delhi (e) None of these
(c) Chennai (d) Kolkata 195. Corey Anderson, who hit the fastest century in one-day
(e) None of these international history recently, belongs to which country?
187. The “Voluntary Compliance Encouragement Scheme, 2013” (a) Australia (b) New Zealand
is related to which one of the following? (c) England (d) South Africa
(a) To encourage self employment among youths (e) None of these
(b) To encourage the defaulters of service tax 196. Which of the following countries has recently assumed the
(c) To promote exporters coveted chair of the ASEAN?
(d) To encourage NRI investors (a) Myanmar (b) Indonesia
(e) None of these (c) Cambodia (d) Philippines
188. Typhoon Haiyan, world’s strongest storm of the year 2013 (e) None of these
recently hit which one of the following countries? 197. According to latest study, which Indian city is expected to
(a) China (b) New Zealand witness the highest demand for office space in the Asia
(c) Philippines (d) Australia during 2014?
(e) None of these (a) New Delhi (b) Mumbai
189. Which of the following countries has recently lost its voting (c) Bangalore (d) Chennai
rights at UNESCO? (e) None of these
(a) India (b) Syria 198. Who among the following is the new Air Force Chief of
(c) USA (d) England India?
(e) None of these (a) Arup Raha (b) Vivek Tyagi
190. Who among the following is the current Secretary General (c) Arun Singh (d) Kishore Chandra
of “South Asian Association for Regional Countries (e) None of these
(SAARC)”? 199. Lieutenant Governor of Delhi, Najeeb Jung on 13 May 2014
(a) Kamalesh Sharma (b) Fathimath Dhiyana Saeed constituted a High Powered Committee to look into the
(c) Ahmed Saleem (d) Abdulla Mohamed pollution levels in national Capital and suggest the steps to
reduce its levels in the city. Who will chair the committee?
(e) None of these
(a) Delhi Chief Secretary
191. Recently, two leaders viz. RJD leader Lalu Prasad Yadav
(b) Environment Secretary
and the Janata Dal (United) leader Jagadish Sharma have
(c) Transport Commissioner
been disqualified from their parliamentary memberships on
(d) DPCC Additional Secretary
account of conviction under the Chaibasa Treasury case.
(e) None of these
E-52 GENERAL AWARENESS
200. UN on 12 May 2014 appointed a woman to head UN 208. Name the Indian who featured on the cover page of the
Peacekeeping Force in Cyprus (UNFICYP). With this business magazine Forbes Middle East's Top Indian Leaders
appointment, the woman became the first-ever female to in Arab World 2014 issue?
head a UN peacekeeping force in its entire history. Name (a) Bollywood actor Amir Khan
her? (b) Infosys CEO Narayan Murthy
(a) Kristin Lund of Norway (c) Bollywood actor Shah Rukh Khan
(b) Rashida Manjoo of South Africa (d) Cricketer Sachin
(c) Phumzile Mlambo-Ngcuka of South Africa (e) None of these
(d) Helen Clark of New Zealand 209. When was the World Migratory Bird Day 2014 observed
(e) None of these across the world?
201. US Senate on 13 May 2014 confirmed the name of an Indian (a) 10 May 2014 (b) 10-11 May 2014
- American as the federal judge in Massachusetts. Name the (c) 20 -21 May 2014 (d) 20 May 2014
person? (e) None of these
(a) Indira Talwani (b) Nikki Haley 210. Recently Supreme Court gave a judgement that Candidates
(c) Kal Penn (d) Padma Lakshmi fighting elections can be disqualified if they fail to disclose
(e) None of these information about assets and liabilities of even their spouse
202. Name the State Government that on 14 May 2014 signed an and dependent children. According to which Article of the
agreement with the TiE Silicon Valley as a part of state's Constitution of India, do the citizens have the right to know
efforts to develop it an IT hub and attract US investment? the information about candidate's assets and liabilities?
(a) Karnataka (b) Andhra Pradesh (a) Article 21 (b) Article 30(a)
(c) West Bengal (d) Odisha (c) Article 19 (a) (d) Article 14
(e) None of these (e) None of these
203. Name the country that on 14 May 2014 announced its 211. Name the former Chairman and Managing Director of Tata
decision to abolish its controversial sponsorship system Steel known as the Grand Old Man of Steel, who died on 16
for foreign workers? May 2014?
(a) Algeria (b) Yemen (a) Russi Mody (b) Ratan Tata
(c) Kuwait (d) Qatar (c) JRD Tata (d) B. Muthuraman
(e) None of these (e) None of these
204. Name the person who was appointed as the Chairman of the 212. Who won the Women's Italian Tennis Open 2014 for the
Cauvery Water Disputes Tribunal on 14 May 2014? third time on 18 May 2014?
(a) Justice NP Singh (a) Sara Irrani of Italy
(b) Justice BS Chauhan (b) Serena Williams of the US
(c) Justice Sudhir Narain
(c) Venus Williams of the US
(d) Justice HL Dattu
(d) Maria Sharapova of Russia
(e) None of these
(e) None of these
205. The Cauvery Water Disputes Tribunal (CWDT) was
213. Who won the Men's Italian Tennis Open 2014 for the third
constituted by the Government of India in which year?
time on 18 May 2014?
(a) 1990 (b) 1998
(a) Rafael Nadal of Spain
(c) 2005 (d) 2010
(b) Roger Federer of Switzerland
(e) None of these
(c) Novak Djokovic of Serbia
206. The Cauvery Water Disputes Tribunal (CWDT) was
(d) Andy Murray of UK
constituted by the Government of India with an aim to give
its ruling on the water dispute regarding inter-state river (e) None of these
Cauvery and the river valley thereof between two states. 214. Name the instant messaging app launched by Sistema Shyam
Name the two states for which the tribunal was constituted? Teleservices for feature phones.
(a) Tamil Nadu and Kerala (a) MyBuddy (b) MBuddy
(b) Karnataka and Kerala (c) WeChat (d) WithU App
(c) Karnataka and Maharashtra (e) None of these
(d) Karnataka and Tamil Nadu 215. On 18 May 2014, President of India, Pranab Mukherjee
(e) None of these dissolved the 15th Lok Sabha. The dissolution of Lok Sabha
207. Name the person who on 18 May 2014 was sworn-in as the was done by the President in accordance with which Article
eighth Chief Minister of Arunachal Pradesh at the Raj of the Constitution of India?
Bhavan in Itanagar? (a) Article 85 (1)(b) (b) Article 85 (2)(a)
(a) Nirbhay Sharma (b) Nabam Tuki (c) Article 85 (1) (d) Article 85 (2)(b)
(c) Gegong Apang (d) Mukut Mithi (e) None of these
(e) None of these
GENERAL AWARENESS E-53
216. The Committee to Review Governance of Boards of Banks 223. 228 light-weight aircraft Nabhrathna was handed over to
in India on 13 May 2014 submitted its report to Reserve Defence Research and Development Organisation (DRDO).
Bank of India (RBI). The Committee was headed by Which organization has developed Nabhratna?
(a) P J Nayak (b) Bimal Jalan (a) Electronics and Radar Development Establishment
(c) Aditya Puri (d) C Rangarajan (b) Armament Research & Development Establishment
(e) None of these (c) Hindustan Aeronautical Limited (Answer)
217. Which party emerged as the third largest party in the recently
(d) Aeronautical Development agency
declared results of General Election 2014?
(e) None of these
(a) Indian National Congress
(b) Bahujan Samaj Party 224. Coca-Cola announced to remove an inflammated ingredient
(c) All India Anna Dravida Munnetra Kazhagam from its cold drinks. What is the name of the ingredient?
(AIADMK) (a) acetate isobutyrate
(d) Trinamool Congress (b) Brominated Vegetable Oil (BVO)
(e) None of these (c) glycerol ester
218. Prime Minister Manmohan Singh submitted his resignation (d) Sucrose rosin
to the President of India on 17 May 2014. In this context, (e) None of these
who was the longest serving Prime Minister of India? 225. What is RuPay? Consider the following statements:
(a) Indira Gandhi (b) Jawahar Lal Nehru
1. An Indigenous Card payment System
(c) Manmohan Singh (d) Gulzari Lal Nanda
2. RuPay is the coinage of two terms Rupee and Payment
(e) None of these
219. Recently, Philippines and the United States signed a defence 3. National Payments Corporation of India (NPCL)
pact. The military agreement between both the countries developed the Rupay platform
will allow a bigger US military presence on Filipino territory. 4. Rupay is the seventh card payment system in the world
Check the correct statement in this regard: Choose the correct Statements:
1. The defence pact will be valid for 10 years. (a) 1, 2 and 3 (b) 2, 3 and 4
2. Philippine defence minister Voltaire Gazmin and US (c) 1, 2 and 4 (d) All are correct
ambassador Philip Goldberg signed the defence pact. (e) None of these
3. The new agreement will not allow the reopening of US 226. Air-to-Air missile Astra was successfully test fired from a
bases in the Philippines. Sukhoi-30MKI fighter jet. Consider the following statements
4. The agreement is seen as another element of Obama's about Astra:
effort to focus US military and economic attention more
1. Astra is India's first Beyond Visual Range (BVR) Air-
heavily on Asia.
to-Air missile. It was designed and developed
(a) 1 and 2 (b) 1,2 and 3
indigenously by the DRDO.
(c) 1 and 3 (d) All are correct
(e) None of these 2. India becomes the fifth country to have BVR missile.
220. Ukranian Government announced to build a 32000 tonnes 3. The 60-km plus range missile possesses high Single
arch over Chernobyl. What is the importance of this arch? Shot Kill Probability (SSKP) which makes it highly
(a) to protect the citizens of Ukraine from radioactive dust reliable.
(b) to mark the 30th anniversary of the Chernobyl disaster 4. Astra is an all-weather missile with active radar terminal
(c) to make the site a heritage monument guidance.
(d) All of the above Choose the correct statements:
(e) None of these (a) 1, 2 and 3 (b) 2,3 and 4
221. Which animal was recently named as the fastest land animal (c) 1, 2 and 4 (d) All are correct
in the world? (e) None of these
(a) Cheetah (b) Southern California mite
227. Which Parliamentary constituencies during the Lok Sabha
(c) Blue Wildebeest (d) Brown Hare
Polls 2014 registered the highest and lowest voter’s turnout
(e) None of these
respectively?
222. Consider the following statements with respect to Akash
missile. (a) Kohima (Nagaland) highest voter turnout and Srinagar
1. Akash is a medium range surface-to-air anti-aircraft (J & K) lowest voter turnout.
defence system (b) Srinagar (J & K) highest voter turnout and Dubri
2. Missile has a strike range of 25 km (Assam) lowest voter turnout
3. It was developed by the Defence Research and (c) Dubri (Assam) highest voter turnout and Srinagar (J &
Development Organisation (DRDO) K) lowest voter turnout
(a) 1 and 2 (b) 2 and 3 (d) Srinagar (J & K) highest voter turnout and Kohima
(c) 1 and 3 (d) All are correct (Nagaland) lowest voter turnout
(e) None of these (e) None of these
E-54 GENERAL AWARENESS
228. Name the person who on 4 May 2014 won the Munich ATP 234. Which museum of the world has launched an online
tournament? database on World War-I on 11 May2014?
(a) Fabio Fognini of Italy (a) National Museum of India
(b) Martin Kilzan of Slovakia (b) Imperial War Museum of UK
(c) Jan-Lennard Struff of Germany (c) National Gallery of Australia
(d) Tommy Haas of Florida (d) National Museum of Germany
(e) None of these (e) None of these
229. The Obama administration on 5 May 2014 gave its approval 235. A new study Renewable Energy and Jobs - Annual Review
to a country's opposition to open its formal diplomatic 2014 was released by the International Renewable Energy
mission in Washington. Name the country? Agency (IRENA) on 11 May 2014. According to it, how
(a) Syria (b) South Korea many jobs were created by Renewable Energy in the year
(c) Afghanistan (d) Bangladesh 2013?
(e) None of these (a) 8.5 million (b) 7 million
230. Name the former British tennis player, who died on 4 May (c) 6.5 million (d) 6 million
2014 due to liver cancer at the age of 30. (e) None of these
(a) Elena Baltacha (b) Bill Allen 236. Which party of South Africa won the national elections for
(c) Andrzej Grubba (d) Henner Henkel the fifth time in a row on 9 May 2014?
(e) None of these (a) Democratic Alliance
231. Constitutional Bench Supreme Court of India on 6 May 2014 (b) United Democratic Movement
held that prior sanction is not required from the government (c) National Freedom Party
to probe the senior bureaucrats on corruption charges. In (d) African National Congress
its decision the Court highlighted an Article of Constitution (e) None of these
of India that says Equality before the Law. Which Article 237. Which Business Group won the International Business of
highlights Equality before the law? the Year award 2014 at the Asian Business Awards Midlands
(a) Article 10 (b) Article 14 in Birmingham on 9 May 2014?
(c) Article 18 (d) Article 156 (a) Caparo Group (b) Tata Group
(e) None of these (c) Hinduja Group (d) Reliance Group
232. Name the organisation/Institute that on 5 May 2014 in its (e) None of these
release termed that by the end of 2014 the world will have 238. Which sports personality was crowned as the World
about 3 billion internet users - two-thirds of them will be Snooker Player of the Year on 8 May 2014 in London?
from the developing world - with mobile-broadband (a) Mark Selby (b) Ronnie O'Sullivan
penetration approaching 32 percent. (c) Ding Junhui (d) John Astley
(a) United Nations International Telecommunications (e) None of these
Union (ITU) 239. Consider the given Chief Ministers and their respective
(b) WikiLeaks states, and choose the correct code?
(c) UK's National Pupil Database (NPD) 1. Jiatan Ram Manjhi A. CM of Gujarat
(d) Google 2. Naveen Patnaik B. CM of Sikkim
(e) None of these 3. Pawan Chamling C. CM of Odhisha
233. BCCI on 6 May 2014 indefinitely suspended a States Cricket 4. Anandiben Patel D. CM of Bihar
Association within hours of appointment of former IPL (a) 1A, 2C, 3B, 4D (b) 1D, 2C, 3B, 4A
commissioner Lalit Modi as the President of that (c) 1D, 2B,3C,4A (d) 1A, 2B, 3C, 4D
Association. Name the state whose cricket association has (e) None of these
been suspended? 240. Name the former Intelligence Bureau Chief, who has been
(a) Punjab (b) Maharashtra appointed as the new Chief information Commissioner of
(c) Rajasthan (d) Tamil Nadu India (CIC)?
(e) None of these (a) Ranjit Sinha (b) S.A. Ibrahim
(c) S. Srinivasan (d) Rajiv Mathur
(e) None of these
GENERAL AWARENESS E-55

ANSWER KEY
1 (a) 21 (a) 41 (d) 61 (c) 81 (a) 101 (d) 121 (b) 141 (b) 161 (a) 181 (c) 201 (a) 221 (b)
2 (b) 22 (b) 42 (b) 62 (c) 82 (c) 102 (d) 122 (d) 142 (b) 162 (b) 182 (b) 202 (d) 222 (d)
3 (a) 23 (b) 43 (a) 63 (b) 83 (b) 103 (a) 123 (b) 143 (b) 163 (b) 183 (b) 203 (d) 223 (c)
4 (b) 24 (d) 44 (d) 64 (a) 84 (b) 104 (b) 124 (a) 144 (a) 164 (c) 184 (a) 204 (b) 224 (b)
5 (b) 25 (d) 45 (c) 65 (d) 85 (a) 105 (c) 125 (d) 145 (c) 165 (b) 185 (a) 205 (a) 225 (d)
6 (b) 26 (a) 46 (a) 66 (b) 86 (d) 106 (b) 126 (b) 146 (c) 166 (a) 186 (a) 206 (d) 226 (d)
7 (b) 27 (a) 47 (a) 67 (d) 87 (a) 107 (d) 127 (d) 147 (d) 167 (d) 187 (b) 207 (b) 227 (c)
8 (c) 28 (a) 48 (c) 68 (a) 88 (b) 108 (b) 128 (c) 148 (d) 168 (c) 188 (c) 208 (c) 228 (b)
9 (a) 29 (b) 49 (b) 69 (c) 89 (c) 109 (c) 129 (a) 149 (a) 169 (a) 189 (c) 209 (b) 229 (a)
10 (a) 30 (c) 50 (c) 70 (b) 90 (d) 110 (a) 130 (b) 150 (a) 170 (b) 190 (c) 210 (c) 230 (a)
11 (b) 31 (d) 51 (a) 71 (b) 91 (b) 111 (a) 131 (b) 151 (b) 171 (b) 191 (b) 211 (a) 231 (b)
12 (b) 32 (b) 52 (c) 72 (a) 92 (a) 112 (b) 132 (a) 152 (c) 172 (d) 192 (b) 212 (b) 232 (a)
13 (b) 33 (b) 53 (b) 73 (c) 93 (d) 113 (c) 133 (b) 153 (d) 173 (a) 193 (a) 213 (c) 233 (c)
14 (b) 34 (a) 54 (c) 74 (c) 94 (d) 114 (a) 134 (d) 154 (b) 174 (d) 194 (d) 214 (b) 234 (b)
15 (c) 35 (d) 55 (a) 75 (b) 95 (a) 115 (b) 135 (c) 155 (a) 175 (b) 195 (b) 215 (d) 235 (c)
16 (a) 36 (c) 56 (a) 76 (a) 96 (c) 116 (b) 136 (b) 156 (a) 176 (c) 196 (a) 216 (a) 236 (d)
17 (a) 37 (d) 57 (b) 77 (c) 97 (b) 117 (a) 137 (c) 157 (b) 177 (a) 197 (c) 217 (c) 237 (a)
18 (a) 38 (c) 58 (a) 78 (c) 98 (c) 118 (b) 138 (b) 158 (c) 178 (c) 198 (a) 218 (b) 238 (b)
19 (d) 39 (c) 59 (b) 79 (d) 99 (a) 119 (c) 139 (d) 159 (a) 179 (b) 199 (a) 219 (d) 239 (b)
20 (d) 40 (b) 60 (b) 80 (d) 100 (b) 120 (d) 140 (a) 160 (a) 180 (d) 200 (a) 220 (a) 240 (d)

Answers &
Explanations
167. (d) Rajiv Gandhi International Airport (RGIA), operated given annually to an outstanding film personality for
by GMR Hyderabad International Airport Ltd, has been his or her contribution to Indian cinema.
awarded the ‘Sword of Honour’ for safety management. 170. (b) Srishsti Rana won the Miss Asia Pacific World 2013
The award was in recognition of its effective crown in late October 2013. This is the second win in a
implementation of occupational health and safety row at the Miss Asia Pacific World pageant. Earlier,
management systems from the British Safety Council Miss Asia Pacific World title won by Himangini Singh
for health and safety works at the airport. The Sword Yadu in 2012. Rana is 21 years old. She belongs to
of Honour is considered to be the Oscar of the Safety Haryana.
World. RGIA is the first Indian airport to receive this 171. (b) India clinched the women's kabaddi World Cup
honour in the world. Championship for the third time in row, defeating
168. (c) German Chancellor Angela Markel will be awarded the debutant New Zealand 49-21 in the summit clash played
“Indira Gandhi Prize for Peace, Disarmament and at Guru Gobind Singh sports complex. The Indian team
Development for 2013”. Prime Minister Manmohan took home a first-place prize of Rs one crore.
172. (d) India's Ganganjeet Bhullar scripted a brilliant chapter
Singh headed jury has chosen her name for the coveted
in India's golfing history, winning the Indonesia Open.
prize. The prize was being given to the German leader
The Arjun awardee carded a blemish-free-three-under
because of the work done by for her for promotion of
68 in the final round to leave behind Thailand's Chapchai
global economic stability, her commitment to universal Nirat and Nicolas Fung of Malaysia by three strokes.
peace and disarmament and her leadership role in 173. (a) The long serving Gurbax Singh Sandhu was retained
strengthening productive and mutually beneficial as the national coach of the men's Indian boxing team
relations with India and other developing countries. till the 2016 Olympics in Rio de Janeiro. Sandhu, was
169. (a) Renowned actress Waheeda Rehman will be offered first appointed as the chief coach in 1993, was picked
with the Centenary Award for Indian Personality of the for a fresh term by a committee under the IBF president
Year at the International Film Festival of India 2013 in Abhishek Matoria.
Panji, Goa. The award has been instituted to 174. (d) Norway's Magnus Carlsen ended 5 times title-holder
commemorate 100 years of Indian Cinema, and will be Viswanathan Anand's reign as the world champion.
E-56 GENERAL AWARENESS
Magnus Carlsen took the crown after a hard-fought 186. (a) At 12 million, Mumbai has more internet users than
draw in the 10th game of the World Chess any other city in the country, according to data released
Championship match. This was the first time in 8 world by the Internet and Mobile Association of India
championship matches starting from 1995 that Anand (IAMAI). New Delhi ranked No.2 with 8.1 million users,
has was unable to win even a single game. followed by Hyderabad (4.7 million) and Bangalore.
175. (b) India and USA have inked another mega contract, the 187. (b) Voluntary Compliance Encouragement Scheme, 2013
$1.01 billion one for six additional C-130J ‘Super has been proposed by the Finance Minister in budget
Hercules’ aircraft. As per the deal six new four-engine for the year 2013-14, to encourage the defaulters of
C-130J will be delivered within three years under the service tax, who have not declared and paid the tax
US government's foreign military sales (FMS) due for the period from 01.10.2007 to 31.12.2012 which
programme. upto first day of March 2013. The eligible assesses
176. (c) Bihar continues to have the highest percentage of girls may voluntary declare the service tax due under this
marrying before completing 18. While the percentage scheme and he will be refrained from imposition of
during the baseline survey (2008-10) was 20.2 per cent, interest and penalty.
it has now come down to 16.5 per cent. 188. (c) The Haiyan, a category 5 strongest typhoon in the
177. (a) Pran Chopra was a veteran journalist and first editor of world this year and possibly the most powerful ever to
The Statesman newspaper. In 1940s he served All India hit land smashed into the Philippines, forcing more
Radio as a war correspondent in China and Vietnam. than a million people to flee, flooding villages and raising
He was also worked with The Hindu and other fears of widespread causalities.
newspapers. 189. (c) The United States has lost its voting rights at UNESCO
178. (c) The Department of Electronics and Information after it missed a crucial deadline, two year after halting
Technology (DeitY), Ministry of Communications & its due payments and protests the decision to make
Information Technology, launched the "E-Inclusion: Palestine a member. The loss of the vote means the US
IT Training for Rural SC/ST/Women beneficiaries. The cannot participate in funding decision for the agency,
project is part of the government of India's initiative which may be best known for its program to protect
under the National e-Governance Plan for creating a the cultures f the world via its Heritage sites.
transparent and accountable governance model for 190. (c) Ahmed Saleem is the current Secretary General of
enabling service delivery at the doorstep of citizens. SAARC.
179. (b) Bollywood star Shah Rukh Khan has been named the 191. (b) The whole story is around the Section 8 (4) of the
most powerful celebrity of 2013 by Forbes India for the Representation of the People Act 1951 (which allowed
second time in a row. Indian cricketer captain M S Dhoni convicted MPs and MLAs to continue in their posts,
moved up from the third spot last year and has come in provided they appealed against their conviction in
at the number two spots with his immense popularity higher courts within three months of the date of
and endorsement deals. judgment). This section was judged ultra vires by
180. (d) Time Magazine named Pope Francis its person of the supreme court.
year. 193. (a) Three Mistakes of My Life; The film ‘Kai Po Che’
181. (c) Google has opened its first ever data centres in Asia as directed by Abishek Kapoor based on Chetan Bhagat’s
it looks to boost its growth further in the region. The novel “The 3 Mistakes of My Life”.
two new centres are based in Taiwan and Singapore. 195. (b) New Zealand all-rounder Corey Anderson cracked the
182. (b) India's first heritage transport museum has been fastest century in one-day international history, in just
opened in Guargaon, Haryana. The museum which has 36 balls against the West Indies. The previous record
been inspired by similar structure in London in the UK was held by Shahid Afridi, who took 37 balls to reach
and Los Angeles in the US showcase the history of his ton playing for Pakistan against Sri Lanka in 1996.
transportation in India. 196. (a) Myanmar took over the chairmanship of Association
183. (b) Pakistani teen activist Malala Yousafzai won the UN of Southeast Asian Nations (ASEAN) on January 1,
Human Rights Prize 2013. The winners of the 2014.
prestigious UN prize in the field of Human Rights for 197. (c) IT hub Bangalore is expected to witness the second
2013 are Ms.Malala Yousafzai and Mr.Biram Dah Abeid highest demand for office space in the Asia Pacific
from Mauritania. region during 2014 in a list topped by Tokyo, according
184. (a) India's ranked 46th at the first ever ‘Environment and to property consultant Cushman and Wakefield. Tokyo
Gender Index (EGI)’ launched at a side event of the is forecast to see the highest office absorption among
Conference of Parties (COP19). The EGI-a project of the 30 cities in Asia in 2014 at 7.6 million sq ft, followed
the International Union for Conservation of Nature by Bangalore at 2nd position and Manila at 3rd, with
(IUCN) claimed to monitor gender equality and women's expected absorption of 6.3 million sq ft and 6 million sq
empowerment in the environmental area. China ranks ft respectively, during 2014.
34th. 198. (a) Air Marshal Arup Raha, an ace fighter pilot took over
185. (a) India is the fifth country to launch a mission to Mars. as the Air Force Chief succeeding Air Chief Marshal
The other countries that have launched missions to NAK Browne. Raha is expected to have tenure of three
Mars include Russia, China, Japan and the US. years as the Chief of Air Staff.

You might also like